Problems On Physical Chemistry

You might also like

Download as pdf or txt
Download as pdf or txt
You are on page 1of 682

PROBLEMS

ON
PHYSICAL CHEMISTRY
"This page is Intentionally Left Blank"
PROBLEMS
ON
PHYSICAL CHEMISTRY
[For BSc (Honours) and Postgraduate students of all Indian universities]

Dola Pahari MSc, PhD


Lecturer, Department of Chemistry
Kharagpur College, Kharagpur, West Bengal

Dr S Pahari MSc, PhD


Retd. Reader, Department of Chemistry
Midnapore College, Midnapore, West Bengal

New Central Book Agency (P) Ltd


LONDON
HYDERABAD ERNAKULAM BHUBANESWAR
NEW DELHI KOLKATA PUNE GUWAHATI
NCBI\
RECD OFFICE
8/J Chintamoni Das Lane, Kolkata 700 009, India
email: ncbapvtltd@eth.net
OVERSEAS
NCBA (UK) Led, 149 Park Avenue North, Northampton, NN3 2HY. UK
email: ncbauk@yahoo.com
EXPORT
NCBA Exports Pvt. Ltd, 212 Shahpur Jat, New Delhi 110 049
email: ncbaexporcs@ncbapvtltd.com
BRANCHES
2127 Ansari Road, !st Floor House No. 3-1-315, !st Floor
Daryaganj, New Delhi 110 002 Nimboliadda, Kac~iguda, Hyderabad 500 027
email: ncbadel@ncbapvtltd.com email: ncbahydb@ncbapvtltd.com

Shop Nos. 3 & 4, Vinayak Towers GSS Shopping Complex, !st Floor
681 /B Budhwar Peth, Appa Balwant Chowk Opposite College Post Office, Convent Road
Pune 411 002 Ernakulam 682 035
email: ncbapune@ncbapvdtd.com 0484-4021054/ 53

Shop No. 15, Bharati Towers, Ground Floor Radhamayee Complex (Second Floor)
Forest Park Opp. SB!, Panbazar, Jaswanta Road
Bhubaneswar 751 00 I Guwahati 781 001
email: ncbabub@ncbapvtltd.com email: ncbaguwahati@ncbapvtltd.com

PROBLEMS ON PHYSICAL CHEMISTRY• Dela Pahari and S Pahari


© Copyright reserved by the Authors
Publication, Distribution, and Promotion Rights reserved by the Publisher
All rights reserved. No part of the text in general, and the figures, diagrams, page layout,
and cover design in particular, may be reproduced or transmitted in any form or by any
means-electronic, mechanical, photocopying, recording, or by any information storage
and retrieval system-without the prior written permission of the Publisher
First Published: January 2009
Reprinted: January 2011, July 2012
Thoroughly Revised, Enlarged and Updated Second Edition: February 2015
Reprinted: July 2016
PUBLISHER AND TYPESETTER
New Central Book Agency (P) Ltd
8/1 Chintamoni Das Lane, Kolkata 700 009
PRINTER
New Central Book Agency (P) Ltd
Web-Offset Division, Dhulagarh, Sankrail, Howrah
TECHNICAL EDITOR
Dipan Roy
PROJECT TEAM
Partha Pratim Biswas and Pradip Biswas

ISBN: 978 81 7381 958 2

Price~ 465.00
Contents
Preface to the Second Edition vii
Preface to the First Edition viii

1. Introduction 1-17
2. GAS 18-73
3. Liquid State (Surface Tension and Vi'.cosity) 74-89
4. Solid State 90-114
5. First Law of Thennodynamics, Carnot Cycle,
Clausius-Clapeyron Equation and Thennochemistry 115-196
6. Second Law of Thermodynamics 197-250
7. Colligative Properties of Solution 251-276
8. Chemical Equilibrium 277-315
9. Chemical Kinetics 316-354
10. Photochemistry 355-379
11. Electrolytic Conductance 380-414
12. Ionic Equilibrium 415-447
13. EMF 448-488
14. Adsorption, Catalysis, Colloids 489-514
15. Quantum Mechanics 515-560
16. Spectroscopy and Some Physical Properties 561-594
17. Statistical Thermodynamics 595-607
18. Miscellaneous Problems 608-673

v
"This page is Intentionally Left Blank"
Preface to the Second Edition
The book first appeared in 2009 with the object of helping the students to learn the subject.
Acceptance of the book by the students as well as the teachers inspired us to revise it. While
revising, we felt that-to give the students the flavour of the subject through short questions
and numerical problems-some more questions and problems of different varieties should be
added. We have increased the number of problems in many' chapters. Few short questions
have also been added. Although we had plan to add few more short questions, we could not
do it to meet the deadline.
We have followed IUPAC recommendations as to symbols of physical quantities so as
to make the students familiar with it. Hence, symbols (e.g., density, velocity etc.) have been
changed. Only few symbols would be found unaltered in this book. Use of molar conductivity,
instead of equivalent conductivity, is strongly advocated by IUPAC, but both the conductivities
have been kept here since value of equivalent conductivity, instead of molar conductivicy,
is often asked in many numerical problems. Molar conductivity, in the book has been denoted
by "m instead of " as recommended by IUPAC. IUPAC recommends using molarity (c)
or molality (M) as to defining concentration of a solution. Since we also use normality, it
is kept in the book. We could not strictly adhere to the SI system as to unit of any quantity.
The problems given by many universities in UG exams do not always follow the SI system.
So we have answered in the unit as required by the problem. Hence, the units of physical
quantities in the numerical problems have been kept unaltered. However, we have added
few problems in ill unit and promise to give more so in the next edition.
In thermodynamics, work has been defined according to IUPAC convention. Hence
its sign has been changed. Although we also had the intention to change the working formulae
required to work out numerical problems in other chapters to. comply with the IUPAC
recommendations, we could not do it for Jack of time. We have tried our level best to
eliminate all the errors and incorporate the alterations uniformly. Since there were many
alterations and corrections, new errors might have crept in while some older ones may
have remained unnoticed despite our sinceremost effort. We deeply regret all such errors
and will try to correct them in the next edition.
Only by solving out different types of problems, students can get a clear conception
of physical chemistry. The more multifarious problems a student tries to solve, the deeper
he/she will go into the subject. So our advice is that students should. look up the solution
to any problem only after giving substantial effort to solve the problem themselves. This
will help them to understand the subject better. Then only our purpose will be fulfilled.
Students should thorougly read Chapter 1 wherein we have tried to give them the idea
as to how to tackle any numerical problem in Physical Chemistry.
We are grateful to Sri Amitabha Sen, Director, New Central Book Agency (P) Ltd, to
agree to publish the Second Edition. We remain truly indebted to our family members-Gita
Pahari, Sri Shubhashis Acharya, Subhajit Pahari-for their constant care, support, and
encouragement. We are truly thankful to Shri Prabhat Jas and other members of the Project
Team of NCBA for their sincere effort. Shri Dipan Roy (Technical Editor) helped us a lot
in proof correction. We express our gratitude to him. The Printer, Cover Printer, Cover Designer
and others have done their jobs quite well .. we may not know them personally but without
their help our effort would not have been fruitful.
Last but not the least, we are grateful to all those teachers and students whose valuable
comments helped us a Jot in the revision of the book.
Raspurnima D Pahari
7 November 2014 S Pahari
Midnapore
vii
Preface to the First Edition
A study of physical chemistry is never complete without solving problems-either theoretical
or numerical. It enables the student to understand the subject clearly. We have written this
book on problems to help the students to learn how to solve the problems.
Numerous books are available either on numerical problems or theoretical problems.
But this book on problems includes both numerical problems as well as theoretical problems.
This book consists of three sections. In the first section, we have introduced various
formulae necessary for solving the numerical problems. Second section is contributed to
theoretical problems, and the last section is for numerical problems. Besides this, few chapters
have Miscellaneous sections at the end with some conceptual questions and tricky derivations.
Major portion of problems have been answered. Few problems have been left for the students
to work out by themselves but hints have been given to guide them. At the end of the
book there is a Miscellaneous chapter having various types of problems consisting of multiple
choice questions, short questions, numerical problems covering the whole subject of the
BSc syllabus. Some short questions as well as all the multiple choice questions have been
answered. Few numerical problems have been solved. Some short questions have been left
unanswered. For most numerical problems in this chapter, only the answers have been given.
They are left for the students to solve to check their grasp of the knowledge of the subject.
It will help them to grow confidence in themselves.
Many different techniques can be employed to solve a problem. In this book we have
tried to stick to one method as far as possible to solve similar type of problems. We have
chosen that method which the students will find very easy to understand, and can use it
to solve other problems. However, sometimes, for some problems, another alternative and
simpler method of solution suitable for students has been shown and students can choose
either one.
We express our gratitude to Shri Gouri Sankar Maity for providing us with IIT and
other Entrance Examination question papers. We are grateful to Late Jogendra Nath Sen
and the publisher Shri Amitabha Sen. We owe much to Shri Prabhat Jas and others of New
Central Book Agency (P) Ltd. We ·are grateful to Shri Dipan Roy (Technical Editor) who
helped us by offering suggestions at various stages of composing of the manuscript. The
editorial staff and the printers, who work behind the scenes, do a thankless job; but without
them the project cannot be completed. We are indebted to all of them.
We invite suggestions and criticisms from the teachers as well as the students which
will help us to make the book more purposeful and error-free in the subsequent editions.

15 August 2008 S Pahari


Midnapore DPahari

vii'i
CHAPTER 1

INTRODUCTION

Long back Prof. W.V. Evans defined a Physical Chemist as a mathematician sitting behind
a mahogany desk with a platinum slide rule in his hand. A popular science and novel writer
Sinclair Lewis held a very high opinion about Physical Chemistry. In one of his novels
Published in 1924, Arrowsmith, the character Max Gotlieb, a teacher of a medical school
asks Arrowsmith, the hero of the novel-"How can you know Physical Chemistry without
much mathematics?" In the same novel Gotlieb utterd, "Physical Chemistry is
power, it is life, it is exactness." In this line we tell you to learn Physical Chemistry by
heart. For that you must be able to solve Problems to learn a basic theory by heart.
For solving a problem at the start :
(i) Write what has been given.
You may write in short in boxes. For example, suppose that a problem is like this-
A. 2 moles of an ideal gas at 27 ·c and 10 atm is allowed to expand to I atm at
327 ·c expanding against a constant pressure of l atm. Calculate the work done, 6.U, f1H
and Q. Comment.
Thus it has been given
n = 2 n =2
T1 = 300 K Expansion T2 = 600 K
P 1 = IO atm against Pex = atm P2 =l atm

And comment on the result.


You may not draw such boxes always. But, however tough the problem may appear,
if you can write a simple summary of what has been given, you will find that the problem
is not· so tough.
Then,
(ii) Write what has to be calculated.
In the given problem A, the quantities to be calculated are' W. Q, !::..U and !::..H. In this
given problem, no extra quantity has to be calculated. The states mentioned in the boxes
are equilibrium states. This may not be always so. For example, let the exercise be :

Prob. Phy. Chem.-i


2 PROBLEMS ON PHYSICAL CHEMISTRY

B. 2 moles of an ideal gas at 10 atm and 27 'C are allowed to expand against a constant
pressure of 1 atm to double the volume at a temperature of 327 'C. Calculate W, 6.U, !:lH
and Q. Comment.
Here it has been given
n = 2 n =2
T1 = 300 K Expansion against T = 600 K
P1 = 10 atm 1 atm Pz =?
v1 2V 1

In this problem, the extra quantity to be calculated is volume V 1 at 300 Kand 10 atm.
And then
(iii) Write formulae connecting known to unknown.
In problem A given,
W = -Pex (Vz - V1)

= _p 2 ( nRT2 _ nRT1 )
P2 Pi
6.U = nC1,(T2 - T 1)
!:lH = nCp<T2 - T1)
6.U = Q + W.
Solve it yourself and comment if you can (see the chapter on first law of
Thermodynamics).
In problem B :
W = -Pex (Vz - VI)
= -Pex (2V1 - V1)

= - p ex· vI -- -
nRT
p ex - -1
Pi
6.U, 6.H are same ;is problem A.
6.U = Q + W.
To comment, calculate the pressure of the final state.
Do not use an equation where it does not apply. Be sure of the fact that you are using
an equation which is applicable.
If you are in difficulty, then do backwards from unknown to known. Then come from
known to unknown.
(iv) Calculate using formulae and modifications. Use a calculator. Round off the
final result to the desired place of decimal.
Rounding off : In calculations, round up if the last digit is above 5 and round down
if the last digit is below 5. Thus, if there has to be 4 significant figures, then report 2·4637
as 2.464 and 2.4634 as 2.463. But for numbers ending in 5 always round to the nearest
even number. Thus 2.4635 is 2.464 but 2.4645 is 2.464.
Writing and reporting results : A number should be written as A x 10 a : A is a
decimal number before decimal point and a is a whole number, positive or negative. Thus
333 is 3.33 x 102 and 0.043 is 4.3 x 10-2.
INTRODUCTION 3

Significant figures : In reported measurement, the digits, the report of which depends
on the accuracy of measurement, are called the significant figures (SF). Thus there are two
significant figures (with 2SF) in 2.4 and three significant figures (with 3SF) in 3.40. Trailing
zero and zero in middle of a number are significant, but leading zeros are not included
in significant figures. Thus there are 3SF in 0.120 (which is 1.20 x 10- 1) and 2SF in 0.0017
(which is 1.7 x 10-3).
When adding or subtracting, the number of decimal places in the result should be equal
to the smallest number of decimal places in the data. Thus, 12.1 added with 7 .22 is 19.3,
not 19.32.
When multiplying or dividing, the number of significant figures in the result should
be the same as the smallest number of significant figures in the data (least exact factor).
Hence, 12.14 x 7.11 = 86.32 (and not 86.3154).
12 4
Similarly, ·1 = 3.78 (and not 3.781931464 or 3.782).
3.21
But if the first significant figure is 1, the number should contain one more significant
.
f1gure. T hus -
12.14
- = 1.707 (an d not 1.71 or 1.70745429).
7.11
However, while using calculator the rounding off should be done at the last stage of
calculation and all digits are to be carried in the memory of the calculator until that stage.
Using different formulae : Let the collision frequency Z be given as

~8RT(NAP)
2 2
z = rt:cr •
{2 rcM RT
All the data including a is given. You can calculate Z. But let a be not given-
A, the mean free path, is given :

Then

z = (NAP)_l ~8RT
RT 2A rcM
Let A be not also given, but viscosity T/ and density d be given. Then

T/ = .!dv'A
3
3ri
'A = dV

Thus, z = (NAP)dvv
RT 61'1
= ( ~A:) 6~ V2.
This is a simple illustration. You have to be free in such uses. These require practice.
The more you practise, the more confident you become in solving. And note the key-
word : Confidence!
Solve similar problems by the same method. Start from simpler problems. Tackle
tougher problems later.
4 PROBLEMS ON PHYSICAL CHEMISTRY .

Be proficient in using formulae. Then you may be able to tackle "Prove that w = ... ),
etc" problems easily.
To solve and write answers to short questions a thorough study and deep thinking on
the chapter are needed.
Be confident that if there is a problem, then there is a solution. 'Problems are solvable,
and by study and practice, I can solve problems, however tough they may appear'. The
same thing applies for short questions.
Method of solving theoretical questions
A student of physical chemistry who cannot solve problems has not understood this
subject. Numerical problems as already outlined can be solved by the given method. Test
it slowly finding a problem appearing tough to you. Then, follow the method outlined blindly.
See what happens.
Similarly, derivations and theoretical problems can be solved if you have understood
the subject by heart. Let us check. It is given that
(i) Cooling due to expansion of an ideal gas occurs due to decrease in internal energy
per unit volume. Is it 0.K?
Hints to help you : This is from first law of thermodynamics. Adiabatic expansion
leads to cooling of ideal gas since it is done at the expense of internal energy.
(ii) Cooling due to expansion of real gas occurs because of reduction in the
intermolecular energy, since it depends on intermolecular attractive forces between
molecules. Is this right?
Hints to help you : Cooling of real gases also take place during adiabatic expansion
even during free expansion. However, it is to be remembered that increase in volume does
not necessarily mean decrease in intermolecular attraction energy.

Suppose, you are asked to derive ( oH)


oP T
for van der Waals gas which is often set

in university examination. It is needed to calculate µ J. T (Joule-Thomson Coefficient) which

is µ
J.T
=_ _I (
cp oP r
oH) .
This can be. solved at least by three different ways. These are the followings
Method A:

van der Waals equation is (p + ; )cv -


2 b) = RT A(l)

ab
Expanding and neglecting V2 in comparison to other terms,

PV +.!!:.. - Pb = RT. A(2)


v
Replacing Vin the denominator by the ideal gas value RT (which will not incur huge
p
error),·

PV + aP - Pb = RT A(3)
RT
INTRODUCTION 5

aP
or, PV = Pb - - + RT A(4)
RT

:. V R: + ( b-;T).
= A(5)

Hence,
· (av)
(JT P =
R a
p+ RT2 A(6)

RT a
... T(av)
(JT P P RT
so V- T(av)
(JT P =(b-3.!!:_)
RT A(7)

Again, we know that dH =T dS + V dP A(8)

:. (-aH)
(Jp T
= T (as)
-
(JP T + V. A(9)

By Maxwell's relation, ((JS)


dP - -((JV)
dT T - p
A(lO)

( aH)
dP
= v _ T(av)
T dT p
From equation A(7),

( aH) T dT = (b-3.!!:_)·
dP = v _ T(av) RT p
A(l l)

.. µ J.T = c- L( i;). b- A(l2)

.. µ = dp ( ~; - b). A(l3)

Related Derivation :

We know that Cp - Cv = T(aP) (av) A(l4)


dT v dT p

van der Waals equation :


RT a
P=----2 A(l5)
V-b V

V = R!._ + (b -
P
_!!_)
RT A(l6)

R
A(l 7)
V-b

Again, A(l8)
6 PROBLEMS ON PHYSICAL CHEMISTRY

Now from the expanded van der Waals equation neglecting a~ and V in denominator
v
of the term !!:_ by RT (the ideal gas value),
v p
aP A(l9)
PV =Pb - - +RT
RT
RT a
or, P(V - b) = - - - . A(20)
P RT
R V-b a
Hence, - = - - + - -2. A(21)
P T RT

Thus, C p - Cv = T(_!!_)(
V-b
R
P
+ --;.-) .
RT
A(22)

Putting A(2 l) in Equation A(22),

Cp _ Cv = RT ( V - b + ~)
V-b T RT2

= R (1 + --3!:_) LV > > b] = R + 2 ~~ [putting V = RT) A(23)


RTV RT . P
Method B:

I 'aH) B(l)
µJT=- cJaP T
Now van der Waals equation :

(P+ ; 2 )c V - b) = RT. B(2)

·
N eg 1ectmg V2
ab term ·m comparison
· to others an d expand'mg,

PV + !!:_ - Pb = RT. B(3)


v
. . _i_(pv +!!:__Pb)= _i_(RT) B(4)
aT V aT

or, (av)
aT av p
{_i_(Pv+!!:.--Pb)} =R.
v p
B(5)

B(6)

B(7)
INTRODUCTION 7

So, T ( ~~) = -P-+---~-T-_-=-_2-a_


P
B(8)

v2 v2
RT
= -R-T--2-a B(9)

V-b -vz-
RT
= ~-------
_RT
V-b
(i- 2a(V-b)).
RTV 2
B(lO)

.. T(av)
ClT p
=(V-b)( 1_2a(V-b))-'·
2
RTV
B(l l)

=(V-b) (i+2a(V-b)) B(l2)


RTV 2

or, T - (av)
()T P
- V = (V - (
b) 1 + 2a(V-b))
RTV 2
- V B(l3)

= [V - 2a(V-b)
b + RT -V- ]
- V B(l4)

or T(()V) - V = V - b +-3!:_ - V - V B(l5)


' ()T P RT

Neglecting b in comparison to V,

v- T(av)
()T P
=(b-3!:)
• RT
B( 16)

µJ.T = - Cp1 (ClH)


()p T B(l7)

dH = TdS + VdP B(l8)

Hence,(aH)
()p T
= T(as)
()p T
+V. B(l9)

But by Maxwell's relation,

B(20)

B(21)

Hence, (-ClH)
()p T
=V - V + b - -2a .
RT
B(22)
8 PROBLEMS ON PHYSICAL CHEMISTRY

B(23)

B(24)

or, µIT = dP ( !; -b). B(25)

Related Derivation :

c _cv = T(aP)
P aT v (av)
aT . p

By van der Waals equation,

p = _.!!!___ - .!!_2 B(26)


V-b V .

Hence, (ap)
dT v
R
V-b
B(27)

From Equation. B(l2),

T(av)
aT p
= (V- b)[l+ 2a(V-b)J·
2
RTV
B(28)

Hence, Cp- Cv = ___..!!___


V-b
x (V- b) [i+ Za(V-b)]
RTV 2
B(29)

B(30)

=R(l+ 2aV) B(31)


RTV 2

= R(I+ R2:v) B(32)

2aP . RT
= R + --ry (putting V =-). B(33)
RT~ P
Method C:

µJT =-L(~;)T C(l)

= __ 1[(au)
cp aP
+{a(Pv)}
aP
] T T
C(2)

= - Cp
1[(au) (av) {a(Pv)} ]
av aP + --a? T T T
C(3)
INTRODUCTION 9

For a van der Waals gas,

(~~)T= :2 C(4)

Since, dU = T dS - P dV, C(5)

C(6)

By Maxwell's relation,

(~~ )T = (~~)V. C(7)

Thus.(au) = T((Jp) - P. C(8)


av T aT v

Now, ((JP) = _I!_, C(9)


(JT v V-b

Thus._ (~~)T = V~b - (vR~b -:2) C(IO)

a
= C(l l)
?'
Hence, ( (JH) = -;-(av) +[(J(PV)] . C(l2)
(Jp T V (Jp T (Jp T
We know that

(P+ ; 2 )cv - b) =RT [For van der Waals gas).

Negleeting ~~ in comparison to other terms and expanding,

PV - Pb+!: =RT C(l3)


v
or, {a{PV)} _ b --;-(av) = {a{RT)} . C(l4)
(Jp T V (Jp T (Jp T

:. {a(Pv)} =b + -;-(av) . C(l5)


(Jp T v _aP r

a(
Now (Jp P+?
a )
= (Jpa ( V-b
RT ) C(l6)

a(
o r l + - a-
'
) - - -RT-)
(JP V 2 - (JP V - b
a( C(l7)
10 PROBLEMS ON PHYSICAL CHEMISTRY

' +~(_::_)(av)
or 1 a( RT ) (av)
av v2 aP T = av v-b aP T
C(l8)

C(l9)

Thus, (aapV) T
= -=2-a__l --.,,R=r=--- C(20)

VJ- (V-b) 2
l
=
2a
VJ -
{i RTV
2a(V-b) 2
3
}
C(21)

C(22)

Now, ( aH)
aP T
= -;(av)
v aP T
+{a(Pv.)}
aP T
C(23)

C(24)

C(25)

C(26)

v3
Since V > > b, 2 = V.
(V-b)

Thus, (aH) = b + ~TV. C(27)


aP T l---
2a
RTV
Now - - > > I. C(28)
2a

Hence, ( aH) = b _ ~ C(29)


aP T RTV
2a

C(30)
INTRODUCTION 11

_1 (b - :!:!!_) C(31)
Cp RT

We cannot proceed further to derive (Cp - Cv) along this path, since both (~;)v

and (av)
aT p
have to be derived anew.

However, (Cp - Cv) can also be derived in a separate method employing another
formula.
Derivation of (Cp - Cv) :
We know that,

C(33)

Now, dU = T dS - P dV.

... (au)
av T
_T(as)
- av _ T
P. C(34)

By Maxwell relation,

(i~l =(~;l· C(35)

Hence, (au)
av r
= T(aP) - P.
aT v C(36)

For a van der Waals gas,

RT a
P= - - - -2 C(37)
V-b V .

. (~;l =-v-~-b C(38)

Putting Equation C(37) and Equation C(38) in Equation C(36),

C(39)

a
=0· C(40)

Putting Equation C(40) in Equation C(33),

cp - c v = (p + :2 )( ~~) p . C(41)
12 PROBLEMS ON PHYSICAL CHEMISTRY

a~
0

Again expanding van der Waals equation and neglecting in comparison to other
v
terms,

PV- Pb+~ =RT C(42)


v
or, p(av) _ _!!-__(av) =R t(43)
aT p v 2 aT p

or, (aaTV)P = P-R__a_ C(44)


vz
R
C(45)

R
C(46)
= RT 2a
V-b- V 2

R
_!!I_[l - 2a(V- b)].
=~~-=-~~~~= C(47)

V-b RTV 2
Since V > > b,

C(48)

_ V- b(
- -- 1--
2a )-l C(49)
T RTV

Thus
' (-av)
aT p
=-T
- l +RTV
V-b( 2a)
--. C(50)

Hence, Cp - Cv = -·-
RT [ -
V-b
V --
T
b ( l +2a
- - )]
RTV
C(5 l)

= R(l + ::v). C(52)

Replacing V in denominator by ideal gas value,


2aP
Cp - Cv =R + RT2 . C(53)

Which one of the above methods will you choose for de(iving µ JT and why? Ask it
yourself.
INTRODUCTION 13

Units and Dimensions


For details see Physical Chemistry-Dr S. Pahari-Chapter
Quantities are to be reported with units consistent with the dimension of the quantity.
In International System of Units (SI) there are 7 base quantities which have their own
dimensioas. Out of these we will never use in this book the dimension, J, and unit, candela,
of the base quantity luminous intensity (Iv). So, the remaining 6 base quantities with symbols
to denote them and their dimensions along with symbols and definitions of their units are
given below :

Physical Symbol for Symbol for Name Symbol Definition


quantities quantities dimension of unit for unit

Length l l metre m length of the path travelled


by light in vacuum during
a time interval of
1/299792458 of a sec

Mass m M kilogram kg mass of the cylinder, kept


in Paris by International
Bureau (BIPM) of Weights
and Measures, made of
Pt-lralloy

Time t T second s duration of 9, 192,631,770


periods of the radiation
corresponding to the speci-
fied transition of the
caesium 133 atom at rest at
a temperature of 0° K

Electric I I ampere A magnitude of the constant


current current which, if main-
tained between two
parallel plates of infinite
length and negligible
circular crosssection and
placed one metre apart in
vacuum, would produce
will exert a force of
2x 1o-7 newton per metre-
length.

Thermo- T e kelvin K fraction of 1/273.16 of the


dynamic
temperature - thermodynamic tempe-
rature of triple point
(273.16 K) of water.

Amount of n N mole mo! amount contains as many


substance atoms as there are in
0.012 kg of 12C.
·------ ------~ --~-"- ·---
14 PROBLEMS ON PHYSICAL CHEMISTRY

All other quantities are expressed in these units or in terms of derived units obtained
algebraically by multiplication or division. Some derived units have their special symbols.
The principal derived units are shown below (to be used in problems) :
Quantity Unit Symbol Definition
Force newton N kg. m.sec- 2
Work, energy, heat joule J N.m = kg.m 2sec2
Power watt w J. sec-I
Pressure pascal Pa N.m- 2 = kg.m-I .sec-2
Electric charge coulomb c A.s
Electric potential kg.m 2.sec 3 .A-I
difference volt v = J.A-I .sec-I = J.c-I

Electric resistance ohm Q kg m2.sec3.A-2


=VA-I = J.c-I .A-I
Electric capacitance farad F A.s.v- 1
= kg-I .m-2.sec 4 .A 2
Magnetic flux density tesla T kg.sec2.A-I
= N.A-I.m-I
Frequency hertz (cycle/second) Hz sec-I

Megnetic fluX" weber Wb Vs


= kg.m 2.sec-2.A- 1
Inductance henry H V.sec.A-I
= kg.m 2.sec-2.A-2
= J.A-2

SI PREFIXES

Submultiple Symbol Pre.fix Multiple Pre.fix Symbol

10- 1 d deci IO deca da


10-2 c centi 102 hecto h
lo-3 m milli 103 kilo k
10-{j µ micro 106 mega M
10-9 n nano 109 giga G
lo-12 p pico 1012 tera T
10-I5 f fem to
10-18 a atto
10-21 z zepto
INTRODUCTION 15

Writing Symbols
Abbreviated units are never plural and also do not end with dots.
Thus, I 0 kg, and not I 0 kg. or I 0 kgs.
In case of derived units, there are three methods of writing, e.g., joule is N.m or Nm
or N x m. The first method is followed here.
Conversion of Units
C.G.S. and practical system of units are commonly used. The C.G.S. unit of energy
is erg: 1J = 107 erg. The conversion factors for mechanical quantities in the two units may
be done by writing dimensions. The dimension of energy is F.l = ML21 2.
I joule = lkg.m 2.s-2 = x erg = x gm.cm 2.s-2

:. x = I ~-(~)2 = 103 gm ( 102cm)2 = 103 x 104 = 107.


gm cm gm cm
There are many practical units. As for example, pressure is expressed in atm-it is
the weight of 76 cm high Hg column.
p = d g h

= 13.5951 gm x 980.665 cm2 x 76 cm


cm 3 sec
= 1.01325 x 106 dyne
cm 2
5
= 1.01325 x 10 Pa.

IPa = I ~ (dimension of pressure = ML-112)


m
= kg.m- 1.s-2
dyne _ _1 _
=x --
cm 2
- x gm.cm .sec 2.

gm lcm
:. x= = J03 gm x 102cm = 10.
P = 1.01325 x 102 kPa (kilo pascal)
dyne
= 1.01325 bar (!bar = 106 - - = 105 Pa).
cm 2
5
325
I torr = I mm of H g = l.OI 760xl0 Pa= 133.32 Pa.

N
Energy : I I-atmosphere= 10-3m3 x 1.01325 x 105-;:;;z

= 1.01325 x 102 J.
I bar-litre = 100 J (exactly).
S.I. system is coherent in the sense that if in an equation there are 2 variables or more
and all are expressed in S.I., then the resulting quantity will be in S.I.
16 PROBLEMS ON PHYSICAL CHEMISTRY

Some of the conversion factors of some special units are indicated in the following
table :
Physical quantity
Length
Name of Unit
Angstorm
.
A
Symbol Equivalent in SI unit
10- 10 moo- 1nm)
Energy electron-volt eV l.602!89x!0- 19 J
= 8065.5 cm- 1
= 96.485 kJ.mo1-1
wave number cm- 1 I .986x I o- 23 J
calorie cal 4.1845 J
erg erg 10-7 J
I-atmosphere 1-atm 101.325 J
bar-litre b-litre 100·0 J
Force dyne gm.cm.s-2 10-7N
Pressure atmosphere atm 101.325 kPa
bar bar 100 kPa
torr tor 133.322 Pa
Electric charge e.s.u. -3.334 X 10-IO C
(electrostatic unit)
Dipole moment debye I0- 18 esu-cm -3.334x10-30 c.m
Magnetic tluX"
density gauss G J0-4 T
Magnetic flux maxwell Mx 10-8 Wb
Viscocity poise p 10- 1 Pl

The values of standard quantities that will be used in the book with symbols are :

Name Symbol Values

l. Velocity of light c 2.9979xl0Sm.sec 1


2. Electronic charge e l.6022xI0- 19 C (coulomb)
3. Planck constant h 6.626x I 0-34J.sec

Ii = 27th l .0546x I o-3 4J.sec

4. Avogadro number NA 6.022xI0 23 mol- 1


5. Mass of one atom

of _!_ of CI 2 isotope 1 a.m.u. 1.6606 x I o-27 kg


12
6. Mass of electron me 9.109 x 10-31 kg
7. Mass of proton mp 1.6726 x 10-27 kg
8. Mass of neutron mn 1.6750 x 10-27 kg
9. Faraday F 9.6484 x 104 C.mo1-1
Contd.
INTRODUCTION 17

Name Symbol Values

I 0. Boltzmann constant k 1.3807 x 10-23 J.K- 1


11 . Gas constant R 8.314 J.K- 1.mol- 1
0.0821 l-atm.K-1.mol- 1
1.987 cal.K- 1.mol- 1
12. Acceleration due to gravity g 9.80665 m.sec- 2
13. Gravitational constant G 6.673x10- 11 N.m 2 .kg-2
14. Standard pressure Po I bar
Standard concentration C° mol.1- 1 of solution
Standard molality mo l mol.kg- 1
15. NTP, STP l atm, 273.15 K (old)
I bar, 273.15 K (new)
16. SATP I bar, 298.15 K

The symbols for quantities commonly used in the book are listed below :
Number of molecules N Time t
Number of molecules per Temperature ·c (J
unit volume N' Number of molecules in zero
Root mean square energy level No
(r.m.s.) velocity c E.M.F. E
Concentration in Internal energy u per mole um
molarity c Enthalpy H per mole Hm
molality m Entropy s per mole Sm
normality N Helmholtz F per mole Fm
Equivalent conductance A. Gibbs free energy G per mole Gm
Ion conductance A.± Number of moles n
Average velocity v Viscosity coefficient TJ
Most probable speed (mps) ii Mean free path A.
Ionic mobility u± Density p
Ionic velocity v± Molar mass M
Volume v Transport number ti(t±)
· Any velocity v Charge Q
Molar volume vm Coulomb c
Conductance L Wavelength A.
Resistance R Frequency v
Pressure p Wave number v
Momentum p CP/Cv y
Temperature T Avogadro Number NA

Prob. Phy, Chem.-2


CHAPTER 2

GAS

A Gas Laws
Al. Required Formulae
1. Ideal Gas Laws

la. PV = nRT = W RT, where W is the weight of the gas taken and M its mole-
M
cular weight.

lb. PM = -w RT = pRT, where p is the density.


v
le. .!l_ = V2 !i
P2 V1. T1
2. Real Gas Laws

~2 )w - nb) = nRT
2
2a. van der Waals equation : ( P +

2b. RT, = ~ for van der Waals gas, where Tc, Pc, Ve, etc. are critical temperature,
P,Vc 3
pressure and volume respectively.
:?.c. Compressibility factor is Z.
PV PV
Z = -- n =
nRT' ZRT
Z = l, ideal gas
Z > l, less compressible than ideal gas
Z < I, more compressible than ideal gas.
nA
nRT RIV
3. Dieterici equation of state : P = - -e
V-nb
18
GAS 19

4. Virial equation of state


A B C
Z= I+-+-+-+
v y2 y3
A, B, C, etc are 2nd, 3rd, and 4th Virial coefficients.
5. Redlich-Kwong Equation :
2

( P + {f
An
TV(V +nB)
) (V - nB) = nRT
6. Law of corresponding states :
PV
Z = -- = F(Tr, Pr),
nRT

where Tr=_!_, Pr=_!__, V =~


Tc pc r Ve
At the same Tr and Pr, all gases have same Vr.
7. Near Boyle temperature, TB, a real gas behaves more ideally. However, in the high
temperature range the major deviation from ideality is closer to 2T8 . For a van der Waals

.
or Dicterici gas,
a
TB=-.
Rb
8. a, b of van der Waals equation in terms of Tc, Pc ,
27 R2 Tc2
a=

8a
p
c
= 27b2'
a T
c
=
27Rb
A2. Short Questions
1. What is the dimension of change?
Ans. : Change is dimensionless. If the change in x is Llx, the dimension of this change
is the dimension of x.
2. When does a real gas behave like an ideal gas?
Ans. : In a real gas there is attraction and repulsion between molecules. When these
two are same, the real gas behaves like an ideal gas. Boyle temperature TB of a real gas
is the temperature at which a real gas behaves like an ideal gas up to certain range of pressures.
At any temperature, every real gas tries to behave like an ideal gas as P.....o,0. At very high
temperatures also a real gas may behave like an ideal gas.
3. Define Compressibility factor. State its significance.
Ans. : Compressibility factor, Z, is defined as the ratio of observed molar volume of
a gas to the molar volume that an ideal gas would occupy at the same temperature and
pressure. It may also be defined as the ratio of pressure that one mole of gas exerts to
the pressure that one mole of an ideal gas would exert at the same temperature and volume.

Vreal PVreaI Preal = Preal V


Z= - - = - - =
Victeal nRT Picteal nRT
20 PROBLEMS ON PHYSICAL CHEMISTRY

Significance : Z is measure of the deviation from ideal gas behaviour. For an ideal
gas, Z is equal to one at any temperature or pressure. A real gas can have negative or positive
deviation. Negative deviation (Z < I) signifies that the real gas is more compressible than
ideal gaf or vreal < videal whereas positive deviation signifies less compressibility of real
gas compared to ideal gas, i.e., Vreal > Videat· However, at Boyle temperature, TB, Z becomes
1 for a certain range of pressure when Vreal = Videat·
4. If P and V are the pressure and molar volume of a real gas and Pid' Vid are those
for an ideal gas, then can we write Pid V = PVid = constant?
Ans. : Acording to the definition of Z (See Q. 3)
v p
Z=
Videal ?ideal

or, z= ?ideal v= p videal


nRT nRT
Thus, ?ideal V = PVideal = ZnRT = constant as T is constant.
5. Is Z in PV = ZnRT extensive?
Ans. : For a particular gas under a particular set of conditions Z is fixed, and in case
of Z parts are not added to give the whole, thus, Z is intensive.
6. What is the basic difference between volume and molar volume?
Ans. : Volume is an extensive property whereas molar volume is an intensive property.
7. Is TB higher than Tc?
Ans. : Critical temperature is the temperature above which a gas cannot be liquified
by application of pressure alone. But attractive forces stil.1 remain stronger than repulsive
forces. A gas above Tc is a gas, below Tc it is vapour. Gases cannot be liquified because
K. E. is so high that they cannot be brought closer. At Boyle temperature, TB, attractive
force is balanced by repulsive force-a gas behaves like an ideal gas. Since repulsion
increases its importance with increase in temperature, T8 is certainly higher than Tc- Thus
TB has to be higher than Tc.
8. What are the dimensions and S.I. units of a, b and R?

~:)
2
Ans. : The van der Waals equation is ( P+ (V - nb) = nRT. Thus, a;2 has

the dimension of pressure, ML- 11 2 . Hence·, a has the dimension of ML 51 2 moJe- 2 and
hence, SI unit is J.m 3 .mol-2 .
nb has the dimension of volume and so the dimension of b is L 3 mole- 1 and S 1 unit
is m 3.mo1- 1.
R has the dimension of energy per deg per mole, so it is ML 21 2K- 1mole- 1. In SI
unit it is J.K- 1.mo1-t.
9, Can we liquify a van der Waals gas for which a = O?
Ans. : There is no attraction between the molecules of a gas for which a = 0, and
thus it cannot be liquified.
10. "A real gas in a container starts behaving ideally as temperature is raised"-
Comment.
Ans. : In a real gas molecule, there are attractive as well as repulsive forces between
the molecuh;s. With increase in temperature;, the kinetic energy of the molecules increases
GAS 21

and hence chaos increases. At a particular temperature, TB, called Boyle temperature, the
intermolecular attraction becomes equal to repulsive forces. The gases behave ideally over
a wide range of pressures. As temperature still increases, only intermolecular repulsive
forces are important up to twice the Boyle temperature. Above that temperature, intermo-
lecular repulsive forces also start losing importance. Only the chaos of motion remains.
Thus, as temperature increases above that, the gas starts behaving ideally.
11. "van der Waals equation takes into account only the attractive forces between
molecules, and repulsive interactions are not considered".-Comment.
Ans.: The above assertion is incorrect. No doubt, the van der Waals constant 'a' takes
into account the attractive forces. 'b' in van der Waals equation is correction factor for
volume of gas molecules-the molecules cannot come closer than 2r, where r is the radius.
There is infinite repulsion at touch. Thus both attractive and repulsive forces are considered.
12. Can you suggest a condition under which a van der Waals gas will behave like
an ideal gas at all temperatures?
a
Ans. : A van der Waals gas behaves like an ideal gas when b - = 0. Thus if
RT

a van der Waals gas has variables a and b so that at all temperatures, b = __!!_, then the
RT
van der Waals gas will behave like an ideal gas at all temperatures.
13. How do you characterize the deviation from ideality of a real gas?

Ans. : We define the compressibility factor, Z = Vreal =


PV . For ideal gas, Z = 1.
Vi deal nRT
If for a gas Z > 1, then it is less compressible than ideal gas. If, on the other hand, Z < 1,
the gas is more compressible than ideal gas. Thus, the more closer is Z to 1, the more
nearly ideal is the gas.
At the Boyle temperature, TB, all gases behave like an ideal gas within a certain range
of pressure. Thus, the closer is the temperature to TB , the more ideal is the gas.
Z goes on increasing from TB to 218. Above 2TB, Z starts decreasing reaching Z = 1
as T~=. Thus, at very high temperatures every gas behaves ideally.
Thus, we characterise deviation by calculating Z, and/or by noting the temperature.
14. Express the compressibility factor Z of a van der Waals gas in terms of reduced
temperature (T,) and pressure (P,).
Ans. : Z of a van der Waals gas,

a
RTV

Replacing V in the denominator by ideal gas value V = R:,

z -( bP)-I aP
- l - RT - (RT) 2 .

But b
22 PROBLEMS ON PHYSICAL CHEMISTRY

.. z = (1- ;~;J-l
=( 1-si, r- ~~;?
15. Is it possible to have gaseous dissociation without the gas showing any abnormal
vapour density?
Ans. : In majority of cases of dissociation, the number of molecules increases, e.g.,
COClz = CO + Clz. PC1 5 = PC1 3 + Clz, etc. and thus the volume increases at constant
pressure, and hence the density, vapour density and apparent molecular weight decrease.
However, if dissociation takes place without change in the number of molecules, then there
will be no decrease of density or vapour density-there will be no abnormal vapour density.
Some examples are 2HI = Hz + Iz. 2NO = Nz + Oz
16. "The compressibility factor of a real gas is greater than unity at high pressure and
temperature" .--Comment.
Ans. : There is a force of attraction and repulsion between molecules in a real gas.
The repulsive force is of very short range and acts when the molecules come closer at

high pressure, Z = PV >I. At high temperature the molecules move with high velocity
RT
and hence, attraction between gas mo!ect•!~s becomes less important. They collide more
often, and hence, only repulsiVf' forces are important and Z > I. At a very high temperature,
however, intermolecul<>r forces lose importance.
17. Cv of Cl 2 is greater than Nz at room temperature, but at high temperature both
the gases have same Cv. Explain.
Ans. : Heat capacity, Cv, is defined as the amount of heat required to raise the
temperature of a system through I °K at constant volume and the heat absorbed by the system

increases its internal energy, U. Hence, Cv = dQv =


dT
(au)
aT
.
v
Internal energy is composed

of kinetic energy and potential energy of constituent molecules. Intermolecular attractions


and repulsions contribute to the potential energy. At room temperature attractions are
predominant. For non-polar molecules like Nz and Cl 2 dispersion energy mainly contribute
to potential energy. Being larger in size, Cl 2 molecule is more polarizable than N2 and
thus, former has more negative dispersion energy. However both the gas molecules have
the same positive contribution of kinetic energy to internal energy since both are diatomic
molecules. Hence, at ordinary temperature, internal energy is more negative i.e., has greater
absolute value in case of Clz. For the same increase in temperature at room temperature
increase in u for Clz is greater than Nz and it absorbs more heat. Thus CV of Clz is greater
than Nz at room temperature. At high temperature, however, intermolecular atractions of
both the gases become negligib!y small while they have, still, the same kinetic energy. Hence,
they have same internal energy at high temperature and, thus their change in internal energy
is same for the same change in t~mperature. So, now they have same Cv.

18. The experimental value of RT is 2.0 for I mole of a non-ideal gas. The gas is
v
at I atm and below its critical temperature. If the pressure is now halved at constant
temperature, would the new volume be twice of original valume?
GAS 23

Ans. : For the given A gas, Z = PV = 0.5. at P = 1 atm below critical temperature T"
RT
Below critical temperature at 1 atm pressure, Z increases with decrease in pressure.
Thus, the new volume will be more than twice the original volume.
19. A non-ideal gas is at Tc but above Pc. Would its volume be halved of the original
volume !f pressure is doubled at constant T?
Ans. : At critical temperature above Pc, Z increases with increase in pressure. So its
volume will be less than halved, if P is doubled.
20. Two separate bulbs are filled with an ideal gas A and a non-ideal gas B respectively
in such a way that PV remains same. B is below its Boyle temperature. Which gas has
greater temperature?
PV
Ans. : For gas A, Z = - - = 1 and PV = RTA .
RTA

For gas B, Z =-PV- < 1 (since gas is below its Boyle temperature)
RTB
RTA
or, -- < (since PV = RTA).
RT8
TA< TB.
Thus, B has greater temperature than A.
21. Doubling the temperature at constant P will be more than double the volume if
the gas is below critical temperature and the volume will be less than double if it is above
its Boyle temperatur.-Explain. Can the gas be ideal?

Ans. : For an ideal gas, Z, the compressibility ,factor, is always one. Now, Z = PV
RT
So if T is doubled at constant P, V would exactly doubles. It could not be more or less
than double. So the gas is certainly not an ideal gas.
Below critical temperature (Tc), at constant pressure Z will increase with increasing
T Let Z 1 be the corresponding compressibility factor at temperature, T 1 and volume, V1.
If now temperature is doubled to T2 at constant pressure to attain new volume V2 and Z2

is the new compressibility factor, then ~ = Ti V2 = ~ and Zz > Z 1. Hence V2 > V1.
Z2 T2 V1 2V1
Thus V should have more than doubled at T is doubled.
Above temperature twice as high as Boyle temperature, TB, Z decreases with increases
in temperature at constant P, since then Z approaches the ideal gas value. Thus, at constant
pressure, above TB at higher T, Zz < Z 1. Hence, V2 < 2 V1. Thus, V should have less than
doubled as T is doubled.
A3. Numercial Problems
1. What is 200 giga pascal in atmosphere and bar?
Soln. : We know that
I atm = 1.01325 x J05 Pa
1 Pa = 9.86679822 x 10---0 atm.
24 PROBLEMS ON PHYSICAL CHEMISTRY

Thus, 200 giga Pa = 200 x I 0 9 Pa


= 9.87 x 10-6 x 200 x 109 atm = 1.97 x 106 atm.
Again, 1 atm = 1.01325 bar
Hence, 200 giga Pa = 1.97 x 106 x 1.01325 bar.
= 1.996 x 106 bar = 199.6 x 103 bar = 199.6 kilo bar.
2. Convert 1013.25 millibar to atm, Pa and torr.
Soln. : 1 atm = 1.01325 bar= 1013.25 x 10---3 bar
Thus, 1013.25 millibar = 1 atm = 760 torr= 1.01325 x 105 Pa.
3. At what temperature are the Celsius and the Fahrenheit scales equal?
Soln. : We know that °C scale is divided into 100 degrees (0-100) and °F scale is
divided into 180 degrees from 32 to 212.
t°C t°F-32
Thus, - - = - - -
. 100 180
or, 9t °C = 5t °C - 160
By the problem, t °C = t °F = x
Thus, 9x = 5x - 160 or, 4x = -160, :. x = -40°
4. Air is approximately 21% 0 2 and 79% N 2. If 8.0 gm H 2 is now added at 0 °C
in a 22.4 litre flask initially filled with air at 1 atm, what is the average molar weight of
the mixture?
Solo. : At NTP volume occupied by I gm-mole of any gas = 22.4 litres.
Now, by the problem, contents of 1 gm-mole of air
= (0.79 x 28) gm of N 2 + (0.21 x 32) gm of 0 2
= 22.12 gm N2 + 6.72 gm Oz
Hence, l gm-mole of air = (22.12 + 6.72) gm = 28.84 gm.
Now, 8 gm H 2 i.e. 4 moles of H 2 are added.~
Now, mass of air = (28.84 + 8) gm = 36.84 gm.
Total number of moles of gases in air = 4 mole Hz + 1 mole air = 5 moles.

Hence, average mo 1ar mass = 36.84g -- 7 .36 gm.mo 1---1 .


5mol
5. Calculate the number of molecules in a container of 0.1 cc volume at 25 °C
and at a pressure of 10---13 torr.
10-13
Soln. : P = 10--- 13 torr = atm
760

Assuming the gas be ideal, n = PV


to
RT
Here, V = 1 cc, R = 0.08206 l.atm.deg--- 1.mol--- 1, T = (273.15 + 23)K = 298.15 K

10- 13 atm x 0.1 cc x 10- 13 1I cc


Thus, n -
- 760x0.082061.atm.K- 1 .mol- 1 x298.15K
= 5.37799112 x 10---22 mole.
Total number of molecules present = 6.022 x 1023 x 5.38 x 10---2z
= 323.86 = 323 molecules.
GAS 25
6. A mixture of Hz and Nz has a density of 0.2 kg.m- 3 at 27 °C and a pressure of
500 cm of Hg. The total volume of the mixture is one cubic metre. Calculate the mole
fraction of Nz and Hz.
Solo. : Let· the total pressure be P.

Thus, P = __5_00_c_m_o_f_H_,g~-
760 cmofHg.atm- 1
500 ~
= - atm x 1.01325 x H>5 atm
760
T = (27 + 273) K = 300 K
V = I m3

PV 500xl01325Paxlm 3
Now, n = - = 1
RT 760x8.314J.K- .mo1- 1 x300K

500x101325 N. m-2 x I m 3
= = 26.72648 mol..
760x (8.314 x 300)J. mor- 1

Let number of moles of Nz be nN •


2

Thus, number of moles of Hz = nH 2 = 26.7 - nN


2

MN
2
= 0.028 kg.mol- 1

Mtt 2 = 0.002 kg.moi- 1


By the problem,
0.2 kg = 0.028 kg.mol- 1 nN 2 mo! + 0.002 kg.mol- 1(26.7 - nN
2
) mo!

or, 0.2 - 0.0534 = 0.026 nN


2

= 0.1466 = 5.6
0 .. 026

Thus, XN
2
= ~ =0.2
26.7
:. XH2 =I - 0.2 = 0.8.
7. 15.0 I of nitrogen measured at 300 torr is compressed to 1.00 at a particular
temperature. Calculate the pressure of the gas.

Solo. : 300 torr = 300


x 1.01325 x 105 Pa
760
= 3.99967 x 104 Pa.
Now at a particular temperature, PV = constant.
.. Pi V1 = P2V2
4
P1V1 3. 99967x10 Pa x 151
i.e., P2 =-V
- = 11
2

= 5.999506579 x 105 Pa= 6·0000 (±0·001) x 10s Pa


= 600 kPa.
26 PROBLEMS ON PHYSICAL CHEMISTRY

8. A faulty barometer reads a pressure of 750 mm of Hg when the actual pressure


is 760 mm of Hg. The evacuated part of the barometer is 20.0 mm long. Calculate the
actual pressure when the barometer reads 730 mm.
Solo. : The barometer may be faulty if there is some gas in the open space. The pressure
exerted by the gas is (760 - 750) = 10 mm of Hg when the open space has a volume
. of 20A mm 3 (area of barometric tube is A sq. mm). When the barometer reads 730 mm,
then the open volume is [(750 + 20) - 730]A = 40A mm 3 .
Thus, by Boyle's law,
P x 40A = 10 x 20A
or, P = 5.
Hence, the real pressure is 730 + 5 = 735 mm of Hg

or, 735 torr = 735 x 1.01325 x I 0 5 Pa


760
= 97.99 kPa.
97991.94079 Pa =
9. (a) A faulty barometer reads 740 torr when the pressure is 750 torr. The barometric
tube is 790 mm long. What pressure will the barometer read when the real pressure is 760
mm of Hg?
Solo. : By the problem, the gas occupies a volume of 50 A mm 3 and exerts 10 torr
pressure. Let the barometer read x torr when the real pressure is 760 torr. The open space
then (790 - x) mm and the gas pressure is P'.
So, ?'(790 - x) A = 10 x 50A

. P' __ 500
1.e., torr.
790-x
But P' + x = 760.

500
+x = 760
790-x
i.e., xZ - l550x + 599900 = 0.
t55o- ~1550 2 -=-~s-99900
• Solving, x = 2
= 748.07418 torr = 748 torr.

(b) A faulty barometer reads 760 mm of Hg when the real pressure is 768 mm of
Hg. There is an open space of 32.0 mm in the barometer. What will the barometer read
when the real pressure is 745 mm of Hg? [Ans. 740 mm]
10. A one-litre bulb is filled at 300 K with acetylene at 1.00 bar and oxygen at 5.00
bar, and the mixture is exploded. Calculate the final pressure if the final temperature is
(i) 300 K, (ii) 353 K and (iii) 600 K. Aqueous tension at 300 K is 26.7 mm of Hg and
at 353 K is 353 mm of Hg.
Solo. : When explosion takes place, CzHz reacts with Oz as follows
2C 2Hz + 50z = 4COz + 2Hz0 (I)

Under given condition, P oc n (n = :~). l mole of CzHz reacts with 2.5 moles of

Oz to give 2 moles of COz. Thus, l bar of CzHz is completely consumed. The remaining
will be 2.5 bar of Oz and 2 bar of COz. Therefore, the total pressure is now
GAS 27

Pco + P0 + aq. tension at 300 K


2 2
26 7
= 2 + 2.5 + ( · xl.01325)bar
760
= 4.5 + 0.03559 = 4.536 bar = 4.54 bar.
At 353 K,
Pco + P0 = 4.5 bar at 300 K.
2 2
4.5 x 353
at 353K, Pco + P0 = = 5.29 bar.
2 2 300
Pco + PH 0 + aq. tension = 5.76 bar.
2 2
At 600 K. H 2 0 is gaseous. As the gas at 300 K, there is a pressure of 4.5 + l = 5.5
bar which at 600 K will be 11 bar.
11. A one-litre bulb at 27 °C is filled with ethylene at l bar and oxygen at 5 bar. The
mixture is exploded. Calculate the final pressure if the final temperature is (i) 27 °C and
(ii) 327 °C. Use data of problem 5. [Ans. 4.04 bar, 12 bar]
12. 11 of N2 at a pressure of 2.1 bar in a 41 flask mixed with 21 of 0 2 at a pressure
of 3.4 bar at a constant temperature without any reaction taking place. Suppose that initial
temperature of 0 2 is 304 K and that of N2 is 402 K. If final temperature is 377 K, what
will be final pressure?
PV
Soln. : For an ideal gas, n =
RT
Initial PN 2 = 2.1 x 105 Pa (l bar= 105 Pa)
VN = II = l x 10-3 m 3
2
P02 = 3.4 bar= 3.4 x 105 Pa
V02 = 21 =2 x 10-3 m3

2.1 x l0 5 Pa x (1x10-3 m 3 ) 2lON.m-2 .m 3


Thus, nN 2 = ( ) =
1
R ).K- .mol- 1 TN K RTN 2 J.mo1- 1
2

201 N.m 210


= =--mol (·: N.m = J)
RTN 2 J.mol- 1 RTN 2

680
= - - mol.

Given, TN = 304 K, T02 :::: 402 K


2
2lOJ 680J
Thus, total number of moles = nT = nN 2 + n 02 = R( k) + R( k)
304 402
Let P1 be final pressure.
, nTRT
1hus, P f = - -1, T1 = 377 k, v1 = 41 = 0.004 m3
VJ
28. PROBLEMS ON PHYSICAL CHEMISTRY

210J 680J R377K


Hence, pf = [ R304K + R4;02K 0.0004m 3
J
= 224534.77 Pa = 224.53 kPa
13. It takes 0.37 gm of N2° to fill a glass tube at 298.2 Kand 0.01 bar pressure. Under
the same set of condition it takes 0.92 gm of an unknown gas (homonuclear diatomic) to
fill the same tube. Find out what the unknown gas was.
Solo. : Let M = molar mass of unknown gas,
W = mass of unknown gas.

Thus, number of moles of unknown gas = n =


w
M
Here, W = 0.92 gm.
By Avogadro's law, same volume of all gases contains same number of molecules at
same temperature and pressure.
Given : WN 2 = 0.37 gm.
MN 2 = 28 gm.-mo1- 1
0.37gm 0.92gm
Hence, n =28gm.mol- 1 =
M

0.92gm x 28gm.mo1- 1
or, M = = 70.7 gm.mol- 1 71 gm.mol- 1
0.37gm
Therefore, the unknown gas was chlorine (Cl2 )
14. Find the density ~f fluorine in S.I. unit at 27 °C and 288 cm of Hg pressure assuming
that it behaves ideally.
Solo. : We know that MF = 38 gm.mol- 1 = 0.038 kg.mol- 1
2
Given that T = (273.15 + 27) K = 300.15 K
P = 288 cm of Hg

2880 mm 'of Hg x (t.01325 x 10 5 ) Pa.atm- 1


= = 383968.4211 Pa.
760mmofHg.atm- 1
1 5
MP
Now, density, p = RT
= 0.038kg.mo1- x3.83968xl0 Pa
8.3!4N.m.mo1- 1 .K- 1 x300.!5K
= 5.8469 kg.m-3 (! Pa = I N.m-2)
In C.G.S. unit, p = 0.0584 gm.cm-3 .
15. 2 glass bulbs of equal volume are connected by a narrow tube and are filled with
a gas at 273.15 K and 1.00 bar. One of the bulbs is then placed in melting ice and the
other in a water bath at 62 °C. What is the new value of P inside the bulb.
[1.1.T. Entrance, 1985]
Solo. : Initially the volume of the two bulbs was 2V and there were
1x2V 2V
n =
Rx 273.15
=- -----
0. 08314 x 273.15
mole.

Under new condition, let the pressure be P. Then


GAS 29

PV PV
" 1 = 0.08314x273.15; nz = 0.08314x335.15
So, n 1 + n 2 =n
,2v
or,
0.08314x273.15 0.08314x335.15 0.08314x273.15
p p 2
--+---=--
273.15 335.15 273.15
608.3? 2
or,
273.15 x 335.15 273.15
2x335.15
:. p = = 1.10923392 bar = 1.109 bar.
608.3
16. 2 glass bulbs of volume 3 litres and I litre respectively are connected by a narrow
tube of negligible volume. Air at a pressure of 1 bar at 30 °C is contained in the apparatus
which is then hermetically sealed. If the 3-litre bulb is now suddenly immersed in steam
at JOO °C, what would be the pressure of air in the bulbs? [J.E.E. 1981)
[Ans. 1.16 bar]
17. 2 bulbs of volume 600 cc and 300 cc are connected by a short tube of negligible
volume at 27 °C. Th'e pressure is 760 torr. Now, the temperature of the larger bulb is quickly
changed to 100 °C. What will be the final, pressure? [Ans. 1.16 bar]
18. Spheric~! baloons of 15 cm diameter are to be filled with oxygen from a cylinder
containing the gas at 25 bar and 0 °C. If the cylinder can hold 2 litres of water at I bar
pressure, calculate the number of baloons that can be filled up. How much oxygen will
go out?
3
l = 1767.375 cc = 1.767 x 10-3 m 3.
Solo. : Volume of the baloon is 4 rt (15) 3"
The cylinder will hold 2 litres of 0 2 at 1 bar.
Thus at 25 bar it can hold 50 litres = 50 x 10-3 m3.
Consequently the baloons will get (50 - 2) 10-3 m3 = 48 x 10-3 m3,
since 2 litres will remain in the cylinder.
48x10- 3
Hence, it can fill ---~,
I. 767 x 10-·
= 27.1646859 = 27 baloons.

Naturally, 0.1646859 x 1.767 x 10-3 m3 = 2.91 x


10-4 m 3 = 0.2911 0 2 will go out.
19. A ~pherical balloon of 21 cm diameter is to be filled up with hydrogen at STP
from a cylinder containing the gas at 20 bar at 27 °C. If the cylinder can hold 2.82 litres
of water, calculate the number of balloons that can be filled up. [Ans. 10]
20. 0.540 gm of an organic liquid displaces 71.52 cc of dry air at 99.80 °C. The
barometer pressure was 691.4 mm of Hg. The compound is saturated and contains only
carbon, fluorine and chlorine. There is 57% Cl 2 by weight. Determine its molecular formula.
Solo. : 99.8 °C = 372.95 °K
PV = nRT = W RT
M
30 . PROBLEMS ON PHYSICAL CHEMISTRY

WRT 0. 54 x 0. 08206 x 372. 95


M =
PV
= 691.4 11.52
gm
--x--
760 1000
0. 54 x 0. 08206 x 372. 95 x 760 x I 000
= gm
69J.4X7J.52
= 253.999 gm = 254 gm.
Let the formula of the compound be CnC1)< 2n+ 2-x.
So, 12n + I 9(2n + 2 - x) + 35.5x = 254.
35 5
% of Cl 2 = 57 = xx · x 100.
254
57 254
:. x = x = 4.078309859 = 4.
35.5 x JOO
Thus, 12n + 19(2n - 2) + 142 = 254
or, 50n = 112 .+ 38 = 150.
n = 3.
The formula of the compound is C3Cl4F4•
21. A meteorological balloon had a radius of I m when raised from sea level and
expanded to a radius of 3 m when it has risen to its maximum altitude. When the temperature
is -20 °C, what is the pressure inside this balloon at this altitude? [B.U.(H) 1996]
Solo. : Assuming sea level to be at SATP,
T; = 298.lSK TF = 253.15K
P; = I bar PF =?
r; = 1m rF = 3m

PFVF = P;V;
TF T;

4 3
37tr; 253.15
= --x-- x I bar
.i 7tr3 298. 15
3 F

3
=
253.15
'313 x 298 _1
5
bar = 0.0314 bar.
22. The Celsius scale is based on 0 °C as the melting point of ice and I 00 °C as the
normal boiling point of water, the respective limiting PV values are 22.71 bar-1.moJ- 1 and
31.02 bar-l.moJe- 1.
A Chemistry Honours student Sandip proposed a special scale on an unknown liquid
taking 0 °SM as the freezing pt. of that liquid and I 00 °SM as the normal boiling pt.' of
that liquid. He also measured that corresponding limiting PV values and found these to
be 20 and 33 bar-1.mole- 1 respectively.
(a) Calculate the m. pt. and· b. pt. of the unknown liquid in °C.

(b) Calculate absolute zero in OsM and the value of the gas constant in bar.I. Ts~
where TSM is absolute temp. in SM Scale.
GAS 31

Solo. : Now 100 °C = 31.02 - 22.71 = 8.31 in PV units.


Thus, temperature in °C may be given by

t 0C = IOO(PV- 22. 71).


8. 31
Melting point of the unknown liquid is then

JOO x 20-22.71 = _ I00x2.7l = _32.61 oc.


8.31 8.31
Boiling point of the unknown liquid is
33-22.71 lOOxl0.29
I 00 x
8.31
= 8. 31
= 123.83 °C.
Now, P1V1 - P2V2 = R.
T T+lOO
20 33
.. - = - - -
T lOO+T
or, l3T = 2000, :. T = 153.85.
0 5M = -153.85 °SM.
Hence, TsM = 0 5 M + 153.85
20
R will be = 0.1299 1-bar.TSM-I .mot-I
153.85
= 0.13 1-bar.TSM-I .mot-I.
23. With the data of the Celsius scale in hand as in problem 14, another student found
0 °n as the f. pt. of another liquid and 100 °n as the b. pt. of that liquid and found PV
as 10 bar.litre.mot-I and 25 bar.litre.mot-I respectively.
Calculate the m. pt. and b. pt. of the unknown liquid in °C and absolute zero in °n
and gas constant R. [Ans. -152.9 °C, 27.5 °C, -66.7 °n, 0.15 bar.litre.mo1-I.T0 -I]
24. The inhabitants of a planet have a centigrade scale, but based on 0 °P as them. pt.
of a substance X in that planet and t = 100 °P as the b. pt. of that substance. They know
about ideal gas behaviour and find limiting PV product to be 20 at 0 °P and 25 at I 00 °P.
Calculate the temperature of absolute zero in degree P.
Hints : A diff. of 100 °P reduces PV by 5 units. So 20 will be reduced to zero at
-400 °P.
25. The space above mercury column in a thermometer is ordinarily evacuated, but
due to faulty manufacture, a particular thermometer has a pressure of 0.665 kPa in this
space when the whole thermometer is immersed in a bath at 0 °C. Calculate the pressure
when the whole thermometer is immersed at 100 °C. At 0 °C, the length of the air space
is 0.012 m, and at 100 °C it is 0.002 m.

Hints : ~ = 0.012A = 6
V2 0.002A

373.15
- - x 6 x 0.665 = 5.45 kPa (40.98 mm of Hg).
273.15
32 PROBLEMS ON PHYSICAL CHEMISTRY

26. A gaseous compound X contained 44.4% carbon, 51.9% nitrogen and 3.7%
hydrogen. Under like conditions 50 cc of X is diffused through a porous plug in 25 sec and
the same volume of hydrogen diffused in 6.8 sec. Deduce the molecular formula of X.
Solo. : Let the vapour density of X be Vc1 ; that of hydrogen being Vc1 . Let the
velocity of diffusion of X be Vx and that of ~ be VH . H
2
2

_!_ oc ~
1
by Graham's Law : V oc
x 25 vdx .

Since VH
2
oc -
1
-
6.8
oc ~ VdH1
• ,
2

6. =
- 8 l rv- (smce,
. vd = l ).
25 'V Vdx H2

2
(25)
vd = - - = 13.51643599 = 13.5.
x (6.8) 2
Thus, molecular weight of X in C.G.S. scale is 27.
In the compound, atomic %

of c = 44.4 = 3.7
12

of N =2!:2_ = 3.7
15555

of H=~ = 3.7.
1
.. C:N:H= 1 l l
Formula : (CNH\ = 27.
x = l.
The compound is HCN.
.27. The time taken for a certain volume of gas to stream through a small hole is 0.96
· min while an equal volume of oxygen took 1.2 min to pass through. Calculate the approximate
molecular weight of the gas. [Ans. 20.51
28. If 5 gm of ammonium carbamate dissociates to 2NH 3 and C0 2 at 200 °C, the
substance occupies a volume of 7.66 litres at a pressure of 740 mm of Hg. Calculate the
degree of dissociation.
Hints : NlhCOONH4 ~ 2NH3 + C0 2
78 gm occupies 3 x 22.414 = 67.242 1 at NTP. Hence, 5 gm occupies 4.3108 I at
NTP.
.. V = P0 V0 x T = 760x4.318 x 473.15 = 7 .68 litres.
T0 P 273.15 740

7.66 0
.. - - x 100 = 99.74, :. ·a= .99.
7.68
GAS 33
29. Calculate the density of moist air in gm-litre- 1 of moist air at 298.15 K and
l bar pressure when dry air contains 79% nitrogen and 21 % oxygen by volume. The relative
humidity of moist air is 60%. The aqueous tension at 298.15 K is 0.032 bar. Given,
R = 0.0821 J.atm.K- 1.moJe- 1•
Solo. : Volume fraction is proportional to mol-fraction which again is proportional
to total pressure.

.. PN2 = 0. 79 = 3.7619 (l)


Po2 0.21
R = 0.0821 l-atm.K- 1.moJ- 1 = 0.0821 x l.01325 l-bar.K- 1.moJ-I
= 0.0831 l-bar.K-1.mo1-1. (2)

Wgram PMbar.gm.mo1- 1 PM _
p = = = RT gm.I 1. (3)
Vlitre RT bar. litre. mo1- 1
But, w = WN2 +Wo2 +WH20
WNz + Wo2 + WHzO PN2MN2 + Po2Mo2 + PH20MH20
.. p = = (4)
v RT
Now, PN + P 02 = l bar - 0.032 x 0.6 bar (since, relative humidity = 60%)
2
= (l - 0.0192) bar= 0.9808 bar.

PN2
Now, - + l = 4.7619 [from eqn. (!)].
Paz

0.9808 bar
= = 0.2059 bar.
4.7619
PN = (0.9808 - 0.2059) bar = 0.7748 bar.
2
0. 7748x 28+0.2059 x 32+0.0192x18
From eqn. (3), p = - - - - - - - - - - - - -
0.08314 x 298.15
28 629
= - - -· - - - gmil'1tre = 1155
. g mil"Itre.
0.08314x298.15
30. Solve problem 29 if the total pressure is l atm and aqueous tension at 25 °C is
23.76 mm of Hg. · [Ans. d = 1.171 gm/I]
31. At 300 K, l 000 cc of H 2 measured under a pressure of 600 mm of Hg and l 000 cc
of N2 measured under a pressure of 400 mm of Hg are introduced into an evacuated vessel
having a volume of 4 litres. Calculate the resulting pressure.

Hints : P = PH 2 + PN 2 = _!_ x 600 +_!_ x 400 = 150 + 100


4 4
=250 mm of Hg.
32. The compressibility factor of 0 2 is 0.927 at 0 °C, and 100 atm pressure. Calculate
the weight of 0 2 necessary to fill a cylinder of JOO litres capacity at the given condition.
Hints : Given, T °K = 273.15 K
P = 100 atm, Z = 0.927
Prob. Phy. Chem.-3
34 PROBLEMS ON PHYSICAL CHEMISTRY

PV lOOx 100 l.atm


n = ZRT
= 0.927x0.08206 l.atm.K- 1.mol- 1 x273.15°K
= 481.26233 = 481.3.
:. wt. in kg = 481.3 x 32 gm = 15401.6 gm = 15.4 kg.
33. The highest allowed presssure of a 20-litre flask containing 1.6 kg of oxygen is
150 atm. To what temperature can it be heated? Assume that 0 2 obeys van der Waals equation
with a = 1.360 litre 2.atm.mole-2 and b = 31.83 cm 3.mole- 1.

(P+ a;: )(V-nb)


Hints : T = [Ans. 7ll.95K = 712K]
nR
34. Calculate the value of R in S.I. unit from the following data. A 20.00 1 container
was filled with 0.25132 gm of He gas and heated up to 500 °C. A student doing this experiment
measured the pressure which he found to be 206 cm of water in a manometer at 27 °C. If
that density of water is 0.9965 gm.cm- 3 at 25 °C and molecular wt. of He is 4.002 gm, did
he measure the pressure correctly?
Solo. : Given that V = 20.00 1 = 0.02 m3
T = (500 + 273.15) K = 773.15 K
Mol. wt. of He = MHe = 4.0026 gm
Mass of He= 0.25132 gm
25132
Hence, number of moles of He = n = O. gm = 0.06279 mol.
4.0026 gm.mo1- 1
By the problem, P = 206 cm of H 2 0 = 206 x 10-2 m of H 2 0, density of water =
p = 0.9965 gm.cm-3.
Thus, P = hpg
= 206 x 10-2 m x 0.9965 x 103 kg.m- 3 x 9.80665 m.s-2
= 20130.9931 N.m-2 = 20130.9931 Pa.

PV 20130.9931Pax0.02m 3
Assuming the gas to be ideal, R = =~~~~~~~~~

nT 0. 06279 mol x 773.15 k


= 8.2936 J.K- 1.mo1-1.
The reading of the student was obviously inaccurate. Since R = 8.3146 J.k- 1.mol- 1,
he should have read the pressure as 206.5 cm of water to get the correct value of R.
35. Pressure exerted by a sample of nitrogen is 202 k Pa at 300 K. Volume occupied
by it under this condition is 1 m 3 . If it is now compressed to a volume of 400 1, calculate
its new pressure.
Solo. : By the problem,
P1 = 202 kPa P2 =?
vi = 1 m 3 V2 = 400 x 10- 3 m 3
T 1 = 300 K T2 = 400 K
GAS 35

__PV T _ 202 k Pa x l m 3
i _i x-2 400 K
P2 - - -----x---
300K 0.4m 3
Ti Vz
= 673.32kPa.
36. The air space in a driving bell is 4.00 m3 on the deck of a boat moving in the
sea. What will be volume of the air space when the bell is taken to a depth of 60 m. Given
that density of sea water is 1.026 gm.cm- 3 ?
Solo. : By the problem, V 1 = 4 m3
P 1 = l atm (say) = 101.325 kPa
p = 1.026 gm.cm- 3
h = 60 m
Vz =?
Pz = P1 + hpg
P2 = 101.325 kPa + (60 m x l.026 x 10 3 kg.m-3 x 9.80665 m.s- 2 )
= I 0 l.325 kPa + 603.69737 kPa = 705.02237 kPa = 705 kPa
If we assume temperature remains constant,
3
Vz =Pi Vi = 101.325 kPa x 4m = 0. 5748 m3.
P2 705 kPa
37. In a glass globe of a particular size, chemical nitrogen prepared by heating NH4 N0 2
(a mixture of NaN0 2 and NH4 Cl) is found to have a mass of 2.2991 gm. In the same
globe nitrogen, made from atmosphere by removing 0 2, C0 2 and water vapour, had a mass
of 2.31018 gm. Suppose that atmospheric nitrogen and argon have molar masses 28.02 gm
and 39.952 gm respectively. Calculate the mole fraction of nitrogen in the mixture.
Solo. : Let T and P be same for both cases. Then same volume of gases will contain
same number of moles.
2.2991
Mole number of N 2 =- - = 0.0820521.
28.02
Let there be A mole of N2 and B mole of Ar in the nitrogen made from air.
Thus,28.02A + 39.9528 = 2.31018 (I)
and A + B = 0.0820521
.. B = 0.0820521 - A (2)
Putting the value of B in Eqn. (I), we get A = 0.081123 mole.
0 081123
Thus, X = · = 0.9886
A 0.082052
and · X 8 = 1 - 0.9886 = 0.0114.
Alternative method :
Let there be A gm of Ar and B gm of N2 , then A + B = 2.31018 g
8 2 2991
and _A_+-- = · = 0.0820521
28. 02 39. 952 28. 02

or, _A_+(2.31018-A) = 0.082052.


28. 02 39. 952
36 PROBLEMS ON PHYSICAL CHEMISTRY

A can be found out from the above equation and nA = _A_ , XA = nA


28.02 nA +n 8
38. At a pressure of 1 atm and a temperature 296 K, density of air is 1.19 gmll. Calculate
the mole fraction of nitrogen in this mixture. There are only oxygen and nitrogen in air.

Soln. : We know, for an ideal gas, M =pRT


p
.

By the problem, p =1.19 gm/\, T = 296 K, P = 1 atm, R ·= 0.08206 l.atm.K- 1. mol- 1


M = 1.19gm.1- 1 x0.082061.atm.K- 1.mol- 1 x296K = 28 .90481 gm. mol-1
latm
Let x be the mole fraction of 0 2 and then (I - x) be the mole fraction of N 2 . If we
assume there is one mole of air, then
Xo2 +(l-x)N2 = I.
x x 32 + (I - x) x 28 = 28.90481
or, 4x = 0.90481 :. x = 0.22.
Thus, mole fraction of nitrogen = (I - x) = 0.78.
39. One mole of gas A at 66 Kand 3.2 atm and gas Bat 608 Kand 18.2 atm occupy
a volume of 1718 cc and 2782 cc respectively. Their critical constant values are shown
below. Find the missing volume in the table and state which gas is more easily liquefiable
and which is more close to ideal behaviour at STP?
Gas Tc Pc (atm) V/I)
A 33 K 12.8 0.065
B 304 K 72.8
Soln. : For gas A, the reduced temperature and pressure under the given condition
66 3.2
are Tr= - = 2, Pr= - - = 0.25, and for gas B the reduced temperature and pressure
33 12.8

are Tr = 608
304
18 2
= 2, Pr = 72.8
· =0.25. Since these are same, they will have same reduced

718
volumes. Vr of A is I. = 26.43.
0.065
:. Vr of B will be 26.43.

Hence, Ve for B is ~.
vr
2 782
(V) = · litre = 0.1052 litre.
.• c B 26.43
(Assuming that they have the same Zc, (Vc)B may also be calculated.)
Tc of B is higher-it can be liquified at 304 K. Thus, in comparison to A (liquefiable
by application of pressure up to 33 K only), B is more easily liquefiable.
27
Boyle temperature for A, (T8 )A = x 33 = 111.375 °K
8

Boyle temperature for B, (T8 ) 8 = ~7 x 304 = 1026 °K:


GAS 37

At STP, A is above twice its Boyle temperature while B is much below its Boyle
temperature.
Naturally, at STP, A will be more close to ideal behaviour.
40. Gases A and B obeying van der Waals equation have Tc and Pc values given below:
Tc (°K) Pc (atm)
A 44 26
B 304 72
Which gas (i) has higher Ve value and (ii) shows more nearly ideal behaviour at 25 °C
and 10 atm pressure? Give reasons for your answer. [C.U.(H) 1992, V.U. 1998]
Solo. : (i) For a van der Waals gas,

RTC 8
= -
PCVC 3
3RTc
.. Ve =
8Pc
3 x 0.082061.atm.K- 1 .mo1- 1 x44K
.. (Ve )A =
8x 26 atm
= 0.0521.mol- 1.

1 1
= 3 x 0.082061.atm. K- • mol- x 304K =0. 1298 l.mol-1= 0.1 3 l.mo1-1.
8x 72 atm
Thus, B with very high Tc has higher Ve value.
27
(ii) Boyle temperature for A, (T8)A = (Tr)A
8

= 278 x 44 °K = 148.5 °K = -124.5 °C.

Boyle temperature of B, (T8 )8 = -27 x (Tc)B = -27 x 304 °K = 1026 °K = 753 °C.
8 8
Boyle temperature of B is much above 25 °C than that of A below it. A is close to 2T8
at 25 °C. Thus, A will show more nearly ideal behaviour than B at 25 °C and 10 atm.
41. The following table gives the values of a, band experimentally measured volume
at 273 K and 200 atm of two van der Waals gases A, B :
a b Volume
atm.1 2 .mol-2 litre.mole- 1 litre.mole- 1
A 0.24 0.026 0.112
B 1.36 0.032 0.111
Calculate (i) the compressibility factor for 1 mole of each at 273 K and 200 atm and
hence state which one is more compressible than predicted by the ideal gas law, (ii) the
temperature at which B obeys Boyle's law over a wide range of pressure and (iii) the
temperature below which A would be subjected to Joule-Thomson expansion for liquefaction.

Solo. : (i) Compressibility factor Z


·
= PV
RT
38 PROBLEMS ON PHYSICAL CHEMISTRY

200x0.I 12
. . ZA = 0. 082 x 273 = 1.000625391 = 1.0006 = 1.00
200x0.I11
Z8 = = 0.99169 = 0.992.
0.082 x 273
Thus, B is more compressible than an ideal gas.

(ii) The temperature is the Boyle temperature : T8 = _!!._


Rb
6 36
(T)
8 = 1.3 = 1. = 518.29 K = 578K.
s Rx 0.032 0.082 x 0.032
(iii) The temper~ture is inversion temperature.
2a
= Rb

2 x 0.24
: . (T)1 = = 225 .14 K = 225 K.
A 0.082x0.026
In this problem R is chosen as 0.082 l.atm.K- 1.mole- 1 rather thau 0.082 I since choice
of R = 0.08206 will lead to both ZA and Z8 less than or:c. What is more, there are only
2-3 significant figures in the reported data.
42. Gases A, B, C and D obey van der Waals equation with a and b values in SI
units given below :
A B c D
a 0.6 0.6 0.2 0.005
I03b 0.025 0.15 0.10 0.02
Which of the gases has (i) the highest Tc (ii) the largest molecules and (iii) the most
ideal bahaviour at STP? [C.U.(H) 2003)
Hints : Calculate Tc and T8 .
T8 of D = 30.IK, C = 240.5, B = 48IK, A = 2886 K
D is most ideal at STP (above 2T8 ).
43. Gases NO and CH4 obeying van der Waals equll:tion have Tc and Pc values given
below:
Pc (atm)
64
45
Which gas has (i) smaller value of a (ii) smaller value of b (iii) larger value of Ve
and (iv) which gas is nearly ideal in behaviour at 300 K and 10 atm pressure? Give reason.
[C.U. 1998)
Hints :

= RTc = 0.028 I.mol- 1


8P,
2 2
27 R T
= ___ c = 1.392 atm.I 2 .mole~ 2
64?,
GAS 39

= 0.125 l.mole- 1
= 19.115 atm.1 2.mote-2
= 3b
1.392
0.0821x0.028
= 605.53 °K

aCH4 = J9. J 15
(TB)CH4 = RbCH 4 0.082lx0.125
= 1862.61 °K.
T8 of NO is more close to 300 K than that of CH4 . So NO is more nearly ideal at
300 K and I 0 atm.
44. At 300 °C, one mole of ether occupied a volume of 740 cc. It is a van der Waals
gas with a= 17.4 atm.1 2.mole-2 and b = 13.4 x 10-2 1.moJ- 1. Calculate the pressure. What
would have been the pressure if it were an ideal gas? Hence, calculate Z.
Solo. : For I mole of a van der Waals ·gas,
T = 573.15 K
V = 0.74 litre
b = 0.134 I.mole-I
a = 17.4 atm.1 2.mole-2
R = 0.08206 l.atm.deg- 1.mole-I
a 0.08206 x 573.14 17.4
p = vRT
- b - 0 = 0. 74 - 0.134 - 0. 74 2
= 77.61169802 - 31.775018
= 45.83667976 = 45.8 atm.
0 08206 573 15
P RT = · x · = 63.55768784 = 63.6 atm.
ideal= V O. 74

p 45.8
z = - - =- - = 0.72012579 = 0.72.
pideal 63.6
Under the given condition, the gas is more compressible than ideal gas.
45. The pressure exerted by 3.02 x 1020 molecules of a gas in a 2 litre vessel is 1.52
cm of Hg. Calculate the temperature of the gas if the gas is ideal. What will be the temperature
if the gas is van der Waals or Dieterici with same a = 1.38 atm. 12.moJe-2 and b = 0.039
I/mole.

Solo. : For ideal gas, T = PV


nR

1.52 2
--x
= 76 K
3. 02 x I 020 x 0. 08206
6.022x 10 23

= l.52x2x6.022x1023 K = 971.99K = 972K.


3. 02 x 10 20 x 0. 08206 x 76
40 PROBLEMS ON PHYSICAL CHEMISTRY

3.02x10 20
n = = 5.01 x 10--4.
6.022x10 23
For van der Waals gas,

(P+~ )(V-nb)
nR
= T.
2
152 1.38x(5.0lx10-4) )( -4 )
~{;+ 2-5.0lxlO x0.0394
( 22
T = 5.01x10-4 x 0.08206

(0. 02 + 8.65 x 10-S )( 2 -1. 97394 x 10-5 )


= 5.01x10-4 x 0.08206
= 0.020000086x 1.999980261 = 972 .9451217 K = 973 K.
5.01x10-4 x 0.08206
For Dieterici gas,
-nA
P(V - nb) RTV
T = ---~e

nR
.

3
In T = In 972 .94 _ 4.210649281x10-
T

Neglecting ..!_ terms, T


T
= 972.94 = 973 K.
46. For a particular gas, b = 4.42 x 10-2 1.mole- 1• How near can the centres of two
molecules approach? [C.U.(H) 1987]
3
Soln. : b = 44.2 cm .mole- 1

4 3
= 4NA x 3m.
3b 3x44.2cm 3
= 16NA1t = 16x6.022xl023 x3.142
= 4.38002 x 10-24 cm3
r = 1.63615 x 10-8 cm.
The centres can approach as close as 2r (cr).
.. cr = 3.2723 x 10-8 cm
= 3.27 x 10- 10 m
= 327 pm.
47. Given that Pc= 48 atm and Tc= 155 K for Argon. Calculate the radius of Argon
atoms.

Hints : b = RT, = 33.123 cm 3


SP,
r = 1.49 A = 149 pm.
GAS 41

48. Using van der Waals equation, calculate the pressure for 2 moles of N 2 occupying
a volume of 5 1 at 40 °C.
Given, a =
1.38 atm.12.mole-2, b = 0.0399 l.mole- 1, R = 0.0821 l.atm.deg- 1.mol- 1.
[C.U.(H) 1974]
[Ans. P = 10.2 atm, Pid = 10.3 atm]
49. For C0 2 , Tc = 31 °C and critical density is 0.341 gm/cc. Its molecular weight
is 44. Calculate a and b and radius of C0 2 . [C.U.(H) 1977]
[Ans. Ve = 129.03 cc, b = 43.01 cc/mole,
a = 3.6 atm.1 2 .mole-2 , r = 162 pm]
SO. Calculate the radius of a van der Waals gas molecule if b = 3.2 x 10-2 l.mo1- 1
and b = 4.42 x 10-3 I.mole- 1. [C.U.(H) 1997]
[Ans. r = 147 pm, r = 75.9 pm]
51. 3 gm-moles of C0 2 occupy l 0 1 at 15 bar. Calculate its temperature if it is
(i) a van der Waals gas, (ii) Dieterici gas and (iii) ideal gas. Given, a =3.59 bar.1 2.mole-2
and b =
0.043 1.mole- 1• [Ans. (i) 606.5 K, (ii) 607.2 K, (iii) 601.4 K]
52. Assuming the intermolecular attraction to be negligible in case of Helium gas, and
taking its molar volume at 0 °C and 100 atm to be 1.107x 10-2 times the volume at NTP,
calculate the diameter of He atom. [C.U.(H) 1983]
Hints : a =0
:. P(V - b) = RT.
By the problem, 100 (l.107 x 10-2 V - b) = 273.15 R
l(V - b) = 273.15 R

or, l.107V - lOOb =l


V-b
07
or, b =· 10 V = 0.0242251 1.mole- 1.
99
er= 268 pm.
53. If the compressibility factor Z for a van der Waals gas be 1.000054 at O °C and
atm, and the Boyle temperature be 107 K, calculate neglecting the higher terms of P,
the values of a, b and molecule diameter. [C.U.(H) 1994]
Solo. : For a van der Waals gas, neglecting the higher terms in Virial form-

z = 1+ ~(b-_E_)
RT RT
= 1.000054

.. b - _E_ = 0.000054 x RT
RT p
= 0.000054 x 0.08206 x 273.15
= 1.2103932206 x 10-3
= 1.21 x 10-3.

TB = ...!!__ = 107
Rb
a = 107 Rb.
42 PROBLEMS ON PHYSICAL CHEMISTRY

b - _I0_7_·_H_b_ = 1.21 x 10-3


Rx273.15
or, b - 0.391726157b = 1.21 x 10-3
or, 0.608273842b = 1.21 x 10-3.
b = 1.988 x 10-3 1.mole-1•
a = 107 x 0.08206 x 1.988 x 10-3
= 0.017466324 atm.12.moJe-2
= 0.0175 atm.1 2.mole-2.
3x b 3x1. 988
r3 = 16NA.1t = 16x6.022xl0-23 x3.142
= 1.97002 x 10-25 cc.
r = 5.81868 x 10-9 cm = 0.58 A= 58 pm.
cr = 116 pm.
54. A certain vapour obeys the van der Waals equation with a =0.52 m6 .Pa.mole- 2,
Its volume is 4.99 x 10-4m 3.mole- 1 at 300K and 3.20 x 103 kPa. Calculate the value of
van der Waals constant., Find the temperature at this volume, when P = 0.
Solo. : van der Waals equation for I mole is

(P+ ; 2 ) (V - b) = RT.
RT
:. V- b = ---
P+!!-
V2
By the problem,
V = 4.99 x 10-4 m3
a = 0.52 m6. Pa. mole- 2
T = 300 K
R = 0.08206 J-atm.deg- 1.moJe- 1 = 8.31473 m3.Pa.deg- 1.mole- 1.
P = 3.2 x 106 Pa
V - b = 4.7164 x 10-4m3
/;J ....... ;:::4.99 x 10-4 - 4.7164 x 10-4
' .. .
= 2.7359 x 10-5 m 3.mole- 1 = 2.74 m3.mole- 1.
Now p = V-b
RT
-
a
V2 .

When P = 0,
RT 0.52
0 = 4. 7164x10-4 2
(4.99x!o-4) ·

V- b a 4. 7164 x 10-4 x 0. 52
T = --x- 2 = 2
R V 8.31473 x (4. 99x 10-4 )

= 118.45809 K "" 118 K.


GAS 43

Since there is attraction, P turns out to be zero above absolute zero. This only indicates
the existence of molecular attraction.
55. One gm-mole of a gas PQ which is dissociated to an extent of 10% at 127 °C
according to PQ = P + Q occupies a volume of 4 x 104 cc. Calculate the total pressure
at this temperature assuming the gas to be perfect. [C.U. 1975; B.U. 1974)
Solo. : Let there be a gm-mole of PQ.
PQ=P+Q
(1 - ·I )a ·la ·la
V = 40 litres.
n = 1.1 a mole = 1.1 mole (since, a = 1)
R = 0.08206 l.atm.deg- 1.mole- 1
08206 400
P = I.Ix O. x = 0.90266 atm = 0.903 atm.
40
56. If the temperature above which a van der Waals gas cannot be liquefied be 32.3 °C
and minimum pressure to be applied at that temperature for liquefaction be 48.2 atm,
(i) find the diameter of gas molecule.
(ii) calculate a and hence pressure of 60 gm of the gas at 27 °C with a volume of
2 litres if its molecular weight be 30.

Solo. : (i) For van der Waals gas, RTc = ~.


PCVC 3
Tc = 32.3 + 273 = 305.3 K
Ve = 3b
Pc = 48.2 atm
1 1
Thus, b = RTc = 305.3Kx0.082061.atm.deg- .mole- = _ litre.
0 065
8Pc 8x48.2atm

Now, b = 4NA(~7tr 3 )
3b 3x 0.065 x J0 3 cc
or, r 1
-
- -
16NA1t - 16x6.022xJ0 23 x7t
= 6.44 x 10-24 cc

or, r = l.86 A .
Thus, cr = 3.72 A .
p "=_a_
c 21b 2 .
. . a = 27 x 48.2 x (0.065)2 = 5.498 atm.12.mol-2,

n = 60 = 2.
30
2
:. p = nRT an = 2x0.08206x300 _ 5.498x4 _
2083
V-nb - V2 2-2x0.065 4 - · atm.
57. A real gas obeys Z =a 1 + a 2P + a 3P2 , Z =compresibility factor. It shows minimum
at some pressure P0 and temperature T1. Find (i) a 1 (ii) sign of a2 and a3 (iii) P0 in terms
44 PROBLEMS ON PHYSICAL CHEMISTRY

of constants. (iv) If the temperature variation of a 2 is given by a 2 = C1 + Cz and the


T
Boyle temperature TB obeys TB > T 1, remark on the signs of C 1 and C2 and express TB
in terms of these constants. [B.U. 2002]
Solo. : (i) As P ~ 0, Z = a 1•
Now as P tends to zero, Z = I. .. a 1 = 1.
Thus, Z = 1 + a2P + a 3P2 .

(ii) and (iii) : (~!)= a2 + 2a3P


When P = P0, az + 2a 3P0 = 0.
Po=--,
az
2a3
Pressure P0 can never be negative, so a 2 and a 3 should have opposite signs.
az
Now at P0, Z < l and Z = .l + a2P0 + a3P02 =1- - 2 .
4~

ai is always (+)ve, Thus, a 3 should be positive so that Z < 1 at temperature T 1 and


pressure P0 . Thus, a 2 is negative.
(iv) At TB, Z =1
As P is very low, a 3P2 can be neglected. Thus az = 0.
Cz .
By the problem, C 1 + - = 0.
TB
C2
TB=--
C1
TB is positive, hence C1 and C2 should have opposite signs.
az increases with increase in temperature from its negative value at T 1, becoming zero
at Boyle temperature TB' .!!:.._ [a2 (T)] =- C~ > 0. Thus C2 should have negative sign while
dT T
C1 sould have positive sign.
58. Two gases N 2 and C 2H 6 obeying van der Waals equation have following Tc and Pc :
Tc pc
N2 126 K 34 atm
C 2 H6 305 K 48 atm
Which gas has (i) smaller value of a (ii) larger value of b (iii) which gas shows nearly
ideal behaviour at 25 °C and 10 atm? [Ans. N2 ,C 2H 6 ,N 2 ]

59. Calculate the temperature at which slope of Z vs P curve (P =0) has a maximum
value for van der Waals gas. Calculate also the value of the slope.

Solo. : At P = 0, for van der Waals gas Z can be written as

z=l + (b-.!!_)_!_.
RT RT
GAS 45

Hence, slope m = (az)


oP
__ (b-.!!_)
P=O -
1
RT RT .

Thus, the slope of Z vs P curve (m) is a function of temperature. This will be maximum
dm
at a temperature when - = 0.
dT

Now, ~; =~T(R~z ) - R~z (b- ;T)

2Q b 2a
Thus, R2T3 - RT2 = 0, .. T = Rb

The slope will be maximum at T =2 Q


Rb

The value of maximum slope,

mmax =rb--Q1x-I
R--
2Q
R.-
b2
2Q = 4a
Rb 2b

Hence, after reaching the temperature twice the Box le temperature of the van der Waals
gas, its Z value starts decreasing and approaches ideal gas value (Z = I) with increasing
temperature.

_60. The behaviour of two gases A and B can be approximated by van der Waals' equation.
The critical constants of these gases are given below :
Gas PJatm Vc/ccm3.moJ-l T/K
A 81.5 81.0 324.7
B 2.26 57.76 5.21

Explain (i) which gas has greater intermolecular force of attraction, (ii) which gas obeys
van der Waals' equation more closely at the critical state. [C.U. 2010]

27R 2 T 2
Solo. : Hints : Q = c
64Pc

QA = 3.68 atm.Al 2.moJ- 2

QB = 0.03 atm.AI 2 .mol-2

Hence, A has greater intermolecular force of attraction.


46 PROBLEMS ON PHYSICAL CHEMISTRY

For A, _ _
RT = 4.04
(ii) c
PCVC

RT
For van der Waals gas, _ _
c = 2.66.
PCVC

So B obeys van der Waals' equation more closely at the critical state.

61. For a van der Waals gas, show that the approximate molar mass of a gas

M = pRT[l + PT" (l- 27 Tc)]·


P 8PcT 8 T

Soln. : For n moles of van der Waals gas, the equation can be written as

PV = nRT + n(b- :T )p

or, PV = -RT
W
M
+
M
a)
-W ( b - -
RT
P

M RT
or, -p = p +(b- :T) [Since, p = wl
v
M RT +(b- 27bRTc) [T 8a ]
or, -p = P 8RT c = 27 Rb

M RT· [ l + bP ( l _ 27 Tc)]
or, - =
p P RT 8 T

or, M = RT [l + RTc ._!_(I - 27 Tc)]·


p P 8Pc RT 8 T

B. Different velocities, energy, Maxwell distribution, equipartition, mean free path,


collision frequency and viscosity
Bl. Required Formulae

=~ ~T = ~ 2: ; .
2
( l) Most probable velocity ii
GAS 47

Root mean square velocity e- __ ~3MRT.

.
Average velocity v = ~SRT
-- .
rtM
(2) Fraction of molecules having velocity between e and e + de :
2 mc 2
dNc
- - =4r e m-
( - )2 ee2-2kTde
N 2rtkT
_ mgh
(3) Barometric formula : P = P0 e kT .
(4) Fraction of molecules having velocity in any direction between ex and ex+ ex
I mc2

=F(e x )dex = (_!!!__)


2rtkT
2 e - Zk; .

(5) Absolute value of ex in one direction


(kT
~~·

(6) . through area A m


Num ber of mo Iecules Ieakmg . trme
. t= I -N_A
rr= e t = -I -N_A
v t
-v6rc V 4 V

(7) Number of molecules leaking through unit area in unit time ~ ~ c.


(8) Number of molecules colliding in unit time with wall in unit area :

z = _I_ N ~ 3kT = N ~ kT
{61i V m V 2rcm
N p
or, V == kT.
p
:. Z = -/2rcmkT ·
(9) Mass striking with lhe wall in unit area in unit time

== µ == Zm ==~2rc:TP=~2:;Tp.
(10) Number of molecules having energy between e and ~ + de :

I
dNe == 2rcN ( - -
)2 e 2e kT de .
2 I -e

rtkT
(11) Fraction of molecules having kinetic energy greater than e :
48 PROBLEMS ON PHYSICAL CHEMISTRY

( 12) Equipartition of energy :

energy per degree of freedom : _!_ kT


2
energy per vibrational degree of freedom: kT.
( 13) Mean free path :

kT 1
= {2:rrP.cr 2 = .fi.rrNcr 2 '

where N' = -N = number of molecules per cc.


v
(14) Collision frequency:

z1 = ~=J8RT x .fi.rtcr ~(rtRT)·


2 2
N = 4cr N
/.., rtm V V M
Frequency of bimolecular collision per unit volume

(15) Viscosity coefficient ri =.!_pv/...,


3

where p = -Nm = density.


.
v
(16) Poiseuille equation for liquid:
rtR 4 (P1 - P 2 )
ri = - -V-- - , where V = volume of gas flowing in unit time.
8 1

rtR
4
( P~ - Pi)
vg = 16Y)P l .
0

( 17) Some Important Integrals :

2n+I -l.x 2 d
(a)
fx
0
e -n !-
x =2t..,n+ I

(b)
18. Error Function (erf)
x
erf (x) = ~se-.v2
{ii,
dy
0

I
~

erfc(x) = I - erf(x) = {ii,


2 e- i dy
x

erfc(x) is co-error function.


Table 1 : Values of erf(x) for different x
x erf(x) x erf(x)
0.00 0.00 0.1 0.11
0.2 0.22 0.3 0.33
0.4 0.43
0.5 0.52 0.6 0.60
0.7 0.68 0.8 0.74
0.9 0.79
0.100 0.84
I.IO 0.88 1.20 0.91
1.30 0.93
1.40 0.95 1.5 0.96
B2. Short Questions

1. Should an ideal gas have viscosity? Why?


Ans.: Viscosity is the internal friction opposing the relative motion of adjacent layers.
In liquids this arises because of attraction between molecules in adjacent layers. In a gas
the reason of viscosity is intermolecular collision between molecules of adjacent layers.
Assuming hard sphere for molecules having collision diameter cr, it can be shown that
viscosity coefficient (7]) is

I -1
1J =3pv11.,

where p is density, ii is average velocity and /... is mean free path. However, if we strongly
adhere to the ideal gas kinetic model and assume molecules as geometric points, then since
points, cannot collide, Tl = 0.
2. Viscosity and thermal conductivity must be related to one another. Why?
Ans. : Viscosity can be interpreted in terms of momentum transfer by molecular
collisions. Thermal conductivity is the phenomenon of transfer of kinectic energy through
a gas by molecular collision. Thus they should be interrelated. The relation is shown below
in brief :
If Q is energy transport, the energy is transported due to collision. Hence,

where dE is K.E transported per unit length through area A.


dy . .
50 PROBLEMS ON PHYSICAL CHEMISTRY

dT
ButQ=KA- .
.· dy
Amount of heat flow per unit area in unit time has to be proportional to temperature
gradient. K is thermal conductivity coefficient in joule per metre per degree in SI unit.
Thus,

KA dT =I_ N vA dE dT
dy 3 V dT dy

K = _!_ N vA dE .
3V dT
But for flow, E = U.
dE
dT = mCv,

where Cv is heat capacity per gram.

= ~pvACv= 1'\Cv,
where 71 is viscosity coefficient.
3. "Viscosity and diffusion coefficient are related."-Explain and show the relation.
Ans. : Diffusion of a gas is a transport of mass across a concentration gradient. The
dC
rate of flow of molecules should be directly proportional to concentration gradient
dy

1J = DdC
-,
dy
where D is diffusion coefficient. It can be shown that

D = _!_vA = ~
3 p'
where p is density and 11 is viscosity coefficient which arises due to momentum transfer
and hence, they are related.
4. Why does collision diameter decrease with temperature?
Ans. : The collision diameter cr0 is the sum of radius of two molecules when they
collide. Because of intermolecular attraction collisions may take place when they are at
a distance greater than cr0 so that apparent collision diameter is greater than cr0. As
temperjlture T increases, the velocity increases so that attraction may be felt only at a shorter
distance and the apparent collision diameter cr' decreases being equal to cr0 only at infinite
temperature. The empirical Sutherland equation expresses this relation as

cr2 = cr2(1+
o T c) ,

where C is a constant.
GAS 51

5. (i) Arrange in decreasing order: most probable velocity (u), root mean square velocity
(c), average velocity (ii). (ii) What would happen to the differences and the ratio between
them if temperature is increased and if molecular weight (M) is decreased?

Solo. : (i) c : v : u = ~-r3MRT : ~ 8pRMT- : ~ 2MRT

= ~: ~: {2
= 1 : 0·92 : 0·82.
Hence, c > v >ii .
Ans. : (ii) Difference between them will increase with incease in temperature but the
ratio of them will remain constant. The same will happen to the difference and ratio between
them if molecular weight is decreased.
B3. Numercial Problems
1. At what temperature will Hz molecules have the same K.E. and r.m.s. velocity as
N 2 molecules at 35 °C ? [M.Sc. Admission Test, IIT, 1993]

Solo. : r.m.s. of Hz at T °K ~ 3 ~T .

r.m.s. of N 2 at 35 °C = 308 K is ~.
3RT 3R x 308
By the problem, - -
2 28
T = 2 x 308 = 22 K.
28
2. In one litre container 10z3 gas molecules each of mass 10-zs kg are present. The
r.m.s. velocity of the molecules is 1000 ms- 1•
(i) What is the total K.E. of the molecules?
(ii) What is the temperature of the gas? [GATE 1996]
Solo. : By the problem,
1 -2 1
K.E. = -mc .xN = -x10-25 x (10 3)Z x 102 3 = _!_ x 10 4 = 5000 J.
2 2 2
I - 2 1 - 2
Again, PV = -mNc 2 = -x N x -mc2 = -K.E.
3 3 2 3

K.E. = lpv = lnRT = l.!!_RT


2 2 2 NA

3 1010
or, 5000 = -x
2 6.022x 10 23
x 8.314 x T

5000 x 6. 022 x 2
.. T = = 2414-40K = 2414 K.
3x 8.314
52 PROBLEMS ON PHYSICAL CHEMISTRY

3. The density of oxygen at 273 °K and I atm is 1.4290 gm/I. Calculate the r.m.s.
velocity of oxygen at 273 °K.

Soln.: c = ~ 3 ::- = ~ 3 :v = f;
P =I atm = 1.01325 x 10sN.m-2
p = 1.4290 gm/I = 1.4290 kg.m-3

c = /3xl.01325;!()5 = 461.2143589 m.s- 1 = 461.21m.s- 1.


~ 1.4290
4. It was found that at 2800 K tungsten (M = 183.80) loses a weight of 2.15 x 10-6 gm
per second from a heated filament of 5 sq cm area. Calculate the vapour pressure of tungsten
at 2800 K.
Soln. : We know that mass leaking per area A in t sec is

µ = /M P At.
~hRT
By the problem,
µ = 2.15 x 10-9 kg
M = 0.1838 kg.mole- 1
R = 8.314 J.K.moJe-1
T = 2800 K
A = 5 x l0- 4 sq. metre.
9
p = µ {2itifi = 2.15 x 10- {2 x 3.142 x 8. 314 x 2800
.. A{M 5x10-4 .V0.1838
7
8.223194714x 10-
, - - . = = = - = 3.84 x
= - -5 -x .l0-4 10-3 Pa
.Vo.1838
2.9 x 10-5 mm of Hg (760 mm of Hg = 1.01325 x 105 Pa).
5. The vapour pressure of a solid at 2600 K was found to be 5.24 x 10-7 atm. It was
found that 6.48 x 10-4 gm of the substance passes through an opening of 3.25 mm 2 in
3.5 hours. Calculate its atomic weight.

Soln. : Mass leaking µ =~ M P.A.t


2rtRT
µ = 6.48
x 10-7 kg
A = 3.25 x 10---0 m2
P = 0.0530943 Pa
t = 3.5 x 60 x 60 sec.

{M = µ.V2rcRT
P.At
or, M = 0.012066072,
:. M = 12.
GAS 53

6. Two flasks (A) and (B) have equal volumes. (A) contains H2 gas and (B) contains
an equal mass of CH4 . (A) is maintained at 300 K while (B) is mantained at 600 K. Assuming
ideal behaviour for the gases, answer the following :
(i) In which flask is pressure greater? How many times one is greater than the other?
(ii) In which flask is the mean free path of molecules greater? How many times one is
greater than the other? (The collision diameter of CH4 is twice that of H 2.)
[C.U.(H) 2002]
Solo. : By the problem : Let w gm H2 be taken, then

(i) PH = wxRx300
2 2xV
wxRx600
PCH4 = 16x V

PH 2 = 300 X ~ = 4.
PCH
4
2 600

(ii)

600R
(since, cr CH 4 = 2cr H2 ) .
.firrPCH4 ( 4cr~2 )

AcH 4 = 2 x 4 = .
2
AH2 4

7. The viscosity coefficient 71 of H 2 is 8.41 x 10-5 poise at O °C and I atmosphere


pressure. Determine A.

Solo. : ri = _!_ pvA.


3

= _!_MP ~8RT A
3 RT rtM
1
= _!_ x 2 gm - mole- x I atm x
3 82.06cc.atm.K- 1 .mole- 1 x273.15K
-------~

4 x 10 7 erg. K- 1 • mo1- 1 x 273.15 K


3.142 x 2 gm - mole- 1

= _!_ x 2xl (8;s:31W()7~~A.


3 82.06x272.15~-
3.!42x2
2 1
= 5.05708894 gm.cm- .sec- A = 5.06 gm.cm-2.seclA..

8.41x10-5
= = 1.66 x 10-5 cm.
5.06
54 PROBLEMS ON PHYSICAL CHEMISTRY

8. Calculate TJ (viscosity coefficient) of nitrogen at 300 K (<J = 4x10-8 cm).


Soln. : TJ = _!_pvt. = MP v = /SRT
3 RT ~~

7
= _!_ 28 {8.314x10 x 2400 x 1
3 ~-3.142x28 2
.f2x3.142x(4xl0 8 ) NA

= 1.038205653 x 10-4 = 1.04 x 10-4 poise.


9. Molecular diameter of CO is 3.19 A. At 27 °C and 100 mm of Hg pressure what
could be (i) collision frequency and (ii) mean free path.

,
H mts 7t<J2 - (NAP)
: Z = {2 .v. RT

By the problem,

Z=
3. 142 x (3.19 x 10-8 )
2

x
{g-;8,314 x lOT~-j(}(j x ( 6. 022 x 10 23 x 100 )
3
{2 3. 142 >< 28 760 x 0. 08206 x 10 x 300

RT
[Ans. 3.46 x 108 sec- 1, 6.87 x 10-5 cm]
.A. = .f2nPNA<J2 .
10. A gas contains 10 15 molecules/I. The mean free path of the molecules is ~
'V 27t
metre. Calculate <J. [B.U. 1991]
1
Soln. : We know that A. =
.f2ncr 2 N''
where cr2 is the square of collision diameter and N' is the number of molecules per unit
volume
By the problem, N' = 10 18 molecules/m 3
1
- 2 -
<J N'
= 1 metre
cr2 = _1_m2
N'
or, <J = ~ 1~18 = 10-9 m.

cr = 1.0 A..
11. The viscosity of hydrogen at NTP is 8.4 x 1o-5 poise and average velocity of gas
moles is 1.7 x 105 cm/sec. Calculate the mean free path. [B.U. 1997]

Soln. : We know that T] = .!.pvt.=_!_ MP VA.


3 3 RT
GAS 55

= _!_ 2x I x 1.7 x I 0 5 A.
3 82. 06 x 298.15
R = 82.06 cc.atm.deg- 1.moJe- 1
Tl x 82. 06 x 273.15 x 3
A. = 2 x I x I. 7 x 10 5
8. 4 x I 0-5 x 82. 06 x 273.15 x 3
= 2xI.7x10 5
= l.661324x!05 cm "'1.7 x lOS cm.
12. At NTP, the viscosity coefficient of oxygen is 0.2 millipoise. Find the collision
diameter of oxygen. [C.U. 1996)

1 _ 1 MP~RT RT
Soln.: Tl= -pvA. = - - - - - - - - -
3 3 RT rtM -firtPNAcr 2

2 I ~8RTM I
<J =3 -rt- -firtNATJ

Tl = 0.2 x I o- 3 poise = 2 x 10--4 poise

cr 2 -
l~x8.314x10 7 x273.15x32 x - - - - - - -
I
- -23- - - -
-3 3.142 -[ix3.142x6.022x10 x2x10-4
= 8.473735682 x 10- 16 cm2
:. cr = 2.91 x 10-8 cm.
13. In a carefully designed vacuum tube it is possible to reach a pressure of
10- 10 mm of Hg. Calculate the mean free path of He atoms at this pressure and at 25 °C.
Given, <J = 2.5 A. [C.U. 1987]
RT 8
Hints : A. = 2 = 1.11117 x 10 cm.
r;;
'Y L. rtPN A(J
14. The molecular diameter of a gas is 2.85 A. What will be the mean free path of
the molecules at 25 °C and 1 atm. [C.U. 1989)
rAns. 1.125 x 10-5 cm]
15. The coefficient of viscosity of He at 500 K is 279 x I o--4 g.cm- 1.sec- 1. Calculate
the collision diameter of He. Given that k = 1.38 x 10- 16 erg.K-1. [C.U. 1991)

. I -~
H mts : T) = -pvt\,
3
R = 1.38 x 10- 16 x NA. [Ans. 1.7 x 10-8 cm]
= 8.3103 x 107 erg (Use this value of R).
16. The van der Waals constant b for C0 2 is 42.9 x 10-3 1 per mole. Calculate T) for
C0 2 at 27 °C. [B.U.(H) 1990)
Hints : <J = 3.329 x 10-8 cm

T) = _!_pvA. = 1.97269 x 10-4 gm.cm-l.sec-1.


3
56 PROBLEMS ON PHYSICAL CHEMISTRY

17. At 15 °C, the visocity coefficient of C0 2 is 172 x l o--4 poise. Calculate its mean
free path and collision diameter. [C.U. 1984)
. I
H mts : ri= -pv11. -1

3
a = 3.47 A
7 67
A. = ·
p
cm (P in dyne/cm 2 ).

18. One mole of HI is present in I dm 3 at 700 K. Find out the number of collision
m l x l0- 3 dm 3 and the mean free path of the gas. [V.U. 1989)


Hmts : ZAA -- 7tCJ2
,,,. ~-
8RT
- (NA x 10-3)2 (since 10-3 dm 3 = l cc)
-v2 1tM
= 2.74 x 1046cr 2
3. 7376 x l 0--22
= (J2

19. A. for N 2 at 0 °C and I atm is 10-5 cm. Calculate the collision diameter of N 2.
What will be the Yalue of A. at a pressure of l torr. Calculate the average time between
collisions at 0 °C and l atm. [C.U. 1997)
RT
Hints : A. =
../21tPNA(J2

a2 = RT
../21tPNAA..
a = 2.89 x lo-8 cm = 2.89 A
760 torr = l atm
A. = 7.6 x 10-3 cm at l torr

~ 1tM
7t 10-5
= = lo-5 x = 2.20 x 10-10 sec.
v = ~8RT
Average time
8RT
1tM
20. Calculate the number of binary collision per cc of N2 (g) per sec at 2 atm pressure
and 30 °C. The bond length of the gas molecules is 1.87 A . [C.U. 1998)
Hints : By the problem,
a = 1.87 x lo-8 cm
T = 303 K
M = 28 gm
P =2 atm

7tCJ2 -( NA p )2 28
ZAA = ../2 v RT = 8.72 x 10 per cc per sec.

21. Calculate the total kinetic energy of all the molecules of 1 mole of an ideal gas
present in 3-1 vessel under 2 atm pressure at a certain temperature. [C.U.(H) 1999]

Solo. : K.E. = -3 PV = 911.92 J.


2
GAS 57
22. A 10-litre capacity metallic container with 0.1 gm-mole of H 2(g) in it has a leakage
hole of radius 0.02 mm. The r.m.s. velocity of H 2 molecule in the container is 400 ml
sec. How many hydrogen molecules escape through the hole per minute? [C.U. 1980)
Solo. : A hole of radius 0.002 cm means an area A of m 2 = 3. I42(2x 10-3) 2 cm2
Molecules striking area A of the container per minute will escape. Z, the number of
escaping molecules

= _l_Nc At
{(;nv

I O. lmole x 6. 022 x 10 23 molecules. mole- 1


= --x x 400 x l02 cm.sec- 1
{(;IT, lOx l0 3 cc
x 3.142 x (2 x 10-3) 2 t:m 2 x 60 sec
= 4.18349 x 10 19 = 4.183 x 10•9.
23. At 2500 K, 0.5 mg of carbon passed through an opening of 3 mm2 in 2.5 hours.
Calculate the v. p. of carbon at 2500 K.

Hints : µ =~ P. A.t.

.. P = µ./2itifi
r.; = 0.6109 dyne/cm2.
-vMA.t
24. The van der Waals constant b for C02 is 42.9 x 10-3 l.mole- 1. Calculate T/ for
C0 2 at 27 °C. [B.U. 1990)
Hints : a from b = 3.239 x 10- cm.
8

I -~
T/ = -pv11.
3

= l MP ~BRT RT =2 G
{MKi
- = 1.99 x lo-4 poise.
3 RT rtM . ..f2rtPNAa 2 3 -Vrt3NA0'2
25. A gas whose viscosity is 200 micropoise flows through a capillary tube 2 mm
in diameter and 2 metres long. If 5 litres of a gas pass through the tube every l 0 11ec,
what must be the pressure head in atmosphere unit under which the gas is flowing?
[C.U. 1988)
Solo. : For any fluid,

V = rtR4(P1 - Pz) A..


8lll

(P _ p ) _ 8Vlll
•• I 2 - 7tR4 '

where V is volume passing per unit time, T/ is viscosity, A. is length, R is the radius of
the tube.
By the problem,
T/ = 200 x I o-6 poise
58 PROBLEMS ON PHYSICAL CHEMISTRY

5000
v = -10- = 500 cc/sec
.
P1 - P2 = P, the pressure head

R = -0.2 cm= 0 .I cm
2
= length of the tube = 2 m = 200 cm

8 x 500 cc. sec- 1 x 200 x I 0--{j poise x 200 cm


p = 4
3.142 x (0.1) cm 4
= 509229.7899 dynes/cm 2
= 0.50257 atm.
26. What will be the ratio of final to initial Z values for a gas, if P is halved at constant
gas density?
p
Solo. : Z = wall collision frequency =
~2rtmkT
N
Now, P = -kT.
v
p
New pressure, P 1 =
2
Nm N
Since p = - - = constant, - = constant. Hence, T 1 =T
v v 2

Thus , ~ = _!i__
fT
x -fl = _!i__ x {2. T1 = _I_
fT r;, = 0.71.
Z 'Y T1 P 'Y T1 2 P1 -y 2

So wall collision frequency will decrease.


27. Two separate bulbs are filled with two ideal gases A and B respectively. The
molecular weight of B is twice that of A. If B is at twice the absolute temperature and
has half the density of A, what is the ratio of wall collision frequencies?
Solo. : By the problem, Ts = 2TA,
Ms = 2MA
Pa =EA.
2

Now, P = 2-Rr.
M

Now, z = PNA
~2AMRT.
GAS 59

28. Determine the ratio v01d /vnew of average velocity and Z01 d I Znew of wall collision
frequencies of an ideal gas, if (a) Tis doubled at constant P (b) Pis doubled at constant T.
l r;:; I
[Ans. (a) r;:;, -v2, (b) I, - ]
-v2 2
29. A bulb contains a mixture of Hz and Oz at 0 °C and I atm total pressure. The
bulb contains 68% by weight of Hz. Calculate the ratio ZH/Zoz·
Soln. : In 100 gm mixture,
weight of Hz. wH
2
= 68 gm
weight of Oz. w02 = 32 gm.
RT RT
Hence, P 02 = n0 zV = V(M 0 z = 32)
RT
= nH2RT
-
= 34-
v (MH
2
= 2).
v
Hence, ZH2 = PH2 ~ Mo2 = 34 /j2' = 136.
Z02 P02 MH 2 fz
30. What would be the molecular weight of a gas if pressure of the gas falls to one
half its value at a height of I m at 25 °C ?
_ Mgh
Hints : Ph = P0 e RT

p
By problem, Ph = _Q_
2
M = 175.23 kg.
31. What will be the percentage change in barometric pressure at 2000 m above the
sea level at 300 K, if it is assumed that air is roughly a mixturi;of 20% Oz and 80% N ?
2
[B.U. 2004]
Soln. : At sea level, total pressure P =I atm.

P002 -
- atm, P~ 2 = -4 atm
5 5
MNzgh Mo21:h
= pNz+
h
p02 -
h -
pN2e
0
----;u- + p02e
0
----;u-
= 32 x 10-3 kg, MNz = 28 x 10-3 kg, g = 9.8 m/sec2
= 8.314 J.K- 1.mole- 1, h = 2000 m, T = 300 K.
= 0.797.
Thus, pressure decreases by 20.3%.
32. At what temperature will Nz molecules have same momentum as Hz at 25 °C?
Soln. : MoJUentum of gas molecules = molar mass x average velocity

Momentum of N 2 molecules =MN 2 v = 28 gm x ~ cm.sec-I


60 PROBLEMS ON PHYSICAL CHEMISTRY

Momentum of H 2 molecules = MH 2 ii = 2 gm x

By the problem, 28 [8Rf = 2 x {8RTH 2


~ls7t ~- 27t
Given TH
2
= 25 °C = (273.15 + 25) K = 298.15 K

28 -T
28
=2
2
x ~298 .15

T = 21.29 K.
At 21.29 K N 2 molecules will have same momentum as H2 at 25 °C.
33. An ideal gas, kept in a thin walled vessel of volume V at a temperature T, is
slowly escaping into vacuum through a small hole of area A. How much time is required

for the pressure in the vessel to decrease to· .!.


of its original value?
e
Solo. : Let N be the number of gas molecules in the vessel.
By the problem, volume of the vessel = V
Temperature =T
Area of the hole = A.
If P is the pressure, PV = NkT
Let dN number of molecules escape through the hole in time dt.

Here dN = _!_4 Nv iiAdt ,


where ii = average velocity.
Thus, the leakage reduces the number of gas molecules in the vessel by dN, thereby
reducing the pressure by dP in time dt.
I p
Hence, dP =- - - iiAdt
4V
p

f -f
e r
vAt
or, dP = iiA dt or, In =--
P 4V e 4V
p 0

4V
Hence, t =-.
iiA
4
Thus, the gas requires V time to decrease the pressure to .!. of its orginal value.
iiA e
34. A container with a small hole in its thin wall contains an ideal gas A at temperature
T and pressure P. The pressure is found to be halved in one hour due to effusion of the
gas into vacuum.
If the same container is filled with an equi-molecular mixture of ideal gases A and

B at temperature T and pressure P, find out the ratio p A after one hour.
PB
GAS 61
Solo. : Initially, let N be the number of A gas molecules, and if V be the volume of
the container, then at temperature T and pressure P,
PV = N.KT
If dN gas molecules effuse in dt time, then

-N=
d p - d t,
-vAA (I)
kT
where vA= average velocity of A
A = area of the hole.
. N VdP
Agam d
,
= --
kT (2)

dP VA
Thus - = --Adt [from eqn. (!)]
' P 4V
p
2 _ I

or, Jd In P = - ; ~ A Jdt.
p 0

Thus, In 2 = ~A. (3)


4V
If NA and N 8 be the number of molecules of A and B respectively,
PAV=NAkT (4a)
P8 V = NskT (4b)
N
By the problem, NA = N8 =
2
where N = total number of molecules = NA + N 8 .
By the problem, PV = NkT = 2NAkT = 2PA V [from eqns. (4a) and (4b)]
p
Hence, PA = P8 = -. (5)
2
Let PA(f) and P8 (f) be the final pressures of A and B after one hour respectively.

PA(!) vAA 1
Thus, ln--=---=-ln2=ln-.
PA 4V 2

Hence, PiJ) = -PA2 . (6)

. Pa(f)
Agam, In - -
v A [ va = average
= - -I _a_ velocity of BJ
Pa 4 V
Va ln2
= - - _ - [from eqn. (3)]
VA

or, Pa J
In ( Pa(!) = ln(2)
¥~ .
62 PROBLEMS ON PHYSICAL CHEMISTRY

or,

_vs
Psf.j) =Ps2 vA (7)

[from eqn. (6) and eqn. (7)]

35. Find the value of most probable kinetic energy of gas molecules.
Solo. : Fraction of molecules having kinetic energy between e and e + de is given

by p = dNe = 27t~( 1_)3 {Ee- k~ de.


E N 7tkT

Most probable kinetic energy corresponds to maximum of energy distribution curve.


At the maximum.

dp£ =0
de

0,, ! (Et '~) = 0

I
I -
--I --- I
E E
l -- I I -- e2
or, -e 2e kT -e2e kTx- =0 or, -e 2 __
=0
2 kT 2 kT

or, .. E = !kT
2
Miscellaneous
1. If 5 gm of ammonium carbamate dissociates to ammonia and carbon dioxide at
200 °C, the products occupying a volume of 7 .66 I at a pressure of 740 mm of Hg. Calculate
the degree of dissociation of NH 2COONH4.
Solo. : The reaction is
NH 4 COONH 2 ~2NH
3 + C0 2
78 gm
l - ex 2ex ex
NH 3 occupies 44.828 I at NTP and C0 2 occupies 22.414 I-a total of 67.242 I at
NTP from decomposition of 78 gm of NH2COONH4 .
Thus at NTP, 5 gm NH4COONH 2 on complete decomposition will occupy
5
- x 67.2 = 4.31 I,
78
GAS 63

which under the given condition is

V = P0 V0 x T = 760x4.31x473.15
T0 P 273.15 740
= 7.6675 1 = 7.67 I.
7 66
So the decomposition is not complete and · x l 00 = 99.869% has decomposed.
7.67
:. (J.. = 0.998.
2. An iron pipe 3 m long closed at one end is lowered vertically into water until the
close end is flushed with water. Calculate the height of the water level in the pipe. Given
diameter of the pipe = 2 inches.
density of water = l x 10 3 kg.m- 3
pressure = l x 105 Pa = 10 m (hydrostatic head of water), temp. is kept constant.
Solo. : Since temperature is constant,
P!Vf =Pin vi
p = p. 3 x area of the pipe = 3P;n
f in h x area of the pipe h
But by the problem,
Pin = 10 m and P1 = hin + 10 m

h + 10 = 3xl0
h
i.e., h2 + lOh - 30 = 0.

h = -
1
o+.fi02mo = 2.4161985 m = 2.4 m.
2
The pipe is 3 m long. Hence, water level would rise up 0.6 m in the pipe (3 - 2.4 = 0.6)
3. A sealed vessel of l litre capacity contains air and a small quantity of water. At
25 °C, the pressure in the vessel is I atm, and it is found that at 200 °C the measured pressure
is 4.6 atm. Assume that the vapour pressure of water at 25 °C is negligible and that of
steam at 200 °C behaves like an ideal gas and that the vessel does not expand on heating.
Calculate the volume of water in the vessel at 25 °C. Given that density of water at 25 °C
103 kg/m 3 , 0 °C = 273 °K, MH = 18 and R = 0.08206 l.atm.K- 1.mole- 1.
20
Solo. : Let n be the number of moles of air and n' be the number of moles of water
at 200 °C, then

PV 4.6atm x I litre
n + n' = = 0.082061.atm.deg- I .mole- l x473K =0.118585.
RT
Now, at 25 °C, n' mole of water occupies
18n'cc = 18(0.118585 - n) cc= 0.018(0.118585 - n) I

latm x[l-0.018(0.118585-n)]l
n = = 0.0408609 = 0.04.
0. 082061. atm. deg- 1 . mole- 1 x 298K
n' = 0.077719 = ().08
V' = I Sn' = 1.398942 cc ..=. 1.40 cc.
64 PROBLEMS ON PHYSICAL CHEMISTRY

4. Hydrogen will dissociate into atoms at high enough temperature, that is Hz = 2H.
Assuming ideal gas behaviour of Hz and H atom, what would be the density of hydrogen
at 2000 °C, if 35% dissociates into atoms? The pressure is I atm.
Soln. : The reaction is Hz = 2H
At 2000 °C and 1 - 0.35 2 x 0.35
1 atm, ex = 0.35 = 0.65 = 0.70
Thus total number of moles is 1.35.
. . total weight 2
Average molecular weight 1s - - - - - " - - - - = - - = 1.481.
toal number of mole 1. 35
PMa Ix l.481
p = =- ----- = 7.94 x 10-3 gm/I.
RT 0.08206 x 2273.15
2
5. Calculate the value of mean square deviation ( (c - (c) ) ) for Oz at 27 °C.

2 2
Soln. : ( (c - (C) ) ) = (c 2 - 2c(c) + (c) )

• = (c 2 )-2(C)(c)+(c 2 ) = (c 2 )-(C) 2

.. (( c-(c))2) -- 3RT - 8RT - (


M 1tM - 3-~
8) M RT

= (3 _!) 1
8.314J.K- .mole- x300K
1

7t · 0.032 kg.mole- 1

= (3-~) 7.794375 x I04 mZ.sec-Z

= 3.53748828 x 10" m 2 .sec-2 •


6. A 12-litre flask is filled with 20 gm neon and an unknown weight of hydrogen.
The gas density is 0.002 gm/cc at 0 °C. Calculate the average molecular weight and number
of grams of Hz added. Calculate the pressure.
Soln. : Given density is 2 gm/I. Thus, there is 24 gm materials out of which 20 gm
of neon has been taken. Automatically 4 gm of Hz. i.e., 2 moles of Hz has to be taken.
Total weight 24 gm and there are 3 moles (I mole neon + 2 moles of Hz).

:. Mav = -243 = 8 gm/mole

P
= nRT = -
3 x 0.08206 x 273.15
--- - - - = 5.603 atm.
v 12
7. An ideal gas at NTP has a most probable velocity ii = 4.20 x 104 cm/sec and a
mean free path of 7.90 x 10-8 cm. Calculate the mean time between collision.

ne k now th at -u
So I n. : UT = ~2RT
-- and average ve Ioc1ty
. -v 1s
. ~8RT
-- .
M 7tM
GAS 65

v = Jn =Jn x
ii 4.20 x 104 cm.sec- 1 = 4.7392 x 104 cm/sec.

Now average time between collision is t = _!_ = ~, where A is mean free path, and
z v
v is average velocity.
7.90x 10-S
t = = l.666948 x 10-12 sec = 1.67 x 10 - 12 sec.
4. 732x 10 4
8. Calculate the average time of collision made by a N2 molecule with successive oxygen
molecules in air at 298.15 K at a total pressure of l atm. Given, cr02 = 3.4IA and crN
2
= 3.71 A.
Soln. : Average time between collision is t =~ (mean free path divided by average
v
velocity). It is also inverse of ZN (o ).
2 2

ZN2(0z) = 7t0' 2AB ~8RT


7tµ Co2

I
µ

28x32
µ = = 32+28 = 14.93 gm.
= 3.4t + 3· 71 x I0-8 cm = 3.56 x I0-8 cm.
2
. N PxNA
Concentrat1on -
v
=
Rx 298.15

6.022x10 23 molecules. mo1- 1 x I atm


= 0.08206 1.atm.K- 1.mole- 1 x298.15K
= 2.46 x
l 0 22 molecules per I
= 2.46 x
10 19 molecules per cc.
Air contains 20 per cent of 0 2 and 80 per cent of N2 .
C0
2
= 2.46 x 10 19 x 0.2 = 4.92 x 10 18 molecules/cc.

107
= 3.142(3.56 x 10-8) 2 {"8; 8· 314 x x
298 15
· x 4.92 x 10 18 collisions/sec.
~- 3.142xl4.93

8 8 314 107 298 5


=:3.9820 x 10- 15 x 4.92 x 10 18 x · x x ·1 collisions/sec
3.142 x 14. 93
= l.9751 x Io4 x 6.5018 x lo4 collisions/sec
= 1.2737 x 109 collisions/sec

:. t = _!_ = 7.85 x 10-10 sec.


z
Prob. Phy. Chem.-5
66 PROBLEMS ON PHYSICAL CHEMISTRY

9. Show that to a first approximation the equation of state of a gas that dimerizes
. . b PV
to a sma II extent 1s given y RT =1- Kc , where Kc is the equilibrium constant for
v
2
the formation of dimer, 2A = A 2 , Kc = [: 2\ . [V.U. 2002)

Soln. : Let a. be the degree of dissociation for the reaction 2A = A2 .


Thus, 2A = A2

1- a. a.
2
a. 1-a.
Hence, [A 2] = - and [A] = - - , where Vis the total volume.
2V V

a.V
Thus, Kc = Z(l - a.)2 .

Now, total number of moles =1- a. + a. =1- a.


2 2
If P is the total pressure, then

PV = (1- ~)RT
or, a. =l _ PV.
2 RT
K
= a. a.
[since, 1 » a. ] =I_ PV.
Hence, _c
v 2(1-a.)2
= 2 RT

PV K
=1 - _c (Proved).
RT v
10. A 2 m long tube is provided with inlets at both the ends so that HCI and NH 3
gases can be admitted simultaneously. Calculate the distance from HCI inlet end to the
tube at which NH4CI will first appear if two gases are admitted at the same time, one from
one end and the other from the o.ther end. · [B.U. 2006)
Hints : Let distance be x m from HCI inlet end, where NH4CI first appears.
By the problem, tNH 3 = tHCI
or, f!E 1£
2-x
8RT
---
=
---
x
8RT (x = 0.81 m)

7tMNH3 7tMHCI

11. A flask contains 1020 molecules of He at 27 °C. Find out the number of molecules
having (i) average kinetic energy (ii) 100 times average kinetic energy.
Soln. : According to Maxwell velocity distribution,
3
...!.._ dNc
N de
= 4 21tRT
7t (~)2 c2e-Mc2/2RT,
-
GAS 67

where _!__ dNc = fraction of molecules having velocity e,


N de
M = molar mass.

Now, _!._ Me2 = E.


2

Hence, de = dE and e = (2EME .


-./2EM fM
E
l dNE
Hence - - - = 27t .jE e - RT
, N dE
~(7tRT)3
where E = energy per mole.

(i) Average kinetic energy, E = '}_RT.


2

l -
Th us· - dNE
- -_ 27t ~(3- RT) e -%
' N dE ~(7tRT)3 2

= f6"( 8.314xl0l
y; 7
x300
)e -% (T = 273
.
+ 27 = 300 K)

= 1.236 x 10-11•
This is the fraction of molecules having average energy. Thus, the required number
of He molecules having average kinetic energy = l.236 x 10- 11 x I 0 20 = 1.236 x 109•

(ii) l 00 times average kinetic energy '}__RT. = 100 x


2
Fraction of molecules having 100 times average kinetic energy

_ l dNE ~600 ( l ) -300/2


- N dE =7 x 8.314xl0 7 x300 e
= 3.976 x 10-15 .
So, number of the molecules having JOO times average kinetic energy
= 3.976 x 10-75 x 1020 = 3.976 x 10-55 "" 0.
12. A flask contains 6.022 x l 023 neon molecules at 300 K. Calculate the number
of molecules having (i) average kinetic energy (ii) 20 times of average kinetic energy.
Hints : Energy distribution of mass is independent.. [Ans. : 7.45 x 10 12 , 13.96]
13. Consider a cubic, l 0 cm on a side, of He gas at STP. Estimate the number of
times one wall is struck by molecule in one second. (Answer should be given in S.I. unit).
[PhD Entrance Examination, Columbia University]
p
Soln. : We know that, Z = At
.,,/2rtmkT

PNA
=-./2rtMRT At.
68 PROBLEMS ON PHYSICAL CHEMISTRY

Given that P = l.01325 x 105 Pa (I atm)


T = 273 K
A = 102 cm 2 = 102 x lQ-4 m2
t = I sec
number of coallisions/sec,
5 23 2 2
l.01325x 10 Pax 6.022x 10 x10- m x lsec
z = --.=====================================~
3
~2x7tx4x10- kg. mole- x 8.314J. K- • mo1- x 273 K
1 1 1

= 8.07 x 1025 .
14. If e m/sec is velocity of gas molecules, then (a) cAlculate the average time required
to travel one metre. (b) Calculate the root mean square deviation of the time from average
time. (c) Calculate fraction of molecules taking more than average time to travel one metre.
Soln. : (a) Velocity of gas molecules = e inlsec
1
So they take - sec to travel I metre.
e

Average time required by gas molecules to travel I metre = t -- (~))

Thus t =(;)
- ( )% e e m 2 _mc2 I m )%f- ee _mc2
f 27tkT
= 47t - -
0
2 kT .-de
e
= 47t. ( 2;ij
0
2 kT de.

-- ~( kmT).
1 1 2
t -( ) - 47t( m
- ~ - 27tkT
)% . (_!!!_) 7t (See page no 48)
2
2kT
(b) Root mean square deviation

Now,
-
(el2) =Jf(e) el2 de
0

= 47t
m )%f- e_mc2 de 'V~
t - t
2
( 27tkT ZkT
0
GAS 69
3 I

= 41t (~) :( 7tkT)2 2 2


. ..!_.[See page no 49]
27tkT m 2
m
= kT

(c) Molecules moving with less than average velocity will take more than average

. to cross one metre.


time s·mce l) I.
. = ( ;: , average ve oc1ty = ( ~).
average time l
l
Let
(~) = u.

Fraction of molecules with velocity less than u = N' ,


m )%J" _mc2
=41t( 27tkT e 2kTe2de =- 41t
0

Now, u = I (7tkT)2
(~) = 2;;
It
N'
N
=e 4
+ B,

m )~ ( l )% Ju e _mc2
where B = 41t ( kT 1t 2kT de.
2
0

me 2
Let
2kT
= x2.

.. de= 2kTx dx:


me
70 PROBLEMS ON PHYSICAL CHEMISTRY

Now, x = ~ 2kT
m c

Thus, de =~(2:T) dx.


When c = 0, x = 0.

c =u, x =~ 2;T , u =~ ,
. 1 FrtkT)
smce u = (~) = ~~2,;)'

..rn
2 2
2
=-::r; j e-x dx
0

= erf ( ~) (See formula 18 in Bl)

= erf (0.886) = 0.79 (See page no 49)

-N'
N
= - 0.4559 + 0.79 = 0.334.
0.334 fraction of molecules takes more than average time to travel one metre.
15. Calculate the fraction of molecules moving with velocity between ii and ii +
0.1 ii. Does it depend on temperature? Explain.
Solo : Fraction of molecules between ii and ii + 0.1 ii

m )% u+JO.lu _mc2 2
= 4rt (- - e ZkTc dc
2rtkT
u
2 2
m )% If.Iii _mc ( m )% Ju _mc
= 4rt( - - e 2 kT c
2
dc-47t - - e ZkT c
2
dc = 11 - 12
2rtkT 2rtkT
0 0

Proceeding like previous problem and putting ii = ~ 2:T ,

2.2 -121
= - .Jrr. e · + 0.88.
GAS 71

12 = -4rt ( -m-
21tkT
)2 kTff;kT
xi.Ox- - e -10· + erf (l.0)
m m

Hence, 11 - 12 =- ~ e-1.o + 0.84 = 0.0849.


Thus, 0.0849 fraction of molecules moving with velocity between ii and 1.1 ii.
This number is independent of tempera-
ture. At any temperature, this number will
remain same. Since ii itself depends on I dN
temperature (increases with increasing tem- NdC
perature), the given range (ii to 1.1 ii)
becomes temperature dependent. However,
curve flatters with increasing temperature,
Hence, number of molecules in that range
temperature independent.
16. Compare the fraction of oxygen molecules to that of Hydrogen molecules moving
with velocity between ii and 1.1 ii.
Solo. : Follow Prob. 15
The value will remain same for both gases which is 0.0849. Compare the temperatures
when the Maxwell distribution curve of a gas will look like that of another gas with twice
of its molar mass.
Solo. : Two Maxwell distribution curves will look alike only when most probability
velocities (ii) match with each other.
Since area under the distribution curve is always one, height of the peaks :"Viii be equal
if two (ii) 's remain same. Let gas A with molar mass MA at temperature TA have same
ii as that of gas B with molar mass M 8 at temperature T8.
By the problem, M8 = 2MA

iiA =~ (at temp TA)

iiB = ~ 2 RTB (at temp TEi)


MB
Now, UA = U8
TA TB
or, -=- or,
MA MB
.. T8 = 2TA •
Short Questions with Hints
1. According to kinetic theory how will the temperature of an ideal gas be affected
by adiabatic expansion into vacuum? Explain briefly.
Hints : No work is done since external force is zero. Energy remains the same because
there is neither attraction, nor repulsion.
72 PROBLEMS ON PHYSICAL CHEMISTRY

2. Explain, in terms of kinetic theory, why an automobile _pump gets heated while
pumping air into a tyre.
Hints : The pumping is done at a higher pressure. So velocity of molecules increases.
Hence temp. increases, while the pump is heated.
3. Show that viscosity coefficient is independent of pressure and give a physical
explanation of why this should be so.
· Ans. We know that
1
TJ == -pvA.
3
1 MP ~8RT RT
::: 3 RT rrM . -fi.rcPNA<J 2 •

p depends directly on P, and A inverses on P. So the pressure terms cancels out. As


P decreases then the number of molecules that cross adjacent layers carrying momentum
decrease but this is cancelled by increase of A as now they have to travel longer distance
(so that A x p remains unchanged).
4. Why is the thermal conductivity coefficient independent of pressure?
Ans. Thermal conductivity coefficient K == TJ x Cv, where TJ is viscosity coefficient
and Cv is molar heat capacity at constant volume. Neither 11 nor Cv depends on P. The
flux of heat carrying molecules across unit area in the gas decreases with decrease of P
but the mean free path increases. The two effects just cancel out.
l -
5. PV == -mNc 2 . If there are two gases having the same volume and same number
3
of moles, will they exert same pressure?
Hints : Not necessarily as molecular weight is different.
6. Under what conditions does real gas approach ideal gas?
Hints : At high temperature and as P ~ 0.
7. In normal practice aqueous tension is subtracted from the observed pressure of a
gas at a particular temperature Why is this done?
Hints : Aq. tension is the vapour pressure of water present in atmosphere.
8. For a fixed temperature indicate graphically how PV of a van der Waals gas depends
upon P when (i) a == 0 (i) b == 0 and (iii) a == b == 0.
Hints :

PV

p
GAS 73

9. Establish the concept of Avogadro number from the expression PV =.!.mnc 2


3

Hints : =n, where N is the number of molecules in volume V and n is the number
!!._
NA
of moles. NA is then ratio.
10. Establish the relation between vapour density (Vd) and density ('P) of a gas.

Hints : .!l = v"' .


P2 V"2
Gas densities are proportional to moleculer weight and hence vapour densities.

11. What is meant by the term : relative humidity ?


Hints : It is defined as the partial pressure of water in atmosphere to the vapour
pressure of water at that temperature.
12. Explain why spark plugs are not needed in a diesel engine.
Hints : The compression of the gas heats the diesel above their ignition temperature.
13. When a tire is rapidly pumped, its temperature rises. Would this be expected if
air is an ideal gas?
Hints : Even if air is an ideal gas, compression would cause a temperature rise because
energy is expended on the gas in the process. The energy converted into K.E. of the molecules
is heat.
CHAPTER 3

LIQUID STATE (SURFACE TENSION AND VISCOSITY)

Required Formulae

1. Excess pressure on the concave side of a bubble P = 4rY


In a cavity P = 2'Y
r
2. Rise and fall of height in a capillary of radius r.
l
y cos(}= -hpgr, where h = height, p = density.
2

3. In stalagmometer, 1L = ~ =Pilli ,
'Y2 m2 P2n1
where m 1 and m2 are masses of one drop of two liquids respectively and d and n are respective
density and number of drops.
. ·11 . rtPr4t
4. P01sem e equation, 1J = SW
1J = viscosity coefficient, V = volume oft.he liquid flowing through a tube of length
I and radius r in time t under a pressure difference P.

5. In viscometer, Piti ..!11 =


ll2 P2t2
6. Stokes laws : F (force) = 61tr1Jv (where v is velocity).
5:fil_
7. 1J = Ae RT , where, Evis = activation energy for flow.
Short Questions
1. It is difficult to blow a balloon initially but becomes easier as time passes.
Ans. : A bubble is formed many times and an inflated balloon is comparable to it.

There is excess pressure in the concave side given by !),.p = 2 'Y. Since"( is approximately
r
74
LIQUID STATE (SURFACE TENSION AND VISCOSITY) 75

constant, when r is small, P is very high. As the balloon is filled up, r increases, thus the
effective excess pressure decreases. Then it becomes easier to inflate.
2. At what temperature surface tension will be zero?
Ans. : At critical temperature, the liquid surface is not formed. Surface tension arises
due to liquid surface. Thus at critical temperature, it should be zero. However, Ramsay
and Shield have given an equation explaining the dependence of yon temperature. The
2
3
eqn. is y x (MV) = K(Tc - T - 6).
Thus y becomes zero (for many reasons) at T = Tc - 6.
3. Why some materials are surface active and others are not?
Ans. : The materials which are surface active are usually a long chain carbon compounds
wi.&.b a soluble part and an insoluble part. Thus they will be thrown back from bulk to surface,
thereby reducing surface tension. Non-surface active materials are generally highly soluble
in bulk.
4. Why does a piece of blotting paper soak water?
Ans. : Blotting paper consists of fine capillaries. When put over water, it soaks water
because water wets paper and there is capillary rise.
S. When pure liquids are boiled in clean and smooth vessels, there is occasional bumping.
Why?
Ans. : Bubbles can grow in presence of rough surfaces. In the absence of rough surfaces,
bubbles are very small. The pressure required to form such small bubbles is very high

(P = 2 '¥ , where Pis the pressure, yis the surface tension and r is the radius). The temperature
r
thus rises above normal boiling point (boiling point of a liquid rises with increase in pressure).
This again serves to increase the pressure sufficiently so that small bubbles are produced.
As these bubbles grow the pressure which has become greater than necessary for their
formation leads to rapid expansion. This leads to occasional bumping.
6. Do electrolytes lead to increase in surface tension of water with increase in
concentration always?
Ans. : Liquid molecules in the surface are attracted strongly to the interior (cohesion).
This is the reason of surface energy and surface tension. Electrolytes attract polar solvents
very strongly and this increased attraction leads to increase in surface tension. In a solution,
ions are solvated. As suggested by M. Dole (1938), a special orientation of hydrated ions
(in water) of the interface may reduce the attractions of surface molecules to the bulk. This
may lead to decrease in surface tension at low concentrations with increase in concentration
of electrolytes.
7. What should be the latent heat of evaporation of mercury? Will it be higher or lower·

than water? Do you expect Le of mercury is as per Trouton's rule?


Tb
Ans. : The latent heat of evaporation of mercury will be lower, since it is non-associated.
That is why, Trouton's rule will be obeyed.
76 PROBLEMS ON PHYSICAL CHEMISTRY

8. Show that ST of I dyne is equal to mN/m.

A ns. : L et X
dyne = 1 mN .
cm m
dyne m
.. x= - - x -
mN cm
dyne IOOcm
= x-- =I.
10-3 x 10 5 dyne cm
1 dyne/cm = 1 mN/m.
9. "Highly viscous liquids are less volatile."-Explain.
Ans. : Attractive forces are operating between liquid molecules. Only those molecules
which have sufficient energy to overcome these forces can get into vapour state. Energy
distribution of liquid molecules is governed by Maxwell-Boltzmann distribution function
which is characteristic of only temperature as long as temperature is sufficiently high. Thus,
the less is cohesive energy of liquid, the greater is number of molecules in vapour state
at certain temperature. Highly viscous liquids have large cohesive energy. Thus, lower
number of molucules will vaporize compared to less viscous liquids at certain temperature.
Hence, highly viscous liquids are less volatile.
10. "Volatile liquids must have lower surface tension and viscosity."-Explain.
Hints : Volatile liquids have small cohesive energy and thus lower surface tension and
viscosity, since they arise due to cohesive forces. See Q. 9.
11. What are bubbles and cavities?
Ans. : A bubble is a region in which vapour is trapped by thin film, probably some
air is also trapped. A cavity is vapour-filled hole in a liquid. What are widely called bubbles
in a liquid, are actually cavities. Tn.~e bubbles have two surfaces, one on each side of the
film. The cavities have only one surface.
12. At critical temperature surface tension of liquid is zero.-Criticize.
Ans. : Surface tension will exist if there is a surface separating two phases . At critical
temperature, interface between liquid and vapour disappears as both have same density.
So, surface tension of liquid is zero at critical temperature.
13. Why are the raindrops spherical?
Ans. : Raindrops are spherical due to surface tension which liquid tends to reduce
by minimizing surface area and sphere has minimum surface area to volume ratio. Although
force of gravity opposes this tendency of liquid, it has little effect on the small size of
raindrops.
14. What is Newtonian fluid? What is laminar flow?
Ans. : If velocity of fluid is al,..yays same in magnitude and direction at a particular
point, the fluid is Newtonian fluid, and the flow is called laminar flow.

Numerical Problems
1. At 25 °C water rose in a capillary of diameter 0.08 cm to a height of 3.72 cm.
Calculate the surface tension of water if the density of water at 25 °C be 0.998 gm/cc and
g =
980.665 cm/sec2.
Soln. : Water wets glass. Therefore,
LIQUID STATE (SURFACE TENSION AND VISCOSITY) 77

r= ..!..r.h.p.g, where r = radius; h = height of water column;


2
p = density; g = acceleration due to gravity.
In this expression 0 has been taken as zero.
Given, r = 0.04 cm
h = 3.72 cm
p = 0.998 gm/cc
g = 980.665 cm/sec2.

:. 'Y = ..!..r.h.p.g = ..!..x 0.04 x 3.72 x 0.998 x 980.665 = 72.8 dyne.cm- 1•


2 2
2. How high will water rise in a capillary of radius 3 x 10-4 metre at 25 °C. Given,
Ytt 0
2
=72.75 x m-
3 N.m- 1

g = 9.80665 metre.sec-2
p = 998 kg.m-2
0 = 0°
Soln. : 'Y = 21 r.p.h.g, :. h
2y
= -rpg
2 x 72. 75 x 10- 3
.. h =- -------
3 x 10-4 x 998 x 9. 80665.
= 0.0495 metre= 4.95 cm.
3. 50 drops each of water and ether weigh 3.75 gm and 0.85 gm respectively.
'Y HzO is 72.75 dyne.cm- 1. What will be for ether? r
Soln. : By the problem, 'Y ether = mether
'°fH20 mHiO

3 75 0 85
By the problem, weight of one drop of water= · gm and that of ether is · gm.
50 50
0 85
· x 72. 75
.. 'Yether = mmether X 'YH20 = _..5-=--0~=--
3. 75
= 16.49 dyne.cm-I.
HzO
50
4. The number of drops of water falling from a stalagmometer is 90, whereas for
an organic liquid it is 310. Surface tension of water is 72.75 dyne.cm- 1. What is the surface
tension of organic liquid, where p = 0.998 gm/cc for water and 0.75 gm/cc for organic
liquid? ·

nH oPo 90 x 0. 75 X 72. 75
: . .'Yo= - -2 - x 'Yw = - - - - - - = 15.87 dyne.cm-.
1
llo·Pw 310x0.998
78 PROBLEMS ON PHYSICAL CHEMISTRY

5. A liquid A has half the surface tension and twice the density of liquid Bat a certain
temperature. If in a capillary the rise is l 0 cm for A, what will be the rise of B in the
same tube at the same temperature? [C.U.(H) 1972]
. h.g.r.p
H mts : YA = 2cos0
, cos 0 = l .
• . YA = -2l x l 0 x 981 x r x 2p

l
y8 = -h x 981 x r x p
2
1
h = 40, YA = 2x· Ya = x.
6. Calculate how far water will rise inside a capillary of internal diameter 0.25 mm;
the surface tension is 72.76 dynes/cm and its density is 0.998 gm/cc in C.G.S. system.
[C.U. 1983; B.U. '96]

Solo. : r= l
-hpgr, h
2
2y
= -pgr = 0. 9982xx980.
2 x 72. 76
665 x 0. 025
= 11.89 cm.
7. Calculate the energy expended in breaking I cc of water into droplets having mean
radius w-5cm. Given.: ST of water (Yw) =
72 dynes/cm.
4
Solo. : Volume of l sphere : -rt(I0-5 )3
3
= 4.1893 x l0- 15 cc.

1
. . number of spheres - - 2 3870 x 10 14
- 4.1893x 10 15 - · ·

Total area 4m2 = 4 x 3.142 x (I0-5)2


= l.2568 x 10-9 sq. cm.
~nergy needed in breaking,
E = (4rtr2)x n x y
= (1.2568 x 10-9 ) x n x y
= 1.2568 x 10-9 x 2.3870 x 10 14 x 72 ergs
= 2.15 x 107 ergs
= 2.15
J.
8. Calculate the' radius of the largest drop of a liquid of surface tension (at 17 °C)
80 dynes/cm that can evaporate at 17 °C. The latent heat of evaporation is 250 cal/gm.
Solo. : Let the radius of the largest drop be r cm.
Then energy needed =m x le, where m is mass of the drop, le is latent heat per gram.
= V x p x le, where V is volume of the drop, p is density.

= im3 x p x le = 4m2y
3
Let d = l gm/cc.
3 x 80 dynes. com -I
= 250cal. gm- x 4.184 x 10 7 erg. ca1- 1 x I gm.cc- 1
1

= 2.294 x to-8 cm.


LIQUID STATE (SURFACE TENSION AND VISCOSITY) 79

9. A liquid of density 0.9 gm/cc is kept in a beaker. A clean capillary tube of diameter
0.033 cm is dipped into it. It rises to a height of 3.8. cm. Calculate the surface tension
of the iiquid.
Solo. : Given, p = 0.9 gm/cm 3
2r = 0.033 cm
h = 3.8 cm.
1
y =2 h.g.r.p.

0.033
= 0.5 x 3.8 cm x 980.665 cm.sec-2 x - - cm x 0.9 gm/cc
2
= 27.68 dynes/cm.
10. A capillary tube of radius O.Olcm is dipped in a liquid of density 1 gm/cc. Its surface
tension is 74 dynes/cm. Find the height to which the liquid will rise in the capillary tube.
2
Hints : h = Y = 15.1 cm.
grp
"11. A sphere of water of radius 1 mm breaks into as million drops of same size. Find
the work done (y = 72 dynes/cm).
4
Solo. : Volume of sphere = V = - 7tr 3
3

3'4 x 3.142 x (0.1) 3 = 4.189 x.10-3cc.


=

Surface area S = 4m 2 = 4 x 3.142 x 0.1 2 = 0.12568 cm 2.


Energy = 47tr2y = 0.12568 cm 2 x 72 dynes/cm
= 9.04896 ergs.
4.189 x 10-3 cc contains 106 drops.
3
Volume of I drop = 4.189x10-
6
= 4.189 x 10-9 cc.
10
4
. . -7tr3
3
= 4.189 x 10-9

or, r3 = 9.9992 x 10-10


r = 9.9973 x 10-4 cm.
Energy of I 0 6 drops = area of one drop x y x 106
= 4 x 3.142 x (9.99 x 10-4 cm)2 x 72 dynes/cm x 106 = 904.8 ergs.
Work done = (904.8 - 9.04896) ergs = 895.75 ergs.
12. An atomiser produces 105 droplets of same size from 0.5 cc of a liquid. Find the
increase in surface energy, if the S.T. is 80 ergs/cm 2 .
Solo. : 0.5 cc gives 105 droplets .

.. 0.5 = ~7tr3 x 10s


3
I
3 5 3
r = ( 4 x 3.142
x O.
x 10 5
) = 0.010607385 cm = 0.0106 cm.
80 PROBLEMS ON PHYSICAL CHEMISTRY

E = 47tr2y x n
= 4 x 3.142 x 0.01062 x 80 x 105 ergs
= 11297. 12 ergs
= 1.1297 x 104 ergs.
13. At 293 K, IO cc of water gave 29 drops and IO cc of ether gave 87 drops in the
=
same stalagmometer. The density of ether is 0. 7 gm/cc and "ftt 0 72 dynes/cc. Calculate
2
the surface tension of ether.
Hints : Yi = rether r2 = rn20
-v 1
-'- = _..n;.p
'--1 -v
or, -'-1 = 29x0.7 , :. y1 = 16.8.·
Y2 n1P2 Y2 87x l

14. A spherical soap bubble of volume ~ cm 3 stands suspended in air. What is the
6
excess pressure inside the bubble? Given the interfacial tension for soap solution air interface
= 27 dynes/cc. [C.U. 2000)
4
Soln. : The volume occupied by soap bubble of radius r is -7tr3. By the problem,
3

i1tr3 = ~.
3 6

or, r3= -I cm 3 .
8
.. r = 0.5 cm.
If P is the excess pressure in the concave side (internal)

P = 41r = 270.5x 4 = 0.5


108
= 216 dynes/cm.
15. Find the change in surface energy when two identical mercury droplets of diameter
2 mm merged isothermally to form I drop ("fHg 49 dynes/cm).= [C.U. 2000)
Soln. : Given, "fttg = 49 dynes/cm, d = 2r = 2 mm.
.. r = 0.1 cm.
4
Volume of one droplet V = -7tr3
3
= 4.189 x 10-3cc

For 2 droplets : V = 8.378 x 10-3 cc.


Total surface energy = 4 7t r2y x 2
= 4 x 3.142 x 0.1 2 x 2 x 49
= 12.3166 ergs.
Two merged : total volume = 8.378 x m-3 cm 3.
4
- 7tr3 = 8.378 x 10-3
3

r =
3 x 8. 378 x I 0-3
4x 3.142
= vl. 99984 x 10-3

.. r • = 0.12538763 cm.
LIQUID STATE (SURFACE TENSION AND VISCOSITY) 81

Surface area of one large drop SA= 4 x 3.142 x (0.125) 2


= 0.196375 cm 2.
Total surface energy of merged drops = SA x 49
= SE = 0.196375 x 49 = 9.622375 ergs.
SE of mixed drop : 9.62 ergs.
Decrease in surface energy = 2.69 ergs.
16. At 20 °C the interfacial tension between water and benzene is 35 mNlm. If the
surface tension y = 28.85 mNlm for benzene and 72.75 mNlm for water, calculate :
(i) the work of adhesion between water and benzene, (ii) the work of cohesion for
benzene and for water, (iii) spreading coefficient for benzene on water. [B.U. 20031
Solo. : Work of adhesion = (YA + Ys - YAs)
By the problem, YA = 72.75 x 10-3 Nim
Ys = 28.85 x w-3Nlm
YAs = 35 x I 0-3 Nim .
. . work of adhesion = (72.75 + 28.85 - 35) 10-3 Nim
= 66.60 x 1o-3 Nim. or Jlm2
= 66.6 mJ/m 2.
Work of cohesion = 2yi
Therefore, work of cohesion for benzene
= 2 Ys = 2 x 28.85 x 10-3 Nim
= 57.7 mJlm 2.
For water = 2 x 72.75 = 145.5 mJ/m2.
Spreading coefficient for benzene on water,
CJsA = YA - Ys - YAs
= 8.9 x 10-3 Nim
= 8.9 mJ/m 2.
17. Show that for a given total volume, the surface area : volume ratio of a cube is
about 514 times of a sphere. [B.U.(H) 2002]
Soln. : Let the total volume of cube be 1000 cc (a 3 = 1000; a :;:: 10). Total surface
area of a cube is 6a 2 = 6 x 100 = 600. So surface area : volume ratio in a cube is 0.6.
Let a sphere with volume I000 cc be of radius r.

.. _± m 3 = 1000 cc
3
I

.. r = (3x1: Y 0
= 6.2032348 cm.
So surface area is 41tr2 = 4 x 3.142 x (6.2032348) 2
= 483.61742 = 480 cm2.
Surface area = 480 = 0.4 8 .
Volume 1000
5
So, 0.48 x = 0.6.
4
Prob. Phy. Chem.-6
82 PROBLEMS ON PHYSICAL CHEMISTRY

Thus, the surface area I volume ratio of a cube is ~ times of a sphere.


4
18. An excess pressure of 364 Pa is required to produce a hemispherical bubble at
the end of a capillary tube of 0.3 mm diameter immersed in acetone. Calculate y.
[B.U. 2000]
Solo. : Pexcess for the formation of hemispherical bubble = 364 Pa.
Given, 2r = 0.3 mm.
:. r = 0.15 mm = 0.015cm = 0.015 x 10-Zm.
2
Now, P = 'Y [y = surface tension, r = radius of bubble, P = excess pressure]
r
2
.. y P x r = 364 x 0. 015 x 10- = 0. 0273 N.m-l
= 2 2
= 27.3 x 10-3 Nim = 27.3 dyoes/cm, i.e., mN/m.
19. A spherical drop of a liquid weighing 0.04 gm is dispersed into 1500 microglobules
of radius 0.02 cm by a suitable experimental device. Find the resultant increase in surface
energy. [C.U. 1998]
Density = 0.8 gm/cc
'Y = 27 dynes/cm 2
m m 0.04gm
Solo. : p = , .. V= = = 0·05 cc.
v p 0.8m I cc

±7tr3= 0.05.
3
3 5
or
,
r3= x0.0
47t
=0.01193662.

r = 0.228539 = 0.228 cm.


Initially surface energy,
E = 4m2y = 4 x 3.142 x (0.228) 2cm 2 x 27 dynes/cm
= 17.64004262 ergs= 17.64 ergs.
After formation of 1500 microglobules,

1500 x ±m3 = 0.05.


3
Hence, r = radius of one micro globule = 0.019 cm.
E = 4rtr2rn
= 4 x 3.142 x (0.019) 2 x 27 x 1500
= 183.7504 ergs.
Hence, the increase in surface energy
= 183.7504 - 17.64 = 166.1104 ergs.
20. A capillary tube of radius 0.001 cm is inclined at an angle of 45° to the surface
of a liquid. The liquid wets the wall. It has a density of 0.85 gm.cm-3 and surface tension
of 36 dynes.cm- 1. Calculate d, the distance along the capillary to the meniscus.
LIQUID STATE (SURFACE TENSION AND VISCOSITY) 83

Solo. : By the problem,

The vertical height of the meniscus is given as :


2 36
h = 2"( = - - --x-- - - - = 86.38 cm
pgr 0.85 x 980.665 x 0.001
h = d sin 45°
h
d = - - = 122.16 cm.
sin45°
21. For a water-oil interface at 25 °C and 1 atm, calculate the rise of water in a capillary
tube of inside diameter 0.2 mm. The surface tension of water is 72 dynes/cm at 25 °C.
The densities of water and air at 25 °C and 1 atm are 0.997 gm/ml and 0.001 gm/ml
respectively. Should the rise be lower at 40 °C? Discuss briefly. [B.U. 2002]
0 02
Solo. : Given, r = · cm = 0.01 cm
2
= 72 dynes/cm
= 0.997 gm/cc
= 0.001 gm/cc
2"( 2 x 72 dynes I cm
h
= pgr = (0.997-0.00l)gm/ccx980.66cm.sec-2 xO.Olcm
144
= - - - - - - - - cm= 14.74 cm.
0. 996 x 980.665 x 0.01
The rise will be lower at 40 °C since y decreases with increase in temperature.
22. A tube of internal diameter 4 mm is dipped into a liquid of surface tension 98
dynes/cm. Find the approximate value of the capillary rise. [B.U. 2003]

2 2x98
Hints : h = 'Y =
pgr 980.665 x 0.2x d
23. At 20 °C pure water with an absolute viscosity of 0.01009 dyne.sec.cm-2 requires
102.2 sec to flow through a capillary of an Ostwald viscometer. At 20 °C tolueQe requires
68.9 sec. If the density of water and toluene be 0.998 gm/cm 3 and 0.866 gm/cm 3 respectively,
calculate the viscosity of toluene. [V.U. 1997]

Hints : llwater = PH20tH20

11 toluene Ptoluene (toluene

• · 11toluene = ( 0.1009 x 0. 866 x 68. 9)


0. 998 x 102. 2
= 0.059 -2
dynes.sec.cm .
84 PROBLEMS ON PHYSICAL CHEMISTRY

24. A spherical drop of water, 0.4 cm in radius is split into 125 identical tiny spherical
droplets. Find the increase in surface energy. [y for water = 72 dynes/cm] [C.U. 2002)
Solo.: Area of the drop:::: 4m2 = 4 x 3.14 x (0.4) 2 = 2.01 cm2.
. 4 4
Volume occupied = -7tr3 = - x 3.14 x (0.4) 3 = 0.268 cc.
3 3
If it is divided into 125 drops and r be the radius of new small drops, then

125 x in;r3 = 0.268 cc


3
3 x 0.268
or, r3 = 125 X 4X 3.14
r = 0.0799 cm = 0.08 cm.
So, total area for this drops
= 4n:r2 x 125
= (0.08) 2 x 4 x 3.14 x 125 = 10.05 cm 2 .
Hence, increasing area= (10.05 - 2.01} cm 2 8.04 cm 2. =
So the increase in surface energy
= increasing area x y
= 8 .04 x 72 = 578.88 ergs.
25. A steel ball of density I0.0 gm/cc having diameter of 4 mm is dropped into a column
of liquid. It takes 5 seconds to fall through a distance of IO cm. Calculate the viscosity
of the liquid. Explain the terms of your working formula; given, dliquid = 3.8 glee.
[B.U. 1995)
Solo. : We know that a falling body of mass m through gravity g produces a force =
mg.
Again, viscous force is 6 lrrT/V,
where the terms having normal meaning .
.'. mg = 6 lrrT/V.
But m = volume x density.
The volume of a spherical ball =in;r3 = V
. 3
:. m = Vp = V(p 5
- p1)
4
- -n:r3(ps - P1 ).
3
4
:. mg = -n:r3(ps
3
- P1) x g

4
or, 6m-11v = -n:r3(ps
3
- P1) g

2 r 2 (p,. -p1)g
:. T/ =9 v
Velocity is 2 cm/sec because it takes 5 sec to fall through a distance of IO cm.
Here r = 0.2 cm, v = 2 cm/sec, (p 5 - p1) = (IO - 3.8) gm/cc.
LIQUID STATE (SURFACE TENSION AND VISCOSITY) 85

2
= 3_ (0. 2)2 cm 2 x (10 - 3. 8) gm I cc x 980.665 cm.sec-
· · 1J 9 2 cm I sec
. = 27.02276889 gm.sec- 1.cm- 1 = 27 poise.
26. A droplet of water (r = 10-2cm) is falling in air at 25 °C. (T/ = 1.8 x J0--4 poise)
What will be the velocity of fall?

Solo. : By the problem, mg = im3 x g (assuming d =l gm/cc)


3

= ~x3.142x(10- 2 )
3
x980.665

= 4.108 x 10-3 gm.cm.sec- 2 = 6m-ryv.


i.e., 6 x 3.142 x 10-2 x 1.8 x 10--4 x v = 4.108 x 10-3 .
4.108 x 10-3 gm. cm. sec- 2
i.e., v = 6x3.142x10-2 cm x l.8x 10--4 gm.cm- 1 sec- 1
= 121.0599522 cm/sec = 121 cm/sec.
27. At 20 °C, pure water with an absolute viscosity of 0.01009 dyne. sec.cm-2 requires
102.2 sec to flow through the capillary of an Ostwald viscometer. At 20 °C, toluene requires
68.9 sec. If the densities of water and toluene be 0.998 gm/cm 3 and 0.866 gm/cm 3
respectively, calculate the viscosity of toluene. [V.U. 1997]

Solo. : We know that 2lL = p,t, ,


112 P2t2
where ry 1 and ry 2 are viscosities of liquid (I) (tolune) and liquid (2) (water) and similarly
d 1 and t 1, d 2 and t 2 are respectively the densities and time taken to fall through the same
length.
Given, ry 2 = 0.01009 poise.

:. T/1 = T/2 x p,t,


P2t2
By the problem, p1 = 0.866 gm/cm 3
t1 = 68.9 sec
p2 = 0.998 grnlcm 3
t2 = 102.2 sec
0.01009 x 0.866 x 68. 9
= - -0.998x
------
102.2
poise

= 5.90264 x 10-3 poise = 5.9 x 10-3 poise.


28. If there is l % error in the value of r, the radius of the capillary, what will be the
error in the viscosity coefficient value calculated by using Poiseuille equation?
[B.U. 1995]
Solo. : We know that by Poiseuille equation,
rtPr 4 t .
Tl = - - , where the terms have usual mean mg.
8/v
Let r 1 and r 2 be correct and wrong radii respectively and ry 1 and ry 2 be correct and
wrong viscosity coefficients respectively.
86 . PROBLEMS ON PHYSICAL CHEMISTRY

Now, ~~ = ( :~ r' where others have their usual value.

211- = 1.04. Hence if ry 1 is the original value, the error is 0.04, i.e., 4%.
111
29. In the absolute method of determination of viscosity coefficient T] by Poiseuille
formula, what should be the error in radius if error in T] is to be kept with 4%. [C.U. 1995)

r'
Soln. : We know from problem no. 28 that

= ( ;: where T]I' rl are the values in error and TJ2, rz are correct values.

.. 2lL
112
= 1.04 =
(;: r
(;: )
I
.. = (1.04)4 = 1.00985340 = 1.0 I.

i.e., the error is 1%.


30. In a tall bath of oil of viscosity 54 poise and density 1.8 gm/cc, steel balls of radius
2 mm when allowed to fall down, the column of oil acquires uniform velocity of I cm/
sec. What is the density of steel? [C.U. 1978)
Soln. : From problem no. 25, we have the formula-
=~ gr (p.,.-P1)
2
v
9 11
...
~ x 98lx(0.2) x(ps-l.8)
2
=
or,
9 54
54x9
or, ds - 1.8 = 2x 981x0.04
ds = 7.991 = 8 gm/cc.
31. At room temperature, the time taken by equal volumes of water artd ,toluene to
flow through a capillary tube are 102 sec and 69 sec respectively. Their densities at the
same temperature are 0.989 gm/cc and 0.86 gm/cc respectively. If the viscosity of water
is 0.01 P, calculate the viscosity of toluene.

Soln. : We know that 2lL = Pitt [11 1 =viscosity of water, 11 2 =viscosity of toluene]
112 P2t2
Here, 111 = 0.01 P, p 1 = 0.989 gm/cc, t 1 = 102 sec
112 = ?, p 2 = 0.86 gm/cc, t2 = 69 sec
O.Olx69x0.86
.. 112 = = 5.88 x 10-3 poise.
0.989x 102
LIQUID STATE (SURFACE TENSION AND VISCOSITY) 87

32. In a viscometer, water and another liquid flow in 100 sec and 250 sec. Calculate
the viscosity of that liquid, if its density is 0.8 gm/cc (11w = 0.01 poise).

Solo. : We know, _!11_ = Pili .


112 P2t2
Given, 172 = 11w = 0.01 poise,
r2 = 100 sec, t 1 = 250 sec, p 1 = 0.8 gm/cc and p2 = 1 gm/cc, 11 1 =?
p1t 1 0.8x250x0.0l .
. . 111 = --x 112 = = 002
. poise.
P2t2 lxlOO
33. A sphere of radius 5 x 10-2 cm and d = 1.10 gm/cc falls at constant velocity through
a liquid of density 1 gm/cc and viscosity 1 poise. What is the velocity of fall?
[C.U. 1987; B.U. 1978]
Hints mg = 67rr71v
4
mg = -m3 (Ps - p1)g = 67rr71v
3

4rcr3(P.,-P1)g 4r2(p,.-P1)g
v =
3 X 6TCT11 1811
34. Two soap bubbles of radius 4 cm and 5 cm are joined together so as to have a
common surface. Find the radius of the surface. [Ans. r = 20 cm]
35. Distilled water rose to a height of 4.8 cm in a certain capillary tube. The same
tube filled with mercurry of density 13.55 gm/cc contained 50 mg/cm of tube length.
Calculate y for water. [Ans. 75.45 dynes/cm]
36. Two limbs of a vertical U-tube have internal diameters 0.05 mm and 0.3 mm
respectively. It is particularly filled with a liquid of density 0.3 gm/cc. The surface tension
of the liquid is 60 dynes/cm. What is the difference in the levels of the liquid in the two
limbs. [Ans. 1.1 cm]
37. In a tall bath of oil of viscosity 54 poise and density 1.8 gm/cc, steel ball of radius
2 mm when allowed to fall down the column of oil acquires uniform velocity 1 cm/sec.
What is the density of steel? [Ans. 8 gm/cc]
38. A ball of radius l cm and specific gravity 10.5 requires 50 sec to fall a distance
of I m through a liquid of specific gravity 1.5. Calculate T]. [B.U. 2003]
Solo. : We know that mg = 67rr71v
4
or,
3
-m3 (Ps - p1)g = 67rr71v.
By the problem,
4
- n;J3 (I 0.5 - J.5) X 980.665 = 67trf\V
3
lOOcm
v = = 2 cm/sec.
50sec
4
)(9) x 980.665 = l2r71
r = 1 cm

11 = -43 x
9x980.665
12
= 980.665 dynes/cm.
88 PROBLEMS ON PHYSICAL CHEMISTRY

39 For a certain liquid, log T] vs / IT plot yields a slope value of 600 K. Estimate the
activation energy for flow. [C.U. 2006]
Solo. : Tl = AeEvis!RT
£.
:. log ry = log A + vis
2.303RT

By th(! pro bl em, 600 = Evis


2.303R
.. Evis = 2.303 x 600,K x 8.314 J.K-1.moJ-l
= 11.488 k,J.mol-1•
40. Calculate the work done in blowing a soap bubble in air of radius 10 cm. Surface
tension of soap solution = 30 dynes/cm. [C.U. 2004]
Solo. : Work done in blowing a soap bubble
= surface energy of the bubble
= surface area x r
=2 x 4m 2 x r(since bubble has two surfaces, total surface area =2 x 4m2 )
= 2 x 4 x 7t x l 0 2 cm 2 x 30 dynes/cm = 75398.22 ergs.
41. The surface tension of water is 72.8 dynes.cm- 1• Calculate the ·energy required to
disperse one spherical drop of radius 3.0 mm in spherical droplets of radius 3.0 x I o-3 mm.
[C.U. 2004]
Hints : Surface energy of one spherical drop =4 x 7t x (3x 10- 1)2 y
4
N x - x 7t (3 x
3
N = 109
Energy required = 4 x 7t x (3 x 10-3 x 10- 1)2 x y x 109 - 4 x 7t x (3 x 10-1 )2 x y
= 82252.53
ergs.
42. A steel ball of density 7.90 gm/cc and 4 mm diameter requires 55 sec to fall a
distance of I metre through a liquid of density 1.10 gm/cc. Calculate the coefficient of
viscosity of the liquid. [V.U. 2004]
2
Hints : h = 2 gr (P.,· - Pt) = 32.6 poise.
9 v
43. One end of a capillary tube 10 cm in length is connected to water supply which
has a pressure of 1.8 atm and other end to a vessel open to air at 25 °C; the tube delivers
1.2 litre/minute. Calculate the diameter of the capillary, if viscosity coefficient for water
at 25 °C be 0.0 l poise. [V.U. 2004]
Solo. : According to Poiseuille equation,
4
( P2 - P1 )1tr t
Tl = 8Vl .
Here, ry= 0.01 poise, P2 = 1.8 atm, P 1 = I atm
V = 1.2 litre = 1200 cc, t = 60 sec, l = 10 cm
LIQUID STATE (SURFACE TENSION AND VISCOSITY) 89

8T1Vl 8 x 0. 01 gm. cm- 1 sec- 1 x1200 cc x 10 cm


:. r4 = (P = (l. 8 -
2 - P1 )m I) atm x I. 01325 x 10 6 dyne. cm - 2 I atm x n: x 60 sec
= 6.283 x 10-6 cm 4 .
Hence, r = 0.05 cm.
:. diameter of the capillary = d = 2r = O.lcm.
44. A large glass bottle is fitted with a siphon made of capillary glass tubing. Compare
the time taken to empty the bottle when it is filled with (a) water of density l o3 kg/m 3
and (b) petroleum of density 0.8 x 103 kg/m 3 . Given that viscosity coefficient of water
is half that of petroleum.

Hints : 2:!1 = Pili , where l corresponds to petroleum and


T\2 P1t2
2 corresponds to water.

(;~ =2.5}
45. One end of a 10 cm long capillary tube is connected to water supply which has
a pressure of 1.8 bar and the other open end to air of 1 bar pressure. The tube delivers
0.21 water per min. Calculate the diameter of the tube if T\ of water is 0.01 P. (All are
at a fixed temerature of 298 K). [V.U. 2006]

1 2 n(P -P )r 4
Soln. : By Poiseuille's equation, T\ = ---'----'--
8Vl
·By the problem,
dyne/cm 2
P 1 - P2 = (l.8 - I) bar = 0.8 bar x 106 = 0.8 x 106 dyne.cm-2
bar
I = 10 cm r = radius of the tube
0.2xl0 3 _
V = 0.2 I/min cc.sec 1, T\ = 0.01 poise
60
6
dyne.cm-2 x r 4
Thus, 0.0 I poise = 3142x0.8x103
0.2xl0 i
8 x -------cc.sec- x lOcm
60
r4 = 0. 0 I gm. cm - l . sec -l x 8 x 10 cm x 0. 2 x 10 3 cm 3 . sec -1
60 x 3.142x0.8x10 6 gm.cm- 2 .sec·- 2
.. r = 0.032 cm.

Thus, diameter of the tube = (0.032 x 2) cm = 0.064 cm.


CHAPTER 4

SOLID STATE

Required Formulae
1. Bragg's equation :
n'A = 2dhkl sin (),
where n is order,
A is wavelength of X-rays,
dhkt is distance between consecutive planes,
() is glancing angle at which X-ray is diffracted by the lattice plane
2. For cubic crystals,

p = -ZM
-3,
NAa
where p = density,
M = moleculer mass,
NA = Avogadro number,
a = length of the edge of the cube,
Z = no. of atoms per unit cell.
3. If OA, OB, OC are intercepts of a crystalline plane on X, Y, Z respectively and
'a,b,c are unit distances along respective axes, then
a
-OA =h -
OB
b
= k -

(hkl) are Miller indices.


c
OC
= l.

4. For an orthorhombic lattice, a -;Cb -;Cc,


a= p= y = 90°.
'C y
B

If dhkl is the distance between two hkl planes or length of


the perpendicular on the hkl plane from origin, then

x
90
SOLID STATE 91

5. Rules for determining the diffracting (hkl) planes, responsible for appearance of
X-ray diffraction patterrn, in a cubic lattices
Simple cube (h,k,l) any value
b.c.c (h + k + /) =even
f.c.c (h, k, l) all odd or all even
Short Questions
1. What is Einstein characteristic temperature? Explain.
hv
Ans. : Einstein characteristic temperature () = k,
where h =Planck constant,
k =Boltzmann constant,
v = single monochromatic frequency with which an isotropic solid vibrates
Heat capacity Cv =3Rf( ()ff). •
This has a fixed value for a fixed term.
For different substances different values of v and ()are obtained, and these are carried
out purely empirically to fit experimental results.
2. What is Debye characteristic temperature?

Ans. : Debye characteristic temperature ()m = hv


where h =Planck const., k =
___!!!_,
k
Boltzmann const., vm =maximum frequency with which an isotropic solid can vibrate .

. At temperature below solid follows T 3 law, () = 7.45Tcv- 113 . As T increases, ()


.i_,
10
no longer remains constant. () can be calculated independently of atomic heat at low
temperature.
3. Metals do not obey T 3 law at a very low temperature--comment.
Ans. : In case of metals, excitation of the electrons must be considered. Electronic
ex9itation contributes to specific heat-a term linear in T.
Cv,el = b. T [b is around 10-4 to 10-3 cal.mo1-t K-2],
2

where b = NAm
2
( 2 7tk)2(~)3·
h 3n
n = no. of electrons per unit volume.
NA =Avogadro number.
For Ag at 300 K, b = 0.046 cal gm- 1 (about 7% of total).
Now, heat capacity, CV = cv,el + cv,l'
where cv,/ =contribution to CV due to lattice vibration.
At a very low temperature, lattice vibrations are negligible. Thus, the term containing
T has much higher contribution. For Ag at (l .5-0.3) 0 K, T 3 law is invalid and above
temperature 3 °K T3 law is valid. For Ni at (1.1-9·22) °K T 3 law becomes invalid.
At room temperature electronic contribution to C 1, is 0.03 to 0.3 cal.mcl- 1.K- 1, which
=
is far less than 3R 6 cal.mol- 1.K- 1• At moderate temperature, the electronic contribution
is thus only a small fraction of Cv. At a very low temperature, the electronic contribution
=
exceeds that of the lattice vibration since cv, 1 aT 3 , where a is a constant, goes to zero
faster than bT, T 3 law is not valid at low temperature.
92 PROBLEMS ON PHYSICAL CHEMISTRY

4. What is the primitive lattice corresponding to f.c.c. and b.c.c.?


Ans. : For f.c.c., the primitive lattice is rhombohedral and for b.c.c. the primitive lattice
is triclinic.
5. Explain why hexagonal closed packed structure and a cubic closed packed structure
for a given element would be expected to have the same density.
Ans. : The two structures have the same coordination number and hence the same·
packing fraction. So, they have the same density.
6. Show that in a crystal lattice there cannot be five-fold rotational axis of symmetry.
360
Ans. : Let AB be two points in a crystal lattice with length a. If it was a fold
e
axis of symmetry, then a rotation by angle 8clockwise and anticlockwise will lead to points
C and D respectively. Then the distance CD should be integer multiple of a, i.e., ma, where
m is an integer.
p Q
c

A
AP and BQ are perpendiculars drawn from A and B respectively on CD. Amongst the
triangle DPA and QCB, angle L.DAP = L.CBQ = 8- 90°. AD= BC= a, AP= BQ. Hence,
the two triangles are identical. So, DP = QC = x (say).
DP • .
But, - = sin (8 - 90°)
AD
or, DP = a sin (8 - 90°).
Now, CD = a + 2x = a + 2a sin ( 8 - 90°) = ma.
ma = a + 2a sin ( 8 - 90°) = a - 2a sin ( 8 - 90°) =a - 2a cos 8.
l - 2 cos 8 = m

or, cos 8 = l-m.


2
cos 8 can vary from 0 to I. Hence, m can be 0, I. 2 and 3. This is shown in the
following table :
m cos 8 8 n-fold axis

0 60 6
2
0 90 4

2 120 3
2
3 -1 180 2
Thus, there cannot be five-fold axis.
SOLID STATE 93
7. Show that for an orthorhombic lattice the distance between two planes dhkl is given
by

+ = (!!_)2
~u a
+ (!5__)2 + (!_)2
b c
Ans. : Let us consider a plane having Miller y
+ ++
indices hkl. Its equation is hx ky lz d = 0. Let
dhkt be its distance from the centre. It can be shown
that the distance between two planes is equal to the
distance between the plane and the centre.
Let the axes and axial ratios a, b, c be as shown
and the plane be ABC.
OD= dhkt
x
OD is perpendicular on the plane from centre.

dhkt = OA cos a =(*)cos a

b
= OB.cos p= -cos p.
k
c
= oc cos "{ = -cos"(.
l
h
cos a = -a dhkl

k
cos p = - dhkl
b
l
cos"{ = -dhkl
c
cos a, cos p, and cos "{ are direction cosines.
Here, cos 2 a + cos 2 p + cos2 "{ = I.
.. (~Yd;k,+(~yd;k,+(~yd;k, =

or, + = (!!_)2
dhkt a
+ (!5__)2 + (!.:)2
b c
In case of a cube, a = b = c.
h2 +k2 +[2
:. -2- =
dhkl

8. Define a unit cell.


Ans. : The smallest collection of atoms from which the whole crystal can be built
up by mentally translating the unit parallel to itself in the direction of its edges by distances
equal to the length of its edges is called unit cell of the crystal.
It represents the symme~ry of the corresponding crystal.
94 PROBLEMS ON PHYSICAL CHEMISTRY

9. What is unit cell content?


Ans. : The unit cell content Z is the number of particles contained within the unit
cell. A primitive ~nit cell contains one particle per unit cell.
A non-primitive unit cell is edge centred, face centred, or body centred and contains
more than one point or particle per unit cell.
10. What are Miller indices?
Ans. : Miller indices are a set of integers (hkl)
that are used to describe a given plane in a crystal.
They are reciprocal of intercepts in terms of the
intercepts. Thus, if ABC is a plane with intercepts
OA, OB and OC on the three axes, then

!!.__ l = ~-
h = _!!___ k =
OA' OB' OC
11. What is the highest order that can be observed in Bragg's reflection from a solid
by X-ray?
Ans. : We know that n'A = 2d sin0.
where n is the order of reflection by the lattice planes with interplanar separation d
and A is the wave length of X-ray.
:. n'A :::; 'li
Since (sin 0)max = l,
• 2d
hence , n < -
- A
So, the highest order that can be observed depends both on lattice plane separation
2
and wave length of the X-ray and it is :

12. What is Bravais lattice?


Ans. : A Bravais lattice is an array of equivalent points that reflect the symmetry of
representative crystal. It is a repeating unit but devoid of any physical properties.
13. What are the close-packed planes in a (i) b.c.c. lattice; (ii) f.c.c. lattice
Ans. : (i) In b.c.c lattice, ( 110) planes are close packed.
(ii) In, f.c.c lattice ( 11 l) planes are close packed.
14. What is the closest distance between two neighbouring atoms in (a) f.c.c lattice;
(b) b.c.c lattice.
Ans. : (a) In f.c.c lattice, the close packed direction is along face diagonal of the cubic

unit cell. If the length of.the unit cell is a A, the closest distance between two atoms= .fia
2
a
= .fi.
(b) In b.c.c lattice, the close packed direction is along body diagonal of the cubic unit

cell. If the length of the unit cell is a A, the closest distance between two atoms = {Ja
2
Numerical Problems
1. A crystal plane intercepts the three crystallographic axes at the following multiples
3
of unit distance - , 2 and l. What will be the Miller indices of the plane? [C.U. 1980]
2
SOLID STATE 95
Soln. : Miller indices of the plane,
a b c 2 l
= -
2 ·.l - -4 · 3
. ·.6 .
3-a 2b . c 3
2
:. (hkl) = (4 3 6).
2. Tungsten has body centred cubic lattice and each lattice point is occupied by one
atom. Calculate the length of the cube edge (density of Tungsten is l9.30 gm/cc and its
atomic wt. is 183.9). [C.U. 1978]

Soln. : We know, p = -MZ-3 .


NA-a
Given, p = 19.30 gm/cc.
z = 2
M = 183.9 gm
NA = 6.022 x 1023
l83.9x2
19.30 = ---~,,....-.--=-

6. 022 x 10 23 x a 3

or, a3 = 183. 9 x 2 = 3.1645 x 10-23


19.30x6.022x10 23
.. a = 3.163039 x 10-8 cm = 3.163 A
So, the length of the cube edge is 3.163 A.
3. Aluminium crystallizes with a face-centred cubic lattice. The inter-ionic distance
(shortest) in a unit cell of Aluminium is 2.86 A. Calculate the density of Aluminium
(At. wt. of Al = 27).
MZ
Soln. : We know, p = - -3
.
NA.a
By the problem, 4r = ,/Ja.
Here, M = 27 gm
z =4
NA = 6.022 x 1023
2r = 2.86 A.
Now, 2 x 2r = .,[?a
or, 2 x 2.86 A = .,[2a.
:, a = 2.86.J2x 10-8 cm.

27gm x4
.. p = 6.022xl0 23 x(2.86xl0- 8 xl.414) 3 cc
= 2.809 gm/cc = 2.81 gm/cc.
4. At room temperature sodium has body centred cubic lattice with a cell of edge length
4.29 A. At -200 °C, the density is only 4% larger but the cell edge is now 5.35 A. What
type of cubic unit cell does sodium possess at -200 °C?
Soln. : At room temperature,

p
96 PROBLEMS ON PHYSICAL CHEMISTRY

23gm x 2
= 23 3
6. 022 x 10 x (4. 29 x l o-S cm )

= 0.967487 gm/cc = 0.97 gm/cc.


At (-200 °C) the density is 0.97 x 1.04
= 1.006186 gm/cc = 1.0062 gm/cc.
. k MZ .
A gam, we now, p = - -3
NA a
23 3
= p.NA-q_ = I. 0062 x 6. 022 x 10 x ( 5. 35 x 10-8 ) = 4.0342 = 4.
3
z
M 23
Hence, at -200 °C, sodium has f.c.c. lattice.
5. Caesium bromide lattice contains 8 bromide ions in the corner of a cube with a
cs+ ion at the centre. Its density is 4.5 gm/cc. Calculate the edge length.

Soln. : We know, p = -MZ-3


NAa
Now, NA = 6.022 x 10 23 , p = 4.5 gm/cc, Mes+= 133 gm and M 81 = 80 gm

:. a3 = MZ =
(Mc_,.++ MBr_ )z
NAP

= (133gm+80gm)xl = 7.86 x 10-23 cm3.


6.022x 10 23 x4.5gm.cm-3
. . a = 4.283 x 10-8 cm.
6. The 222 planes of Tantalum (cubic) give a reflection at 54° 7.2', when the X-ray
has 'A= 1.5418 A. At what angles would these planes give reflection, when 'A= 0.7107 A?
What is the edge length? What are the spacings between 100, 110 and 111 planes?
Soln. : Here 'A = 1.5418 x I o-8 cm
() = 54° 7.2'
Now, sin () = sin 54° 7.22' = 0.81
Again, 'A = 2d sin ()

'A l.5418x 10-8


or, d =
2sin8
= 2x0.81
= 0.95 x 10-8 cm = 0.95 A.

'A 0.7107x 10-s


Again, sin()= 2d = 2 x 0. 95x10-8 = 0.37405.
() = 21.96577 = 21° 58'.
a
We know, d = 2-y 3
,,, (for 222 planes)

or, a = d x ../3 x 20.95 x I o-8 x ../3 x 2


= I .6454 x 10- 8 x 2 = 3.28 A.
SOLID STATE 97

a a
=a:..fi.:.fj

3 28 3 28
= 3.28 : • : · = 3.28 : 2.32 : 1.9.
1.414 I. 732
7. The crystal structure of magnesium oxide is similar to sodium chloride. The distance
between Mg++ and o= ions in the cell is 2. l 0 A. What is the density of MgO?
Soln. : Here, a = 2dMgO [a = 2(r+ + r_ )]
= 2 x 2.10 x 10-8 cm = 4.20 x io-8 cm.
'MZ
.. p
= N a3
A

Here M = 40 gm
z
=4
NA = 6.022 x 1023
a = 4.20 x 10-8 cm
p =density.
40x4
.. p= 3 = 3.586 gmlcc = 3.6 gm/cc.
6.022x 10 23 x(4.20x10-8 )

8. KCl has a face centred cubic lattice. Density of KCl is 2 gmlcc at a certain
temperature and edge length is 6.28 A. Find the number of K+ and c1- ions per gram.
Soln. : Given,
p = 2 gmlcc
a = 6.28 x 10-8 cm
z 4. =
Volume of unit cube = a 3 = (6.28 x 10-8 cm)3 = 2.477 x I 0-22 cc.
Hence, weight of unit cube = 2 x 2.477 x 10-22 gm = 4.954 x 10-22 gm.
Unit cube contains 4K+ and 4c1- ions.
4
:. number of K+ or c1- ions per gram = = 8.1 x 102 1 ions/gm.
4. 954 xi 0-22
9. The element chromium exists in a body centred cubic lattice whose unit cell edge
is 2.88 x 10-8 cm. The density of chromium is 7.2 gmlc.c. Find out the Avogadro number
(At. wt. of Cr = 52).
MZ
Soln. : We know, p =
NAa 3
Here, p = 7.2 gmlcc
M = 52 gm
z = 2
a = 2.88 x l0-8 cm
.. NA = 6.047 x 1023•

0.-~1.. f'l..,. f"'L ,..


98 PROBLEMS ON PHYSICAL CHEMISTRY

10. For LiBr the heat of formation is reported as 97.4 kcal. The heat of vaporisation
of Li is 37 kcal. Ionisation energy of Li as 123.9 kcal per gram-atom. Calculate the lattice
energy of LiBr. Assume that electron affinity of Br 2 is 87 kcal/gm and the heat of dissociation
of bromine is 53 kcal.
Solo. : As given in the problem,
-87 kcal
Lt+ Br
53 kcal

L.E.
123.9 kcal
Li(g) + Br2

37 kcal
LiBr(s)
Li(s) + Br 2
~H
-97.4 kcal

37 + 123.9 - 87 + 53 + L.E. = -97.4


.. L.E. = -37 - 123.9 + 87 - 53 - 97.4 = -229.3 kcal.
11. A powder pattern is obtained for a-phase of a metal. Successive sin 20 values are
in the ratio : I, 2, 3, 4, 5, 6, 7, 8. Which type of cubic unit cell is present? Discuss the
probable hkl value responsible for sin 2 0 = 7 in the above series. The density of a-metal
is 7.91 gm/cm- 3 . The wavelength of the X-rays is 0.5 A. Calculate a for unit cell. Calculate
the value of 0 for Bragg reflection for 110 type planes and 100 type planes. Calculate the
radius of the atom. (atomic wt. of the metal is 56). The ~-form is f.c.c. cube. Its a = 3.60
x 10-8 cm. Also calculate the radius of the metal in a- and ~-forms. Calculate the ratio
of their densities.
Solo. : 7 cannot be the sum of squares and it must actually be 14. The series is then
2, 4, 6, 8, 10, 12, 14, 16.
The crystal (a- form) is thus b.c.c.
h2 + k2 + t2 = 14
hkl = 321, not necessarily, may be 231 or 123 in that crystal.

a3
MZ 56x2
=NAP= 6.022xl0 23 x7.91.
Thus, a = 2.864839 x I o-8 cm = 2.86 A

dwo =
a
= 2.86A = 1. 43 A
2 2

sine = ~ = 0.174825, .. e = 10.01°.


2.86

a 2.s6A
d110 = {2 = {2
= 2.022325 A = 2.02 A
SOLID STATE 99

0.5
sine =
2x2.02
= 0.123762, .. e = 7.11°.
for a-form,
4r = ./?a
.f3a .f3 x 2. 86 x 10-8
r = 4
= 4
= 1.238416'A = i.24'A

112
= ------=--
3 3
( 2. 86 x 10- ) . NA

For the ~-form,


MZ 224
= 3
= 3 (since Z = 4)
NA"a NAx(3.6x10- 8 )
and 4r .j2a
{2x3.6x10-8
r = 4
= i.21 A.

Now £l = 112
3
x
NA x (3.6x10- 3 r
P2 (2.86x 10-8 ) NA 224
3
(3. 6) 46.656
.. £l = = = 1.
P2 (2.86) 3 x2 46. 787
12. An orthorhombic unit cell has the following parameters :
a = 0.5 A, b = l A, c = 1.5 A. What is the spacing of the ( 123) planes?
[C.U. 1990 1996; B.U. 1988]
Solo. : For orthorhombic unit cell, a ;t!b ;r: c,a = ~ = y = 90°.
The· distance between two planes having Miller indices (hkl) is
1. h2 k2 [2
= -a2+ b2
-+-
c2
dKkl
12 22 32
or,
dfkl
= -(0.5)
-+ 2
-.- +--
(1) 2 (1.5) 2

I 4 9 9+9+9 27
or, = --+-+--
0.25 I 2.23
= 2.25
= 2.25
= 12
dKkl

.. dfk/ = 12
.. dhkl = o.2886 'A = o.289'A = o.289 x I o-8 cm = 2890 pm.
13. Find the simplest formula of a solid containing Aand B atoms in a cubic arrangement
in which A occupies corner and B the centre of the faces of unit cell. If side length is
5 A, estimate the density of the solid assuming atomic weights of A and B as 60 and 90
respectively. [C.U. 1977]
!00 PROBLEMS ON PHYSICAL CHEMISTRY

Soln. : A atoms occupy 8 corners. Each corner is shared by 8 cubes. So, the single

cube shares _!.th of each point. Thus, there is one 'A' atom (8 x _!.A). The B at6ms occupy
8 8
the centre of faces of the unit cell. Thus,, there are (6 x ..!_) = 3B atoms occupied by the
2
single cube. Thus, the formula of the compound is AB 3 .

We know that density p =


MZ3 . Here, MZ is the full weight of the occupying atoms
NAa
which here is weight of the AB 3 = (60 x I) + (90 x 3) = 330 gm.
330
Thus, density = 3 = 4.3839 gm/cc = 4.39 gm/cc.
6. 022 x 10 23
x (5 x 10-8 )

14(a). An element occurs in two crystalline forms a and p. The a-form has f.c.c.
with a = 3.68 A and the P-form has b.c.c. with a = 2.92 A. Calculate the ratio of their
densities. [C.U. 1978]
Soln. : As given in the problem-
4M 4M
Pa = N a3 =
A

2M 2M
and p~ = -NA-a3 = 23
3 .
6.022x10 x(2.92x10-8 )

Pa 4M {2.92) 3
Pp
=- x
2M (3.68)
3

3
Pa 2 x (2. 92)
or,
Pp
= {3.68) 3

Pa 49. 794176
= 49.836032 = 0.9991660125.
Pp

Pa == I.
Pp
Thus, the densities are in the ratio 1 : 1.
14(b). Same as 14(a), only difference is now a = 3.65 A for f.c.c. and a = 2.90 A
for b.c.c. Calculate the ratio da: d f3 as above. [C.U. 1999]
3
Soln. : Pa = 4 x {2. 90)3 = 1.0031.
Pp 2{3.65)

.. Pa ::::: I.
Pp
15. A metal has a b.c.c. cubic lattice and the length of a unit cell is 2.95 A. If the density
of the metal b.e 9.95 gm/cc, calculate the atomic weight of the metal. [C.U. 1995]
SOLID STATE 101

MZ
Hints : p =
N Aa 3 '

where Z = 2
a = 2.95 x 10-8 cm
p = 9.95 gm/cc
.. M =?
Ans. : M = 76.91302 gm = 76.9 gm.
16. Potassium crystallizes with a b.c.c. lattice and has a density of 0.835 x I 0 3 kg.m- 3.
What is the length of the side of the unit cell.' a' and the distance between (200) planes and
that between (110) planes. [C.U. 1993)
Soln. : By the problem,
Z =2 for b.c.c.
At. wt. of potassium
= 39 x I o-3 kg/gm-atom.
p = 0.835 x 103 kg.m-3
MZ
p = NAa3

39x 10-3 x 2
or, a3 =
6. 022 x 10 23 x 0. 835 x 10 3
.. a = 5.37306 x 10- 10 m = 537.306 pm.
a 537.306
.. dzoo = = = 268.653 pm.
2 2
a 537.306
d110 = ..,/2 = ..,/2 = 379.93 pm.

17. A face centred lattice has all the positions occupied by atom A The body centred
octahedral hole in it is occupied by an atom B. For such a crystal calculate the void space
per unit volume of unit cell. Predict the formula of the compound. [V.U. 2005]
Soln. : In the face centred lattice 4rA =· .,/2a, where a is edge length of the cube
and rA is the radius of A atom.
:. a = 2../2 rA.
The body centred octahedral hole h is occupied by B atom .
. . 2r8 + 2rA =a= 2.,/2rA.
. . 2r8 = (2.,/2 - 2)rA.
oA
. . !Ji_ = (.,/2 - I) = 0.414. oB
rA

Volume of the cube is a3 = (2.,/2r A)3 = 16.,/2 r A3


Occupied volume by A and B :

3 3)
4 x -43 7tr A + -43 7tr 3B = -4 7t ( 4rA + r 8 .
3
3
102 PROBLEMS ON PHYSICAL CHEMISTRY

Volume occupied per unit volume of the unit cell is

i rr(4r A3 + r 83 )
3
=
l6.J2r~

= -2.!i.__(4+0.414 3 )
12xl.414
3.14 x 4. 070957944 3.14X 4.071
= 12xl.414
= -12Xl.414
- - - = 0.7534.
So, 75.34% is occupied.
:. void space per unit volume of unit cell = 24.66%.
There are four A atoms and one B atom per unit cell. So the probable formula is BA4 .
18. Calculate the ratio of the distances among (I 00), ( 110) and (I ! I) µlanes of a simple
C'ubic crystal with a lattice side of 1.40 A. [C.U. 1994]
Solo. : Let a be the length of the unit cell. We kr.0w that, for a simple cube the ratio
of the distances are-
a a
=a: ~·
.J2 ....;3
In this case, a = 1.40 A
Hence, d 100 =a = 1.40 A

d110 = a = I.40 A
.J2 .J2
I.~oA'.
d111
a =
= -J3 3
'V.J

Thus, d100 : d110 : di 11

a a
= a:
.J2 -J3
I l
=
.J2 -J3
= 1 : 0.707 : 0.577
= 1 : 0.70 : 0.58.
19. Using X-ray of wavelength 'A = 1.79 x 10-8 cm, a metal produces reflection at
28 = 47.2°. If this be the first order reflection from the (110) plane of b.c.c. unit cells,
what is edge length of the cube? [V.U. 1994)
Solo. : Given that, 'A = 1.79 x 10- cm.
8

By Bragg's law, for lst order reflection, 'A = 2d sin e.

'A I. 79 x 10-3
:. sin()= =
2d 2d
SOLID STATE 103

For b.c.c. unit cell,


a
dl 10 = .fi..
20
By the problem, e = 472· = 23.6°.
So, 1.79 x 10-8 = 2d sin 23.6 = 2d x 0.40035.

1.79x10-8
. . 2d110 = 0.40035

79 10 8
d 110 1. x - = 2.23554 x 10-8 = 2.24 A.
= 2x0.40035

a
Now d 110 = .fi..
a = .,/2 X d 1IO
= .,/2 x 2.24 A = 3.1678 A = 3.11 A.
20. The first order reflection from a crystal plane in a cubic crystal occurs at 13°41'.
Find the Miller indices of the plane. Given, a= 5.63 A, A= 1.54 A [sin 2 13° 41' = 0·056].
[B.U. 1992]
Hints : Use the formula,
n'A. = 2d sine.
n = 1, A. = 1.54 x 1o-8 cm
'A
2sin0

Again, dhkl = a = 3.25 x I o- 8 cm.


~ h2 + k2 + /2
5.63x10-8
Thus, ~h 2 +k2 +1 2 =
= 3.25x10-8 = 1·73 ·
h2 + k2 + 12 = (l.73) 2 = 3.
Hence, h = k = l = 1.
.. Miller indices (111).
21. The characteristic Ka. lines of Cr, Fe and Ni have wavelengths of 2.29 A; 1.94 A
and 1.66 A respectively. Which of these may be used to determine a lattice spacing of 1 A?
[B.U. 1994]
Solo. : By Bragg's equation we know,
n'A. = 2dsin e.
Now, for maximum value of A, sin e = 1 and n = l.
:. 'A= 2d.
Given, d = 1 A, :.
A = 2 A.
Thus, Ka. lines of Fe and Ni can be used since the value of A has to be lesser than
or equal to 2 A.
104 PROBLEMS ON PHYSICAL CHEMISTRY

22. Sodium crystallizes in b.c.c. structure with a = 4.24 A. Calculate (i) theoretical
density and (ii) radius of Na atom. [B.U. 1994)

. MZ
Hmts: p = -- 3
NAa
Here, Z = 2, NA = 6.022 x 1023 , a = 4.24 x 10-8 cm, M = 23 gm.
.. p =?
23x 2
p = 23 3
6. 022 x 10 x ( 4. 24 x 10-8 )

= l.0021194 gm/cc =l gm/cc.

Again, r =
-l3Q .J3
= - x 4.24 x l o-8 = 1.83597 A.
4 4
:. r = t.84 A.
23. Metallic gold crystallizes in f.c.c. lattice, the length of the cubic unit cell being
4.070 A (l) Calculate the closest distance between gold atoms. (2) How many nearest
neighbour does each gold atom have at the distance in (l)? (3) Calculate the density of
gold.
Soln. : ( l) In f.c.c., the closest distance of an atom is to the atom at the centre of
the face.
Now, for f.c.c., ffa = 4r.
4 70
·~L. A
2
0

= 2r = ../2
a = 'V
0

.
. . the c Iosest d1stance be tween the go Id atoms -- 2.878 A •

(2) Each gold atom has 12 neighbours at a distance


of 2.878 A
MZ
(3) p
= N Aa3

197x4
= 3
6.022x10 23 x(4.070x10-8 )
o -Au atom
= 19.408 = 19.41
glee.
24. The density of NaCl is 2.17 x 103 kg/m 3 and (100) reflection using X-ray of wave-
length A occurs at f) = 6°. Calculate A.

Soln. : p = -MZ-3
NA a
p =
2.170 x 103 kgtm3
z = 4
M =
58.45 x 10-3 kg
By the problem,

4x58.45x10-3
2.170 x 10 3 =
6. 022 x 10 23 x a 3 •
SOLID STATE 105

a3 =
4x58.45x w- 3
2.170 x 6. 022 x !0 23 x l o3
= 1.789 x w-28m3 .
a = 5.634 X l0-10 m.
Thus, "A = 2d 100 sine

= 2 ~sin e (since, in NaCl, d 100 = d200 = ~)


2 2
=a sin 0 = 5.634 X lO-IO sin 6°
= 5.889 x w- 11 m = 58.89 pm = 0.589A.
25. The first order reflection from ( lOO) plane of KC! occurs at 5°30' but for NaCl,
it occurs at 6°, for the rays of same wavelength. The length of NaCl unit cell is 564 pm,
then what is the length of KCJ cell? The densities of NaCl and KC! are 1.99 gm/cc and
2.17 gm/cc respectively. Do these values support the X-ray analysis?
Solo. : "A= 2d 100 sine.
d =_"A_
·· too 2sin0
dtoo (KCI) sine (NaCl) sin 6° 0.104528
---"-=...:.--'- = = = = 1.09058.
dto0 (NaCI) sine (KC!) sin 5° 30' 0.095845

:. aKCI = 1.09058 aNaCI


= 1.09058 X 564 pm= 615.08 X l0-1 2 m = 6.15 A_
. Now, both NaCl & KC! have f.c.c. structures.
58.44xZ 74.45xZ
·· PNaCI = 3 and PKcI = 3
NA.aNaCI NA.aNaCI
3
58.44 I. 99
.. PNaCI a KC!
= - - x -3- =
PKCJ 74.45 a NaCl 2.17
I
aKCI
= (1.99 x 74.45)3 = (1.168282409)1/ = 1.053212357.
3

a NaCl 2.17 58.44


.. aKCI = 1.0532 x 564 = 594.0118 pm = 5.94 A.
Thus these values do not support the X-ray analysis.
26. In a powdered diffraction result a KC! crystal gave lines at the following distances
from central spot, when 70.8 pm X-ray was used in a camera of radius 5.74 cm.: 13.2 mm,
18.42 mm, 22.81 mm, 26.21 mm, 29.42 mm, 32.2 mm, 39.65 mm. Identify the unit cell
and determine the dimension.
t..,2
Solo. : We know, sin 2 8 = --2 (h 2 + k2 + !2) (Sec 4.28 from Vol. 1)
4a
t..,2
=A (h 2 + k 2 + t2 ) [where A= - ] 2
.
4a
In a powdered diffraction, distances of the lines from central spot are related to the

corresponding diffraction angles by () = !!.__, where R is radius of camera.


2R
106 PROBLEMS ON PHYSICAL CHEMISTRY

D
e = -· ~
nl\ = 2 d sme
. n'A.
·. I -
e = sm-
2R' ' . 2d

.. O/degrees =_!_2 ( 57.4mm


D )(
360
2rt
)= .!.D/mm.
2
The following table is drawn :
D/mm 13.2 18.42 22.81 26.21 29.42 32.2 39.65
()/deg 6.6 9.21 11.41 13.11 14.71 16.l 19.82
100 sin 2 8 1.32 2.56 3.91 5.14 6.44 7.69 10.2
32
The common divisor is I. . Divide by common divisor to identify h 2 + k 2 + t2 :
100
2 3 4 5 6 8
The corresponding
indices are : ( 100) (I I 0) (I II) (200) (210) (211) (220)
h2 + k2 + 12 ~ 7 since, all possible fl kl) planes are present, the unit cell is simple
cube.
1..2

4a 2
= - sin2e
- 1.32
- - = --
h 2 +k2 +/ 2 100
= 0. O132 (. 2
smce h +
,
k2
+
12
= I)
2
A.2 = (70. 8)
or, 4a 2 =
0.0132 0.0132
2
7 8
or, a2 ( 0. ) = 94936.36
4x0.0132
a = 308.1 pm.
27. The molar volume of KC! is known to be 1.3 times that of NaCl. If the glancing
angle for first order Bragg reflection from the (100) plane of NaCl is 5.9°, calculate the
same for KC!. [B.U. 1997]
Hints : For NaCl, Z = 4.
3
x 23 x N
. . molar vol. of NaCl = _ix _x_N_A and that of KC! = A
4 4
where x 1 and x 2 are side-lengths of NaCl and KC! respectively.
. xix NA
G1ven, =
4
:. x2 = l.091x 1

For NaCl, d 100 = -5_ (first order reflection).


2

For KC!, d 100 = 1.091 x X1 (first order reflection).


2

:. ~
"NaCl =2 X -5_ s1'n 5.9°
2

an d
~
"KCI =2 x 1.091
- - x 1sm u.
2
. ll

A NaCl = A,KCI' :. (J = 5.40°


SOLID STATE 107
28. CaF2 is f.e.c. lattice with four CaF 2 groups per unit cell. The (Ill) reflection with
X-rays of wavelength 0.1542 nm has 0 = 14.18°, sin 0 = 0.245. Calculate (i) the length
of the unit cell edge and (ii) the density of the crystal. ' [C.U. 1997]

a
Hints : For f.c.c., d 111 = {3.

:. a=-/3d111
Now, A. = 2d sine [first order reflection]

:. d =_'A_= __
0._1_54_2_ = 0.3147 nm.
2 sine 2 x sin 0. 245
(i) :. a= (-/3x 0.3147) run= 5.450 x to-8cm.

MZ
(ii) :. density p = N a3
A

78x 4gm
= 23 3
6. 022 x 10 x ( 5. 45 x 10-8 ) cc

78x4x10
= gm/cc = 3.2005 gm/cc.
6.022x(5.45) 3
29. Ag is known to crystallise in f.c.c. form and the distance between the nearest
neighbour atoms is 2.87 A. Calculate the density of Ag. [At. wt. of Ag = 108]
[B.U. 2000]
Hints : For f.c.c., 4r = -/'Ja.

.. a = Ji
4 ·r = ( Ji
4 x l.435 ) A 0

a = 4.058 x 10-8 cm.

MZ 108x4
.. density p = N a3 = 23 3
A 6. 022 X 10 X ( 4. 058 X 10-8 )

108 x 4 x IO
= gm/cc = 10.735 gm/cc.
6. 022 x ( 4. 058) 3
30. Polonium is the only metal whose lattices are primitive with a = 334·4 pm.
X-rays of wavelength 153.6 pm is now used in powdered diffraction experiment. What will
be its density? If it were a f.c.c. crystal with the same atomic radius, what will be its density?
Soln. : We know that

sinO(hkl) = 2~~(h2+k2+12)
A. = 153.6 pm
MZ
.. density p = NA-;;'3
At. wt. of Polonium = 210 and for simple cube, Z = 1.
108 PROBLEMS ON PHYSICAL CHEMISTRY

210
.·. p = 6.022x10 23 x a 3
Given, a = 334.4 pm = 334.4 x 10- 12 m = 3.344 x 10-8 cm.
210x 1 .
:. p = = 9.326 gm/cc.
23 8 3
6. 022 x 10 x (3. 344 x 10- )

Radius of the atom (r) = ~(for simple cubic)


2
334.4
= pm = 167.2 pm.
2
In f.c.c. lattice, 4r = ..f2a.
4r 4xl67.2
.. a={2= {2 pm = 472.9130 pm = 4.73 x 10-8 cm .

. 4xM 4x210
. . dens1ty = - - 3 = -----------=- 3 = 13.18 gm/cc.
NA'a 6.022x10 x (4. 75x 10-8 )
23

31. a-phase of Fe crystallizes in the cubic system. In a powder pattern experiment,


ratio of sin 2 (}values obtained are l, 2, 3, 4, 5, 6, 7, 8, etc. Which type of cubic unit cell
is present? What is the probable Miller indices for sin 2 8 = 7?
The density of alpha iron is 7 .92 gm/cc. The wavelength of X-ray used is 45 pm. Calculate
the length of side of unit cell. Also calculate the value of (} for Bragg reflection from 100
type planes and radius of iron atom. (atomic weight of iron = 55.85 gm-mole- 1)
Solo. : As h2 + k2 + !2 can never be 7, it must actually be 14(3 2 + 22 + 12 = 14).
So the sin 28 are 2, 4, 6, 8, 10, 12, 14 and 16, etc. The unit cell is b.c.c. Probable Miller
indices are (321) (not necessarily in this order).

55 85 2
or, a3 = MZ = · x = 2.342 x io-23 cm 3 .
NAP (6.022xl0 23 )x7.92
:. a = 2.861 x 10-8cm = 286.1 pm.
a
For (200) type planes, d = - .
2
For b.c.c. lattice, first order diffraction is observed from (200) planes rather than ( 100)
planes.

sin 2 (} = _}{__ (h2 + k2 + [2)


4a 2
2 2
45 x4 45
- 2 = 0.024739408.
= 4x(286.1) 2 = -286.1
or, sin (} = 0.15728766
.. (} = 9.04949731 = 9.05°.
SOLID STATE 109

The system is b.c.c. Thus ,,/3a = 4 r.

r = -
../3a ../3 x 286. l
= - - - - pm= 123.884934 pm= 123.9 pm.
4 4
32. MnO has a cubic unit cell ·with a = 447 pm. The density is 5200 kg.m- 3.
(a) Calculate the number of MnO units per unit cell and determine from this whether
it is simple, b.c.c. or f.c.c. Find out the coordination number of Mn and oxygen.
(b) Assuming the unit to be b.c.c. of side 447 pm, calculate sin ()for reinforcement
of 50.0 pm X-ray reflected from (210) planes. From which of the following three planes
the first order reflection will be more intense and why? Planes : (100), (110), (211). Given
at. wt. of Mn = 55.
Soln. :. (a) p = 5200 kg.m- 3
a = 447 pm = 447 x 10- 12 m
NA = 6.022 x 1023 , X = ?
MZ
p =
N A a3

dN_A_
_ a3
or, z =
M
5200x6.022x10 23 x{447x10- 12 )3 52x 10-8 x6.022x(447) 3
= =
71x10- 3 71

= 3.939 = 4.
. . the unit cube is f.c.c.
Coordination no. of M?,+ = 6 and coordination number of o 2- = 6, since there
are 6 M?,+ ions surrounding each o2- ion and vice versa.
(b) 'A = 2dsin (} (for 1st order reflection)
a2 a2
d}w = h2+k2+t2
=- --
4+1+0
447
d 210 = a pm = 199.91055 m.
.. {.5 = 2.236 p

sin () = A = 50 · Opm = 0.125056.


2d 2x l99.91055pm
() = 7.1839°.
The first order reflection from (211) planes is more intense than the other two planes.
In f.c.c. lattice for other two types of planes, (100) and (110), another plane is interposed
halfway between two successive planes, that destructively interfere with the radiation
scattered from the respective planes, thereby reducing intensity.
33. A certain solid crystallizes in a b.c.c. lattice. The first order X-ray (A= 0.154 nm)
reflection maximum from a set of (200) planes occurs at an angle of 16.6°. Calculate the
dimension of the unit cell. [M.Sc. Admission Test, 1994, 1.1.T. Bombay]
110 PROBLEMS ON PHYSICAL CHEMISTRY

Solo. : We know that n/.., = 2d sine.


Here n = l, A= 0.154 nm, sin()= 0.286.
n'A 0.154nm
So, dzoo = = = 0.269 nm.
2sin0 2 x 0.286
a
dzoo = 2
So, a = 2d200 = 0.538 nm.
34. CsF crystallizes in CsCI type structure and KCI in NaCl type structure. Lattice
length of KCI is 1.5 times that of CsF and molecular weight of CsF is twice that of KCI.
Calculate the ratio of density of KCI to that of CsF.

Solo. : p = -MZ-3
NAa
NaCl has f.c.c. type structure and CsCI has b.c.c. type structure.
Thus, for CsF, Z = I.
_ McsF
PcsF - 3
NAaCsF
4x MKcI
For KCI, Z = 4, dKCI = 3 .
NAaKcl

. Thus, PKCI = 4 .x MKcI x ( acsF J3


PcsF M CsF aKCI
3
1
=4 x ..!..x (--)
2 l. 5
(Given, MesF = 2MKCL• aKCI = I .5acsF)
= 0.59.
35. KCI crystallizes in NaCl type structure. The ionic radius of Na+ is 0.5 times that
of c1- and 0.7 times that of K+. Calculate (a) the ratio of the side of the unit cell of NaCl
to that of KCI. (b) the ratio of density of NaCl to that of KCI.
Solo. : KCI and NaCl have f.c.c. type structure. Thus, 2(r+ + r_) = a = side of unit
cell.
Given, rNa+ = 0.5 rc 1-
rNa+ = 0.7 rK+.

rNa++ rci- rNa + + 2rNa + 3


(a) a NaCl = = = = 0.88.
aKCl rK+ + rci- r Na + + 2rNa + 3.42
0.7

MZ
(b) p = N a3
A

Here, Z =4 (for both cases).


3
Thus, PNaCI = MNaCI X ( aKCI J = 58. 5 X( 3. 42 )
3
= l. l 6.
PKCI MKCI aNaCI 74.5 3
SOLID STATE 111
36. The density of Lithium metal is 0.53 gm.cm- 3 and the separation of the ( 100) planes
of the metal is 350 pm. Determine whether the lattice is f.c.c. or b.c.c.
[M(m) Li = 6.941 gm.moI- 1]
Solo. : Given, d 100 = 350 pm = a.
ZM
Again, p = - - 3
.
NA a
Here, p = 0.53 gm.cm- 3, M = 6.941 gm.mo1- 1
3
0. 53 x 6. 022 x 10 23 x ( 350 x 10- 10 )
z = 6.941
= 1.9 = 2.
Hence, the lattice is b.c.c.
37. Polonium crystallizes in a simple cubic lattice. It has an atomic mass = 209 and
density= 9.4 gm/cm 3. What is the edge length of its unit cell? What are the distances between
two neighbouring (110) and two (I II) planes. What are (i) the crystalline atomic radius
of Po and (ii) per cent void in Po crystal? [V.U. 1997]
Hints : Z = I, M = 209 gm, p = 9.4 gm/cm 3
ZM a a
p = --3
NAa
= 9.4, d110 =
-v2
r;;;, d111 = 'V.J/,;3
(a = 3.32 A, d110 = 2.35 A,
= 1.92 A. r = 1.66 A, 48%)
d111
38. The density of CaO is 3.35 x 10 kg. m- 3. This oxide crystallizes in one of the
3
cubic system with a = 480 pm. Calculate the number of molecules in unit cell. Which type
of cubic system is it? [V.U. 1996]
Hints : p = 3.35 x 10 kg.m- , a = 480 x 10- m, M = 56 x 10-3kg, z = ?
3 3 12

[Ans. Z = 4, f.c.c.}
39. Calculate the wavelength of X-ray which shows a second order Bragg reflection
angle of 14°10' from the (100) plane of KC!. The density of KC! is 1.984 gm.cm- 3 and
there are four atoms in the unit cell. [V.U. 2002]
Hints : 2d 100sin () = n'A
Here, n= 2, d 100 = a, () = 14°10', p = 1.984 g.cm-3, Z = 4
4xM
P = -- 3 [Ans. A = 1.54 AJ
NAa
40. The effective radius of an iron atom is 124 pm. Iron occurs in b.c.c. structure.
Calculate the mass density of iron in SI system.
(Fe= 56, I pm= 10- 12 m, NA= 6.023 x 1023 mot- 1) [V.U. 2003]

Hints : Here, Z = 2, M = 56 x 10-3 kg. r = 124 pm = 124


x 10- 12 m, .f3a = 4r
[Ans. p = 7918.96 kg.m-3]
41. A crystal having simple cubic lattice has the length of its unit cell a0 pm. One
of its planes show a first order Bragg reflection at an angle of 60°. Taking the wavelength
of the X-ray as a0 pm, find Miller indices for the plane.

Hints •• 'A = aO' a = aO' 2d sine = rv,~ d = ~ ao ' [Ans.'(! I I) planes]


-yh2+k2+!2
112 PROBLEMS ON PHYSICAL CHEMISTRY . ·

42. Is it possibfe to obtain Bragg reflection from (I 00) planes of simple cubic crystals
with edge 1.5 A by using X-ray with wavelength of 3.5 A.. Explain. [B.U. 1981]
Soln. : For simple cubic crystal, d 100 = a = 1.5 A.
Now, 2d IOO sine = A.
For maximum possible value of 'A, sin e = I,
'A =2 x 1.5 = 3.0 A.
Hence, X-ray with wavelength of 3.5 A cannot be used.
43. X-ray powder diffraction angles of silver crystal are obtained at 19.11°, 22.18°,
32.34°, 38.84° and 40.89°. (a) Find the type of unit cell. (b) What is the radius of Ag atom,
if 'A of X-ray is 154.43 pm.
Soln. : e 19.11° 22.18° 32.34° 38.84° 40.89°
sin 2 e 0.11 0.14 0.28 0.39 0.42
Dividing by common
divisor 0.11 1.3 2.5 3.5 3.8
Multiplying by 3 3 4 8 II 12
h2 + k 2 + 12 3 4 8 11 12
The corresponding
indices are (111) (200) (220) (311) (222)
Hence, the unit cell is f.c.c.
Now, 2dsin e = A.
A.2 A.2
sin 2 e = --
4d 2
= --(h
4a 2 ·
2 + k 2 + ! 2 ).

~54 .42
32
Hence, 0.11 = (I + l + I) !Given, 'A= 154.43 pm]
4a
a2 = 162604.26
.. a = 403.24 pm.
Now, {2.a = 4r.
{2
Thus, radius of Ag atom = r = x 403.24 m = 142.56 pm = 1.42 A.
4
· 44. The first six observed Bragg diffraction angles from a powdered sample of iridium
crystal are 21.97°, 25.60°, 37.66°, 45.75°, 48.43°, 59.75°. Determine the type of unit cell
and assign corresponding Miller indices for every observed angle.
Soln. : e 21.97° 25.60° 37.66° 45.75° 48.43° 59.75°
sin 2 e 0.14 0.19 0.37 0.51 0.56 0.75
Dividing by common 1.36 2.64 3.64 4.00 5.36
divisor 0.14 multiplying
by 3 3 4 8 11 12 16
h2 + k2 + [2 3 4 8 11 12 16
Corresponding Miller
indices are (111) (200) (220) (311) (222) (400)
The unit cell is f.c.c.
SOLID STATE 113

45. X-ray power diffraction angles from the Mo crystal are at 20.26°, 29.30°, 36.82°,
43.82°, 50.70°, 58.80°, 66.30°. Find the type of unit cell and also corresponding Miller
indices for the planes responsible for the above diffraction angles.

Soln. : () 20.26° 29.30° 36.82° 43.82° 50.70° 58.80° 66.30°


sin 2 e 0.12 0.24 0.36 0.48 0.60 0.73 0.84
Dividing by 0.12 and 2 3 4 5 6 7
multiplying by 2, 2 4 6 8 10 12 14
since 7 cannot be
sum of squares. 2 4 6 8 10 12 14
h2 + k2 + t2
Corresponding Miller
indices are (110) (200) (211) (220) (310) (222) (321)
The unit celll is b.c.c.
46. Calculate the distance between successive layers of graphite if carbon-carbon
distance in the molecular layers is 1.43 A. Its density is 2.23 gm/cc.
Soln. : Graphite has a layer structure with each layer consisting of hexagonal network
of carbon atoms. Three layers of graphite constitute a unit cell.
Let carbon-carbon distance in the molecular layers be a cm and distance between
successive layers be d cm.

c Fig. 2 : A layer in graphite

Fig. 1 : Unit Cell of graphite


Solid lines are the boundaries of the
graphite unit cell that intersects the bottom
layers.
Thus, if we draw the quadrilateral con-
stituting the bottom face of the unit cell a
reducing the carbon atoms to points
A

Prob. Phy. Chem.-8


114 PROBLEMS ON PHYSICAL CHEMISTRY

DB= -f3a cm
c
AO= -f3a cos 30° = ~a cm.
2
3
Area of MDB = J_DB x AO = .f3- a 2 cm2
2 4
Thus, area of the quadrilateral (rhombus) ABCD D
= area of the top face or bottom face of the unit cell

=2 x 3-!3 a 2 = 3 -!3 a 2 cm 2
4 2
Thus, volume of the unit cell,

3
V = -!3 a 2 x 2d cm 3 = 3{3 a2d cm 3
2
The unit cell has 8 atoms at comers, 2 atoms at the centres of top and bottom faces,
4 atoms at the middle of vertical edges and 1 atom at the centre of the unit cell.
Thus, number of atoms in each unit .cell

= Z = -1 x 8+1x2 +-1 x 4 +Ix I = 4.


8 4
Given, density of graphite = 2.23 gm/cc
and a = 1.43 A= 1.43 x lo-8 cm
Atomic weight of carbon = M = 12 gm

Thus ZM = 2.23
NAV

4X 12
or,
23 2 = 2.23
6.022 x !0 x 3{3 x (1.43x 10-8 ) x d

:. d = 3.36
x 10-8 cm = 3.36 A.
47. Fe below 912 °C has f.c.c. structure and density of 7 .86 gm-cm- 3. If its atomic
mass is 55.85 g, find its atomic radius.

. d .
A ns. H mts : ens1ty, p = - MZ
3-
a NA
a ,;, 2.86 x 10-8 cm

4r = -f3a
r = 1.24 x 10-8 cm = 1.24 A
48. An element has cubic unit cell. if its molar volume is 9.44 cm 3-mol- 1 and a =
3. l 5A, determine its crystal structure.

. a3
Ans. Hmts : zNA = 9.44,
Z = 2, So the structure is b.c.c.
CHAPTER 5

FIRST LAW OF THERMODYNAMICS, CARNOT CYCLE,


CLAUSIUS-CLAPEYRON EQUATION AND THERMOCHEMISTRY

Convention Adopted
1. W = Work done upon a system (+ )ve
= Work done by a system (-)ve
[Reverse of these is the old convention.]
2. Q = Heat absorbed by a system (+ )ve
= Heat evolved by a system (-)ve
Required Formulae
1. dW and dQ are inexact differential.
dU and dH are ex.act differential.
2

fdH = H2 - H 1 = llH.
I

fdU = U 2 - U1 = t:i,,U.
I

But fd-W = W, fdQ =Q.

2. Formula of mechanical work


dW = - PexdV (old convention dW = Pex dV)
Pex =external pressure on the system.Wis (-)ve, when dV is (+)ve (expansion) and
(+)ve, when dV is (-)ve (contraction).
2

W = - JPexdV can be integrated.


I

If Pex remains constant throughout the process,


W = - P(V2 - V 1) = - P/:i,,V, (Pex = P) = P (V1 - V2 ).

115
116 PROBLEMS ON PHYSICAL CHEMISTRY

If Pex remains infinitesimally close to ?system throughout the process (i.e., reversible

process), - JPdV can be integrated if P is known as a function of V.

Reversible Isothermal change :


2 2

Ideal gas : W = -J Pdv = -JnRTdVv = -nRTln Vv 2


= nRTln-1 •
v
I I
1 Vz

van der Waals gas, W


f(...!!.!!!_ -
2
1
V-nb
an,2 )dv
v-

= nRT In V2 -nb - an 2 ( - I - -1 ) .
V1 -nb V2 V1

= nRT In Vi -nb + an2(_!_ __


I )·
V2 -nb V1 V2

3. For solid or liquid, a= J_(av)


V dT
, P

where a = coefficient of thermal expansion,

and p= _J_(av)
V dP T

where p = isothermal compressibility.


V =f(P, 7)

dV =(av)
dT
dr+(av) dP
p dP T

=av dT- pv dP
2

W = -f PdV = - f P(aVdT-PVdP) = p: (Pi-Pf} (dT = 0).


I

I
K = Bulk modulus =
p

5. According to first law of thermodynamics, for a system at rest in absence of external


field,
dU = dQ+dW.
For the system doing only mechanical work,
dU = dQ - Pex dV
FIRST LAW OF THERMODYNAMICS 117

i!Q = dU + Pex dV.


If the system has external Kinetic energy (T) and is subject to a conservative external
force (e.g., gravitational force), then
dE = i!Q + dW,
where E = T + V + U
and V = potential energy due to external field.

6. nCp =(dQ]
dT
=(aH)
aT p
,where H=enthalpy.
p

nCv=(dQ]
dT
=(au), where
aT v
U=internal energy
p

7. For ideal gas,

(~~)T =(~~)T =(~;l =(~~)T = O.


Cp - Cv = R.
8. For solid, liquid, real gas, i.e., for any system,

C - C - azTVm
p v- ~

where Vm = molar volume.


9. For exact differential dz,
fdz =O
z = f(x,y).
If dz = Mdx + Ndy,
then (aM)
ay
= (aN) .
ax
x y

10. Adiabatic work :


W = nCv (T2 -
T1), if Cv is constant and the system is an ideal gas.
For adiabatic reversible change in an ideal gas,
PvY = constant.
Tvr-- 1 = constant.
T
-R- = constant.
pCp

11. For free expansion, W = 0, since Pex = 0.


12. Considering source of gravitational force part of the system for free fall of a body
from a height h, W = 0. '°
118 PROBLEMS ON PHYSICAL CHEMISTRY

13. For ideal monatomic gas : Cv = ~ R.


2

For ideal diatomic gas : Cv = ~ R.


2
14. Generally we assume, U =f (T, V), H =f (T, P)

du= (au)
av
+(au)
r
dv
aT v
dT

dH = (aH)
(JP T
dP+(aH) dT
(JT p

For ideal gas,

dU = (au)
aT v
dT = nCvdT.

dH = ((JH) dT = nCPdT.
CJT p

For adiabatic work against constant Pext for ideal gas,


dU = dW
If Cv is constant,
nCv dT = - Pext dV
or, nCv (72 - Ti) =-Pex(V2 - Vi)

_ p (nRT2 nRT1 )
or, n C v (.TI - T)
2 - ext -p- -- - p ·
2 I

Thus, -3 R (Ti - T2) = R ( T2 - Po) •


T1 - ·
2 ~

.. ~ (T 1 - T2) = ( T 2 - T1 ;~).
Short Questions
I. State with reasons whether the following statements are correct or incorrect :
(i) Every isolated system is closed.
Ans. : For closed system, mass is not exchanged with surroundings. For isolated system
neither mass nor heat is exchanged with surroundings. So the statement is correct.
(ii) Every closed system is isolated.
Ans. : The statement is incorrect. Closed system exchanges heat with surroundings
but isolate0 system neither exchanges heat nor mass with the surroundings.
(iii) Thermodynamics is dependent in the microscopic properties of matter such as
work, heat.
FIRST LAW OF THERMODYNAMICS I 19
---~~~~~~~--~~~~~~~~~~~~~~·

Ans. : In thermodynamics, states are defined by a number of microscopic properties.


w and q are not properties of states-these are dependent on path of change. So the statement
ts incorrect.
(iv) !},.H is a state function.
Ans. : A state function refers to a state. Although H is a state function, !},.H cannot
be a state function since it refers to the difference in H between two states (which, however,
is path independent). Thus, the above statement is not correct.
(v) Cv is independent of temperature for a perfect gas.

An~ : C,, the mol.r heat capocity, i' defined" [ ~~ t whe<e dQ = heot change

for one mole and dQ= dU + Pex 1 dV for a system performing mechanical work only. So

at constant V, dQ = dU. Hence, Cv = (dU)


. Now, for a perfect gas, U dpends only
dT v
on temperature. Therefore, Cv will depend only on temperature for a perfect gas. Hence,
the above statement is false.
(vi) A process in which the final temperature is equal to initial temperature must be
an isothermal process--comment.
Ans. : It is no doubt that in an isothermal process, temperature is constant and so
initial and final temperatures are same. But for an adiabatic free expansion of an ideal gas
the final temperature is equal to the initial temperature. So, the above statement is not correct.
(vii) U remains constant in every isothermal process in a closed system.
Ans. : The above statement is true only for an ideal gas. Because only for an ideal
gas U =f(D. But, for real materials, U depends on temperature and volume. Thus, U cannot
remain constant for any material in an isothermal process. So, the above statement is not
correct.
(viii) Q =0 for every cyclic process.

Ans. : We know that Q = L1U + W. For a cyclic process, f du = 0 [since, U is a


state function].
But, W may not be zero because it depends on path and, therefore, Q may not be
zero. Thus, the statement is not true.
(ix) !},.U = 0 for every cyclic process.

Ans. : U is a state function and, therefore, fdU = 0. Thus, !},.U = 0 for a cyclic process
and the statement is correct.
(x) When a system expands, work is done on the system.
Ans. : If the external pressure is less than internal pressure, then the system expands.
Thus, work is done by the system. So, the statement is false.
(xi) If a closed system at rest in the absence of external fields undergoes an adiabatic
process that has W = 0, then the temperature of the system must remain constant.

Ans. : We know that for an adiabatic process, dW = -dU.


120 PROBLEMS ON PHYSICAL CHEMISTRY

Now, u = j(V, T).

dU = (au)
av
+ (au)
ar T
dV
v
dT.

aw= - (au) (au) dV - dT.


av ar T v

If W =0, 0=-f f(a_!!_ )


2
nCvdT-
2

av T
dV.
I I

If Cv is constant, then

J(au)
av T
dV
I

i.e., 0 =

Thus, if (au)
av r
-;e 0, T1 is not necessarily equal to T2 and for an ideal gas, (au)
av
=
T
0.

The above statement is thus true for an ideal gas and is incorrect for any other material.
(xii) For a closed system doing mechanical work only, a constant pressure process
that has Q > 0 must have t:i.T > 0.

Ans. : dQ = dU + Pext dV
If Cv is constant, and U = f (T, V), then

.. dQ = nCvdT +[(~~ldV+Pexr.dV]
...

or, dQ = nCvdT + [( ~~l +Pexr ]dv.

:. Q = nCv (T2 - T 1) + J[(~~l +Pexr]dV.


Now, if Q > 0, t:i.T m!ly be zero, less than zero and greater than zero, because it depends
on the second term in the above equation.
(xiii) P - V work is usually negligible for solids and liquids.

Ans.: For any system, dW = PexrdV. For solid and liquid, Pexr has a very negligible
effect and coefficient of compression (/3) is around IO--{i atm- 1• Thus, P - V work is usually
negligible for solids and liquids. The above statement is correct.
2. Show that if Z = PV, then Z is a state function.
Ans. : Z = PV
Thus, dZ = PdV + VdP.
FIRST LAW OF THERMODYNAMICS 121

Now, (~~)v =

( av) =
av p 1
Therefore, f dZ = 0, so, Z is a state function.

3. A gas obeys the equation of state P(V - b) = RT, where b is a constant.


For the gas,
(a) plot V vs. T at constant P.
(b) plot P vs. V at constant T.
(c) plot P vs. T at constant V.
Ans. : (a) Plot of V vs. T at constant P
The equation is

':.<;)~ (V - b) = RT
,~o
p
v Slope =R R
b p V = -.T
p
+ b.
Intercept = b

T .
(b) Plot of P vs. v (at constant n :
The equation is
I
I con st.
b~I
P=
I V-b
p
.. p oc -1-
isotherm V-b
V~ooasP~O

V~basP~oo

(-b~ v
(c) Plot of P vs. T at constant V

Sl ope = -R- .The equation


. .
1s P = RT
V- b const.
p Intercept = 0.

4. Find
(-aH)
av r
.
for van der Waals and ideal gas.
122 PROBLEMS ON PHYSICAL CHEMISTRY

Solo. : H =U + PV
dH = dU + PdV + VdP

(~~l ~(~~l +P+V(~~)T


= T(aP)
aT v
+v(aP)av T
,since (au)r
av
=T(aP) -P.
aT v

nRT an 2
For Viln der Waals gas, P =- -- V2 .
V-nb

v(aP) = v( nRT + 2an


2
)= _ nRTV + 2an
2

av T (V-nb) 2 V3 (V-nb) 2 V2 ·

Hence, ( -aH) - - nRT


av T - V-nb
(i -~
V )+
V-nb V2 -
2
2an- -- n RTb +
(V-nb) 2
2n -
a
V2 •
2 2

For ideal gas, P = nRT


v
T(aP) = nRT = P, v(aP) = - nRT = -P
aT v V av T v

Hence, (aH)
av r
= 0.

5. Define heat capacity C1, and calculate from the I st principle the numerical value
of Cv of a Cu penny in calorie/°C using your best physical knowledge.
[PhD Entrance Examination, University of California, Berklay, USA]
Ans. : The amount of heat needed to raise the temperature of an object through l °C

is dcfinod" its heat capacity. It is sufficient to Mite C, o[ ~ l- The atomic wt. of Cu

is 63.54 gm, i.e., roughy 64 gm.mol- 1. A Cu penny is about 32 gm. A solid consisting
of N atoms has U = 3NkT. It comes out from the fact that in a solid only vibrational degrees
of freedom are possible. From the equipartition principle we know that energy per vibrational
degree of freedom is kT. For a system of N particles the vibrational degrees of freedom

are 3N. Thus, total energy is 3NkT. If there are n moles of particles n = !!-..., where NA
·NA
is AvQgadro number. Thus,

U = 3NkT = 3N _!!__ T = 3nRT.


NA
:. Cv = 3nR.
By the problem, WCu = 32gm.
Thus, ncu = 0.5 mol
FIRST LAW OF THERMODYNAMICS 123

Hence, Cv = 3 x 0.5 mol x 1.987 cal.K- 1.mo1-1


= 2.9805 cal.K- 1= 2.9805 calorie/°C
6. An insulated chamber is divided into two halves of same volume; the right-half
is evacuated and left-half contains an ideal gas at temperature T. Now a small hole is opened
between the two halves. Thus the gas will flow from left-half to right-half and the system
ultimately comes to equilibrium. If no heat is exchanged with the walls, find the final
temperature of the system. [PhD Entrance Examination, Columbia University, USA]
Solo. : The gas flows continuously from left to right into vacuum. Since opposing
pressure (Pext) is zero, W = 0. Q is also zero. Since by the first law of thermodynamics,
fi.U = Q + W, fi.U = 0. For an ideal gas, internal energy ( U) is a function of temperature
only. Thus, the temperature will not change. It will remain the same after the expansion
against vacuum as before.
7. What will happen if in the above case the gas is real?
Ans. : For a real gas,
U = j{T, V) .

.. dU = (au) dT+(au) dV = Cv dT + (au) dV.


ar v av r av T

:. ti.U = f CvdT+ f(aa~ )dV.


2

I
2

1
But again, W = 0, Q = 0, :. ti.U = 0.
2 • . 2

Thus, JCvdT = - J(au)


av r
dV.
1 1

In real gas, U generally increases as volume increases. So (au) is +ve.


' av T

Thus, J CvdT must be -ve.


1
:. C.y(_T2 ~ T 1) = Cv<T2 - T) ( ·: T 1 = T)
= -ve
i.e. T > T2 .
Hence, temperature will fall, i.e., decrease.
8. Two systems with heat capacities C 1 and C2 respectively interact thermally and
the temperature becomes TF. The initial temperature of system- I is T 1.What was the initial
temperature of the system-2? The total energy of the system is constant.
[PhD Entrance Examination, Wisconsin University, USA]
Solo. : Let the initial temperature of the system-2 be T2 , (T2 > T). Then by the problem,
C1(TF - T1) = C2(1'2 - TF)
= C1TF - C 1T1 + C2Tp
or, C2 T2

.. T2 =.s_(TF-T1 )+ Tt.
C2
9. A solid object has a density d 1• Show that the pressure P and the heat capacities
3aMP
Cp and C v are re 1ate d by C p - C v = ---, where M 1s . mass o f the o b'Ject and a 1s
.
di
the coefficient of linear expansion. [PhD Entrance Examination, Wisconsin University, USA]
124 PROBLEMS ON PHYSICAL CHEMISTRY

Soln. : By the first law of thermodynamics, dU =d"Q+d-W .

. . d"Q= dU - d"W = dU + PdV (since, dW = -PdV).

For a solid there is almost no difference between (au)


aT p
and (au)
dT v.

:. Cp - Cy= P(av) .
aT p

We know that a =_!_(av)


v ar
[·:(av)
ar p p
=av for one dimension]

Thus, Cp - Cy= PV. ~(~~t= PVa.


For, three dimensions, (av)
aT P
= 3a V.
M M
:. Cp - Cy= 3a-P (where V =-).
d1 d1
10. An ideal gas has been kept in a large jar of volume V0 . A metal ball of mass
M which is fixed snugly in the glass tube of cross-sectional area A is fitted to the jar. The
equilibrium pressure in the jar is slightly higher than atmospheric pressure because of ball's
weight. If the ball is displaced slightly from equilibrium it will undergo S.H.M. If the gas
state represents a quasistatic adiabatic process, find the relation between oscillation frequency
and the variable of the problem. [PhD Entance Examination, University of California, Berkeley]
Soln. : Let the pressure in the jar be P. As the process is adiabatic PvY == constant,

where V is the volume and y is the ratio of specific heat


(~:).
.. dP +ydV == O.
p v
.. dP == _ yPdV.
v
Now F = A.dP
::::::::::::::::::::::
[where A is the area, dP is the change in pressure J
But a system undergoes S.H.M. So, F = -k./
:::::::::::::::::::::·
[where I = length =dV ,i.e., change of volume per area]
A
:. AdP =-kl.

:. A. (-y P~V) = -kl

=> AyP dV = k dV
V A
FIRST LAW OF THERMODYNAMICS 125

A"{P k
~ --=-
v A
A2"fp
.. k= - -
v
where P is the pressure in the jar and higher than atmospheric pressure.
mg
P =Po+ A
Now oscillation frequency v -
-
~
27t
-
1
- - (k [k
- 27t ~-;;;
= 47t2m v2]

l ~ --
2
=- A "{P [where co =1, h"f =hcco].
27t mV c

11. l mole of a gas with molar internal energy given by U = CT - 5-., where V is
v
molar volume. and a and care constant. Calculate Cp and Cv. The gas obeys van der Waals
equation. [PhD Entance Examination, Wisconson University]

(au)
Soln. : We know that Cv =
aT v
.
cP =(aH)
aT
=(au)
aT
+ P(av)
p aT p P.

:. (~~t= =(~~t +(~~l·(~~)p·


Now, U f(T, V)

or, dU = (au)
aT v av
dT+(au) dV
T

cP = Cv +P(av)
aT
+(au) (av) = Cv + [P+(au) ](av)
av aTp T av aT p T p

= Cv +(P+ :2 )(~~)P·
By van der Waals equation,

(P+-;.)
v
RT [U =CT-5-., (au) =-;.]
=V-b v av v T

.. cP = Cv + _!!!..__(av)
V-b aT p

But from van der Waals equation, ( P + : 2) (V - b) =RT. Expanding and negleeting
V2
ab . comparston
m . wit. h other terms and rep1acmg
. Vm
. the denommator
. by RT th e ideal
. p (
ab
gas value), PV - Pb + = RT
RT
126 PROBLEMS ON PHYSICAL CHEMISTRY

PV = RT + P ( ~ - :T). (I)

.. V R: + ( b- R~).
= (2)

:. (av)
oT p = P
R
+ RT 2
a
.

Again, from equation (I),


(V-b) R a
-T-= P- RT 2 .

R V-b a
.. P =-y:-+ RT 2
Now Cp =Cy+ -RT
- x (V-b
- - + -2a-2 ) .
V-b T RT
2a
:. Cp =Cy+ R + (V ... b)T

~ Cp = Cy+ R + l:!!_ [since, V >> b]


VT
2aP
~ Cp =Cy+ R +--
2
RT
Numerical Problems
1. A mass of mercury at a pressure of I atm and a temperature of 0 °C is kept at
a constant volume and the temperature is raised° to I 0 °C. Calculate the final pressure.
Given, a = I ·81 x 10-4 deg- 1; f3 = 3·87 x 10-6 atm- 1.
Solo. : Given, a = I ·81 x I0-4K- 1
f3 = 3·87 x I0-6 atm-1
0
We know that (
oT v
P) l3Cl atm.K-I .
(l
:. dP = r;:dT.
f' '.

(l .
.. P2 = P 1 + 13·I0 (Given, T2 = IO °C, T1 = O °C)
8 P = l atm )
=Pl+ 1.. lxI0-4 x _ ·IO atm.K-. 1K
=(I+ 46 7.700 26 ) atm (given,
.
3 87 10 6 1

= 468.70026 atm 468.7 atm. =


2. A metal whose volume expansivity is 5.0 x 10-5 deg- 1 and isothermal compressibility
is 1.2 x l o-6 at_m- 1 is at a pressure of I atm and a temperature of 20 °C. A thick surrounding
cover of invar of negligible expansibility and compressibility is fitted to the metal very
m~~ •
FIRST LAW OF THERMODYNAMICS 127

(a) What will be the final pressure if the temperature is raised to 32 °C?
(b) If the surrounding cover can withstand a pressure of 120 atm, then what .is the
highest temperature to which the system can be raised?
Solo. : (a) Proceeding as in Problem I,

5x10-5
P2 = P 1 + x 12
I. 20x10-6
= 501 atm (given, P 1 = I atm)
(b) P 2 = 120 atm

5x10-5
.. ~p = P2 - Pi= 119 = l.20xl0-6 ·tJ.T

2
ti.T = 119 x I. x l0-6 = 2.85 °C
·· sx10-5
. . T2 = 20 + 2.85 = 22.85 °C.
3. A block of the same metal as in Problem 2 occupies 5 litres. As a result, temperature
is raised to 32 °C, the volume is increased by 0.5 cc. Calculate the final pressure.
Solo. : P =f(T, V)

dP = (ap) v
aT
dT + (aP) r av
dV

2
:. ti.P= ati.T
p
+f(aP) r av
dV
. I

Given, V2 = 5.0005
l, P 1 = I atm,
V 1 = 5 1, ti.T = T 2 - T 1 = (32 - 20) °C = 12 °C,
a = 5 x 10-5 deg- 1, p = 1.2 x 10---0 atm- 1.

p2 - p1 = ti.P = a .ti.T -
13
I V2 [ smce
[3Inv;- . A
I-'= -VI (av)
aP T ]

= 5x10- x
5
12
_ 10--{i ln5.0005
I. 2 x 10-6 I. 2 5
= 500 - 83.329 = 416.67 atm.
P2 = 416.67 atm + I atm = 417.67 atm.
4. A block of copper at a pressure of I atm and 0 °C is kept at constant volume. What
will be the final pressure if the temperature is raised by 10 °C? What will be the final
pressure if the volume changes from I 000 cc to I 000.1 cc ?
Given, a = 4.8 x 10-sdeg-l
Bulk modulus = 1.3 x 10 12 dyne.cm-2
l.3x10 12
Solo. : K = atm = 1.2830002 x 106 atm
1.01325x10 6

p = _!_ = 7.79423. x 10-7 atm.


K
128 PROBLEMS ON PHYSICAL CHEMISTRY

Proceeding as Problem and Problem 2,


a
P2 = P1 + px!O

= 616.840 atm (given, P 1 =I atm, a = 4.8 x I0-5 deg- 1)


If V changes from I 000 cc to I 000.1 cc,
7
IO In 1000.1
7. 79423 1000
= 615.8402 atm - 128.29362 atm = 487.54656 atm = 487.5 atm.
:. final pressure = I atm + 487.5 atm = 488.5 atm.
5. 2 mol of an ideal gas undergo isothermal and reversible expansion from V1 to
IO V1 litre. The initial pressure is I 00 atm. Calculate the initial volume and temperature,
i~ the work done by the system is 40000 cal.
Solo. : Given, n =2 moles, V2 = 10V1, P 1 = 100 atm.
W = - 40000 cal, T 1 = T2

The work done, W = -nRT In IOVi = -2RTlnIO.


V1
.. -40.000 cal = -2 mol x 1.987 cal.K- 1.mol- 1 x T In 10.
40.000
T = 2 x 1. 987 x In 10
K = 4371.35865 K = 4371.36 K.

PV = nRT = 2 RT
2RT 2 mol x 0.08206/-atm.K- 1.mol- 1 x4371.36K
Thus,
P1 IOOatm
= 7.174276032 litres = 7.17 litres
T2 = T 1 = 437.36 K.
6. 2 mol of an ideal gas undergo isothermal and reversible expansion at 1096 K from
2 litres to 20 litres. -Calculate the work done.

Hints : W = -nRT In V2 . Ans. -41.96 kl.


V1
7. 2 mol of Nitrogen at STP is compressed reversibly to 10 litres. Calculate the work
done upon the system if (a) the work is carried out isothermally and (b) adiabatically.
1 1
2 x 0.08206/-atm.K- .mol- x273.15 K
- ------ - - - - - - - = 44.829
I atm
litres

10
(a) W1 = - 2 x R x T In - -
44. 829
IO
=- 2 x 8.314 J.K- 1.mol- 1 x 273.15 K x I n - - = 6.814 kJ.
44.121
FIRST LAW OF THERMODYNAMICS 129

= 273.15
Ans. : 9.34 kJ
( ;~
44 29
r
4
-) = 497.77 °K.

8. Calculate the rise in temperature of a piece of iron (mass 10 kg) at 25 °C if it is


dropped from a height of 100 metres. What is the K.E. of this piece just before it hits
the ground? What is its speed? Let the gravitational acceleration constant (.g to be
9.80 m/sec 2 and let the moler heat capacity of iron be CP = 25.12 J.mol- 1.K- 1.
Soln. : Just before the mass hits the ground, all of the potential energy will be converted
into kinetic energy. So,
1:!.U = mgh = 10 kg x 9.80 m.s- 2 x 100 m
= 9800 kg.m 2 .s-2 = 9800 J = 9.80 kJ.
I
.. kinetic energy is thus 9.800 kJ. This is given by mv 2 ,
2

. I . {2K.E.
where v ts ve oc1ty; v = ~--;;-
= 8-x 9800J
V- IOkg

= ffx 980 m.s- 1 = 44.27188724 m.s- 1


= 44.3 m.s- 1.
For solids, cp = CV .
.. 1:!.U = nCP l:!.T .
.. l:!.T = !!.U.
nCP
Given, CP = 25.12 J.mol- 1.K- 1. M = 55.85 x 10-3 kg.mo1-1.

Hence, !!.T = 9800Jx0.05585 kg.mo1- 1


10 kg x 25.12J.mo1- 1.K- 1
= 2.178861465 K = 2.18 K.
.. the final temperature is 27.18 °C.
9. Calculate the rise in temperature of a piece of copper (C = 6.4 cal.deg-1.mol-I)
when dropped from the top of Qutab Minar (height 270 feet) assu~ing that no heat is lost.
Hints : Since, W = 0, Q = !!.U = mgh = nCv(T2-T 1). [Ans. 1.92 °C]
10. Water is brought to boil under a pressure of I atm. When an electric current of
0.6 ampere from a JO V source is passed for 5 minutes through a resistance in contact with
it, 0.8 gm is vaporised. Calculate !!.U, !!.H and W.
Soln. : Given, I= 0.6 A, V = JO volt, t = 5 min, m = 0.8 gm, T = 100 °C = 373.15 °K
Heat Q= I ampere x V volt x t sec
= 0.6 ampere x I 0 volt x 5 x 60 sec
= 1800 J = 1.8 k.J,

Prob. Phy. Chem.-9


130 PROBLEMS ON PHYSICAL CHEMISTRY

Work done, w = -P!lV = -P(Vg - V1), (Vg » V1).


:. W = -PVg = n(mol) R(J.K- 1.mol- 1)T (K)
= -0.0444 x 8.314 x 373.15 J = -137.8 J.
!lU = (1800 - 137.8) J = l662.2 J = 1.67 kJ.
The heat change has taken place at constant pressure.
:. Qp = !lH = 1.8 kJ,
11. 200 gm of CaC0 3 was heated to 727 °C when it decomposed. The operation was
carried out in a container closed by a piston, which was originally resting on the sample
and was retained throughout by the atmosphere. (a) How much work was done? (b) What
would be the work in an open vessel? (c) What would be the work in a closed vessel?
Hints : (a) 200 gm CaC03 = 2 moles
2CaC0 3 = 2Ca0(s) + 2C0 2(g)
T = 727 °C = 1000 K.
C0 2 occupies a volume of 2 x 22.414 = 44.828.
1x V2 = 1x44.828
1000 273

Then the volume at 1000 K and 1 atm, V2 = 44.828X 1000 litre.


273
Work done against constant pressure of 1 atm,
x 1000 I. atm x IO I.
W = -P!lV = -PVg = 1x 44. 828273 325
_I_
1.atm
= -16638.08462 J = -16.64 kJ,
(b) -16.64 kJ (c) 0.
12. 1 mole of a van der Waals gas at 300 K expands isothermally and reversibly from

20 I to 60 I (a = 0.556 m6 .Pa .mole-2 and b = 0.064 dm 3.mole- 1). For van der Waals (au)
av T

= Va2 . Calculate W, Q, !lU and !l.H.

~2
2
Soln. : van der Waals equation of gas is ( P + } V - nb) = nRT.

nRT an 2
So, P = v-..::,;--v2·

v -b
= -1 molx8.314 J.K- 1.mol- 1300K In - 2 - -
V1 -b

3
1000
lmol 2 x 0.556m6 .Pa.mol-2 ( - 1000 )- m-3
- - --
60 20
FIRST LAW OF THERMODYNAMICS 131
·-~-----------------

59 9361
= - 2494.2J In · + 18.533 m3.Pa
19.9361
=- 2494.2 J x In 3.006420 + 18.533 m3 .Pa
=- (2745.4911 - 18.533)J
= - 2726.96 J.
.. wiso, rev = - 2.727 k.J,mol- 1.
2
= -va2 dV = - a = 18.533 J.mol-1
a --
!:i.U
I
f V1 V2

Q = !:i.U - W = 18.533 J + 2.727 kJ


= 18.533 J + 2727 J = 2745.5 J = 2.746 kJ
Aff = !:i.U + (P2 V2 - P1V1)

= 18.533 J + [( ~~·~~ - : 2
)-( ~~~ - :I)] J

= 18.533J + ·[(RT.Vz
V - b
- RTV1
V1 - b
)+(.!:..-~)]
V V
J
2 1 2

= 18.533 J + 8.314 x 300(~-~) J + 0.556(__!_ _ __!_)x1000 J


59. 936 19. 936 20 60
= (18.533 J - 2494.2 x 2.142 x 10-3 + 18.5333) J
= (18.5333 - 5.3426 + 18.5333) J = 31.72 J.
Aff can also be calculated in the following way :

(~~)T = T (~~)v +(~~l·v ·


For van der Waals gas,

oH) RTb 2a
( avT=-(V-:-b) 2 + -v2 ·

1 1 1
Hence, Aff = RTb ( -- - -- ) + 2a(- _ __!_)·
V2 - b V2 - b V2 V1

Putting the corresponding values same result will be obtained.


13. 16 gm of 0 2 at 27 °C and 10 atm undergoes reversible adiabatic expansion to I
atm. Calculate the final temperature, work done, !:i.U and !:i.H. If the same expansion takes
place against a constant pressure of I atm, what will be the final temperature, W, !:i. V and
Aff?
If the same expansion takes place against a constant pressure of I atm but till the
volume is trebled (then pressure is raised to match the internal pressure), what will be the
final temperature, W, !:i.U and Aff?
132 PROBLEMS ON PHYSICAL CHEMISTRY

Soln. : (a) Pz = Ti(;~ yp. For diatomic gas, CV = %R; cp = ~R


By the probem. T1 = 300 K P 2 = 11 atm

1
P1 = 10 atm n = - mol
2
2 2

300(;~ J = 300 x c~r=


7
.. T2 = 155.38424K = 155.4 K.

.!_ mol x 0.08206 /.atm.K- 1 .mo1- 1 x300K


~2_ _ _ _ _ _ _ _ _ _ _ _ = 1.23091.
IOatm

W = nC1, (T2 - T 1) = __ .!_ x ~R(300 - 155.38) = -1.502 kJ.


2 2
/',.U = -1.502 kJ; /',.H = nCP(T2 - T 1) = -2104.585 J = -2.!04 kJ.
(b) Against a constant pressure of I atm
pex CV:~ - V1) = nCi, (Tl - Tz)

nRT nRT )
or, P 2 - -2 - - -1
( P = nCv (T1 - T2)
P 2 1

or, T2 - 30 = 750 -
5
2
_r 2

7 2
-T2 = 780, or, T, = - x 780 = 221.86 K = 51.15°C.
2 ~ 7
. . W = nC/T2 - T1) = -812.06 J.
/',.U = -812.06 J
/',.H = nCp<T2 - T 1) = -1136.891 = -1.14 kJ.
(c) pex (V2 - V1) nCv (Tl - T2) =
5 5
or, Pex (3V1 - V2) =n x 2.R (300 - T2 ) or, I x 2V1 =n x - x
2
R(300 - T,)
~

nRT 5 5
or, I x 2 '>< - -1 = n x - x R(300 - T ) or, -T2
~ 2 2 2
= 690.
.. T2 = 276 K = 2.85 °C
W = nC/T2 - T 1)
I • 5
=-mol x - x 8.314 J.K- 1.moJ- 1 x (276 - 300) K = -249.42 J.
2 2
7 nRT,
/',.H = --x 8.314 x 24 = -349.14 J, P2 = - - - = 3.06 atm.
4 V2
FIRST LAW OF THERMODYNAMICS 133
14. (a) I mo! of an ideal gas is allowed to expand against a piston which supports
0.04 l MPa, the temperature being constant at 0°C (273.15 K). The initial pressure is 1.01325
MPa and the final pressure is 0.041 MPa. Calculate W, Q, tiU and tiH.
(b) If the change is carried out in such a way that the confining pressure at all time
is less than the gas pressure, find the value of W, Q, tiU and !iH.
Solo.: Given, P 1 = 1.01325 MPa = 1.01325 x 106 Pa= 10.1325 bar.
Similarly, P2 = Pex = 0.041MPa = 0.41 bar.
T °C = 273.15K, R = 0.08314 bar.litre.K- 1.mol- 1•

0.08314/.bar.K- 1 .mol- 1 x273.15K


· - - - - = 2.2412 l.mol- 1.
10.1325 bar

RT 0.083141. bar. K- 1 . mo1- 1 x 273.15 K 1.


= - - - - - - - - - - - - = 55.389 l.mol-
P2 0.41 bar
(a) W = -Pex (V2 - V 1) = -0.41 (55.389 - 2.241) = -21.79052 I.bar
= -2.179 k.J.
- w = Q = 2.179 k.j.
tiH = tiU = 0.

55 389
(b) W = -RT/n V2 = -8.314 x 273.15 In ·
v1 2.2414
= -7283.63 J = -7.28 k.J.
Q = -W = 7.28 k.J.
tiU = !iH = 0.
15. 104 gm of He were kept in a cylinder covered by a piston at 127 °C and 25 atm
pressure. The pressure is suddenly released to l atm adiabatically. (a) Calculate the final
temperature and work done. (b) What would be the final temperature and work done if
the change was carried out reversibly and adiabatically? (c) What would be the work done
if the change was reversible and isothermal? Calculate tiU and tiH.
104 gm He
Solo. : Rev.

~
127 cc 127 cc Ad. Irrev.
I atm !so. 25 atm I atm m

Ad.
Rev.

~
~
(a) P 1 = 25 atm P2 = 1 atm
n = 26 mo! T 1 = 400 K
T2 = ?

Assuming ideal monatomic gas. C 1 = ~ R.


2
134 PROBLEMS ON PHYSICAL CHEMISTRY

For adiabatic irreversible e.xpansion,


Pex (Vz - V1) = nCv (Tl - Tz)

or, P2
nRT nRT1 )
- -2 - - -
3
= n x -R(T1 -T2 )
( P2 P1 2

400 3
or, T2 - 2s = (400 - T2 ) or, T2 - 16 = 600 - l.5T2
2
or, 2.5T2 = 616.
. . T2 = ~i = 246.4 K = -26.6 °c.
2.5
.. w = nCV(Tz - T1)

= 26 mol x 2RJ.K- 1.mol- 1 (246.4 - 400)K =- .t.6 x 2x 8.314 x 153.6 J


2 2
= --49804.186 J = -49.80 kJ,
:. !:lU = - 49.80 kJ,

Ml = nC/T2 - T 1) =-'L6 x t" x 8.314 x (400 - 246.4) J = - 83.01 kJ,


(b) For an adiab<.4tic reversible process,
T
-R/c
p p
= constant.

= 400 x (;

= 110.3784 K
5
r 4
= 400 x (0.04)°·4

"" 110.4 K = -162.8 °C.


3
W = nCv(T2 - T 1) = 26 mol X lR(J 10.4 - 400)

= -26 mol x 2x 8.314 J.K- 1.mol- 1(400 - 110.4) K


2
= -93201.64 J = -93.90 kJ,
Now, t.U = w = -93.90 kJ,

= nCP (T2 - T 1) = -26 mol x l5 x 8.314 J.K-1 .mol- 1 x 289.6 K

= -156.50 kJ,
v
(c) Hints : Wrev,iso = -nRT In - 2 ; Q = -W; t.U = Ml = 0.
V1
FIRST LAW OF THERMODYNAMICS 135
16. What would be the work done, final temperature, Aff, etc., in the above case if
the pressure is suddenly released adiabatically to 5 atm and then (a) pressure is changed
to l atm reversibly and adiabatically and (b) the pressure is released to vacuum, and after
attainment of the volume corresponding to I atm, the pressure is made I atm. What is the
volume?
n = 26 n =26 n = 26
5 atm
Soln. : 400 K T2=? W2
25 atm Irr, Ad. P2 = 5 atm Rev., Ad
W1
Wz(Vacuum)
~

r3 = T2
V3 =?
P3 =I atm

(a) Given, n = 26 mol, Cv = 23 R, T1 = 400 K, P 1 = 25 atm.

W = W1 + W2 or, W1 + W2 '
For irreversible, adiabatic expansion,
WI = nCJT2 - T1) = -Pex <V2 - V1)

3
or, n x -R(T = -5 (nRT nRT
- -2 - - -1 ) = Pex = 5 atm)
2- 400) (given, P2
2 P 2 P 1

400
or, 2(400 - T2 ) = 5(T2 - ) = T2 - 80
2 5 25
or, 600 - l.5T2 = T2 - 80
or, 2.ST2 = 680.
. . T2 = 680 = 272 K
2.5
W1 = nC/T2 - T1) = 26 x l.5R(272 - 400)
= -41503.488 J = -41.503 kJ
t1U 1 = -41.503 kJ
Aff1 = 26 x 2.5 x 8.314 x (272 - 400) = -69172.48 J = -69.172 kJ.
For rev. adiabatic expansion : W2 = nCJT3 - T2 ).
4
T3 = T2( ~: )RfCp = 272(~r K

= 142.88 K.

W2 = 26 mot x 2x 8.314 J.K- 1.mot- 1 x (142.88 - 272) K


2 .. ·
= -26 x l.5 x 8.314 x 129.12 J = -41,866.64 J
= -41.87 kJ"J
136 PROBLEMS ON PHYSICAL CHEMISTRY

llU2 = -41.87 kJ.

llHz = -26 mol x ~x 8.314 J.K- 1.mo1-1 x (272 - 142.88) K


2
= - 69,777.7392 J = - 69.78 kJ.
w kJ.
= W 1 + W2 = - (41.50 + 41.877) kJ = - 83.37
llU = llU 1 + llU2 = (-41.50 - 41.87) kJ = -83·37 kJ.
llH = ll H 1 + ll H 2 = - (69.17 + 69.78) kJ = -138.95 kJ.
(b) For adiabatic free expansion,

w; = 0, T; = T2 = 272 K (since, the gas is ideal)


ll u; = 0, ll H; = 0
w = W1 + w; = W 1 = -41.503 kJ
llU = llU 1 + ll u; = -41.503 kJ
= llH + llH; = -69.172 kJ
1

nRT; 26molx0.082061.atm.K- 1 .mol- 1 x272K


=T = latm
= 580.32832 I I. = 580.3
17. If I mol of an ideal gas undergoes a reversible polytropic expansion, the relation
PV n = C holds where n and C are constants with n > I. (a) Calculate the work done. if
the gas expands from V1 litres to V2 litres with T 1 = 300 K and T2 = 200 K and n = 2.
5
(b) If Cv = -R, calculate Q, t..U and llH. [B.U. 1994; C.U. 20041
2
2 2

Solo. (a) W =- fPcxdV = -- JPdV. since the process is reversible.


I I

Now, PV2 = C, :. P = -c2 , n = I.


v

W = - J~ dV = [ ~r
I I

( -y;--v;- = ----y;--y;- , smce C 1 = P 1V 1


= C C) ( P Vi P V1 2 21 ) •

= (P 2 V2 - P 1V1) = R(T2 - T 1), since the gas is ideal


= - 8.314 J .K- 1.mol- 1 x I mol x I 00 K (given, T 1 = 300 K, T2 = 200 K)
= -831.4 J.
(b) llU = nCv(T2 - T 1)

5
= n-R(200 - 300) =-I mol x ~ x 8.314 J.moJ- 1.K- 1 x 100 K
2 2
(given, n = I mol)
= -2078.5 J = -2.078 kJ.
FIRST LAW OF THERMODYNAMICS 137

l'lH = nCP(T2 - T 1) = 1 mo! x l7 x 8.314 J.mol- 1.K- 1 x (200 - 300)K

= _'}__ x 8.314 x 100 J = -2.909 k,J.


2
Q = AU - W = (-2078.5 + 831.4) joule
= -1.247 k,J.
18. At 25 °C, ex for water is 2.07 x 10-4 deg- 1 and its density is 0.9970 gm/cc. If
the temperature of 200 gm of water is raised from 25 °C to 50 °C under a constant pressure
of 2 atm, calculate W, and if CP = 75.3J, calculate Q, AH and AU.

Solo. : ex = l( av)
V oT P

dV = exVdT.
2 2
W = - f PdV = - f PexVdT
I I

f
= -P ex V dT = -P ex V(T2 - T 1).
I
Given, ex = 2.07 X 10-4 deg-I T2 = 323 K m = 200 gm
P = 2 atm T1 = 298 K d = 0. 9970 gm/cc
200gm
.. v =
0. 9970 gm I cc
= 200.60 cc = 200.60 x 1o-3 litre.
.. W = - 2 atm x 2.07 x l(Y 4 K- 1 x 200.60 x 10-3 litre x 25 K
= -- 2.0762 x 1o-3 1-atm
= -(2.076 x 101.325 x 10-3 )1 [since 1 litre-atm = 101.325 J]
= - 0.21035 J.
200
AH = nCpdT = mo! x 75.3 J.mol- 1.K- 1 x 25 K
18

= (21~
0
x 75.3x 25 )1=20.917 kJ.
At constant pressure, Q = AH= 20.917 kJ
AU= Q + W
= (20.917 - 0.002) kJ
= 20.915 kJ
19. 2 mo! of an ideal monatomic gas with Cv = 15 J.K- 1 initially at 300 K and
atm is put through the following reversible cycle :
Path A : state l ~ 2, heated at constant volume till the temperature is 500 K.
B : state 2 ~ 3, expanded adiabatically till the temperature is 300 K.
C : state 3 ~ 1, compressed isothermally back to state 1.
Calculate Q, W, l'lU, l'lH for each path and whole cycle.
138 PROBLEMS ON PHYSICAL CHEMISTRY

Solo. :

T1 = 300 K
T A
T2 = 500 K
n = 2 mol, Cv = 15 J.K- 1

c
v
Path A WA = 0 (since, Li V = 0).
QA = L\UA = nCv (T2 - T 1) = 2 X 15 X 200 J = 6000 J = 6 kJ,
AflA = nCP (T2 - T 1) = 2 x 23.314 x 200 J = 9325.6 J = 9.32 kJ,
Path B Q8 = 0, W8 = L\U8 = 2 X 15 X (T1 - T2 )
= -2 x 15 x 200
= 6 kJ,
L\H8 = -9.32 kJ.
Path C : LiUc = Aflc = 0 (since the process is isothermal).

QC =-We= nRT1 In~ (aW= -PdV).


V2
Since path B is reversible adiabatic,
T2 vY- 1 - TI vY- 1
I - 2 ·

.. ~ = (2i ),',
V2 T2 .
= (2i
T2
f
.. QC =-We= nRT 1 In

= 2 x 15 x 300 In
UfG:} = nCvT11n( ;: )

= - 4597.4 J = - 4.597 kJ,


Qcy = Wey= WA + W8 + We= -1.4 kJ, L\Ucy = Aflcy = 0.
20. 10 gm of copper initially at 0 °C and 1000 atm is cqmpressed to 1 atm isothermally
and reversibly. Calculate the work done. Given that p = 0.725 x 10-6 atm, and density
of Cu = 8.93 gm/cc.

Solo. : p = __!_(av)
y ()p T
or, dV = -PVdP. i;
FIRST LAW OF THERMODYNAMICS 139

2 2
Now, work done W = - JPexdV = - JPdV [for rev. process]
I I

= JP~VdP =
Given, ~ = 0.725 x 10--0 atm-i, Pi = 103 atm m = 10 gm
P2 = 1 atm d = 8.93 gm/cc

. . v = __lO_;g:::...m__ = 1.1198 cc = 1.1198 x 10-3 litre.


9.93gm I cc

.. work done, W = ~V (P 22 - pi2)


2
(0. 725 x 10---0 atm- 1 ) x ( 1.1198 x 10-3 litre) x ( 1 - 10 6 ) atm 2
= 2

(o. 725x10-9 x1. l 198)x (1-10 6 )litre.atm


=
2
= --4.0592 x 10-4 litre.atm = -(4.0592 x 10-4 x 101.325) J = - 0.04113 J.
21. An automobile tire contains air at 300 kPa at 20 °C. The valve stem is removed
and air is allowed to expand adiabatically against the constant external pressure of 50 kPa
until the pressure is same inside and outside. Calculate the final temperature, W, Q, t'J.U

and L'lH per mole. Given, Cv = ~R. Assume air to be an ideal gas.
2
Hints : Pex<Vi - V2 ) = nC/T2 - Ti)

or, 50 (nRT, - nRT2


P1 P2
)= n~R(T2
2
- 293)

7 293
or, - T2 - 2. 5 x 293 + -
2 6
= 223.24 K = 223 K = -50 °C.
5
w = -R(T2 - T1) = -1.45 kJ,
2
L'lU = -1.45 kJ. L'lH :::: -2.04 kJ,
22. A 32 gm sample of methane gas initially at 1 atm and 27 °C is heated to
277 °C. The empirical equation for the molar heat capacity of methane at constant pressure
is CP :::: 3 + (2 x 10-2)T cal.mo1-i .K-i. Assuming ideal behaviour calculate Q, Ml, L'lU
for an isobaric reversible process. [V.U. 1997]
Vz Vz
Solo. : Work W = - JPexdV = - JPdV
V1 V1

Since P is constant,
140 PROBLEMS ON PHYSICAL CHEMISTRY

W = -P(V2 - VI)= -P(nRT2 - nRT1 ). p = P2.


P2 P1
Given, nr = 32 gm, P 1 = I atm, T 1 = 27 °C = 300.15 K
T2 =--277 °C = 550.15 K, P 2 = I atm, CP = 3 + (2 x 10-2 )T cal.moi-1.K--I

32 gm methane = -32 or 2 mol methane.


16

Hence, W = -2 mol x 8.314 J.K- 1.moi- 1( T 2 - p~~1 )K


= --2 mol x 8.314 J.K- 1.moI- 1 x (550.15 - 300.15) K
= -2 mo! x 8.314 J.K- 1.moi- 1 x 250 K = --4157 J = - 4.16 kJ,
,
!l.U = f nCvdT
I

= 2 mo1J(3-l.987)+(2xI0- 2 T)cal.mol- 1.K- 1dT(since, Cv = CP-R)


I

2 2
= 2f1.013dT f
+ 4 x 10-2 TdT
I I

= 2 mol x 1.013 cal.K-I .mol -I (T2 - 7.1) K + 4xl0-2 cal.K-2 .(T-" 2 - T2 1)K 2
2
= 2 mol x 1.013 caI.K- 1.moI- 1 x 250K + 2 x 10-2 (550 2 -300 2 )cal.
= 506.5 cal + 4250 cal = 4756.5 cal = 4756.5 cal x 4.184 J/cal
= 19901.1961 = 19.90 kJ,
2 2
. .
S1m1Iarly, !lH =2 x 3 x 250 cal + 4 x 10-2 ( -550- -300
-- )
cal
2 2

4x10- 2
= 1500 cal + x 212500 cal = (1500 + 4250) cal
2
= 5750 cal x 4.184 J/cal = 24058 J = 24.058 kJ,
23. One mole of an ideal monatomic gas at 25 °C and 5 atmosphere pressure is expanded
to a final pressure of I atm (a) isothermally against a constant pressure of I atmosphere
(b) adiabatically reversibly. Calculate in each case (i) the final temperature, (ii) the heat
absorbed, (iii) the increase in internal energy and (iv) the increase in enthalpy of the gas.
[V.U. 2000]
Solo. : Given, n = I mole, T 1 = 25 °C = 298.15 °K, P 1 = 5 atm, P2 =I atm.
(a) For isothermal expansion, T 2 = T 1 = T = 298.15 K.
Work done by the gas, W = -Pex<V2 - V1)

= -P2 ( p;-p;-
nRT nRT) ·
= -nRT ( I- P?)
p~
FIRST LAW OF THERMODYNAMICS
~~~~~~~~~~~~~~~~~~~~-
141

= -lmol x 8.314 J.K- 1.moJ- 1 x 298.15 K x(1-~)


= -1983.055 J = -1.983 kJ.
Since the gas is ideal, internal energy ( U) and enthalpy (H) are functions of T only.
:. f:i.U = f:i.H = 0.
Hence, heat absorbed, Q = W = 1.983 k.J.
(b) For reversible adiabatic expansion,
W = C/T2 - T1)

and T2 = Tl ( P2 )R/Cp [_!__ = ; - = 0.4J.


P1 Cp -R
2

~ )°
4
T2 = 298. I 5K x ( = 156.62 K.

Q =0
W = n 2_R( 156.62 - 298) K
2
=-I mole x 1.5 x 8.314 J.moJ-'.K- 1 x 141.38K
= -1763.15 J = -1.76 kJ.
ti.U = W = -1.76 k.J.

f:i.H = 1x
2
8.314 x 141.38 J = 2938.58 J = 2.94 k.J.

20
24. The pressure of a gas is represented by atm, where V is the volume of the
v
gas. If the gas expands from 5 I to 50 I and undergoes an increase in internal energy by
200 cal, how much heat will be absorbed during the process? [V.U. 2002)
20
Solo. : Since is in atm unit, let the unit of 20 be 1-atm, where V in litre.
v

v
= -20 In - 2 (1-atm) = -20 In JO 1-atm (V2 = 50 I, V1 = 5 I)
vI .
= -46.051702 1-atm x 101.325 J/l.atm
= -4666.1887 J = -4.67 kJ.
Q = f:i.U - W = 200 cal x 4.184 J/cal + 4.67 kJ
= 836.8 J + 4.67 kJ = 0.837 + 4.67 kJ = 5.507 kJ = 5.51 k.J.
25. One gm-mole of an ideal gas is kept at JO atm and 300 K. The pressure is
(i) reduced to I atm adiabtically and reversibly, (ii) the external pressure is made I atm
and the gas is allowed to expand till its pressure is also I atm, (iii) the external pressure
142 PROBLEMS ON PHYSICAL CHEMISTRY

is reduced to 1 atm and the gas is allowed to expand adiabatically till the volume is doubled
and then the pressure is raised to stop the expansion. Calculate W, Q, M and tiU.
Soln. : wad = nCV (Tz - T1), Q = ~
tiU = nCJT2 - T 1) = W
= nC/T2 - T 1)
= ~R.
2
By the problem, Ii= I mol, P 1 = 10 atm, T1 = 300 K.

(i) T2 = T1 ( ~~ r/Cp = 300 c~r 4


= 300 x 0.10.4 (given, P2 = 1 atm)

= 119.4321512 K = 119 K.
W = 1 mol x 1.5 RJ.K- 1.mol- 1(119 K - 300 K)
= -1.5 x 8.314 x 181 1 = -2257.2511 = -2.26 kJ,
tiu = w = -2.26 kJ.
M = 2.5R(l 19 - 300) 1
= -2.5 x 8.314 x 181 1=-3762.0851 = -3.76 kJ,
(ii) For irreversible adiabatic process, nCv(T2 - T1) = -Pe/V2 - V1)

= _p (nRT 2 _ nRT1 )
2
P2 P1

or, n~R(T2 - 300) = -nR(T2 -T1 Pz ). n = 1


2 ~

or,
450 - ~2 T2 = T - ( 300 x I )
2 10
5
or,
2 T2 = 480.
2x480
.. T2 = = 192K.
5
3
Q = 0, W = n-R(l92 - 300).
2
.. w = -1 mol x 1.5 x 8.314 1.K- 1.mol- 1 x 108 K
= -1.5 x 8.314 x 1081 = -1346.8681 = -1.35 kJ,
.. tiU = -1.35 kJ,
M = 2.5 x 8.314 (-108) 1 = -2244.781 = -2.24 kJ,
(iii) CJT2 - T1) = -Pz(2V1 - V1) = -P2 V1 = -P2 x nRT1
P1
FIRST LAW OF THERMODYNAMICS 143

2
or, 420 x - = T2, . -. T2 = 280 K.
3

.. w = -~ x 8.314 x 20 =- 249.42J.
2
13.U = -249.4 = - 249.4 J.

13.H = -~x 8.314 x 20 = - 415.7 J.


2
26. An ideal gas is expanded adiabatically against a constant pressure P2 until it doubles
its volume. If the initial temperature and pressure are T 1 and P 1 respectively, calculate the
final temperature. [V.U. 2005]
Soln. : See Problem 25.
27. Calculate 13.H and !iU for the transformation of one mole of ideal gas from 27 °C
and I atm to 327 °C and 17 atm. Given, CP = 20.9 + 0.042 J.K- 1.mot- 1. [V.U. 2005)
Soln. : CP = 20.9 + 0.042T J.K- 1.mot-I.
Cv = 12.586 + 0.042T J.K- 1.mot-I.
Given, n = 1 mo!, T 1 = 27 °C = 300.15 K, P 1 = 1 atm,
T2 = 327 °C = 600.15 K, P2 = 17 atm.

~U J
= n CvdT = n U f 12.184 + 0. 042T}K1.mol-ldJC

Since the gas is ideal, 13.U and liH foe change of P are zero.
:. 13.U = I mo! x 12.586 J.K- 1.mot- 1 x (600 - 300)K

+I mol x 0.04 2 J.K-z.mol-I x (600 2 - 3002)K 2


2
= I mole x 12.586 J.K- 1.moI- 1 x 300 K + 0.021 x 900 x 300 J
= 12.586 x 300 J + 5670 J
= 9445.~ J = 9.44 kJ,
Similarly,
13.H = 20.9(600 - 300) J + 0.021 x 900 x 300 J
= 11940 J = 11.94 kJ,
28. One mole of an ideal gas expands from 10 litres to 50 litres at 27 °C in the following
two ways : (i) isothermally reversibly and (ii) against a constant pressure 'of 1 atm. Show
by calculation in which case more heat will be absorbed during expansion. [C.U. 1993)
Soln. : Given, n = 1 mole, V1 = IO I, V2 = 50 I, T 1 = 27 °C = 300.15 K.
(i) For isothermal reversible expansion,
2
Q = -W = f nRTdV
V
= nRT In~
V2
I

50
= 1 mol x 8.3141.K- 1.mol- 1 x 300.15 K In- = 4016.267 J = 4.016 kJ,
IO
144 PROBLEMS ON PHYSICAL CHEMISTRY

(ii) In the second case,


W = -Pext (V2 - V1) = -1 x (50 - 10) = -40 litre-atm
= -40 litre-atm x 101.325 J/litre-atm = -4053 J = - 4.053 kJ.
In case (ii) more work is done, and more heat may be absorbed. The second change
cannot take place isothermally-the temperature has to change. Ideal gas can expand only
up to 25 I against a pressure of 1 atm if the process is done isothermally. Since in the
second case gas expanded to 50 I, temperature must have changed.

29. One mole of an ideal gas with Cv = 3 cal.deg- 1.mol- 1 initially at S.T.P. is put
through the following reversible cycle :
(i) State I to State 2-heated at constant volume to twice the initial temperature.
(ii) State 2 to State 3 -expanded adiabatically until it is back to initial temperature.
(iii) State 3 to State 1 -compressed isothermally back to state 1.
Depict the cycle in a V-T diagram and calculate Q, W and !l.U for the cycle.
[C.U. 1993; V.U. 2004]
Soln.: 3 (V2, T1)

adiabatic
expansion

2(V1, Tz)
(VJ. Ti) isochoric
process
T
Given, n = I mol, Cv =3 cal.deg- 1.mol- 1, T 1 = 273 K, P1 =I atm,
T2 = 2T 1.
1 1
nRT1 I mol x 0. 08206 l - atm. K- . mo1- x 273 K
=- - = ----·-------------
P1 I atm
= 22.40238 I = 22.4 I.
At constant volume, W = 0,
Q tlU
= nCV (Tz - Tl) = nCV (2T1 - T2)
= I mole x 3 cal.deg- 1.mol- 1 x 273 K
= 819 cal = 819 x 4.184 J
3426.69 J = 3.427 kJ.
!l.H = nCP x 273 J = 1 mo! x 4.987 cal.deg- 1.mol- 1 x 273 K
= 1361.45 cal = 1361.445 x 4.184 J
= 5696.307 J = 5.696 kl.
(ii) Adiabatic reversible change,
Q =0
tlU = W = nCv(T 1 - T2)
= -1 mo! x 3 cal.deg- 1.mol- 1 x (2T 1 - T 1) K
FIRST LAW OF THERMODYNAMICS 145

= -3 x 273 cal = -819 cal x 4.184 J/cal


= -3.427 kJ.
w = -3.427 kJ.
For adiabatic reversible change,
T2VIY-I = T1V/-I
2T1vi1.66-1 = Tl v2 l.66-I (y = 1.66)
or, 2 x 273 x (22.4) 0·66 = 273V20.66
v 20· 66 = 15.5665972 I
. . V2 = 64.02566123 I = 64.02 litres.
(iii) Isothermal compression.

Q
v1
= -W = .nRT1 ln-
V2

22
=l mot x 8.314 J.K- 1.mot- 1 x 273 K x In .4
64.02
= -2383.513549 J = -2.384 kJ.
!lU = 0.
Total !lU = 3.427 kJ - 3.427 kJ + 0
= 0.
Total W = -(0 + 3.427 kJ - 2.384 kJ) = -1.043 kJ.
Total Q = 3.427 kJ + 0 - 2.384 kJ
= 1.043 kJ.
30. l mot of an ideal gas with Cv = 20 kJ.mole- 1 initially at l bar and 300 K is put
through the following reversible cycle :
A. State l to State 2 by heating at constant volume to thrice the initial temperature.
B. State 2 to State 3 expanded adiabatically until back to initial temperature.
C. State 3 to State I, compressed isothermally back to state I. Calculate Q, W, !lU
and Ml for all the steps and the cycle.
Draw the change in a V-T diagram (See Problems 19, 29).
31. 0.5 mol of an ideal monatomic gas initially at 5 atm pressure and 0 cc is allowed
to expand against a constant external pressure of 0.5 atm. Conditions are such that the final
volume is 10 times the initial volume. The final gas pressure equals the external pressure.
Calculate Q, W, '1.U, Ml, L1S, '1.G for the process. [C.U. 1999, 2005]
Soln. :
Given, n = 0.5 mol
Pex = P2 = 0.5 atm
P1 = 5 atm
T1 = 0 cc = 273.15 K

0.5mol x 0.082061-atm.K- 1.mor 1 x273K
5atm
= 2.2414689 I = 2.24 I.
Prob. Phy. Chem.-10
146 PROBLEMS ON PHYSICAL CHEMISTRY

By the problem, V2 = 10V1 = 22.414689 I.


P2 V2 0. 5 atm x 22.4146891
T2 = ---;;/? = 0.5molex0.082061-atm.K- 1.mol- 1
= 273.15 K.
Since the process is isothermal,
ll.U =0
/),,ff = 0.
Q = -W = Pz(V2 - V 1) (·:d-W= P dV)

-
nRT
_ p2 ( - 2 nRT1 )
----
P2 P1
_
- nR (T2-
TI P2 )
P1

= 0.5 mol x 8.314 J.K- 1.mo1-1 x ( T- T;z)


= 4.157 J.K- 1 x 273.15(1- 0~5 )K
= 4.157 x 273.15 x 0.9J = 1021.9361 J = 1.021 kJ,
See chapter 6 for the calculation of !lS and ll.G.

!lS = nR In V2 = _!_mo! x 8.314 J.K- 1.mot- 1ln 10


V1 2

= 19.14 J.K- I x -I -_ 9.57 J.K-I .


2

ll.G = ]__RT In V2
2 V1

= __!_ x 8.314 J.K- 1.mol- 1 x 273.15 K In JO= -2614.5498 J = -2.61 kJ,


2
32. One mol of oxygen expands adiabatically against a constant external pressure of
1 atmosphere until the pressure balances. The initial temperature and volume are 200 °C
and 20 litres respectively.
(i) Explain whether the process is reversible or not.
(ii) Calculate the final temperature, work done and the change in entropy.
(iii) Will the quantaties be different if n moles of oxygen were taken. Give reason.
Assume ideal behaviour.
(iv) Calculate ll.U and /l.H. [C.U. 1997]
Solo. : (i) The change is not reversible since increase or decrease of pressure cannot
reverse the process.
(ii) Q =0
ll.U = W = nC/T2 - T1), if Cv is constant.
But W = -Pex(\12 - V1)
Pex = P2 = 1 atm; T1 = 200 °C = 473.15K.
V1 = 20 I,
FIRST LAW OF THERMODYNAMICS 147

nRTi lmolx0.08206 /-atm.moi-i.K-i x473K


= -- =
Vi 201
= l.941334 attn = l.941 atm .
.. Wad = - nC/T2 - T1) = -Pex (V2 - V1)

.. n x ~R(473.15 - T2) = P2(nRT2 - nRTi)


2 ~ ~

= nR(T2 -Ti-- ) =
1
l. 94
nR(T2 -~)
l. 94
5 473.15 _ 7T
. + - - - - 2·
or, - x 47315
2 l. 94,. 2

.. T2 = 2.5x473.15~243.89l75K = 407.627071 K = 407.63K.


2

W = n~R(407.63 - 473.15)
2

= -lmole x ~ x 8.314 J.K- 1.moJe- 1x 65.52 K


2
= -1361.833 J = -1.362 kJ,
AU = -1.362 kJ,

!ili = _2 x 8.314 x 65.52 J = -1906.566 J = -1.906 kJ.


2

AS = nCP In T2 - nR In p 2 (See second law of thermodynamics in Chapter 6)


T1 Pi

407 63 1
= 2_R In · - R ln-- = +1.172 J.K-1.
2 473.15 l. 94
(iii) If there are n moles, then AU, AH, AS, etc. will change. T2 will remain the same,
since Tis intensive and U, H, Sare extensive. W will change since adiabatic work is extensive.
33. A cylinder is fitted with a frictionless piston and is kept in a thermostat. It contains
2 moles of an ideal gas at 27 °C and 2 atmospheric pressure. Following (i), (ii) and (iii)
are three separate experiments carried out independently with the above.
(i) The piston is all on a sudden withdrawn to a position where pressure is reduced
to l atm and equilibrium is restored.
(ii) Pressure is reduced at a single step from 2 to 1 atm.
(iii) Pressure is slowly reduced to l atm in such a way that the position of the piston
remains unaltered if left to itself at any moment during the operation. Calculate
in each case AU, !ili, Q and W.
148 . PROBLEMS ON PHYSICAL CHEMISTRY

Soln. : Given, n = 2 moles, P1 = 2 atm, T1 = 27 °C = 300.15 K.


(i) .The expansion of the gas from 2 to I atm is against vacuum. So W = 0, AU = 0,
Ml= O; Q = 0.
Since the cylinder is kept in a thermostat the processes (i), (ii), and (iii) are isothermal.
(ii) So W = -Pex(V2 - V1) = -P2(V2 - V 1)

= -P2[nRT2 - nRT1] = -nR[T2 -Ti P2]


P2 P1 P1

= -2 mo! x 8.314 J.K- 1.mot- 1 T(1-~)K


= -2 mol x 8.314 J.K- 1.mol- 1 x 300.15 x _!_ K
2
= -2495.45 J = - 2.495 k.J.
For ideal gas, U = f(7) only,
Thus, Q = -W, AU = 0, Ml = O,
(iii) The work is reversible.
p
-W = Q = nRT ln-1
P2
= 2 mo! x 8.314 J.K- 1.moi- 1 x 300.15 K In 2
= 3459.42 J = 3.459 k.J.
AU = 0, Ml = 0.
34. One mole of a monatomic ideal gas at 80 °C and 5 atmosphere pressure is expanded
adiabatically against a constant external pressure I atmosphere in such a way that the final
pressure of the gas becomes I atmosphere. Calculate the final temperature, AU, Ml, M
for the process. [C.U. 1999]
Soln. : Given, n = I mol.
T1 = 353 K, P 1 = 5 atm.

pex 3
= P2 = 1 atm; CV = -zR, cp = ~R
2 .

W = -Pex(V2 - V 1) = nCv(T2 - T 1)

or, -P2(nRT2 - nRT1


P2 P1
)= n~R(T2
2
- T1)

or,

_ 353 x I __ 3
or, T2 2.(353 - T2 )
5

or, = 1.5 x 353 + 0.2 x 353

= 1.7 x 353
FIRST LAW OF THERMODYNAMICS 149

.. T2 = ~ x 1.7 x 353 = 240.04 K = 240K.


5

W = 1 mol x ~ x 8.314 J.K- 1.moJ- 1(240 - 353) K


2
= -1.5 x 8.314 x 113 J
= -1409.223 J = -1.410 kJ.
Q = 0.
f).U = W = -1.41 kJ.

Aff = 2. x 8.314 (240 - 353) J


2
= -2.5 x 8.314 x 113 J = -2348.7051 = -2.35 kJ.

i1S = nC In Tz - nR In Pz
P T1 P1

=I mol x 2. x 8.314 J.K- 1.mol-lx In 240 -I mo! x 8.314 J.K- 1.moJ- 1 x In.!
2 353 5
= 5.361 J.K-1•
35. 2 moles of an ideal monatomic gas initially at 100 °C and 5 atmosphere pressure
expands adiabatically and reversibly to I atm pressure. Calculate (i) the work done by the
. gas, .(ii) final molar volume, and (iii) change in enthalpy. [C.U. 2000]
R

~~ YP (~)°'
4

Hints : T2 = T 1( = 373 = 195.93K = 196 K.

nRT2
Aff= nCP (T2 - T 1); V2 = --.
P2
36. For a diatomic ideal gas near room temperature, what fraction of the heat supplied
is available for external work if the volume of gas expands at constant pressure and at .
constant temperature? [PhD Qualification Exam., Wisconsin University]
Soln. : Let the volume increase from V1 to V2 , temperature change from T 1 to T2 at
constant external pressure P, then
PV1 = nRT1
PV2 = nRT2.
Work done by the system, W = P(V2 - V1).
W = -P(V2 - V1)

= -P(nRT2 _ nRT1 ). Pi = Pz =p
P2 Pi .
= nR(T1 - T2).
f).U = nCV (Tz - Tl).
150 PROBLEMS ON PHYSICAL CHEMISTRY

= -W = -W nR R
Fraction of heat available for work
Q !':i.U-W
= = 5 .
-R+R
2

Since the gas is ideal diatomic, Cv = %R.


-W R 2
Q = '!_R·= 7·
2
At constant T, !':i.U =0 (since the gas is ideal) .
.. Q = -W
-W Q
Hence, - =- = I.
Q Q

37. Will the fraction W remain the same for both the cases in the last problem if
Q
atomicity of the gas changes?
Solo. : W, whether done at constant pressure or at constant T, will not be affected
with the change of atomicity. But heat supplied at constant P changes with the change of

atomicity. Hence, W remains 'the same for the process done at constant T but changes
Q
for the process done at constant P, if atomicity changes.
38. l mole of an ideai monatomic gas at 298 K expands to double its volume at constant
pressure. Calculate heat absorbed by the gas. [C.U. 2006]
Solo. : P 1 = P 2 = P, V2 = 2V1
T
Hence, T2 = V2 x --1. = 2T1 [:. V2 = 2V1]
Vi
Heat absorbed by the gas = nCP (T2 - T 1) = nCPT1

= l mot x ~x 8.314 J.K- 1.mot- 1 x 298 K = 6.19 kJ,


2
39. An ideal gas undergoes the following cycle-
(i) State l ~State 2, heating at constant volume to raise the temperature from T 1 to T2 .
(ii) State 2 ~ State 3, expansion with decrease of temperature from T2 to T 1• Work
done by the gas is 1000 J while 500 J heat enters the system.
(iii) State 3 ~State I, isothermal compression, which needs 600 J work.
Calculate W, Q, !':i.U for each step. 3
Solo. : State l ~ State 2, W1 = 0, !':i.U1 = Q 1.
State 2 ~ State 3, W2 = -1000 J, Q2 = 500 J, v
!':i.U2 = Q2 + W2 = -500 J.
State 3 ~ State 1, !':i.U3 = 0, W3 = -Q3 = 600 J.
Now, !':i.U cycle =0
or,!':i.U1 + !':i.U2 + !':i.U3 = 0,:. !':i.U 1 : -!':i.U2 = 500 J.
Hence, Q1 = 500 J. T Tz
FIRST LAW OF THERMODYNAMICS ~ 151
40. 2 moles of steam at 120 °C are condensed isothermally and reversibly to water
at 120 °C. Vapour pressure of water at 120 °C is 1490 mm of Hg, le= 526 cal.gm- 1. Calculate
W, Q, t:i..U and Ml.
Solo. : Given, n = 2 moles
le = 526 cal.gm-I
T1 = 120 °C = 393.15 °K
v8 = 1490 mm of Hg
W = -P(V 1 - Vg) = PVg (V 1 « Vg)
= nRT = 2 mol x 8.314 J.K- 1.mol-1 x 393.15 K = 6537.30 J = 6.53 kJ,
Q =Ml= -2 moles x 526 cal.gm-I x 18 gm.mo1-1 x 4.184 J.cat- 1
= -79228.22 J = -79.23 kJ
t:i..U = Q + W = (-79.23 + 6.53) kJ = -72·7 kJ,
41. At 40°C, the dissociation pressure of the system CuS0 4 , 5H20 ~ CuS04 , 3H20
+ 2H20 is 23.2 mm of Hg. At the same temperature vapour pressure of water is 55.1 mm
of Hg. Calculate maximum work for the change.
CuS0 4 , 3Hz0 + 2Hz0 = CuS0 4 + 5H20
Soln. : P 1 = 55.l mm of Hg, P 2 = 23.2 mm of Hg, n = 2 moles
55
W = -nRT In .!l = -2 moles x 8.314 J.K- 1.moJ- 1 x 313 K In ·1
P2 23.2
= -4501.93 J = -4·5 kJ,
42. l gm-mole of methane (van der Waals gas) is compressed reversibly and isothermally
from a pressure of l atm to 400 atm at 0°C. Calculate Q, W, t:i..U and Ml (a= 2.264 atm.litre2/
mole 2, b = 0.0428 litre/mole). For volume calculation assume ideal gas behaviour.
[C.U. 1979]

- b - a (-- _ ~)
1
Hints : W = -RT ln V2
V1 -b V 2 V1

V _ RT V _ RT
I - P:' 2 - p;
Hence, W = 12.8 kJ,

Again,

t:i..U =a (~-- 1
V1 Vz
)= -4.08
.
kJ.

Ml= t:i..U + t:i..(PV) = t:i..U + P2 V2 - P1V1 = t:i..U.


Since, V = pnRT (by the problem)

Q = t:i..U -
W = -16.9 kJ,
43. l rnol air is heated from 291 Kand 21 l to a final state of 319 Kand 22 l. Calculate
the heat absorbed by the system as well as change of internal energy. Air is assumed to
be an ideal gas with y = l .40.
152 PROBLEMS ON PHYSICAL CHEMISTRY

Solo : Initial pressure = Pi =nRT;


V;

I mol x 0.08206 l.atm- 1K- 1.mol- 1 x291Kx I mol


·-
211
= 1.137 atm = 1.14 atm
nRT
Final pressure = P1 = - -1
Vf

1molx0.08206/.atm- 1K- 1• mo1- 1 x319Kx1 mol


= 22/
= 1.189 atm = 1.19 atm.
The corresponding PV diagram is
PA= 1.14 atm, VE= 211 A c
PB= 1.19 atm,VD = 221
The work done is - (are a ABDEA)
= -[area of rectangle ACDEA + area of the triangle ABC]
v (I)
E D
I
Thus, work done =-[(ED x AE) + -(AC x BC)]
2
I
=-[(IL x 1.14 atm) + (- x I/ x 0.05 atm)] = -(1.14 + 0.025) /.atm
2
= -1.165 /.atm = -1,165 x 101.324 1 = -ll8.04 1
AU= nCy(Tf - T)

7
By the problem, y = 1.4 =~=I
2

Y=
c
-1:..
Cv

Thus, Cv = ~R.
2

.. AU= l mol x ~Rx


(319 - 291) K
2
= I mol x 2.5 x 8.314 1 k- 1.moJ- 1 x 2SK
= 581.98 1
Q = AU - W = 581.981 - (-ll8.04)1 = 700.021.
44. A cylinder contains 1/ of ideal gas which is kept at 3 atm pressure. How much
work is done by the gas if it is expanded isothermally and reversibly to l atm. Compare
this work with the work between the same two states but proceeding in a straight line in
the P-V diagram.
FIRST LAW OF THERMODYNAMICS 153
Solo. : The processes in the P-V diagram are as shown below :

The isothermal reversible expansion work done by the gas = W = nRT In~.
v
Vs
By the problem, VA = I litr~, PB = 1 atm A

PA= 300 atm, TA= TB"

.. W =PA VAln Ps (since, PAVA = nRT = PBVB) 1 c B


PA

= 3 In -I 1·ttre-atm
3 D E
V(litre)-
m3 N
= -3.2958 lit.atm x 10-3 fux 1.01325 x 105 mz.atm
= -333.95 J.
If A and B are joined by a straight line the work done = - (Area of ABEDCA)
= -(Area of triangle ABC + Area of rectangle CBED)
l
=-(DE x EB) - - x BC x AC.
2
Now, DE= CB = VB - Ve= (3 - l) lit= 2 lit
AC = pA - pc = pA - pB = (3 - l) atm = 2 atm
EB = P8 = l atm.
Thus, W AB(linear) = (-2 x l) lit. atm - ( ~ x 2 x 2 }t.atm =-(2 + 2) lit.atm

= -4 lit.atm = -4 x 101.325 J =-405.3 J.


45. The bulk modulus B of water at 0 °C is 2.032 x Hf atm. The average volume
expansion coefficient a = 4.33 x 10-4 deg- 1 at l atm. Find out the same coefficient at
1000 atm. It is further given that the bulk modulus is 2.137 x Hl4 atm at l00°C.
Solo. : By the problem, a 1 = 4.33 x 10-4 deg- 1.

Now the coefficient of compressibility, 13 =_!_,


B
when B is bulk modulus.

l
:. 130 at 0 °C = atm- 1 = 4.92 x 10-5 atm-1
2.032x 10 4

A
and t"IOO at 100 °C = l atm -I = 4.68 x 10-S atm- I .
2.137X 104

Now, a = _!_(av) (la)


V aT p

13 = __!_(av) . (lb)
V aP T

Again, V = j(P, T)
154 PROBLEMS ON PHYSICAL CHEMISTRY

or,dV = (av)
oP dP+(av)
T oT dT= (-PV)dP + (a.V)dT
p
[from Eqns. (la) and (lb)]

Since, V is a state function, dV is perfect differential

Thus, [a{a.V)J =[a(-PV)J


aPr p aT
or, a.(av) + v(aa.) = -P(av) _v( ap)
aP r aP r aT p aT p
0 0
or, -a.pv + v( a.) = -Pa.V - v( P) [from Eqns. (la) and Eqns. (lb)]
oP T oT p

(2)

Let temperature coefficient of p be x.

oTP) p = x
0
Hence, (

or, dp = x dT or,P100 = Po + x(T2 - T1).


Now, T2 = 100 + 273 = 373 K, T1 = 273 K .
5 5
... x -_4.68xl0- -4.92xl0- atm -lK-1-
. . x io-Satm-l.K-1.
- - 24
373-273
Let pressure coefficient of a. be y.
Thus, ( ~~ )T = y.
Now, from Eqn. (2). y = -x = 2.4 x 10-8 atm- 1 K- 1.
Thus, a. 1000 = a. 1 + 2.4 x 10-8 atm- 1.K- 1 (P2 - P 1).
Given that, P 2 = 1000 atm, P 1 = I atm.
Thus, a. 1000 = 4.33 x Io-4K- 1 + 2.4 x 10-8 atm- 1.K- 1(1000 - I) atm
= (4.33 x 10-4 + 2.4 x 10-8 x 999)K-I = 4.57 x 10-4 K- 1
Thus, volume expansion at I 000 atm = 4.57 x 10-4 K- 1.
46. A gas enclosed in a cylinder with a moveable piston undergoes reversible adiabatic
expansion according to the following equation p30 = constant.
(a) Find the work done by the system.
(b) System can also reach the final state (B) along any of the three paths ACB, ADB
and direct linear AB as shown in the figure. Calculate W and Q along each path.
Soln. : (a) Since the process is adiabatic Q = 0.
Given that, P 3 0 = constant. (I)

constant
Th US, P = V4/3 (2)
FIRST LAW OF THERMODYNAMICS 155

4/3
Hence, PAV A = P8 V 84/3 = constant = K. (3)

B 8 A c
Now, W~~abatic= - fPdV = -K f :~3 ~
1o5

l
u

~
A A
Q..

=-PAV!f
3

[
y--1/3
~± - :±y--1/3
Pa.m3
l 105
-
16 D B

10-3 V(m3) - 8 x 10--3

= 3x 10' x(io-'ri [(sx :0_, )i -Cd-' l ]Nm = -1501.

(b) For adiabatic work, AUAB = w:giabatic = -1501, since QAB = 0.


WACB = WAC+ WcB
WAC= -lCP (8 x 10-3 - 10-3)1 = -7 x 10-3 x ta5 1 = -7001.
Wc8 = 0, since AV= 0.
Hence, WACB = -7001
QACB = AUACB - WACB
= AUAB - WACB (since U is a state function AUAB = AUAcB)
= (-150 + 700)1 = 550 1
WADB = WAD+ WDB

105 --3 --3 105


=0 - - x 10 (8 - 1)1 = -7 x 10 x - 1 = -43.75 J.
16 16
QADB = (-150 + 43.75) = -106.25 J.

w~~ar = -[~( 10 5
- \
0 3 3 0 3 3
: )8x10-- -10-- ) + \ : x (8x10-- -10-- ) }

= -371.88 J.

Q~~ar = AUAB - W~;'.ar = (-150 + 371.88) 1 = 221.88 J.

Carnot Cycle and Thermochemistry


Required Formulae and Information

1. AH; is standard heat of formation per mole. For elements in their standard states
AH; is taken as zero at 298.15 K and one bar pressure.
2. For reaction between ideal gases, AH = AU + AnRT at constant temperature.
3. The heat change for the forward reaction is exactly identical in magnitude but
opposite in sign to heat change for the reverse reaction.
4. The enthalpy change of a reaction is independent of the number of steps.
156 PROBLEMS ON PHYSICAL CHEMISTRY

5 •. Efficiency of Carnot engine, T/ = -Wey =-Wey = Wey = T - T'


Q -W1 W1 T
Efficiency of Refrigerator and Heat Pump Refrigerator :
T'
Refrigerator : Coefficient of Performance, P = JI._ = =
T-T'
Wey
T
Heat Pump : Coefficient of Performance P = -Q =--W_,_i_ =
Wey W1 +W3 T-T'

.Here,· T = Temperature of hot reservoir


T' = Temperature of cold reservoi1:
-Wey = work done by the engine
Wey = work done upon refrigerator or heat pump
Q = heat absorbed by the eng!ne
= heat absorbed by refrigerator or heat pump
Q'
-Q' = heat rejected by engine
-Q = heat rejected by refrigerator or heat pump
Also, for carnot engine = -Wey = -(W 1 + W2 + W3 + W4 )
= -(WI + W3) =Q - (-Q') = Q + Q'.
Wey (or -Wey) is written also as W (or -W).
6. Clapeyron equation,

-dP = _(___
Ml
) for phase change I ~ 2,
dT T V2-V1
where T is the phase transition temperature.
. Cl apeyron equation,
. d In P Le
7• Cl ausius- - - =
dT RT2 '
where Le = latent heat of evaporation per mole.
8. Kirchhoff equation :·
2

Afl2 = Afl1 + fI
!!i.CPdT.

9. Effect of external pressure on vapour pressure of liquid :


If P 1 and P2 are the vapour pressures of liquid at external total pressures P11 and P 2,
res pee ti vel y.

In P2
p
= Vm
RT
(P2
I
- P1 )
I
I

where Vm is molar volume of liquid at temperature T. If P 1 is the vapour pressure when


there is no external pressure then P 1 = P 11 •
Short Questions
1. We know that heat passes from high to low temperature. How does, then, heat passes
isothermally from reservoir to system in Carnot cyC!e at the high temperature (HTR) and
from reservoir to system at low temperature.
FIRST LAW OF THERMODYNAMICS 157
Ans. : In case of source (HTR), it is assumed that the source is at temperature T +
dT while the system is at T. Heat passes from HTR to system reversibly. In case of LTR,
it is assumed that the source is at T ' - dT ', so that heat passes from system at T' to LTR.
2. Two systems with heat capacities C 1 and C2 respectively, interact isothermally and
come to a common temperature TF· If the initial temperature of system l was T1, what
was the initial temperature of system 2? [PhD Qualification Exam., Wisconsin University]
Ans. : Let the initial temperature of system 2 be T2. Assuming that total energy of
the .combined system remains unchanged, heat released by system l is equal to heat absorbed ·
by system 2. ·
Assuming T2 > T 1,
Ci(TF - Ti>= Cz(Tz - TF)
C1TF - C1T1 + CzTF = C2T2.
C1
. . Tz = -(TF - T1) + TF.
C2
3. A compressor designed to compress air is used instead to compress helium. It is
found that the compressor overheats. Explain this effect, assuming that the compression
is approximately adiabatic and starting pressure is the same for both gases.
[PhD Entrance, Wisconsin University]
Soln. : Let the change be adiabatic reversible, then

P(~)r = Po• y = cP ,
Vo Cv

P and P0 are final and initial pressures respectively and Vand v0 are final and initial volumes.
Because of compression, V0 > V and Ytte > Yair·
Thus, p He > pail'
But, PV = nRT.
·· THe > Tair·
Same result can also be obtained using T-V relation instead of P-V relation in the
following way.

To vJHe-1 = THe V'YHe-1


and

T \!'. )'YHe-1 ( V )'fair-I


Hence, -1:k.. = ( _Q_ -

r
Tair V Vo

= (~ He -y air ,

since V0 > V and 'Ytte > 'Yair• THe > Tair·


4. 10 litres of a gas at atmospheric pressure is compressed isothermally to a volume
of I litre and then allowed to expand adiabatically to 10 litres.
(a) Sketch the process in P-V diagram for a monatomic ideal gas.
(b) Make a similar sketch for a diatomic ideal gas.
158 PROBLEMS ON PHYSICAL CHEMISTRY

(c) Is a net work done on or by the system?


(d) Is it greater or less for diatomic gas? [PhD Entrance, Wisconsin University]
Soln. : 1(10 /, I atm) -t 2(1 I, P2 )
Since 1 -t 2 is isothermal, P 1V1 = P2V2 2
II -t diatomic
.. P2 = 10 atm.
II' -t monatomic
2 -t 3 is adiabatic. Assuming reversiblity,
p
I (101, I atm, n
(a) For monatomic gas, y = 5 = 1.6.
3 (10 I, 0.398 atm T3 )
5 3' (10 I, 0.215 atm, T/)
P; = P2 (~~r = 10(i~)3= 0.215 atm. v 3'

(b) For a diatomic gas, y = 73 = 1.4.

= !Oc~r = 0.398
4

P3 atm.

Work done on the system in the process I is similar for both gases and is greater than
the work done by the gas during processes II and II'. Hence, net work is done on the gas. It
is less for diatomic gas since work done during II is greater than the work done during II'.
S. The temperature T K is the temperature of a building which is heated by a heat
pump using a river at T0 K as a source of heat. The heat pump has a perfect performance
and consumes power W. The building loses heat at the rate a(T - T0 ). where a is a constant.
(a) Show that the equilibrium temperature Te of the building is given by :

Te = T0 + 2: [ 1 + (I+
4
~0 ) l·
(b) The heat pump is replaced by simple heater which consumes power W(constant)
and converts this into heat with 100% efficiency. Show explicitly why this is less desirable
than a heat pump. [PhD Entrance Examination, Columbia University]

Soln. We know that for a heat pump, -Q = _T_


W T-T0

:. -Q = WT
T-T0
By the problem, -Q = a(T - T0 ).
Let Te be the equilibrium temperature of the building.

At equilibrium, WT. = a(Te - T0 )


T.-To

or, WTe = a.CT/ - 2TeTo + T02 ) or, aT/ - (W + 2T0a)Te + aT02 = 0.


+ ~(w + 2T a) -4a 2 T
2 2
W + 2T a
0 0 0
.. T
e
=----.......;..--------
2a ·
'
FIRST LAW OF THERMODYNAMICS 159

w 1/2
or, Te = -+T0 +--vW +4T0 ex. +4WT0 cx.-4cx. T0
22 22
2cx. 2cx.
W l J 2
=To+ -+--vW +4Tocx.W
2cx. 2cx.

Thus, Te= T0 + W 4cx.T0 )] .


cx. [ I+ ( l+-W
2

(b) By the problem, for a heater -Q =W


Now -Q = cx.(T - T0 ).

At equilibrium, W = ex.( T~ - T0 )

'
Te =To+ aw
T; < Te. Thus consuming same power heat pump keeps the room warmer than ordinary
heater.
6. The temperature of a room is T2. It loses heat to the outside at temperature T1•
The rate is A(T2 - T 1) where A is a constant. It is warmed by a heat pump operating between

T 1 and T2 (T2 > T 1). The power supplied by the heat pump is dW
dt

(a) What is the maximum rate dQmax at which heat is delivered by the heat pump
dt
to the room. Evaluate the gain when T 1 is 275 K and T2 is 300 K.
(b) Derive the expression for equilibrium temperature of the room T2 in terms of T 1,
dW
A and - . [PhD Entrance Examination, California University, Berkeley]
' dt
Solo. : We know that for a heat pump,
Q T2
=
T2
. . - aQmax = ---=-- aw.
T 2 -T1

But the rate of heat lost to the outside = - aQ = A(T2 - T 1).


dt
Thus, at equilibrium,
T2 dW
A(T2 - T 1) = --=---
T2 -T1 dt

Hence, - aQmax = A(T2 - T1).


dt
Given, T2 = 275 K, T1 = 300 K
160 PROBLEMS ON PHYSICAL CHEMISTRY

Thus, dQmax = 300 =300 = 12.


dW 300-275 25
12 is the maximum rate at which heat pump delivers heat to the room.
(b) At equilibrium

Let dW = Y.
dt

.. A(T22 -2T1T2 +T12 ) = T2 .Y or, AT2 2 - (Y+ 2T1A)T2 + AT12 = 0.

Since T2 > T1 and ~ > 1,

dW
Now, Y= -
dt
2
Thus, T2 = T1 + - I (dW)
- + I- ~dW)
- +4AT1 (dW)
- •
2A ~ 2A ~ ~

7. A building at temperature T °K is heated using the heat from surroundi!tg at T0


K by an ideal.heat pump. The building loses heat at a.rate of a.(T - T0). The heat pump
consumes a power W. Find the equilibrium temperature of the building.
[PhD Entrance Examination, MIT]
Hints : See problem 5.
8. Define efficiency of a heat engine. Derive an expression for the efficienc;y of a
reversible cyclic process (following fig.) with I mole of an ideal gas.
Given that the isothermal steps are A~ B, C ~ D and A
e ~ F, adiabatic steps are B ~ C, D ~ E, F ~ A, where
VA, V8 , Ve, V0 etc. are the volumes of the systems at the f
VA Ve VF
respective steps. It is also given that - =- =-
VB VD VE E

V-+
FIRST LAW OF THERMODYNAMICS 161

Ans. The efficiency of heat engine is given by T] =- W,


where Wis work done by
Q
the engine on the surrounding and Q is the heat absorbed at some high temperature for
doing some work in a cycle. ·
For the heat engine working in the given reversible cycle,
-W = total work done by engine = -(W 1 + W2 + W3 + W4 + W5 + W6). (I)
For an ideal gas subject to isothermal processes, 11.U = 0.
Now, t:i.U = Q + W,
where Q = heat absorbed by the engine
W = work done upon it.
Thus, for isothermal steps.

WI = WA-->B = -Q 1 = RT1 In ~A, (2)


B

and W5 =WE_,F= -Q3 = RT3 In vVE. (4)


F

Since Q =0 for an adiabatic process,


W2 = !1.U2, W4 = AU4, W6 = !1.U6 .
Thus,W2 = CiJ..T2 - T 1), W4 = CiJ..T3 -
T2), W6 = CiJ..T1 - T3).
Hence, W2 + W4 + W6 = 0. (5)
Hence, -W = -(W1 + W3 + W5) [From eqns. (I) and (5)]

VA
= - ( RT1 In-+RT Ve VEJ
2 ln-+RT3 Jn- .
VB Vo VF
v v
By the problem, VA = __£ = -L
v
B VD VE

Thus, W = -R(T1 + T2 - T3) In -VA . (6)


VB
Now heat absorbed Q = Q 1 + Q2 .
Thus, Q = - (W1 + W3) [From eqns. (2) and (3)]

Hence, efficiency T]
W
=~ = -W =
Q -(W1 + W3 )

= T1 + T2 -T3 = I-
T1 +T2
162 PROBLEMS ON PHYSICAL CHEMISTRY

Numerical Problems
Carnot Cycle
1. A steam engine operates between 14 °C and 30 °C. What is the minimum amount
of heat that must be withdrawn to obtain l 00 J of work?

-W,ov T~T'
Solo. : We know that, =
Q T
By the problem,
-W = 100 J' T' = 14 °C = 287 °K, T = 30 °C = 303 °K.
The heat that must be withdrawn from high temp. reservoir =Q
= -W x -T- = 100 x -303 J = 1893.75 J = 1.894 kJ,
T-T' 16
2. An ideal heat engine operating between a source having a temperature 0 °C, and
a sink T2 produces 1000 cal of work per cycle rejecting 6400 cal of heat. Calculate the
efficiency of the engine and the temperature of the sink. [V.U. 1987]
Solo. : By the problem,
-Q' = 6400 cal, -Wey= 1000 cal.
Q = -Q' - Wey = 6400 + 1000 = 7400 cal.
T= 273.15 °K (HTR) .
.. -Wey = T-T'
Q T
1000 273.15 - T' T'
or, = =1-
7400 273.15 273.15
T' 1000 6400
or,
273.15
=l -
7400
= 7400
64
T' = x 273 .15 = 236.2378 K.
74
3. Calculate the minimum amount of work in ergs required to freeze 1 gm of water
at 0 °C by means of a refrigerator which operates in surrounding at 25 °C. How much heat
in calorie is given to the surrounding? Given that latent heat of fusion = 80 cal/gm for
ice at 0 °C. What will be change in entropy of the room?
Q'
Solo. : For a refrigerator, - = -T'-,
W T-T'
where Q' is the heat absorbed from LTR, W is work done on the refrigerator and Q is the
heat rejected at T (25 °C)
By the problem, Q' = 80 cal/gm, T= 25°C = 298.15K, T'= 0°C = 273.15K
80 273.15 273.15
w = 298.15 - 273.15
= 25

.. W = 80 cal x~ = 7.321984 cal x 4.184 J/cal


273.15 .
= 30.63518106 J = 3.0635 x I 0 8 ergs.
FIRST LAW OF THERMODYNAMICS 163

Q = 80 + 7.32 cal = 87.32 cal.

tlS
room
= tlSsurr = -QT = 87.32cal
298.15 K
= 0.293 cal.K-I .
4. An ideal operating Carnot cycle operates on a temperature difference of l 00 °C
and half of the heat absorbed at high temperature T is wasted as heat at low temperature
T'. The cycle does 150 J of work. Find Q, Q', T and T'.
Soln. : By the problem,

-Q ' = _!_ Q, T - T' = 100.


2
-W T-T' 100
=--=-
Q T T
-W Q-(-Q') l
Tl = Q
=
Q
=2
-W = 1501.
150
.. = 2' Q = 300J .
Q

Again, --W
Q
= 2
=
100
- , T = 200K.
T

T' l
.. l - = -2
200
l T'
-
2
= 200,
.. T' = lOOK.

5. An ideal Carnot engine operates on a temperature difference of 250 °C and _!_th


4
of heat absorbed at high temperature Tis wasted as heat discharged at lower temperature
T'. The cycle does 500 J of work. Calculate Q, Q', T and T'
[Ans. Q = 666.67 cal, Q' = -166.67 cal, T = 333.333 K, T' = 83.333 K]
· 6. A reversible heat engine absorbed Q heat at 800 K per cycle rejecting heat Q' at
200 K. Its work output is used to run a hoist and owing to the friction in the pulleys l 0%
of the work is converted into heat at 200 K. For the system engine + pulley the total heat
evolved is l 0000 J. Calculate Q, Q' and W.
Soln. : Let heat be absorbed = Q, heat rejected = -Q'.
Thus, efficiency of the engine
-W = T-T' = 800 - 200 = 600 = 0.75.
Q 7' 800 800
:. -W = 0.75 Q.
10% W = 0.075Q, converted into heat.
Now, Q - (-Q') = -W.
-Q' = Q - 0.75Q = 0.25Q.
164 PROBLEMS ON PHYSICAL CHEMISTRY

Heat released = 0.25 Q + 0.075 Q = 0.325 Q.


But by the problem, 0.325Q = I 0000 l

Q = lO.OO<:?_ = 30769.23 l = 30.77 kJ.


0.325
Q' = -0.25Q = -7692.3075 = -7.692 k.J,
w = - 0.75Q = -23076.9225 = -23.08 k.J.
7. A reversible heat engine absorbs heat Q at 1000 K per cycle and evolves heat Q'
at 350 K. Its work output (W) is used to run a machine. 20% of the work is converted
into heat due to friction. For the system engine plus machines, the total heat evolved is
I 5000 l per cycle. Calculate Q, Q' and W.
[Ans. Q = 312501, Q' = -10937.51, W =-20312.51]
8. A heat engine operating between 800 °C and 25 °C produces work (absorbing heat
Q at 800 °C) that is entirely used to run a refrigerator operating between 0 °C and 25 °C.
Calculate the ratio of the heat absorbed by the pump at 0 °C (Q') to that absorbed by the
engine (Q). Assuming ideal operation for both, calculate Q, Q', W and the heat rejected
by the heat pump at 25 °C, if Q' heat absorbed at 0 °C leads to conversion of I 00 gm
of water to ice, CP = 325 l/gm. How much heat is rejected by the engine at 25 °C?
Soln. : By the problem,

HTR 25 °C
800 °C

Refrigerator

Q'
25 °C
LTR 0°C

-W T-T' 775
For engine, - = = (25 °C = 298 K, 800 °C = I 073 K).
Q T 1075
775
.. -W = Q
1073
775
.. W= -Q
1073
For the refrigerator,
Q' T' 273
= =
w T-T' 25

. . Q' -- 273 .w -- _273 x 775 x Q [since engine runs the refrigerator]


25 25 1073 .
FIRST LAW OF THERMODYNAMICS 165

Q' .
:. - = 7.814404432 = 7.81
Q
or, Q' = 7.81
Q
i.e., Q = 0.13 Q'.

Q
Thus, -
Q'
= 0.13.
By the problem, Q' = 325 x 100 J = 3.25 x 104 J.
Heat rejected to high temperature reservoir
= Q' + w
4
= 3 .25 x 1041 + 25x3.25xl0 J
273
= 32.5 kJ + 2.98 kJ 35.48 kJ. =
Q = O. I 3Q' = 4225 J = 4.2225 kJ.
w = 2976.19 J = 2.98 kJ.
9. An ideal refrigerator works between 0 °C and T °C. It freezes 2 kg of water at
0°C per hour. At the same time, the total heat output to the room is 200 kcal.h- 1. Calculate
T 0 C. Latent heat of fusion of water at 0 °C 80 cal/gm. =
Solo. : 2 kg of water freezes at 0 °C. So Q' (heat absorbed) = 2 x 80 000 cal = 160
kcal per hour. The refrigerator works between 0°C and T °C.
Given that Q = 200 000 cal.
Q'
Now, - = -T'-
W T-T'
-Q = Q' + W = 200 000 cal
W = -Q - Q' = (200 000 - 160 000) cal = 40 000 cal.
W
Thus, - = T-T' = -T - I.
Q' T' T'
40000
.. - - + =- + I = !__,
160000 4 T'

. . T = 7" x ~ = 273 x ~ = 341.25 K.


4 4
10. An ideal refrigerator works between 0°C and a higher temperature T °C. It freezes
3 kg of water at 0 °C per hour (if= 80 cal/gm) and leaves 300 kcal per hour at T °C.
Calculate T.
. W T-T'
Hmts: - = - -
Q' T'
T' = 273 K
Q' = 240 000 cal
W = Q - Q' = 60 000 cal
T = 341.25 K.
166 PROBLEMS ON PHYSICAL CHEMISTRY

11. A heat engine operating between l00°C and 25 °C absorbs 1000 J work from high
temperature reservoir. How much heat is rejected to low temperature reservoir?
Soln. : Given, Q = 1000 J, T = 100°C = 373.15 K.
T' = 25°C = 298.15 K
-W -W T-T' 75
Thus, Tl = Q = lOOOJ = T = 373.15 = 0.201.
:. -W = 1000 J x 0.201 = 201 J.
Hence, heat rejected = Q' = Q - (-W) = (1000 - 201) J = 799 J.
12. Two idential bodi.:s~f constant heat capacity at temperatures T 1 and T2 respectively
are used as reservoir for a heat engine. If they remain at constant pressure, show that the
amount of work obtainable is W = Cp<2T - T1 - T2), where Tis the final temperature attained
by both the bodies. Show that when T = ~T1 T2 , then W is maximum.

Soln. : The heat engine, working in a cycle, takes tfQ1 heat from high temperature

reservoir (HTR) at T1 and rejects i!Q2 lteat to low temperature reservoir (LTR) at T2.
Eventually temperature of HTR decreases to T and that of LTR increases to T. If work
obtained from the engine is W. and Q 1 is the total heat gained by the engine and Q2 is
the heat rejected by the engine, then
-W = QI + Q2 = CpCT1 - n - cp (T - T2)
= CP(T1 + T2 - 27).
Hence, W = Cp(2T - T 1 - T2).
If the engine is a Carnot engine, then W will be maximum.
For a Carnot engine for one cycle,

aQ1 + aQ2 = o.
T1 Tz

Now, aQ1 = CpdTl, and -aQ2 = -CpdT2


dT1 dT2
.. =--
T1 Tz
T
Now,
T

sd;1 = _ dT2
Tz
f
T1 I Tz
T T T2
or, In - =-In- = ln- .
T, Tz T

.. T= ~T1 T2 .
FIRST LAW OF THERMODYNAMICS 167

13. A reversible engine converts ..!.th heat which it absorbs at heat source into work.
6
When the temperature of the heat sink is reduced by 82 °C, its efficiency is double. Calculate
the temperature of the souce and the sink.
T-T'
Solo. : We know efficiency, T\ = -W = - -
Q T

By the problem, - W = _!_Q,


6
1 T-T'
Thus, T\ = - = (la)
6 T
5T
:. T' = (lb)
6'
when T~ew = T' - (273.15 + 82)K = T' - 355.15K

1
Then T\new = 2T\ = )

Thus_!_ = T-(T' -355.15K)


(2a)
'3 T

5T
or, T = 3T - 3 T' + 1065.45K =3 x - - 1065.45 K [from eqn. (lb)]
. 6
or, 0.5T = 1065.45 K :. T = 2130.9 K. (2b)

1 2130.9K-T'
From eqn. (la),
6 2130. 9K
T'
or, 1 -
2130.9K
= 6

T' 5
or,
2130.9K
= -6
..T' = 1775.95 K.
Thus, temperature of the source =2130.9 K and temperature of the sink = 1775.95 K
14. l mo! monatomic gas is used in a Carnot engine working between 700 K and 350
K. The smallest volume occupied by the gas during the course of operation is 500 ml and
it absorbs 910 J heat per stroke.
(a) Calculate W, Q, !l.U, !lS for each step in Carnot cycle.
(b) Calculate Wey• Qcy' !l.Ucy' D.Scy
Solo. : Efficiency of a Carnot engine irrespective of nature of the working material
and its amount,
-W T-T'
= T\ =Q =-T- (I)

Given: T = 700K, T = 350K, Q = 910 J, n = 1 mol.


168 PROBLEMS ON PHYSICAL CHEMISTRY

Thus, ri = 700- 350 = 0.5. (2)


700
(a) Step A ~ Step B,
Assuming the gas to be ideal, !lU = 0. p
= 910 J
l
Hence, -W 1 = Q (3)

/lS = 910 J.k-1 = l.3Jk-I


I 700
(b) Step B ~ Step C
v--
Q = 0,
llU 1 = W2 = nC./f - 7)

= 2R(350 - 700)K (·:Cv =l_R for a monatomic gas)


2 2
3
= - x 8.314 x (-350) J
2
= -4364.85 J = --4.364 kl. (4)

!lS =0 (since the process is reversible adiabatic)


Step C ~ Step D, !lU = 0, W3 = -Q'
From eqn. (l), -W = 0.5 x 910 J = 455 J. (5)
-W = Q + Q' = 4551 (":Q - (-Q') = Q + Q')
Q = 910 J
Thus, -Q' = (910 - 455)1 = 4551 (6)
55
Hence w3 = 455 J, !lS2 = --4 J.K-1 = -1.3 JK-1
350
Step D ~ Step A
Q =0
llU2 = W4 = nCy(T - T') = 4.364 kl (7)
!lS = 0.
(b) From eqns. (5) and (6), Wey = --455 J
Qcy = 455 J, llUcy = 0, !lScy = 0.
15. An air conditioner is nothing but a refrigerator in which work done upon it and
heat removed from the cooler inside which is rejected at the warmer outside.
Such an air conditioner operating on a Carnot cycle (which is reversible) taking heat
from the inside at temperature T2 and rejecting heat to the outside at temperature T1 consumes
W I/sec from the power line.
(a) The air conditioner absorbed Q2 (J) from the inside and rejected Q 1 at the outdoor.

Derive a formula for the term Qz as a function of T 1 and T2 (T 1>T2 ).


w
(b) Heat leakage into the house follows the equation Q =A(T1 - T2). Develop a formula
for T2 as a function of T1, W and A for the continuous operation of the conditioner under
constant outside temperature T1, and T2 is the inside temperature (uniform)
(c) The air conditioner works between room temperature 293 K and the outside
temperature 303 K. The machine operates only 30% of the time. Find the highest outside
temperature (H.T.R.) for which it can maintain 293 K temperature.
FIRST LAW OF THERMODYNAMICS 169

(d) In the winter, the cycle is reversed and the system becomes a heat pump that absorbs
heat from outside and releases heat into the inside, find the lowest temperature in K for
which it can maintain 293K inside. (T2 > T1)
Outside Temp
Soln. : (a) According to the problem, Q2 +W = -Q 1. (I) T 1 (const)
Since air conditioner works on a cyclic process, Afsys =0. (2)
Since the cycle is reversible,
w
Afl =Q2 ' Af2 = - QI. (3)
T2 T1
From eqn. (2) Af 1 = Af2. Inside Temp T2
(varies but ulti-
mately unifonn)
Hence, - Qi = Q2
T1 T2

Q1+W T1
or, - - = - [From eqn. (I))
Q1 T2
Q, T,
or, -- =----.
W T1 -T2

(b) Now Q2 = w( T2
T1 -Tz
) =A(T1 - T2) [at equilibrium] (4)

Hence, T2 W = A(T1 - T2) 2


or, T2 W = A(T12 - 2T1T2 + Tl)
or, AT22 - (W + 2AT1)T2 + AT12 = 0. (5)

= (W+2AT1 )±~(W+2AT1 )2-4A T1


2 2
Thus, T2
2A
2
lw +4WAT +4A 2 T 2 -4A 2 T 2
= -W+ -2AT
- 1 ± '\' I I I
2A 2A 2A
1w2 +4T WA
= -W + T ± ~'\'____ 1 _
2A I 2A

= T1 +~[: ± ~ ( ~W 2 +4T1WA )]

But T1 > T2 .

Thus, T2 = rf+ ~[: - (:)' +


4
~,VJ
170 PROBLEMS ON PHYSICAL CHEMISTRY

A(T1 -T2)2
(c) When the machine is 30% efficient then W 30 o/o =
By the problem, TI = 303 K; T2 = 293 K

A( J02) JOO
Thus, W30 % =~ = 293
A.

When it operates continuously, i.e., 100% of the time, then

W =W 100 =JOO ·A x lOO = l.1377A.


JO% 30 293 30
When T2 = 293 K,
we solve equation for TI in terms of T2, W and A.
From Equation (5),
ATI 2 - 2ATIT2 - (WT2 - ATz2) =0

(6)

Since TI > T2,

:. TI= Tz +~w.2.
When T2 = 293 K, we get TI = T2 +~W. 2 = 293 + l.1377Ax
293
A

= 293 + ..J 1.1377 x 293


= 293 + 18.257 = (293 + 18.26)K = 311.26K = 38.26°C.
(d) Hence, QI + W = -Q2.
Thus, W =- Q 2 - QI.
As a heat pump the coefficient of performance,

p = -Q2 = T2
W T2 -T1
Solving eqn. (3) for TI, we get

2AT2 ±~4A
2
T2 2 -4A(AT22 - WT2 )
TI = 2A

= T2 ± ~T2 :.
FIRST LAW OF THERMODYNAMICS 171
Since T2 > T1,

Tz = Ti + ~T2:. (7)

Hence, T1= T2 - ~: T2
= 293 - ~l.1377x293 = 293 ~ 18.26 = 274.74K = l.74°C.
16. With an intake temperature at 300°C and exhaust temperature of l00°C in a Carnot
cycle in operation is a steam turbine. What is maximum work the turbine can do for a
given heat intake Q.
Solo. : For a Carnot cycle with heat absorbed Q at higher temperature T and heat
-W T-T'
rejected Q' at lower temperature T' the effickncy is T\ = - =
Q T
By the probllem, T = 300 °C = 573.15 K
T' = 100 °C = 373.15 K
-W = Q + Q'
(Q + Q') is equal to (1- ~)Q.

-W = (1-T')Q= (1-373.15)Q=0.35Q.
max T 573.15
It is the maximum work a turbine could do since it is working in a reversible cycle.
The work will be less than 0.35Q if the turbine operates irreversibly.
Clausius-Clapeyron Equation
1. The pressure at triple point of water is 4.56 mm of Hg. Under this condition the
volume of liquid water per gram is 1.0001 cc and that of ice is 1.0906 cc. Find out the
temperature of the triple point. If= 80 cal/gm. [C.U. 1968]

T(V2 -V1 )
Solo. : We know, !ff = .!iP. [Clapeyron equation]
Ml
Let the change from I ~ s.

But,
75~.44
Af' = -(4.56 - 760) mm of Hg= ---atm.
760
.. vs - v1 = (t.0906 - 1.0001) cc
= 0.0905 cc/gm = 0.0905 x 10-3 litre/gm
for I ~ s, If = - 80 cal/gm.

!iT = 273.15Kx0.0905x 10- litre.gm- x """(-_7_55_._44~)atm x


3 1
. cal
1 24 21
-80cal.gm- 760 litre.atm
= 0.0074 °C .
172 PROBLEMS ON PHYSICAL CHEMISTRY

Thus, for a decrease of pressure from 760 mm of Hg to 4.58 mm of Hg, freezing point
of water increases from 0 °C by 0.0074 °C. Hence, temperature of triple point = 0.0074 °C.
This may also be calculated as follows :

dP Aff - 3ocal.gm-t = -883977.9006 ca1.1- 1 x


dT = T!J.V = 0.0905xl0-3 Iitre.gm- 1 XT T

dT
or, dP = -883977.9-
T

Tz
or, P2 - P1 = -883977.9 ln-
T1

or, In T 2 = 755.44x24.21
T1 760 x 883977. 9
.. T2 = 273.1574 K = 0.0074 °C.
2. The vapour pressure of water changes by 27.17 mm from 99.5 °C to 100.5 °C. The
specific volume of water and steam are 1.04 cc/gm and 1674 cc/gm respectively. Calculate
le.
Soln. : V1 = 1.04 cc/gm
vg = 1674 cc/gm
T(V2 - V1 )
!J.T = !J.H
!J.P [Clapeyron equation; for l -1 g]

By the problem, !J.T = (I 00.5 - 99.5)°C = I °C = I °K


M = 27.17 mm
T = 373.15 K.
1
:. Aff = T(V2 -V1) !J.P = 373.15x{1674-l.04)cc.gm- . mm
27 17
!J.T IK
373.15xl672.96x27.17 .
= 1 x I 03 x 760 1itre-atm.gm- 1

= 2261.3 J.gm- 1 [since 1 litre.atm = l01.325J] = 2.26 kJ.gm- 1


3. le for H2 0 is 540 cal/gm at 100 °C. At what temperature will wat.:!r boil at 800
mm of Hg?
Hints : Here, le = 540 cal/gm
T 1 = 373 K, P 1 = I atm
800
P 2 = 800 mm = atm
760
Tz =?

P2
In
P,
= leR [ TT2 -T
T
1]

1 2

800 x 18 [ T2 - 373.15 J
or, In
760
= 540
I. 987 T x 373.15 2
FIRST LAW OF THERMODYNAMICS 173

T2 = 374.62 K
T2 = 101.62 °C.
4. Given that dP is 2. 72 cm of Hg per degree for water at I 00 °C. Calculate the
dT
approximate heat the vaporisation in cal/gm at this temperature. Calculate 6.V.

Hints : dP = ___!::s_
dT T.6.V
I dP L.
or, p dT = RT2 [Vg >> Vil

RT 2 dP
.. Le = P.dT
8.3141.K- 1.mole- 1 x(373.15) 2 K 2 2 72atm K- 1
= x · · = 41431.65 J.moJ- 1•
latm 76
:. Le = 2301.7583 J.gm- 1
= 550.133 cal.gm- 1.
6. V Calculation
Again, from Claperyon equation,

M = ___!::s_
dT T.6.V

41431.65 J. mo1- 1
or, = Le = 2 72
TdP 373.15 K x · atm. K- 1 x 101.325 JI I. atm
dT 76
= 30.6 = 1.7
litre/gm.
litres/mo!
5. The latent heat of fusion of deuterium at 18.56 K is 47.0 cal/gm-mole. The slope

dP is 40.5 kg/cm 2 .deg- 1. Calculate the difference between molar volume of solid and liquid
dT
deuterium at this temperature. [C.U. 1981]
Solo. : From Clapeyron equation,

dP = ___!::s_
dT T.6.V

or, v1 - V5 = 6. V = /Jp = 47cal. mole- 1 x4.184J I cal


18.56K x 40.5 x 9.80665m.sec-2 x10 4 kg. m- 2
dT

= 196 · 648 N.m.mole-l = 2.668 x 10--0 m3.moJe- 1 = 2.667 cc.moJ-1.


73714626. 7 kg. m- 1. sec 2
6. The latent heat of evaporation of water is about 2.25 x I 06 J/kg and the vapour
density is 0.0598 kg.m- 3. Find the rate of change of boiling point with altitude near sea
level in °C per km. The air temperature may be assumed to remain constant at 300 K.
Solo. : By Clausius-Clapeyron equation,
174 PROBLEMS ON PHYSICAL CHEMISTRY

_ Mgh
According to barometric formula, P2 = P1e RT

:. In P2 = - Mgh .
P1 RT
By the problem, T = 300 K, M = 29.04 (mol. wt of air),
h = 105 cm,
le = 2.25 x 106 J/kg = 2.25x106~107x18 erg/mole
10
6 7
_Mg!!_= 2.25xl0 xl0 xl8(-l---l-)··
RT 10 3 x R 373.15 T2

5 11
or, _ 29. 04 x 980. 665 x 10 = 2. 25 x 10 I ).
x 18 ( - 1 - __
Rx 300 103 x R 373.15 T2
:. T2 = 369.91°K = 96.76 °C.
:. t:.T = T2 - T 1 = 96.76 - 100 = -3.24 °C.
:. rate of change of boiling point = -3.24 °C per km.
7. For MgS04 , 7H2 0 the dissociation pressure is 35.6 mm of Hg at 35 °C and 47.2
mm of Hg at 40 °C. Calculate Aff for MgS0 4 , 7H 20 = MgS04 + 7H 20.

Soln.: In P2 = 6.H[T2 -Ti]


P1 R T1T2

or, In 47.2
35.6
= Aff [ 5
8.314 313 x 308 .
J
Aff = 45.212 kJ,
But if the reaction occurs producing H20 as a gas,
MgS04 , 7Hp = MgS04 + 7Hp(g)
Kd = p~20
6.H = 7 x 45.212 kJ = 316.5 kJ,
8. The vapour pressure of liquid fluorine expressed in mm of Hg is reproduced by
350 6
the equation Iog 10 P = 70.1 - · . Calculate the molar heat of evaporation of fluorine.
T
350.6
Soln. : log 10 P = 7.01 - -T-

350.6
.. log 10 P 1 = 7.01 - --
T,
350.6
And log 10 P 2 = 7.01 - - - .
Tz
FIRST LAW OF THERMODYNAMICS 175

350.6
Putting P 1 = 100, log 10 IOO = 7.01 - (i)
T1
350.6
and P 2 = 200; log 10200 = 7.01 - - - (ii)
T2
Solving (i) and (ii), we get T 1 = 69.98; T2 = 74.45.
Now, P 1 = 100 mm of Hg, when r 1 = 69.98 K
P2 = 200 mm of Hg, when T2 = 74.45 K.
From Clausius-Clapeyron equation,

In P2 = Le [ T2 - T1 ]
P1 R T1T2
i

or, In 2 = 8 .~ 4 x[~:::::~!:!!J .. Le= 6716.861.mole-I = 6.72 kJ.mole-1.


7440
9. The vapour pressure of a solid selenium is given by log P(mm) = - + 12.72
T
5390
and ofliquid selenium by log P (mm)= - + 8.63. Determine the triple point temperature
T
of selenium. [C.U. 2003]
Solo: : At triple point,

- 7440 + 12.78 = - 5390 + 8.63.


Tp Tp

7440 7440
- + = 8.63 - 12.78.
Tp Tp

Tp = 493.97 K = 220.82 °C.


10. The vapour pressure of n-butyl alcohol is given by the equation log P(mm)
= -2443.0/T + 9.136. Calculate the heat of vaporization per mole of the liquid at the normal
boiling point, 117 °C. [C.U. 2006]
Solo. : According to Clausius-Clapeyron equation for I ~ v transition,
dlnP dH
-;;:r- = RT 2 .

. dH
.. In P = - - +constant
RT
dH const.
or, log p = - 2. 303RT + 2. 303

f:t.H
By the problem, = 2443.
2.303x R
.. f:t.H = 2.303 x 2443 x 8.314 = 46776.47 J.mole- 1
= 46.78 kJ.mole- 1.
176 PROBLEMS ON PHYSICAL CHEMISTRY

11. Calculate the change in pressure required to change the freezing point of water
by l °C. At 0 °C the heat of fusion of ice is 335.5 J.gm- 1. The density of water is 0.9998
gm.cm-3 and the density of ice is 0.9168 gm.cm-3. [V.U. 2003]

Hints : dP = Afi
dT T(Vw - Vice)

335.5 I
dT = 1°C, tiH = 335.5 J/gm = lOl.3 25
l.atm.gm-

T = 273.15K, Pw = 0.9998 gm/cc = 0.9998 x I03gm/I


= 0.9168 gm/cc = 0.9168 x I0 3 gm/I

335.5
:. dP = 1
(1
) = -133.87 atm.
IOl.325x273.15x - - --
Pw pice
Thermochemistry
1. Given the following heats of reaction at 25 °C,
C 2H4 (g) + 30 2(g) = 2C0 2(g) + 2Hz0(1) Afi = -337.3 kcal.
2Hi(g) + Oz(g) = 2Hz0(1); liH = - 136.6 k cal.
2CzH6 (g) + 70i(g) = 4C0z(g) + 6Hz0(1); Afi = -745.6 kcal.
Calculate the liH for the reaction,
CzH4(g) + Hz(g) = CzH6 (g) at 25 °C.
Soln. : By the problem,
2CzHig) + 60 2 (g)= 4C0z(g) + 4Hz0(1); Afi = --674.6 kcal
2Hz(g) + Oz(g) = 2Hz0(1); Afi = -136.6 kcal
4C0z(g) + 6Hz0(1) = 2CzH6 (g) + 70z(g); Afi = 745.6 kcal.
Adding : 2CzH4 (g) + 2Hz(g) = 2CzH6(g); Afi = 65.6 kcal.
:. CzH4 (g) + Hz(g) = CzH6(g), :. !iH = 32.8 kcal.
Hence, the heat of hydrogenation of C2H4 is + 32.8 kcal, i.e., 137.24 kJ.
2. Calculate the heat of formation of cane sugar from the following data :
(a) C + Oz = COz ; Afi = -94 000 cal
(b) 2Hz + Oz = 2Hz0 ; Afi = -136 000 cal
(c) C 1zHzz0 11 + 120z = 12COz + 11Hz0 ; Afi = -1350000 cal.
Sofo. : By the problem,
I 2C(s) + l 20z(g) = l 2C02 (g); Afi = -1128 kcal.

211 .
x [2Hz(g) + Oz(g)J = 1IHzO(I); Afi = -748 kcal.

12C0 2(g) + l IHzO(I) = C 12 H2z0 11 + 1202(g); liH = 1350 kcal.


FIRST LAW OF THERMODYNAMICS 177

= =
Adding, l 2C(s) + 11 H 2(g) + .!..!..o2(g) C 12 H220 11 ; !:ill -526 kcal.
2
Hence, the heat. of formation of cane sugar· is - 526 kcal = -2200.78 kJ,
3. Heat of solutions of l mole of KC! in 200 moles of water under l atm pressure
is 4339 cal at 21 °C and 4260 cal at 23 °C. Determine the heat of solution at 25 °C. What
are the assumptions needed? •
Soln. : If !:ili2 and !:ili1 are heat of reaction at T2 and T 1 respectively and ACp is
difference between heat capacities of products and reactants, by Kirchhoff equation,
2
!:ili2 = !:ili1 + JACpdT.
I

!:ili2 = !:ili1 + ACp (T2 - T1>·


Here, basic assumption is that ACp is constant of temperature.
By the problem, !:ili2 = 4339 cal, T2 = 21 °C = 294.15 K
!:ili1 = 4260 cal, T1 = 23 °C = 296.15 K.
:·. ACp =-39.5 cal.K- 1•
Hence, !:ill at 25 °C = !:ili298 .15 = !:ili296.15 + ACp(298.15 - 296.15) K
= 4260 cal - 39.5 cal. K- 1 x 2 4181 cal. =
4. Both cis- and trans-azobenzene can be reduced to aniline by SnCI2 and HCI. When
400mlofethanolic of0.1995(M) SnCl2 and 1.085(M) HCl are added to 100 ml of0.2426(M)
trans-azobenzene in a calorimeter, the temperature rise was 5.405 °C. When a similar
treatment was given to 100 ml of 0.2423 (M} cis-azobenzene in the same calorimeter, the
temperature was found to rise by 6.38 '°C. The heat capacities of the solution were determined
by electrical heating. Passage of 0.69 A at 48.0 volts for 190 sec produced a rise of
4.65 °C. Calculate the heat of conversion of cis- to trans-azobenzene.
Solo. : By the problem, if x moles of cis- or trans-azobenzene in 100 cc are added
to 400 ml of ethanolic solution of 0.19?5 (M) SnCl 2 and l .085 (M) HCl, then in both the
cases the final product is aniline. The given solutions of SnCl2 and HCI are heated by 0.69
A current at 48 volts for 190 sec to lead to a temperature rise of 4.65 °C. So, water equivalent
of the solution is

JL, where AT = rise in temperature


AT ·
Q = heat supplied
= iVt
= 0.69 amp x 48 volts x 190 sec = 6292.8 J.
8
.. water equivalent = 6292
4.65
· = 1353.29 J.K-1.

Again, heat generated is ( 1353.29 x 5.405) when l 00 cc of 0.02426 moles of trans-


=
azobenzene is added. [100 ml 0.2426 molar 'ft.02426 moles]
f b .. 1353.29X 5.405
Hence, AU & •
ior conversion o trans-azo enzene to am 1me
1..111 =- - -0.-02426
---
= -301505.87 J.lllol-1.
Thus, !:ilitrans = 301505.87 J.moJ- 1.
178 PROBL~MS ON PHYSICAL CHEMISTRY

Similarly, AHcis = 1353. 29 x 6.38


. 0.02423
= 356334.72 J.mol- 1•

. . heat of conversion from cis- to trans-azobenzene


= llf{cis~trans
= wtrans - llf{cis
= (301505.87 - 356334.72) J.mo1- 1
= -54828.85 J.mo1- 1
= -54.83 k,J.moI- 1•
5. A solution of composition CuC1 2. IOH20 is diluted with a mol of water, the heat
5023
generated is given by ax . The heat of solution of CuC1 2 in 600 moles of water
a+21.24 ·
is -11.08 kcal. Find the heat of solution of anhydrous CuC1 2 in IO mol of water. How
much heat will be evolved. if a so1ution of CuCl 2. 600H 20 be infinitely diluted?
Solo. : By the problem, if CuCl 2. IOH20 is converted to CuCl 2.600H20 by addition
. 590x5023 '
of 590 moles of water, the heat change 1s Q = - = - 4.848 kcal.
590+21.24
Again, CuCl 2 + 600Hp = CuC1 2.600H20; !!H = -l l.08. kcal; (i)
CuC1 2. IOH 20 + 590H20 = CuCl 2.600H20; 611 = - 4.848 kcal. (ii) .
(iii)
' And adding (iii) and (i), we get
CuCI 2 + lOHp = CuCl 2.JOH20; !!H. = (-ll.08 + 4.848) kcal= --6.232 kcal
= -26.07 kJ.
CuCI 2. IOHp + 590Hp = CuC1 2.600Hp, llH = - 4848 cal.
But, CuCl 2. JOH 20 + aq. (= infinity water) = CuCl 2 .aq.~ = -5023 cal.
. h ·- . fi . Q a x 5023
S mce, w en a = m m1ty, = - = ax 5023 = -5023 cal.
a+ 21. 24 a
.. CuCl 2.600H20 + aq. = CuCI 2.aq; /'JI= (-5023 + 4848) = -175 cal= -732.2 J.
6. Calculate the heat of formation of AlCl 3 from the following data :

llH
2Al + 6HCI (aq.) = 2 AICl 3.aq + 3H2 -1003.2 kJ
H2 + Cl 2 = 2HCI -184.0 kJ
HCI + aq = H~l.aq -72.40 kJ
AICl 3 + aq = AIC1 3.aq -321.5 kJ
[Vikram Univ. 1978]
Soln. : By the problem.'._
!lH in kJ
2AI + 6HCl(aq.) = 2A1Cllaq.) + 3H2 -1003.2
3H2 + 3Cl 2 = 6HC1 -(184 x 3) = -552
FIRST LAW OF THERMODYNAMICS 179

6HCI + aq. = 6HCl(aq) -72.4 x 6 = - 434.4


2AICl 3(aq.) = 2AICI 3 + aq 321.5 x 2 = 643

Adding, 2AI + 3Cl 2 = 2AICI 3 -1346.'6 kJ.

3 -1346.6 kJ.
. . Al + -CI 2 = AICI 3 2
2
.. Ml =- 673.3 k.J.mole-1•
7. Calculate the heat of formation (6}{) of solid AI 2CI 6 from the following data :
3 .,
Al(s) + 3HCl(aq.) = AIC1 3(aq) +-H2 (g) Ml= -127.0 kcal
. 2
H 2 (g) + CI 2 (g) = 2HCl(g) Ml = - 44.0 kcal
HCl(g) + aq. = HCI(aq.) Ml= -17.5"kcal
AI 2CI6 (s) + aq. = 2Al91 3(aq.) Ml = -155.8 kcal
[V.U. 2004]
Solo. : 2Al(s) + 6HC1 (aq) = 2A1Cl 3(aq) +3H 2(g) !!.H = -254.0 kcal
3H2 (g) + 3CI 2 (g) = 6HCl(g) Ml= -132.0 kcal
6HCl(g) + aq. = 6HCl(aq) Ml= -105.0 kcal
2AlC1 3(aq.) = Al 2Cl6(s) +aq. Ml = 155.8 kcal
Adding, 2Al(s) + 3Cl 2 (g) = Al 2CI 6 (s) !!.H = -335.2 kcal.
·,
8. 8 gm of NH4 N0 3 were dissolved in 400 gm of water taken in a calorimeter (water
equivalent of the calorimeter 50). A fall in temperature of 1.3 °C was noted. Calculate the
enthalpy of solution.
Solo. : 8 gm NH4 N0 3 = 0.1 gm-mole NH4 N0 3
Assuming heat capacity of water remains same after addition of NH4 N0 3, total heat
capacity of calorimeter and solution = (400 + 50) cal.K- 1 = 450 caI.K- 1.
This leads to the decrease in temperature by 1.3 °C.
:. total heat change = -450 cal.K- 1 x 1.3K = -585 cal.
The heat change is due to dissolution of 0:1 gm.mole NH4N0 3 .
Hence, enthalpy of solution =heat change for 1 mole
= -585 x 10 cal.mole-I ,
=-5850 cal.mole-i'
= -24476.4 J.mole- 1 = -24.48 J.mole-1.
9. A sample of sucrose C 12H 22 0 11 weighing0.1265 gm is burned in a bomb calorimeter.
After the reaction is over, it is found that to produce an equal temperature increase
electrically, 2082.3 J must be expended. Calculate the heat of combustion of sucrose, if
the "temperatrue increment is 1.743 °C. What is the heat capacity of calo.rimeter and its
contents?
Solo. : Molecular weight of C 12 H 2p 11 = 342 gm
0.1265 gm releases 2082.3 J heat
2082.3
. . 342 gm release - - x 342 ::= 5629.62 kJ heat.
. 0.1265
180 PROBLEMS ON PHYSICAL CHEMISTRY
=
. . heat of combustion 5629.62 kJ,
Now, 2082.35 J raises the temperature by) .743 °C.
2082 3
Thus l °C is raised by · J = 1194.66 J.deg- 1.
I. 743°C
Thus, heat capacity of calorimeter = 1194.66 J per degree.
10. Calculate the enthalpy change at 1500 K for the reaction 2Hz(g) + Oz(g) 2Hz0(g) =
for the following data.
MI300 K =
-115.0 kcal and the values of Cp (cal.deg-I.mole-I) are for
Hz(g) = 6.95 - 0.002 T
Oz(g) = 6.10 + 0.0032 T
HzO(g) = 7.19 + 0.0024 T [C.U. 1994]
1500
Soln. : fJ..H 1500 = Ml 300 + f l!.C pdT
300

l!.Cp = 2CP. HzO - 2CP. Hz -Cp. oz


= (2 x 7.19 - 2 x 6.95 - 6.10) + (2 x 0.0024 + 2 x 0.0002 - 0.0032)T
= -5 .62 + 0.002 T.
Given, M/300 = -115.0 kcal = -115000 cal.
1500 1500
Hence, Aff 1500 = -ll5000 cal - J5.62dTcal· + J0.002TdTcal
300 300

. 0 002
= -115000 cal - 6.52 x 1200 cal+ - · - x 1800 x 1200 cal
7
= -ll9584 cal = - 119.58 kcal.
11. The heat of neutralization of HCN by NaOH is 2900-caf. Calculate the heat of
ionization of I mole of HCN; given that 13800 cal as heat of reaction of ff++ oH- = HzO.
[B.U. 1995]
Solo. : By the problem,
HCN = H+ + cN- Mi
x
ff+ + OH- == HzO Y = -13800 cal
HCN + oH- = HzO + cN- Z = -2900 cal
By Hess's law, X + Y =Z
x = z - y = -2900 + 13800
=
-10900 cal 45.6 kJ. =-
12. The heat of fonnation of NH3(g) at constant pressure is 46.1 kJ .mole- 1 at
27 °C. Cttlculate the heat of formation of NH 3(g) at constant volume assuming ideal
behaviour. How would the value differ if the gases under consideration were van der Waals
gas? [B.U. 1990]

Solo. : The reaction is 2"l Nz + 23 Hz = NH3


FIRST LAW OF THERMODYNAMICS 181

= l - .!. - ~ = -1
2 2
=t:.U + RT&. = t:.U - RT
46100 = !lU - 8.314 x 300
llU = 48594.2 J = 48.6 kJ.
Calculation of heat of formation at consu~nt volume from the corresponding value at
constant pressure consists of the following two steps :
reactant (P, V. 1) !lU1 product (P, V', 1) llU2 product (P', V, 1)
-+ -+
llU = llU1 + llU2 = Aff - PllV + tJ.U2
= Aff - RT!ln + llU2.
=
For ideal gas, llU2 0 as U is independent of volume, but for van der Waals gas it
depends on V and so llU2 exists. Thus, !lU will change depending how U depends on V.
13. Molar heat capacities of gases are given (in calorie) by Cp(N 2) = 6.5 x 10-3 r,
Cp(H2) = 6.5 + 9 x I0-4T and Cp(NH3) = 8.0 + 7x Io-4T. Calculate Aff (300 °K) for
the reaction N2(g) + 3Hi(g) = 2NH3(g), when MI (0 °K) = -19 kcal. (B.U. 1992]
Solo. : Given, Cp(N 2 ) = 6.5 x 10-3r
Cp(Hz) = 6.5 + 9 x to-4T
Cp(NH3) = 8.0 + 7 x to-4T
Aff(O °K) = -19 kcal = -19000 cal.
The reaction is : N2(g) + 3Hz(g) = 2 NH3(g)
300

Aff300 = Aff0 + f llCpdT


0
300
= -19000 + f[2cp{NH 3 )-3Cp(H 2 )-Cp(N 2 )]dT
0

300 300
=-19000 + f-3.5dT - f1.8xl0- TdT
3

0 0

7.8x10-3
= -19000 - 3.5 x 300 -
2
x 3002 = -20401 cal = -20.401 kcal

14. Calculate the latent heat of fusion of ice at -20 °C, if that at O °C is - 1440 cal
per mole, the specific heat of ice being 8.7 cal.mole- 1• (C.U. 1998]
Solo. : The phase transition is ice-+water
T1 =0°C = 273.15 K, T2 = -20°C =253.15 K
Aff1 = 1440 cal.mole-I, CP,ice = 8.7 cal.mole- 1.
253.15
Thus, Aff2 = Aff1 + J(Cp.water-Cp.ice)dT
273.15
= 1440 + (18 - 8.7) (253.15 -- 273.15) = 1254 cal.mole-I.
182 PROBLEMS ON PHYSICAL CHEMISTRY

15. Calculate bond energy of 0-H bond in HzO(g) from the following data :
l
Hz(g) + 20z(g) ~ HzO(g) l:!.H° = -241.814 kJ/mole

Hz(g) ~ 2H(g) l:!.H° = 435.994 kJ/mole


Oz(g) ~ 20(g) l:!.H° = 498.34 kJ/mole
Solo. : By the problem,

Af/0 = 241.814 kJ/mole

Hz(g) ~ 2H(g) Af/0 = 435.994 kJ/mole


l
2 Oz(g) ~ O(g) Af/0 = 249 .17 kJ/mole
Adding, HzO(g) ~ 2H(g) + O(g) Af/0 = 926.978 kJ/mole.
Thus, bond energy of 0-H bond
Af!O
= .--
2 = 463.49 kJ/mole.

16. The combination of Nz and Oz at 27 °C occurs :i.ccording to 2Nz + Oz = 2Nz0.


Ml= 39.4 kcal. What will be the heat of formation at 227 °C? Use the relation Cp =a
+ bT in calculation where C p is the molar heat capacity.
g a bxl03
Nz 6.52 1.25
Oz 6.15 3.1
NzO 7.0 · 7.1 [C.U. 1976]
Hints ; 2N 2 + Oz = 2N20
l:!.Cp = 2cP.N20 - 2CP.Nz - CP.Oz
= (2aNzO - 2aNz - a0 z) + (2bNzO - 2bNz - b 02 ) X l o-3T
= 5.19 + 8.6 x 10-3T.
500

l:!.H500 = 11H300 + f l:!.CpdT = 39.050 kcaL


300

:. Afl
1
= -39.050
-
2
= 19.525 kcal.

17. At 18 °C the heat of solution of anhydrous CuS0 4 in large volume of water is


-15800 cal/mole while that of CuS0 4.5HzO is -2750 cal/mole. Find at 18 °C the heat of
reaction CuS0 4 (s) + 5Hz0(1) = CuS0 4 .5HzO(s). [C.U. 1984]
Solo. : CuS04 (s) + aq. = CuS04 .aq. Ml= -15800 cal/mole (l)
CuS04 .5HzO(s) + aq. = CuS0 4 .aq. Ml = -2750 cal/mole (2)
Rearranging eqn. (2), CuS0 4 .aq. = CuS0 4 .5HzO(s) + aq. Ml = 2750 cal/mole (3)
Adding eqn. (l) and (3),
=
CuS04 (s) + 5Hz0(l) = CuS0 4 .5H20(s), Afl -13050 cal/mole.
FIRST LAW OF THERMODYNAMICS 183
18. Enthalpies of formation of COz(g) and HzO{I) under' standard conditions are
-94.8 and 68.3 kcal/mole respectively. If the enthalpy of combustion of acetaldehyde is
-279.07 kcal/mole, find its enthalpy of combustion. [Delhi University, B.Sc.,_ 1975]
Hints :

2COz(g) + 2Hz0{1) = CH 3CHO + i5 Oz 279·07 kcal

2C + 20z = 2C0z(g) -94·8 x 2 kcal


2Hz +Oz = 2Hz00) -68·3 x 2. kcal

. . 2C + 2Hz + i Oz - 47·13 kcal.

· 19.' •Calculate 0-0 bond s.trength. Given that


HzO(g) ~ 24(g) + O(g) m 0
= 930 kJ/mole
HzOz(g) ~ 2H(g) + 20(g) m = 1070 kJ/mole.
0
[C.U. 2006]
930.
Hints : 0-H bond strength =- kJ/mole.
2
930 .
0-0 bond strength = 1070 - 2 x - = 140 kJ/mole.
2
20. Calculate increase in vapour pressure of water per atmosphere rise in external
pressure at 10°C (vapour pressure of water at 10 °C in 9.2 mm). [V.U. 2002]
Soln. : Given pl = 9.2 mm, vm = 18 CC, T = 10 °C = 283.15 K
p Zt - p It = !1P = I atm
Vm11P
RT

P I atm x l8cc. mo1- 1 x10- 3 1I cc


or, In -2 = = 7.747 x 10-4
9.2 0.82061.atrn.deg.mol- 1 x283.15K
Hence, Pz = 9.207 mm
Miscellaneous
1. Let a perfect gas be forced isobarically through an orifice into an evacuated rigid
vessel until its pressure in the vessel rises to the isobaric value and the process stops. Show
that Tz = "(T1, where T2 is the final temperature of the gas in the vessel before it loses .
heat, and T 1 is the temperature before entering the vessel.
Soln. : From first law, Wad = nCp dT = nCv (Tz - T 1) (i)
Total work done = P1V1 - P0 V0
= P 1V1 [for expansion into evacuated vessel, P0 = O]
= nRT1. (ii)
.. nRT1 = nCv (Tz - T1)
c
or, [since cp - CV = R and _£_ = 'Y]
CV
184 PROBLEMS ON PHYSICAL. CHEM!STRY
or, YT1 = T2.
T2 = yT1 (Proved).
2. A thick-walled insulated chamber contains n; mol of He at a high pressure P;. It
is connected through a valve with a large and almost empty gas holder where the pressure
is maintained at a constant value Per very nearly atmospheric. The valve is opened slightly
and He flows slowly and adiabaticaliy into the gas holder until the pressure on the two
sides of the valve is equalised. Let nF be the final number of moles of He left in the chamber
and PF be the final pressure. Also let V;, VF and V' be initial, final molar energy of He
in the chamber and molar energy of helium in the gas holder, and h' V' + P0 V0 =
(V0 =molar vol. of He gas in the gas holder), TF and T' be the final temperatures of the
chamber and gas holder, and assume that He is an ideal gas with constant heat capacity.
Now, show that-
I

= (:~y
nF h; -Vi !!:L
(1)
n;
= h;-UF
(2)
nF

y-1 1- PF
= T{~)1
r.
(3) TF (4) T =--L
P;
I
'Y
1-(~ y
Solo. : Let there be a chamber fitted with an adiabatic jacket which contains ni moles
of He with internal energy Vi at temperature Ti. This is joined to a holder at nearly atmospheric
pressure P0 having very large volume V. The volume Vis so large that addition or removal
of finite gram-moles do not alter the volume. But if n moles of He are introduced. the
piston is slightly pushed and the work done is nP0 V0 , where V0 molar volume of He=
in holder.
Let the adiabatic en-
closure contain a valve at v,
' 0
temperature T;. the valve is
opened and the gas mo!- PF, TF, nF
ecule escapes to the holder (Final) T'
until the pressure P; be-
(Final temperature)
comes equal to PF· Let there
be nF moles of the gas after
exit of the gas from the
adiabatic enclosure to the
holder.
Initial number of moles = n;
Temperature =T;
Energy/mole = V;
Pressure = P;
Assuming very closely reversibility, we known that
y-1

TF = r{~ )
1
. where y= ~ [Proved (3)).
FIRST LAW OF THERMODYNAMICS 185

Now,

Assuming, the whole system to be isolated, the energy lost in the chamber is equal to the
energy gained in the holder.
Loss of energy in the chamber n;U; - nFUF =
Gain of energy in the holder = (n; - nF)U' + (ni - nF)P0 V0
:. npi - nFUF = (n; - nF)U' + (n; - nF) P0 V0 .
By interchanging,
~~-~U+~~=~~-~U+~~
or, = nFUF - n~U' + P0 V0 )
npi - ni(U' +"P0V0 )
or, ni(U; - h'i) =n,AC.:F - h';) [:. h' = U' + P0 Vo].
!!:i_ = UF-hi =hi-VF
.. . nF U; -hi hi-U;

or, !!L = hi - U; [Proved (1)].


n; ~-UF

But, for monatomic ideal gas, U = ~RT/mole


2
. 5
h = R77mole.
2
~Rr' -~RT.
.. nF = 2 2 I
5 I 3 =
2RT -2RTF

On cross-multiplying,
nF(5T' - 3TF) =n;(5T' - 3T;) .
.. ~T' = nFTF -n;T;
3 nF -n;

= r{nF·T-n;)
yT,
nF-ni

:. T'
= ~(T-~)
1- !!:.L
nF
186 PROBLEMS ON PHYSICAL CHEMISTRY

Dividing and multiplying by


( PPF,· )y,

T•::;
;[(~ f -( :: )~l (~ l I x I

1-(:~r (~ r

= f[(~H = 7[,-(~)J [Proved (4)).

(~Y-1 1-(~Y
3. A polytropic process is one that obeys the relation PV'1 c, where n and c are =
constants. Derive a general expression for W, Q, l'::.U for such a change from state I to state
II carried out reversibly. Show that they provide familiar relation when n = 0, I, oo, y.
2 2
Solo. : W = - JPdV= -n JC.~~
. I I

= -f cv-ndv

= (n ~I) [ V2:_1·- V1:_1]


= I [P2V2n P1V1n]
(n -1) v/-1 - vln-l

I
= (n - I) [RTz ~ RT1] [·: PV == R1]

_ .!!.!J._
(n-I)
[TT
2

1
-I]= RT1 [P2V2
n-1 P1V1
-l].
FIRST LAW OF THERMODYNAMICS 187
Now, Q = !::.U- W

= C (T -
v 2 T)I + n-1
RT1 [1- P2 Vz]
P1V1

= !!Ii
n-1 {(l- P2V2)+~cv(T2 -1)}
P1V1 R T1
But P2 V2 = RT2, P 1V1 = RT1
1-(RT1-RT2)J+cv(Tz-T1) = [RTi (1-r2)-cv(r1-T2)]
=
n-1
[-
n-1 T 1

= .!!!i[(1-T2)-
n- l T
Cv(n-l)(1-T2)]
R T1
= El[1-1'2][1- Cv (n-1)]
n- l T1 R 1

= :~\ [ l - ~~ ] ( l - ~ =~) = :~\ ~ =~ [l - ~~ ]


= RT1 y-n[ 1 _P2 V2 ]·
n-1 y-1 P1V1

PV1 = C
aP nP
=--
av v
aP 0 . b .
When n = 0 , av = , !SO anc process (P = constant)

ap
n = l , -;- = - -p , isot
. h l
erma process (T = constant)
uV V
ap . h .
n = oo,
av = oo,
- 1soc one process (V = constant)
n p ' revers!'bl e
aP = -y v
= y, av act'1ab at1c
. process (Q = 0)

n =y
n = oo

v
188 PROBLEMS ON PHYSICAL CHEMISTRY

4. For reversible polytropic process described by the general relation PV n =C, derive
the following expression for work and heat.

W= RT1 {( P2 )n:t _I}


n-1 P1

Q =
(n-y}RT1
(n - l)(y-1) {(~: f-l
Hints : Derive the general formula of work and heat,

W= RT1 [P2 V2
n- l P1V1
_ 1J.

Then put V1 = ( PiC )~ and V2 = (PC2 )~ to obtain the above expression.

Q RT1 ) ( "f- n)
= (n-l y-1
[1- T2] (as found in the last problem)
T1

putting V1 = ( ~ Y
v, =(~);

I).
1

= (n -y)RT1 {( Pp2 ) ' ; -


(n-l)(y-1) 1 .

5. 3 moles of an ideal gas at 300 K is isothermally expanded to 4 times its initial


volume and then heated against constant volume till the pressure is equal to its initial value.

The total heat absorbed in the process is 75 kJ. Calculate ......E.. =y.
c
CV
Solo. : Let V1 = V
. Given, T = 300 K
P 2 x 4V = 3R x 300
P 1 x 4 V = 3R x T '.
v
!i -
300
"p1-T'.
FIRST LAV{ OF THERMODYNAMICS 189
But, P2 x 4V =P 1 x V.

.. !l = 4.
P2
.. T' = 300 x 4 = 1200 K.
Q =QI+ Q2
Q 1 = -W1
v2 = 3 x 300 x 8.314 In 4 = 10373.08619 J.
=nRTln-
Vi
By the problem, W2 =0
Q2 = nCv (T' - = 3 x Cv (1200 -
1) 300) = 2700 Cv
. . 2700 CV + 10373.08 = 75000
••. CV = 23.935 J = 23.94 J.
.. cP = cv + R = 32.249J.

.. r = cp = 1.35.
CV
6. A gas at 500 K and 1 atm is passed at the rate of I00 cc/sec into an apparatus where
it flows over an electrically heated wire of resistance 200 ohms. When the heating current
is 0.06 A, the gas leaves the apparatus at 510 K. Calculate CP and Cv.
Solo. : Let V0 be the volume at NTP.

P1V1 = PoVo
T1 To

lxlOO . lxV0
or,
500 = 273 .

V0 = 273x
500
100 .
= 54.6 cc = 0.0546 htre.
Heat Q = i2Rt = (0.06)2 x 200 X IJ

72
= 0.72 J = 4.184
O. cal = 0.172 cal.
.
0.172 cal heat leads to an increasse of 10 °C for 0.0546 litre gas.

. . O. l 72 cal leads to an increase of 1 °C for 0.0546 litre.


10
0.172 •
or, cal leads to an increase of 1 °C for 1 litre.
lOx0.0546

or, O. l 72 x 22.4 cal leads to an increase of 1 °C for 22.4 litres.


lOx0.0546
0.172 X 22.4 -I -1
.. CP =
IOx0.0546
cal.deg .mole . = 7.0564 cal.deg-I .mole-I .
= 29.52402 J.deg-1.moJe-1
•. Cv = (29.5240 - 8.314) J,deg- 1.mole-l
=. 21.21 J.deg-1.mole-1.
190 PROBLEMS ON PHYSICAL CHEMISTRY

7. A gas at 273 °C and I atm is passed at the rate of 80 cc/sec into an apparatus where
it flows over an electrically heated wire of resistance 150 ohms. When the heating current

is 0.04 A, the gas leaves the apparatus at 278 °C and I atm. Calculate CP and Cv assuming
the gas to be ideal.

Solo. :
P1V1 Pa Va
=--,or,
Ix 80 = Ix Va
T, Ta 546 273

80x 273
. . V0 = = 40 cc = 0.04 litre.
546
The volume is 40 cc at N'TP.
The amount of heat is i2Rt joule
= (0.04 2 x 150 x I) J = 24 x 10-2 J = 0.05736 cal.
0.05736
- - - cal heat requires for rise in I °C for 0.04 litre
5
0.05736
or, - - - cal heat requires for rise in l °C for I litre
5 x0.04

or, 0.05736x22.4 ca1 heat requires


. "1or nse
. m. 1oc"1or 224 ·
. 11tre.
5x0.04
0.05736x22.4 Id _1 . _1
•• CP =
5
x O.
04
ca . eg .i;no1e
...
= 6.424 cal.deg- 1.moJe- 1 = 26.88 J.deg-1.mole-1•
:. Cv = (6.424 - l.987) cal.deg-1.mole- 1 ,
= 4.437 cal.deg- 1.mole- 1 = 18.564 J.deg-1.mole-1•
8. A bottle at 27 °C contains an ideal gas at 126.9 kPa pressure. The gas expands
adiabatically after the removal of rubber stopper against the constant pressure of the
atmosphere at that time, IO 1.9 kPa. Sorrie gas is expelled and the stopper is quickly replaced
when the pressure of the bottle equals atmospheric pressure. The gas which is cooled in
the adiabatic expansion slowly warms up to 27 °C.. The final pressure now becomes 111.07
kPa. Calculate y. . .
Solo. : This is Clement-Desorms method for determination of y.
Let the initial temperature and pressure be T1 and P 1 respectively. Let the initial volume
be V 1, and it expands (or contracts) to V2 and the final volume be V3. Since the bottle
=
volume is fixed, V2 V3. Number of moles expelled or entered is assumed to be negligible.
.. PjV1 =P3V3
V2 = V3
Ti = T3
Assuming the change to be reversible,
,.,
FIRST LAW OF THERMODYNAMICS 191

p
log-1
Pz
.. r= --p-·
log-1
P3
By the problem, P1 = 126.9 kPa
P2 = 101.9 kPa
P3 = 111.07 kPa.
lo 12.6. 9
g 101.9 logl.245338567
r= = --"'------- = 0.0952874 = 1.7 4.
lo .126. 9 log l.34048257 0. 0546315
g 11l.9

Assu~ing the change to be irreversible,


W= -Pe/V2• - V1) = nCv (T2 - T 1)
Pz(Vz - VI) = nCV (Tl - Tz)

R.(T2 P2 - T1 P2) = cv <T1 .:... Tz)


P2 P1

.. R(T -T ;~)2 1 = CJ1 - CvT2

• or, T2 (R + Cy) = T1 ( Cv + R ;~ ) .•
P2
Cv+ R -
.. Tz = . P1 T
Cv +R I
,· '

Tl = T3; V3 = Vz
P1 = 126.9 kPa; P2 = 101:9 kPa; P3 = 111.07 kPa.

P3 =
Tj

.. CV =
R
192 PROBLEMS ON PHYSICAL CHEMISTRY

IOl. _ 111.07xl01.9
9
Thus, _cv = _______12=6.-.::.9'--- = 101.9-89.1886 = ---
12.7114
= 1.39.
R 111.07-101.9 9.17 9.17

CV +l
r = .JL_c
_v
= 2.39 = 1.12.
1.39
R
9. Let the volume of the bottle in Problem 8 be 20 I. Calculate the amount of the
ideal gas that is expelled if its molecular weight is 28. [Ans. 3.46 gm]
10. Consider the case similar to Problem 8. But now let the pressure be 151.2 kPa
and the abnospheric pressure be 100.8 kPa. What will be the final pressure, if 1.66? r=
[Ans. 118.4 kPa]
3
11. The molar specific heat at constant volume of an ideal gas is 2 R. Suppose that
I mole of such a gas is subjected to a cyclic quasistatic process which appeared as a circle
in the P vs. V diagram (see fig.). Show that-
(i) The net work done by the gas in one cycle is about 3.14 J.
(ii) The internal energy difference of the gas between states, C and A is about 600 J.
(iii) The heat absorbed by the gas in going from A to C via ABC is 1107 J.
Calculate Q, !!.U and W for path CDA.

B
3
pj
A
'\ c-
2

/
0 l
"' v~
2
D

103 cc
Solo. : (i) Net work done by the gas = - area of the ellipse ABCD = -rtab
= - 3.14 x 106 ~x
cm
103 cc

= -3.14 x 106 x 103 ergs =-314 J.


(II") TA -- -R-,
PAVA TC = PcVc
R
Hence, !!.UcA = Uc - VA = Cy(Tc - TA)

= ~R(PcVc - PAVA)
2 R R
FIRST LAW OF THERMODYNAMICS 193

=~ x 2 x 106 (3 - 1) x 1o3 dyne


-:--z-CC
2 cm

=6 x ergs = 600
109 J.
(iii) Work done for the path ABC = -(area under ABC)
= -(area of half ellipse + area of 4 boxes)
1 . .
= -(- x 314 + 4 x 100) J
2
(since area of each box = 106 x 103 ergs)
=- 557 J.
Thus. Q for the path ABC= !lUcA - WABC = (600 + 557) J = 1157 J.
For return path CDA, !lU = - 600 J.
W = (area of 4 boxes - area of half ellipse)
= (400 - 157) J = 243 J.
Q = -600 - 243 = -843 J.
12. Show that Cv and Cp of a perfect gas are independent of both volume and pressure.
Do they depend on temperature?

Solo. : Cp = ( 0aT") p • cv = (au)


oT v

since for ideal gas, (au)


av r
= o.

Similarly. (
0
0; l = [a~(~~\L~ [a~(~~)Jv = o,
since for ideal gas,
.
(au)
av r
= 0.

[ ~(oH)
oT aP r
] _o p - •

since for ideal gas, ( oH)


oP r
= O.

Similarly, (oCp)
oP r
= o.
Since U and H depend on temperature, Cp and Cv will depend on temperature.

13. Find the value of (oU)


av r
and (oH) for the gas obeying PV =RT+ Pb and hence
oP r .
comment which one provides better thermodynamic criteria for describin~ an ideal gas.

Prob. Phv. Chem.~n


194 PROBLEMS ON PHYSICAL CHEMISTRY

Solo. : ( au) r = T(aP)


av aT v
_P
For the gas obeying PV = RT + Pb,

R
V-b
Hence,
RT - p =p - p = 0.
V-b
Now, (aH)
aP T
=v- T (av)
aT
.p

:. (~;)T = ;.

Hence, (aH)
aP T
=v - T (av)
aT
= RT + b -
pp
RT = b.
p

Thus, (aH).
aP T
is zero only for ideal gas but (au)
av T
is zero not only for ideal gas

but also for a gas having little deviation from ideal behaviour. ~ence, ( aH) = 0 is better
' aP T
tbennodynam. ic criteria for an ideal gas than
. . ~·
(au)
av r
= O.
14. What is perpetual motion machine of first kind? Does it exist? Is the orbital motion
of elecctron around the nucleus an example of perpetual motion?
Ans. : A machine which creates its own energy itself for doing work is called perpetual
motion machine of I st kind.
It does not exist as it contradicts the I st law of thermodynamics which states that energy
cannot be created or destroyed .
. Orbital motion of electron arround the nucleus is due to the Coulomb's force of attraction
between nucleus and electron. So, it is not an example of perpetual motion.
15. Write the thermodynamic e.quation of state .

. 16. Show that


av r = 0 for an ideal gas, given that (au}
(aH) av r = 0.
Ans. : H = U + PV
or, (~~)r = (~~l+(~~)rv+ P
FIRST LAW OF THERMODYNAMICS 195

= - (avaP) r
. (au)
V + P [given, · -
av r
= O]
For ideal gas, PV = RT.
:. ( ~~ )T =- ~~ =- ~
Hence, (aH)
av r
= -P + P= 0.

17. A perfact gas is allowed to expand adiabatically against vacuum to a finite volume
and then it is compressed reversibly and adiabatically to its initial volume. Is this a cyclic
process? Indicate the process in P-V diagram.
Ans. : State 1~ State 2, adiabatic expansion against vacuum.
Thus, Q = o. w = 0, au = 0.
Since the gas is ideal, T 1 = T2 .
By the problem,
Adiabatic Adiabatic
(Pl' VI, T1) (Pz. Vz. T1) (P', VI, T')
expansion against vacuum Rev. contraction
2 3
1
Now, P'V1 = P2 V'J.
and P 1V 1 = P2 V2. State 1~ (P 1, V 1, T1)
2
~~ y-1
State 2~ (P2, V2, T 1)
:. P' = P1 (
State 3~ (P', V1, T')

Since y > I,
V2 > Vi' hence P' > P 1• v-
.Thus, the process is. not cyclic, since state 3 is not same as state 1.
...

18. When will be l1H = aU? ·


Solo. : For a reaction at constant pressure involving ideal gases if l1n (difference of
total number of moles of products and reactants) is zero, then l1H au. =
m = au + RT11n,
if /1n = o. m = au.
Also for solid or liquid if any process leads to the change of temperature only, then
l1H = au.
19. In which case will the efficiency of Carnot cycle be higher : when the temperature
of the hot reservoir is increased by aT or when the temperature of the sink is decreased
by the same amount.
Solo. : Let the temperature of the high-temperature reservoir be T and that of 1ow-
T - T' x
temperature reservoir be T ', the efficiency, 71 =- -=
T T
196 PROBLEMS ON PHYSICAL CHEMISTRY

x-i-!lT,
If T is increased by !lT, the efficiency, 77 1 = T + !lT- T' =--~
T+!lT T+!lT
and when T' is decreased by t:,.T, the efficiency

T-(T'-t:,.T) T-T'+t:,.T x+flt


772 = T
= T
= - T
-.
Obviously, 772 > 17 1• Hence, the efficiency is higher when the temperature of the sink
is decreased by !lT.
For further problems and short questions, consult the book on Physical Chemistrry
(Vol. I) by Dr S. Pahari.
CHAPTER6

SECOND LAW OF THERMODYNAMICS

Required Formulae

1. Entropy change 65 = Qrev


T

2. 65mix (per mole) =-R LX; Jn X;


Xi= mole fraction of ith component in the mixture.
3. dU = TdS - PdV
dH = TdS + VdP
dF (or dA) =-PdV - SdT
dG =VdP - SdT.

4. !l.G = nRT In p 2 (for ideal gas)


Pi
= V!l.P (for liquid)

=nRT In pP12 + n(b-_!!_)<P2


RT
- P 1) (for van der Waals gas).

-
s. 65phase change - TLf or TLe
6. <J(!l.G IT)) =_Aff
( (JT p T2
!l.G =A - BTlnT + lc.T, if Aff = A + BT.

7. 65 = JnCp dT
T
-J nR dPp. (for ideal gas)

= f nCv TdT + fnRV


dV .
(for ideal gas).

197
198 PROBLEMS ON PHYSICAL CHEMISTRY

Short Questions
1. A body at constant heat capacity CP at a temperature T 1 is put in contact with a
reservoir at a higher temperature Tz. The body comes to equilibrium with the reservoir
at constant pressure. Show that the entropy change of the universe is ·

nCP { T1 ;2T2 - In( l + T1 ;2T2 ) } .


Solo. : Let there be n moles with constant CP.
T2
Afsy = nCP In-.
T1
Reservoir remains at constant temperature T2.
Aflsy = nCP (T2 - T1)
!J.HR = -nC/T2 - T1) = nCP (T1 - T2)
Af!R 1 2 nCp(T -T )
f)SR = - - = ----~
T2 T2

Afun = Afsy + t.SR = -nC In..!!_ + nC (T1 -Ti)


P Ti P Ti

= -nC In (1 + (r1 -Ti))\ nC (T, -Ti)


P , T2 P T2

= ncp[T1;2T2 -10(1+ T1;2T2 )].


2. Show that

a~) =
( aP v
~Cv
aT
and (as)
av p
= .EL
aVT

Solo. : We know that, dS = Cv dT + ( aP) dV for 1 mole of gas.


T oT v

or, dS =C dT + a dV
v T ~

Again, dS = cP dT
T
- (av)oT p
dP

dT
or, dS = CPT - aVdP.
SECOND LAW OF THERMODYNAMICS . ~: 199

3. The compressibility factor for a van der Waals gas is given by

:~ = l + (b- ;T) :T. Show that (~;)T = b- ~;.


Solo. : By the problem,

PV = RT + ( b- ;T )p

or, V
• ·.
= RT
P
+(b-_!!_)
RT
R a
-+--·
P RT 2 .

.. T( ~~ t = 1; + ;T .
But (aH) = v - T(av) .
aP T aT p
Since, dH = TdS + VdP,

.. (~;)T = T(~~)T +' V

= v - T(av) [byMaxwellrelation.(as) =-(av) ]


aT_ p aP T aT p

Hence, (~;)T = R: + (b- R~ )- R: - ;T =b- !~·


4. Define coefficient of compressibility at constant entropy.

Show that' Ps = _!_pT·


"{

Solo. : We know that PT = _ _!__(av) and Ps = _ _!__(av) .


v aP T v aP s
Now, dS = CP dT
T
- a..VdP (i)

dT a..
= CVT + p;-dV. (ii)

(~~)s = __ p~-~-v [putting dS = 0 in eqn. (i) and eqn. (ii)]

(~~)s
Cp
and
= a.TV
200 PROBLEMS ON PHYSICAL CHEMISTRY

(~~)s = (~~)J~~)s = _ p~~v. a;: =-Pr~


. Cp
[smce, - = y]
Cv

or, _ _!_(av)
V aP s = f3s = Pr.
y

a27l3sV
5. Show that, Cv = ------
Pr(Pr -Ps)'
Soln. : We know that, f3s =- ..!.. (av) = Pr
V aP s y

Now,

Cp-Cv
= = Cv.
Cp -I
Cv

a 2 TV
6. Sho.w that CP = .
Pr -Ps
Hints : Proceed as Problem 5 and use f3s = Pr
'Y

7. Show that (aF)


av s = s(aP)as v - P.
Soln. : We know that dF = - PdV - SdT.

.. (aF) = _ P_s(ar) .
av s av s
According to Maxwell relation, (aT) - - (aP)
av s - as v.
Thus, (~~)s = s(~~)v - P.
8. For each of ~he following processes, state which of the following quantities AU,
AH, AS, AF and AG are equal to zero for the system specified.
(a) A non-ideal gas is taken around a Carnot cycle.
Hints : All zeros since cyclic change of state function is zero.
SECOND LAW OF THERMODYNAMICS 201

(b) A non-ideal gas expanded through a throttling valve.


Hints : Joule-Thomson effect, Aff = 0.
(c) Water is vaporised at 100 °C and 1 atm.
HintS : !:lG = 0.
(d) H 2 and 0 2 react to form H20 in a thermally isolated box.
Hints : All non-zeros.
(e) An ideal gas is adiabaticallly expanded through a throttling valve.
Hints : Aff = 0, !:lU = 0.
(f) An ideal gas is allowed to expand from P 1-tP2 at constant T
Hints : !:lU = Aff = 0.
(g) HCI and NaOH react to form Hp and NaCl at constant T and P.
Hints : All non-zeros.
9. At what condition is !:lU = AG = -W true ?

Ans. : By first law, ctQ = dU + ctW. Under adiabatic condition ctQ = 0,


:. ctW = - dU, or -W = AU. For a system performing mechanical work only,
-W = !:lG at constant temperature and pressure. So if an adiabatic change takes place at
constant T and P, then !:lG = !:lU = -W.
10. "Any spontaneous process is always accompanied by an increase in the entropy
(/:lS) of the system." Is the statement correct? Briefly explain.
Ans. : The second law of thermodynamics predicts that !lS of the universe is incre?sing.
!lSuniverse increases for spontaneous change. This prediction is for isolated system Thus,
the statement is wrong.
11. A solid body initially at a higher temperature is left in air. It is observed that the
body will spontaneously cool down to room temperature. Does the free energy increase,
decrease or remain constant? Does your calculation contradict the second law of thermo-
dynamics? Give reasons for your answer.
Ans. : It can be shown that for a spontaneous process /:lG decreases at constant T

and P. At constant pressure, dG =-SdTand (O/:lG) =-!lS. Decrease of temperature leads


CJT P

to decrease in entropy, so that for a temperature decrease, Gsystem increases. A solid body
at ,!ligh temperature will cool down with increase in /:lG.
Here temperature is not constant. There is no prediction for /:lG regarding spontaneity
when T changes. So this cooling is not against the prediction of thermodynamics.
12. Correct or justify the following observations-
(a) Below 0 °C, supercooled water changes spontaneously into ice.
(b) If the same change of state from state A to state B be effected in a system in different
ways, the free energy change in each case will be same provided the processes are reversible.
Ans.: (a) For the system ice_,water, as the temperature decreases, the vapour pressure
of both ice and water decreases. But vapour pressure bf ice decreases at a higher rate so
that below 0 °C, vapour pressure of ice is lower than that of water. Here, /:lG is -ve for
liquid to solid at constant T and P. since !:lG = VM. So supercooled water changes
spontaneously to ice.
202 PROBLEMS ON PHYSICAL CHEMISTRY

(b) For a state change, t:i.G = constant. and it does not depend on path history. For
state change from A~B, t:i.G =G8 - GA and it is immaterial whether the change is reversible
or irreversible.
13. What is the unit of entropy? 100 gm .of water is kept at 100 °C, what will happen?
Will the entropy of water increase or decrease or remain constant? Is your finding· against
the second law of thermodynamics? Explain.
Ans. : Unit of entropy is J.K- 1. In the given case, system will spontaneously cool
down. So entropy of water will decre~se ..
By second law of thermodynamics, entropy of universe has to increase for a spontaneous
change (t:i.Suniverse is (+)ve)." There is no prediction for entropy change of closed or open
system regarding spontaneity. Second law predicts that for a spontaneous change t:i.Ssyste~
is (+)ve, if it is isolated. Here system is not isolated. It loses heat to surrounding, thereby
increasing Jatter's entropy. So. it is in conformity with the second law.
14. One kg of water at 25 °C and 1 atm is subjected to a pressure of 100 atm, the
temperature remaining constant. Calculate the change in free energy. Is t:i.G positive? Is
your finding fn conformity· with the 2nd law?
Ans. : For the given change let the volume be constant. If the density is assumed
to be 1 gm/cc then volume is 1000 cc and t:i.G = Vll.P = 1 x 99 lit.atm = 99 lit.atm. So
t:i.G increases. By second law, t:i.G is (-)ve for a spontaneous change at constant P and T.
There is no prediction for t:i.G regarding spontaneity if P or Tchanges. Thus it is in conformity
with the second law.
15. What are the differences between J-T cooling and adiabatic cooling?
Adiabatic Cooling J-T Cooling

1. Temperature decreases and t:i.H decreases. 1. Temperature decreases below inversion


temperature of a real system,
but t:i.H = 0.
2. There is no inversion temperature. 2. There is an inversion temperature above
which a real gas gets heated on passing
from high to low pressure.
3. In adiabatic cooling, ideal gases are also 3. For ideal gas, there is no J-T cooling.
cooled due to expansion from high to low
pressure.

16. An adiabatic process is always isoentropic.-Comment.


Ans. : The above statement is incorrect. For adiabatic reversible process, Q =0; this
is reversibly taken or, released heat. So t:i.S = Qrev = 0. In adiabatic irreversible process
T
=
Q 0, but that is irreversible heat change. There is no relation between t:i.S and irreversible
heat change. Actually irreversible heat change leads to increase in entropy of system and
the universe.
17. If t:i.H0 =0 fot a reaction, then show t:i.S° = 0 for the same reaction.

Ans. : We know that for reaction


SECOND LAW OF THERMODYNAMICS 203

:. M1° ::::: !le; T + le [on integrating].


Since Ml0 0, Ile; and le must be zero.
:::::

'(J!:J.So) !le~
We know that ( Tr P ::::: T

.. !:J.SO ::::: n !le; ln T.

Now !le; ::::: 0.


.. t'lS° ::::: 0 .
18. If the standard Gibbs function change of a reaction is zero, the reaction is
thermodynamically impossible. State whether the statement is true or false. Justify.
Ans. : The statement is false.
The standard Gibbs energy change !:J.G 0 is the difference between the Gibbs free energy
between the unmixed products and unmixed reactants at their standard states. There is no
relation between standard free energy change and spontaneity. For a spontaneous change,
!:J.G has to be negative at constant T and P. Again, !:J.G 0 ::::: - RT In KP. If !:J.G 0 is zero, then
KP ::::: l. A reaction with KP ::::: I is quite possible. ·
19. State briefly indicating reasons whether the following statements are correct or
incorrect:
(A) Any spontaneous process is always accompanied by a decrease in free energy.
Ans.: The above statement is incorrect. The free energy, G, decreases for a spontaneous
process at constant temperature (7) and pressure (P).
G=H-TS.
At constant temperature,
!:J.G ::::: Ml - T!:J.S ::::: Ml - Qrev.'
At constant P, !:J.G ::::: Q - Qrev· ::::: 0, for reversible change.
= (-)ve, for spontaneous irreversible change, since Qrev > Q.
Thus, the above statement is true only at constant T and P.
(B) For any adiabatic process the entropy change t'lS ::::: 0, since Q = 0.
Ans. : The above statement is incorrect. t'lS is gi~en by t'lS ::::: Qrev , where Qrev is
T
the reversible heat change at temperature T and there is no relation between entropy change
and irreversible heat change. In an irreversible adiabatic process, the heat involved is zero,

but it is irreversible heat (Qirre) and t'lS is not equal to Q;rrev .


T
Q 0
For a reversible adiabatic the heat term Qrev is zero and !:J.S is ;v ::::: T =0. Thus,
t'l S ::;:. 0 for any adiabatic process.
(C) When water is evaporated at 100 °C and at 1 atm, the change in free energy is zero.
Ans. : We know that, !:J.G ::::: 0 at constant T and P at equilibrium. But in such a case,
the equilibrium can be attained if liquid water is kept in the vessel at 100 °C and I atm
covered by a movable piston. Evaporation means a process where the vessel is open to
atmosphere, and vapour escapes. Naturally, equilibrium between two phases cannot be
204 PROBLEMS ON PHYSICAL CHEMISTRY

attained. AG is not equal to zero. The process is spontaneous vaporisation and AG will
be (-)ve. However, at I atm and I 00 °C the vapour ipay not escape since the external
pressure is also I atm. Thus AG may, in fact, be equal to zero.
(D) Free energy increases with increase of temperature at constant pressure.
Ans. : The above statement is incorrect. We know that

((JG)
()T P
= -S.

or, AG =- f dT.
S

S increases with increase of temperature. Since AG is negative, G decrease~ with increase


of temperature and the above statement is incorrect. The plot of G vs. Tis shown in figure.

T-

(E) Free energy increases with increase of pressure at constant temperature.

Ans. : We know that ((JG)


()p
=V. Thus, the slope of G vs. P is positive and G increases
T .

with increase of pressure. Hence, the above statement is correct. For ideal gas,

AG = nRT In ( ~~ }
.and for solid or liquid, AG = V(P2 - P 1).
(F) Entropy is a state function and hence does not depend on the amount of matter
in the system.
• Ans. : The above statement is incorrect. We know that entropy is an extensive property.
Hence, it depends on the amount of matter in the system and thus the portion 'does not'
is incorrect.
Entropy is a state function, i.e., a property. But a property may be either extensive
or intensive. Thus, the fact that entropy is a property does not mean that it is an extensive
property. As such the word 'hence' is wrong. The correct statement is :
Entropy is a state function and it depends on the amount of matter in the system.
20. Predict the sign of AS for each of the following processes :
(a) C(s, graphite)~ C(s, diamond)
(b) Br 2 (l)~Br2 (g)
(c) Hard boiling of an egg
(d) 0 2 (g)~20(g)
(e) Devitrification of glass
(f) Nz(g) + 3H2 (g)~2NH 3 (g)
(g) N 2 (g, 10 atm)~ N 2(g, I atm)
(h) Desalination of sea-water
(i) C(s) + H2 0(g)~CO(g) + H2 (g)
SECOND LAW OF THERMODYNAMICS 205

Ans.:
(a) Negative. Diamond, being a harder solid, would be expected to have more restricted
atomic motions within the crystal.
(b) Positive. S is always greater for a gas than for its corresponding liquid.
(c) Positive. The fundctmental process in the "boiling" of an egg is not a literal boiling,
in the sense of vaporization, but a denaturation of the egg protein. A protein is
a large molecule which exists in a particular configuration in the so-called native .
state but may occupy a large number of almost random configurations in the
denatured state, resulting from rotations around· the bonds. The increase in the
number of possible configurations is analogous to the melting process.
(d) Positive. There is an increase in the number of gas molecules.
(e) Negative. Devitrification is the onset of crystallization in an amorphous solid, i.e.,
supercooled liquid.
(f) Negative. There is a decrea.se in the number of gas molecules.
(g) Positive. Entropy increases on. expansion.
(h) Negative. Desalination iS the opposite of dissolution in which solute must be
removed from a solution.
(i) Positive. There is an increase in the number of gas molecules.
21. Why /:J.S is not a state function?

Ans. : dS is exact differential and J


2
dS :::: /:J.S. It is the change of S, the entropy, which
I
is a state function and thus, /:J.S depends only on states l and 2 and not on the nature of
the path. However, a state function refers to a state but llS does not refer to any state. ·
It refers to a state change and thus not a state function.
22. H 2 at room temperature is warmed as a result of passage from high to low pressure
in the J-T cooling-explain.

Ans.: We know that J-T co-efficient µ1_r= ((JT) . For a van der Waals gas ((JT)
(Jp H (Jp H

( l!!__ _ b)
= RT , where a and b are van der Waals constants and T is the temperature at which
Cp

J-T experiment is carried. At inversion temperature, Ti = !: ,there will be no J-T cooling


or heating.

2a b)
= (RT- .
((JPaT) H Cp

bT; -b)
(
dT = T .dP
Cp

or, !ff= b(T; - TJ .Af'.


TCn
206 PROBLEMS ON PHYSICAL t:HEMISTRY

If T> Ti, the system will be heated, if AP is (-)ve. For H 2, Ti is much below room
tempsrature and thus, it is heated in the J-T cooling.
23. Show that, AGsystem = -T.ASuniverse at .constant temperature and pressure.
Ans. : We know that, Gs =Hs - T?s where G is the Gibbs free energy of the system,
S is the entropy of the system, T is temperature and H is the enthalpy of the system. For
a large change at constant temperature,
AGs = AHs - TsASs ['s' indicates 'system'.]
But at constant pressure,
ANS= Qp

The system receives Qp heat from the surrounding which loses Qp heat at temperature
T. Thus,
Qsystem = - Qsurrounding·
·· AGs = -Q surrounding - T µ s· ..
But surrounding is assumed to exchange heat reversibly.

AS _ Qsurrounding
surrounding - T

·• AGs = - TASsurrounding - TASsystem


= -T (ASsurrounding + ASsystem)
.. AGs = -TASuniverse· [Proved].
24. Ice. is
melted at 0 °C and I atm. State which of AU, AH, ASuniverse• M and AG
must be zero for process.
Ans. : For this change which is an equilibrium at constant temperature and pressure,
AG= 0. As because it is a reversible process ASuniverse = 0 (AGsysem = -TASuniverse' thus
. if AG = 0, ASuniverse must be zero). .

25. What are the values of ( as ) and ( as ) · for an ideal gas?


.. . .. av p aP v

Ans. : Frorri problem 2, (as)


av p
Cp
= -.,....,
a.VT and

RT
For ideal gas, V = p
(i)

Thus, a.= t(~~)P = PRv·


Hence, (as)
av p
= CpPV
RVT
= PCp
RT
= Cp [from (i).]
v
.
A gam, /3 =·- -1 (av)-.- .
. ·V ()P T
'. ' ·.• '
~
SECOND LAW OF THERMODYNAMICS 207

Thus _
• v
_!_(av)
oP
= RT
P2 v ·
T

Hence, ( asoP )v = RTCvPV


P VRT
= Cv
p 2

26. Find the values of ( ;~) P and ( ;; ) v for a gas obeying PV = RT + Pb, where

b is independent of temperature and pressure.


Ans. : Proceeding as Problem 27,

(avas) p= CpP
RT
a~d (as) = _cv
oP .v P

27.
(acoP ) for ideal gas and van. der Waals gas.
Calculate __ P

Ans.: (acp) =[}_(T as) J _T[}_( as) J


oP r oP. oT p r - aP oT p r

T[}_(as)
aT oP r
J= p·

.
. .(as)
By Maxwell relation, -
oP
=- (av)
- .
aT r p

Thus (oCp) =
.·· ' oP r
-T(azv)·
oT p.
2

For an ideal gas, PV = nRT.


Hence, (av)
oT
= nR.
P P

:.
2
a
( oT2
v) P = 0.

Thus, ( oCp) =O.


oP T
2
nRT
For van der Waals. gas, P =- - - -an2 •
V-nb V .

It can be written as, V = nRT + n(b _ ~).


P. RT

Thus, (.-av)
oT p
nR
=-+--na
P RT 2.
208 PROBLEMS ON PHYSICAL CHEMISTRY

Hence, (aCp) = n~
2
aP r RT
.

Alternative Method : See Q. 12 of Miscellaneous in Chapter 5 for ideal gas.

(aH)
aP T
=V - T(av)
aT p
[using Maxwell relation,( as) =-(~V)
aP T oT
]. p

For van der Waals gas, V = n~T + n( b - ;T).

(av) nR na
aT = :-p + RT
p 2 •

Thus, ( aH) =V _ nRT _ ~ = -


nRT
- + nb -
na nRT na
aP T P RT P RT P RT

2na
= nb - -.
RT

2na
Thus,
RT 2 •

Here a and b are assumed to be temperature and pressure independent.

28. Find out the value of ( acv ) for ideal gas and van der Waals gas.
av r

Ans. (acv) =[_i__(Tas) J =T[_i__(Tas) J =T[~(as) J


· · av r av aT v av aT v aT av
r v· r r.

. ·(as)
By Maxwell relation, -
av r
= (aP)
- .
aT v

av r =T(aaT~) v
2
Thus, (acv)
For ideal gas, PV = nRT,
(~~ )v = 0.
Hence, ( aac; l =0.
2
nRT an
For van der Waals gas, P =- - - -
V-nb V 2 •
SECOND LAW OF THERMODYNAMICS 209

az ~) = 0. (since a and b are temperature independent)


Again, (
aT v
So, (acv)
av r-_ 0.
Alternative Method : See Q. 12 of Miscellaneous in chapter 5.

(au) = T(aP) - V[using Maxwell relation, (as) = (aP) ]


av r aT v av r aT v
2
nRT an
For van der Waals gas, P =- - -
V-nb V2
-
nR
V-nb
2 2
Thus, ( aauv )T = Vn-RTnb _ nRT + an2
V-nb v
_
-
an2
v •

Hence, ( acv) = [_i_( au) ] = 0 (since a and bare considered to be temperature


av r aT av r v
independent)
29. Give examples of a process in each of which for the system (i) !lG = 0,
!lS > 0, (ii) !lG < 0, !lS < 0, (iii) !lG = 0, !lS < 0, (iv) !lG < 0, !lS > 0.
Ans. : (i) Melting of ice to water at 0 °C and 1 atm : !lG =0, !lS > 0.
(ii) Freezing supercooled water to ice at -5 °C and l atm : !lG < 0, !lS < 0.
(iii) Condensation of vapour to water at 100 °C and 1 atm : !lG =0, !lS < 0.
(i•) Vaporization of superheated water to vapour at 105 °C and l atm : !lG < 0,
!lS > 0.

30.1:1e Helmholtz energy of one mole of a certain gas given by A =-( ~ )- RT In


(V - b) + j{T), where a and b are constants and f (T) is a function of T only. Set up the
expression for pressure of the gas. ·

Ans. : A = (~) - RT ln(V-b) + j{T)

or, ( aA) r
av
= a
VT
- RT
V-b
= -P.
.

P=_!!I_- a
.. V-b VT'
31. Though entropy is a fundamental state function and free energy is a derived one,
the latter can be used more conveniently.-Explain.
Ans. : Both the entropy (S) and the free energy (G) indicate direction of a process,
.. For a spontaneous process, !lG of the system is (-)ve at constant temperature and pressure,
and !lG = 0 at the same condition indicates equilibrium, while positive value of !lS for
universe inc,iicates spontaneous process, and zero value of the same indicates reversible
210 PROBLEMS ON PHYSICAL CHEMISTRY

process. Sum of entropy change of system (Msystem) and surrounding (Msur) comprises
. entropy change for universe. Calculation of Msurr is not always easy. So free energy is
preferred to entropy as far as direction of a process is considered.

Numerical Problems
1. Consider the following cycle using I mole of an ideal gas initially at 25 °C and I atm.
(a) Isothermal expansion against zero pressure to double the volume.
(b) Isothermal reversible compression from .!. to I atm.
2

Calculate W, M for a and b in the cycle for the system and Muniverse·
Soln. : By the problem, n =I mole, T = 298.15 K, P =I atm, V = V.
(a) For free expansio~. W = 0.
For ideal gas /J.U = 0 at constant T. :. Q = 0.

But Msystem = nR In Vz = 8.314 J.K- 1 In 2 = 5.76 J.K-1.


V1
= 0. :. Muniverse = 5.?6 J.K-l.
(b) Msystem = nR In~ = -5.76 J.K-1.
P2
W = -nR Tin.!. =RT In 2 = 1.72 kJ.
2
Q = -W = -1.72 kJ
M
surr
= ~
298.5
= 5.76J.K-1.

Muniverse = 0.
For this cyclic change, Muniverse = 5.76 J.K- 1.
2. A quantity of ideal gas expands (i) isothermally and reversibly at 500 °K from a
volume of V1 to V2 . During the expansion the gas absorbs 400 cal of heat from the reservoir
in contact with it. Find M for (a) gas, (b) reservoir and (c) the whole system. (ii) If the
expansion was carried out isothermally but irreversibly for the same volume change from
vi to Vz absorbing 200 cal of heat, what W?uld be Mg, MR and Muniverse?
Soln. : (i) By the problem, T = 500 K
Q = 400 cal.

Since the change is reversible, Mg = Q = 400 = 0.8 cal.K- 1 = 3.35 J.K-1.


T 500
MR= - 3.35 J.K-1. :. Muniverse =Mg +MR= 0.
200
(ii) Mg = 3.35 J.K- 1 but MR = - = - 0.4 cal.K- 1 = -1.675 J.K-I.°
500
:. Muniverse = (3.35 - 1.675) 1.K-I = 1.675 J.K- 1 = 1.68 J.K- 1.
3. The molecular specific heat of C0 2 at constant pressure varies with absolute
temperature according to the equation,
CP = (7.0 + 0.0071 T - 1.86 x 10-{j T2) cal.
Calculate M when 50 gm of C0 2 i& brought from -10 °C to 50 °C.
SECOND LAW OF THERMODYNAMICS 211

Soln. : !lS =
f nCp TdT
= J50 [7.0 In
323 5 86
· 1 + 0.0071(323.15 - 263.15) - 1. (323.15 2 - 263· l5 2)x 10-6]
44 263.15 2

= SO (1.4377 + 0.426 - 0.0327155) = 2.08 cal.K- 1 = 8.7J.K-1.


44
4. For benzene, a = l.24 x 10-3 deg- 1 and {J = 4.93 x lo-s atm- 1 at 20 °C and
l atm. Assuming a and {J to be constant, find the change in molar volume which will be
required to produce an entropy change of 6.75 J.deg- 1. ·
Solo. : According to the given problem-
<X = l.24 x 10-3 K- 1 constant at a constant temperature.

Similai-Iy, f3 =4.93 x 10-5 atm- 1 constant at constant temperature.


Here, T = 293.15 K.

= <X ~v.
13
Now, !lS

1. 24 x 10-3 K- 1
.. 6.75·J.deg- 1 x ~V lit x 101.325 J/lit.atm.
- 4. 93 x 10-5 atm -I
= 2548.5396 x ~v lit.
or; ~v 6 75
= - --·- - = 2.6486 x 10-3 lit = 2.6486 cc.
2548.5396
5. Calculate, ~Vin the above case if the entropy change is 0.82 cal/deg.
Hints : Same as Problem 4.. [Ans. l .34 cc]
6. At 30 °C, a for a certain liquid is l.11 x 10-3 deg- 1. Its density is 1.03 gm/cc and
33 gm of the liquid was taken. The liquid was kept at l 0 atm and the pressure was released
to 1 atm. ~alculate !lS.
Solo. : According to the given problem,
a = l.11 x 10-3 K- 1, which is constant at a constant temperature.
p = 1.03 gm/cc., m = 33 gm.

Hence, volume of liquid is ~ = 32.03883495 cc.


l.03
We know that, !lS = aV(P 1 - P 2)
32 0388
= l.11 x 10-3 K-1 x · lit x (IO - l) atm x l 01.325 J/lit atm.
1000
= 0.032 J.K- 1•
7. For an ideal gas, CP = 9.037 + 35.6 x 10-3T - 216.9 x 10-1T2 cal.moJ-1.deg-1.
Calculate !lS when 5 moles of the gas are heated from 500 °K to 1000 °K, and the volume
changes from JOO lit to 50 lit.
Solo. : According to the problem,
CP = 9.037 + 35.6 x 10-3 T - 216.9 x 10-7 T2 cal.moJ-1.deg-1.
212 PROBLEMS ON PHYSICAL CHEMISTRY

Here, n =5 moles, T1 = 500 K, Tz = 1000 K, V1 = 100 lit, Vz = 50 lit.


IOOO

Af =n f-(
500
Cv ~)+nRln ~2
I

216 9
= 5[7.05
In lOOO + 35.6 x 10-3(500) - · x 10-7(1000z - 500Z)] + 5R ln2
500 2
Here, Cv = CP - R = 9.037 + 35.6 x 10-3T - 216.9 x 10-7 TZ - 1.987
= 7.05 + 35.6 x 10-3T - 216.9 x 10-7 TZ
Af = 333.3 J.K- 1•
8. For a certain ideal gas CP = 7.05 + 17.8 x 10-3 T - 308 x 10- 7 TZ cal/mole deg.
Calculate Af if the state changes from 500 °K and 50 atm to I000 °K and 120 atm and
there are 10 moles of the gas.
Hints : Same as Problem 7. [Ans. 20.81 J.K- 1]
9. 3 moles of Nz originally at I atm are mixed with 5 moles of Hz at I atm isothermally
to yield a mixture whose pressure is also I atm. Calculate Af.
Soln. : In the given problem, we have 3 moles of Nz and 5 moles of Hz. So, mole

fraction of N z = ~ and that of Hz = ~


8 8
Af
We know that, - -
mole
= -RI.xi lnxi

= -R(%1n%+%1n%)J.K-1.
_ 3 5. -I
Now, Aftotal - -R(3 In S + 5 In S) J.K
. .8 8
= 8.314(3 In-+ 5 In -)J.K-1
3 5
= 8.314(2.94 + 2.35) J.K- 1 = 8.314 x 5.29 J.K-1
= 43.9812 J.K- 1 = 44 J.K-1.
10. If two moles of an ideal gas A ac 120 °C and I atm is mixed with 3 moles of
another gas Bat 70 °C and I atm and as a result the pressure remains at I atm. Calculate
Af.
Soln. : By the problem, nA =2 moles, TA = 120 °C = 393.15 K, P =I atm,
and n 8 = 3 moles, T8 = 70 °C = 343.15 K, P = 1 atm.
Here heat transfer occurs from A to B. Let the final temperature be T. Thus,
2(120 -7) = 3(T - 70)
:. T = 90 °C = 363.15 K.
Let the gas A and B be mixed at 90 °C. Thus, this is an example of free mixing.

So,
Af. .
- - = -R'I:.x.Inx;
1
=- . (2 2 3 3)
8.314 J.K- 1.mole- 1 -Jn-+-ln-
mole 5 5 5 5
= 5 .59542 J.K- .mole- .
1 1

. . l1S 101a1 = 5 mole x 5.59542 J.K- 1.mole- 1 = 27.977 J.K- 1.


SECOND LAW OF THERMODYNAMICS 213
Now, entropy change due to temperature change is

I
363. 15 C I 363.15
Af = nA CPA n 393.15 + nB PB n 343.15

363 15 36115
=2x l Rln · +3x l Rln =0.1396J.K- 1•
2 393.15 2 343.15
Hence, total entropy change = Afmix = (27.977 + 0.13960) J.K- 1 = 28.12 J.K- 1•
Alternative way :

T PA 2
AfA = nACPA In TA - nAR In p
(PA =s· P = I)

T PB 3
AfB = nBCP In- - n# In- (PB= - , P = I)
B TB p 5
and Afmix = AfA + AfB.
11. If two indentical monatomic perfect gases A and B with same pressure and the
same number of molecules N kept in 2 bulbs of volumes VA and VB at temperatures TA
and TB respectively are mixed together, calculate Af for mixing.

Solo. : A B

Let T' be the final temperature attained by both gases. Thus,


CiTA - T') = CB(T' -TB) .
.. T' = TA +TB. since CA= C
8
2
Let, after mixing, PA and PB be partial pressures of gases A and B respectively.
By the problem, P = PA = PB, since two gases are idential.
Afmix = AfA+ AfB
T' T' T' T'
= nACPAln- + nBCP In- = -25 NKB 5
In- + - NKB In
TA B TB TA 2 TB

T' 2 (TA +TB)


= -25 NKBln -- = 5 NKnln
TATB 2~TATB .
12. One mole of an ideal gas, originally at the volume of 5 lit at 500 °K is allowed

to expand adiabatically until the final volume is 15 lit. For the gas, Cv = lR. Calculate
2
final temperature and Af, when
(a) the expansion takes place reversibly,
(b) the expansion takes place against a constant pressure of 3 atm,
(c) the change in volume is due to free expansion.
214 PROBLEMS ON PHYSICAL CHEMISTRY

Soln. :
5 I 15 I
500 °K T2oK C
v
= ~R
2
n = I mole n = I mole

(a) For reversible and adiabatic process, we know that-

T1 V1Y- 1 = T2V2Y- 1, where"(= CP


CV
V
or, T2 = T1 ( V
1)y-I = 500 (J55 )~-I = 240.5 K.
2

Since it is an adiabatic and reversible process, Af = 0.


(b) We know that, Pe/V2 - V1) = nC/T1 - T2)
3
or, 3(15 - 5) = -R(500 - T2) <Jiere, R
2
= 0.0821 l-atm.K- 1.moJe- 1)

or, T2 = 256.39.

256. 39 15
. . Af = nCv In - - - + nR In -
500 5
= 0.8044J.K-I.
(c) For free expansion, W = 0.
Since the process i.s adiabatic, Q = 0.
.. t:.U =Q + W = 0.
Since the gas is ideal, AT = 0.
. . T2 = T1 = 500 K.
v
2 = 8.314 In 3 = 9.13J.K-I.
Hence, M = nR ln-
Vi
13. For the fusion of ice, t:.H is given by L'lH =1100 - 9T. Calculate t.G for the change-
atm)~H 0(s,
Hp (I, I 2 latm) at -IO °C.
Soln.: At 273.15 K, L'lH = 1100- 9T= 1100- 9 x 273.15, (-)ve.
Here l~s transition is considered.

We know that,

or,d(~)= -J~dT=-J1 10~; 9 T dT.

t:.G 1100 .
or, T = T + 9 lnT + lc [where lc = Integration Constant]

or, t:.G =· 1100 + 9 TlnT + lc.T. [Since the temperatures are different, there is a
difference in these cost values.]
SECOND LAW OF THERMODYNAMICS 215
At 273.15 K, AG = 0 for the above change.
. . IC = -54.52.
.. AG = 1100 + 9 T In T - 54.52 T.
Now, AG at -10 °C is
AG = llOO + 9x 263.15 li:i, 263.15 - 54.52 x 263.15 = - 48.78 cal = -204.10 J.
14. Find out AG at I atm per mole at -17 °C for HzO(l)~HzO(s). Given, Cp<H 2 0, I)

= 18 cal/mole, Cv (H20, s) = 9 cal/mole, Aff273 = -1440 cal.mole- 1.


Solo. : ACP for the change = -9 cal/mole
[since, ACP = CP (H20,s) - CP(HzO, I) = 9 - 18]

Now, we know that, c~;) P = ACP.


or, dMf = ACPdT
or, AH = ACp· T + I c [where I c = Integration Constant]
At 273.15, Mi = -1440, and ACP = -9.
:. IC= -1440 + (9 x 273.15) = 1018.35
:. AH = 1018.35 - 9T.

Following Q. 13 we can write,


AG= 1018.35 + 9T In T + lc.T.
But at 273.15, AG = 0.
• . IC = -54.22
.. AG = 1018.35 + 9 Tin T - 54.22 T.
At -17 °C,
AG= 1018.35 + (9 x 256.15 In 256.15) ~ (54.22 x 256.15)
= -85.18 cal.moJe- 1 = ...;.JS6.39 J.mole- 1•
15. I mole of He initially at 27 °C and 1 atm is heated to 327 °C, the final pressure
of the gas being adjusted to IO atm. Calculate AS and AG assuming ideal behaviour. Given
that, s;7 = 30.20 cal/deg.
Solo. : The process may be depicted as-

300.15 K 600.15 K 600.15 K


i-- f---+
I atm I atm 10 atm
n = 1 mole n = 1 mole n =I mole
600.15 10
Here, !'lS = nCP I n - - - n R In-.
300.15 I

Putting n = 1 and CP = %R, we get


!'lS = -1.13 cal/deg = - 4.74 J/deg.
Now, AG 1 =AH - (T2S2 - T1S 1).
216 PROBLEMS ON PHYSICAL CHEMISTRY

Again, !).H = nCP dT and Sz = S 1 + nCp In(~~ J


T T
Thus, Sz = S° + CP In T! and S 1 = S° + CP In T~

By the problem, T 1 = T 0 and S 1 =S 0


= 30.20 cal/deg.
T
Hence, !).G 1 = (2.5 x 1.987 x 300) cal - 600.15(S0 + CP In T!) + 300.15 x S°

= 1490.25 - 600.15 (30.2 + 2.5 x 1.987 x 0.693) + 300.15 x 30.2


= -9635.45 cal.
10
Again, !).Gz = +1.987 x 600.15 In -cal = + 2745.8282 cal.
I
!).G = (-9635.45 + 2745.8282) cal
= -6889.6218 cal x 4.184 J/cal = -28.83 kJ,
16. I mole of Oz escapes adiabatically into an evacuated chamber of such size that
its final volume is 10 times the initial volume. Given that Cv = 4.90 cal and 'a' for oxygen
is 1.36 atm.litz .moie-2. Calculate Af if the initial temperature is 0 °C (For volume calculation
use ideal gas equation).
Solo. :

n=l 1n=l
Volume = VI = V Free expansion Volume = Vz = IOV.
I
P = I atm · ·
It is an example of free expansion.
The initial volume of the gas (Oz), V
nRT I mole x 0. 08206 l. atm. K- 1• mo1- 1 x 273. J5K
= -
P
=
latm ..
= 22.414 lit.

Now, we know that, J(~~)T dV = J; 2dV =a. V~1~~1 = a. 1 ~~ 2


VJ

1.36x 9
= !Ox 22.414
Iit.atm.

6 9
Again, !).U = nCv (T 1 - Tz) - 1.3 x x 24.21cal.
JOx22.414
Now, the given problem is an example of adiabatic and free expansion. So, Q = 0 and
w = 0.
1.36 x 9
Hence, !).U = 0 = nCv (T1 - Tz) - x 24.21 cal
!Ox 22.414
1.36x 9
or, 4.9(273.15 - Tz) - JOx .4 x 24.21 = 0
22 14
or, 1338.435 - 4.9Tz - 1.32208 =0
or, 4.9Tz = 1337 .1129 or, Tz = 272.88.
SECOND LAW OF THERMODYNAMICS 217

Now, !lS = nCv In T2 + n R lnlO


Ti

= 4.9 Jn 272 ·
273.15
88
caJ.K- 1 + 4.575 cal.K- 1(since, Cv = 4.9 caJ.K- 1.moJ- 1).

= -0.00484 cal.K- 1 + 4.575 caI.K- 1 = 4.570 cal/K = 19.12 J.K-1.


17. For copper p = 0.8 x
10-6 atm- 1. What is the change in Gibbs free energy when
63.546 gm of copper is subjected to an extra pressure of 1000 atm? What would be AG
if P is assumed to be zero? Given that the density of copper is 8.93 gm/cc.
Soln. : We know that co-efficient of thermal compressibility,

p =-~(~~)T.
.. dV =- PdP
v
Integrating we get-
ln V = -PP + lc [Integration Constant].
When P = 0, lc = lnV0.

.. In~= -PP
Vo
i.e., V = V0e-f3P.
If p is very smaJI, V = V0 (l - PP).
Again, AG = JV dP = JV0(l - PP) dP.
Assuming that, V0 is the volume at zero pressure,
AG = JV0(l - PP)dP.
Integrating, we get
IOOO

AG = f0
V0 (l-PP)dP

= Vo<Pz - P1) - P~o (Pi- Pi2)


= (V0 x 1000) - PVo (1000)2 [taking P1 = O]
2
0 8 10
or, AG = (V0 x 1000) - · x --0 x v0 x (I 000)2.
2
63 546
Again, v0 = • = 7.116 x 10-3 lit.
8.93
.. AG = 7.116 x 10-3 x lOOO(lit.atm) - 4x10-7x 7.ll6x10-3x (1000)2Jit.atm

= 7.116 Jit.atm - 0.0028464 lit.atm = 7.113 lit. atm = 720.725 J.


If p= 0, AG = 7.116 lit-atm
= 721.0287 J.
Ll8 PROBLEMS ON PHYSICAL CHEMISTRY

18. For water ~ = 4.94 x 10-5 atm- 1 and d = 0.997 gm/cc at 25 °C. Calculate AG,
when the pressure of 18 gm of water at I atm is raised to 101 atm at 25 °C. ·
.
H mts : AG= V0(P 2 - P 1) -
2 -~Vo
( P 2 - p2)
1- [Ans. AG= 182.8 J]
2
19. Calculate the entropy change of the universe as a result of each of the following
processes-
(a) A zinc block of 500 gm mass and with a total heat capacity at constant pressure
of 150 J/deg at I00 °C is placed in a lake at I 0 °C.
(b) The same block at I0 °C is dropped from a height of 100 m into the lake.
(c) Two such blocks at 100 °C and 0 °C are joined together.
Solo. : (a) The lake is a reservoir. Its temperature remains fixed at 10 °C. The zinc
block attains 10 °C. Thus it rejects 150 (100 - 10) = 150 x 90 J heat and lake receives
it. Thus
AC' = 150x90 =47.68J/de.
U>J Lake 283.15 g
T2 373.15 .
AS8 1ock = nCP ln- = -150 I n - - = -41.4005541/deg.
T1 283.15
· · AS universe = 47.68 - 41.40 = 6.28 J/deg.
(b) Q = mgh = 500 x 980.665 x Ht erg = 490.33 J.
This heat is rejected to the lake.
490.35
:. ASBlock = 0, ()Slake = + - - - = l.73 J/deg.
283.15
:. ASuniverse = +1.73 J/deg.
(c) Two such blocks are joined. They have the same mass. Hence, final temperature
: 50 oc = 323.15 OK.
323.15 In 323.15.
AS 1 = 150 In . and AS2 = 150 .
273 15 373 15
:. AS = AS 1 + AS 2 = 3.63 J/deg.
20. One mole of an ideal gas is allowed to expand isothermally at 300 K from I atm
to 0.1 atm (i) reversibly and (ii) irreversibly. Calculate AG for (i) and state with reason
whether AG for (ii) would be different.
Solo. : By the given problem state changes from I to II.

State-I State-II
n = I mole n = I mole
P1 = 1 atm P 2 = O.l atm
T1 = 300 K T2 = 300 K

Same state is reached both by path I and path II. G is a state function. Thus, AG for
both paths I and II are identical. We know that,

2
p I
AG= nRT ln- = I x 8.314 x 300 In- = -5.74 kJ,
P, IO
SECOND LAW OF THERMODYNAMICS 219

21. I gm of water enclosed in a vial is placed in an evacuated flask miantaining at


298 K. By means of a lever, the vial is broken so that the water is free to vaporise, and
at equil1brium it is found that half of the water has vaporised. The vapour pressure of water
at 25 °C is 24 mm· of Hg and the enthalpy of vaporization is 2470 J/gm. Calculate Q, W,
Mi, !lG and !lS.
Solo. : The process is-

2l gm I
of Hp (I, 25 °C, 24 mm) ~ - gm of H 20 (g, 25 °C, 24 mm).
2

Q = 2470 = 1235 J.
2
Let Q = Qv, :. W = 0 (no external work is done).
or, !lU = 1235 J.
Now, Mi= QP = Qv + P!lV = Qv + PV
8
1 1
= Qv + nRT [n = 2x18 = 36]
I
or, Mi= 1235 + ( - x 8.314 x 298) = 1304 J.
36
This Qrev (slow expansion against P to vaporise half the water).

... !lS -- -1304 -- 4.38 J.K-1 .


298
1304
Now, !!G = Mi - T!lS = 1304 - 298 x - - = 0.
298
22. An isolated system comprises of I mole of ideal gas, a heat reservoir at T 1, a
machine and a source of energy for the machine. The gas at T 1 first expands freely to three
times its initial volume; it is then compressed by the machine (at T 1) reversibly back to
the initial volume; the gas is again allowed to expand freely to twice the initial volume.
Calculate !lS for the process.
Solo. : Step-1 + Step-2 together returns ~he gas to the initial state.
Thus, for the gas, !lS = 0.
!lS of the gas for Step-3 = R In 2 = 5.76 J.K-1
For the Step-I, W1 = 0, Q1 = 0, !lU = 0,

For the Step-2,


v
W = RT ln-1. = RT In 3 = 9.13T
2 1 1 1
V1
Since, !lU = 0, Q = -W = -9.13 T 1J
W3 = !lU = Q = 0.
Thus, 9.13 T 1 heat would be delivered to the heat reservoir in the· whole process and
it gains
9.13T1
-- = 9.13 J.K- 1 entropy. Hence, !lS = 9.13 + 5.76 = 14.89J.K-1•
Ti
23. One mole of an ideal monatomic gas undergoes a process whereby its volume
and temperature are trebled. Calculate !lS and !lG if the gas is initially at 273 K and
I atm. Its molecular wt. is 40 gm/mole.
220 PROBLEMS ON PHYSICAL CHEMISTRY .

It may be assumed that for a monatomic ideal gas of molecular wt. M,

s; = [Rln[ri M% )-2.31 lcal.K- 1mole- 1•

Solo. : We know that,

=C In Tz + R In Vz
v T1 V1
= Cv In 3 + R In 3
= CP In 3 = l5 R In 3 = 22.83 J/K.
Now, !lG = Ml - ll(TS)

But Ml

= 1.987 In [(273)%(40)%] - 2.31 = 36.55 cal = 152.9 J.

= 1.987 ln[(819)%(40)% ]- 2.31 = 42.01 cal = 175.8 J.


:. llG =Ml - T2S2 + T 1S 1•
Let P be constant at l atm.
:. S2= s;, SI = s;.
:. llG = Ml - (819 x 175.8) + (273 x 152.9)
or, llG = - 90.89 kJ,
s~J9 can also be calcufated as-
0

S819 = S273 + nCP l n819


0

-
273

= 152.9 + ~2 819
x 8.314 In
273
=
175.8 J/K.

24. l mole of an ideal monatomic gas is taken from the state (24.6 I, l atm 300 K,
S0 = 81 kJ- 1) to the state (2 atm, 320 K). Calculate llU, Ml, llS and llG for this change.

Solo. : We know that, llU = nCv llT ="23 R x 20 = 249.42 J


5
and Ml = nCP llT = lR x 20 = 415.7 J.
Now, State-I State-II

V = 24.6 I P = 2 atm
T = 300K T = 320 K
n = l mole n = l mole
SECOND LAW OF THERMODYNAMICS 221

nRT 0. 082 x 320


P = l atm, :. V = - - = = 13.12 lit.
p 2

We know that, !JS = nC ln T 2 - nR ln p2


P Ti Pi

5 320 2
= -R I n - - R In- = -4.42J.K-1 .
2 300 l
Calculation of t::.G : ·
300 K Step-I 320 K Step-II 320 K
l atm ~ l · atm ~ 2 atm
!::.GI = Afl1 - T2S2 + T1S1

= (%Rx20)- 32o(s 0 +Cpln~~~) + (300 x 81)

= 415.7 - 320(81+~Rln 320


2 300
)+ (300 x 81) = -1.63 kJ.
AG 2 = RTln p2 = 8.314 x 320 In 2 = 1.84 kJ.
Pi
.. AG= AG 1 + AG 2 = 210.5 J.
25. Calculate Ml and !JS when a l kg bar of copper at 90 °C is placed in 2 kg of
water at 283 K in an insulated container maintained at l atm.
Heat capacity : Cu (0.41 J.K- 1.gm-1)
Hp(I) (4.2 J.K- 1.gm- 1).
Solo. : Let the final temperature be TF· So,
1000 x 0.41 (90 - TF) = 2000 X 4.2 X (TF - 10)
or, TF = 13.723042 °C = 286.873042 °K
Hence, Aflcu = nCP (TF - Tcu) = 410 J.K- 1(286.873042 - 363.15) K
= -31273.55278 J.
AflH 2o = 2 x 4200 J.K- (286.873042 - 283. J5)K = 31273.5528 J.
1
Ml = Aflcu + AflH 2o = 0.00002 J = 0.

Now, /'J.Scu = 410 In 286.873042 = -96.668 J.K-1 .


363.15

/'J.SH20 = 8400 In 286.873042 = 109.729 J.K-1


283.15
= 13.06 J.K- 1•
26. One mole of an ideal and monatomic gas'initially at 300 K imd l atm (occupying
24.6 lit) expands isothermally and reversibly to 49.2 lit under conditions that, W = -400 J.
Calculate AG and !JS. •
Solo. : Since the process is isothermal, AU = 0. Hence, from the 1st law of
thermodynamics, we get-
Q = -W = 400 J.
222 PROBLEMS ON PHYSICAL CHEMISTRY

In the given problem, V1 = 24.6 lit, V2 = 49.2 lit,


1
P 1 = I atm, .. P 2 = 2' atm.

. . llS = R ln_!l_ = R In 2 = 5.76 J.K-1•


Pz
p
!!.G =-RT In - 1 = -RTin 2 = -1.73 kJ,
Pz
27. One mole of a perfect monatomic gas initially at volume V1 = JO lit, pressure
=
P 1 and temperature T1 300 K experiences the following reversible changes :

(a) isothermal compression to one-half the volume ( ; , P2 )

(b) cooling at constant volume until the pressure is returned to the original value of
Pl' the final temperature being T2.
(i) Calculate P 1, P2 , T2 , Q, W, !!.U.
(ii) Are the magnitudes of !!.U, Q, W same by path C.

5 JO ..
V(lit)

Soln. : In the given problem, p 1 = nRT1


Vi
lmolex0.082061.atm.K-i.mole- 1 x300K
= JOatm
= 2.46 atm.
After isothermal compression, P is doubled. Hence, P 2 = 4.92 atm. To reduce this
300
pressure to P 1 by cooling, T2 = IJ_ = = 150 K.
2 2

. 'pT =-1..
T -2 P = T]
[smce
. p or • T
_.!_
.I 2
2
= TI.p
I
_.!_
2

(a) For isothermal change, !!.U = Ml = 0.


W = - nRT In Vz =-RT In.!_ = 8.314 x 300 In 2 = 1728.8477 J = 1.73 kJ,
Vi 2 ·
Q = -W = - l.73 kJ
.v . v
llS = nRln-1.. = -Rln 2 = -5.76J.K- 1; !!.G = nRTin - 1 = 1.73 kJ.
~ ~
SECOND LAW OF THERMODYNAMICS 223

(b) For isochoric change, W = 0.


3
.. Q = liU = nCV (Tz - Tl) = -150 x -R = -1.87 kJ .
2
5
Afl = nCP (T2 - T 1) = -150 x ZR= -3.12 kJ

T 3 I ·
1iS = nCv In....l = -R In- = -8.64 J.K- 1
Ti 2 2
liG = Af{ - TzSz + TISI.
Here liG cannot be calculated because S0 is not known.
N.B. : (i) S 0 can be calculated by using the formula in Problem no. 23 and then liG
can be calculated. Molecular wt. M is needed.
(ii) U is a state function. Hence liU will be same by path C. Area under C less than
area under A + B. Thus, W will be smaller and Q will be smaller as Q = W + liU.
28. 1 mole of H 2 (g) at 373 °K is compressed adiabatically and reversibly from 1 to
10 atm. Calculate liU, Afl, liS and /iG assuming ideal gas behaviour.
Given, CP = 6.9 cal.deg- 1.moJe- 1. S 0 at 298 = 49 cal.deg- 1.
Solo. : Now, P 1 = I atm, T 1 = 373 °K, P 2 = 10 atm. ·

..!!_ 1.987
Tz = T 1 ( ~~ YP = 373 (10)6.9 = 723.9K.

Cv = 6.9 - 1.987 = 4.913 cal.deg- 1.mole-t.


liU = nCV (Tz - T1) = I x 4.913(723.9 - 373) = 1.72 kcal.
Afl = nCP (T2 - T 1) = 6.9(723.9 - 373) = 2421.21 = 2.42 kcal.
liS = o.
!iG can be calculated for the following change :
n = I mole n = 1 mole n = I mole
373 K
I atm
I _
723.9 K ,__II _., 723.9 K
atm IO atm
l'lG1 = Af{ - TzSz + TISI

723 9 373
= 242.21 - 723.9(s 0 +Cp In · ) + 373(s 0 +Cp In )
298 298

= 2421.21 - 723.9(49+6.91n
728
·
298
9
)·+ 373(49+6.91n
.
373
298
)

= - 18628.41432 cal = -18.628 kcal.


p
liG 2 = RTJn....l = 1.987 x 723.9 lnlO = 3312.01376 cal.
Pi
.. !iG = liG 1 + liG 2 = (3312.01376 - 18628.41432) cal
= -15316.40056 cal= - 15316.4 cal = - 15.32 kcal.
29. Calculate the difference in molar entropy and fI!Olar free energy (a) between liquid
water at -5 °C and ice at -5 °C. (b) between liquid water and water vapour at 95 °C and
1 atm. Distinguish between liSsystem and liSsurrounding· .
224 PROBLEMS ON PHYSICAL CHEMISTRY

= 37.6 J.K- 1 (s~I) [Cp.t - Cp)


= - 43.2J.K- 1 (1~v) [Cp.v - Cp.11
= 6.02 kJ.mole- 1
Le = 40.67 kJ.mole-1
cp.l => specific heat at constant pressure of liquid water.
cp.s => specific heat at constant pressure of solid water.
cp.v => specific heat at constant pressure of water vapour.
Soln. : !lS system :
(a) It is given that-
~-~

HzO(l) l HzO(l) i----2- H 20(s)


-5 °C 0 °C 0 °C -5 °C
!lS = !lS I + Af2 + Af3

!lS = C In 273.15 _ (6.02x1000) + C In 268.15


or, p.l 268.15 273.15 p,s 273.15

= (c p.I
In 273.15 -C In 273.15)- 22 .04
268.15 p.s 268.15

273 5
= AC In · 1 - 22.04
p,m 268.15
273 5
= 37.6 In · 1 · - 22.04 = 0.6946 - 22.04
268.15
= -21.3454 J.K- 1 = -21.34 J.K- 1.
__ Affm(l ~ s) Affm(s ~ l)
T T
Now, Affsurr = -Cp.l (TF - T) + Affm (s~I) - Cp.s (T -TF)
= Cp.l(T- TF) + Affm (s~I) - Cp.s(T -TF)
= ACp,m(T- TF) + Affm
= (-37.6 x 5) J + 6020 J = 5832 J.
_ 5832J _ -I _ -I
.. Afsurr - .l K - 21.749 J.K - 21.75 J.K .
268 5
.. !lSuniverse= (-21.34 + 21.75) J.K-1.
= 0.41 J.K- 1.
We know that, G = H - TS.
. . at constant temperature, AG = Aff - T!lS.
. . AGm (l ~ s,T) = Affm (l ~ s,T) - T!lS (l ~ s,T)
= -5832 - {268.15 x (-21.34)} = -5832 + 5722.321 = -109.68 J.
AG can be calculated in other way :
Affm = 6020 J.mole- 1
ACp,m =.37.6 J.K- 1
SECOND LAW OF THERMODYNAMICS 225

(am) aT
=!le
P

d (Ml) = !lCPdT
Ml = !lCPT + lc [lc = lntegretion Const.]
-6020 = (-37.6
x 273.15) + IC
. . Ic = 4250.44 J.mole- 1
. . Ml= 4250.44 - 37.6 T

!lG = A - BTlnT + lc.T [since, (a(llG/T)) =-Ml]


aT p T2
= 4250.44 + 37.6 TlnT + lcT
.. 0 = 4250.44 + 37.6 x 273.15 In 273.15 + lc x 273.15
or, lc = -226.49762
. . !lG at 268.15
= 4250.44 + 37.6 x 268.15 ln268.15 - (226.49762 x 268.15)
= -108·46 J.moJe- 1.
(b) H 20(1) HzO(I) HzO(v)
95 °C 100 °C 95 °C
!lS = !lS1 + !lSz +. !lS3
=C In 373.15 + 40670 + C In 368.15
p.s 368.15 373.15 p,v 373.15
373. 15
= 108.99102 +(Cpl. - Cpv) I n - -
' 368.15
373 15
= 108.99102 - 43.2 In
·
368.15
= 108.99102 - 0.58277 = 108.40825 J.K- 1.
= 108.41 J.K- 1.
N !lS - -Mim(l ~ v)
ow, surr - T

Mfsurr= -Cp.l(Tb-T)-Mfm(l~v,Tb)- Cp,v(T-Th)


= -Mfm - (Cp,l - Cp) (Tb - T)
= -40670 - (43.2 x 5) = - 40886 J.
- -40886 - -1
.. !lSsurr - 368.15 - -111.058 J.K .

.. !lSuniverse = -2.648 J.K-1 = -2.65 J.K-1.


!lG = Mfm (l ~ v,T) - T!lSm (l~v,T)
= 40886 - (368.15 x 108.41) = 974.86 J.
30. An infinitesimal crystal of ice is added to 10 gm of supercooled liquid water at
10 °C in an adiabatic container. What is Ml for the process if the system reaches equilibrium
at a fixed pressure of 1 atm? (The equilibrium state will contain some ice and will, therefore,

Prob. Phy. Chem.-15


226 PROBLEMS ON PHYSICAL CHEMISTRY

consist either of ice plus liquid at 0 °C or, of ice at or below 0 °C.) What is present at
equilibrium? Calculate D.S for the process. Given that,
t1 = 80 cal/gm. cp.t = I cal.K-I .gm-1
Soln. : 10 gm supercooled water at -10 °C is a non-equilibrium system. Addition of
infinitesinal of ice will take it to equilibrium. Since there is an adiabatic jacket, heat lost
for a process will be equal to heat gain. It will try to reach in equilibrium. The change
may be depicted as below :
IO gm super- 10 gm x gm ice
cooled water ~ water +
at -10 °C at 0 °C (10 - x) gm water at 0 °C
For moving from -10 °C to 0 °C, the system gains AH
= cp (Tz - T1) = Cp{O -(-10)}
= cp x 10.
CP for water is l cal.K-l .gm-1.
So, AH = 10 gm x l cal.K- 1.gm- 1 x lOK = 100 cal.
Thus, energy needed 100 cal. This must be supplied by conversion of x gm of liquid
. lOOcal
water to ice at 273.15 °K. Thus, x = = 1.25 gm.
80cal I gm
Thus, 1.25 gm of ice and 8.75 gm of water is present -at equilibrium at 0 °C.
Now, D.S = D.S 1 + D.52
- ~ cal.K- 1
273 15
= nC In ·
p 263.15 273.15

- ~ cal.K- 1
273 15
= IOcaI.K- 1 In ·
263.15 273.15

=, (+0.373-~)caI.K-
273.15
1.

= (+ 0.373 - 0.366) cal.K- 1 = + 0.007 caJ.K- 1•


31. 0.5 mole of an ideal monatomic gas initially at 5 atm pressure and 0 °C is allowed
to expand against a constant external pressure of 0·5 atm. Conditions are such that the final
volume is 10 times the initial volume; the final gas pressure equals the external pressure.
Calculate iv, Q, !!..U, AH, D.S and !l.G for the process. [C.U. 2005)
Soln. : In the given problem,
n = 0·5 mole n = 0·5 mole
p = 5 atm p = 0·5 .atm
Ti= 273·15 °K Vz = lOV
v = V1 Tz =T
/
From the given change, we get

- - =P2V2
P1V1
--
Ti Tz
.. T1 = T2
SECOND LAW OF THERMODYNAMICS 227

So, this is an isothermal and irreversible change.


For such a case, !J..U = 0.
:. w = -Q.
Now, from the 1st law of thermodynamics,
W = -Pext (Vz - V1)
= Pz W1 - Vz)

= Pz [nRT _ nRT]
Pi P2
= nRT[ P2 -
P1
l].
W = 0.5 x 8.314 x 273.15 (0.1 - I) J = -!021.94 J. = -1.02 kJ.
Q = -W = 1.02 kJ
Now, Mi = nCP dT = 0 [for ideal gas and for isothermal process]

!J..G = nRTln p2
P1
0 5
= 0.5 mol x 8.314 J.K- 1.moJ- 1 x 273.15 K tn( ; )

=- 2614.5498 J =-2.61 kJ,


v
6S = nR In - 2 = 0.5 x 8.314 x In( 10) = 9.5718 J.K- 1•
. Vi
32. I mole of an ideal gas at 2 atm pressure expands irreversibly and adiabatically into
vacuum to decrease its pressure to l atm. Calculate 6S for the change. [B.U. 2003]

Solo. : 6S = nR ln2· = 8.314 x ln(~)1.K- 1 = S.763 J.K-1•


~ 1 .
33. At -5 °C the equilibrium vapour pressure of ice is 3.012 mm of Hg and that of
supercooled liquid water is 3.163 mm of Hg. Latent heat of fusion of ice is 5.85 kJ.
mole- 1 at -5 °C. Calculate !J..G and 6S per mole for a transition.
HzO(l, -5 °C)~HzO(s, -5 °C)
Comment on the spontaneity of this process. [C.U. 2004]

Soln. : !l.G = RT ln( 33.163


·
012
)

3 012
= 8.314 J.K- 1.mol- 1 x 268.15K x tn( · ) = -109.0SJ.
3.163
Since equilibrium vapour pressure of ice is lower than that of supercooled water at
-5 °C, freezing is not a reversible process at that temperature. Hence, for calculation of
6S the following alternative reversible path can be conceived through which the change
can be done.
Rev. and isothermal Rev. phase change
HzO (l,P1,-5 °C) expans10n HzO (l,Ps,-5 °C) at const.Tand P HzO(s,Ps,-5 oq

!J..SI
228 PROBLEMS ON PHYSICAL CHEMISTRY

Assuming the vapour Behaves ideally,


Pl Mffusion
Msystem =Ml+ M2 = R In- - - - ' - -
P.1. T

=8.314J.K-'.mo1-'1n3.163 - 5.85x103J.mol-1
3.012 268.15K
= -21.409 J.K-1.moJ-1.

MJ(l~s) _ Mffusion 5850 J. mo1- 1


=- = -268.15K
--- = 21.816 J.K- 1.moJ- 1.
T T

Hence, Muniverse = Msurr + Msystem


= (21.816 - 21.409) J.K- 1.moJ- 1 = 0.407 J.K- 1.mot- 1•
Since llG for the process is negative at constant temperature and pressure, the process is
spontaneous. Positive value of Muniverse also indicates the same thing.
34. One mole of water vapour is compressed reversibly to liquid water at 373 K.
Calculate W, Q, Ml, !lS, !lG and M.
Mfvap at 373 K = 2258 J.gm- 1. [C.U. 2002]
Soln. : We know that, work (W) = -P(V2 - V 1).
Here, V2 = V1 and V1 = V8 .
:. w = -P <V, - Vg) = PVg [:. vg » v,1.
RT
Assuming ideal behaviour, V = nRT =
g p p
[n = I].
W =RT
= 8.314 x 373 = 3101.122 J.
llH = -2258 x 18 J = - 40644 J.
Now, llH = llU + ll(PV).
At constant pressure,
llH = llU + PllV
= llU + P(V1 - V8 ) = llU - PV8 = llU - RT
or, llU = Ml + RT.
. . Q = llU - W = Ml + RT - RT = Ml
or, Q = -40644J.

M = 406441
373K
= -108.965 J.K- 1.

!lG = 0, since the process is reversible at constant temperature and pressure.


I

M =- f Pdv = -P (V1 - V8 ) = PV8 = RT = 3101.122 J.


R
35. Calculate the change in entropy experienced by 2 moles of an ideal gas on being
heated from· a pressure of 5 atm at 50 °C to a pressure of I atm at 100 °C.
Given, CP = 9.88 cal.moJ- 1.deg- 1. [V.U. 2000]
SECOND LAW OF THERMODYNAMICS 229

Hints : Use formula given below,

t!.S =n[Cp In TT1 2


- Rln p 2
P1
]= 38.67 J.K- 1.

36. 0.5 mole water at l atm pressure undergoes the process :


H2 0(1, -IO °C)~H 2 0(s, -JO °C)
Compute t!.S and t:i.G for the process from the following data : specific heat capacity
of water and ice over the temperature range is 1.0 cal.deg- 1.gm- 1 and 0.5 cal.deg- 1.gm- 1
respectively, latent heat of fusion of ice is 80.0 cal.gm- 1 at 0 °C. [C.U. 2000]
Soln. : The process may be depicted as below :
Hp(l) H 20(1) 2 H 20(s) Hp(s)
-IO °C 0°C 0°C -IO °C
By the problem, n = 0.5 mole, .. m = 9 gm.
P = 1 atm
Cp.l = I cal.deg- 1.gm- 1
cp,s = 0.5 cal.deg- 1.gm- 1
z1 = 80 cal.gm- 1
t!.S = t!.S I + t!.52 + t!.53

t!.S I
=9 x 1 x In 273.15 _ 80x9 + 9 x O.S ln 263.15
263.15 273.15 273.15
= 0.335672 - 2.6359 - 0.167836
= -2.468064 = -2.468 cal/K = -10.326 J.K-1.
See Problem 14 for calculating t:i.G. [t:i.G = -85.49 J]
37. Calculate t:i. A for vaporisation of 0.1 mole of H 20 at I atm pressure and
100 °C. Given molar volume of Hp(l) at IOO °C .is 18.8 ml.mol- 1 and that of Hp(g)
is 30.6 litre.mol- 1. [C.U. 2000]
Soln. : By the problem, V
8
= 30.6 lit. moi- 1
= 3.06 lit for 0.1 mole.
and V1 = 0.0188 lit.mol- 1 = 0.00188 lit for 0.1 mole.
Now dA = -PdV - SdT.

At constant temperature M for the process given by M = J-PdV.


At constant pressure,
M = -P(V - V1)
8
= P(V1 - V )
8
= 1(0.00188 - 3.06) lit.atm
= -3.05812 lit. atm = -3.05812 x 101.325 J
= -309.864 J.
38. A sample of hydrogen is confined to a cylinder fitted with a piston of 50 cm2
cross section. It occupies 500 ml at 25 °C and 2 atm. What is the change in entropy when
the piston is isothermally lifted through 10 cm? [B.U. 1991]
Soln. : From the given data, the initial volume of the gas (Vi) = 500 ml and final
volume (V1) = Vi + (50 x 10) ml = 500 + 500 :;:: IOOO ml.
230 PROBLEMS ON PHYSICAL CHEMISTRY

Now, according to thr given problem-


~ = 500 ml V1 = IOOOml
P; = 2 atm 1 T = 298.15 °K
T; = 298.15 °1\.
From the above data we can calculate the number of mole of hydrogen gas in the
cylinder.
P.V 5 2
i.e., n =-'-' = 00x = 0.0409.
RT; 298.15x0.082x 10 3

1000
:. entropy change,Af = nR In VJ =0.0409x 8.314x ln( )1.K- 1=0.2357J.K- 1
V; 500
39. For a certain reaction !l.G (cal/mole)= 13580 + 16.IT log 10 T- 72.59T; fine! M
and t1H of the reaction at 27 °C. [C.U. 1983, '87; V.U. '97]
Solo. : From the given problem we get
!l.G = 13580 + 16.1 TloglOT - 72.59 T.
d
We know that, Af = - - (!l.G).
dT
d d
:. Af = - dT(!l.G) = - dT[l3580 + 16.l T log 10T- 72.59 T]

d d
= - dT (16.l Tlog 10 7) + dT(72.591)

d ( '6 l ) d
= - dT \ 2·.3~3 Tin T + 72.59 dT (1)

= -6.99088 !!.__
dT
(T In 7) + 72.59
.
= ~9088 [I + In 71 + 72.59
= -6.99088 [I+ ln300.15] + 72.59 = 25.721 cal.K- 1.mol- 1
= 107.62 J.K-I.mol-1.
Now, !l.G at 300.15 °K is-
!l.G = 13580 + (16.l x 300.15 x logl0300.15) - (72.59 x 300.15)
= 3763.638 cal/mol x 4.184 J/cal = 15747.06 J/mol.
· We know that,
!l.G= t1H - TAf at constant temperature
.. t1H = !l.G + TAf
or, t1H = 15747.06 J.mol- 1 + (300.15 x 107.61) J.mol- 1
= 48046.2 J.moi- 1 = 48.05 kJ.mol-1.
40. Calculate !l.H and Af when I kg bar of copper of I 00 °C is placed in 2 kg of
water at 0 °C in an insulated vessel maintained at I atm pressure. Heat. capacity of H20(1)
is 4.1 J.K- 1.gm- 1 and Cu is 0.4 J.K- 1.gm- 1• Can you calculate !l.G of this change?
Solo. : Let final temperature be t °C as a result of mixing.
:. by the given data (1000 x 0.4) (100 - t)
= 2000 x 4.l(t - 0) that is 0.4 x (100 - t) = 8.2 t
or, 40 = 8.6 t.
SECOND LAW OF THERMODYNAMICS 231

t = 4.6511628 °C = 277.801628 °K = 277.8 °K.


277 8
•• ~Cu = nC In · = -118.03 J.K- 1•
p 373.15
277.8
For water, ~ = 2000 x 4.1 In - - = 138.42 J.K- 1•
373.15
.. ~total = 20.39 J.K-1
f>.Htotal = f>.Hcu + dHH20
f>.Hcu = 1000 gm x 0.4 J.K- 1.gm- 1 x (277.8011628 - 373.15)K
= -38139.53 J.
dHH o= 2000 x 4.1 x (277.8011628 - 273.15) = 38D9.53J.
2
.. f>.H = =
0 and dG f>.H -T2S2 + T1S 1 and it (dG) cannot be calculated because
value of at least one S is required.
41. The standard Gibbs free energy (dG 0 ) for a certain reaction at different temperatures
(T °K)· is given by
dG 0 (cal) = -126400 - 5.44 Tin T + 104.7 T.
Calculate the standard enthalpy change for the reaction at 2000 °K. [C.U. 1977]
Hints : See Problem 39. Only remember that in Problem 38, there is log 10, but here
In = loge.
42. Two mole of a monatomic gas initially at 4.0" bar and 47 °C undergoes reversible
expansion in an insulated container. Calculate the temperature at which the pressure is
reduced to 3.0 bar. [GATE 2001]
Soln. : We know that,

where T1 is the temperature of state 1, P1 and P2 are the pressures of states 1 and 2 respectively
and T2 is the final temperature. By the problem, T 1 = 47 °C = 320.15 K, P 1 = 4 bar and
P2 = 3 bar. ·

:. Tz = T ( P2
I P1
)~ =T
I
( P2
P1
)o.4 '
R
where = -5- = 0.4.
-R
2
3)0.4
.. Tz = 320.15 x ('4 = 285.35 °K.
43. Establish the condition for spontaneous vaporisation of water, given f>.H = 9590
cal mole- 1 and ~ = 26 e.u. for the process. [B.U. 1982]
Soln. : The condition for any spontaneous process is that dGP.T < 0.
Now we know that G =H - TS.
At constant temperature dG = dH - T~.
232 PROBLEMS ON PHYSICAL CHEMISTRY

Thus, for spontaneous vaporisation, tJf - TtJ.S < 0 [ :. t:i.G P.T < 0]
tJf
or, T >
tJ.S

or, T > 9590 0 K


26
or, T > 368.846 °K or, T > 368.85 °K.
44. Describe a reversible process by which one mole of water at I 00 °C can be converted
to vapour at 0.5 atm and 100 °C. Calculate tJf and t:i.G for the process. How far these
quantities would be affected if the operation were performed irreversibly? Assume water
vapour to behave ideally and latent heat of vaporisation of water 540 cal. gm- 1. =
[C.U. 1970]
Hints : The problem can be depicted as following :

Hp(I)
100 °C, I atm
I
r---+
H 20(v)
JOO °C, I atm - 2 H 20(v)
100 °C, 0.5 atm

tJf = !::i.H1 + tJ/2.


Now, t:i.H 1 = 18 x 540 cal.mote- 1
t:i.H2 = 0, due to isothermal change.
Similarly, t:i.G = t:i.G 1 + t:i.G2•
p
Now, t:i.G 1 =0 and t:i.G 2 = RTin-1
Pz
H and G are state functions and thus tJf and t:i.G would remain same if initial and
final states are same irrespective of the path.
45. Calculate the change in Gibbs potential when 36 gm water initially at 100 °C
and 10 atm pressure are converted to vapour at 100 °C and 0.01 atm pressure. [C.U. ~971]
Soln. : The process can be written as below :

H 2 0(1) l Hp(I) 2 Hp(v) 3 Hp(v)


!--------. r--. --+
100 °C, 10 atm 100 °C, l atm 100 °C, l atm 100 °C, 0.01 atm

t:i.G = t:i.G 1 + t:i.G2 + t:i.G3


Now, t:i.G 1 = f VdP = V(P 2 - P 1) lit.atm.

= 0.036(1 - 10) (volume of 36 gm water = 36 cc = 0.036 I)


= - 0.324 lit.atm x 101.325 J/lit atm = -32.83 J.
= 0, since it is an equilibrium process.
P? 1 1 O.Olatm
= nRTln-- = 2 mol x 8.314 J.deg- .mot- x 373.15 deg x I n - - -
~ I~
= -28573.875 J
:. t:i.G = -28606.71 J.
46. 5 gm-moles of water initially at 27 °C are converted to a final state of vapour
at 227 °C, the conversion being effected under I atmospheric pressure. Assuming the vapour
to behave ideally, compute the total change in entropy.
SECOND LAW OF THERMODYNAMICS 233
. [Heat capacity of water= l cal/gm; heat capacity of water vapour= 0.40 cal/gm; latent
heat of vaporisation of water = 540 cal/gm.] [C.U. 1972]
Hints : The process can be depicted as following :

H20(1) -L H20(1) L HzO(v) -L H20(v)


27 °C, l atm 100 °C, l atm 100 °C, l atm 227 °C, l atm

Aftotal =Aft+ Af2 + Af3


T2 mxl T2
Now, Af 1 = mC.ln-; Af2 = _ _e; Af3 = m.C.ln-.
~ T ~
47. Calculate the entropy change when one kilogram of water at 27 °C is converted
to super heated steam at 200 °C under constant atmospheric pressure. [Specific heat of liquid
water = 4180 J/kg. Specific heat of steam = ( 1670 + 0.49 1) J/kg at T °K and latent heat
of vaporisation = 23 x 105 J/kg.]
. Solo. : The process can be written as below :
H20(1) ~ H20(1) ~ HzO(v) ~ H20(v)
27 °C, l atm 100 °C, l atm 100 °c, 1 atm 200 °C, l atm

So, Af = Af 1 + Af2 + Af3.


T 373.15
Now, M 1 = mC ln-1. = kg x 4180 J.kg- 1 x I n - - = 909.978 J.K-.t.
P Ti 300.15

= l x 23 x 105 J.K-t = 6163.74 J.K-t.


373.15

T
= 1670 In -1. + 0.49(T2 - T1)
Ti

= 1670 In 473· 15 + (0.49 x 100) = 445.5 J.K- 1.


373.15
M = 7519.2 J.K-1.
48. Calculate the change in Gibbs free energy per mole for the process :
HzO (I, -5 °C) ~ H20 (s,-5 °C)
The vapour pressure of supercooled water at -5 °C is 3.163 mm of Hg, and the vapour
pressure of ice is 3.012 mm of Hg. [C.U. 2002]
Hints : See Problem 33 for calculation of !lG.
49. One mole of He gas is heated from 200 °C to 400 °C at a constant pressure of
I atm. Assuming He to be a perfect gas, calculate !lG, !lH and Af for the process. Given
that the absolute entropy of He at 200 °C is 135 J.K- 1.mole-t. [C.U. 2003]
Hints : See Problem 15.

Af = nC In T2
P Ti
234 PROBLEMS ON PHYSICAL CHEMISTRY

Here n =l and CP = %R, 7'2 = 673.15 K, T1 = 473.15K.

llS = 7.33 J.K- 1


5 5
= ZR(T2 - T 1) = x 8.314 x 200 J = 4157 J = 4.157 kJ.
2

Tz
= !lH - T2(S 1 + nCPln - ) + T 1S 1•
T1
In this problem, S 1 = 135 J.K- 1.mol- 1.
50. Calculate the change in entropy when l 0 gm of tin is heated from 293 K to 573 K.
The melting point of tin is 505 K. Its latent heat of fusion is 14 cal.gm- 1 and specific heat
of solid tin and liquid tin are 0.055 cal.gm- 1 and 0.064 cal.gm- 1 respectively.
[C.U. 2005]
Soln : The process can be depicted as below :

Sn(s), 293 K I Sn(s), 505 K 2 ISn(!), 505 Kj 3 ., j Sn(I), 573 KI

Here llStotal = Af1 + Af2 + !lS3.

T
2 = IO x 0.055 x I n - caI.K- 1 505
Now, !lS 1 = m.Cps· ln-
· T1 293
= 0.29941 cal.K- 1 = 1.2527 J.K- 1•
4
= IOxl cal.K- 1 =0.2772 cal.K- 1 = l.15980 J.K- 1•
505
573
= IO x 0.064 x In - cal.K- 1 = 0.080849 cal.K- 1 = 0.3383 J.K- 1
505
.. !lS = 2.75 J.K- 1•
51. 5 moles of an ideal gas, initially at 50 atm and 300 K is expanded irreversibly
where the pressure suddenly drops to IO atm. The work involved is 4000 J. Show that the
final temperature is greater than a reversible adiabatic expansion to the same pressure. If
Cv ::; l.5R, calculate the entropy change during the irreversible expansion. [B.U. 2002]
Soln. : Let the irreversible work be adiabatic also.
By the problem, Cv = I.SR.
Now, W = nCvdT = nCv(T2 - T1).
Here, n = 5 mol, T1 = 300 °K, W = -4000 J
.. -4000 J = 5 mol x 1.5 x 8.314 J.K- 1.mol- 1 (T2 - 300)
or, T2 = 235.85 °K.
Now, under adiabatic reversible condition,
R

T1 = T1 ( ~~ YP ·
By the problem, T1 = 300 °K, P 2 = ID atm, P 1 = 50 atm.
SECOND LAW OF THERMODYNAMICS 235

:. 10)- = 300 x (0.2)


T2 = 300 x (50 Cp
0 ·4 = 157.59 °K.
So, the final temperature for the given process in irreversible adiabatic process is greater
than that in reversible adiabatic process.
In the irrversible process,
Ti = 300°K, T2 = 235.85 °K .
Pi = 50 atm, P2 = 10 atm.
T p2
.. !!.S = nC \n--1. - nR ln- [Here cp = CV +R = l.5R +5 = 2.5R]
P T1 P1
235 85
= 5 x 2.5 x 8.314 In· J.K-i - 5 x 8.314 ln_!.2.J.K- 1
300 50
1
= -25.0029 J.K-i + 66.90 J.K- = 41.9 J.K-1.
N.B. : If irreversible work is written as Pexr(V2 - Vi), using Pext = P 2 = atm, 10
the equation will be wrong, because it has not been stated that work is done against a constant
pressure of atm. 10
52. A mass of water at T is mixed isobarically and adiabatically with an equal mass

. of water at T'. Show that the entropy change is positive and is given by !!.S =2mCP In T-;;{;, .
2-vTT'
[Here m is the mass of water.]
T+T'
Solo. : The final temperature will be
2

Since there ism gm of water in each, the entropy change form gm from Tto T + T' and
2
T+T'
T' to --can be added.
2
· T+T' T+T'
.. !!.S = mCP I n --
2T
+ mC l n - -
P 2T

-m T +-
_ . C In ( - +-
T'xT- T')
P 2T 2T
2
(T+T') T+T'
= mCP In = 2mCP In -::r:::=:;; [Proved].
4TT' 2-yTT'
This quantity is positive. This can be shown by the following :
(T + T') 2 - 4T T' = (T - T')2
Now since, (T - T')2 ~ O,
.. (T + T') 2 - 4T T' ~ 0
or, (T + T') 2 ~ If' T'
2
or, (T+T') >
_ 1.
4TT'
:. !!.S is positive.
236 PROBLEMS ON PHYSICAL CHEMISTRY·

53. l mot of H2 0 at 1 atm pressure undergoes the following process H2 0 (I, -10 °C)
~H 0
2 (s,-10 °C) in a thermostate maintained at -IO °C. The heat of fusion of water is
334.720 J.g- 1 at 0°C and specific heat capacity of water and ice over this temperature range ·
iJ 4.184 J.c- 1.gm- 1 and 2.092 J.c- 1.gm- 1 respectively. Is the process spontaneous?
Soln. : See Problem 36.

:. Af = 18 x 4.184 In 273.15 - 334.720x 18 + 18 x 2.092 In 263.15


263.15 273.15 273.15
= -20.653 J.K- 1•
263.15

Af/263.15= Af/273.15 + f
273.15
!:lCpdT

= -334.720 + (2.092 - 4.184) x (-10) = -313.8 J.g- 1•

Af = _ Aff263.15 (l~s) = 18x313.8 = 2 1. 46 J.K-l.


surr 263.15 263.15
Afuniverse = 21.46 -20.653 = 0.807 J.K-1.
Aff for the process can also be calculated in the following way :
Aff263 . 15 = 18 x4.184x(273.15-263.15)-334.720 x 18 + 18 x 2.~92(263.15 - 273.15)
= -5648.4 J
_ 5684.4 _ -l
and Afsurr - 263.15 - 21.46 J.K .

This process is spontaneous since for this process Afuniverse is positive.


54. Calculate Af when 1 mol of ice at 273 K is converted to steam at 373 Kat 1 atm.
u1 = 80 cal/gm, le = 540 cal/gm, CP = 1 cal/gm)
Hints :

Ice
273 K, 1 atm
Af1 Water Af2 Water
273 K, 1 atm 1--+ 3733 K, I atm
Af3
-- Vapour
373 K, 1 atm

11x18 373 le X 18
Af = M 1 + M 2 + M 3 = ~ + 18 x Cpln +
273 373
1
= 36.95 cal. deg- = 154.6 J.deg- . 1

55. Find !:lU, Aff, Af, !:lF (or L\A), !:lG in expanding 1 litre of an ideal gas at 25 °C
to i 00 litres at the same temperature.
Hints : !:lU = Aff = 0, Af = 38.3 J.K- 1, L\A = !:lG = -11.42 J.K- 1.

56. Calculate the !:lG; for liquid water at 25 °C. Given that Af/ of H20(1) is
0

-285.8 kJ and standard molar entropies are S~ 2 o(i) = 69.9J/k, S~z(g) = 130.61/K and s;z(g) =

205 J/K. [V.U. 2007)

Soln. : H2(g) + '21 0 2(g) = HzO(I), T = 25 °C = 298.15 K.

Given, Aff~zO(I) = -285.8 kJ, s~zO(I) = 69.9 J/K,


SECOND LAW OF THERMODYNAMICS 237

s~z(g) = 130.6 J/K, s~z(g) = 205 J/K


t:.G~ = t:.H° - Tt:.5°
0 0 0

= t:.f/HzO{I) - !':.
. H Hz(g) J_ !':.HOz(g)
-.2 - 298.15 X !':.S 0

0 1 0. 0 )

= -285.8 kJ- 298.15 K SH 2o(i)-SH 2 (gJ-2So 2 (g)


(

(since, !':. H~ 2 (g) = 0 and !':. H~ 2 (g) = 0)

= -285.8 kJ - 298.15 K(69.9 - 130.6 - J_ x 205) J/K


2
= -237141.92 J = -237.14 kJ,
57. 2 moles of He is heated from 473 K to 673 Kat a constant pressure of 1 atm. Assuming
He is a perfect gas, calculate t:.G, t:.H and t:.S for the process. Given that 73 = 135 J.K- 1.s;
mole- 1. Is this process spontaneous?
. 5 673
Hmts : t:.S Y 1 = 2 mol x - x 8.314 J.K- 1.mot- 1 x I n - = 14.66 J.K-1
ssem 2 473

5
t:.H = 2 x -x 8.314 (673 - 473) = 8314 J.
2
t:.G = t:.H ~ (T2S2 - T1S1)

5 673
= 8314 - 673 (2 x 135 + 2 x - x 8314 In-)+ 2 x 473 x 135
2 473
= -55552.18 J = -55.55 kJ,

t:.S = - Qp = - t:.H = - 8314 J.K-1 = - 12.35 J.K-1


sur T T 673
Hence, t:.Suni = 2.31 J.K- 1.
So the process is spontaneous.
58. An ideal gas expands isothermally at 27 °C, but somewhat irreversibly, producing
1500 J of work. The entropy change is 10 J.K- 1. Calculate the degree of irreversibility,

i. . ewactual
.,--
W,ev
Hints : Wactuat = -1500 J.
t:.U = 0, Wrev = -Tt:.S = -IO x 300 = -3000 J.

wactual = 0.5.
wrev
59. 1 mol of water vapour at 373 K and 2 atm undergoes the following process :
HzO(g), 373 K, 2 atm)~H 2 0 (I, 373 K, 2 atm)
Calculate t:.G, t:.H and t:.S for the process assuming the vapour to be ideal and
liquid to be incompressible. ,
1
(CP for vapour= 7 cal.deg- .mol- , CP for liquid ,;,18 cal.deg- 1,mot- 1, Le =IO kcal.mol- 1).
1
238 PROBLEMS ON PHYSICAL CHEMISTRY

Hints: The process is at constant P. Thus, for the process, Ml= Qp =-10 K.cal.mo1- 1.
The process can also be carried out in the following reversible steps ..

H20 (g, 373 K, 2 atm)----H 20(g, 373 K, 1 atm)

H20(1, ·373 K, 2 atm)--- - H20 (I, 373 K, 1 atm)


! 2

Aff 1 = 0, Af 1 = R ln p:p = 1.38 cal.deg-1.moJ- 1.

AG 1 =RT ln Pz = -514.7 cal.mol-1.


Pi
Aff3 = AG 3 = Af3 = 0, since water is incompressible.
Aff
AG 2 = 0, Aff2 = Qp = -10 kcal.mole- 1, Af2 = - - -
2
= -26.81 cal.K- 1.mole- 1.
T
AG = - 514.7 cal.moJe- 1,Aff = -10 k.cal.mole-1, Af = -25.43 cal.K- 1.mole-1•
60. In what proportions should n-hexane and n-heptane be mixed to achieve the greatest
Afmix; assumingg ideal mixing. [B.U. 2004)
Solo. : Afmi./mol = -R (X 1lnX 1 + X2InX2) = Af
Now, X1 + X2 = I, :. X2 = 1 - X1.
Thus, Af = -R [X1InX 1 + (1 - X1) ln(l - X1)].

.. -dAf
dX
= -R [lnX1 +I -.ln(l - X1) - l].
1

·For maximum value of Af.

-dAf
dX
= 0 = lnX1 - In (1 - X1)
1

or, X I = 1 - X 1• .
••
xI -- 2.
1.

:. _n_
1_ = 1 or, I + nz = 2.
n1 +nz 2 n1

Thus, nheptane : nhexane = I : l.


Hence, nhexare and nheptane should be mixed in the same proportion to achieve maximum
AfmiX' .
61. 100 gm of water cools from 100°C to 25 °C when kept in air. Calculate Afuniverse·
[V.U. 2007]
lOOgm
Solo. : Given, nHzO = 18gm I mole
= 5..56 mole.

T1 =
100 °C = 373.15 K
T2 = 25 °C = 298.15 K
SECOND LAW OF THERMODYNAMICS 239

2 298 · 15
H ence, A<'
ti..} system = n CP I n
.
-T
~
= 5.56 mo I x 18 caJ.K- 1.mol- 1 x In
373.15
= -22.46 cal.K- 1
Q = nCP(T2 - T1)
= 5.56 mol x 18 cal.K- 1.mot- 1 x (1298.15 - 373.15)K = -7506 cal.
_ 7506cal _ -I
.. Afsurr - 298.15 K - 25.18 cal.K .

= Afsystem + Afsurr .= 2.72 cal.K-l ..


• · !l.Suniverse
62. 2 mol of benzene vaporizes at 80 °C (boiling point). Calculate M, !lG, W, Q, AU,
/llf, Af if final pressure is 0.5 atm at 80 °C. Given that vapour behaves ideally and latent
·heat of vaporization of benezene is 422.6 J.gm- 1•

Hints :
c6Ift; (l) 80 °C c6Ift; (g) 80 °C c6H 6 (g)
2 mol latm 2 mo! 0.5 atm 2 mol
/lH = llH 1 + llH2 = 422.6 x 78 J.moJ- 1 x 2 mol + 0 = 65.9 kJ.
p1
W = (W 1 + W2) = - P(V - V1) - nRT ln-
g P2
p2
= P(V1 - = - 9.94 kJ,
V ) + nRT In -
8
Pi
!lU = llU 1 + llU2 = llU 1 = llH 1 - P!lV = llH 1 - nRT = 60.03 kJ,
p
1
Q = Q 1 + Q2 = llH 1 + nRT In - ·
. P2

Af = !lHi - nR In p 2 = 198.13 J.K- 1•


T P1
p
llG = llG 1 + llG2 = 0 + nRT In _1.
Pi
p
M = M 1 + M 2 = -nRT + nRT In _1. (·: M 1 = !l.G 1 - P!lV).
P1
63. A heat engine operates through the following Carnot cycle between T1 = 273 K
and T2 and produces 418 J of work per cycle. Calculate Q1, Q2 , Q 3, Q4 and T2.

160 250
-I
S(J.K ) - .
Hints :
Step 1 :·Reversible Isothermal expansion, Q1 = Af 1 x T2 = 90 T2.
240 PROBLEMS ON PHYSICAL CHEMISTRY

Step 2 : Reversible lsoentropic expansion, Qi = 0 (Rev adiabatic process).


Step 3 : Reversible Isothermal contraction,
Q3 = Af3 x T1 = -24.57 kJ.
Step 4 : Reversible isoentropic (adiabatic) contraction,
Q4 =0
QI + Qi + Q3 + Q4 = 418 = 90 Ti - 24570
Ti = 277.6 K = 278 K
.. QI = 90 X 278 J = 25.02 kJ.
64. Calculate t'ilfm and fl.Sm for the process :

HiO (/) HiO (s)


T= 263 K T=263 K
P=O.I MPa P=O.I MPa
Assume that at 273 K the molar enthalpy of fusion of ice is 6006 J. mol- 1, the heat
capacity Cp.m_(s) of ice is 38 J.K- 1 and the heat capacity Cpm.<l) of liquid water is 76 J.K- 1
mol- 1. Consider the heat capacities to be constants. (JAM 2010]
Hints :
Hp (l) Hp (l) Hp (s) Hp (s)
T = 263 T = 263 T = 273 T =263
P = 0.1 MPa P = 0.1 MPa P = 0.1 MPa P=O.lMPa
· Afm = -20.58 J.K- 1 mo1-1
t'l.Cp = CP,m(s) - CP,m(l) = -10 J.K- 1
t'illi73 (l -t s) = - 600 J.mo1- 1
t'illi73 (l -t s) = - t'illi73 (l -t s) + t'l.Cp (268 - 273)
= -5996.38 J.moI- 1.
65. Consider a thermally insulated container divided by a partition into two compart-
ments in such a way that left copartment is twice as big as right one. Both the compartments
contain same number of moles of ideal gas at temperature T. The partition is now removed.
Calculate
(a) the total change of entropy if the gases are identical;
(b) the total change of entropy if the gases are different.
Soln. : Let the volume and pressure of right compartment be V and PR respectively.
By the problem, initial state is as follows :
where n = number of moles of gas,
T,n T,n PL = pressure of left compartment.

Thus, PRV = nRT (l)


PL 2V = nRT
nRT P
or, PL= - . - = -1i. [from eqn. (l)]. (2)
2V 2
Let final pressure be P after removing the partition.
SECOND LAW OF THERMODYNAMICS 241

Thus P(2V + V) = 2nRT


2 nRT 2
or, P= - - - = -PR [from eqn. (l)] (3a)
3 v 3
4 (3b)
=-PL [from eqn. (2)].
3
(a) For the right compartment,
p 3
MR= nR!n-1!..
p
= nR In- [from eqn. (3a)]
2
For the left compartment,
p
ML= nRln~ = nRln-3 [from eqn. (3b)]
p 4
Thus, total change in entropy, when the gases are identical,
3 3 9
M =MR+ ML= nR lnl + nR ln = nR ln .
4 8
(b) When the gases are different, partial pressure of the gas in right compartment
after mixing : .
, n P PR
PR = P. - - = - = - (4a)
n+n 2 3
and partial pressure of the gas in left compartment after mixing :
' p 2
PL= -
2
= -PL.
3
(4b)

Thus, total change in entropy when the gases are different,


PR pl
M =MR+ ML= nR1n-, +nRln-:
PR PL
3
= nR1n3 + nR1n- [from eqn. (4a) and (4b)]
2
9
= nR1n-.
2
66. Suppose a container of volume V is divided into two equal compartments by a
thin partition with a small hole of area A in it. If it is filled with an ideal gas at temperature
T such that the initial pressure on the left compartment is PL (0) and that on the right side
is PR(O),
(i) find the pressure of the gas in the left compartment as a function of time.
(ii) calculate M of the whole gas after the equilibrium has been reached

PL(O) PR(O)
Soln. : Initial State : T T
v
- -v
2 2
If NL(O) and NR(O) are initial numbers of gas molecules in left and right compartments
respectively,
Prob. Phy. Chem.-16
242 PROBLEMS ON PHYSICAL CHEMISTRY

PL(0)2
v = NL(O) kT (I)

v
2 = N R(O)
P R(O) kT. (2)

Let total number of molecules be N which remains constant.


Thus, NL(O) + NR(O) = N. (3)
Let NL and N R be number of molecules in two respective compartments at any instant.
Hence, number of molecules effusing through hole of area A from left to right
compartment in time dt

= dN 1 = ±~ vAdt. (4)

2
and similarly number of molecules effusing from right to left compartment in time dt

±
= dN2 = ~ vAdt. (5)

2
Here v = average velocity.
Net change in number of molecules in left compartment in time dt,
dNL = -dN1 + dN2
=_!NL vAdr+! NR vAdt
4 v 4 v
2 2

N (N-N )
= __ L vAdt + L vAdt (since, NL+ NR = N)
2V 2V

NLvAdt N _Ad
= +-v t. (6)
V 2V
Due to effusion corresponding change in pressure in the left compartment = dP L'

Hence, dP L
v = dNL kT.
2
Let the final pressure in both compartments be P when effusion no longer changes
the pressures of the compartments.
Hence, PV = NKT = [NL(O) + NR(O)] kT

= P 1 (0)~ + PR(O)~ [from eqn. (I) and eqn. (2)]


~ 2 2
I
Thus, P = °2[PL(O) + PR(O)].

Hence, from eqn. (6) we obtain

dp -V- -_- PLvAdt +--v


P _Ad '·
t
L 2KT 2kT 2kT
SECOND LAW OF THERMODYNAMICS 243

or,

vAt
PL(t) =P-[P-PL(O)] e v.

I
= 2[PL(O)+PR(O)] [I I ] --
- 2Pi(O)+lPR(O)-PL(O) e v.
vAt

Hence, at time t pressure at the left compartment,


_ vA1
1 1
PL(t) = 2[PL(O)+PR(0)]-2[PR(O)-PL(O)]e v [from eqn. (7))

(ii) D.S = D.SL + D.SR =NL(O)kln PL(O)


p
+NR(O)kln PR(O)
p

_ V [ 2PL(O) 2PR(O) ]
- - PL(O)ln ( ) ( ) +PR(O)ln ( ) ( ) [from eqn. (7))
2T PL 0 +PR 0 PL 0 +PR 0
· Miscellaneous Problems

1. Show that ( as ) = ~CV '


aP v a.T

as) _ cp
( ap P a.VT
Solo. : We know that, dV = dQ - PdV.
For I mole of gas undergoing reversible change,
dU = TdS.- PdV

or, TdS = CvdT + [(~~l +P ]dv


Again, (au)
av
= r(aP) -
r ar v
P.

:. TdS = + r(aP)
CvdT dV.
ar v

:. dS =
. r (aP)
dT +
CV-
ar . -
v
.dV.
244 PROBLEMS ON PHYSICAL CHEMISTRY

When dS = 0, (av) cv
aT s = -T(·~)
1

aT v

= - ~Cv
aT
.
[since
(ap) <XJ
aT . v =p .

By Maxwell's relation,

(~~)s = -(~~)v·
.. (~~)v = -(~~)s
.. (as)
aP v
~C" =
aT
[Proved].

Again, TdS = dH - VdP

or dS
'
= CPT
-dT - (av)
-
aT p
dP.

cp
=--
TaV

as) = !:.L
:. (av avrp
[Proved].

2. Show that for an ideal gas (av)


as p
= RT
and
RT
VCv

Soln. : dS = nCP-
dT
T
- (av)
-
aT p
dP

or, (as)
av p
= nCp
T
(aT).
av p

PV p
For an ideal gas, T = , ..
nR nR
SECOND CAW OF THERMODYNAMICS 245

Thus
'
(av) _I_(av)
oS p nCp oT
=
P
=
RT
PCp

:. (oV)
as p = Pep
RT [Proved].

. dS
Agam, dT
= nCv- + (oP)
- dV
T oT v

or, (as) v =
oP
nCv (oT) .
T oP v

For an ideal gas, ( oT) = V


oP v nR

Thus, ( ~; )v = VCv
RT
[Proved].

3. Find the value of µ1 T for a gas obeying (i) P (V - b) =RT, (ii) ( P + : 2 ) V= RT.

Comment on the results.

T(av) - v
Ans. : µJT = ( oT) = ___;,_oT--'-'p__
oP H Cv
(i) P(V - b) = RT
RT
or' V = - p + b.

(oV)
aT p
= R. Hence, T(oV) - V = RT - V = -b.
P aT p P
.• µJT = -b < 0.
Temperature will always increase on reduction of pressure since the gas has only
repulsion but not attraction between its molecules. So it does not have any inversion
temperature.

(ii) (P+ : 2 )v =RT

or, V = -RT - -a .
(replacing V by RT)
p
P RT
2
Hence, T(oV) - V = a
oT P RT
2a
•• µJT = RT > 0.
Temperature will always decrease on reduction of pressure, since the gas has only
attraction but not repulsion between its molecules. Result indicates that it does not have
any inversion temperature.
246 PROBLEMS ON PHYSICAL CHEMISTRY

4. Find out the values of the following c!ifferentials in terms of CP' Cv, a, ~ and
P, V, n, T :

(a) (~~l· (b) (~~)P, (c) (~~)T, (d) (~~)T, (e) (~~)v, (f) (~~t·
Soln.:(a)(au) =T(as) =Cv·
"dT v aT v
(b) dU = TdS - PdV

( au) _ T (as) _ P (av)


"dT p - aT p aT p

= CP - PaV [since, a = _!_(av) ]


V aT p

(c) dU = TdS - PdV

( au)
av T
= T (as) _ p = T (aP) - p =Ta - P [by Maxwell's relation]
av T oT v p
(d) dU = TdS - PdV

( au) =T(as) -P(oV) =-P(~V) -T(av) [byMaxwell'srelation]


aP T ap T ap T oP T aT p .

= f3PV - aVT, sin~e ~ = __!_(av\ .


V ap )T
(e) dU = TdS - PdV

(au)ap v = T (as)
aP. v = -T (av) .
aT s [by Maxwell's relation]

=Cp ! [·:(~~\ =-~; l


(f) dU = TdS - PdV

( -au)
av p
= T (as)
-
av p
- P = T (aP)
-
aT s
.
- P [by Maxwell's relat10n]

= ~~ - P [·:(~~)s =~ ac~ l
5. Find out the values of following differentials :

(i) (aH) , (ii) (aH) , (iii) (aH) , (iv) (aH) , (v) (aH) •. (vi) (aH) .
aT p aT v · av T aP T aP v av p
Hints : dH = TdS + VdP and then proceed as Problem 4.

(i) CP, (ii) ( Cv + V ~}(iii) {J (aT- l), (iv) V(l - aT), (v) ( V + ~. Cv} (vi) ~~.
SECOND LAW OF THERMODYNAMICS 247

6. Draw Carnot cycle diagramatically in V vs. P, T vs. S, S vs.T, P vs.T, V vs.T, U


vs. S and S vs. U with reference to P vs. V diagram. Indicate each step.
Reference plot :

-
3

v
Process Pressure (P) Volume (V) Temperature (T) Entropy (S) Internal energy (U)

1. Isothermal Decreases Increases Const. Increases Const.


rev. expansion
2. Adiabatic. rev. Decreases. Increases Decreases Const. Decrases
expansion
3. Isothermal and
rev. contraction Increases Decreases Const. Decreases Const
4. Adiabatic rev. Increases Decreases Increases Const. Increases
contraction

Thus, the other diagrams would be as follows


l.V vs. P

B
3
vf
A

- p

A B

2. T vs. S
rl 4 2

D 3 c

- s
248 PROBLEMS ON PHYSICAL CHEMISTRY

A
D

pi 3
3. P vs. T
B
c
- T

4. V vs. T
vj 3

-
A

T
A 2 B

5. S vs. T
sl 3

D 4 c

A
- T
I B

6. U VS. S
ul 4 2

D 3 c

A
- s
I B

s vs. u
7.
Tl 4 2

D 3 c
- s
SECOND LAW OF THERMODYNAMICS 249
Similarities between T vs S and U vs S diagrams, and S vs T and S vs U diagrams
are obvious since for ideal gas U is a function of T only.

7. Prove that for an ideal gas, . (au)aT P = Cv and (aH)


aT v = Cp.

Soln. : dU = TdS - PdV

or, (au) = T(as) - (av)


aT p aT p
p
aT p
= Cp - R = Cv.
[since, for ideal gas, PV = RT]
or, P (av)
aT P
= P. R = R.
p
Again, dH = TdS + VdP

or, (~~)v = T (~:)v +V (~~)v = Cv + R = Cp,


since, for ideal gas, v(ap)
aT v
= V. R = R.
V
It can also be proved from lst law considering,
H = U + PV
(aH) v = (au) v + v (aP) v =
aT aT aT
Cv + R = Cp [Proved].
And U =H - PV

or, (~~t = (~~t - (~~t


P = Cv'[Proved].

8. State Trouton 's rule.


Ans. : Trouton's rule state that molar entropy of vaporization of various liquids at
the standard boiling point (the boiling point at I bar pressure) is same and it is about
88 J.K- 1.mol- 1•
9. Joule-Thomson process is an isoentropic process-'--Comment.
Ans.: Joule-Thomson process is an irreversible adiabatic process. So entropy increases

in this process, since for an irreversible process dSsystem > dTQ and here d'Q = 0. It is

rather an isoenthalpic process.


10. Change of Gibbs free energy for a process at low temperature at a constant pressure
is given by :

!lG =a + bT + cT2 , show that b


aT p
= 0 and
aT p
( a!lG) = (aw)
have values equal

in magnitude and opposite in sign and both the values tend towards 0 as T--tO.
[B.U. 2006]
Ans. : !lG = a + bT + cT2
250 PROBLEMS ON PHYSICAL CHEMISTRY

-f.S = ( "d!iG) = b + 2Tc.


"dT p

.. f.S =- (b + 2Tc) and Lt !J.S


T-40
= -b.
Now, according to the third law of thermodynamics, Lt !J.S
T-40
= 0.
:. b = 0.
Thus, /iG =a + cT2 and ( d!iG) = 2Tc.
"dT P

Aff = !iG - T
·
(d!iG)
"dT p
=a + cT2 - 2cT2 = a - cT2.

:. ("d!iH) = -lTc, :. ("d!iG) = _("dAff) = 2Tc.


"dT . p "dT p l "dT p

.
Thus, Lt
r_,o (-"d!iG)p
Lt ("()Aff)
ar- = T_,o --
ar = 0. p
CHAPTER 7

COLLIGATIVE PROPERTIES OF SOLUTION

Required Formulae
1. Osmotic pressure (0) = RTC
R ~ universal gas constant.
T ~ temperature

C ~ concentration in molarity (M) = n2


v
where n2 is the number of moles of solute and V is volume in dm 3.

2(a). po -P r = x .. , where Po is the vapour pressure of pure solvent, P is .the


-~
vapour

pressure of solution and x 2 h the mole fraction of solute.


2(b). For ideal solution mixture of two liquids A and B, total vapour pressure
p = pA + pB = XA p: + XB p~
If x~•P and x~•P are mol.e fractions of A and B respectively in vapour state,
p _ X vap p p _ X vap p
A- A '8- B ·
o2
3. t:i.Tb = RTb , where m is concentration in molality and is given by
10001.
IOOOW2 · .
m = M 2 xW1
, where M 2 ~ molecular weight of solute.

W2 gm of solute is dissolved in W1 gm of solvent.


!':i.Tb ~ elevation of boiling point
r; ~ normal boiling point of pure solvent.
le ~ latent heat of evaporation per gram of solvent.
.. !':i.Tb = Kb x m
Kb ~ ebullioscopic constant.

251
252 PROBLEMS ON PHYSICAL CHEMISTRY

,2
RT1
where Kf ~ cryoscopic constant and K1 = .
100011

where T; ~ freezing point of pure solvent


1 ~ latent heat of fusion per gram of solvent.
1
t-.T ~ depression of freezing point.
1
If there is association or dissociation, the quantities are multiplied by van't Hoff
factor i.

i = nabs , etc.
rrthea

. . . i-1
5 . Degree o f d 1ssoc1at1on, a = -- ,
v-1
where v is the number of particles into which solute dissociates.
. . f3 v(I - i)
6 • D egrees of associat10n, = ---,
v-1
where v is the number of molecules of solute associating.
7. In S.I. unit,

fl =RT n2 =RT W2 kg
V M 2 kg.mole-' x V m 3
o2
f..T =K W2 kg = RTb.f m
b.f b.f M 2 kg. mole-' x W1 kg Le.f
Le = latent heat of evaporation per kg
1; = latent heat of fusion per kg.

Short Questions
1. What is van't Hoff factor i?
Ans. : When solute molecules associate or dissociate in an ideally dilute solution, the
simple relations relating mole-fraction, molarity or molality of solute molecules with different
colligative properties of the solution will no longer be valid, since concentration changes.
So observed value deviates from calculated value. van 't Hoff factor i is the ratio of observed
colligative property to the theoretical value at the same concentration and temperature.
i can be defined as the ratio of observed osmotic pressure, fl obs• to its theoretical value,
fltheo• at the same concentration and same temperature .

. - nabs - nabs
1- - - - - -
Iltheo RTC
: . flobs = iRTC.
. . (-
po_p)
S1m!larly, -- = i Xz.
p0 obs
(f..Tb)obs = iKbm
(f..Tiobs = iKfm.
COLLIGATIVE PROPERTIES OF SOLUTION 253

2. When can the van't Hoff factor i be integral?


Ans. : The van't Hoff factor i can be integral when a solute is completely dissociated
to v particles. The colligative properties depend on number, and for a C molar solution
the real concentration becomes vC (and for m molar it is vm) so that
n = vRTC
tiT = vK m
1 1
= v.
3. "Colligative properties are intensive."-Comment.
Ans. : A property is extensive when sum of its values of constituting parts of a system
gives the whole. When parts do not add to give the whole, the property is intensive. This
is so about colligative properties. Let the osmotic pressure n of a 800 cc solution of glucose
of certain molarity m be A, then n of I litre solution is also A. That is why colligative
properties are intensive.
4. Is there any advantage in using molality instead of molarity as a concentration unit?
Ans. : Yes, there is an advantage. When the temparature of the liquid phase changes,
and yet we need to compare relative concentrations at two tempartures, then the temparature
independent concentration unit 'molality' may be used.
5. When can there be freezing point elevation?
Ans.: In an ideal solution the solute is insoluble in solid solvent, and tiT (the freezing
1
point depression)= T0 - T (T0 , freezing point for solvent and T , freezing point of solution)
1 1
is (+)ve. Thus, there is freezing point depression. In the case when the solute is soluble
in solid solvent and the solubility in the solid solvent is greater than that in liquid solvent,
then there can be freezing point elevation, i.e., tiT is (-)ve. It can be easily shown that
1
tiT = K (l - K), where K is the ratio of mot-fraction of solute in solid solvent to mol-
1 1
fraction of solute in liquid solvent. When K >I, there will be freezing point elevation.
6. "Lowering of vapour pressure of a liquid by a non-volatile solute is due to the
attraction of solvent molecule through solvation."-Justify or criticise.
Ans. : Lowering of vapour pressure is an entropy effect. A solvent vaporises when
kept in a container because. that leads to more chaos and randomness, thereby increasing
entropy. In a closed vessel the molecules in the vapour phase collide with the wall giving
a pressure P 0 -this is the ·vapour pressure. Addition of non-volatile solute introduces extra
randomness in liquid phase. As a result the tendency of solvent to escape to vapour decreases.
Consequently in a closed container now there will be lesser amount of solvent in vapour
phase and naturally some extra molecules of solvent are there in the liquid solvent. It is
comparable to arresting of solvent molecules, maybe through sQlvation. Hence, the above
statement.
7. Why is it necessary that the solid dissolved in the liquid solvent be non-volatile
(in case of colligative properties)?
Ans. : When a solute is dissolved in a solvent, the chemical potential of the solvent
in the liquid phase will decrease in comparison to that of vapour phase in equilibrium with
it if the solute is absent in the vapour phase. Thus the solute has to be non-volatile for
lowering of vapour pressure, and other related colligative properties. The chemical potential
in liquid phase can, of course, be less than that in the vapour phase if the concentration
of the volatile solute is less in the vapour phase in comparison to liquid phase. But it cannot
be said unequivocally that addition of solute leads to the decrease in chemical potential
of the solvent in the liquid phase.
254 PROBLEMS ON PHYSICAL CHEMISTRY

8. When will the van't Hoff factor i be fractional?


Ans. : The van't Hoff factor i expres-ses the deviation from ideality of a real solution
in their colligative properties. It can explain easily the deviation when there is association
or dissociation. When there is associatiun, complete or fractional, i must be a fraction

(because, say, for complete dimerisation i = ±


= 0.5). ,Also i is fractional when there is
partial dissociation.
9. Can we use the same Beckmann thermometer for boiling point elevation and freezing
point depression measurements?
Ans.: In Beckmann thermometer, the bulb volume error has been eliminated by keeping
a mercury reservoir at the top. Thus, the same Beckmann thermometer may be used for
both measurements. However, the thermometer needs setting before being used so that the
bulb volume is adjusted to the desired range. '
10. Show that for a solution of a solute in a non-polar solvent at the particular
concentration !lTb /Tb is independent of the nature of solvent.

Ans. : We know that !lTb = __


RT2
b x 2.!lTb depends on the number of moles of solute
L,
and nature of the solvent, since Tb (boiling point) and Le, the latent heat of evaporation
per mole, depen.d on the nature of solvent.

where R = 2 cal and ~ = 21 cal, by Trouton's rule.


Tb

!lT x2
Hence, __ b is independent of the nature of solvent and is equal to - -, where x
2
Tb 10.5
is mole fraction of the solute. It has been assumed that Trouton's rule is valid.
11. Which one is expected to be higher among K1 and Kb?

R(Tb')2
2
R(T; )
Ans.: K1 = and K =
lOOOlr b 10001,

' ' _le /


The ratio of squares of Tr and Tb is generally above 0·25, whereas is rately less
Lr r

K
than 5. So _J_ > 1, i.e., K > Kb.
Kb 1
12. Why is camphor more suitable than water as a solvent in determination of molecular
weights of organic substances by cryoscopic method?
Ans. : For determination of molecular weights of organic substances by cryoscopic
method depression in freezing point (!lT ) of the dilute solution has to be measured. Since
1
COLLIGATIVE PROPERTIES OF SOLUTION 255

!:.T = K m and m (molatity of the substance in the so.lvent) has to be very low, !J.Tf will
1 1
be reasonably high to be detected even by ordinary thermometer, if and only if K is high.
1
Camphor has appreciably high K (40) while that of water is only 1.86. So, camphor is
1
preferred to water as a solvent in determination of molecular weights of organic substances.
13. Why is effervescertce observed when a soda water bottle is opened?
Ans. : According to Henry's law, solubility of gas in water increases with pressure.
Soda water contains C0 2 at high pressure. When bottle is opened, the pressure is released .
and solubility of C0 2 also decreases. So the effervescence is observed.
Numerical Problems
1. At 25 °C, I 0.50 litres of N 2 at 760 mm of Hg are passed through an aqueous solution
of a non-volatile solute, whereby the solution loses 0.246 gm in weight. If the total pressure
above the solution is also 760 mm, what is the vapour pressure of the solution and mole
fraction of the solute? Given that the vapour pressure of pure water at this temperature
is 23.76 mm of Hg.
Solo. : The partial pressure of water in N 2 is the vapour pressure of the solution. Thus,

Now, I 0.5 litres of N 2 at 25 °C contains


Ix 10.5
n = mole = 0.42916 mole.
0. 08205 x 298.15

0.246
18
.. PHzO = 0 246 x 760 mm of Hg
-· +0.42916
18

0.013666
= -0-.0- 1-36-6-6-+-0-.-42-9- 1-6 x 760 mm of Hg = 23.45 mm of Hg.

By Raoult's law, the mole fraction of the solute in solution is


0
= P -
0
P = 23. 76 - 23. 45 = O.Ol3.
P 23. 76
2. At 25 °C, 10.5 litres of pure N 2 measured at 760 mm of Hg is passed through an
aqueous solution of a non-volatile solute whereby the solution loses 0.2455 gm in weight.
If the total pressure above the solution is also 760 mm, what is the vapour pressure of
the solution and mole fraction of the solute? Po at 25 °C = 23.756 mm of Hg.
Hints : See Problem I.
3. A 5% solution of cane sugar is isotonic with a 8.77% solution of urea. Calculate
the molecular weight of urea (the molecular weight of cane sugar is 342)
[Agra B.Sc. 1973; Meerut T.D.C.I. 198~]

Solo. : The cane suger solution contains 50 gm of cane sugar in I 000 gm of water.
Let the density of the solution be I gm/cc. Then

C = 50 . 7t = 50 RT.
342, 342
Similarly, for urea with molecular weight M 2 ,
256 PROBLEMS ON PHYSICAL CHEMISTRY

8 77
·n = · RT
M2

50 8 77
Thus, by the problem- RT = · RT
342 M2
or, M2 = 59.9868 ,,, 60.
4. 10 gm of a substance was dissolved in 250 ml of water and the osmotic pressure
was found to be 600 mm Hg at 15 °C. Find out the molecular weight of the substance.
Solo. : n = RTC
600
By the problem, n = atm.
760

c = 40, 10 gm in 250cc ]
M [ :. 40 gm in lOOOcc

where M = molecular weight.


600 40
.. = 0.08206 x 288. 15 x
760 M
. . M = I l 98.0432 ,,, 1198.
5. How much urea must be dissolved in 10-2 m 3 of water to yield a solution whose
osmotic pressure is 2.03 x 105 N.m-2 at 300 K?
Solo. : The mo!. wt. of urea = 60
Now n = 2.03 x 10s N.m-2 M 2 = 60 gm
v = lo-2m 3 T = 300 K
W2 =? R = 8.314 J.K-1.moI-I

W2 -I -I W2 kg
.. n =RT-- = 8.3141.K .mol x 300 K X 3
M2 V 0.060kg.mol- 1 x 10-2 m

2.03x10 5 N. m- 2 x 0.06 kg. mole- 1 x10- 2 m 3


or, W2 = 8.314J.K-1.mole-1 x300K = 0.04883 kg= 48.83 gm.

6. Consider a vertical tube with a cross-sectional area of 0.5 cm 2. The bottom of the
tube is closed with a semipermeable membrane and 0.12 gm of urea is placed in the tube.
The closed end of the tube is just immersed in water. What will be the height of water
level in the tube at equilibrium? The final density of the solution is l.01 gm/cc and the
temperature is 27 °C. What is the osmotic pressure of the solution in SJ. unit?
Solo. : Let the height of water raised be h cm. Since the area is 0.5 cm 2, the volume.
2 103
is 0.5h cm 3 . Since the weight of urea taken is 0.12 gm, C= O.l x moles. liet.
60x0.5h
:. n = RTC
2 103
= 0.08206 lit-atm. K- 1.mole- 1 x 300.15 K x O.l x mole.lie!
30h
98.5212
= h
atm.
COLLIGATIVE PROPERTIES OF SOLUTION 257

Again, n = hpg
= h cm x 1.01 gm.cm-3 x 980.665 cm.sec-2
= h x 990.47165 gm.cm- 1.sec-2
2
= hx990.47165 dyne.cm- = h x 9 _77519 x 10-4 atm.
I. 01325 x 10 6 dyne. cm-2 . atm- 1

98 5212
· atm = 9.77519 x lo-4 x h atm
h
98.5212
or, h2 = 9.77519x 10-4
.. h = 317.47 cm= 3.17Sm.

n = 98.5212atm x 1.01325 x 10s Pa = 31.44 kPa.


317.47cm atm
7. The osmotic presssure is measured between water and a solution containing l gm
glucose (M =
180) and 1 gm sucrose (M =
342) in 1000 gm water. The temperature is
maintained at 25 °C. What will be the osmotic pressure of the solution? If this pressure
is measured without knowing that the solute is a mixture, what will be the resulting molecular
weight? What kind of average is it?
1 1
Soln. : Total number of moles in l 000 gm of water =(-180
- +-- ) gm. Assuming the
342 •
density of water 1 gm/cc, the molar concentration of the solution is

C =(--+-1
180
1
- ) moles.litre- 1
342
T = 25 °C '= 298. 15 K
1 1
Il = RTC = 0.08206 lit-atm.K- 1.mol- 1 x 298.15 K x(--+--)mol.licl
180 342
= 0.20746 atm = 0.21 atm.
Again, Il = RTC

or, 0.21 atm =0.08206 lit.atm.K- 1.mol- 1 x 298.15 K x ~ mol.lir- 1


M1
[where M 2 = resulting mol. wt.]

0. 08206 lit. atm. K- 2 mo1- 1 x 298.15 K x 2 mol.lit- 1


·or, M2 =
0.2latm
or, M2 = 233.011 gm.
average weight ~W
The average means ---~~ = --'
mole ~n;

8. Using van't Hoff equation, calculate the osmotic pressure of 0.865(M) sucrose
solution at 20 °C and compare your result with the experimental value which is 26.64 atm.
Explain the discrepancy. [C.U.(H) 1971]
Soln. : We know that for non-electrolytic solute, Il = RTC
Prob. Phy. Chem.-17
258 PROBLEMS ON PHYSICAL CHEMISTRY

or, TI= 0.08206 litre.atm.K- 1.mol- 1 x 293. 15 K x 0.865 mol.lir- 1


= 20.808 atm.
Now, rcexpt = 26.64 atm and Titheo = 20.808 atm .

.
·. 1' = nexpt = 26.64atm
·--- = 1.28.
ntheo 20. 808 atm

i-1
.. a= - - = 1.28- l = 0.28.
n-1 2-1
Since n = 2, sucrose breaks into glucose and fructose with a = 0.28.
9. The normal boiling point of chloroform is 61.2 °C and its heat of vaporisation is
59.0 cal/gm. If 0.50 gm of an organic substance is dissolved in 50 gm of CHC1 3 , the boiling
point of the solution is 61 .42 °C, calculate the molecular weight of the solute.
[C.U.(H) 1970]
Solo. : Let the molecular weight of the solute be M 2 . Then the molal concentration
of the solution will be

m = 1000 x 0.5
0
2
RT
Now, IJ.Tb = __h- x m.
1000/e

Here, Ti = 334.35 K
le = 59 cal/gm, :. IJ.Tb = 0.22 K.
1 1 2 2
I. 987 cal. K- • mo1- x (334.35) K 1000 gm ( 0. 5)
.. IJ.Tb = 1 x x - mo 1
lOOOgm x 59cal.gm- 50gm M2

_ I. 987 cal. K- 1• mo1- 1 x (334. 35) 2 K 2 x 1000 gm


or, M2 x 0.5 gm
- IOOOgm x 59cal.gm- 1 x50 gm x 0.22K
= 171.12 gm.
10. A and B are liquids and form ideal solution. The total vapour pressure at 50 °C
of a solution containing I gm-mole of A and 2 gm-moles of B is 250 mm of Hg, and that
of a solution containing 2 gm-moles of A and 2 gm-moles of B is 300 mm of Hg at 50 °C.
Calculate the vapour pressures of the pure liquids. [C.U.(H) 1968]
Solo. : The total pressure P is the summation of the partial pressure of the components.
p A and P8 are partial pressure]
[ of A and B respectively

where xi and P? are the mole fraction and vapour pressure of pure i-th substance.

:. PA= xAP~ and PB= X3P~.


By the given problem-

-j P~ mm of Hg + %P~ mm of Hg = 250 mm of Hg
COLLIGATIVE PROPERTIES OF SOLUTION 259
-----------~--------------

1 1
and - P~ mm of Hg + - P~ mm of Hg = 300 mm of Hg.
2 2
From the above two equations we get-
P~ = 450 mm of Hg.
P~ = 150 mm of Hg.
11. 122 gm of benzoic acid which completely dimerises in benzene solution are dissolved
in 1000 gm of benzene at certain temperature. If the vapour pressure of pure benzene is
66.6 cm of Hg at that temperature, then what will be the vapour pressure of the solution?
[C.U.(H) 77]

po -P
Solo. : We know that - - - = __!!]__
po n2 +n1
where Po and P are the vapour pressures of pure solvent and solution respectively, and
n 1 and n 2 are the number of mole of solvent and solute respectively.
By the given problem,
Po = 66.6 cm of Hg
1000
= 12.8205 mol
78
122 Benzoic acid dimerises, ]
= 0.5 mol [ so its molcular wt.= 2 x 122 = 244
244
p =?
From the above equation, we get
p
I - -po =

p
or, - __!!]__
po = n2 +n1

or, P = (1--n2 )x po
n2 +n1

0.5mol )
or, P = (I- x 66.6 cm of Hg = 64.1 cm of Hg.
0. 5 mol + 12. 8205 mol
12. Calculate the osmotic pressure of a 5% solution of cane sugar at 25 °C in SI unit.
Solo. : We know that
n = RTC,
where T = absolute temperature
C = molar concentration.
By the given problem, concentration of solution is 5%,
i.e., 5 gm cane sugar is present in I 00 cc solvent.
4
. M) 0.005x 10 I
. . mo Iar concentration ( = mol m 3
0.342
[Mo!. wt. of cane sugar = 0.342 kg/mole)
260 PROBLEMS ON PHYSICAL CHEMISTRY

5 104
.. CT = 8.314 J.K- 1.moJ-I x 298.15 K x ( O. 00 x ) mol.m-3
0.342
= 362400.45
Pa = 362.4 kPa.
. 13. A I% solution of AgN0 3 is isotonic with the 2% solution of glucose. Calculate
the degree of dissociation of AgN0 3 .
20
Solo. : 2% glucose =
(M) glucose solution.
180
[ ·: molecular weight of glucose = 180)

Again, 1% AgN0 3 solution = _!_Q_(M)


169.8
AgN0 3 solution.
.
Now in case of glucose, since it is a non-electrolyte,
CT = RTCg, where Cg is concentration of glucose solution.
Again, in case of AgN0 3 solution, since it is an electrolyte,
n = iRTCAgN03 [i = van't Hoff factor]
According to the given problem,
iCAgN03 = Cg
20
-moe. 1 1·it -1
or, i = 180 = l.8866.
__!9_ mole. lit- 1
169.8
i- I
So, degree of dissociation (ex) = v-1
Here, v = 2.
1.8866-1
• • <X = = 0.8866.
2-1
. . degree of dissociation = 0.8866.
14. Water is used in car radiators. In winter season, ethylene glycol is added so that
water may not freeze. Assuming ethylene glycol to be non-volatile, calculate the minimum
amount of ethylene glycol that must be added to 6.0 kg of water to prevent it from freezing
at 272.7 K. K for water = 1.86. [Punjab B.Sc. 1975]
1
Solo. : By the problem, in winter season the freezing point of water is 272.7 K. The
freezing point of water in car radiator must be less than or equal to 272.7 K. With glycol,
(CHpH) 2, (0.062 kg.mol- 1) as solute the freezing point of the solution has to be 272.7 K
or lesser. Thus, freezing point depression is (273.15 - 272.7) = 0.45 Kor greater.
~ T1 ~ 0.4:K
.. 0.45 K ~ Kpm
I W2 kg
:::; 1.86 l:g.K.mo1 x
0. 062 kg. mol- 1 x 6 kg

.. Wz ~ 0.45Kx0.062kg.mol~ x6kg
1

I. 86 kg. K. mol-
or, W2 ~ 0.09 kg.
COLLIGATIVE PROPERTIES OF SOLUTION 261
Therefore, the minimum amount of ethylene glycol to be added to 6 kg water is 0.09
kg (or, 90 gm).
15. How many gm of methyl alcohol should be added to IO-litre tank of water to
prevent its freezing at 268 K? K for water is 1.86.
1
Hints : See Problem 14. [Ans. 886.022 gm]
16. A solution of Hg(CN) 2 containing 3 gm per litre has an osmotic pressure 231.9 mm
of Hg at 298 K. What is the apparent molecular weight and degree of dissociation of
Hg(CN) 2?
Solo. : By the problem,
231. 9
Il = 231.9 mm of Hg= --atm.
760
Weight of Hg(CN) 2, i.e., W 2 = 3 gm per lit,
3
= mol.lit- 1,
M2
where M2 = mol. wt. of Hg(CN) 2 .
23 9 3
1. = RTC = 0.08206 lit.atm.K- 1.mol- 1 x 298 K x - - mol.lic 1
7~ M2
0.08206 lit.atm.K- 1. mo1- 1 x298K x 3mol. liC 1
= 231. 9 = 240.42 gm.
--atm
760
So, the apparent mol. wt. is 240.42. It is reduced from the real value 252.6 because
of the dissociation to three ions, Hg~+ and 2CN-. Therefore, Il = iRTC.
231.9
----atm
.. i =
n = ________!_7~60"--------,,-----~
RTC 0.08206 lit.atm.K- 1 • mo1- 1 x 298 K x -~ mol. lit- 1
252.6
231. 9 x 252.6
.. i = ---------
760 x 0. 08206 x 298 x 3
= 1.0506.
So, a = i-1 = 0.0506 = 0.025
v-1 2
.. degree of dissociation = 0.025.
17. When 2 gm of a non-volatile hydrocarbon containing 94.4% carbon is dissolved
in 100 gm of benzene, the vapour pressure of benzene at 20 °C is lowered from 74.66 mm
to 74.01 mm. Calculate the molecular formula of the hydrocarbon. [C.U.(H) 1984]
Solo. : By the given problem, the hydrocarbon contains 94.4% of C and the rest, i.e.,
94 4
5.6% of H. Thus, the number of C atom may be · =7.86 and the number of H atom
12
5.6 6 .
may be - = 5. or, m the numbers of atom, C : H = 1.4 : I.
I
So, the approximate formula is (C 1.4H>x·
262 PROBLEMS ON PHYSICAL CHEMISTRY

Now, let M be the molecular weight of the hydrocarbon.


By the problem,
2
po -P M
po = -rToo
--+---
M 78
po
or, =l+ IOOXM
po _p 78 2
100 M po p
or, --x- = po -P
-I=
po _p
78 2
M = _P_ x 78x2
P0 - P LOO
74.01 mm of Hg 78x2
= ------- x
0.65mmofHg 100
or, M = 177.6
.. (12 x 1.4 + I) xx= 177.6
or, x = 10.
So the correct formula is (C: .4H 1 ):o = C 14H 10.
18. A mixture of 1 gm of napthalene (M = 128) and
10 gm of camphor freezes at
147 °C whereas pure camphor freezes at 177 .5 °C. Calculate the cryoscopic constant of
camphor. LC.U.(H) 1983]
Soln : We know that,
AT!= Krm
. lOOOx W,
A gam, m = -,
M 2 xW1

where W 1 and W2 are wt. of solvent and solute respectively.


M 2 = mol. wt. of solute.
By the given problem,
ti.Tl= 30.5 °C
W 1 =IO gm; W2 = 1 gm; M2 = 128gm
:. 30 .5 0 C = K x lOOOx 1
f 128x10
or, K
1
= 39.04 K.gm.mo1-I.
19. A solution containing 0.684 gm of cane sugar in 100 gm of water freezes at
-0.037 °C, while a solution containing 0.585 gm of NaCl in 100 gm of water freezes at
-0.342 °C. Calculate K for water, i and % dissociation of NaCl. [C.U.(H) 1982]
1
Soln. : We know that, for non-electrolyte,

ti.T = K x Wz [in S.I. unit]


1 1 M 2 xW1

ti.T1 Kx M2 kg.mo1- 1 x W1kg.


or,
Kf = 0. 684 x 10-3 kg.
COLLIGATIYE PROPERTIES OF SOLUTION 263
For water,

0. 037K x 0. 342 kg. mo1- 1 x 0.1 kg


K = ---------"------~
3
f 0.684xlo- kg
or, K = 1.85 kg.K.mo1-1.
1
For NaCl solution, which is an electrolyte,

/J.Tf = i.K1 x W2 - [in S.I. unit]


W1M2

0. 342 K x 0. 0585 kg. mo1- 1 x 0.1 kg


or,
.
1 = t:i.T/ x M2 x WI = I. 85 kg. K. mor- 1 x 0. 585 x 10- 3 kg
KI xW 2

or. i = 1.8486.
:. a = i- l = 0. 8486 = 0. 8486 .
v-l 2-1
or, a = 0.85, :. percentage dissociation = 85%.
20. An organic acid (molar mass = 60) associates in benzene to form a dimer when
1.65 gm of the acid is dissolved in 100 gm benzene, the boiling point of benzene is raised
by 0.36 °C. Calculate the van 't Hoff factor and the degree of association of the acid in
benzene. Kb = 2.6. LV.U. 2000]
Soln. : We know that, for electrolytic solution- .
t:i.Tb = iKb.1n
t:i.T,,
or, i =
Khm

or, i = - - - - 0.36K
- - - - - -1
2.6kg. K. mo1-1 xmmol.kg- ·

3
Here m = - - -1. 65 x 10-3 kg·=----1
0. I kg x 60 x 10- kg. mo1- ·
.. i = 0.5.
Now, degree of association, p= v(l - i)
v-1
A 2X0.5 l
or' f' -
- -)-- - '

. . the degree of association is I (100% association).


21. Benzoic acid dimerises when dissolved in benzene. The osmotic pressure of a
solution of 5 gm of benzoic acid in JOO ml of benzene is 5.73 atm at IO 0 C. Find van't
Hoff factor and degree of association. [B.U. 1994]
Soln. : We know that, fltheo = RTC.
5x10- 3 kg
In the given problem, C = W2 = -------"'-----
M2 xW1 0. l 22 kg.mo l - \ x 0. Ix 10-3 m 3

0. 08206 lit. atm. K- 1. mo1- 1 x 283.15 K x (5 x 10- 3 )kg


. . rrtheo = ~ ·- Q.523 atm .
0.122kg.mol-'xo.1nt , ·
264 PROBLEMS ON PHYSICAL CHEMISTRY

= 5. 73atm
=-
flabs - - = 0.60173.
rrtheo 9. 523 atm

.. degree of association ~ = v(I - i) = 2


(J -0. 6 ) = 0.8.
v-1 2-1
22. The freezing point of pure benzene is 5.44 °C and that for a solution of 2.092
gm benzaldehyde in 100 gm of benzene is 4.44 °C. Calculate the molar mass of benzaldehyde,
given K for benzene is 5.1 kg.K.mol- 1• [B.U. 1997]
1
Solo. : By the problem, ti.T = I K.
1
. 0.002092 kg 0.02092
Now, mola 1 concentrat10n (m) = 0.1 kgx M kg.mo!- 1
= M2 kg.mol- 1

2
But, ti.T = K m
1 1
0.02092
or, IK = 5.1 kg.K.mo1- 1 x
M 2 kg.mol- 1

5.l kg. K. mo1- 1 x 0. 02092


or, M 2 = JK

= 0.1066924 kg.mol- 1 = 106.69 gm.


23. Heavy water (Dp) boils at 102 °C and its Kb value is 10% higher than that of
water. How does its latent heat per mole compare with that of water? [B.U. 1991]

Solo. : Let Kb and K~ be the ebullioscopic constant for wata and heavy water
respectively. Then by the given problem, since Kb value for D 20 is 10% higher than that
of H 20, K~ = 1.1 Kh.
Again, let Tb and T~; Lv and L~ be the boiling temperatures and latent heats for H 20.
and D 2 0 respectively.

Now, Tb = 373.15 °K; T~ = 375.15 °K.


We know that,
RT 2
Kb = __b_ and K~
1000/e
,2
Kb Tb Le
or,
Kb
= -2-,
TbLe
2
le Tb Kb
or,
Le
= -Tbz XKh
-,

or,
·•Le
{.

le
= ( r
375.15
373.15
Kb
x l.IKh

or, = 0.91886.
Le
COLLIGATIVE PROPERTIES OF SOLUTION 265

24. x gm NaCl, y gm sucrose and z gm urea are mixed and dissolved in 0.5 dm 3 of
water. Suppose that no chemical reaction sets in, what should be the osmotic pressure of
the solution? [B.U. 2000]
Soln. : We know that-
n = RTC
R = 8.314 J.K-1.mol- 1
T =T °K
C = A mol.m- 3.
By the problem, the solution is made of x gm NaCl, i.e., (xx 10-3 )kg NaCl, y gm
sucrose, i.e., (y x 10-3) kg sucrose and z gm urea, i.e., (z x 10-3) kg urea. Hence, number
xx 10-3 kg
of moles NaCl per m3 = .
0.0585 kg.mo1- 1 x0.5x 10-3 m 3

. yx10-3 kg
That of sucrose is .
0.342 kg.mo1- 1 x0.5x10- 3 m 3

zx10- 3 kg
And that of urea = .
0.060 kg.mo1- 1 x0.5xl0- 3 m 3
.. total concentration to be calculated for which we assume that NaCl completely
dissociates.
2xxx10- 3 kg
Its number of mole = -~~~~~~~=-~~~

0.0585 kg. mo1- 1 x0.5x10- 3 m 3 ·

4x 2y 2z ) 3
So, C = ( 0. 0585 + 0. 342 + 0. 06 mol.m-- ·

.. n =CRT

= (~+~+~)mol.m- 3 x 8·3141.K- 1.mol- 1 x TK


0. 0585 0. 342 0. 06

= 8.314 T x ·(~+.2L.+~) J.m- 3 or N.m-2 or Pa


o. 0585 0. 342 0. 06
25. A mixture which contains 0.55 gm of camphor and 0.045 gm of organic solute
(containing 93.46% of C and 6.54% of H) freezes at 157 °C. Calcuate the molecular formula
of the organic compound (Given : freezing point of camphor= 178.4 °C and K for camphor
1
= 37.7 °C kg.mole- 1). [C.U.(H) 1985]
Soln. : By the problem, t:i.T = 21.4 K
1
W1 =
0.55 x 10-3kg, Kf = 37.7 kg.K.mo1-t
W2 = 0.045
x 10-3 kg.
We know that, t:i.T = K m
1 1
0.045 x I0- 3 Kg
or, t:i.T
1
= 37.7 kg.K.mol- 1 x
M 2 kg. mo1- 1 x 0. 55 x 10-3 kg

37. 7 kg. K. mo1- 1 x 0. 045 x 10-3 kg


or, M2 =
0.55x10-3 kgx 21.4K
266 PROBLEMS ON PHYSICAL CHEMISTRY

or, M 2 = 0.144 kg.mo! 1.


Again, from % composition data,
. % of
gm-atomic c 93.45
= - - = 7.7 8
12
6 54
and gm-atomic % of H · =
= 6.54.
I
:. atomic ratio, C : H = 7.78 : 6.54 = 1.19 : I.
Multiplying by 10, molecule is C 12 H 10 ; molar mass =
0.154 kg.mol- 1.
But the error in molar mass is very high. [Taking 1.19 as I. I, the molecular formula
is C 11 H 10 with molar mass 0.142 kg.mol- 1. The error% in molar mass is reasonable. So,
the molecular formula may be written as CnH 10 J
26. Calculate the freezing temperature of an aqueous solution having its boiling
temperature JOO. I °C. The molal ebullioscopic constant and the molal cryoscopic constant
are 0.513 K.kg.mo1- 1 and 1.86 K.kg.mol- 1 respectively. [V.U.(H) 1987]
Solo. : By the problem, t1Tb = 0.1 K.
K" = 0.513 K.kg.moi- 1.
We know that. t1T" = K".m

or, m = O.lK
-----,- = ( ~) mol.k -I
0.513 K.kg.mol- 1 0.513 g
Again, 11T
1
= Klm
= 1.86 K.kg.moi- 1 x O. I mol.kg- 1 = 0.36 K.
0.513
. . the freezing temperature is -0.36 °C
27. A 2.5 molal HCl solution freezes at -9.88 °C. Calculate the apparent percentage
of dissociation and explain your answer (Kl = 1.86).
Solo. : By the problem,
t;.Tf = 9.88 K
K
1
= 1.86 K.kg.mol- 1
m = 2.5 mol.kg- 1.
We know that,
11T1 = iKlm
. 9.88K
or, I = I I = 2 .12.
1.86 kg.K.mol- x2.5 mo!. kg-
. . . i- I 1.12
So, degree o f d 1ssociat1on, a = -- = 1.12.
v- i 2-1
So, percentage of dissociation = 112%.
The reason for this absurd result is that the equation 11T = iKlm is not valid for this
1
case. HCl is soluble in solid solvent. It is volatile and the solution is reasonably concentrated.
28. (i) Benzene freezes at 5.6 °C. Its value for K is 5.1 Find t1Hr
1
(ii) An aqueous 20% (by wt.) solution of sucrose has density 1.08 gm.cm-> at 25 °C.
Express the concentration of solute in molar, molal and mole fraction units. [B.U. 2000]
COLLIGATIVE PROPERTIES OF SOLUTION 267
Solo. : (i) By the problem,

RT12
K1 = - - = 5.1 kg.K.moJ- 1 [Le = latent heat per kg]
Le

8. 314 J. K- mol- x 2~8. 75 K


1 1 2
= . J.kg-l
or, L e = 5.1 kg. K. mol-
126668 78

.. latent heat of fusion per mole = 126668.78 J.kg- 1 x 0.078 kg.moJ- 1


[ m. wt. of benzene = 78)
.. f..H = 9880.16 J.mol- 1•
1
(ii) By the given problem,
p = 1.08 gm.cm- 3
M = 0.342 kg.mot-I
The solution is 20% by wt.
So, 20 gm of sucrose in 80 ml of water.
Total weight of solution = I 00 gm.
Density of solution = 1.08 gm.cm- 3 .

. 20 gm x lOOOcc 1
.. C (molar concentration) = = 0.6316 mol.lit- .
342gm x (10071.08)cc

20 gm x I 000 gm 1
m (molal concentration) = = 0.7309 mol.kg- .
342gm x80gm

20
x (mole fraction) 342
= 20 80 = 0.013.
----+---
342 18
29. (i) For a certain solvent, the boiling point is 127 °C and the latent heat of evaporation
1s i 20 cal/gm. Find its molal elevation constant Kb.
(ii) On addition of a solute the vapour pressure of a liquid reduced to 91I0th of its
original value. If 2 gm of the solute (molar mass =100) is added to JOO gm of the liquid
to achieve the reduction, find the molar mass of the solvent assuming ideality. [B.U. 2003]
Solo. : (i) By the given problem,
T~ = 400.15 K

le= 120 cal/gm= 120 x 4.184 x IOOOJ.kg- 1.

We know that,
RT°2
Kb = --"- [Le is the latent heat per kg]
L,
8.314J.K- 1 .mol- 1 x (400.15/K 2
= 120 x 4.184 x IOOOJ. kg 1

= 2.6514 K.kg.mol- 1
268 PROBLEMS ON PHYSICAL CHEMISTRY

(ii) By the problem,

P = .!l_po
10
po_p = 0.1
po
po_p
Again, po

2
or, 0.1 = 100
2 100
-+-
100 M1
or, M1 = 555.56 gm.
30. Two grams of benzoic acid dissolved in 25 gm of benzene (K = 4.90 K.kg. mo1- 1)
1
produces a freezing point depression of 1.62 K. Calculate the molar mass of benzoic acid.
[V.U. 2005)
Solo. : From the given problem,
W1 = 25 gm of benzene
= 2 gm of benzoic acid
W2
K = 4.90 K.kg.mo1- 1
1
6.TI = 1.62 K
We know that-
6.T1 = K1 m
-- KrM2.W1
W2-
-

or, M 2
6.Tf. W1

4. 9 K. kg. mo1- 1 x2x10- 3 kg


= = 0.241975 kg.mo1-I
1.62Kx25x10-3 kg
or, M 2 = 243 gm.
Molecular weight of benzoic acid = 144 gm.
Since it demerizes, M2 gets doubled.
31. 0.01 molal aqueous solution of K 3Fe(CN)6 freezes at 0.062 °C. Calculate the
percentage of dissociation of the solute. Given K for water = 1.86 K.kg.mol- 1.
1
[V.U. 2004)
Solo. : According to the given problem,
m = 0.01 molal
6.Tf = 0.062 K
K
1
= 1.86 kg.K.mol- 1.
We know that-
6.T1 = i.K1 m
COLLIGATIVE PROPERTIES OF SOLUTION 269

or,

0.062K
= I l = 3.33.
1.86 kg.K.mol- x0.01 mol.kg-
Now, K3Fe(CN)6 dissociates producing 4 ions.
K3Fe(N) 6 ~ 3K+ + Fe(CN) 6- 3
i-1
So, degree of dissociation (a) = v-1
=
2. 33 = 2. 33 = 0.7433.
4-1 3
.. percentage of dissociation is 74.33%.
32. A 0.5% aqueous solution of potassium chloride is found to freeze at -0.24 °C.
Calculate the van't Hoff factor and degree of dissociation of the solute at that concentration.
[Kf = 1.86 K.kg.moJ- 1] [V.U. 2003]
Solo. : According to the given problem-
the solution is 0.5%, i.e. 0.5 gm KC! in 100 cc HzO.
So, molal concentration(m)

= __
w-=-2 -
M xW
mole.kg- 1
2 1

0.5kg I
= ---------
1
100 kg x 0. 07457 kg. mol-
= 0.067 mole.kg- .

We know that

0.24K
= I
1.86 K.kg.mol- x0.067 mol.kg-
I = 1.925.
KCl dissociates to give 2 ions.
KC!~ K+ + c1-

. . . (a) = -
. . degree o f d1ssociat10n i-1 0.925 092
- = - - = • 5.
v-1 2-1
Thus, percentage of dissociation = 92.5%.
33. Raoult's law is obeyed by a mixture of benzene and toluene. A mixture of the two
containing 0.34 mole fraction of toluene boils at 88 °C under atmospheric pressure. The
vapour pressures of pure benzene and toluene at this temperature are 960 mm and 380 mm
respectively. Find out the composition of the vapour on its boiling. [V.U. 2002]

Solo. : Let x 1 and x 2 be the mole fractions of tolune and benzene respectively and P1°
and P~ be their corresponding vapour pressures in pure states.
270 PROBLEMS ON PHYSICAL CHEMISTRY

Given, x1 = 0.34
P1° = 380 mm of Hg

P~= 960 mm of Hg

.\'.2 = I -- Xi = 0.66
Let P = total vapour pressure of mixture
P 1 = vapour pressure of toluine in mixture.
1
.
x 10 /uene (111 vapour state) = -Pi = X1P{
x 1Pl +x 2 P~'
1
P
0.34x380
= -----
(O. 34 x 380) + (0. 66 x 960)
= 0.1693759 = 0.17.
Similarly, xbenzene (in vapour state) = (I - 0.17) = 0.83.
34. Given the density of water to be I gm/ml (at a given temperature), which of the
following two solutions will have lower vapour pressure?
(i) 1.0 molal aqueous solution of cane sugar.
(ii) A cane sugar solution in which the mole fraction of cane sugar is 0.1.
[V.U. 2001]
Soln. : 1 molal aqueous soln. of cane sugar means that 1 mole of cane sugar has been
added to 1000 gm of water. Thus, mole fraction x 2 of cane sugar is-
n2
Xz = - - -,
n1 +n2
where n 2 is the number of moles of solute and n 1 is the number of moles of solvent (H 20).
I
Thus, x2 = = 0.01768.
1000
1+--
18
Hence, x 1 =(I - 0.01768) = 0.98231 = 0.98.
:. vapour pressure (P) = P 0 x 0.98.
A cane sugar solution of mole fraction 0.1 has a mole fraction ( 1 - 0.1) =0.9 of water.
Thus, in this solution P = 0.9 P 0

Hence, the 2nd solution has lower vapour pressure.


35. The boiling point elevation constant for benzene and water are 3 and 0.5 K.kg.
mo1- 1. When 0.585 gm of NaCl is dissolved in 100 gm of water, it causes an elevation
of 0.1 °C in the boiling point. Calculate the molecular weight of NaCl. Will the elevation
in boiling point of an NaCl solution (of the same molality) in benzene be higher? Assume
that the solubility NaCl in benzene is I% by weight.
[M.Sc. Adminission Test 1994, UT Bombay]
Soln. : By the problem,
W2 = 0.585 gm = 0.585 x 10-3 kg.
W 1 = 100 gm= 0.1 kg.
Assuming complete dissociation of NaCl, i.e., a = I,
COLLIGATIVE PROPERTIES OF SOLUTION 271

we get
i-1
a=
v-1
i- I
or, 1= or, i = 2.
2-1

We know that, 11Tb = 1. Kb.m = 1'Kb' - -


W?- -
M2. W1
Here 11Tb = 0.1 K and Kb = 0.5 K.kg.mol- 1.
iKh.W2
= 11Th. W1

2x0.5K.kg.mo1- 1 x0.585x10- 3 kg
= 0.1 KxO.l kg
or, M2 = 0.0585 kg. moI- 1•
In benzene, the solubility of NaCl is 1% by weight, i.e., 1 kg of NaCl in 99 kg of
benzene.
We know that,
!1Tb = Kb.m
0.585x10- 3 kg
=3 K.kg.mo1- 1 x
0.0585 kg.mo1- 1 xO.lkg
= 0.3 K.
So, 11Tb will be more in benzene than in water.
36. Liquids A and B form an ideal solution. In a binary solution of A and B the mole
fraction of A is 0.33. Calculate the composition of the vapour in equilibrium with the solution.
Given : P~ = 75 mm of Hg and P~ = 22 mm of Hg.
[M.Sc. Admission Test 1993, IIT Bombay]
Soln. : By the problem
XA = 0.33, .'. x 8 = ( 1 - 0·33) = 0.67

P~ = 75 mm of Hg

P~ = 22 mm of Hg.
We know that, for ideal solution total vapour pressure,

P = PA + P 8 = XA P~ + x8 P~ = (0.33 X 75) + (0.67x 22) = 39.49.


Let, in the vapour phase, mole fraction of A be x~ and mole fraction of B be x 8

0.33 x 75
= 0.62.
39.49

.. x 8 = 0.38.
37. At 140 °C the vapour pressure of C 6 H 5Cl is 939.4 mm and that of C 6 H 5Br is
495.8 mm. They form an ideal solution that boils at atmospheric pressure. Find out the
composition of the solution. [M.Sc. Admission Test 1992, IIT Bombay]
272 PROBLEMS ON PHYSICAL CHEMISTRY

Solo. : By the problem,

P 1° = 939.4 mm

P~ = 495.8 mm.
Atmospheric pressure (P) = 760 mm.
We know that for ideal solution,

P = x I P1° + x 2 P~ = XI PJ° + (I - XI) P~


or, 760 = (x 1 x 939.4) + (I - x1) x 495.8
or, x 1 = 0.6
.. x2 = 0.4
.. xc6HsCI = 0.6 and xc6HsBr = 0.4.
38. What is the freezing point of a 10% (by weight) solution of methanol in water?
Molal freezing point depression constant for water K = 1.86 K.kg.moJ- 1
1
[M.Sc. Admission Test 1993, HT KGP]
Solo. : By the problem, the solution is 10% by weight, i.e., 10 kg CH30H in 90 kg
of H20.
lOkg 1
.. m = - - - - = - . . ,- - - = 3.4722 mol.kg- .
= -0.-032 kg. mo!- 1 x 90 kg
AT! = K1 m
= 1.86 K.kg.moJ- 1 x 3.47 mol.kg- 1
= 6.454 K
T °C = -6.454 °C.
39. What is the freezing point of a 0.01 molal solution of K3 [Fe(CN)6 ], which is 78%
dissociated in water (K1 = 1.85)? [M.Sc. Admission Test 1992, HT KGP]
Solo. : By the given problem-
a = 0.78
v = 4.
.. i = 3.34 [a=.£=_!_]
v-1
We know that AT1 = iK1 m
x 1.85 K.kg.mole- 1 x 0.01 mole.kg- 1 = 0.06.
= 3.34
:. freezing point will be - 0.06 °C.
40. A solution of KI is isotonic with a 0.01 M solution of I2 at 27 °C. When equal
volume of two solutions were mixed together, the osmotic pressure dropped by 18.5% of
that of the individual solutions. Calculate the percentage of conversion of I - to 13-. Assume
that the solutions behave ideally and the salts are completely dissociated.
[M.Sc. Admission Test 1990, HT KGP]
Solo. : Osmotic pressure (CT) for 0.01 molar solution of I2 at 27 °C is-
n12 = 0.01 x 0.08206 x 300.15
= 0.246 atm.
COLLIGATIVE PROPERTIES OF SOLUTION 273

By the problem,
solution of Kl is isotonic with O.OIM solution of 12 .
:. ITKI = 0.246 atm at 27 °C
Let the concentration of KI solution be C mole.lie 1
:. IT = ix RTC
KI dissociates into K+ and I-.
Assuming complete dissociation, i = 2 .

.. c = rr - 0.246atm
ix Rx T - 2x 0.08206 lit.atm.K- 1 . mo1- 1 x 300.15 K
= 0.00499 mole.lie 1 = 0.005 mole.lie 1•
Now since equal volume of two solutions are mixed, the concentration will be half.
5 1
i.e., = 0.00 = 0.0025 mol. lir
2
0.01 . I
C 12 = = 0.005 mol.lir- .
2
Now by the given problem. IT of resulting solution,

= 100-18.5 x () .246 atm


100 •
= 0.2005 atm.
The reaction is :
KI + 12 = KI 3
Let x moles of KI react. So conentration of KI and I2 will be-
CKI = (0.0025 - x) mol.lie 1 and C 1 ~ = (0.005 - x) mol.lie 1
rr due to KI = 2(0.0025 - x) RT atm.
IT due to I2 = (0.005 - x) RT atm.
IT due to KI 3 = 2 xRT atm.
(since, KI 3 ~K+ + I3- and assuming complete dissociation i ::: 2)
So, [2(0.0025 - x) + (0.005 - x) + 2xJ RT = 0.2005
or, x = 0.00186 .
.. C 1 - = 0.00186 mole.lie 1.
3
Before reaction, C ,- = 0.0025 mole.lit-I.
After reaction, C _ = 0.00186 mole.lit- 1.
1}
86
.. % of conversion= _Q.OOl x JOO= 74.3%.
0.0025
41. The vapour pressure of a solution containing 6.69 gm of Ca(N0 1 )~ in 100 gm
of water is 746.9 mm of Hg at 100 °C. Calculate a (M = 164). [c~U.(Hl 1974]
Solo. : P0 at I00 °C is 760. Hence, by the problem,
po -P
po = ix2
274 PROBLEMS ON PHYSICAL CHEMISTRY

6.69
760- 746. 9 164
or.
760
=ix = ix 100 6.69
--+--
18 164
or, i = 2.3647.
Thus, the degree of dissociation,

a. = 2. 364 7 - I = 1.3647 = 0. 68235 .


v-1 3-1
42. A solution containing 10 gm of a dibasic acid in 1000 gm of water freezes at
-0.15 °C. 12 ml of O.l(N) NaOH solution is required for the neutralisation of 10 ml of
this solution. Given K for water is 1.86 K.kg.moI- 1, calculate the van't Hoff factor for
1
the acid. [C.U.(H) 1994)
Soln. : By the problem, the strength of acid x is given by 12 ml x 0.1 (N) = I Ox,
or, x = 0. I 2(N).
Let the density of the solution be I.
Then, JO gm in 1000 cc = O. l 2(N)

or, __!_Q_
0.12
gm in 1000 cc = I (N) = 83.33 gm in 1000 cc.

Hence, the equivalent weight is 83.33. The acid is dibasic.


Thus, its molecular weight is 83.33 x 2 = 166.66.
Now, by the problem,
freezing point = -0.15 °C
.. !ff1= 0.15.

0.15M 2 W1 0.15x166.66x100
or, i = ---~~-=
K1 x lOOOx W 2 1.86 x JOOO x JO

or, i = 1.34.
43. A and B are two non-volatile solids. A is dissolved in a solvent to give a dilute
solution which is dissociated (degree of dissociation a.). In the same solvent with the same
molal concentration Bis dimerised (degree of association p ). The elevation of boiling point
in the A solution is twice than that of B solution. If a. be 0.8, what should be p?
[C.U.(H) 1993)
Soln. : Let A dissociate to two particles. Then
i = 1 + a. = l + 0.8 = 1.8.
.. by the problem,
2 x ti.Th= 1.8 K 1rm and ti.Th= i'Kh.m
I. 8
or, 2 = -.-,
l

or, i' = 0.9.


COLLIGATIVE PROPERTIES OF SOLUTION 275

v(l-i') 2(1-0.9)
We know that, ~ = = = 0.2.
v-1 2-1
' 2.6
[N.B. : If A dissocites to 3 particles, i = 2.6, i = 1.3.
2
:. ~
> 1, impossible.]
44. An aqueous solution of sucrose freezes at -0.200 °C. Calculate the normal boiling
point and the molality of an aqueous solution of NaCl having the same vapour pressure.
Given, K1 for HzO = 1.86 K.kg.mole- 1 and Kh = 0.52 kg.K.mole- 1.
Solo. : By the problem, 11.T_r = 0.200 K.
We know that, 11.Tf = Kf"m.
Hence, the molality of sucrose solution (m) is

m = 11.Tf
Kf

.. m = 0.200K -- 0 • 1075 mo Ie. kg-I


I. 86 K. kg. mol- 1
.. 11.Th = Kh.m
= 0.52 K.kg.mole- 1 x 0.1075 mole.kg- 1 = 0.0559 K.

:. Ti = 373.15 + 0.0559 = 373.2059 K.


Assuming NaCl to be completely ionised, i = 2.
Since the solution of NaCl has the same v. p. as that of sucrose, it will have the same
boiling point.
.. 11.Th = 0.0559 K .
. . 0.0559 =2 x Kh x m.
0 0559
.. m = · = 0.05375.
2 x 0.52
.. molality of NaCl soln. is 0.05375.
45. A certain mass of substance when dissolved in 100 gm of benzene lowers the freezing
point by 1.28 °C. The same mass of solute dissolved in I00 gm of water lowers the freezing
point of the latter by 1.40 °C. If the substance has a normal molecular weight in benzene
and is completely dissociated in water, into how many ions does it dissociate in water?
Given, K1 (water)= 1.86 K.kg.mol- 1; K (benzene)= 5.12 K.kg.mol- 1• [C.U. 1975]
1
Hints : !1.T1 (benzene) = K (benzene).m
1
11.Tf (HzO) = v. Kf (H 20).m
!1.T1 (benzene) = 1.28
11.Tf (H 20) = 1.40
:. 1.28 = 5.12 x m for benzene (i)
and 1.40 = 1.86 x m x v for H 20. (ii)
Dividing eqn. (ii) by eqn. (i) we get the value of v.

:. v = 5.12x 1.40 = 3.0l == 3.


1.86 x I. 28
276 PROBLEMS ON PHYSICAL CHEMISTRY

46. (a) Find out osmotic pressure of a 0.001 (M) aq. K 2 S0 4 solution at 27 °C.
(b) Boiling point of acetone is 56.5 °C and its latent heat of vaporisation is 6920 cal.
mole- 1. Hence, calculate the molal boiling point elevation constant of acetone.
Hints : See previous problem. [Ans. (a) 0.0738 atm., (b) 1.81 kg.K.mole- 1 J
47. A solution of HCl in water contains 18.25 gm HCl per 100 gm solution. and its
density is 1.10 gm.cm·- 3 . Calculate the molarity and mole fraction of HCI.
Hints : See previous problem. [Ans. X2 = 0.099, concentration = 5.5(m).]
48. Calculate the van 't Hoff factor and the apparent degree of dissociation of a 0.2
molal aqueous solution of NaN0 3 which freezes at -0.675 °C.
[Given : Kr= 1.86 K.kg.mole- 1] [C.U. 2006]
Hints: See previous problem. [Ans. i = 1.814, ex= 0.814]
49. The vapour pressure of A is 939.4 mm of Hg and that of B is 495.8 mm of Hg
at 140 °C. Assuming that they form an ideal solution, what will be the composition of a
mixture, which boils at 140 °C under I atm? What will be the composition of the vapour
at this temperature? [C.U. 2004]

Hints : 760 =P~xA + P~x 8


vap -
P~xA [xA = 0.596, x 8 = 0.404 ]
xA - 760 x~"P=0.737, x~ap=0.263

50. The total vapour pressure at 25 °C of a mixture of benzene and toluene in which
the two mole fractions are equal, is 62 mm Hg. The vapour pressure of pure benzene at
25 °C is 95 mm of Hg. Calculate the mole fraction of benzene in the vapour in equilibrium
with the liquid mixture (assume ideal behaviour of the mixture). [C.U. 2001]
Hints : See previous problem. [Ans. 0.77]
CHAPTER 8

CHEMICAL EQUILIBRIUM

Required Formulae
1. Let a standard reaction be
L,viAi = 0.
In this equation v/s are negative for reactants and positive for products. Taking A 1
and A 2 as two reactants and A 3 and A4 as two products, then
v 3A 3 + v4A 4 - v 1A I - v 2A 2 =0
i.e., the reaction is v 1A 1 + v 2A 2 = v 3A 3 + v4A 4 .

2. For this reaction if a 3 and a4 be activities of products and a 1 and a2 be activities


of reactants and the activities are the activity at any instant except equilibrium, then reaction
quotient Qa is written as
v1 v4
aA3. ail.+
-i1P1.
0
( i)
·aA2
111

At equilibrium Q0 is converted to Ka, the equilibrium constant.


pV4, p"3
K is defined as K
p p
= -p'1A.+ AJ
p'?.
, (ii)
Ai. A2

where the terms are equilibrium partial pressures. We may define reaction quotient QP.
We can define Kc also, where
cv3 cv4
K - _A3· A±_
(iii)
c- C 11 C""
A1. A2

Kp and Kc are related to K 0 .


Ka= KP.Ky,
where y is the activity coefficient.
Similarly, Ka = Kc-Ky

277
278 PROBLEMS ON PHYSICAL CHEMISTRY

3. Unitless Kp and Kc :

The standard state P0 is l atm.

Similarly, Kc =

The standard state C 0 is l mole.lic 1.


4. Using unitless Kp and Kc,
AG = AG 0
+ RT lnQP and AG =-RT lnKp. 0

5. Kp and Kp be equilibrium constants at temperatures T 1 and T2 , then-


! 2

::~ ) = ~ T~,1~.~I
0
In ( ( )

and d In Kp Ml~. =
dT RT 2
6. Distribution coefficient or partition coefficient K is one type of equilibrium constant.
If a solute undergoes distribution between two immiscible liquids retaining molecular weight,
say ~ater 11.nd an organic solvent, then at equilibrium

K = Cor,
caq

where C0 r is concentration of solute in gm-mole in organic solvent and Caq is the


corresponding concentration in aqueous layer.
Short Questions
1. Show that to a first approximation the equation of state of a gas that dimerizes
to a small extent is given by

PV = l- Kc
RT V
where Kc is the equilibrium constant for the formation of dimer,

2A ~ A2; Kc~ [A2}.


[A]
Ans. : The reaction is 2A ~ A2
Let a be the degree of reaction. Then, if we take l mole of A initially, at equilibrium
CA is (I-a) moles and A 2 formed is a;2 moles. If the total volume is V,

(a/2)
Kc= V
{(J-:-a)/v} 2 ·
CHEMICAL EiiQUILIBRIUM 279

Now, l>><X,
a.V
.. Kc = ---
2.1
Kc a
.. =
v 2

At equilibrium there are n moles, n being (I -~} If the system is an ideal gas,

PV = nRT = (I -~)RT.
PV a
RT
= 2
PV Kc
or, - - = I -
RT V
2. Comment on the statement : "All reactions are reversible".
Ans. : We know that all reactions are reversible and characterised by an equilibrium
constant K0 , so that for the reaction,
v 1A + v2B = v 3C + v4D
a;.3. a~
Ka = vi vi '
aA .as
where a 's are activities of reactants and products.
3. Predict the nature of changes on the equilibrium for the following reactions brought
about by the variation ot the operational parameters mentioned against each equation within
parenthesis :
Ans. : (i) 2S0 3(g) = 2SOz(g) + 0 2(g) (heat applied)
This is an endothermic reaction. Thus, application of heat will lead to increase in the
equilibrium constant and equilibrium composition of products. This can be ascertained from
the relation :

In Kz = t-.H . Tz - T, .
K1 R T 1T 2
Since tJ.H is positive, application of heat will lead to increase of temperature and
K 2 > K 1. Consequently, the activities, i.e., concentration of S02 and 0 2 will be higher at
higher temperature
(ii) PC1 5(g) = PCJ 3(g) + Clz(g) [pressure increased]
For this reaction, if a is the degree of dissociation, then

Ppc13· Pch
Kp = ~ =
PPc/5

where Ppc 13 , Pc 12 , etc. are partial pressures of products and reactants, and P is the total
pressure. Hence, 1f total pressure Pis increased, a will decrease as Kp is constant. In other
words, increase of pressure means decrease of volume, and such decrease of volume will
lead to a decrease in the yield of products.
280 PROBLEMS ON PHYSICAL CHEMISTRY

(iii) C(s) + H20(g) = CO(g) + H 2(g) lC(s) increased at const. temperature]


For this reaction,
2
Pco· Pfh a P
Kp = - = --,,,
PH20 I-a.-
where PCO' PH., and PH.,O are partial pressure of CO, H 2 and H 20(g) respectively, and
P and a are the fotal pressure and degree of the reaction. Thus, if increase of C(s) at constant
temperature is made at constant pressure also, then there will be no effect. However, if
P increases, then the yield of CO and H 2 will decrease.
(iv) Nigl + 2H2 (g) = 2NH3(g)
[Inert gas introduced at constant volume and constant pressure]
If we start with a moles of N2 and b moles of H 2 , then it can be shown that the KP
for the reaction,
2 2
K _ 4x (a+b-2x)
P - (a - x )( b - 3x ) 3 P 2 '
where 2x is the number of moles of NH 3 at equilibrium and P is the total pressure (x moles
of N 2 spent and 3x moles of H 2 spent). The total number of moles at equilibrium is
(a + b - 2x) so that PV = (a + b - 2:\') RT, where P is the total pressure, V is the total
volume and Tis the temperature (assuming gas to be ideal). Addition of inert gas at constant
volume will lead to proportionate increase of P (at const. 1) so that x, the yield, will not
be affected.
But, if inert gas is added at constant P, (a + b - 2x) increases and hence x, the yield
must decrease to keep Kp constant.
(v) C (s) + 0 2(g) = C0 2(g) [system cooled]
Cooling will lead to a decrease in temperature. The above reaction is exothermic. Hence,
decrease of temperature will lead to an increase fl, KP and the equilibrium yield.
(vi) N 2 (g) + 3H2 (g) = 2NH 3(g) [The catalyst is removed after equilibrium is attained.]
The catalyst is a foreign substance and has no role in equilibrium composition. Thus.
the removal of catalyst will not disturb the equilibrium composition if total pressure is
constant. However, if total pressure decreases, then in the above case, the equilibrium yield
of NH 3 will decrease.
4. Does the equilibrium constant of a reaction depend upon (i) the standard state chosen
for the reactants and products, and (ii) the stoichiometric representation of the reaction?
Justify your answer. [C.U.(H) 2000]
Ans. : Equilibrium constant of a reaction depends upon the standard state chosen
unless !iv for the reaction is zero. If the standard state is I gm-mole/lit, resulting K is
Kc and if the standard state is I atm, the resulting K is Kp and they are different unless
11v = 0.

since, K p = n ( ;:, )v; and Kp = Kc ( ~: RT) Av' where po is 1 atm pressure and C°

is 1 mol.1- 1•
A B C D
i1G 0 is an extensive property and if a reaction is written as -+-=-+-
2 2 2 2
and KP is K. then liG 0
= -RT lnK =L.
CHEMICAL EQUILIBRIUM 281

If the reaction is A + B =C + D,
the mass is doubled, !:!.G 0
= -2RT lnK = -RT lnK2 = 2L.
Hence, Kp = K 2 . Thus, the equilibrium constant should depend on how we write the
equation in stoichiometric representation.
5. Under what condition will equilibrium constant of a reaction not change with
temperature?
Ans. : The temperature dependence of Kp is given by

d In Kp = !:!.H~. Thus, if !:!.H = o: 0


i.e., the standard heat of reaction is zero and
dT RT-
constant, then K will not change with change of temperature.
6. The relation Kp = Kc(RTl"v is not compatible with unitless Kp and Kc
Ans. : This is obvious since both R and T have units. Unitless Kp for the reaction
V1A + V2B = V3C + V4D is really

c~~ )"(~~ r 4

Kp =

( r(
~~ ~~-r
2

..
( r
~~~z~ c~~:(r c~-r(~~RT
Co po r(~Qr(
Co COR!r
po
Kp = =
(~~!r(c~~~~r (~~ r( c:~Tr(~~ r( c:~~r
= K
C
( co
po
r
RT +1'4 - vi
'"2

= Kc
(
~0
0
RT rv
If C 0 is 1 mole/lit, P 0 =1 atm, R is lit-atm.deg- 1.mole- 1, then ( ~: RT }s dimensionless,

and this Pc , etc. in K p should be written as ;~


p )l>v
7. Show that Kp = K, ( Po

Ans.: Kp =

= K.., (_!___)!>"
po
282 PROBLEMS ON PHYSICAL CHEMISTRY

8. Obtain an expression for Kp in terms of x(x = moles of N 2 reacted per mole) and
the total pressure P for the reaction.
N 2 + 3H 2 = 2NH 3
Show that x -t 0 as P -t 0 and x -t I as P -t oo.

Ans. : The reaction is N 2 + 3H2 = 2NH3 . If we take I mole of N 2 and 3 moles of


H 2 , then let the equilibrium number of mole of NH 3 be 2x, i.e., the equilibrium number
of moles of N 2 = (I - x) and that of H 2 = 3(1 - x).
2
4x 2 x(4-2x)
:.Kp =(1-x) 4 x27P 2 .

2
4x 2 (4-2x)
.. p2 = 27(1-x) 4 .Kp .
2x(4-2x)
Hence, P =
3(1-x)2.~3Kp .

8x
In general, I >> x, hence, P = ..J3K; . Thus, x -t 0, as P = 0.
3

Similarly, when x -t I, the denominator -t 0 as it contains (I - x). Thus, as P-too,


x -j I.
9. N 2 and 0 2 combine at a given temperature to produce NO. At equilibrium the yield

of NO is x per cent by volume if x = ./ Kab - K( a+ b) , where K is equilibrium constant


4 .
of the reaction at the given temperature and a and b are the volume perentage of N 2 and
0 2 respectively in the initial pure mixture. What should be the initial composition of the
reacting mixture in order that maximum yield of NO is ensured?
Ans. : By the problem yield,

x = ./Kab - K(a+b)
4

(~:)b and ( dx)


db a
will be equal to zero at maximum value of x.

(~:)b = 2-,Ja
.J/<b K =0 (i)
- 4

{Ka
(~;)a = 2.Jb
K
and
4
= 0. (ii)

. . from (i) and (ii),

or, ~ = [f
or, a = b.
CHEMICAL EQUILIBRIUM 283

Hence the yield will be maximum if we take equilibrium amount of N 2 and 0 2 at


the beginning.
10. What would be the effect of adding solid CaC0 3 to the following system at
equilibrium at a definite temperature, CaC0 3(s) = CaO(s) + C0 2(g)? [C.U. 2006]

Ans. : For the reaction CaC0 3(s) + C0 2(g) the equilibrium constant, Kp = Pco .
Thus, addition of CaC0 3 at equilibrium at a definite temperature would not affect if pressufe
remains constant. However, if P changes, equilibrium will be affected.
11. Consider two equilibria,
A 2 = 2A (i)
AB= A+ B (ii)
and assume that the b..G 0 and, therefore, Kp are the same for both. Show that the equilibrium
value of advancement for the reaction (ii) s 2 is greater than the corresponding value of
the reaction (i), s 1. What is the physical reason for this result? [V.U. 2001]

Ans. : If vf and vi are number of moles of species i at the beginning and at equilibrium
respectively, then
ni = n1° + lvds.
vi is (+)ve for products and (-)ve for reactants.

Let n~2 and n~ 8 be I, and n~, n~ be zero initially. Then by the problem,
nA
2
= I - SI' nA = 2s 1 for reaction (i)
and nAB =l - s 2 , nA = s 2 , n8 = s 2 for reaction (ii).
Since, Kp is same for both reactions,
4s2 s2
.. --'- = __2_
1- s, 1- s2
or, 4 sf = s~ (since, I » S)
:. s 2 = 2s 1, and thus, s 2 >.s 1.
One molecule of A 2 gives two molecules of A while one molecule of AB gives one
molecule each in both A and B. Equilibrium constants of both reactions will be same if
fraction of molecules of A present at equilibrium for reaction (i) is less than the fraction
of molecules of A and B present at equilibrium for reaction (ii). Hence, s 2 is greater than
S1·
0

12. Since b..G = -In K (the terms having their usual significance), a plot of In K
RT
l
vs. T would give a straight line passing through origin. Justify or modify the statement.

[V.U. 2006]

Ans. : Since b..G 0 depends on temperature, plot of In K vs. ~ will not be a straight
T
line passing through origin.
284 PROBLEMS ON PHYSICAL CHEMISTRY

13. Predict the direction of the slope of ( ac)


a<; T.P
and thus, direction of reaction, if

(i)Qp > Kp (ii) Qp = Kp (iii) Qp < Kp, wheres= extent of reaction and Qp is pressure
quotient.

Ans.: (ac)
-
OS T.P
-
= 6.G = -
6.Go + RT lnQp

= RT!n( ; : J (since 6.G


0
= -RTlnKp).

(i) If Qp > Kp, slope is positive and reaction will go backward, i.e., reactants will
be formed at the expense of products to reach equilibrium.
(ii) If Qp = Kp, 6.G =0 and reaction is at equilibrium.
(iii) If Qp < Kp, slope is negative and reaction will go forward, i.e., product will be
formed.

14. "If 6.G 0 =0 for a reaction, the reaction is thermodynamically impossible."-Justify.


Ans. : Spontaneity of a reaction is determined by 6.G and not 6.G 0 and

6.G = 6.G 0 +RT In Qp =-RT In Kp +RT In Qp.

6.G 0 = 0 means Kp = I, i.e., equilibrium concentration of reactants is same as that


of products. If initially Qp < Kp so that 6.G < 0 reaction proceeds in forward direction
to attain equilibrium, where 6.G =0 at constant P and T.

15. When will the plot of In Kp vs. _!_ give straight line? How does the plot of In Kp
T

vs. _!_ look for exothermic and endothermic reactions? Explain.


T

dlnKp t:,.Ho
Ans. By van't Hoff equation, =
~~)
R

If !':,.H 0 is independent of temperature, then plot of In Kp vs. ~·will give a straight line.

For exothermic reaction, t:,.H 0 is -ve, thus,

lnKp /

For endothermic reaction, t:,.H 0 is +ve, thus, lnKp~~+---

T
CHEMICAL EQUILIBRIUM 285

For exothermic reaction as T decreases or increases, yield increases, thereby


T

increasing K p· For endothermic reaction as T decreases or _!_ increases, yield decreases,


T
thereby decreasing K p· The shift of equilibrium in both the above cases takes place to nullify
the applied change. If T decreases at equilibrium, it will! shift in such a way that more
heat is generated. Hence, Kp increases for exothermic reaction and the same decreases for
endothermic reaction.
Numerical Problems
l. When nitrogen and hydrogen are mixed in the proportion I : 3 at 50 atm and
650 °C, the equilibrium concentration of ammonia is 25% by wt. Calculate the equilibrium
constant. [C.U.(H) 1969, 1980]
Solo. : The reaction is
N 2 + 3H2 = 2NH3
Let a moles of N 2 and 3a moles of hydrogen be taken initially. Thus, the mass is
{(a x 28) + (3a x 2)} = 34a gm. This weight is constant and its 25% is 8.5a gm. Thus,

the moles of NH 3 is ~a = 0.5a. So at equilibrium,


17
N2 + 3H2
a(l-x) 3a(l-x) 2ax
Now, 2ax = 0.5a .
.. x = 0.25.
Total number of moles is a + 3a - ax - 3ax + 2ax = (4 - 2x)a = 3.5a.

.. 0.75a x
PN2 = 3.5a
50

2.25a x
PH
2
= 3.5a
50

PNH = 0.5a x 50.


3 3.5a
(a cancels out. Thus, in problems we may forget about a and take the ratio as per
equation.)
If the equation is :

_!_ N 2 +
2
2
H.,
2 ~
= NH 3

( -~~)
2

x(50) 2
3.5
3
-0.75
---x50x {2.25
---x50 }
3.5 3.5
= 1.4 x 10- 4.
286 PROBLEMS ON PHYSICAL CHEMISTRY

2. When nitrogen and hydrogen in the molar ratio I : 3 are allowed to react at 100 atm
and 200 °C, it was found that the conversion to ammonia is 25% by volume. Calculate the
equilibrium constant for the reaction, N2 + 3H2 = 2NH3 .
Solo. : Let I mole of nitrogen and 3 moles of hydrogen be taken and let 2x mole
of ammonia be formed at equilibrium.
N2 + 3H2 = 2NH 3
(I - x) 3(1 - x) 2x
Since equal volumes contain equal number of moles, the total volume at equilibrium.
is (4 - 2x-) and volume of NH 3 is 2x. As per problem,
2x
- - x 100 = 25
4-2x
or, x = 0.4.
:. I - x = 0.6 and 2x = 0.8
or, 3(1 - x) = 1.8.
Total number of mole = 0.6 + 1.8 + 0.8 = 3.2.
0.6
PN2 = - x 100
3.2
I. 8
PH2 = - x 100
3.2

= ~ x 100
3.2
(0. 8) 2 x (3. 2) 2
:. Kp = = 1.872 x 10- 4 •
0.6x1.8x (100) 2
2x
N.B. : If the conversion were 25% of original volume, then - x 100 = 25,
4
or, x = 0.5.
3. For the reaction 2A + 38 = C + 4D, the partial pressures of A, 8, C and D at
equilibrium are 101.325 kPa, 10.1325 kPa, 1.01325 kPa and 202.650 kPa respectively.
Calculate KP.
Solo. : The reaction is-
2A + 38 =C + 4D
Let PA' P El' Pc and PD be the partial pressures of A, 8, C and D respectively, then-

where Po is the standard pressure = I atm = 101.325 kPa.

!:._~g_25 ).( 202_.:_650 )


( IOl.325 101.325
4

= 160 _

( _!_2!:~~)2
101.325
.(LC!:I3_~2)3
101.325
CHEMICALEQ.~U_I_L_IB_R_l_U_M _ _ _ _ _ _ _ _ _ _ _ _ _ _ _ _ _ _~ 287
4. A mixture of S0 2 and 0 2 in the molar ratio 2 : 1 was kept over platinum catalyst
in a reaction vessel at 650 °C, the total pressure at equilibrium was found to be JO atm.
If 60% S0 2 be converted to S0 3 , calculate Kp for 2S0 2 + 0 2 = 2S0 3 .
[B.U. 1977; C.U. 1973]
Solo. : Let 2 moles of S0 2 and 1 mole of 0 2 be taken (with 2a and a the result will
be same). Since the conversion is 60%, x = 0.6.
2S0 2 + 0 2 ~ 2S0 3
2-2xl-x 2x
.. nso2 = 0.8 ; no2 = 0.4 ; nso3 = 1.2.
Total number of mole (n) = ns 02 + n 02 + ns 03 = 2.4.
.. p S02 = 0. 8 x I 0 = 130
2.4
0.4 IO
Po2 = 2.4
x 10 = -6
I. 2
Pso 3 = 2.4
x 10 = 5.
2
( Pso3 ) (5)2
.. Kp = ---2-- = = 1.35.
( Pso2) . Po2
c~r x-~
5. Deduce the expression for Kp for the dissociation equilibrium
2H 2S(g) = 2Hi(g)
+ S 2(g)
in terms of total equilibrium pressure P and the degree of dissociation a at equilibrium starting
from pure reactant (H 2S) only. Hence, calculate values of KP, when a =
0.0118 (I 065 °C)
and a = 0.026 ( 1132 °C), the total pressure being I atm in each case. [C.U.(H) 1982]
Solo. : Let us consider that, there are 2 moles of H,,S and the degree of dissociation
is a. Hence, ~
2H 2S = 2H 2 + S 2
2(1-a) 2a a
Total number of moles = (2 + a).
(2-2a) P
.. PH2s = 2+a
2a a
= - - P and P5 = --P.
2 +a 2 2+a

= (!+~.P Y·h-~c;·P)
.. Kp

e2-+ !·p
2
r
or, Kp
= (2-2a) 2 (2+a) .P.
288 PROBLEMS ON PHYSICAL CHEMISTRY·

When a = O.oI 18 and P = 1 atm,


3
4(0.0118) 3 x1 (0.0118) xl
= 2 =
4(1-0.0118) (2+0.0118) (1-0.0118)2x2.0J18
= 8.36 x 10-1•
Again, when a = 0.0260 and P = I atm,

4(0.026) 3 x I
= 4(1- 0. 026) 3 (2 + 0. 026)
= 9.14 x to-6.
6. The equilibrium constant between ethyl alcohol and acetic acid to form ethyl acetate
is 4.0 at 293 °K. What weight of ethyl acetate will be obtained from 50 gm of alcohol
and 50 gm of acetic acid at 293 °K in a closed vessel? [Delhi B.Sc. 1975]
Solo. : By the problem,
50
nalcohol = = 1.087 [m. wt. of C 2H 50H = 46]
46

nacetic acid = ~~ = 0.833 [m. wt. of CH 3COOH = 60]


Let x be the number of mole of ester formed.
. . CH3COOH + C 2H5 0H == CH COOC H
3 2 5
(0.833 - X) ( 1.087 - x) X

.. 4 = x
(1.087-x)(0.833-x)
Solving we get x = l .938
and x = 0.623.
1.938 mole cannot be formed from 0.833 mole.
:. x = 0.623 .
. . amount of ester produced = (0.623 x 88) = 54.82 gm.
[Molecular weight of CH 3COOC 2H5 = 88]
7. 5 gm of PC1 5 kept in a flask of 0.4 lit capacity was completely vaporised at 523 °K.
The molecular weight was found to be 158. Calculate the eqilibrium constant.
[Lucknow B.Sc. 1979]
Solo. : The reaction is PC1 5 = PC1 3 + Cl 2
a(l-a) aa aa
where a = initial concentration of PC1 5 and
a = degree of dissociation.
We know that in this case,
0
P-P M -M
a =- - = --=-o__
po M
since pressure is inversely proportional to molecular weight at constant volume.
CHEMICAL EQUILIBRIUM 289

:. a = 208 · 5 - lS
8
= 0.3196. (molecular weight of PC1 5 = 208.5).
158

Initial concentration of PCI 5 = a = 51208 · 5 = 0.05995 mol.1- 1.


0.4
At equilibrium, CPC 13 = C02 = 0.31% x 0.05995 = 0.01815 mol.1- 1
.. CPCts = 0.05994 - 0.01915 = 0.04079 mou- 1

.. Kc= CPCl3Cc12 = 0.01915x0.01915 = 9 x 10-3.


Cpc1 5 0. 04079

8. At I 00 °C and a pressure of lo- 4 atm, C0 2 is 30% dissociated into CO and 0 2 .


What will be the degree of dissociation if the pressure is raised to I atm? What will be
the degree of dissociation when a mixture of 50% C0 2 and 50% 0 2 is heated to I 00 °C
at l atm?
Solo. : Let l mole of C02 be taken. The reaction is :
I
C02 =CO + 02
2
I
I-a a -a
2
0.7 0.3 0.15
By the problem, a= 0.3. Hence the number of moles of C0 2 , CO and 0 2 at equilibrium
are 0.7, 0.3 and 0.15 respectively (n = 1.15). Thus,

_Ql_ p)(~ p)l/2


( l.15 I.IS
=
(!!2p)
~~--'---'-~--'-~

1.15
112
= 0. 3 x (0.15P)
112
0. 7 x (1.15)
By the problem, P = 10- 4 atm.
:. Kp = 1.5478 x 10-3.
Let the degree of dissociation be a at l atm.
I
Hence, C0 2 = CO + 202

a
1-a
2
Total = 2+a
2
12
2a- ) P. ( - a- P )
(-
'
2+a a+2 a.a112p112
.. Kp = =
2(1-a).P (2 + a) 112 (1-a) ·
a+2
Neglectinga in comparison to l,
2 K~ = a 3.
. :. a= 0.016858537 = 0.017.

Prob. Phv. Chem.-19


290 PROBLEMS ON PHYSICAL CHEMISTRY

Now, in the second case, there are 1 mole of C0 2 and 1 mole of 0 2 initially. Hence,
the equilibrium composition will be-
!
C0 2 = CO + -0 2.
2
a
(I- a) l+-
2
2
a{2 +a)"
So, Kp =
{4+a)112 {1-a) ·
Neglecting a in comparison to 1, 2 and 4,
ax2112 a
Kp = 4 112 = .Ji
:. a= .../2Kp = 2.18893 x 10-3 = 2.2 x 10-3.
N.B. : Note that there is cubic power of a, and this type of problems can be solved
by normal methods, only if we can neglect a in comparison to 1.
9. In the reaction COCl 2 = CO + CI 2 at 550 °C, when the initial pressures of CO
and Cl 2 are 250 mm of Hg and 280 mm of Hg respectively, the equilibrium pressure is
found to be 380 mm of Hg. Calculate the degree of dissociation of COC1 2 at 1 atm. What
will be the extent of dissociation when N2 at a pressure of 0.4 atm is present, and if the
total pressure is (i) 1 atm or (ii) 1.4 atm?
Solo. : The reaction is : COCl 2 = CO + Cl 2
x a-x b-x
where a = 250 mm of Hg
b = 280 mm of Hg.
Let x be the pressure of COC1 2 at equilibrium.
Then Pco = 250 - x and Pc 1 = 280 - x
2
But by the problem,
Pco + Pc12 + Pcoc12 = 530 - x = 380
:. x = 150 mm of Hg.
Hence, at equilibrium,
Pco = 100 mm= o.:J2 atm
Pc 12 = 130 mm= 0.171 atm
P coc1
2
= 150 mm = 0.197 atm.
Hence, Kp for COC1 2 ~ CO + Cl 2 is
1- a a a
Kp = Pco·Pc1 2 = 0.132x0.171
Pcoc1 2 0.197
= 0.1145786 = l.15xl0-I.

. a 2P
But again, Kp = - - = 0.115.
1-a 2
CHEMICAL EQUILIBRIUM 291

When P = 1, by the problem-


o.2 = 0.115 - O. l 15a2
0.115
or, a2 =
1.115

:. (J. = ~ = 0.32115 = 0.32.


Now, N2 is an inert gas for this reaction. When N2 is present at 0.4 atm, the total
pressure of reactants and products is 0.6 atm. Thus,

0.115 = af x0~6.
1- 0.1

:. at = ~ 15
15 = 0.4.
Thus, when N2 is present at a pressure of 0.4 atm, the total pressure being I atm,
the degree of dissociation will increase from 0.32 to 0.4. If the total pressure is 1.4 atm
and the pressure of N 2 is 0.4 atm, then the pressure exterted by reactants and product is
1 atm, and hence, a = 0.32.
10. For the dissociation of phosgene gas, the value of Kp at 100 °C is 6.7 x 10-9 ,
when the pressure is expressed in atmosphere. Find the fraction of phosgene dissociated
at this temperature when 1 mole of phosgene is placed in a 100 lit vessel containing N 2
at a partial pressure of 1 atm. [C.U.(H) 1978]
Solo. ~ N2 is an inert gas here and since volume is constant, it will have no effect
on the yield. The fraction of phosgene dissociated,

(J. = [P:~p.
But P >> Kp.

.. a =~p
{K;, but P = RT, since a
v = 1 mol,. (and neglecting a in comparison to a)

. . by the problem,

a = ~ K p. V = )6.7 x 10-9 x 100 = 1.48 x 10 _ 4 .


RT ~ Qo8206 x 373.15
11. I2(g) is 1% dissociated to 2 I at 873 K and 25% dissociated at 1073 K. Calculate
f'.J{o.

Solo. : The reactions is-


12 ~ 21..
Let 2x be the amount of I produced.
. . C12 at equilibrium = ( 1 - x) and C1 = 2x.

. . partial pressure of I, i.e., P1 = ~P 1-x


and P1 = - P
l+x 2 l+x
292 PROBLEMS ON PHYSICAL CHEMISTRY

Since P is I atm,
2
4x
Kp = -1-x
-2.

For I% dissociation, x = 0.0 I.


2
K - 4 x (O.Ol) = 4.0004 x J0-4.
2
P1 - 1-(0.01}
For 25% dissociation,

Kp = 4x(0.25): = 0267 .
2 1-(0.25)
We know that

0
In 0. 267 = Af/ x 200
4X J0-4 8.3141 873X 1073

or, Af/0 = 8.314x873x 1073 x In( 0.26:) = 253.25 kJ.mol-1.


200 4x 10
12. The vapour pressure of solid NH4 HS at 25 °C is 50 cm of Hg. Assuming complete
dissociation of the vapour into NH 3 and H2S, calculate the total pressure when NH4 HS
is allowed to dissociate at 25 °C in a vessel containing ammonia at a pressure of 32 cm
of Hg. [C.U. 1976]
Solo. : The reaction is NH 4HS(s) = NH3(g) + H 2S(g)

Since there are two gases NH3 and H 2S in equal amount, the mole fraction of each is ..!.. .
2
.. PNH3 = 50 x -2I = 25 cm of Hg
I
PHzS = 50 x -2 = 25 cm of Hg

.. Kp = PNH 3.PH2S = (25


- x25)
- ·
76 76

If excess NH3 is added, the partial pressure of each will change due to excess ammonia.
Let there be x cm of Hg of NH 3 dissociated from NH4 HS.
32+x
·· PNH
3
= 76
x
PH S
2
= 76
2
x(32+x) {25)
. . Kp = (76)-
2 = (76) 2
or, x2 + 32x - 625 = 0
2
-32+:}(32) +2500
or, x = = 13.68.
2
. . total pressure = x + x + 32 = 59.36 cm of Hg.
CHEMICAL EQUILIBRIUM 293

13. Three solids A,B,C break into two gases, each as


A = A' + H2 S, B = B' + H2 S, C = C' + H2S.
The total pressure over A is 6 cm, over B is 8 cm and over C is 24 cm of Hg, when
they are kept in 3 separate vessels at 25 °C. What will be the total pressure if they are
kept in the same vessel at 25 °C?
Soln. : The total pressure over A is 6 cm of Hg, when A is kept in a vessel at 25 °C.

Mole fraction of each A' and H2S is _!_


2
I
:. PA' = Z x 6 = 3 cm of Hg and PHzS = 3 cm of Hg.

Similarly, when B and C are kept separately the pressure over B and C are 8 cm of
Hg and 24 cm of Hg respectively.

PB' = -2I x 8 = 4 cm of Hg

PH s = 4 cm of Hg
2

PC' = ZI x 24 = 12 cm of Hg

PHzS = 12 cm of Hg

K = -
A
9

(76)2 '
K
B
=~·
(76)2 '
K _ 144
c - (76)2 .
Let, when they are mixed together, PA' = x cm of Hg, P8 . = y cm of Hg and PC' =
z cm of Hg .
.. PHzS = (x + y + z) cm of Hg.

K x(x+y+z) 9 (t")
.. A = {76) 2 = (76) 2

y(x+ y+z) 16
Ks= ( M y = (76) 2 (ii)

K = z(x+y+z) =~ (iii)
c (76) 2 (76)2 .
Adding equations (i), (ii) and (iii), we get

(x+ y+z)2 169


(76) 2 = {76)2
or, (x + y + z) 2 = (I ).)
,.,
2

or, x + y + z = 13 cm of Hg.
Now, total pressure over the mixture is
P =PA'+ Ps· +Pc·+ PH2s
= X + y + Z + (x + y + Z)
= 2(x + y + z) = 2 x 13 = 26 cm of Hg.
294 PROBLEMS ON, PHYSICAL CHEMISTRY

14. Two solids A and B break in the vapour phase completely to A' + NH 3 and B'
+ NH 3 respectively. The pressure over A is I 0 cm of Hg and over B is 24 cm of Hg at
I 00 °C, when they are allowed to break in separate vessels. Calculate the total pressure
over solids A and B, when they are kept together in a vessel at JOO 0 C.
Hints : See Problem 13. [Ans. 26 cm of Hg]
15. For the reaction Nz0 5 ~ Nz0 3 +Oz at a certain temperture and 7 atm pressure,
the yield of Oz at equilibrium is found to be 20% of Nz0 5 present. Calculate the equilibrium
constant. Nz0 5 was allowed to decompose in another vessel at 7 atm, but now COz at
a partial pressure of 3.5 atm was added. Find out the yield of Nz0 3.
Soln. : The reaction is :
NzOs ~ Nz03 + Oz
I - a

:. Kp = (J-.-:a)(l+a)
Now, according to the problem,

~ x 100 = 20
I-a
I
or, 120a = 20 or, a=
6

(iY· 1

(i +~)(I -i) = 0.2.

Now, COz is an inert gas, here let b moles of COz be added. Let the degree of dissociation
now change to x. Thus, at equilibrium there is-
( I - x) + x + x + ..b. = (I + x + b) moles .
b
---.P
l+x+b
b
or, 3.5 = -l+x+b
--·7

l+x
or,
b
= 1, or, (1 + x) = b.
2 2
Now, Kp = 0.2 = (1- x )(1x +Px + b) = (I - x )(1x+ Px +I+ x)
x 2P
or, Kp = 0.2 = ( )(
21-x l+x
)

or, 0.4(1 - xZ) = xZP


or, xZ(p + 0.4) = 0.4
or, x
10.4
= YP+OA.
CHEMICAL EQUILIBRIUM 295

By the problem, P = 7 atm.

:. x = ~ 07.4· 4 = 0.23.

16. For the reaction H2(g) + COi(g) = Hp(g) + CO(g), the equilibrium constant at
a particular temperature above 100 °C is found to be l.2x10-2. A mixture of 0.5 mole of
Hi and 1 mole of C0 2 was taken initially. Calculate the mole per cent of steam formed.
Now a metallic oxide MO was introduced, and it was found that 20 mole per cent steam
is formed. The metal oxide reacts as follows :
MO + H2(g) = M + H20(g) (i)
and MO + CO(g) = M + C0 2(g) (ii)
Calculate the equilibrium constants K 1 and K2 of the two reactions (i) and (ii).
Solo. : The reaction is :
H2(g) + C02(g) = Hp(g) + CO(g).
It is a reaction with !w = 0. Hence, the total number of moles at equilibrium is 1.5.
Let x moles of H20 be formed. Then at equilibrium,
nH 2 = 0,5 - x ; nH 20 = x
nco2 = 1- x ; nco = x
x2
.. K -
P - (0. 5 - x )(1 - x)
= 1.2 x 10-2 .

Solving, we get x2 = (0.5 - l.5x + x2) (1.2 x 10-2)


or, 0.988x 2 + 0.018x - 0.006 = 0.

_ -0.018+~(0.018) 2 +(4x0.988x0.006)
or, x
2 x 0. 988
= 0.07 (neglecting the negative root).

mole percent of steam formed = 0.07 x 100 = 4.67.


l. 5
Now, the reaction (i) is :
MO(s) + H 2(g) = M(s) + H20(g).
Here also /w = 0. Hence, the total number of moles = 1.5. But now the total mole
per cent of steam is 20, i.e., there are 0.3 moles of steam. Thus,
nH
2
= 0.5 - 0.3 = 0.2
nH20 = 0.3
Ki = PH20 = ~ = 1.5.
PH
2
0.2
The 2nd reaction (ii) is-
MO(s) + CO(g) = M(s) + C02(g)
Pco2
K2 =
Pco
296 PROBLEMS ON PHYSICAL CHEMISTRY

But PcoxPH20 =l.2x10-2


PH2 x Pco2

PH~o
and -~- = 1.5.
PH2

Thus, Pco
- - x 1.5 = 1.2 x 10-2 .
Pco2

Pco 2 l.5x 100


K2 = - - = = 125.
Pco 1.2

17. KP for the reaction H2 + C0 2 =Hp+ CO is ~


at 450 °C. 0.1 mole of H2
119
and 0.2 mole of C0 2 are introduced in a flask at 450 °C-the equilibrium pressure is I
atm. What will be the mole percentage of steam at equilibrium? A mixture of CoO and
Co (both solid) is then introduced so that additional equilibrium
CoO(s) + H 2(g) = Co(s) + H 20(g) (I)
CoO(s) + CO(g) = Co(s) + COi(g) (2)
are established. The equilibrium mixture now contains 30 mole percentage of steam.
Calculate K 1 and K2 .
Hints : See Problem 16. [Ans. 10%, 9, 119]
18. In the gaseous reaction 2A + B = A 2B, !lG 0 = -1200 cal at 227 °C. What total
pressure would be necessary to produce a 60% conversion of B into A 2B, when a mixture
of A and B in the mole ratio 2 : I be used. [C.U.(H) 1975]
Soln. : The reaction is :
2A + B = A 2B !lG0 = -1200 cal
At equilibrium, at T = 227 °C + 273.15 = 500.15 K
2A + B
(2- 2 x 0.6)a (l-0.6)a
0.8 a 0.4a
Total number of moles
= 0.8a + 0.4a + 0.6a
= l.8a
= 0.6a p = ..!_
1.8a 3
p = ip
PA = 0.8a
l.8a 9
p = 3._p
Ps = 0.4a
l.8a 9
where P = pressure required for 60% conversion of B into A 2B.

Kp = -PAzB
2-
PA-Pn
CHEMICAL EQUILIBRIUM 297

_!_p _!_x(~r
3 4
x~2
3
=
(4p
9
y9 x 2P
= p2

7.59375
= p2
0
Now, liG = -RT In KP
or, -1200 = -1.987 x 500.15 In Kp.

.. lnKp = _ _12_0_0_ _ = 1.2075


l. 987 x 500.15
Kp = 3.345

So, 7.59;75 = 3.345


p
p2 = 7.59375 = 2.2702
3.345
p = 1.5067 atm. = 1.5 atm.
19. What total pressure will be necessary for Problem 18, if in case, /iG 0 =+ 1200 cal.
[C.U.(H) 1979]
Soln. : The reaction is :
2A + B = A2B. liG 0 = 1200 cal at 500. LS K
7.59375
= p2
Now,
1200 = -1.987 x 500.15 In Kp .
. . In Kp = -1.20749
Kp = 0.2989 = 0.30.
.. p2 = 7. 59375 = 25.4056
0.30
.. P = 5.04 =5 atm.
20. At 25 °C, /iG for the reaction N 20 4 = 2N0 2 is + 1380 cal. What is the degree
0

of dissociation (a) at 25 °C and total pressure of 10 atm? Calculate a', if the total pressure
is 10 atm, but a partial pressure of C0 2 equal to 5 atm is present at equilibrium.
Soln. : The reaction is :
N204 = 2N02' liG 0 = 1380 cal at 25 °C.
At equilibrium, Np 4 = 2N02
I - a 2a
Total number of moles present at equilibrium
= I - a + 2a = I + a
I-a
.. PN204 = -I+a
-P
298 PROBLEMS ON PHYSICAL CHEMISTRY

= ~/'>
I +a.

=
c~aa pr
~---'--
4a 2 P
- ----- =
_1-_a P (I +a)(l-a) l-a 2 •
I+a
Now, fl.G 0 = -RT In Kp
or, 1380 = -1.987 x 298.15 In Kp .
. . In Kp = -2.33
.. Kp = 0.097.
2
4
S
o, l-a2
a P = 0.097
or, 4a2P = 0.097 - 0.097a 2
or, 40a 2 + 0.097a 2 = 0.097 [ ·: P = IO atm]
or, 40.097a 2 = 0.097.

097
a= O. = 0.0492.
40.097
Now, in another condition P co is 5 atm, but the total pressure is I 0 atm. So the pressure
exerted by N 20 4 and N0 2 is (10 atJi - 5 atm) = 5 atm. Let a' be the degree of dissociation
under this condition.
So now,
4a' 2 P
-- 2 = 0.097, where P = 5 atm.
l-a'
or, 20a' 2 = 0.097 - 0.097a' 2
or 20.097a' 2 = 0.097
a' = 0.0695.
21. Kp is 0.05 (pressure expressed in atm) at 20 °C for the reaction NH4 HS(s) ~
NH 3(g) + H2 S(g). 0.06 mole solid NH4 HS is introduced into 2.4 litre flask at 20 °C.
(i) Calculate the percentage of solid that will get decomposed into NH 3 and H2S
at equilibrium.
(ii) Calculate the number of moles of NH 3 that would have to be added to the flask
to reduce the decomposition of solid to I%. (Assume all the gases to be ideal)
[C.U.(H) 2003]
Soln. : For the dissociation-
(i) NH4HS(s) ~ NHig) + H 2S(g)
Kp = PN
83
x P 825 = 0.05
If the total pressure is P atm,
I
PNH3 = 2p atm
CHEMICAL EQUILIBRIUM 299

p p
·So, - x -
2 2
= 0.05
or, p2 = 0.05 x 4 = 0.2
P = 0.447 atm.

447
Hence, pressure due to NH 3 or H 2S is O. atm = 0.2235 atm. Under the given
2

condition, number of moles of NH 3 1s n


. PV
= -R-T = -0-.0-0.223x 2.4 O
- - -x-- - -_1- = .0222 .
8206 2 5
93
So, the number of moles of NH 4 HS that has got decomposed = 0.0222. Hence,
..
percentage of decompos1t1on = ---
0.0222
0.06
x JOO =37 111.
-10.

(ii) Now, by adding NH 3 from outside, the decomposition of the solid has to be reduced
to 1%.
Let the number of moles of H 2S be x.

So, xx 100 = l.
0.06
06
:. x = O. = 0.0006 moles.
100
nRT
Thus, PHzS = v
0.0006 x 0.08206 x 293.15
= = 0.006 atm.
2.4
Hence, Kp = PNH3 x PHzS

or, 0.05 = 0.006 x PNH3·

:. PNH3 = 0.0.05 8
006 = .333 atm.

So, the pressure of NH 3 added from outside is (8.333 - 0.006) atm = 8.327 atm. Hence,
the number of moles of NH3 added from outside
PV
= RT

= 8.327x2.4 =O.S 3 1.
0. 08206 x 293.15
22. The equilibrium constant Kp for the reaction H 2 (g) + S(g) = H 2S(g) is 20.2 at
945 °C and 9.21 at 1065 °C. Calculate Af-/ 0 assuming it to be temperature independent.
[C.U.(H) 2001)
Solo. : By the given problem,
Kp
1218·15
= 20.2 at 945 °C

.
Kp
1338·15
= 9.21 at 1065 °C.
300 PROBLEMS ON PHYSICAL CHEMISTRY

K
We know that, In _!:1_ =
KPJ

9.21)
or In ( - -
= !l.f/
0
[1338.15-1218.15]
' 20.2 8.3141.K- 1. mo1- 1 • l338. l5x 1218.15

or, !l.f/0
= 8.314xl338.l5xl218.15xln(9.21)
120 20.2
= -88.7 kJ,
=
23. For the reaction NH4HS(s) NH 3 (g) + H2S(g), KP is 0.06 (atm) 2 at 300 K. 0.08
mole of solid NH 4HS is introduced into a 2 lit flask at 300 K.
(a) Calculate the per cent of the solid that will have decomposed to NH 3 and H 2S
at equilibrium.
(b) Calculate the number of moles of NH 3 that would have to be added to the flask
to reduce the decomposition of solid to 2%.
(c) Having reached this state of equilibrium explain whether the addition of more
NH4 HS(s) would increase/decrease, if the NH 3 pressure is unchanged?
Hints : For parts (a) and (b) see Problem 21. [Ans. 24.9%, 0.246 moles]
For part (c) : No change. Addition of more solid does not affect its thermodynamic
activity.
24. Kp has the value 10-9 atm 3 and 10--0 atm 3 at 25 °C and 50 °C respectively for
the decomposition of CuS0 4 ,3H2 0 = CuS04 + 3H2 0(g).
(a) What is the minimum number of mole of H 2 0 vapour that must be introduced
into a 3-lit flask at 25 °C in order to completely convert0.001 mole ofCuS0 4 to the trihydrate.
(b) Calculate !l.f/0 for the reaction.
Solo. : For the given reaction,
Kp = P~ 20 = 10~ at 25 °C.
Thus, equilibrium pressure of H 20 = 10-3 atm.
Then the number of moles of H 2 0, n = PV
RT
o_._OO_l_x_3__ = t.22618
= __
0. 08206 x 298.15
x 10-4

= 1.23 x 10- 4 mole.


The total number of moles of H2 0 needed
= 3 x 0.001 mole + J.23 x 10- 4 mole
= 3.123 x 10- 3 .
By van't Hoff equation,
l o-6 w 0
25
In 10-9 =R x 298.l5x323.15
or, !l.f/ 0
= 221.334 kJ.
CHEMICAL EQUILIBRIUM 301

25. For the reaction at 25 °C, PCl 5(g) =PCl 3(g) + Clz(g), 6.H =22 kcal/mole. Calculate
Kp at 25 °C and 600 °C, assuming tJ.H to be independent of temperature. Given standard
free energy of formation 6.G; (in kcal/mole) at 25 °C.
Cl 2(g) = 0.0, PCJ 3(g) = - 68.42
PCl 5(g) = -77.6 [C.U.(H) 1987; V.U.(H) 1988]
Solo. : From the given data,
6.G 0 = -68.42 + 0 + 77.6 = 9.18 kcal.mole- 1.
Therefore,
9180cal. mole- 1 tJ.Go
In Kp =- 298.15Kx1. 987 cal. K- 1 . mol- 1
[·: lnKp = --]
RT
= -15.495657.
Thus, Kp = 1.86 x 10-7•
298·15
K 73 5
Again, In Ps i = tJ.H x ( 575 )
KP29s.1s /? 298.15 x 873.15

= 24.455074
=_ _2_2_000_x_5_7_5_ _
1. 987 x 873.15 x 298.15
K
In Ps 73 t 5 = 24.455074
or, I. 86 x 10-7
= 1.86 x 10-7 x 4.17545 x 1010
= 7766.34 = 7.766x HP.
26. For the reaction PCl 5 (g) ~ PCl 3 (g) + Cl 2 (g), tJ.H =92 kJ. mol- 1 at 25 °C. Calculate
Kp at 25 °C and 300 °C, assuming tJ.H to be independent of temperature. Given standard
free energy of formation 6.G; in kJ.mol- 1 at 25 °C. Clz(g) = 0.0, PCl 3(g) = -286.2,
PCl 5(g) = -324.7.
Hints : See Problem 25.
27. When N 2 and H 2 are mixed in I : 3 mole ratio, 15 gm of NH3 is found in a mixture
of 100 gm at equilibrium. The pressure is 44 atm and the temperature is 350 °C. Find an
approximate value of Kp. [B.U. 2003]
Solo. : Let the reaction be
N 2 + 3H2 2NH3 . =
Then proceed as Problem 1, where ex.= 0.25 but here ex.= 0.15.
28. For a reaction 6.G 0 = -4 kJ.mol- 1 at 301 K and w =-50 kJ.
0
• moi- 1. Estimate
its Kp at 500 K.
Solo. : 6.G 0 =-RT In Kp
.. Kp = 4.945.
Again, by the problem,
In~ =' -50000 x 199
4.945 8.314 500x301
Solving, K = 1.74 x 10-3 •
302 PROBLEMS ON PHYSICAL CHEMISTRY

29. At 1000 °K, the value of Kp for the reaction 2S0 2 + 0 2 = 2S0 3(g) is 3.5, the
pressure being expressed in atmospheres. Calculate the standard free energy change (L'iG 0 )
for the reaction at 1000 °K. Calculate Kc for the reaction from Kp at 1000 °K, and obtain
the value of L'iG 0 from Kc· Clearly explain why the two values of L'iG 0 are different; although
both represent standard free energy change for the same reaction at the same temperature.
[C.U.(H) 1986)
Solo. : We know that-
L'iG0 =-RT In Kp
= -1.987 cal.K- 1.mol- 1 x 1000 K In 3.5
= -2.49 kcal.
For any reaction,

Kp =Kc- ( ~o RT
0
)ilv

or, Kc =Kp. ( ~: RT rilv


Since L'iv = -I for the given reaction,

I mol.l- 1
= 3.5 x 0.08206 1.atm.K- 1.moJ- 1 x 1000 K x
latm
= 287.21.
Thus, L'iG~ =-RT In Kc= -1.987 cal.K- 1.mol- 1 x 1000 K x In 287.21
= -11.25 kcal.mol-1.
L'iG 0 calculated from Kp is the free energy change when each gas is at a particular
pressure ·of J. atm; and L'iG 0 calculated from Kc is the free energy change when each gas
is at a concentration I mole.lir- 1. Thus, the reference states in the two cases are different,
and they can be identical only when L'iv = 0 so that P = CRT becomes P = C for the
absence of RT terms as a whole.
30. Kp is 8 x 10-9 atm at 100 °C for the equilibrium COCl 2 (g) = CO(g) + Cl 2(g).
L'iS 0373 K = 30 cal/deg.
(a) Calculate degree of dissociation of phosgene at 100 °C and 2 atm pressure.
(b) Calculate L'ifl;
73 K for the reaction.
(c) At what temperature phosgene will be 0.1 % dissociated at 2 atm pressure?
[C.U. 1998)
Solo. : (a) The reaction is :
COCl 2 (g) = CO(g) + Cl2(g)
Let us have initially I mole COCl 2 (g), and the degree of dissoiation is a. Thus, the
number of moles at equilibrium are :
ncoc12 = (I - a); nco = a ; nc12 =a
Total number of moles = I + a.
1-a a a
Pcoc12 = --.P
l+a
; Pco = --.P
l+a
; Pc12 = --.P
l+a
CHEMICAL EQUILIBRIUM 303

a2
.. Kp = --2.P
I-a

or, a = rt[·: I >> a, so neglecting a in comparison to I]

= 6.3 x 10-5 .
or, a
(b) We know that
fl.G 0 = -RT In Kp
or, fl.G 0
= -1.987 cal.K- 1.mot- 1 x 373.15 K ln(8 x io-9)
= 13.82 kcal.mol- 1.

Af{;73K = !J.G + Tt.5°


0

= 13820 + 373.15 x 30
= 25.01 kcal.mot-I = 104.66 kJ.moI- 1.
(c) By the given problem, a= 0.1%, i.e., a= J0- 3.

a 2P
Thus, Kp = - - = a 2P = 2 x J0--6.
1-a 2
Now, we know that,

or, In 2xJ0---0 = 250JO T 2 -373.15


8x10-9 1.987 T2 x373.15
(where T2 = final temperature)
.. T2 = 446.19 °K.
31. The dissociation equilibrium constant (Kp) of Np 4(g) = 2N0 2(g) at 25 °C and
1 atm is 0.141. Show that dissociation can be made spontaneous for the production of N0 2(g)
at 1 atm from N 20 4(g) at JO atm at 25 °C. [B.U. 1988; V.U. 2006]
Solo. : We know that the condition of spontaneity for any reaction is that !!G for that
reaction should be negative. By the problem, the reaction is-
N204(g) ~ 2N0 2(g).
We know that-
2
!J.G = !!G 0 + RT In PNoz
PN204

or, fl.G = -1.987 x 298.15 In 0.141 + 1.987 x 298.15 In~


JO
or, !J.G = -203 .55 cal.mol-1.
So, the reaction is spontaneous.
304 PROBLEMS ON PHYSICAL CHEMISTRY

32. The equilibrium constant Kp for the dissociation 2H 2S(g) = 2Hz(g) + S2(g) is 0.0118
at 1065 °C. Find the value of Kp at 1200 °C if heat of dissociation L1H = 42400 cal in
this temperature range. [C.U. 1988, 1996)
Hints : Proceed as earlier.
33. At l 000 °A the pressure of the iodine gas is found to be 0.112 atm. due to partial
dissociation. Had there been no dissociation the pressure would have been 0.074 atm.
Calculate Kp for the reaction, [B.U. 1984)
12 <== 21.
Soln. : By the problem, the reaction is-
12 <== 21.
Let there be I mole of 12 initally and a be the degree of dissociation. ~en at equilibrium
the number of moles of 12 = ( 1 - a) and n 1 = 2a. Total number of moles = ( l + a). If
it is done at constant volume, then pressure ratio will be equal to the mole ratio, i.e.,

?final (1 +a)
-- =--
?initial I
By the problem,
?final = 0.112
?initial = 0.074.
0.112
.. - - = 1 +a
0.074
.. a= 0.51.
Now, we know that-

c~aa pr 4a 2 P
2
4x(0.51) x(0'.112)
= = --2 = = 0.157 atm.
1-a p I-a 1-(0.51) 2
l+a

34. Equilibrium constants Kp for the reaction..!..Nz(g) +~H (g) = NHig) are 2.62 x
2
2 2
10-2 at 350 °C and 1.27 x 10-2 at 400 °C, pressure being expressed in atmosphere. Hence,
calculate the enthalpy change, assuming it to be independent of temperature. Also calculate
the standard free energy change at 400 °C. If the reaction be written as
N 2 (g) + 3Hz(g)
= 2NH3 (g), will the corresponding thermodynamic quantities will be different from the values
you have calculated? [C.U. 1972)
Soln. : We know that-
(KP)2 L1f/o (T2 -Ti)
In-- - -- ----
(Kp )1 - R . T 1T 2

Here, (Kp) 1 = 2.62 x 10-2 ; T1 = (350 + 273.15)K = 623.l5K


(Kp)2 = 1.27 x 10-2 ; Tz = (400 + 273.15)K = 673.15K.

( Kp )0 !>.H 0 (Tz -Ti)


In--~ = -- ----
( Kp ) 1 R T 1T 2
CHEMICAL EQUILIBRIUM 305

Here, (Kp) 1 = 2.62 x 10-2 ; T 1 = (350 + 273.15) K = 623.15 K


(Kp) 2 = 1.27 x 10-2 ; T2 = (400 + 273.15)K = 673.15K.

11H° = (KP)2 x RxT1T2


(KP)1 T2 -T1

1 1
=In( l.27x 10-2) x 8.3141. K- . mol- x623.15 K x 673.15 K
2.62x10- 2 (673.15 - 623.15)K
= -50510.00933 J.mol- 1
= -50.51 kJ.mol- 1
Let the standard free energy change at 400 °C be 110°
/1G 0
= -RT ln(Kp) 2
= -8.314 x 673.15 x ln(l.27 x 10-2)
= 24435.47856 J. = 24.44 kJ.
Equilibrium constant depends on how we write the equation. If the equation is written
as Nz(g) + 3H2 (g) = 2NH3(g}, Kp will be the square of the previous Kp and /1G will be
0

twice the previous one and !1H 0 will also be twice.


35. Starting with equimolecular amount of CO and H20, calculate the molar percentage
of the water gas system (H 2 + C0 2 = H 20 +CO) at equilibrium at 600 °C.K at this temperature
is 0.16.
Solo. : H 20 + co= H 2 + C0 2
(a - ax) (a - ax) ax ax
Let (a - ax) moles of CO and H20 be available initially.
Then after reaction,
the moles of C0 2 = ax
and the moles of H 2 = ax.
-i:otal number of moles = 2a( I -· x) + 2ax = 2a.
PH2 Pco2
.. Kp = =
PH 20 . Pco 0.16

ax ax
2a 2a = I
.
la - ~2 (a - axJ 0.16
2a 2a

C~xr
l
or, = --
0.16
1-x
or, = 0.4
x
or, I - x = 0.4x
or, l.4x = l

or, x= = 0.7143.
1.4

Prob. Phy. Chem.-20


306 PROBLEMS ON PHYSICAL CHEMISTRY

:. molar percentage of reactants, i.e.,

each of H 20 and CO is = a(l-x) x 100 = 1-0.7143 x IOO


2a 2
= 14.285 = 14.3.
And molar percentage of products, i.e., each of H 2 and C0 2 is = a.c
2a
0.7143
= - - x 100 = 35.7.
2
36. At high temperature C0 2 dissociates according to the equation 2C0 2 (g) = 2CO(g)
+ 0 2(g). At one atmosphere pressure the fraction of C0 2 dissociated is 2 x I o-5 at 1000 K.
At 1400 K it is 1.27 x 10-2. Assuming that the enthalpy change (Af/0 ) of the reaction is
0
independent of temperature, calculate /J.G and /J.5° at 1000 K. [B.U. 1980]

Soln. : 2C0 2(g) = 2CO(g) + 0 2(g)


2(1--a) 2a a
Let a be the degree of dissociation.

P~o·Po2
:. Kp =
P~o2

( -~~)2. p2 __c:.__, p
= 2+a 2+a
2( 1-
( 2+a
a))
--'---~---"-~---,,.--~-

2·p2
[P is the total pressure = I atm]

a3
[a «I].
2
.. a3 =2 x Kp.
At 1000 Ka.= 2 x 10-5.
3
( 2 x 10-5 )
= =4 x 10-15.
2

1.0242 x 10--6.

Now by van't Hoff equation,

In (KP)1400 ~ Afl
0
(T2 -Ti)·
(Kp )1000 R T1T2

In I. 0242 x 10- 6
Af/
0
( 1400 -1000)
·· 4x 10- 15 = 8.314 x 1400x 1000

or, /J.H 0 = ln(~0242xl0-6)x8.314xl400xl000


4x10- 15 400

= 563382.343 joules = 563.4 kJ.


CHEMICAL EQUILIBRIUM 307

Again, AG 0 = -RT In KP
(AG 0 ) 1oooK = -8.314 J.K- 1.mot- 1 x 1000 K x ln(4 x 10-IS)
= 275629.7356 joules = 275.63 kJ
. . AG = AH0 - T.D.S 0
0

0 0
.. !).So= 1'1.H -1'1.G = (563.4-275.63) x lQ3 = 287 _77 J.K-1.
T 1000
37. For the gaseous reaction-
C2H2 + HzO =
C 2D 2 + HzO; AH is 530 cal. At 25 °C Kp is 0.82. Calculate how
much C 2D 2 will be formed if I mole of C 2H 2 and 2 moles of D 20 are put together at
a total pressure of I atmosphere and at I 00 °C. [C.U.(H) 1989; V.U. 2000)
Solo. : From van't Hoff equation,

In (Kp )2 = !:lH Tz -Ti


(Kp ) 1 R T 1T2
Here, (Kp) 1 = 0.82
T 1 = (273.15 + 25)K = 298.15 K
T2 = (273.15 + lOO)K = 373.15 K
AH = (530 x 4.184) joule.
Let (Kp) 2 be equilibrium constant at 100 °C.

(Kp )2 AH T 2 -T1
:. In
(Kp ),
= R T1T 2

(Kp )z 530 x 4.184 x (373.15 - 298.15)


or, In
(KP )1
= 8. 314 x 373.15 x 298.15

I (KP)z = 0.1798
:. n (Kp),

or, (Kp)z = (Kp)J x e0·1798


= 0.82 x e 0 · 1798 = 0.982.
Now, C 2H 2 + DzO = C 2D 2 + H20
(I - x) (2 - x) x x
Let x mole of C 2D 2 and x mole of H 20 be produced.
Then at equilibrium moles of C 2H2 = I - x
and moles of D 20 = 2 - x.
Total number of moles 3. =

.. Kp [Let P be the total pressure.]

=----
(1-x)(2-x)'
308 PROBLEMS ON PHYSICAL CHEMISTRY

.. x2 = K p(2 - 3x + x2)
or, x 2 - x 2.Kp + 3x Kp - 2Kp =0
or, (I -Kp) x 2 + 3x.Kp -- 2Kp =0
_ -3 K p ± ~9K~--;4(l~-K-;f2/<--;
or, x - ( )
2 1-Kp

= 0.66 [neglecting (-)ve value] (putting Kp = 0.982)


So, at JOO °C 0.66 moles C 2 D 2 will be produced.
38. Calculate the partial pressure of HCI gas at 27 °C above a sample of NH4Cl(s)
as a result of its decomposition. Given /1G 0 values of NH4Cl(s), NH 3(g) and HCl(g) are
-203 kJmoI- 1, -16.48 kJmoI- 1 and -95.3 k.J.mol- 1 respectively at 27 °C. [V.U. 2003]
Solo. : NH4 Cl(s) = NH 3 (g) + HCl(g)
By the problem,
'1G 0 reaction = [(-16.48 - 95.3) - (-203)1 kJ.mol- 1
= 91.22 kJ.mol- 1 = 91220 J.moi- 1.
t1G 0 = -RT In Kp
912201. mo1- 1
or, lnKp = - - - - - ~--~--~~---
RT 8. 314 J. K -1. mo Ie - 1 x300.15K
or, Kp = 1.3 x 10-16.
But, Kp = PNH3.PHCI
= p2HCI (since, PHCI = PNH ).
3
= {K;
= 1.14 x 10- 8 atm = 8.66 x 10-6 mm of Hg
= 1.16 x 10-3 Pa.
39. For the reaction H2 (g) + I2 (g) ;:-~ 2Hl(g), Kp = 50.0 at 448 °C"and 66.9 at 350 °C.
Calculate !1.H 0 for the reaction. [V.U. 2005]
Soln. : Proceed as earlier. [Ans. -11.1 kJ]
40. At 2155 °C and I atm pressure H20 (g) is 1.18% decomposed into H 2 (g) and
0 2(g) in accordance with the equation 2H2 0(g) = 2H 2(g) + 0 2(g).
Calculate Kp for the process. [C.U.(H) 1997]
Hints : Here a is 0.0118.
3
4a
Kp of the reaction is, KP = ------.,,.-
2
' (2+a)(2-2a) ·

Putting the value of a,


we get Kp = 8.36 x 10-7 atm.
41. The equilibrium constant of the reaction,
Az(g) + B 2(g) ~ 2AB(g) at 100 °C is 50. If a one-litre flask containing I mole of
A 2 is connected to a two-litre flask containing two moles of B 2 , how many moles of AB
will be formed at the said temperature? [B.U. 2001]
CHEMICAL EQUILIBRIUM 309

Hints : A 2(g) + B 2(g) == 2AB(g)


(I-a) (2--0.) 2a
Total volume = 3 litres.

..
(2;)2
Kc = I-a 2-a = 50
----
3 3
:. a = 0.934
:. 2a = (2 x 0.934) = 1.868.
42. Consider the gas phase reaction :
A+ 2B ~ 4X + Y
It was found that when 2.0 mol of A, I .0 mol of B and I .0 moles of Y were mixed
and allowed to come to equilibrium, the resulting mixture contained 0.80 moles of X at
25 °C. Calculate the equilibrium constant of the reaction at I .0 bar and 25 °C.
(1.1.T. Bombay 2001)
Soln. : A + 2B == 4X + Y
(2-a) ( 1-20.) 4a (I + a)
Let the degree of dissociation be a.
Since 0.80 moles of X is produced,
.. 4a = 0.8
or, a = 0.2.
Now, moles of A = (2 - a) = (2 - 0.2) = 1.8
moles of B = (I - 2a) = (I - 0.4) = 0.6
moles of X = 4 a = 0.8
moles of Y = l + X = 1.2.
Total number of moles = 4 + 2a = 4.4.

.. PA = -l. 8 xP PB = -4.4
0.6
xP
4.4
l. 8 1.2
Px = -4.4 xP Py = -4.4 x P.

( _9_:~ p) (-1.2
4

4.4
x
4.4
p)
( -1..:_~
4.4
p) x (_Q~ p)2
4.4

-392
= - - '(0.8)4xl.2xP2
----'----2---, - • x -2
10 .
(1.8)x(0.6) x(4.4t
43. For the reaction, PCl 5(g) = PCI 3(g) + Clz(g) :
(A) Calculate the equilibrium constant using the following data at 25 °C :
Substance t:i.H0 (kJ.mole- 1) S0 (J.K-l.mole-l)
PCI 3 (g) -287 312
310 PROBLEMS ON PHYSICAL CHEMISTRY

-375 365
0 223
(B) Using the result from above, calculate the equilibrium constant of
2PCI 5(g) = 2PCI 3(g) + 2Clz(g). [Joint M.Sc. Entrance 2003]

Solo. : (A) We know that /lG 0 = /lH 0 - TllS 0 •


By the problem,
llG 0 PCl = (-287000_- 298.15 x 312) J.moJe- 1 = -380022.8 J.mol- 1
3
llG 0 PCls = (- 375000 - 298.15 x 365) J.mole- 1
= - 483824.75 J.moJe-1
llG c 12
0
= (0-298.15 x 223) J.mole- 1 = -66487.45 J.moJe- 1.
fl.Go reaction = llG 0
PCl3 + llG° Clz - /lGo PCI
5
llGo reaction = 37314.5 J.mole-1.
Now, llG 0 =-RT In Kp
fl Go
= - -RT = - 15.0533.

Kp = 2.9
x 10-7.
(B) 2PCl 5 (g) = 2PCI 3(g) + 2Cl 2(g)
n2
1 rc1]. Pc12
'
2
K' p = - - ·. - - = (Kp)
Preis
= (2.9 x 10-7) 2 = 8.41 x 10-•4.
44. The equilibrium constant of the reaction C (graphite) + 2H2i(g) ;:::::= CH 4(g) is
3.036 x 108 at 35 °C. The standard enthalpy of the reaction is 749 kJ.mo1- in the temperature
range 25 °C - 35 °C. Calculate the standard free energy change for the reaction at 25 °C.
[l.I.T. Joint Admission Test 2002]

Hints: In (KP)2 = llH T2 -Ti


(Kp) 1 R T1T2

Here, (Kp)z = 3.036 x Ios


llH° = 749000 J.moi- 1
T2 = 35 °C = 308.15 K
T1 = 25 °C = 298.15 K.
From calculation, (Kp) 1 = 1.6741 x 104 .
Now, llG 0 zsoc = - RT In (Kp) 1
= -24108.04 J.mo1-l
= -241.1 kJ.mol- 1.
45. Write down the expression for the equilibrium constant of the reaction
N2(g) + 3H2(g) = 2NH 3(g), llG 0 = -33.4 kJ.mo1- 1. What is the value of Kp at 300 °K.
[l.I.T. M.Sc. Entrance Bombay 1998]
Solo. : Proceed as usual.
CHEMICAL EQUILIBRIUM 311

46. At 2000 K the standard free energy change in calories for the reaction
N 2 + 0 2 ~ 2NO is given by !lG 0 = 22000 - 2.5T. Calculate Kp at this temperature.
[C.U.(H) 2005]
Hints : !lG 0 = (22000 - 2.5 x 2000) cal = 17000 cal.
0
And !lG = -RT In Kp.
!lGo -t.G'
In Kp = - - - , :. Kp = e RT = 1.39 x 10- 2 •
RT
47. If we like to extract at a time 51 % of the iodine present in 100 ml of an aqueous
solution of the same, what volume of CCl 4 is needed? Given that at the experimental
temperature the distribution coefficient of 12 between CCl 4 and H 20 is 85. [C.U. 2006]
Soln. : Let x gm be the weight of 12 present in water. Let V cc be the volume of
CC1 4 required to extract 51 % of the 12 at a time.
So after extraction,
12 present in H 20 = (x - 0.5 lx)gm = 0.49x gm
and 12 present in CCl 4 = 0.5 lx gm.
0.49x x 1000
Thus, concentration of 12 in H 20 in gm-mole/litre = Caq =
lOOx M 12

0.5lxx 1000
and concentration of 12 in CCl 4 in gm.mole/litre = Corg =

c
Now,~= 85.
Caq

0.5lxx!OO
H ence, = 85.
Vx0.49x
:. V = 1.224 ml.
48. The solubility of a solute is three times as high in ether as in water. Compare
the amounts extracted from 100 ml of the aqueous solution by (i) 100 ml of ether in one
step (ii) successive extractions each with 50 ml of ether. [V.U. 1999]
Soln. : Let x gm be the weight of solute in 100 ml aqueous soh,ition. Let y gm be
extracted by 100 ml of ether.
(i) So, after extraction, amount of solute in water = (x - y) gm.

By the problem, ~
c = 3.
Caq

lOOOy .
Now, cor = -----gm.mol/htre
lOOx Mether

lOOO(x- y) .
caq = gm.mol/htre.
100 X Mether

_Y_ 3
x-y
= 3 or, y = -x
4
= 0.75x.
Thus, 75% is extracted.
312 PROBLEMS ON PHYSICAL CHEMISTRY

(ii) Let z gm be extracted by 50 ml of water.

Hence, z x JOO = 3.
50~(x-z)

:. z = 2 x or 60% is extracted and 40% remains unextracted. Second time 50 cc will


5
again extract 60% of the remaining unextracted solute, i.e., 24% will be extracted. So total
(60 + 24)% = 84% is extracted.
Thus, extractions in several steps are much more efficient than a single step.
49. Distribution coefficient of a solute between CCl 4 and water is 25. The solute is
extracted from 250 cc aqueous solution by successive extractions. Calculate the number
of extractions needed for 99% recover with (i) 25 cc (ii) 50 cc CC1 4 .
Hints : (i) 4 times, (ii) 3 times.
50. For a certain reaction, L'iG 0 =- 4 kl/mole and l'l.H 0 =-57 kJ/mole at 298 K. Calculate
Kp for the reaction at 500 K assuming L'iH0 remains constant over the given range of
temperature. [C.U. 2006]
Hints : K = 1.48 x 10-4 .
51. The value of Kp forthe reaction N2 + 3H2 =2NH3 at 400 °C is 1.6 x 10- 4 . Calculate
0
the value of the standard free energy change L'iG with respect to Kp and Kc and interpret
0
the value of L'iG in two cases, if these are different. • [V.U. 2007]

co )-2
Hints : Kp = Kc ( po RT , Kc = 0.488

8.G 0
= -RT In Kp = 48.916 kJ
fl.G 0 = -RT In Kc = 4.015 kJ.
L'iG 0 are different in two cases as the standard states are different.
52. Calculate the percentage dissociation of N 20 4 for the reaction
N 2 0 4 (g) ~ 2N0 2 (g), if the equilibrium constant is 640 mm of Hg at 500 °C and the
gas mixture is at 160 mm of Hg. At what pressure will the dissociation be 50%?
Hints : Np 4 (g) ~ 2N0 2 (g)
I-a 2a
where a = degree of dissociation
4a 2 P 4a 2 P
Kp = (I +a)(l-a) = l-a 2 •
Putting the corresponding values of Kp and P. we get a= 0.707, or 71% N 2 0 4 will
dissoicate. When a = 0.5, P = 480 mm of Hg.
53. At what temperature Kp and Kc of the following reaction are same?
A(g) = B(g) + C(g)

co)~v = Kc (RT cpo


0

Soln. : Kp = Kc (
RT po
)
[!iv = l]

When RT~: =I, Kp =Kc,


CHEMICAL EQUILIBRIUM 313

P0 l atm
T = -0 - = = 12.19 K.
C xR l gm. mol.l- 1 x0.082061.atm.K- 1 . mol- 1
At 12.19 K, K p = Kc for the given reaction.
54. An equilibrium mixture of : Xe F 2(g) + OF2(g) = Xe OF 2(g) + F2(g) was found
to contain 0.3 mole of Xe F 2, 0.3 mole of OF2, 0.1 mole of Xe OF2 and ·0.2 mole of F 2
in one-litre container. How many moles of OF2 must be added to increase [Xe OF2 ] to
0.3 mole? ·

• K _ 0. I x 0. 2 _ ~
I
Son.. - - .
0.3x0.3 9
Let x moles of OF 2 be added to increase [XeOF 2] to 0.3 mole.
XeF2 + OF2 ~ XeOF2 + F2
0.3 - 0.2 0.3 + x - 0.2 0.1 + 0.2 0.2 + 0.2
[0.3][0.4) 2
[O.l+x)[O.l]
=9
9 x 0.12 = 2(0.01 + O.lx] = 0.02 + 0.2t. Thus, x = 5.3.
55. 98% of 2 gm NH 4CI dissociated while heating in a one-litre flask at 300 °C. If
to this flask 2.0 gm. of dry NH 3 was added, what would be the percentage of dissociation?
Hints : NH4 Cl(s) ~ NH/g) + HCl(g)
0.037 - 0.98 x 0.037 0.98 x 0.037 0.98 x 0.037
0.037 - a. x 0.0377 0.118 + a. x 0.037 0.037a.
(0.98 x 0.037) 2 = (0.118 + 0.037a.) 0.037a.
Thus, new percentage of dissociation = 27.3%.
56. Kp is I x 10-5 for the equilibrium.
C0 2 + H2 = CO + Hp(g)
At 298 °K and !iS 0 =-IO cal/deg, I mole of CO, 2 moles of H2 and 4 moles of C0 2
. are introduced in a 5-litre flask at 298 °K. Calculate (a) !iG and !iH (b) equilibrium pressure
0 0

(b) moles of each species present at equilibrium.


Hints : !iG 0 = -RT In Kp, !iH0 = !iG 0 + T!:lS 0
C0 2 + H 2 =CO+ Hp
4-x 2-x I+x x Kp = Kx Since !iv = 0

10_5 = (I +x)x , x = 7.99 x lo-5


(2-x)(4-x)
At equilibrium, C0 2 + H2 = CO + Hp
4 moles 2 moles I mole 7.99 x 10-5 moles
P
. .b.
E qui11 num = nRT
v -
_ 7x0.08206x298 _
5
- 3424
. atm.
57. At 929 K the total gas pressure is 0.98 atm for the reaction.
2 FeS0 4 (s) = Fep 3 (s) + S0 2 (g) + S0 3(g)
(a) Calculate Kp.
314 PROBLEMS ON PHYSICAL CHEMISTRY

(b) What will be the equilibrium total pressure if excess FeS04 is placed in a
flask at 929 K containing S0 2 at 0.4 atm
Hints : (a) Kp = Pso 2 .Pso3
98
Pso = Pso = O. atm = 0.49 atm.
2 3 2
Thus, Kp = 0.49 2 = 0.2401 atm 2 .
(b) (0.4 + Ps )Ps = 0.2401
03 03
Pso = 0.329 atm
3
Pso = 0.729 atm
2
Peq = 1.058 atm.

58. !lH0 is 30 kcal for the reaction AgCI =Ag + .!.c1 2 at 25 °C. Standard free energy
2
for formation of AgCI is -26 kcal at 25 °C.

(a) What is !l.G;98 for the above reaction?


(b) Calculate dissociation pressure of Cl 2 .

d(Tln Kp)
(c) What is the value of ?
dT

Hints : !l.G;98 = -!lc;.AgCI = 26 kcal

!l.G 0 =-RT In Kp. Kp = ~Pc 12 , Pc = 7.58 x 10-39 atm


12

d(T In Kp) I dT = _...!_ d!l.Go =


R dT
!l.Ho-!lco
D.S 0 = = 13.42 cal/deg
T

!l.So = 6.75.
R
59. Calculate Kp for the reaction 2NOCI = 2NO + C1 2, if at equilibrium the total pressure
is I atm and partial pressure of NOC! = 0.7 atm at 200 °C. Calculate !lH 0 , if Kp increases
by 1.5% per degree around 200 °C. Also calculate the pressure at which degree of dissociation
'of NOCI will be 0.4 assuming Kp = 0.1 atm at 200 °C.
Hints : (a) PNO + Pc12 = l - 0.7 = 0.3
2
PNO = 0·3 x ) = 0.2, Pc 12 = 0.1

Kp
-- P~o·Pc12
P~oCI
= 8.16 x 10-3 .
CHEMICAL EQUILIBRIUM 315

(b) dlnKp
dT

RT 2 (!!!_f_)
:. !J.Ho = Kp = 8.314 J.K-1 x 473.152 K2 x 0.015 K- 1 = 27.92 kJ,
dT
When degree of dissociation of NOC! is 0.4, then
a.3P
:. Kp = ( a.) = 0.1, P = 1.35 atm.
2( I - a.) 2 I + --
2
60. Np 4 is 19% dissociated at a total pressure of 1 bar and at 298 K. (a) Will the
addition of N2 at constant volume affect the equilibrium? (b) If N 2 is added at a constant
total pressure of 1 bar, what will be the degree of dissociation when partial pressure of
N 2 is 0.6 bar?

Hints : (a) No, since, Kp = n~02 [~


nN204 £..,ini
]·here !J.v is +1. Thus, addition of N2 at

constant volume ine<eases total numbe< of moles [ ~ n;} but at tOe same time P also
.increases so that -RT remams
.
. constant. (PV
v = Di R n.
I

(J. 2 p
(b) Kp = --, , here
l -a.- P = 0.4 bar, a. = 0.292.
61. A reaction vessel of one litre containing 0.23 mol of N 2 and 0.34 of PCl 3 is heated
to 523 K. Calculate Kp of the reaction PCl 5 (g) =PCl 3(g) + Cl 2(g), if the equilibrium total
pressure is 29 bar.
Hints : PCl 5(g) = PCl 3(g) + Cl 2(g)
0.34 - x x x
Total number of moles = n = 0.34 + x + 0.23 = 0.57 + x
nRT
29 = - - , :. x
v
= 0.097 (here R = 0.08206 x 1.01325 bar.l.K- 1.mol-t)
PN
2
= 10.00 bar, PPCI = 10.57 bar, Pct = PPCI = 4.22 bar.
5 2 3
Pc12 x PPC13
Kp = = 1.68.
PPCl5

62. Calculate Kp and !J.G 0 for the following reaction at 20 °C,


CuS04 , 4NHicrystal) = CuS04 , 2NH3 (crystal) + 2NH 3(g).
The equilibrium pressure of NH 3 is 8.26 kPa.
Hints : Kp = 6.65 x 1o-3 , !J.G 0 = 12.22 kJ.mo1- 1.
(1.01325 x 105 Pa = I atm)
CHAPTER 9

CHEMICAL KINETICS

Required Formulae
1. For an elementary reaction,
v 1A + v2B = v3C + v4D,

the rate equation is _ __!__ dCA = _ _!_ dCn = J_ dCc = J_ dCD = k C"IC"2.
V1 dt Vz dt V3 dt V4 dt A B

The order is v1 + v2.


2. Dimension of rate constant = (conc.) 1-nr-I.
Unit of k = (mole. 1- 1)1-ns-I,
n is the order of the reaction.
3. Rate equations of reactions of simple order.
I a
1st order : k = -ln--
t a-x
a = initial concentration of reactants
x = amount spent at time t
0.693
l112= -k-.

I x
2nd order : k = ( )
ta a-x
1
t1 = - , when a = b.
12 ka

n-th order : k - - ' - [ I


- (n-l)t (a-xr- I
1 an-I l
I [2n-I - I]
tllz= (n-l)k -an-I
-- ·

316
CHEMICAL KINETICS 317

4. k = pA e-Ea!RT
p = probability factor
A = frequency factor
Ea = activation energy at temperature T

In~ = Ea Ti - T2 .
k2 R T1T2
5. If t 1 and t2 are half-lives for two different initial concentrations a 1 ' and a2
respectively, then

n =

6. t314 = 2n-l + l,
t112

where t314 = three-fourth life


t 112 = half-life.

Theoretical Problems

1. Will the order of reaction be integral always?


Ans. : The rate equation of a reaction is an experimental quantity and may be like

- dCA = k. c~' for the reaction nA --t product n' may be different from the coefficient n
dt
and is evaluated experimentally. Thus, it may be integral or fractional. It need not be integral
always.
2. Can the order of a reaction be negative?
Ans. : For a reaction of the type n 1A + n 2B --t n 3C + n4D, the rate equation may
be of the type-

r
= kcn'1 cn'2
A B '

where n( + n2 is the order of the reaction. n( is defined as the order with respect to the
reactant A and n2 is defined as the order with respect to B. The order is an experimental
quantity and n( and n2 may be anything positive or negative, integral or fractional. It is,
however, difficult to explain negative order. The substance with negative order acts as an
inhibitor. Thus, there should be more than one reactant for the exhibition of negative order
by one reactant. Experimentally some reactants in heterogeneous catalytic reaction have
been found to show negative order.
3. The order of a reaction may often be changed by varying the concentration of the
reactants. Correct or justify.
Ans. : The order of a reaction is the sum of exponents of the concentration terms
in the rate equation and is an experimental quantity. The rate equation for the reaction,
v 1A + v2 B --t v3 C + v4 D
318 PROBLEMS ON PHYSICAL CHEMISTRY

may be R = k C,'.;' C~ under a particular set of conditions. Then the order with respect to
A is m and with respect to B is n. These are constants under a particular set of conditions
and is independent of rate of concentration terms.
Hence, the order of a reaction cannot be changed by varying the concentration of the
reactants, and the above assertion is incorrect. However, if a reaction is such that the rate
equation is changed when the concentrations of reactants are varied after a specific
concentration, then the order may depend on concentration. But that is very rare.
4. The rate of a reaction changes exponentialy with time but not the specific rate constant.
Explain.

Ans. : The rate of reaction A~ product is


normally given by -dCA oc C~ in simple
dt
cases, where CA is the concentration of A at a particular time and n is an experimental
quantity.
-dCA
Hence, - - =ken.
dt
k is the proportionality constant and is the specific rate constant. Hence, it is independent
of both, the rate and the concentration. The integration of the above equation will give
concentration CA as a function of time, and CA changes continuously with time usually
exponentially (linearly only for zero order reaction so that in that case rate is independent
of time) and since rate depends on concentration, it obviously depends on time. For first
order CA = ae-k1, where a is initial cone. so that rate = ake-kt, i.e., it varies exponentially
with time. For other order reactions, the variation is inverse or some power inverse, etc.
The concentration goes on decreasing with time never becoming zero. The variation is,
thus, comparable to exponential variation for rate. Thus. although the rate varies with time,
the specific rate is constant.
S. Zero order reaction must be multistep.-Explain. Does such a reaction go to
completion?
Ans. : For an elementary reaction v1A ~ Product,

the rate equation for this reaction will be r = k. C:,{ and the order will be v1. Thus, if a
zero order reaction is elementary, then the reaction has to be OA~products, which violates
the law of conservation of mass. So, a zero order reaction must be multistep.
The rate equation for a zero order reaction has to be
dC
-dt = k.Co.
.. dC = -kdt.
Integrating C = -kt + le , where le is integration constant and C0 is the initial
concentration at t = 0.
:. C = C0 - kt.

Hence, a zero order reaction will go to completion (C = 0), and time period is Co
k
6. The stoichiometric representation of a reaction is given by A + 2B~3C + D. Express
the rate of the reactions in terms of change in concentration of each of the constituents
and also in terms of the advancement of the reaction.
Ans. : With respect to concentration the rate equation may be written as--
CHEMICAL KINETICS 319

-dCA
- - = k.CA.CB2
dt

and -dC8 , dCc and dCo may be defined similarly. With respect to degree of advancement
dt dt dt
s, the rate may be defined as follows. Let n~ be the initial number of moles of A, n~

initial number of moles of B and n~ and n~ are initial numbers of of moles of products
C and D respectively. Then, if s
is the degree of advancement at time t, the number of
moles of reactants and products at that time are

nA = n~ - S
nB = n~ - 2s
nc = n~ + 3s
(} s.
nD = no+
_ dnA ds
.. =
dt dt
_ dnB
dt
= 2dsdt
dnc
dt
= 3dsdt
dn 0 ds
dt
= dt

Defining ds as rate,
dt
ds = _ dnA = __!_ dn 8 = I dnc = dn 0
dt dt 2 dt 3 dt dt
If the rates are now expressed in terms of concentration C gm-moles/lit,

c = .!!.. .
v
n = VC.
If V is constant, then

dn = V. dCA.
dt dt
.. ds = - v. dCA = _..!..v. dCB = _!_v. dCc = v. dCo.
dt dt 2 dt 3 dt dt

Since the volume is constant, the rate is defined as ds. __!__.


dt v
·If the reaction is elementary, then the rate equation can be written .as

r = __!__, dS = dCA = __!_ dCB = _!_ dCc = k. C C2'


V dt dt 2 dt 3 dt A B
where k is the specific rate constant.
320 PROBLEMS ON PHYSICAL CHEMISTRY

7. Arrhenius A factor always have the same unit as the rate constant. Comment.
-Ea
Ans. : We know that specific rate or rate constant, k =A. e RT , where A is the Arrhenius
A factor and, E0 is activation energy, R is the gas constant and T is the temperature in
-Ea
0
K. Since e RT is unitless A must have the same unit as k.
8. The specific rate constant for a reaction has the unit lit 2.mole-1.sec 1. What is the
order of the reaction?
-dC
Ans. : For a reaction of n-th order, = k. en.
dt
dC
k

Therefore, the unit of k is (gm.mole/lit) 1-n.sec-I,


since C is generally expressed in gm-mole/lit. Thus, unit can be (gm-mole/lit)-2.sec- 1, when
n = 3. Therefore, the order is 3.
9. For the mechanism,

~ C
A+ B ..---
k~
C~D.
(i) Derive the rate law assuming the steady state approximation to eliminate the
concentration of C.
(ii) Assuming that k3 << k2, express the pre-exponential factor A and the overall
activation energy E for the formation of Din terms of A 1, A2 and A 3 and El' E 2 and E3
for the three steps.
(iii) What is the significance of (E 1 - E 2)?

Ans. : (i) The rate of formation of D is dCo = k3Cc,


dt
where C0 is the concentration of D and Cc is the concentration of C.
Similarly, CA and C8 will be taken as the concentration of A and B.
Assuming steady state in Cc, the rate of formation = rate of decomposition, i.e.,
k 1 CACB = (k2 + k3) Cc-
k1
Cc= --CACB.
k2 +k3

Thus, r -- --CAC
k3kl
8.
kz +k3

(ii) If k3 << k2 , then r = kik 3 .CAC8 .


kz
:. specific rate k = kik 3
kz
CHEMICAL KINETICS 321

By Arrhenius equation,
E
k=AeRT
_!!1_ -~
k2 = A2 e RT k3 = A3 e RT.
where A's are frequency factors, k's are rate constant and Tis the temperature in °K and
E's are- activation energy.
By the problem,

k = k1k3
k2

Ae-EIRT = A1A3. e
A2
. . overall activation energy,
E = E 1 + £ 3 - E2

and A= A1A3.
A1
(iii) E 1 - E2 is the difference in activation energy of reactants and products in a reversible
process. Thus, it is the heat of reaction W.
10. "All reactions are not necessarily characterised by a,1 order."-Explain.
Ans. : The rate equation of a reaction is an experimental quantity and the order of
a particular reactant is power of the concentration term of that reactant in the rate equation.
The total order is the sum of the exponents of the concentration terms.
If the rate equation is such that there is concentration terms only in the numerator
and all concentration terms have a definite and fixed power, then the reaction is characterised
by a fixed order. But, if the rate equation is such that fixed power cannot be ascertained
to a concentration term, then that reactant has no fixed order. If a concentration term occurs
in the denominator, this may happen. Let the rate equation for t~e reaction,
v 1A + v 2B ~ v 3C + v 4D,

be r = k-,c;
--CB,
n

k. +C~
then there is a power series in CA, and A does not have any fixed power. Hence, the reaction
does not have a fixed order. Thus, all reactions are not necessarily characterised by a fixed
order.
11. What is the difference between an unstable intermediate and an activated complex?
Ans. : An unstable intermediate is an actual chemical species which perhaps may be
stabilized by changing reaction conditions. The bond order of atoms is normal in th.ese
intermediates and it occurs at a minimum, albeit a small minimum, on the potential energy
curve.
The activated complex is a postulated species having maximum energy in the reaction
pathway of the conversion of reactants to products. Stabilization results only by a change
of bond order and length, and vibration in the direction of product or reactant. The bond
order of atoms of an activated complex are sometimes unusual.

Proh. Phv C'hPrn - ?1


322 PROBLEMS ON PHYSICAL CHEMISTRY

12. The reaction 2NO + 0 2 ~ 2N0 2 proceeds through the following steps.
2NO ~ Np 2 (fast)
Np 2 + 0 2 -1 2N0 2 (slow)
Find the order of the reaction. The overall rate constant is found to decrease with the
rise in temperature.-Explain.
J dC
Ans. : The rate of formation of N0 2 is r = 2 ; 02 = k3.CN202 .C02 ,
where k 3 is the specific rate of the 2nd stage. Since, the first step is fast, it must be reversible
with the specific rate k 1 and k2 for the forward and backward reactions.
. . the equation is :
2NO ~ N20 2

Assuming steady state in N20 2 ,


k1 C~o = k2CN 20 2 + k3CN 20 2.Co2
k1C~o
=

And r

If k2 >> k3,
k1k3
then r = --. 2
CN 0 .C02 .
k2
Hence, the order of the reaction is 3. Now, the overall specific rate,

k = -k1k3
k2
= Keq.k 3,
where Keq is the equilibrium constant for the !st step.
.. Ink = In Keq + In k 3 .
dlnk din Keq dlnk1
.. -- = +---.
dT dT dT
Ea Ml E3
.. = RT2 + RT2 .
RT 2
.. the overall activation energy,
Ea= liH + E3.
If liH is highly negative, then overall Ea is -ve. Hence, rate decreases with increase
in temperature.
13. Show that if A undergoes two simultaneous reactions to produce B and C according
to A ~B, A ~C. then Ea, the observed activation energy for the disappearance

of A, is given by E0 = k1E1 +k1E2


k1 +k2
CHEMICAL KINETICS 323

Ans. : We know that

where k 1 and k2 are specific rate constants for the 1st and 2nd steps of the reaction and
A 1 and A2 are Arrhenius factors for steps 1 and 2. E1 and E2 are activation energies for
steps 1 and 2. The specific rate for overall change for A to B and C is k 1 + k2 .
.. k = k 1 + k2 •
dlnk dln(k1 +k2) Ea
.. dT = dT = RT 2 '

where E0 is the overall activation energy for the decomposition of A.

But dln(k1+k2 ) = _ l_ !!_(ki +kz) = _ l_ (dk1 + dk 2 )


dT k1 +k2 dT k1 +k2 dT dT

= _I_[!!_
k +k dT
.(A1 .e -E1IRT) + !!_(A
dT
. 2 .e -EzlRT)]
1 2

=k __l_[A _§_ 1· 2
-E tRT A ~
.e 1 + 2· 2 .e
-EzlRTJ
1 +k2 RT RT

E = E1k1 + E2k2
a k1 + kz .
14. Comment on the following :
Elementary reactions with molecularity greater than 3 generally don't occur.
Ans. : The reactants are activated as a result of molecular collisions. But the collisions
generally requires high amount of energy because of internuclear and inter-electronk
repulsions. For a collision two molecules are must. If the molecularity of an elementary
step is 3, then three molecules must collide requiring very high energy. For a 4-body collision,
energy required will be still higher and, therefore, unknown. It is far easier to have two-
body collisions and that will take place. Therefore, elementary reaction of molecularity 4
or greater is unknown.
15. What is stoichiometric number ?
Ans. : Stoichiometric number of an elementary reaction is the number of times this
step occurs in the mechanism for the overall reaction as given.
For example, the mechanism of decomposition of N 20 5 may be as follows
Step-I N20 5 ~ N0 2 + N0 3
Step-II N0 2 + N0 3 ~ NO + 0 2 + N0 2
Step-IH NO + N0 3 ~ 2N0 2
The stoichiometric number of Steps I, II and III are 2. 1 and l respectively. If there
is a rate limiting step in a reaction, then sometimes stoichiometric number of the rate limiting
stage is defined as the stoichiometric number of the reaction.
324 PROBLEMS ON PHYSICAL CHEMISTRY

16. "Unimolecular reactions are not always of first order."-Justify the statement using
Lindemann mechanism.
Ans. : According to Lindemann mechanism even in unimolecular reactions the
molecules are activated as a result of collision with a second body. Let the reactant be
A and the second body be M. But there is a time lag between activation and reaction. Thus,
by Lindemann theory, the mechanism of the unimolecular reaction is :

A+ M A*+ M

A* __, product
With k2 << k_ 1 the rate
r = kz.c:.
Assuming steady state in c;,
k,CACM = k_, c:.cM + kzC:.
k1CACM
.. c·A = k_ 1CM+k 2
k1k2CACM
Thus, r = k_ 1CM + k 2
Since k 2 << k_ 1,
k1k2CA
.. r = k.1
This is the first order. But at a very low pressure the number of collision is much
less. As a result at some low pressure depending on the system, k 2 will be much greater
than k_ 1. Then the rate, ·
r = k 1 CACM.
If M = A, then r = k 1• C~ ,
i.e., order is 2.
Thus, unimolecular reactions are. not always first order.
17. What is 'Bottle-neck' principle in chemical kinetics?
Ans. : The slowest step in a multistep reaction determines its, rate. Steps before it
are in rapid equilibrium. This slowest step serves as bottle-neck, since in the rate expression
the steps after it need not be considered.
18. What is the essential condition of a chain reaction to occur?
Ans. : Chain reactions are multistep reactions where one or more intermidiates are
produced. These intermidiates may be produced by collision or from radiation. The
intermidiates will have to be highly reactive and may be able to reproduce the same
intermidiate in later reactions so that the reaction continues through a series of steps which
are termed as chain propagating steps.
. B . f
19. The rate of a reaction is given by log k = A - - + C log T. Fmd the. value o
T .
activation energy. [B.U. 2006]
CHEMICAL KINETICS 325

Ans. : Given, log k =A - ~ + C log T

2.303B
or, In k = 2.303 A -
T
+ C In T

2 3
or, In k - In Tc = 2.303 A - 2.303 · 03B
T
:. k = Tc e2.303A e-2.303Brr = A e-EJRT
Hence, activation energy f.'a = 2.303 BR.
20. (a) The following initial rate data were obtained for the reaction :
2NO(g) + 0 2 (g) ~ 2N0 2 (g)

Partial Pressure of Initial rate


NO
-------~--t------------·-------~-----------+-----------

Run I v

Run 2 4v

Run 3 2v

(i) What is the rate law for this reaction?


(ii) One of the mechanisms proposed for this reaction is :
kl
NO(g) + 0 2 (g) ~ N0 3 (g)
k_l

N0 3(g) + NO(g) -----+ 2N0 2(g)


Obtain the rate law predicted for this mechanism, assuming a steady state
concentration of N0 3.
(iii) Predict the rate law for this mechanism, if the first equilibrium step 1s
established quickly and the second step is slow. [JAM 2009]

Hints : (i) r = _!_ dCN02 = kCNof:o,_·


2
2 dt

(ii) dCN03 =
dt

= k1k1 C~oC02
k_ 1 +2k 2 CNo

(iii) k_ 1 > > k2 •


326 PROBLEMS ON PHYSICAL CHEMISTRY

Numerical Problems
1. Determine the rate law for the reaction A + B + C ~ Products
from the following information when the initial concentration of A is doubled, the initial
rate of reaction doubles. Doubling the initial concentration of B doubles the rate twice and
doubling the initial concentration of C cuts the reaction rate by half.

Soln. : Let the rate equation be r = k Ci C";}- C2 .


Let the initial concentration of CA be a, that of C8 be b and that of Cc be c.

Thus, r = ka"'b 112 c"3.


By the problem,
if CA = 2a,
r'I = k(2a)"' bv2c"3
r;
= (2)"' = 21 = 2 as r; = 2', .. VI = 1
r1

r'2 = ka"'(2b) 112 c"3 = 4 r


So ,r2- = 2v2 = 4 = 2 2.
r
:. v2 = 2.
r} = ka"1b"2(2c)V3
r'
....1
r
= 2v:i = --2'I .. v3 -- -1 ·

Thus, r = kCAC~Cc 1

2. For the reaction A + B =
C + D, it is found that doubling the concentrations of
A and 8, the rate increases 8-fold, but on doubling the concentration of A, the rate increases
4-fold. What is the order of reaction with respect to B?

Soln.: Let r = kCiC~2= ka"'bvz.


a = initial concentration of A
b = initial concentration of B.
By the problem,
Sr= k(2a)"' (2b)v2
Thus, 8 = k2"'2"2 = 2"'+vz = 23.
,', VI + V2 = 3.
But 4r = k(2a)"' b 112 •
:. 4 = 2 11 = 2 2 •
Thus, v 1 = 2.
Hence, v2 = l.
Thus, the order of reaction with respect to B = 1.
3. Given that for the decomposition of gas of a gaseous substance at 825 K, the specific

rate k is 2.89 x 10-2 litre.mole- 1.sec- 1. According to the equation : - dCAB =:::: kC~ 8 •
dt
CHEMICAL KINP.TICS 327

how many molecules of AB will decompose per sec per cc at atmospheric pressure? (Assume
one way reaction.)
n P latm
Solo. : CAB = V =
RT
= 0. 082061. atm. K- 1 . mol- 1 x 825 K

= 1.477 x10-2 mole.lic 1

dC =2.89 x 10- 12 x (1.477 x 10-2)2


dt
=6.3046 x 10- 16 mole.lic 1.sec- 1.
dN
=6.3046 x 10- 16 x 6.022 x 1023
dt
=3.7966 x 108 molecules.lic 1.sec 1
= 3.79 x l 05 molecules.cc- 1 .s~c- 1 .
Thus, 3.79 x 105 molecules will decompose at atmospheric pressure per sec per cc.
4. In the reaction (COOH) 2-7CO + C0 2 + H20, the following results were obtained :
Time (min) 0 300 450 600 1200
Volume of KMn0 4
required to titrate 22 17.0 15.0 13.4 7.9
oxalic acid (ml)
Show that the reaction is of 1st order, and determine the value of the velocity constant.
[Bangalore B.Sc. 1978]

Solo. : We know that k =~ In a


a-x
By the problem, a = 22.
1 22
:. k = - - ln = 8.594 x 10- 4 min- 1
300 17
1 22
k = - -ln = 8.510 x io- 4 min- 1
450 15

k = - - ln ~ = 8.263 x 10- 4 min- 1


1
600 13.4
1
k =- -ln -33.._ = 8.534 x 10- 4 min- 1.
1200 7.9
k is approximately constant for each case. So, the reaction is of 1st order. The average
value of velocity constant is 8.5 x 10-4 min-I.
5. From the following data show that the decomposition of an aq. solution of H20 2
is of l st order.
t (min) 0 10 20
N (ml) 22.8 13.8 8.20
where N is the volume of KMn0 4 solution in ml needed to decompose a definite volume
of the peroxide solution.
328 PROBLEMS ON PHYSICAL CHEMISTRY

Hints : Choose the formula,

k = -1 I n -a- .
t a-x
Find out a = 22.8 ml.
Then proceed as in Problem 4.
k = 5.1 x 10-2 min- 1.
6. 1 gm-mole of ethyl acetate was hydrolysed with 1 gm-mole of NaOH when the
concentration fell according to the following data. Show that the reaction is of 2nd order,
and calculate the value of k.
t(min) 0 4 6 IO 15 20
(a-x) 8.4 5.3 4.58 3.50 2.74 2.22
[Deihl University B.Sc. 1978]

1
Soln. : We know that - =kt.
a-x a
x
= k.
a(a - x)t
By the problem,
a = 8.4 gm-mole/lit, (x =0 when t = 0)
x = 3.1 gm-mole/lit, when t = 4 min
x = 3.82 gm-mole/lit, when t =6 min
x = 4.9 gm-mole/lit, when t = 10 min
x = 5.66 gm-mole/lit, when t = 15 min
x = 6.18 gm-mole/lit, when t = 20 min
1 3.1
.. k = - x = 0.0174079 lit.moJe- 1.min-1
4 8.4x5.3
1 3.82
= -6 x = 0.0165488 lit.mole- 1.min-1
8.4 x 4.58
4.9
= -x = 0.01666 lit.mole- 1.min- 1
10 8.4 x 3.5
5.66
= - x = 0.016394 lit.mole- 1.min- 1
15 8.4x 2. 74
6.18
=- x = 0.01657 lit.mole- 1.min- 1
20 8.4 x 2.22
k is approximately constant and the average value is 1.6 x 10-2 lit. moI- 1sec- 1• So,
the reaction is of 2nd order.
7. The rate constant (k) of a second order reaction has the value 2.4 x 10--24 , the
concentration being expressed in molecules/cc and time in seconds. What will be the
magnitude of k if the concentration be expressed in moles/lit and time in minute?
Soln. : Second order rate constant is 2.4 x 10-24 cc.molecule- 1.sec- 1.
Let 2.4 x 10- 24 cc.molecule- 1.sec- 1 = x litre.mole- 1.min- 1.
CHEMICAL KINETICS 329

.. x = 2.4 x 10 _24 (~). mole . min


litre molecule sec

= 2.4 x 1o-24 ( cc )x 6.022 x I 023. molecule . 60 sec = 0.0867.


I000 cc molecule sec
. . rate constant is 8.67 x 10-2 litre.mol- 1.min- 1.
8. The activation energy for the decomposition of·a gas is 80 kJ. What percentage
of gas molecules possess sufficient energy to react at 500 °K?
Soln. : We know that,
n = n0 e
-Eu I RT
.
By the problem, Ea = 80 kJ = 80000 J; T = 500 K.
80000
n ---
.. = e 8.314x500
no
80000
.. In~= = -19.25 .
no 8.314 x 500
n
.. = 4.3635 x 1o--9.
no
. . the percentage of gas molecule possess sufficient energy to react at 500 °K is
4.36 x 10-7 .
9. A substance decomposes according to a second order rate law. If the rate constant
is 6.8 x l<J-4 lit.mole- 1.sec- 1, calculate the half-life of the substance when the initial
concentration is 0.05 mole.lit. [C.U.(H) 1989)
Soln. : We know that,
I 1
kt= - - - - .
a-x a

At half-life (t
112
) x =a
2

tl/2 x = kxa
By the problem,
k = 6.8 x 10-4 lit.moJe- 1.sec- 1•
a = 0.05 mole.lit.

t
112
= 6. 8 x 10-4 x 0. 05
= 29411.76 sec = 8.17 hours.
So, the half-life of the substance is 8.17 hours.
10. The first order gas reaction AB 2 X2 ~AB 2 + X2 has k1 = 2.28 x 10-5 sec-l. What
per cent of sample of AB2X2 would be decomposed by heating at 320 °C for 2 hours.
[C.U.(H) 1989]
1 a
Soln. : We know that, t = In
k a-x
330 PROBLEMS ON PHYSICAL CHEMISTRY

By the problem,
k = 2.28 x 10-5 sec-I
t = 2 hours = 2 x 60 x 60 sec.

.. 2 x 60 x 60 = ln-a-
2. 28 x 10-5 a- x
a
:. In = 2.28 x 1o-5 x 2 x 60 x 60
a-x
a
or, In-- = 0.16416.
a-x
.. _a_ = 1.178402844
a-x
a-x
or, = 0.848606234
a
x
or, 1 - = 0.8486
a
x
or, =1 - 0.8486 = 0.1514.
a
So, the percentage is 0.1514 x 100 = 15.14.
11. Half-life of decomposition of a compound was found to be 50 min. When the
concentration to start with was halved, the half-life becomes 100 minutes. What is the order
of the reaction? [Delhi University B.Sc. 1972]

log !.L + log az


tz a1
Solo. : We know that, n =
log az
a1
By the problem,
a 1 = a, t 1 = 50 min

a2 = !:!.., t2 = 100 min.


2
50 1
log-+log-
n = 100 l 2 = 2.
log-
2
So, the order of the reaction is 2.
12. A 1st order reaction has rate constant equal to 1.25 x 10- 4 sec- 1 at 298 °K and
8.5 x 10-4 sec- 1 at 318 °K. Calculate the activation energy of the reaction.

Solo. : We know that, In --1.. k = E(T-T) 2 1


k1 R TT __fL
1 2
,

where k 1 and k2 are two rate constant at T 1 and T2 and Ea is the activation energy.
CHEMICAL KINETICS 331

. . by the problem,
k1 = 1.25 x 10-4 sec- 1
k2 = 8.5 x 10-4 sec- 1
Tl = 298 °K
T2 = 318 °K.

In 8. 5 x 10-4 = Ea ( 318 - 298)


1.25x10-4 8. 314 318 x 298
8.5 20
or, In - -
I. 25
= -Ea- x
8.314 318x 298

= 8.314x318x298 In~
Ea 20 1.25
= 75514.09 J = 75.51 kJ,
So, the activation energy of the reaction is 75.51 kJ,
13. On raising the temperature from 27 °C to 37 °C, the rate of reaction is doubled.
Calculate the acivation energy. [C.U.(H) 1981]

Soln. : We know that, In~ = Ea T2 - Ti


k1 R T1T2
By the problem,

~. = 2
k1
T2 = 37 °C = 273.15 + 37 = 310.15 K
T1 = 27 °C = 300.15 K.

.. In 2 = Ea (310.15-300.15) Ea · 10
8.314 = 8.314 x 310.15x300.15.
310.15x300.15

= 8.314x310.15x300.15 1n 2 = 53647 .02 1 = 53•64 kJ,


. 10
So, the activation energy is 53.64 kj.
14. The decomposition of NH 3 catalysed by hot tungsten surface was studied at
1100°C and the following data were obtained for the time of half decomposition with different
initial pressure.
P0 mm of Hg 300 200 100
t 112 min 8.6 5.6 2.9
Find out the order 9f the reaction and the rate constant. [C.U.(H) 1977]
Soln. : We know that,

log !L +log a 2
t2 a1
n =
log a2
a1

where t 1 and t 2 are two half-lives at pressure a 1 mm of Hg and ai mm of Hg.


332 PROBLEMS ON PHYSICAL CHEMISTRY

By the prnblem,
t1 = 8.6 min
tz = 5.6 min
al = 300 mm of Hg
az = 200 mm of Hg.
8.6 200
log-+log-
.. n = 5.6 200 300 = - 0.058.
log---
300
Again, using the data,
t1 = 5.6 min
t2 == 2.9 min
a = 200 mm of Hg
az = 100 mm of Hg,
5.6 100
log-+log-
n 2. 9 200 = 0.051.
= 100
log--
200
.
So, the order of the reaction =-0.05+0.06 = 0.
2
For zero order reaction,

k =

Putting a = 300 mm of Hg = -300 atm and t 1 = 8.6,


760 12

300
k = = 0.023 atm.mln-1.
2 x 760x8.6
lS: The decomposition of a gas is a second order reaction. Using the gas at 200 mm
of Hg initial pressure at a certain temperature, if 25% of the gas decomposes in 30 min, in
what time 50% of the gas will decompose? Calculate the specific rate constant. [C.U.(H) 1976]
Solo. : We know for the 2nd order rea~tion,

x
k = ta(a-x)'
1 x
=k a(a-x)

I I
-a 1 4a
= 4 = =
k~
k ax(a-~) ax-
4
3ka
CHEMICAL KINETICS 333

a a
1
Again, t 1 = 2
= - _2_ =
k ax(a-~)
/2 a
k ax-- ka
2

~ =3.
t114

.. t, = 3 x t, .
/2 14
By the given problem, t1
14
= 30 min.
:. t 1
/2
=3 x 30 = 90 minutes.
So, the time required for 50% decomposition is 90 minutes.

:. k = -Ix -I,
t112 a
200
By the problem, a = 200 mm of Hg = atm.
760
1 I
.. k = x 200 = 0.0422 atm.min- 1.
90
760
16. At 100 °C, the gaseous reaction A~38 + C is observed to be !st order. Starting
with pure A, it is found that at the end of I 0 minutes the total pressure of the system is
130 mm of Hg and after a long time 400 mm of Hg. From these data, find (a) the initial
pressure of A (b) the pressure of A at the end of I 0 minutes (c) the rate constant and
(d) the half-life.
Soln. : A ~ C + 38
Given that, after 10 minutes, pressure = 130 mm of Hg.
After a long time, pressure = 400 mm of Hg.
We may assume that after a long time, the decomposition is complete. So, one gas
molecule leads to four gas molecules as shown above. Since after a long time the pressure
is 400 mm of Hg with four molecules (38 + C), So
400
(a) the initial pressure is -mm of Hg = 100 mm of Hg.
4
(b) Let, after 10 minutes, the pressure of A be reduced by x mm of Hg.
A ~ 38 + C
10 min, 100 - x x 3x
So, the total pressure, (100 - x + 3x + x) = 100 + 3x mm of Hg.
By the problem, 100 + 3x = 130
or, x = 10.
So, the pressure of A at the end of 10 minutes is (100 - 10) mm of Hg = 90 mm
of Hg.
(c) We know that for 1st prder reaction,

k =!In-a-.
t a-x
334 PROBLEMS ON PHYSICAL CHEMISTRY

By the problem,
t= 10 min
a = 100 mm of Hg
a - x = 90 mm of Hg.

k = _!_ Jn lOO = 0.0105 min-1.


10 90
(d) We know,
0.693
t
112 = k
0.693
.. t
112 = 0.0105 = 66 min.
17. At 200 °C the gaseous A~ 2B + C is observed to be second order. Starting with
pure A it was found that at the end of I 0 minutes the total pressure of the system is 176.0
mm of Hg and after a long time 270 mm of Hg. Calculate the rate constant and half-life
of the substance.
Soln. : A ~ 2B + C
We may assume that after a long time, the decomposition is complete. So, one gas
molecule leads to three gas molecules as shown above.
Since after a long time the pressure is 270 mm of Hg with three molecules (2B+C).
270
So, the initial pressure is mm of Hg = 90 mm of Hg.
3
A ~2B + C
10 min 90 - x 2x x
So, the total pressure at I 0 minutes is (90 - x + 2x + x) = 90 + 2x mm of Hg.
By the problem,
90 + 2x = 176
x = 43 mm of Hg.
We know that for 2nd order reaction,

k =! x
t a(a-x)
Here t = I 0 minutes
x = 43 mm of Hg
(a - x) = (90 - 43) = 47 mm of Hg.
43
.. k = _!_. = 1.0165 x 10-3 mm of Hg.- 1min- 1.
10 47x90
A . I I
gam, t 112 = ka = 1.0l65xl0-3 x90 = 10' 93 min.
18. A dilute solution of cane sugar was hydrolysed into glucose and fructose by addition
of di!. HCI. The progress of the reaction was followed in a polarimeter tube by observing
the angle of rotation at different times. The results are :
CHEMICAL KINETICS 335

t (min) 0 10 20 30 80 00

Angle of roation 32.4 28.8 25.5 19.6 10.3 14.1


Show that the reaction is of l st order and calculate the half-life. [D.U. 1982)

. ts
H ID :
k = -n
11 eo-e= . [Ans. 118 min]
r e, -e=
19. Methyl acetate is hydrolysed in approx(N) HCI at 25 °C. Aliquotes .of 5 ml each
are removed at intervals and titrated with O. l 85(N) NaOH. Calculate the first order rate
constant from the following data :
t (sec) 339 1242 2745 4546 oo

V (ml) 26.34 27.80 29.70 31.81 39.81 [Utkal 1980]


Soln. : We know that-
k =~ V= - Vo
t v= - v,
l V= - V0
=k V= -V1

t1 = -1 1n V=-Vo
k V= -V, 1

t2 = J_ln V=-Vo.
k V= -V, 2

l V= -V,
(t2-t 1)=-ln I
k V= -V,
2

By the problem,
t2 = 1242 min
t1 = 339 min
V 00
= 39.81 ml
VTJ = 26.34 ml
vt2 = 27.80 ml
39 31 26 34
.. 1242 - 339 = J_ln · - · = 0.11472535
k 39.81-27.80 k

:. k = 0.11472535 = 1.27 x 10-4 min-1.


903
So, the value of rate constant is 1.27 x 10-4 min- 1•
20. l gm-mole of ethyl acetate was hydrolysed with l gm-mole of NaOH when the
concentration fell according to the following data. Show that the reaction is of second order
and calculate k.
t (min) 06 10 4 15 20
a-x 5.3 8.04
4.58 3.50 2.74 2.22
Soln. : We know that for 2nd order reaction,
336 PROBLEMS ON PHYSICAL CHEMISTRY

I
= - + kt.
a-x a
l
.. = - + kt1 (i)
Q-X1 a

(ii)
Q-Xz

Subtracting equation (ii) from equation (i), we get

By the problem,
a - x1 = 8.04, when t 1 =0
a - x2 = 5.3, when t2 = 4.
l
---- = -4 k
8.04 5.3
-2.74
or, ----
8.04x5.3
= -4 k
2.74
or, k = 8.04x5.3x4
= 1.607 x 10-2 mole-I .cc.min-I.
Again, putting a - x2 = 4.58, when t2 = 6, we get

_I_ - _I_= - 6k
8.04 4.58
or, k = 1.567 x 10- 2 mole- 1.cc.min- 1.
Again, putting a - x 2 = 3.50, when t 2 = 10, we get
k = 1.613 x 10- 2 mole- 1.cc.min- 1.
k is approximately constant. So, the order of the reaction is of 2nd order. The average
rate constant is 1.6 x 10-2 mole-I .cc.min-I.
21. When ethyl acetate is saponified by NaOH, the progress of the reaction can be
measured by titrating the unchanged alkali against a standard acid at definite intervals. Using
equal concentration of ester and alkali the following results were obtained in the experiment :
t (min) 0 5 25 55 120 oo

acid (cc) 16 10.24 4.32 2.31 I. I 0


Show that the reaction is of second order. What fraction of ethyl acetate does decompose
in 30 minutes? [C.U.(H) 1969)
Soln. : We know that

k = !. x
t a(a -x)
By the problem,
when t = 5; a = 16; (a - x) = 10.24 an~ x = (16 - 10.24) = 5.76.
CHEMICAL KINETICS 337

1 __
5 ._7_6_ = 7.03 x 10-3 cc- 1.min- 1•
k :;: -x
5 16x 10.24
~gain, using t = 25; (a - x) == 4.32 and x = 11.68.
68
k = _!__ x l l. = 6.75 x 10-3 cc- 1.min- 1•
25 16x4.32
Again, using t == 55; (a - x) = 2.31; x = 13.69.
k = 6.73 x Io-3 cc- 1.min- 1.
k is approximately constant and order of the reaction is second. The average rate constant
is 6.83 x 10-3 cc 1.min-1.

k =! x
t a(a- x)

6.83 x 10-3 = _!__ x


30 16(16-x)

x
or, 6.83 x 10-3 x 30 ==
16(16-x)

x
or, 3.2784 = 16-x
or, 4.2784x = 16 x 3.2784
or, x = 12.26.
12 26
So, the fraction of ethyl acetate which decomposes in 30 minutes is · = 0.77.
16
22. A certain substance A is mixed with an equal quantity of a substance B . .At the
end of 1 hour, 75% A reacted. How much A will be left unreacted at the end of 2 hours
if the reaction is of (a) first order in A and independent of B (b) I st order in A and first
order in B and (c) zero order in A and independent of B?
[Delhi University B.Sc.(H) 1972)

a
Solo. : (a) k = -tI ln--
a-x

= -601 I n a
- - (by
0.25a
the Problem)

= 0.0231049 min-1.
. . after 2 hours,
a
In - - =kt= 0.0231049 x 120
a-x
= 2.7725887
a
or,
a-x
= 16
I
i.e., a - x = -a.
16
100
A left unreacted will be - %
16
= 6.25 % ,
338 PROBLEMS ON PHYSICAL CHEMISTRY

a
(b) = -I +kt
a-x a
by the Problem,
1
- - - _!_ = 60k
0.25a a
3 I
.. - = k or, k = -
60a 20a
x l
Now,
a(a-x)
= kt = 20a x 120 = -6a .
x a-x I
or,
a-x
= 6, .. x
=6
.. a
x
=6
7
or, -ax = 67
6
x = -a.
7
l
Hence, a - x = -a = 14.285714% = 14.29%.
7
(c) By the problem,
x = kt, 0.75a = 60 k.
0.75
k =-·-a.
60
The reaction will be complete in time,
60 .
t = -ak = 075
a
. -a
___
= --
0. 75
= 80 mm.
60
Thus, no amount will remain at the end of 2 hours.
23. A drug is known to be ineffective when it has decomposed to the extent of 30%.
The original concentration of the sample is 500 units/ml. When analysed 20 months later,
the concentration was found to be 420 units/ml. Assuming that the decomposition is of
first order, what will be expiration time of the sample? What is the half-life of this drug?
[C.U.(H) 1988]
Solo. : By the problem,

k = -1 ln
500
= 8.71766 x 10-3 month- 1.
20 420
The expiration time t (when x = 0.3a)
= I In a
k a-0.3a

= 8. 71766I x 10-3 In 0. 7
= 40.91405 months = 41 months.
The half-life here is the time when it decomposes to the extent of 15%, since the drug
expires after decomposition to 30%. Then
CHEMICAL KINETICS 339

t II = I ln-a-
2 8.71766x!0~3 o.g5a
= 18.642475 months = 19 months.
24. The decomposition of a gas at an initial pressure of 600 mm of Hg was studied
in a closed vessel at a certain temperature. The gas was found to be 50% decomposed in
30 minutes and 75% decomposed in 90 minutes. Show that the reaction is of second order,
and calculate the rate constant. [C.U.(H) 1981]

Soln. : We know that ' 314 = 2n-I + J.


l112

90
.. = 2n-l+I
30
.. n = 2.
I 760
Hence, k = - - = = 0.0422 atm-I.min-1.
at112 600 x 30
25. The thermal decomposition of acetaldehyde vapour CH 3CHO --t CH4 + CO is
an. irreversible process whose rate at 500 °C is given by the following two sets of d\lta
for .the pressure change at constant volume and temperature :
Initial Pressure of CH3CHO Total Pressure after I 00 sec.
400 mm of Hg 500 mm of Hg
200 mm of Hg 226 mm of Hg
Find the order with respect to CH3CHO. Calculate the rate constant at 500 °C. The
activation energy is 180 kJ. At what temperature will be rate constant be twice of what
it is at 500 °C?
Soln. : Let the initial pressure of CH3CHO be P0 . Then, the pressure of CH 3CHO
after some time will be (P 0 - x) when the total pressure will be (P 0 + x).
CH3CHO --t CH4 + CO
P0 - x x x
Thus, when P0 = 400 mm, after JOO sec, x = JOO (P 0 + x = 500) and
so, P0

r1 =-
300-400
- x
.
= 300, and hence,
. dC dx
= I (a 1 = 400) (smce, r = - - = - )
.

100 dt dt
Again, when P0 = 200 mm, after 100 sec, x = 26 and P0 - x = 174. And thus,

r2 = - 176-200
100
= + 0.26 (a 2 = 200).

log2
_ _r_2
Now, n =
log~
a2

=
tn(ok)
Therefore, n ln( 400 ) = J.~434165 = 2.
200
340 PROBLEMS ON PHYSICAL CHEMISTRY

x 100
.. k400 = - = -x = 8.33 x 10-6mm- 1.sec- 1•
t a(a -x) 100 400x 300

I 26 1 1
kzoo = - x 200x 174 = 7.47 x Io-6mm- .sec .
JOO
·. kav = 7.9 x to-6mm-1.sec-I.
Again, by the problem, using Arrhenius equation,

In
2 = _ 180000 (-1 __I_) = 180000 x (T2 - 733)
8.314 T 2 773 8.314xT2 x773
.. T2 = 792.61578 = 793 °K = 520 °C.
26. A piece of wood buried by a glacier had 25.6% as much C 14 as a recently grown
piece of wood. If the amount of C 14 in the atmosphere was the same when the old wood
died as it is now, when was the wood buried? (f 11 for C 14 = 5600 year).
2
Solo. : Let us assume that decay started when it got buried. Radioactive decay is of
1st order.
I 0.693
.. = - In No , but t 11 = - -
k N 2 k
0.693
.. k =
t112

f112
.. = In
0.693 0.256

= . 5600 In _ I_
0.693 0.256
= 11010.73 years -= 11011 years.
Hence, the wood got burried 11011 years ago.
27. A study by temperature jump method gave a relaxation time t* = 38 µs at 20 °C
for H 20 ~ H+ +OH-. Calculate k and kb for the reaction. At 20 °C, Kw= aH+a0 w
1
= 1.04 x 10- 14 .
Solo. : From this data,
xe =~l.04x 10- 14 = 1.0198 x 10- 7 mole/lit.
C +·C
And K = H ow = 1.04x10-14 = 1.87218 x 10-16.
e CHzO 55.55
Now, for the reaction, H 20 ~ H+ + OH-,

t* = ----

. . by the problem, 38 x I 0---0 =


( kJ + 2kb x I. 0198 x 10-1 ) .
CHEMICAL KINETICS 341

k
But, _j_ = Ke.
kb
.. kf = Ke.kb = 1.87218 x 10-l6kb.

38 x 10- 6 = I .
{I.87218x 10- 16 xkh +2x l.0198x 10-7 )

kb = 1.29024 x I 0 11 l.mo1- 1.sec 1


= 1.3 x 10 11 lit.mole- 1.sec-1•
k = Kekh = 2.41550 x 10-5 = 2.4 x 10-s sec-•.
1
28. For a bimolecular decomposition of HI, the rate constant is 2.8 x Io- 7 lit.
mole- 1.sec- 1 at 556 °K. The activation energy is 44300 cal. Calculate the entropy of
activation (h = 6.6 x 10-27 erg.sec, R = 2 cal/deg mole, NA = 6.023 x 1023 per mole.)
[C.U.(H) 1987]
Soln. : By the eqn. 2.18.27 in Physical Chemistry, Vol. 2 by Dr S. Pahari,

kT -
=2 e - e6S~
( RT J
' IR e - Ea/ RT
h po

E
Now, In k 2 =In A - _a
RT
44300
In A =In k 2 + .S... = In 2.8 x 10-7 + .
RT 2x556
A = 5.605 x 1010.
Naturally,
RT RT 6St!R
In A = 2 + In - - + In - - + e 0
NAh po
RT
Putting R = 8.314 x 107 erg.deg- 1.mole- 1 m and
NAh
RT
R = 0.082061 Iit.atm.deg- 1.mole- 1 in
po

8.314xl0 x556
7
0.08206lx556 .&s',;
In A= 2 + In 27 + 1n +-
6. 023 x 10 x 6. 6 x w-
23
I R

or, 24.749 = 2 + In l.162 x 10 13 + In 45.625 + M~


R

or, 24.749 = 35.904 + M,;


R
or, M~ = -92.74 J/K.
342 PROBLEMS ON PHYSICAL CHEMISTRY

29. The rate constant of a second order gas reaction, H2 + 12 ~ 2HI is 0.0234
lit.mole- 1.sec- 1 at 400 °C and the activation energy is 150 kJ/inole. Calculate !'.H!, !'.G,:
and Af,: for this reaction.
Soln. : Since the reaction is of second order,

!'.H,: = Ea - 2RT
= 150000 J.mol- 1 - 2 x 8.314 J.K- 1.mol- 1 x 673 K
= 138.81 kJ/mole.
Now, k = RT (RT)e-G6tRT
NAh po
R = 8.314 x I07erg.deg- 1.mole- 1
= 0.08206 lit.atm.deg- 1.mole- 1
T = 673.15K
NA = 6.023 x 1023
h = 6.626 x l 0- 27 erg.sec
and k = 0.0234. lit.mo1-l .sec-I.

.. 0.0234 = 7.74674 x 101 4 x e "


-G* I RT
!'.G,~
.. = 38.038483 .
RT
+
.. !'.G; = 212.83 kJ,

138 81 212 83
.A ,-.:j:
.'.:..:io = · - · = -0.109961 kJ.K- 1 = -110 J.K- 1•
673.15
30. For a first order reaction the specific rate was determined at three different
temperatures. Calculate the activation energy and the frequency factor (A). Find out !'.H;,
!'.G; and Af; at 37 °C.
T °C 27 °C 37 °C 47 °C
2.1 4.2 8
t.s+ -Ea
kT __l!_
Hints : Proceed as in Problem 28. However, here k2 = e - . e R . e RT .
h
Determine activation energy (Ea) using the given table :
[Ea = 53.4 kJ
A = 4.1 x 107

!'.H,; = 50.8 kJ

llG! = 85.66 kJ

Af~ = -107.73 J.K- 1)


CHEMICAL KINETICS 343

31. The effective rate constant of.a 1st order reaction following Lindemann mechanism
has the following values : 2.5 x I o- 3 sec- 1, when C = 5 x 1o- 2 gm-mole.lir- 1, and
4.2 x 10-3 sec-I, when C = 9.8 x 10- 2 gm.mole.sec- 1.lir- 1. Find out the rate coefficient
for the activation step.
Solo. : By Lindemann mechanism,

I k
.. = - - + - -2 .
keff k1CA k1k3

.. k -I
eff = k-1c-1
I A + ~
k3kl
Putting the value,
103 102 -I _k2
- =-kl +
2.5 5 k3kl

103 = 102 k-1 +~.


4.2 9.8 I k3k1
Subtracting, we get
2
103 - 103) = 102 10 ) I
( 2.5 4.2 ( -5-- 9.8 ·-;;;

or, (400 - 238.095) = (20 - 10.204)


k1
9 796
= · = 6.05 x 10- 2 lit.gm-mole- 1.sec- 1
161. 905
The keffequation comes from Lindemann mechanism as outlined in Physical Chemistry
Vol.2 by Dr S. Pahari, equation no. 2.14.4. Replacing M by A, i.e., CM by CA,
k3k1Ci
r =
k3 +k2CA
-dCA
--;;;--- = kefiCA.
32. Some PH3 is introduced into a flask at 600 °C containing some inert gas. Phosphine
proceeds to decompose into Pig) and H2(g) and the reaction goes to completion. The total
pressure is given below as a function of time.
t (sec) 0 60 120 00

P (mm of Hg) 260 270.5 273.14 274


The rate equation for the backward reaction is given by

r = k2 P;4 · P~2 ·
344 PROBLEMS ON PHYSICAL CHEMISTRY

Calculate a and b. Let the initial pressure of phosphine be 20 mm of Hg and the half-
life be 25 sec. Calculate k, if the decomposition is of I st order and if it is of second order.
Solo. : The decomposition reaction may be written as
4PH3 ---t P4 + 6H 2
If the initial pressure of phosphine is 4a mm of Hg and that of P 4 at some time is
x mm of Hg, then at that time,
PPH
3
= 4a - 4x
PH = 6x
2
and total pressure is 4a + 3x. Now, there is always some inert gas. Let its pressure be
b mm of Hg.
260 = 4a + b.
But at oo, all phospine decomposes. 4 particles yield 7 partides.
.. 274 = 7a + b.
14
3a = 14, i.e., 4a =4 x = 18.67 mm of Hg.
3
After 60 sec,
4a + 3x + b = 270.5
and 4a + b = 260
3x = 10.5 mm of Hg.
4a - 4x = 18.67 - 14 = 4.67 mm of Hg.
After 120 sec,
4a + 3x + b = 273.14
and 4a + b = 260
.. 3x = 13.14 mm of Hg
and 4a - 4x = 18.67 - 17.52 = 1.15.
Hence, assuming the reaction is of !st order,
4
k =! In a
t 4a-4x
1 18 67
= In · = 0.02309595 sec- 1
60 4.67
1 18 67
and k' = - - In · = 0.0232263 sec- 1.
120 l.15
.. kaverage = 0.023 sec-.1•
Hence, t 11 = 30.13043 sec.
2
Now for the above equation, the equilibrium constant

K = Pr4: P~2. = (5-)s = [ ~4 xP~2 )s


PPH3 k2 PPH3

Consequently, S = 4. Hence, a= 0.025 and b = 1.5. Thus, for the backward reaction,
with equation written as 4PH3 = P4 + 6H2, the rate equation is :

r = k • p0.25pl.5
P4 H2.
CHEMICAL KINETICS 345

If the reaction is written as

I 3
4 2
Pp4. PH2'
K= =
PPH3
But here S = I.
:. a= 0.25 and b = 1.5 again. Even if we write the equation as 8PH3 = 2P4 + 12H2 ,

a = 0.25, b = 1.5 (since S = 8), and if 4a = 20 mm and t 11


2
= 25, I st order k =O.25693
= 0.02772 sec-1, 2nd order k = 25760
x 20
= 1.52 sec- 1.atm-
.
1 = 0.002 sec- 1.mm of Hg- 1.

33. The decomposition of PH 3 at 950 K is followed by noting the change in total


pressure as a function of time. The reaction is :
4PH3(g) -t P 4 (g) + 6H2(g).
The following measurement were made on the system containing only PH 3 initially.
Time (sec) 0 50 100
P (total) (mm of Hg) 200 299 332
Show that the order of the reaction is one. Calculate the rate constant.
Solo. : By the problem, Jet phosphine decomposed be 4x and the reaction be-
4PH 3
4a - 4x
Thus, the total pressure is 4a + 3x.
So, 4a = 200
4a + 3x at 50th second is 299.
3x = 99.
x = 33.
Similarly, at IOOth second 4a + 3x = 332.
.. 3x = 132
.. x = 44.
At 50th second,
PpH is 4a - 4x
3
= 200 - 132 = 68 mm of Hg.
At IOOth second,
P PHJ is 4a - 4x = 24 mm to Hg
k = -1I l na-x
a
--

= !1n 4a
4a-4x

= _!_In 200 = 0.0215 sec- 1•


50 68
346 PROBLEMS ON PHYSICAL CHEMISTRY

200
k 100 = 100 ln-
24
= 0.0212 sec 1.
k is constant. So, the reaction is of 1st order and kaverage = 0.0213 sec- 1•
34. The decomposition of phosphine at 950 K is following by noting the change in
total pressure as a function of time. The reaction is :
4PH3(g) ~ P4(g) + 6H 2(g).
The following measurements were made on a system containing PH3 initially.
Time (sec) 0 40 80
P (total) (mm of Hg) 100 150 166.7
Show that the order of the reaction is one. Calculate the rate constant.
Hints : See Problem 33. [Ans. k = 0.0275 sec- 1)
35. The second order rate constant fo.r the neutralisation of 2-nitroporpane by oH-
ions in aqueous solution at T °K is given by the expression,

IoglOk 3163.0
= - --T- + 11.90

k is expressed in litre.mole- 1.min- 1. Calculate the energy of activation, and the half-
life period at 20 °C, when the initial concentrations of base and acid are each 0.008 mole.
litre- 1. [C.U. 19971
Solo. : By the problem,
3163
Iog 10 k2 =- - - + 11.90 at 20 °C,
293.15
where k2 is specific rate constant at 20 °C.
.. log k2 = 1.110301893
.. k2 = 12.89 litre.mole- 1.min- 1
·3163
and log k 1 =-
273
.JS + 11.90 at 0 °C,
where k 1 is specific rate constant at 0 °C.
. . log k 1 = 0.32028196.
.. k1 = 2.09 Iit.moJe- 1.min- 1.
20
x
2.303x 8.314 273.15 x 293.15
Ea = 60569.05 J = 60.569 kJ,
l l
t 11 at 20 °C = - = = 9.69 min.
2 k.a 12.89litre.mol- 1 .min- 1 x0.008mol.litre- 1
36. In a particular reaction, the time required to complete half of the reaction was
found to increase nine times when the initial concentration of the reactants was reduced
to one-third. What is the order of the reaction? Mention the unit of the related rate constant.
(C.U. 1996)
CHEMICAL KINETICS 347

Solo. : We know that

log a2 + log!!_
a1 t2
n = t
log i
. t2

t 1 is the half-life period when initial concentration is a 1, and t2 is the half-life period
when the initial concentration is a 2 .
By the problem,

a1 t1
log--+log- l l
3a1 9t 1 log-+log-
n = ----'----~
t
= __
3 -i --2. = 1.5.
log-1- lob9
9t1

.. the unit of the rate constant is Ii(112.mole-112 .min-I.


37. A certain reaction is 20% completed in 15 minutes at 25 °C. But for the same
extent of reaction it takes only 5 minutes at 35 °C. What is the activation energy of the
reaction? - [C.U. 1995]
Solo. : The change of temperature from 25 °C to 35 °C increases the rate 3 times.
Hence, by standard formula,
Ea x 10
In 3 = 8.314x 308.15 x 298.15
E0 = 83.92 kJ,moJ- 1•
, 38. A certain first order reaction is 20% complete in 15 minutes at 27 °C but for the
same extent of reaction it takes 5 minutes at 37 °C. What is the activation energy of the
reaction? [C.U. 2000]
Hints : See Problem 37. [Ans. 85.03 kJ.mole-l]
39. A gas decomposes according to second order kinetics. When the initial pressure
of the gas is 500 torr, 40% decomposition occurs in 30 min. Find out the time required
for 75% decomposition of the gas and the value of the rate constant. [C.U. 1999]
Solo. : We know that the specific rate (k) for 2nd order is given by

k = ~. x .
t a(a-x)

Here t is the time when the amount of decomposition is x and a is the initial amount.
By the problem,

k = __!_, 0.4a l 0.4a


=-X
30 ax(a-0.4a) 30 ax0.6a

0.4
k =- x torc 1.min- 1 = 4.44 x 10-5 torr-I.min- 1•
30 0.6 x 500
348 PROBLEMS ON PHYSICAL CHEMISTRY

0.75a
•• 131
4
=:
k ax 0.25a
=
3
=- ---
4.44xl0-5
x - -
500
= 135. 135 min = 135.14 min.
40. At 1100 K the following data were obtained on the homogeneous reaction,
2NO(g) + 2Hi(g) ~ N2(g) + 2Hz0(g)
[NO] [Hz] Rate, r
(mol.dm-3) (mol.dm-3) (mol.dm-3.sec- 1)
(!) 5.0 x 10-3 2.5 x 10-3 3.0 X 10-S
(2) 15.0 x 10-3 2.5 x 10-3 9.0 X 10-S
(3) 15.0 x 10-3 !0.0 x 10-3 36.0 X Jo-S
(i) Calculate the order of the reaction with respect to NO, with respect to Hz
and the overall reaction order.
(ii) Write the rate law expression for the reaction.
(iii) Calculate the rate constant for the reaction.
(iv) What will be initial rate of the reaction of
[NO] = [H 2 J = 8.0 x 10-3 mole.dm-3? [C.U. 2001)

Solo. : (i) Let the rate equation be


r = k.C~ 0 .C~ 2 .
By the problem, let from the first set of data,
CNo = a = 5 x 10-3 mol.dm- 3 . and CH = b = 2.5 x 10-3 mol.dm- 3 . and rate(r) =
2
3.0 x 10-5 mol.dm- 3.sec-I.
.. r = k. a \b Y. (i)
From the second set,
3r = k(3a]X.bY. (since, now, rate = 9.6 x 10-5 mol.dm- 3.sec- 1) (ii)
Dividing eqn. (ii) by (i),
3 = 3x,
x = l.
From the third set of data,
12r = k.(3a)X.(4b)Y.[since now rate =36x 10-5 mot.dm- 3.sec- 1} (iii)
Dividing eqn. (iii) by (i),
= J'AY.
12
Since, x = I, 12 = 3.4.l'
.. y =I
(ii) Rate law is r =.k.CNo·CH
2
(iii) 3 x 10-5 = k x 5 x io-3 x 2.5 x 10-3
Ir. = 2.4 dm 3.moJ- 1.sec- 1•
(iv) r i
= 2.4.(& x Hr3 = 2.4 x (8 x w-3>2
= J.536 x Jo-4 mole.dm-3.sec-1.
CHEMICAL KINETICS 349

41. A substance decomposes according to a second-order rate law. If the constant is


6.8 x 10-4 lit.mole-I .sec- 1, calculate the half-life of the substance if the initial concentration
is 0.05 mole/lit. [C.U. 2002]
Soln. : We know that for 2nd order reaction,
I
t 11
2
= -ka'
where k is the rate constant and a is the initial cone. and t 11 is the half-life of the reaction.
2
By the problem,
1
t 11 - = 29411.76 sec= 490.19 min= 8.17 hours.
2 - 6. 8 x 10-4 x 0. 05
42. The gas phase reaction, 2Nz05 ~ 4N0 2 + 0 2 has k
-(24.65 kcal.rnol-1/RT)sec-l
= 2.05 x 10 13 exp
(i) Give the values of A and Ea,
(ii) Find k at 0 °C.
(iii) Find r11 at 0 °C. [C.U. 2002]
2
Soln. : (i) By the problem,
-(24.65 kcal.rnol- 1t RT)sec- 1
k = 2.05 x 10 13 e
We know that by Arrhenius equation,
-Ea
k =A. e RT.
where A is the frequency factor and Ea is the activation energy.
Comparing above two equations,
A = 2.05 x toI3 sec-I.
Ea = 24650 cal
= 103.136 kJ.
(ii) At 0 °C, i.e., 273.15 K the value of k is :
-24'50
2.05 x 1013 el.987x273.15

= 3.86748 x 10-7 sec-I.


0.693 0.693
(iii) t11 = = . x _ = 1791864.47 sec.
2 k 3 86748 10 7
43. A third order reaction is 50% completed in 100 sec. Calculate the time for 75%
and I 00% completion. [B.U. 2000]
Soln. : We know that for a third order reaction,
l l
--~2 = 2 + 2kt, (i)
(a-x) a
where a is initial concentration and k is the rate constant and x is the amount spent in
time t.
350 PROBLEMS ON PHYSICAL CHEMISTRY

3
By the problem, putting in eqn. (i), x = 0.75a, k = a ,
200 2

( o.~5aY - ~2 = Co~a 2 } ·

1-0.25 2
3 1-0.25 2 3
or,
(0.25a)2
= - - 2t
100a
or,
0.25 2
= -100t .
15x100
.. t = 3
= 500 sec.
A 3rd order reaction is never complete, a can be 0, i.e., reaction is 100% complete
only at t = oo.
44. Consider the parallel reaction.
k1
~---B

A "- k2
•c
In an experiment it was observed that 60% decomposition of A takes place in 20
min and analysis of product showed that 75% of B and 25% of C are present. Calculate
k1 and k1 • [B.U. 2000)
k1
,,------'--- B
Solo. : •c

By the problem, !::i = 3.


k2
kl = 3k2.
We know that for parallel reaction,
I a
- I n - - = (k 1 + k2).
t a-x
By the problem,
I a
-In
20 a-0.6a
= 3k2 + k2
I I
or, -In -
20 0.4
= 4k,.-
k2 = 0.4H14 min-I.
k1 = 3k2 = 3 x 0.0114 = 0.0342 min-I.
45. The first order reaction 2A --7 28 + C is 35% complete after 325 seconds. How
long will it take for 70% completion? [B.U. 1998)
Solo. : We know that for first order reaction,
I a
k = - In--.
t a-x
CHEMICAL KINETICS 351

By the problem-::-

k = -!l n - -a- -
325 a-0.35a

= - 1-In 1
- - = 1.325 x 10- 3 sec- 1.
325 0.65
The time required for 70% completion is

= ln-a-
1.325 x 10-3 0. 3a
= 908.659 sec.
46. A I st order reaction is 20% complete in I 0 min at 25 °C and 40% complete in
12 min at 40 °C. Calculate the energy of activation for the reaction. Find also the value
of k at a very high termperature.
Soln.: The reaction is 20% completed in 10 min at 298.15 K. The first order rate

coefficient in k =~ In _a_. x = 0.2a. Thus a -- x = 0.8a.


t a-x
1
k = _!_ In - - = 0.02231 min- 1.
10 .0.8
Similarly, at 313.15 °K,
x = 0.4a at 12 minutes

k = _!_ln-1- = 0.0426 min- 1


12 0.6

In 0.0426 = Ea . 15
0.02231 R 313.15x298.15
E0 = 33472.56 J = 33.471 k,J.
=59._
k=AeRT.
At a very high temperature k~A.

47. At 400 K, the half-life period for the decomposition of a sample of gaseous
compound initially at 55.5 kPa was 340 sec. When the pressure was 28.9 kPa the half-
life period was 178 sec. Determine the order of the reaction. [V.U. 2004)
Soln. : We know that, the order

log !L + log !1
t2 a1
n=
log az
a1

where t 1 is half-life when initial amount is a 1, and t2 is half-life when the initial amount
is Gi·
By the problem,
a1 = 55.5 kPa t1 = 340 sec
a2 = 28.9 kPa t2 = 178 sec
352 PROBLEMS ON PHYSICAL CHEMISTRY

340 28.9
log--+ log--
178 55.5
n = 28.9
1og--
55.5

=
0. 28 I05 - 0. 28339 = 0. 00234 = 8.24 x 10-3 "" 0.
-0.28339 0.28339
48. Calculate activation energy (in SI) for a reaction whose rate coefficient, at 27 °C
is doubled by 10 °C rise in temperature. (R = 8.314 J.K- 1.mole- 1) [V.U. 2003]
Hints : See Problem 37.
49. A solution containing equal concentration of ethyl acetate and NaOH is 25%
saponified in 5 minutes. What will be the % of saponification after IO minutes? [V.U. 2002)
Solo. : We know that for a 2nd order reaction with a single reactant or two reactants
with same initial concentration,

k = !. x ,
t a(a- x)
where a is the initial concentration and x is the amount spent at time t, and k is specific
rate constant.
k = 1 0.25a l
= -x-l
= l5a
5 0.75ax a 5 3a
After 10 minutes,
x
k = IO a(a-x)'
x
=
l5a IO (a-x)a
x IO
~
a-x
= 15
a-x 15
~
x
= -IO
~
a
x
1 = -23
a 5
~
x
= 2
x 2
~
a
=5
So, the percantage of reactant converted is 40%.
50. Rate constants of a first order decomposition are 1.37 x 10-5 sec- 1 and 5.15 x
10-5 sec- 1 at 25 °C and 37 °C respectively. Calculate the activation energy and the entropy
of activation for the reaction. [V.U. 2001)
Solo. : We know that,

lnkz=Eu
k1 R
·[Tz-T1],
T T 1 2
CHEMICAL KINETICS 353

where k2 and k 1 are specific rates at temperature T2 and T 1 and Ea is the activation energy,
R being gas constant.
By the problem,
k2 = 5.15 x 10-5sec- 1 at 310.15 K
k1 = 1.37 x 10-5sec- 1 at 298.15 K.
5
ln5.15x10- =_s_x 12
·· l.37xto-5 8.314 310.15x298.15
5.15 x 10-5
298.l5x3!0.l5x8.314xln 5
Ea= ~~~~~~~~~--'l~.3~7~X:...:..:..;10~--
12
= 84836.815 J = 84.836 k.J.
For I st order reaction,

rate constant, k = e RT e ~ 1Re-Ea 1RT


NAh
putting k = 5.15 x 10--5 sec-- 1 R = 8.314 J.K-- 1.mol- 1
T = 310.15 K NA = 6.022 x 1023 mol-- 1
h = 6.626 x 10--34 J.s Ea = 84836.815 J

entropy of a~tivation, Af~ = -62.11 J.K--1.


51. What would be AV"" per mole of reaedon for which the rate constant increases
by a factor of 1.5 for an increase of pressure from I to 3000 atm at 300 K.
Soln. : By equation 2.19.3, Physical Chemistry, Vol. 2 by Dr S. Pahari,

AV+ = (()AG+) .
()p T

It can be shown from the equation that

dlnk2 ) = _ 6V+.
. ( dP T RT
k1(P2) 6V+
.. In k1(P1) = - RT (P2 - P,).

where k2 and k 1 are srecjfa: rate constants.


By the problc111.
P1 = 1 atm
P2 = 3000 atm.
Av*=_ lnl.5x0.082061.atrn.K- 1.mole- 1x300K
2999 atrn
= -3.328 x10--3 lit/mole.
52. The reaction 2N0 2 + F 2 ~2N0 2 F is 1st order with respect to F 2 and N0 2, where
dC
__!j_ = kCF CNO • k = 3.40 x 104 cc.mole- 1.sec- 1 at 25 °C. If F2 initially at 2 mm of
dt 2 2
Hg reacts with N0 2 initially at 5 mm of Hg in rigid vessel at 25 °C, then what will be

Prob. Phy. Chem.---23


354. PROBLEMS ON PHYSICAL CHEMISTRY

their pressures and the pressure of N02 F after 30 sec?


The reaction is 2nd order with different initial concentrations of reactants.
Solo. : 2N0 2 + F 2 ~ 2N0 2F Temp~rature, T = 273 + 25 = 298 °K
b-2.x a-x 2x

Thus, - 1-tn a(b- 2 x) =kt.


b-a b(a-x)

or,
3.40x10 4 cc. mole- 1 .sec- 1 x 30sec x 10-3 I.cc-I = _1_ 1n a(b-2x).
7600 mmof!-fg x0.08211-atm.deg- 1.mole- 1 x298°K b-a b(a-x)
atm
Now, b = 5 mm of Hg, a = 2 mm of Hg
2(5-2x) 3x3.40xl0 4 x!0- 3 x30
or, In = · ~ 0.16457
5(2-x) 760x0.0821x298
2
or,
2
((- ~) = 1.17888 or,
5
-
2
x = 2.94 72.
52-x 2-x
Hence, x = 0.94 mm of Hg.
Thus, after 30 sec, pressures of N0 2, F 2 and N0 2F are 3.12 mm of Hg, 1.06 mm
of Hg and 1.88 mm of Hg respectively:
53. At 20 °C, two gases A and 8 2 r~act as follows to give 2A + B 2 ~ 2AB(g). The
rate equation is given by r = kCACB with k = 3.24 xI0 2 cc.mole- 1.sec- 1 at 20 °C. The
initial pressure of A is 25 mm of Hg and that of B is IO mm of Hg. What will be the
pressure of A, 8 2 and AB after IO minutes?
Hints : Proceed as Problem 52. [Ans. 15.67, 5.34, 9.33]
54. D,ecomposition of N 20 5 , a first order reaction, was studied at few temperatures
and the results were :
T"C 25 35 45 55 65
k xi OS sec- 1 1.72. 6.65 24.95 75 240
Calculate frequency facter, activation energy and

L\GJ, MIJ, L\U(,, M! at 45 °C

Hints : In (kz)Tz =Ea (Tz -Ti)


(k2)r1 R T1T 2
For I st order reaction,

k,,
-
= ekT
-
h
e
e.sJ!R -Ea/RT --Ae-Ea!RT_
e

Again, In k2 = In A- E"
RT
L\H*o = L\U*o = Ea - RT L\G+o = L\H*o - 7T Af*o

A = 2.27 x 10 13sec 1, Ea = 103.24 kJ, Ml~ = 100.59 kJ, L\U~ = 100.59 kJ,

Af~ = 1.91 J.k-1, L\G(, = 99.98 kj.·


CHAPTER10

PHOTOCHEMISTRY

Required Formulae

I. log 10 = D, where Dis extinction or absorbance or optical density a~d 10 is the


Ir
intensity of the incident radiation and 11 is the intensity of tn1nsmitted radiation.
Again, D = FC.l,
where E is molar absorption or extinction coefficient or molar absorptivity, l is the path
length and C is concentration.
Optical desnity (D) is additive.
n
.. D = LD;.
i=I

2. Quantum efficiency (¢)


Number of molecules reacting
= Number of quanta absorbed
Number of moles reacting
= Number of einstein absorbed
Number of moles reacting per unit time
=
Number of einstein absorbed per unit time
Rate
=---
Intensity
If the reaction is : v1A ~ P
and 10 is the intensity in einstein of photon absorbed per unit time,

¢=

355
356 PROBLEMS ON PHYSICAL CHEMISTRY

3. The energy corresponds to one einstein of photon is

E -- N·A h v -- NAhc
A ,
where· NA is the Avogadro number, c is the velocity of light and A is the wavelength of
the radiation.
E = 6. 022 x 10 23 x 6. 626 x 10-34 joule. sec x 2. 9979 x 10 8 m I see
·· Ax io- 10 m
where. A is expressed in A.
9 6 5
E = 1.1962 x 10 J = 1.1962 x 10 kJ = 2. 859 x 10 k.cal.
A. A. A.
4. C is concentration of a solution in molarity C(M).
m is concentration in molality.
Theoretical Problems

1. Is the Einstein's law of photochemical equivalence always valid?


Ans. : Einstein's law of photochemical equivalence as stated for any photochemical
process is otten violated if there are secondary processes along with the primary process.
Even for primary processes the law may be invalid for laser beam. Laser beams contain
a very high density of photons, and a substance may absorb more than one photon.
Sometimes as reported by E.A. Orgryzlo in 1965, a single photon can excite two molecules
in contact with it. Thus, the Einstein's law is not always valid.
2. What are the factors on which the optical density of a solution depends?
Ans. : The optical density of a substance primarily depends on the wavelength of
the radiation absorbed, nature of the light absorbing substance, concentration of the light
=
absorbing substance and the length of the cell containing the solution (/) E.C./.). It also
depends on the medium or solvent. Normally it does not depend on temperature.
3. Will the quantum yield be greater than one for any chain reaction?
Ans. : Generally the quantum efficiency is different from 1 if there are secondary
processes. If the reaction takes place by a chain mechanism, then the primary process is
the chain initiating step and the secondary processes are chain propagating (the last step
being chain terminating). For a laxge number of chain reactions, <I> is grater than I. A classic
example is the photochemical combination of 1-k and Clz (<I> "" 106 ). However, if a chain
propagation step is slow (and endothermic) and requires high activation enegy, <)l may
be less than 1. A classic example is H2 and Br2 reaction for which Br+ H2 ~ HBr + H
is a step requiring 75 kJ/.mole as Eact· There is also a chain inhibiting step and <)l is about
0.01. Therefore, for high quantum yield, the secondary processes should be fast.
4. Is it absolutely essential for light to be absorbed directly by a molecule A2 for
its photochemical decomposition?
Ans. : The Grothuss-Draper law states that only those radiations that are absorbed
by the reaction system are effective in producing a chemical change.
This law is apparently obvious since it appears that reaction by radiation cannot take
place in absence of light. However, if by the system we only mean the reactant (which
we normally do), then the absorption of radiation by A2 is not a must. Let there be a
substance B present which can absorb radiation, and Az cannot. Then B is excited to B*,
PHOTOCHEMISTRY 357
and it collides with A2, as a result of which the energy of excitation is converted to thermal
energy and A2 reacts. Thus, it is not absolutely essential that A2 has to absorb radiation
for photochemical reaction of A2. The above is an example of photosensitization. Howeve~
the system considered above is a mixture of Az and B, and the system absorbs radiation.
5. What are the exceptions to Beer's law?
Ans. : Beer's law has a number of exceptions. It is generally found that at high
concentrations the substance absorbs more readily than the equation requires. The Jaw also
fails when coloured substances ionise, dissociate, or associate in solution and in highly
intense light such as laser beam that contains latge density of photons and induces multi-
photon processes.
6. "Thermodynamically non-spontaneous reaction may also take place spontaneously
in presence of light"-Explain.
Ans. : A reaction A ---1 B is said to be spontaneous if the equilibrium lies toward
B, i.e., at equilibrium there is a very high concentration B. Normally, in such cases the
free energy of the reaction defined by 1':!.G =GB - GA is negative. Thus. if the free energy
of A is greater than B, then a substantial amount of A is converted spontaneously into
B. Major portion of B will not be converted into A, since GA - GB is positive (However,
pure B will be converted into a mixture of A and B, since GJ - G; is always negative).
However, let us assume that B can absorb a particular radiation while A cannot and let
tl\at radiation pass through the system. B then reaches an excited state of higher free eneigy,
and now GA - GB is negative and at equilibrium a higher proportion ofA may be present.
This finding is normally said to be the occurence of an apparently non-spontaneous reaction.
But in reality the spontaneity is judged by 1':!.G = Gfinal - Ginitiat. and that is always negative.
7. How can you explain the decrease in rate with temperature observed in the
photochemical chlorination of benzene and some other reactions of this type?
Ans. : The effect of temperature on rate is given by the temperature coeficient of
the specific rate of the reaction. For the chlorination of benzene or such other reactions,
the specific rate k of the final rate equation may be expressed as k = k' K, where k' is
a single specific rate or a product of several specific rates for the given multi-step
photochemical reaction and K is the equilibrium constant involved in the formation of some

o f the .mterme d"iates or pro d uct. Now -


dlnk
- = -E-2 ,
0
where Ea is the activation energy
dT RT

of the reaction. Hence, Ea = E' + tJl0 , where d In K =


dT RT
tJI;and l':!.H 0 is the heat change

in the reversible step. E' is the sum of the activation energy of the composite steps. Now,
if E' + l':!.H 0 is positive, Ea is positive, and the rate will increase with temperature. If
E' + l':!.H 0 = 0, the rate will be independent of temperature, and if E' + 1':!.H0 is negative,
the rate will decrease with increase in temperature. Therefore, the above mentioned reaction
must be such that E' + tJl0 is negative. This can easily happen if l':!.H 0 refers to an exothermic
heat change (-ve) and the magnitude is greater than +E'. Therefore, if there is a reversible
step, as is normally assumed, then it is a highly exothermic one. {$0 can be negative also,

if !:i
= K, where k1 and k2 are single or product specific rates so that Ea = E1 - E2
k2
and E2 > E1).
8. Let the energy of radiation sent through a system be slightly greater than the bond
energy of A-A in the system. Will there be absorption always?
358 PROBLEMS ON PHYSICAL CHEMISTRY

Ans. : A photochemical reaction can take place if the bond of the type A-A or
A-B breaks as a result of passage of light. Thus, the electron of the system should be
excited from the ground electronic state to the highest vibrational level of at least the 1st
excited electronic state. Therefore, it may appear that if the eneigy of radiation sent to
a system slightly greater than the bond eneigy, then absorption should take place.
However, the ground electronic.state of a majority of stable systems are singlet and
the corresponding 1st excited electronic state is triplet. And singlet ~ triplet electronic
transitions are forbidden for systems like A-A. Naturally in most of those cases, there will
be no absorption.
9. What is fluorescence?
Ans: : Fluorescence is the radiative transition between two states of same multiplicity
The process may be such that frequency of transmittance radiation may be equal to or
less than that of incident radiation. Generally, tfle frequency of the transmitted radiation
is less and it takes place from the ground vibrational level of the I st excited singlet state
to the ground singlet state.
10. What is phosphorescence?
Ans. : Phosphorescence is the radiative transition between two states of diferent
multiplicity. In this case the frequency of the emitted radiati0 •. is always less than the
frequency of absorbed radiation.
11. Give an example of a photochemical reaction •;,hich may be used to measure the
intensity of the radiation.
Ans. : A photochemical reaction of kmw:~, quantum yield may be used in actinometers
to measure the intensity of radi>ltion. A common reaction is decomposition of oxalic acid
sensitized by u0:2++. A mixture of oxalic atid (about 0.04M) and U~S04 (O.OIM) 1s
taken. uo2++ absor•:.; radiation in the region 250 to 440 nm and the reaction is :

U02++ ~ (uo~+ r
(uo;+ f + (COOH)2 -tU02++ + C02 + co + H20
The quantum efficiency is 0.57. The amount of oxalic acid decomposed is determined
by the titration of standard KMn~ and hence the amount of radiation absorbed 1s
ascertained.
12. Discuss with an example what is meant by photostationary equilibrium.
Ans. : The photostationary state is a steady state which arises if the forward or backward
or both reactions are affected by the radiation. In that case, the thermal equilibrium state
may be shifted to another minimum which remains the same as long as radiation is not
cut off. One of the well-known examples of photostationary equilibrium is the dimerisation
of Anthracene (A).
Anthracene absorbs radiation to give the dimer and hence, the equilibrium state between
A and A2 is shifted more towards A2 in comparison to thermal conversion.
Numerical Problems
1. k in the expression I = /0 e-k.IC is 6.63 for light of 3 I 30 A for acetone vapour
C is the concentration in moles/litre, and/, the cell length in cm. Calculate the percentage
of light absorbed by acetone vapour at 100 mm pressure at 50°C, when a beam of light
is passed through a cell 15 cm long. [C.U. 1967) ·
PHOTOCHEMISTRY 359

Solo. : By the problem,


k = 6.63
JOO
p -atm
c ::: = 760
RT 0.08206lit.atm. mole- 1K-I x 323.15K
100
= 760x 0.08206x 323.15 moles/lit

= 15 cm.
I
In-= -k.IC
(,
= -6.63 x 15 cm x _____JOO
_ _ __ moles/lit = -0.493464.
760 x 0. 08206 x 323.15
I
= 0.6105079 = 0.61.
I()
Hence, the percentage of light transmitted is 61 %. Therefore, % of light absorbed
is 39%.
2. A 0.003 (M) solution of a coloured substance transmits 75% of the incident light
of 500 mm, when piaced in a cell of length I cm. Calculate the molar extinction coeficient,
and hen•e, the optical density of a O.OOl(M) solution in the same cell at Che same wavelength.
[C.U. 1974] .
Solo. : By the problem,

.!J._ = 0.75; C = 0.003 M; I = I cm.


lo
We know that,
I
log ....L = - E.C.l.
lo
I 1
.. E = --Iog-
C/ 1
1

= 0.003(M)x !cm
x log 0.75

= 41.646245 cm- 1.(M- 1) "" 41.6 cm-1.(M-1).


. . optical density (D)

= E.C.l
= 41.6 cm- 1.(M- 1) x 0.001 (M) x 1 cm = 0.0416.
3. A l o- 3 (M) solution of A also contains some B, and the solution
when placed in
a 2 cm cell absorbs 80% of the incident light at a certain wavelength. If the extinction
coefficients of A and B at this wavelength are 250 and 1000 respectively, find the
concentration of B in the solution. [C.U. 1967]
Solo. : By the problem,
I 20
tog....!... =Jog-.
/0 100
360 PROBLEMS ON PHYSICAL CHEMISTRY

I
D = log _!!... = log 5 = 0.69897.
II
But D = D 1 + D 2.
Again, D 1 = E1C 1/ 1
= 250 cm- 1.(M- 1) x 10-3 (M) x 2 cm
= 0.5.
D2 = E2C2l2
= 1000 cm- 1.(M- 1) x C2 x 2 cm
= 2000 C2 (M- 1).
. . 2000 C2 (M- 1) + 0.5 = 0.69897.
C2 0.19897 (M) = 9.9485 x 10-5 (M).
=
2000
. . the concentration of B is 9.9 x 10-5 (M).
4. A 1 x I 0-3 M solution of a dye X shows an absorbance of 0.20 at 450 nµ and
an absorbance of 0.05 at 620 nµ. A I x 10-4 (M) solution of another dye Y shows 0.00
absorbance at 450 mµ and an absorbance of 0.42 at 620 nµ. Calculate concentration of
each dye present together in a solution which exhibits an absorbance of 0.38 and and 0.71
at 450 mµ and 620 mµ respectively. The same cell is used in all measurements and its
thickening is I cm. [C.U. 1979]
Soln. : By the problem,
Dye X Dye Y
C = 10-3 M C = 10-4 M
D450 0.2 0
D620 0.05 0.42
mixture of D450 D620
X and Y 0.38 0.71
.. D45o
x = 0.2 = e!soc.1.

= 10-3 .(M) x E450X


x !cm
.. E450
x = 200 cm- 1.(M- 1) .
Again, D.~ 20 = 0.05 = E620
x x Cl
E620 x 10- 3 .(M) x
= x I cm.
.. E620
x = 50 cm- 1.(M- 1)
D450
y = 0, E450 = 0)'

0620
j' = 0.42 = e 620
y x Cl = E620
y x 10-4 (M) x 1 cm.
.. E620
y = 4200 cm- 1.(M- 1) .
Now, in the mixture at 450 mµ,

D!~~ture = 0.38 Cx x 1 + E~
50
= e!50 x x Cy x I
= 200 cm- 1.(M- 1) x Cx x 1 cm + 0 x Cy x 1 = 200 Cx (M- 1).
PHOTOCHEMISTRY 361

= O. 38 = 1.9 x 10-3 (M).


200
2 2
= 0.71 = E~ °Cxl + E~ °Cyl
620
And Dmixture
=50 cm- 1.(M- 1) x 1.9 x l0-3(M) x 1cm+4200 cm- 1.(M- 1). Cy x 1 cm
= 50 x 1.9 x 10-3 + 4200 Cy (M- 1)
or, 0.71 = 0.095 + 4200 Cy (M- 1)
or, 0.615 = 4200 Cy (M- 1)
or, Cy = 0.615 (M)
4200
= 1.46428 x l o-4 (M)
= 1.5 x lo-4 (M).
5. The percentage of transmittance of an aq. solution of a dye.at 450 rrµ and at 25 °C
is 30% for a 2 x 10- 3 (M) solution in a 2 cm cell. Calcufate the optical density and the
molar extinction coefficient. Find the cone. of the same dye in another solution where
the percentage transmittancy is 20% in a l cm cell at the same temperature and same
wavelength of light. [C.U. 1980]
Solo. : By the problem,
ii_ = 30
10 100

1- = 100
11 30
I 100
.. D =Jog~ =log- = 0.5228787 = 0.52.
11 30
Now, D = E.C.l
E = ClD =2x-10-
0.5228787
-3- - - -
(M)x 2cm
= 130.71961 cm-'.(M-1)
= 130.7 cm- 1.(M- 1).

Now, log .!J... == -E. C. l.


lo

.. C = __!_ log !J..


El /0
20
= - 130.7cm-1.(M-1).1cm log-
100

= 5.348 >< 10-3 (M).


6. In a cell of a certain length, and at a pressure of l 00 mm of Hg, gaseous acetone
transmits 25. t % of the incident light of wavelength 2650 A.Assuming Beer's law to apply,
calculate the pressure at which 98% of the incident radiation will be absorbed by gaseous
acetone (obeying ideal gas laws) in the same cell at the same temperature. [C.U. 1987)
362 PROBLEMS ON PHYSICAL CHEMISTRY

I
Solo. : By the problem, .....L
. lo
= 10-ECI
I
or, log .....L = -€Cl
Jo

or, log 0.25 I = - Ex (_!QQ_)


760RT
x I.

:. € = - ~( 76 0RT)x
log 0.251.
I IOO
Again, when 98% is absorbed, 2% is transmitted.

I (760RT)
:. log 0.02 = --. - - x log 0.251 x C.l.
l IOO
760RT) p
or, log 0.02 = - ( - - x log 0.25 I x
IOO 760RT
. . the required pressure in mm of Hg is

p = -100 x log0.02 = 283.00777 mm of Hg= 283 mm of Hg.


logo. 251
7. Monochromatic light is passed through a I mm path length cell containing 0.005
moles/dm3 solution. The light intensity is reduced to 16% of its value. Calculate the molar
extinction coefficient of the sample. What would be the transmittance if the cell path is
2 mm? [C.U. 1987)

I IOO
Solo. : By the problem, log-2..= log- = D = €.C.I.
II 16
Here C = 0.005 moles/dm3 .
=I mm = 0.1 cm
l IOO
€ = D =
Cl
ogJ6
0.005 moles I dm 3 x 0.1 cm
= 1591 lit/mole.cm- 1•

If the path length is 2 mm, i.e., 0.2 cm, then

log~ = 1591 lit.mol- 1.cm- 1 x 0.005 mole/lit x 0.2 cm= 1.591.


II

log IOO = l.59 l.


I,
11 = 2.56.
8. A 0.01 molar solution of a compound transmits 2~% ofNa-D line when the absorbing
path length is I .5 cm. What is the molar extinction coetficient of the substance? Solvent
is assumed to be completely transparent. [C.U. 1989]

Hints : D = log IOO


20
log I 00/20
E = = 46.6 (M-1).cm-1•
O.Olxl.5
PH01DCHEMISTRY 363
9. An aqueous solution of a compound A of concentration 10-3 moles/litre absorbs
50% of incident radiation in a cell length 1 cm and another compoundB of concentration
2 x 10-3 moles/litre absorbs 60% of the incident radiation at a particular wavelength.
Calculate the percentage absorbed by a solution containing X)- 3 moles/litre of A and B
each in the same cell at the same wavelength. [V.U. 1994]
Soln. : For comp0und A, 50% light is absorbed.
100
D = log-- = EA'C/.
0.50
logl/0.50
EA = 10- 3 moles I lit xi cm [by the problem).

EA = 301.02 lit. mole-I .cm-I.


For compound B, 60% light is absorbed, i.e., 40% is transmitted.

D = log l~i = EB'CL

log!00/40
·· ·EB= 2xl0-3 moles/litxl cm [by the problem]

= 198.97 lit.mole- 1.cm-1.


So, for solution containing 10-3 moles.litre- 1 of A and B,
D = (EA + E 8) x Cl
= 499.99 lit.mole- 1.cm- 1 x 10- 3 mole.lic 1 x I cm
= 0.49999.
I
log ....Q. = 0.49999.
II
100
log- = 0.49999.
JI
11 = 31.64 .
.. (100 - 31.64)% = 68.36% light is absorbed.
10. At 460 nm a blue filter transmits 75% of the light and a yellow filter transmits
40% of the light. What is the transmittance at the same wavelengths of the two filters
in combination? [V.U. 1996]
Solo. : By the problem,

D of blue filte~ = log lOO = 0.125


75

D of yellow filter = log lOO = 0.398.


40
As optical density is an additive property, so optical density of combination of blue
and yellow filter. is (0.125 + 0.398) = 0.52.3.
100
log-= 0.523.
II
= 29.99.
So, the transmittance is 29.99%.
364 PROBLEMS ON PHYSICAL CHEMISTRY

11. A 2 x 10-4 (M) solution of a dye when placed in a cell of 1.0 cm path-length
transmits 10% of the 460 nm radiation incident on it. Calculate molar extinction coeficient.
If the same solution be placed in a 5 cm cell, what would be the absorbance? Will the
molar absorption coefficient change if the measurement be made with (i) 5 x 10- 4 (M)
... <lution in 2 cm cell with 460 nm (ii) 2 x 10- 4(M) solution in 1 cm cell with 500 nm?
[C.U. 1997]
Hints: See similar problem. [Ans. 5000 l.mol- 1.cm- 1, 5]
E depends on nature of substance and wavelength. So, it will not change in case
(i), but will change in case (ii).
12. Tryptophen residues in proteins absorb light with molar extinction coeficient of
E = 54 x Ia5 cm2 .rnole- 1. If a protein sample of 5 x 10-5 mole.litre- 1 shows absorbance
A = 0.54 for I cm path-length. Calculate tryptophen residues in the sample. [B.U. 1996]
Hints : Calculate cone. by using Lambert Beer's law.
13. A compound absorbs U.V. radiati<;m at wavelength 3000A. What would be the
total energy in kilocalories absorbed by I mole of this compound when i1Tadiated with
this radiation? (I cal = 4.18 x 107 erg; h = 6.625 x 10-27 erg-sec)
Soln. : We know that,
E = NAhc
')..

6. 022 x 10 23 x 6. 625 x 10-27 erg. sec x 2. 9979 x 10 10 cm I sec


= 3000 x I 8 cm o-
12
= 3.9867 x 10 erg
= 95375.6 cal = 95.376 kcal.
14. A gas, when exposed to radiation of')...= 3310 Aundergoes decomposition, and
per kilocalories of light energy absorbed, 0.0230 moles of the gas is decomposed. Calculate
the quantum efficiency. [C.U. 1976]
I cal = 4.18 x I 0 7 erg; h = 6.62 x 10-27 erg-sec
Soln. : Energy of one einstein of 3310 A is

E = NAhc
/...
6. 022 x 10 23 x 6.62 x 10-27 erg. sec x 2. 9979 x 1010 cm I sec
=
3310x 10-8 cm
= 86.379 kcal = I einstein.
I kcal = 0.01157 cinstein.
quantum yield (I\>)
No. of moles of gas decomposed 0. 023
= No.of einsteinabsorbed
= 0.01157
= 1.987899 "' 1.99.
15. The photochcmicaf dissociation of gaseous HI to form normal ~ and 12 requires
radiation of 4040 A or less.
(i) Determine the molar heat of dissociation of HI.
(ii) If radiation of 2537 A is used, how much eneigy win appear as kinetic energy
of the atoms?
PHOTOCHEMISTRY 365

NAhc
Solo. : (i) Now, E = ')...

6.022x10 23 x6.625x10- 34 J.secx 2.997x10 8 cm I sec


= 4040x 10 10 m
= 296048.19 J
= 296.048 kJ
= 70.572 kcal.
(ii) Now for 2537 A,
EF = 471436.61 J
= 471.43661 kJ
= 112.6761 kcal.
E-EF = (112.6761 - 70.7572) kcal
= 41.919 kcal energy will appear as K.E. of the atom.
16. The total amount of radiation of wavelength 4200 A falling over a certain time on
a solution was calculated from radiation intensity to be 2.5 cal. In this time l .8x 10-5 moles
of the reactant was decomposed. Calculate the quantum eficiency.
Solo. : We know that the energy of l einstein of photon of /... = 4200 A,
23
x 6. 625 x 10-34 J. sec x 2. 9979 x 10 8 m I sec
6. 022 x 10
= 4200 x 10- 10 m
= 284770.1641 J
= 68061.7 cal.
I
. . l calorie = einstein .
68061. 7

.. 2.5 calorie = 2.5 x einstein


68061. 7
= 3.67424 x 10-5.
: • ..t. = 8 10 - 5
l. x = 0.4898 05
'I' 3.67424x10-5 • •

17. The dissociation energy of Hz is 102.9 kcal.mole- 1. If Hz is illuminated with


radiation whose wavelength is 2537 A and is thereby dissociated, how much of the radiant
energy per mole will be converted to kinetic eneigy?

Solo. : Now, E = N ~he = 6.022x10 23 x6.625x10-34 J.sec x 2. 9979x10 8 m I sec


A. 2537 x 10- 10 m x 4.1841.cal- 1
= 112.676 kcal.
Hence, the energy per mole converted to K.E. is
(112.676 - 102.90) kcal = 9.776 kcal.
18. Absorption of each calorie gives 1.44 x 10-5 gm of Hz in the photo-dissociation
of HI vapour using radiation of wavelength 2070A. Calculate the quantum yield.
Solo. : Now, energy of 1 einstein of /... = 2070 A,
E = NAhc
')...
366 PROBLEMS ON PHYSICAL CHEMISTRY

6.022 x 10 23 x 6.625 x 10-34 J. sec x 2. 9979 x 108 m I sec


=
2070x 10- 10 m x 4.184J.ca1- 1
= 138096.21 cal.
l
:. 1 cal = ----
138096.21
einstein.

Now, 2HI = H2 + Ii
1.44x10- 5
¢> =- -- = 1.99.
I
138096.21
Since, 1.44 x 10-5 gm of hydrogen is the amount in gm-atm that should result from
HI per (Photon x NA) unit amount.
19. In the photochemical combination of ffi(g) and Cb(g) a quantum yield of 1 x 106
is obtained with light of wavelength 4800 A. How many moles of HCl(g) would be produced
under these conditions per calorie of light energy absorbed? [C.U. 1982)
Solo. : By the problem,
N Ahc 6. 022 x 10 23 x 6. 625 x 10-34 J. sec x 2. 9979 x I0 8 m I sec
E
= -/...- = 4800x!0- 10 mx4.184J.cal- 1
= 59553.99 cal.
I
:. 1 cal =- - - einstein.
59553.99
Now; H 2 + Cl 2 ~ 2HCI
I
- x number of moles of HCI produced
•• ¢> = -"'2~-----------
number of einstein absorbed
. . number of moles of HCI produced = 2 x ¢> x number of einstein absorbed

=2X 106 x - - - = 33.583 = 33.6.


59553.99
20. An uranyl oxalate actinometer is irradiated for 15 minwes with light of wavelength
4350 A and oxalic acid equivalent to 12.0 cc. of O.OOI(M) KMnQ is found to have been
decomposed. The quantum efficiency of the actinometer at this wavelength is 0.58. Calculate
the average intensity of the light used in (i) erg/sec and (ii) quanta/sec. [C.U. 1983]
Solo. : (i) The reaction is :
2KMn04 + 3H2S04 + 5H2C204 = KzS04 + 2MnS04 + l OCQi + 8H20
Thus, 2 moles of KMn04 react with 5 moles of H2C204 i.e., 1000 cc., 2 molar KMnOi
= 5 moles of HzC204.
Now, 1 cc 1 molar KMn04 = 2.5 x 1o-3 moles of HzC204.
Number of moles ofoxalic acid decomposed is equivalent to 12 cc O.OOl(M) KMnQ
.. number of moles of H2C204 decomposed = 2.5 x 10-3 x 12 x l0- 3 moles
= 3 x 10-5 moles.
numberofmolesofH 2 C 2 0 4 decomposed __ n..
Thus, ...,
number of einstein absorbed
PHOTOCHEMISTRY 367

- 5
.. number of einstein absorbed (NE) = 3x10- = 5.17241 x 10-5.
0.58
(ii) Thus, number of quanta absorbed/sec

= 5.17241X10-S X 6. 022 X 10 23 = 3.4609 X 1016 = 3.5 X t0l6,


900
NA he6. 022 ~ l 0 23 x 6. 625 x 10-27 erg. sec x 2. 9979 x 1O'° cm I sec
Now, E
= -1.-= 4350 x 10-8 cm
= 2.7495 x .1012 erg.
. Ex NE
. . energy absorbed 1s per sec
900
2. 7495x10 12 x5.17241x10- 5
= 900
= 1.58 x 105 erg/sec.

21. Consider the photochemical reaction :

A~B+C
When irradiated with light of wavelength 2537 Aduring a certain period, the light energy
absorbed = 3.436 x 108 erg and the number of moles of B formed = 3.64 x 10-6 mole.
Calculate the quantum yield. - [C.U. 1984]

Soln. : By the problem, energy of one einstein = N ~he


_ 6.022x10 23 x6.625x10-27 erg.sec x 2. 9979x 10 10 cm I sec
- 2537 x 10-8 cm
= 4.71436 x ma erg.
3 436 108
3.436xl08 er = · x einstein = 7.28837 x 10- 5 einstein.
·· g 4.71436x10 12

:. 4> = 7. 3.64~ 10...{)


28837 x 10-
5 = 0.04'J94 "'0.05.

. 22. A 100 cm 3 vessel containing H2 and Cl2 was irradiated with light of 400 nm.
Measurements with a thermopile showed that 11 x 10- 7 J of light energy was absorbed
by the chlorine per second. During an irradiation of I minute the partial pressure of chlorine,
as determined by the absorption of light and the application of Beer's law, decreased from
27.3 to 20.8 kPa (corrected to 0°). Determine the quantum yield.
Solo. : We know, P = CRT
The decrease in pressure of Cl2 is (27.3 - 20.8) kPa = 6.5 kPa
6.5kPa
= 101.325kPa
atm = 0.06415 atm.

Hence, the concentration of Clz consumed is


C 0.06415atm
- 0.08206 lit.atm.mole- 1 .K- 1 x273.15 K
= 2.86195 x 1o- 3 moles/lit.
368 PROBLEMS ON PHYSICAL CHEMISTRY

Since, the vessel is 100 cc, the number of moles consumed is 2.86195. x 10--4.
The light energy absorbed in one minute was 11 x 10-7x 60 J = 6.6 x 10-5 J.
N ow, Energy o f one emstem
. . == -A-
NAhc

6. 022x10 23 x6.625x10-34 J.secx 2.9979x 10 8 m I sec


== 4000X10-IO m
== 299008.67 J [since 400 nm = 4000 AJ.
6.6x 10-5
. . 6.6 x 1o-5 1 = einstein = 2.2072938 x 10-IO einstein.
299008.67

:. qi = 2.86195x 10-4 == 1. 296 x lo'i.


2.2072938Xl0-IO ·
23. The quantum yield is 2 for the photolysis of gaseous HI to H2 and h. by the
light of 253.7 nm wavelength. Calculate the number of moles of HI that will be decomposed
if 300 J of light of this wavelength is absorbed.
. "' _ no. of moles decomposed
I .. 'f' -
Son
no. of einstein absorbed
Now, energy of one einstein is
N Ahc = 6. 022 x 10 23 x 6.625 x 10-34 J.sec x 2. 9979 x 10 8 m I sec
A. 253. 7 x 10-9 m
= 471436.6 I = 4.71436 x 105 J.
300
. . 300 J == einstein
4. 71436 x 105
= 6.3635 x 10-4 einstein.
no. of moles decomposed
By the problem, 2 == . x -4
6 36355 10
:. number of moles decomposed
=2 x 6.3635 x w-4 = i.2121 x 10-3.
24. The incident radiation from a monochromatic source of 2537 A on a slit is 80
ergs/sec/sq. mm. The radiation is passed through a slit whose area is 5 sq. mm and into
a cell whose face transmits 30% of the incident radiation. If the solution in the cell transmits
10% of the radiation, how much energy in calories is absorbed by the solution per hour?
Solo. : The radiation is passed through 5 sq. mm slit and hence, 80 x 5 = 400 ergs/sec
of energy falls on the cell. In one hour, 400 x 60 x 60 ergs energy falls on.
By the problem,
30% falls on the solution and 10% of it is transmitted, i.e., 3% of the original energy
is transmitted. The solution, thus, absorbs (30 - 3) = 27%. i.i:
. . light absorbed,
27
- x 400 x 60 x 60 ergs = 388800 ergs
100
== 9.2925 x 10- 3 cal = 9.3 x 10-3 cal. (since, 4.184 x 107 ergs =I calorie)
PHOTOCHEMISTRY 369

25. In an experiment to measure the quantum efficiency for a photochemical reaction.


the absorbing substance was exposed to 480 nm light from a 150 W source for a period
of 30 minutes. The intensity of the transmitted light was 40% of the intensi~ of the incident
·light. As a result of this, 0.35 mole of the substance decomposed. Calculate <)>.
Solo. : Energy of 1 einstei"n
6. 022 x 10 23 x 6. 625 x 10-34 J. sec x 2. 9979 x 10 8 m I sec
= = 349173.891.
480x 10-9 m
Energy coming out from 150 W source in 30 minutes
J
= 150- x 30 x 60 sec = 270000 J.
sec
270000
.. number of einstein of the incident light = = 1.08358 "" 1.08.
249173.89
.. number of einstein absorbed = 0.6 x 1.08 = 0.65 .
35
• • <1> = O. mole = 0.5384615 = 0.54 mole. einstein- 1.
0. 63 einstein
26. In a photochemical reaction, the stoichiometry for which A ~ i.B+C, the quantum
efficiency with 400 nm light is 2. After exposure of 3x10- 3 moles of A, 2.4 x 10-3 moles
of Bare formed. How many photons were absorbed by A? How much energy was absorbed?
. ,.. _ No.of moles reacting
SoIo .. 't' -
No.of einstein absorbed
Number of mlecules reactiRg
:. number of photons absorbed (N0 ) =

Since 2.4 x 10-3 moles of B are formed, 1.2 x 10- 3 moles of A must have reacted.
1. 2 x 10-3 x 6. 022 x 10 23 - 3. 6 x 10 20
N0 - = 0.598 x I o- 3 einstein
•• - 2 - 6. 022 x 10 23
Now, energy of one einstein
6. 022 x 10 23 x 6. 625 x 10-34 J. sec x 2. 9979 x 10 8 m I sec
= 400x 10-9 m
= 299008.67 J.

. . energy absorbed = 0.598 x 1o- 3 x 299008.67 J = 178.81 J.


27. In three seperate experiments of passage of light through uranyl oxalate actinometer,
the actinometry solution consisted of (a) 5.232 gm, (b) 8o47 gm and (c) 4.852 gm of oxalic
acid respectively in 50 cc of water. After exposure for 4 minutes, the remaining solution
was titrated in these 3 cases seperately with KMn04, and the KMn04 solution required
for complete oxidation of ~he remaining oxalic acid was (a) 18 cc of 0.98(M), (b) 25 cc
of l.ll(M) and (c) 13 cc of 0.88(M), solution respectively. What is the average intensity
of the light used in photons/sec and joules/sec in these 3 cases separately, if, in alt these
cases, Jight of wavelength 4050 A was used. The quantum efficiency is reported as 0.55.
5 232
Solo. : (a). Number of moles of oxalic acid= · = 0.0581333 in 50 cc.
. 90
2KMn04· + 3H2S04 + 5H2C204 = KzS04 + 2MnS04 + 8H20 + 10 C02
2000 cc 1 (M)KMn04 = 5 moles of oxalic acid
.. 1 cc 1 (M)KMn04 = 2.5 x 10- 3 moles of oxalic acid.
370 PROBLEMS ON PHYSlCA°L CHEMISTRY

Thus,
(a) 18 .cc 0.98(M) KMn04
= 18 x 0.98 x 2.5 x 10- 3 moles of oxalic acid
= 0.0441 moles in 50 cc.
Now energy/einstein
NA he 6.022x10 23 x6.625x10- 34 J.sec x 2.9979x10 8 m I sec
= -')..- = 4050 X 10-IO m
= 295317.21 J
= 295.32 kJ.
(a) Oxalic acid decomposed = (0.0581 - 0.0441) moles
= 0.014 moles in 240 sec
0.014
No. of moles decomposed/sec = = 5.8333 X 10-S.
240
No. of moles decomposed I sec
.. </J = Number of einstein absorbed I sec

. . No. of einstein absorbed/sec = No. of moles decomposed I sec

5. 8333 x 10-5
= 0.55
= 1.0606 x 10- 4 = 1.0606 x I o- 4 x 6.022 x I 0 23 photons/sec.
= 6.38693 x 1019 photons/sec.
Energy in J/sec = 1.0606 x 10- 4 x 295317 .21 J
= 31.321343 = 31.3 J/sec.
Similarly, (b) and (c) can be calculated.
[Ans. (b) 1.13 x 10 20, 55.3; (c) l.15 x 10 20 , 56.6]
28. An actinometer contains 20 cc of 0.05 moles/lit oxalic acid through which light
of 350 nm was passed for 2 hours. This light was absorbed by the uranyl oxalate. After
exposure the solution required 34 cc KMn04 to titrate the undecomposed oxalic acid. The
same volume, i.e., 20 cc oxalic acid required 40 cc of KMn04 before exposure. Calculate
the energy absorbed in joule/sec, if <!> = 0.57.

. contains
SoIn. : The so Iut1on . - 0.05
- x 20
1000
= 10-3 moles of oxalic acid.
The complete reaction is :
2KMn04 + 3H2S04 + 5H2C204 = K1S04 + 2MnS04 .+ 8H20 + I 0 C02
1000 cc 2(M) KMn04 =5 moles of oxalic acid .
5
. . I cc x(M) KMn04 =- - xx moles of oxalic acid
2000
= 2.5 x 10- 3 x moles of oxalic acid.
Hence, 40 cc x (M) K~n04 = 2.5 x 10- 3 x 40x moles of oxalic acid.
The solution contained 10- 3 moles of oxalic acid.
PHOTOCHEMISTRY 371

By the problem,
40 x 2.5 x 10- 3x = 10-3

x = _I_ = 10-2.
100
Hence, the strength of KMn04 is io- 2(M). Again. by the problem, H2C204
decomposed is equivalent to (40 - 34) = 6 cc 10- 2(M) KMn04.
Now, I cc of l(M)KMn04 = 2.5 x 10- 3 moles of oxalic acid .
o- o- o-
. . 6 cc of 1 2(M) KMn04 = 6 x I 2 x 2.5 x I 3 = 1.5 x I 4 moles of oxalic o-
acid
. . number of einstein absorbed

= I. 5 x I 0--4 = 2.63157 x 10- 4


0.57
Hence, the energy absorbed in joule/sec. is :
6.022x!0 23 x6.625x!0- 34 J.secx2.9979xl0 8 m/sec
2.63157 x I 0 - 4 x
350x 10-9 m x 2x 60x 60 sec
= 1.25 x 10- 2 J/sec.
29. An actinometer contains 20 cc of 0.05 moles/lit oxalic acid through which light
of 350 nm was passed for 2 hours. This light was absorbed by uranyl oxalate. After exposure
the solution required 28 cc of KMn04 to titrate the undecomposed oxalic acid. The same
volume of solution required 40 cc of KMn04 before exposure. Calculate the energy
absorbed, given that quantum efficiency wavelength is 0.57.
Hints : Proceed as Problem 28. [Ans, 2.5 x 10- 2 joule/sec]
30. At 480 nm, the quantum yield for the production of Fe 2+ in the photolysis of
K3Fe(C204)3 in 0.05 moles/lit of H2S04 is 0.94. After 15 min irradiation in a cell containing
107.4 cc of solution, the solution is mixed thoroughly and 5 ml of sample is pipetted into
a 25 ml volumetric flask. A quantity of I, 10 phenanthroline is added and the flask is filled
to the mark with a buffer solution. A sample of this solution is placed in a 1.00 cm colo~imeter
cell and the transmittance is measured relative to a solution containing no iron. The value

.!J_ is found to be 0.548.


lo
If the extinction coefficient is 1.15 x 103 m 2/mole for rhe complex, how many quanta
were absorbed by the original solution? What was the absorbed intensity in J/sec and in
photon/sec?
Solo. : By the problem-
£ = l.15 x 103 m 2.mole- 1
l = 10-2 m

.!J_ = 0.548.
Io
I
We know that, log _i_ = -£.Cl.
lo
I
or, C = - - log _i_ = x log 0.548
El 10 J.15x!0 3 m 2 .mole- 1 x10- 2 m
= 0.0227147 mole.m- 3 = 2.27 x 10-5 mole.lic- 1.
372 PROBLEMS ON PHYSICAL CHEMI~TRY

5
. l . 2.27 x 10- l f . . . ·
T he co 1onmeter eel contains x 25 mo es o iron. This 1s present m 5
1000
cc of.original solution. Therefore, 107.4 cc of original solution contains

2.21x10-5 107.5 f'


x 25 x - - mo 1es o iron.
1000 5
i.e., 1.220125 x 10-5 moles of iron as a result of exposure. Now, the reaction is :

2Fe 3+ + C 2 0~- ~ 3Fe 2+ + 2C02.

no.of gm moles of Fe 2+produced


Now, <1>
Fe
+2 = no. ofeinsteins absorbed (NE)

l. 220~~~: 10 - = 1.298
5

.. by.the problems, NE= x 10-5 . [".·<j>Fe+ 2 = 0.94]

Thus, the no. of quanta absorbed is :


1.298 x 10-5 X 6.022 x 1023 = 7.816556 x 1018 = 7.82 x 1018.
Now, the energy corresponding to 1 einstein,

E _ 6. 022 x 10 23 x 6. 625 x 10-27 x 2. 9979 x 10 10


- 4800x10- 8 x10 7
So, the intensity absorbed in J.sec- 1 is
1
- - = Ex NE = 3.59 x 10-3 J.sec- 1
sec 900
7.82xl0 18
No. of photon/sec = = 8.69 x 1015 photons/sec.
. 900
31. An actinometer uses a solution of K3 [Fe(C2Q4)3] in which Fe 3+ is reduced and
the oxalate ion is oxidised. Assuming <1> = 1.24 at 310 nm, calculate the intensity of the
incident light which produces l.3 x 10- 5 mole of Fe 2+ in 36.5 min. The same light source
is used to irradiate a sample of CH2CO for a period of 15.2 min. If the quantum yield
of C2H4 is 1.0 and that of CO is 2.0, determine the amount of each gas produced by
the photochemical reaction. [B.U. 1995]
Solo. : According to the given problem,
<1> = 1.24; no. of mole of Fe 2+ produced = 1.3 x 10-5 mole in 36.5 min.
,., no.ofmolesofFe 2 +produced
We k now that, 't' = no. of einstein absorbed NE
( )
or, no. of einsteins absorbed (NE)
= 2
l.3x 10- = l.04 x
no.ofmolesofFe +produced
8 10
=
_5
5

<1> 1.24
In the given problem, time of absorption = 36.5 min = 36.5 x 60 sec.
So, intensity of the incident light in terms of no. of quanta absorbed/sec

l. 048 x 10-5 ~ 6. 022 x 10 23


= = 2.882xl0 15/sec.
36.5 x 60
PHOTOCHEMISTRY 373

Not\', inten~ity in terms of ergs/sec is


Intensity = n.h.v,
where n = no. of quanta absorbed/sec.
h = Planck constant
v = frequency of the incident light.
c 2. 9979x10 10 cm I sec
Here v = =
A 310xl0-7 cm
2. 882 x 10 15 x 6. 626 x 10-27 x 2. 9979 x 10 10
. . Intensity =
310x 10-7
= 1.847 x to4 ergs/sec.
I
CH2CO + hv ~ -C2H4 + CO
2
Energy of I einstein
6. 022 x l 0 23 x 6. 626 x l 0- 27 erg. sec x 2. 9979 x l 0 10 cm I sec
=
310xl0-7 cm
= 3.859 x 1012 ergs.
Energy absorbed by the sample in 15.2 mins
sec
= 1.847 x 104 ergs/sec x 60 - - x 15.2 min
min
= 16844640 ergs.

168 44640
Energy absorbed in einstein = · ergs = 4.36503 x 10-6 einstein.
3.859x 10 12 -~~~~
einstein
no. of moles of C 2 H 4 produced
Now, <f>cH =
2 4 no. of einstein absorbed
Hence, number of moles of C 2H4 produced = <Pc H x 4.36503 x io~ 6
2 4 -6
= 4.36503 x 10 C<Pc2H4 = l)
<P = no. of moles of CO produced
co no. of einstein absorbed
Hence, no. of moles of CO produced = <Pco x 4.36503 x 10-6
=2 x
4.36503 x 10-6 = 8.73 x 10-6 mole.
32. An acid solution of K3[Fe(C204)3] is used as an actinometer to measure the
intensity of light incident upon a vessel. After exposure to light of wavelength 253.7 nm
for 103 sec, the concentration of Fe2+ ion in a l lit solo. was found to be 5.0 x 10-5 (M).
The fraction of light absorbed by the actinometer is 0.1. What is the intensity of the incident
radiation? Given <)> = 1.25.
Solo. : Energy of light per einstein :
= NA X h. v.
6. 022 x l 0 23 x 6. 626 x 10-34 x 2. 9979 x l 0 8
= 253. 7 x 10-9
= 471.5078 kJ.
374 PROBLEMS ON PHYSICAL CHEMISTRY

Number of einstein per sec = 471.5078 J/sec.


2
no. of moles of Fe + produced
Now, ¢ =
no. of einsteins absorbed (EN) ·
= 5x w-s =4 x w-5.
I. 25

·Now, 10 =~ ·/
or, .....E.... = 0.1.
0.1 10
Here la = 4 x w--5 x 471.5078 J/sec = 0.0J 886 J/sec.
I
.. 10 = _a_ = 0.1886 J/sec.
0.1
33. A photographic film is exposed for 0.1 sec to a 250 W incident light at a distance
of 3 m. If I 0% of the power is emitted as visible light of 625 nm to which the film is
sensitive, then estimate the number of Ag atoms that will be produced in a AgBr grain,
J5 µm in diameter.
Solo. : By the problem, the power emitted as visible light sensitive to the film is
JO
- x 250 = 25 watts.
100
Since the exposure is for w- 1 sec, the energy emitted is 25 x 0.1 J = 2.5 J. Now,
the area receiving the energy is 4rcr 2 = 47t(3 )2 :::: l l 3.112 sq. metre.
Intercepting area of AgBr grain is r.r: (circular exposure)
= 3.142 x (15 x w- 6 ) 2 7.0695 x =
10 sq. m. w-
Therefore, energy received by AgBr is :

2. 5 X 7. 0695 X Jo--IO = 1.
5625
x w-l l J.
113.112
Now, the energy corresponding to I einstein is :
23 34 8
E = N Ahc = 6. 022 X !0 X 6. 626 X I0- X 2. 9979 x !0 = 5
'A 625 x w- 9 l. 9 J x !0 J.
I J = 5.2248 J x 10-6 einstein.
The total einstein absorbed is= 1.5625 x 10- 11 x 5.22~8J x 10- 6 = 8.16 x 10- 17
photons.
The number of Ag atoms produced = 8 J6 x 10- 17 x 6.022 x !0 23 = 4.91 x 107 .
34. The quantum efficiency for the conversion of molecule x to y is 0.5 at 300 nm.
For what length of time must irradiation of J 20 W source continues in order to destroy
J mole of x?
Solo. : Energy corresponding to I einstein,
23 34 8
E = N Ahc = 6. 022 x 10 x 6. 626 x I 0- X 2. 9979 x I 0 = 3 .9873841 x l05 J.
'A 300x 10-9
Thus, a 120 W source produces-
170
= - moles of photons/sec
3.9873841 x JO 5
= 3.00949 x w- 4 moles of photons/sec.
PHOTOCHEMISTRY 375

Thus, 0.5 x 3.00949 x io- 4 moles are destroyed pe; sec. So the time required is
6645.6442 sec = 1.846 hour.
35. A reaction responds to both red and violet light (A.= 8000A and 4000A respectively)
with an equal quantum yield. Will there be more photochemical reaction per 100 cal of
light in the red than in the violet or vice versa? [C.U.(H) 1992]
Soln. : Let <h be efficiency with violet light and <1>R be efficiency with red light. ·Let
YR be number of moles reacting per 100 cal for red light of wavelength AR. and Y1. be
the number of moles reacting per 100 cal of violet light of wavelength A,, ..
no.of moles reacting
<1>v = NA x no. of violet quanta absorbed
no. of moles reacting
<!>R = ----------=---
NA x no. of red quanta absorbed
Now, energy corresponding to I quanta is

E= he = I. 9864 x 10-25 J. metre


A A
1.9864x 10- 16 J.metre 4. 74763 x 10- 17
= A(nm)
J =
A(nm)
cal

4. 75x10- 17
= A.(nm)
cal.

A
.. I cal = 4. 75 x 10- 17 quanta

A
and 100 cal = quanta.
4. 75x10- 19

Hence, there will be more photochemical reactions for the red light in the given case.
36. A 100 watts Na-vapour lamp radiates most of its energy in the yellow D-line at
589 run. How long will it take to excite more than half the molecules of an absorbing
species in a 10-3 mole.dm- 3 sample if all radiant energies are absorbed by the sample?
[C.U. 1991]
Soln. : One photon of 589 nm carries energy
= 6. 626 x 10-34 J. s x 2. 9979 x 108 m I sec
1
589x 10-9 m
= 3.3725 x 10- 19 J.
A 100 watts lamp emits 100 J/sec. So, no. of photons emitted per sec

= JOO = 2.963 x 10 20 .
3. 375 x 10- 19
376 . PROBLEMS ON PHYSICAL CHEMISTRY

Now, I o-3 mole.dm- 3 = 6.022 x I 0 20 molecules.dm-3 . To excite more than half


6 022
molecules we require more than · x 10 20 photons.
2
3.0llx!0 20
i.e., > 3.0J1 x I 020 photons. So, time required is greater than . x .
2 963 1020
i.e., > 1.016 seconds.
37. In the decomposition of ethylene iodide (C 2H412) with radiation of 424 nm, the
I,, formed after 20 min required 41.4 ml of 0.005(M) sodium thiosulphate solution. The
i~tensity of light source was 9.15 erg.sec- 1. Calculate the quantum yield. [B.U. 2000]
Soln. : 2Na 2S 20 3 + 12 ----7 Na2 S4 0 6 + 2Nal

Thus, !000 cc l(M) NaSz0 3 = -.!.mole of 12 .


2
41 5
Hence, 41.4 cc 0.005 (M) Na 2 S20 3 = .4 x 0.00 = 1.035 x 10- 4 moles of Iz.
2xl000
Energy absorbed by the system in 20 min from the light source of intensity of
9.15 erg. sec- 1 = 9.15 x 60 x 20 erg
= 10.980 erg.
Energy corresponding to I einstein of photon of A of 424 nm

6. 022 x 10 23 x 6. 626 x 10-27 erg.sec x 2. 9979 x !0 10 cm I sec


= 424x 10-7 cm
= 2.82 x 10 12 erg.
. . .
Th us, energy absor be d m emstem = I 0980 erg x I12einstein = 3.8936 x I _9 . .
0 emstem.
2.82xl0 erg

Hence, quantum yield = cj> = number of moles of I 2 formed


number of einstein absorbed

= 1.035 x 10-4 = 2.65 x 104.


3. 8936 x I0- 9
38. When acetone vapour is irradiated with monochromatic light of wavelength 313 run,
it decomposes to form ethane and carbon monoxide.
(CH 3 ) 2CO(g) ----7 C 2H 6(g) + CO(g)
The reaction cell used has a volume of 59 cm 3. Irradiation for 7 hours at a temperature
56.7 °C produces a change of pressure from 102.16 kPa to 104.42 kPa. If the incident
intensity is 48.1 x w-- 4 J.sec- 1 and acetone vapour absorbs 91.5% of it, calculate quantum
yield.
Soln. : Energy of one einstein corresponding to the radiation of A (313 run)

6.022 x !0 23 x6.626x10- 34 J.secx 2. 9979x !0 8 m I sec


=
= 382177.388 J.
Now change in pressure due to decomposition of acetone
= (I04.42 - 102.16) kPa = 2.26 kPa.
PHOTOCHEMISTRY 377

n1 - x x x
Initial pressure, Pin"" nl
· Final pressure, Pri ex: n 1 - x + x + x
ex: n
1 + x
Thus, x ex: Pfi - Pin·
So, acetone decomposed in gm-mole/litre
(Pfi - Pin)
= (since, P = CR1)
RT

2. 26 x 10 3 N I m 2
= NI m2
1.01325xl0 5 ---x0.0821 l.atm.deg- 1.mole- 1 x329.85 °K
latm
= 8.236300 x 10-4 moles/litre.
The cell is of 59 cc capacity.
No. of moles of acetone decomposed
8. 236300 x 10-4 mole I litte x 59cc
=
1000 cc I I itre
= 4.8594 x I o- 5 moles.
Energy absorbed by acetone vapour in 7 hours
9 5
= 48.1 x 10-4 J/s x 3600 sec x 7 hr x 1. = 110.90898 J
hr 100
. . . 110. 90898 J
Th us, energy a bsorbe d m emstem =
3. 82177 x I 0 5 J I einstein
= 2.90203 x 10-4 einstein.
5
Hence, <!> = 4. 8594 x 10- = 0.167.
2. 90203 x 10-4
39. The quantum yield for the photolysis of acetone (CH3)2CO = C2H6 + CO
with the radiation of wavelength 300 nm is 0.2. How many moles of CO are formed per
second if the intensity of the 300 nm radiation absorbed is 10-2 J. sect?
Solo. : Energy corresponding to l einstein

6. 022 x 10 23 x 6. 626 x 10-34 J I sec x 2. 9979 x 10 8 m I sec


= 300 x 10-9 m
= 398738.4076 J.
10-2 J
Energy absorbed in einstein =
398738. 4076 JI einstein
= 2.50791 x 10-8 einstein.
no. of.moles of CO formed
Now,<!> = no. of einstein absorbed
378 PROBLEMS ON PHYSICAL CHEMISTRY

= no.ofmolesofCOformed
Th us, 0 .2 8 .
2. 50791 x Io- einstein
Hence, moles of CO formed = 0.2 x 2.50791 x 10- 8 = 5.0158 x 10-9 •
40. When acetone vapour is irradiated with light of wavelength 3130 A, it decomposes
to form ethane and carbon monoxide :
(CH3)2CO(g) + hv ~ C2H6(g) + CO(g)
Using a reaction cell of 60.3 ml capacity and a temperature of 56 °C, irradiation for
23000 sec at a rate of 85200 erg.sec- 1 produced a change in pressure from 760 mm to
790.4 mm of Hg. Calculate (i) the number of molecules of acetone decomposed and
(ii) quantum yield. · [C.U. 2006)
Hints : Proceed as Problem 38.
[Ans. (i) 5.375 x 10 19 no of acetone molecules, (ii) 0.174]
41. A solution of a coloured compound of concentration I .Ox Io- 4 (M) has 20% trans-
mission in cell of path-length 1.0 cm at 450 nm wavelength. Calculate the molar absorption
coefficient (E) of the substance. If the path-length and the concentration are both halved,
calculate the percentage transmission. Will the value of E change if light of wavelength
550 nm is used? =
[Ans. E 6989.7 cm- 1.(M- 1), 67%, E depends on wavelength]
42. The quantum yield for the photochemical decomposition 2HI ----? H 2 + 12 at
'A = 254 nm is 1.99. 3070 I of energy is absorbed, find the number of moles of HI
decomposed. [B.U. 1998)
Solo. : Energy of I einstein
23
x 6.626x 10-34 I.sec x 2.9979x 10 8 m I sec
=6.022 x 10 254 x 10-9 m
= 470950.88 J.

Energy absorbed by the system = 3070 J.


3070
It corresponds to einstein = 6.5187 x I o- 3 einstein.
470950.88

= no. of moles of HI decompose'!_


no. of einstein absorbed
Hence, no. of moles of HI decomposed
=
1.99 x 6.5197 x 10- 3 =
1.29 x 10- 2 moles.
43. A dye solution of concentration 0.01 gm/cc absorbs 40% of blue light in a cell
of thickness 1 cm. What should be the concentration to ensure 90% absorption in the same
cell? [B.U. 1998)

Solo. : By the problem, !J_ = 0.6, I = I cm, C = 0.01 gm/cc


lo
log 0.6 = - tCl.
Let C' be the concentration when !J_ = 0.1.
lo

Thus, log 0.1 = - EC'/ or,


C' = logO.l
c log0.6
PHOTOCHEMISTRY 379

Hence, C' = 0.01 gm/cc x logO.l = 0.045 gm/cc.


log0.6
44. The mechanism of the photolysis of HCHO vapours to H1 and CO at 313 nm
is possibly as the following :

(i) HCHO ~ CO + H1
(ii) HCHO ~ H + CHO

(iii) H + HCHO -~ H1 + CHO

(iv) CHO + M ~ CO + H + M

k1 . I I
(v) CHO + wall __,,__,-CO +-HCHO
2 2
If quantum yields for step (i) and step (iiJ are $1 and $2 respectively, obtain the
expression for quantum yield for H1 in terms of $1 and $2.

dCH2
Solo. : Let 4>tt 2 be quantum yield for H1. Thus, 4>tt 2 = -f-.
a

dH
Now - -
dt
2
= la$1 + k1CHCHCHQ, (I)

dCHCHO
where $1 = _ __.d"-'-t_
la
Again from step (ii),

dCCHo
= _d_t_
la
Applying steady state conditions to H and CHO, we obtain
dCH
- - = 0 = la$2 + k1CcHoCM - k1CttCHcHO (2)
dt
dCCHo
- - and = 0 = la4>2 + k1CttCHcHO - k1CrnoCM - k3Cetto. • (3)
dt
Adding Eq (2) and Eq (3), we get
2la4>2 - k3Cetto = 0.
, . Cctto = 2$2la
k3
CHAPTER 11

ELECTROLYTIC CONDUCTANCE

Required Formulae
KA K
1.G=-=
l cell constant '
where G = conductance,
K: = conductivity or specific cenductance

= distance between two parallel electrodes


l
A = area of the electrode.

l.(a) A = IOOOK,
N
where A = equivalent conductance or equivalent conductivity
N = normality of the solution.
If N* is concentration in gram-equivalent per cc, then
1(
A= N*.

(b) A = 1000K
m C .
C = molarity of the solution and N = ZC.

A= Am.
z
For an electrolyte, A +B _ ~ v+ A z+ + v- az-
v v

A = Am = Am = Am .
v+Z+ v_Z_ Z

3.(a) For strong electrolyte, A0 = A.+ + A.-,


where A 0
= equivalent conductance at infinite dilution
A.+ = equivalent conductance of positive ion
A.- = equivalent conductance of negative ion.
380
ELECTROLYTIC CONDUCTANCE

(b) For an ele~trolyte A +B _


v v

Am = v+f"m.+ + v_A.m.-
"'-m.+= molar conductance of positive ion
"'-m.- = molar conductance of ne~ative ion.

4. v = u +~<\>
± - l
v± = velocity of the ion
u± = mobility of the positive or negative ion
~<\> = potential difference between two electrodes or voltage applied

~<\> = potential gradient.

5.(a)

t± = transport number of positive or negative ion


I± = current carried by positive or negative ion.
I+ = current carried by positive ion
/_ = current carried by negative ion.
For an electrolyte A + B _
v v
.
t = Z+C+Fv+A = ~
± Z+C+Fv±A+Z.CFv_A v++v_
Since, C± = v±C and Z+v+ = Z_v_
C± = conccentration of positive or negative ion in gm-mole. m-3
C = concentration of electrolyte in gm~mole. m- 3
In a mixture where C's of different salts may vary.

tB = ZaCava
r.z,.c,.v,.
ZB = charge of ion B
CB = concentration of ion B in gm.mole.m-3
vB = velocity of ion B
zi = change of ith ion
Ci = concentration of i 1h ion
vi = velocity of ith ion

<\> ~
Since v = u.~-, tB =. ZBCBUB and ,...,.1 = Fu., .
thus tB = ZBCBA.B ,
1 · r.z,.c,. v,. 1
r.z,.c,.A.;
where, Ai = equivalent conductance of ith ion; KB = ZBC8A.8 and Kj = ZiCiA.i, thus.
Ka
tB =
LK;

5.(b) ~,...,m,8 = FZBuB• thus tB ="'-maCa


---.
I.AmI C;
382 PROBLEMS ON PHYSICAL CHEMISTRY

6. Transport number by moving boundary method :


laFN
f. = ---
l IOOOQ'
where l is the length through which boundary moves and a is the inner area of the tube
N
in which ion moves. Q is the charge in faraday that passes through number of
F 1000
gm-equivalent per unit volume. In C.G.S., it is gm-eq.mJ- 1.

Also t; = laFC
'z Q
where C = concentration of electrolyte in gm.mole.m- 3.
7. Hittorf's Method
t± = loss or gain in gm-eq per faraday in a given chamber.
If the electrodes are inert, then loss of cation in cathode chamber is t_ gm-eq per
faraday and loss of anion from the same chamber is t_ gm-eq per faraday. Thus, loss of
electrolyte from cathode chamber is t_ gm-eq per faraday. Similary, loss of electrolyte from
anode chamber is t+ gm-eq per faraday.
Loss of anion from cathode chamber per faraday is t_ gm-eq. in cathode compartment.
If the electrodes are reactive, i.e., electrolysis of CuS04 between copper electrodes,
of AgN0 3 between silver electrodes, then gain in cation, anion and S<,ilt in anode chamber
is t_ gm-eq per faraday. If the cathode is reactive which will be a rare case, then the net
gain of electrolyte in cathode chamber is t+ gm-eq. For detailed analysis go through Physical
Chemistr)I Vol. I by Dr S. Pahari (page nos. 559-561)
Short Questions
I. "During measurement of conductance a.c. is used."-Explain.
Ans. : If d.c. is used in conductance measurements and current flows through the
cell, there will be electrolysis. The concentration will change as a result of this, and hence,
also the conductance as the conductance depends on concentration. Moreover, there will
be polarisation at each electrode due to this irreversible phenomenon. Thus, to avoid
electrolysis and polarisation, a.c. is used. If a.c. is used, electrodes change sign regularly.
However, use of a.c. cannot eliminate polarisation completely, but it may reduce polarisation
considerably. The higher the frequency, the better is the reduction of polarisation.
2. The electrodes used in conductance measurements are platinised.-Explain.
Ans. : When a.c. is used, the polarisatioin of electrodes is greatly reduced. But unless
extremely high frequency is used, polarisation is not completely eliminated. H+ and oH-
ions, being very fast, tend to be liberated at the electrodes by successive pulses of current.
Electrodes are coated with finely divided platinum black for aiding the elimination
of polarisation effects by alternating current. This is platinisation. The larger surface area
of the finely divided platinum appears to catalyse the union of hydrogen and oxygen, which
tend to be liberated by successive pulses of the current (eliminating bubble over voltage).
What is more, platinum coated black surface absorbs gases formed effectively. The
polarisation e.m.f. is thus almost completely eliminated.

3. "The relation A= IOOOK between eq. conductance and specific conductance means
c
that A is inversely proportional to concentration".-Comment.
ELECTROLYTIC CONDUCTANCE 383

Ans. : The proportionality equations are as y =Kx, where K is proportionality constant,


and thus, y oc x. Therefore, in the above equation A oc _!_, if 1000 K; i.e., K is constant.
c
But K depends on concentration. Hence the above relation does -not mean that A oc ~
However, A increases as cone. (C) decreases. K, on the other hand, decreases as C
decreases. The reason why A increases as C decreases is that K decreases with decrease
.
in concentration at a rate lower than the rate of decrease of concentration .

4. "_!_ is plotted against number of drops in a conductometric titration. "-Explain.


R

Ans. : _!_ is the conductance G of the system. The conductance G depends on the
R
number and type of ions for a solution at a fixed temperature and volume. If titrant is
added freely, volume will change. Thus the concentration of the titrant is kept much higher
than the solution to be titrated, so that the volume does not change much and titration
is completed just by a few drops.
Now, since G changes regularly as per change in number and type of ions as a result
of addition of titrant, the analysis of plot of G versus drops of titrant will give an idea
of titrant needed for complete titration. Since the type of ions is a factor, they are so chosen
tha,.t rate of change of conductance G before and after end point are different. Thus, a
study of G versus drop will give the end point. That is why G is plotted against drops
of titrant.
5. "For weak electrolytes, A increases with dilution indefinitely and hence, there is
no A0 ."-Comment.
Ans. : The statement is wrong. At average concentration, dilution leads to increase
in the number of ions and hence, conductance increases. [:\n ionogen BA dissociates as
BA ~ B+ +A-
(I - a.C) a.C aC where a is the degree of dissociation and C is
a.2C
concentration, so that K is - - (approx),
1-Ct.
since a = C as y = 1. So, as C decreases, a. increases.]
But there is electrophoretic and asymmetric effect, and hence, increase in conductance
is very small. After a certain dilution, these effects start losing their importance (as the
distances between ions become very large) and a also
increases many-fold, and hence conductance (L) and
equivalent conductance (A) increases sharply in A versus
C plot. This may appear almost vertical. If we could
measure up to C =0, we would have noted that A merges
with the axis and that would be measured /\ 0 • But there
is a concentration below which no measurement could
be done and thus increase appears as indefinite increase.
A is finite at theoretical C = 0 and is A which is the
0
,

sum of conductance.
:. A =
0
"-+ + A._.
384 PROBLEMS ON PHYSICAL CHEMISTRY

6. What should be the nature of plot of conductance values of strong electrolytes of


varying concentration measured in a particular cell against respective specific conductance
values? Does this plot give any information about the cell? How essential is the information
for any absolute measurement in the cell?

Ans. : We know that conductance G = K Al , where I\


.
is specific conductance and

= K, the cell constant. Hence, a plot of G versus I\ measured in same cell will be
A
linear passing through origin.

The slope of the plot is A or So this plot gives information about the cell.
l K
If we want to· measure I\ or A, we need to know K, the cell constant, because we

measure only G (A= 1O~OK). Thus the information is very essential for any absolute
measurement.
7. Comment on the statement : "Debye-Hi.ickel theory is applicable only to slightly
contaminated distilled water."
Ans. : The basic assumptions in the Debye-Htickel theory are as follows
(i) Ionophores are completely ionised at all concentrations.
(ii) Ions are point charges.
(iii) The bulk relative permitivity of the solvent controls the attraction between the
ions and remains unchanged in presence of solute.
(iv) Inter-ionic attraction are due to Coulombic force only. There are no other inter-
ionic force.
(v) The electrical potential energy is much smaller than its thermal energy so that
Poisson equation can be linearised.
Linearisation of Poisson equation fails to allow the interaction between the ions that
constitute the ion atmosphere. This can be valid only at infinite dilution. That is why it
is sometimes asserted that Debye-Hi.ickel theory has application only to slightly contami-
nated distilled water.
8. Why does the u+ ion move slower than K+ ion in water?
Ans. : At a particular temperature and under the same set of conditions, a smaller
ion should move faster than a larger ion. u+ ions are smaller than K+ ions. But water
is a polar solvent, and water molecules are attracted by the positive ions forming tight
layer surrounding the ions. u+ and K+ both are unipositive, but the radii of Li+ ions are
smaller than the radii of K+ ions and thus the former have higher charge density.
Consequently, u+ ions will be more solvated in water and the number of water molecules
attached to u+ will be much higher. As a result the total apparent mass of u+ will be
much higher than· that of K+ in water, and Li+ ions will move slower. Li+ ion will
consequently ·be less conducting also.
(Remember that in a solvent where there will be no solvation or negligible solvation,
the speed of u+ ions will be greater than those of K+ ions.)
9. Why should equivalent conductance of an electrolyte at finite concentration be less
than that at infinite dilution?
Ans. : At infinite dilution, all electrolytes are completely ionised and there is no
asymmetric and electrophoretic effect. No ion pairs are formed also. Thus the equivalent
ELECTROLYTIC CONDUCTANCE 385

conductance of an electrolyte at a particular temperature becomes the highest for any solvent
(termed A 0 ). At a finite concentration all these effects are present. What is more, ionogens
are not even completely ionised. Consequently, A values will be lower than A0 •
10. Explain why the changes in equivalent conductance of KCl and HOAC with
concentration are widely different.
Ans. : KC! is a strong electrolyte or ionophore, and is completely ionised at any
concentration. Therefore, the conductance at any concentration is higher than that of weak
electrolytes or ionogens like HOAC which are only partially ionised (and the degree of
ionisation << 1 at normal concentration.)
In ionophores, surrounding every ion, an ion atmosphere of opposite charge is present.
When it moves from one place to another, the old atmosphere is destroyed and a new
ion atmosphere is created. There is a time lag between the two incidents, and until the
new atmosphere is created, ion is attracted or pulled backwards by old ion atmosphere.
The conductance decreases with increase in concentration due to the increase in this
asymmetric factor. Ion pair formation and electrophoretic effect are also responsible for
the reduction of conductance. As C decreases, these effects decreases and conductance
increases. This decrease or increase is linear in ,,/C from a very low concentration to a
reasonably high concentration. The variation with concentration is small. For ionogens like
HOAC, a << I at normal concentration. The ions are far off and asymmetric effect,' etc.
are unimportant. With decrease in concentration a increases and conductance increases.
But with increase in number of ions, the asymmetric effect, etc. become important. The
nature of variation is different from ionophores. The variation is lai;ger with concentration.
11. Will the conductance of a substance change if the voltage is changed?
Ans. : The voltage ~<I> is related to current i through the proportionality constant
conductance G, i.e., ~<I> G =
i. Thus if ~<I> is changed, i will be changed and G (and
hence K: and A) will remain unchanged. However, if Ohm's law is not valid (which may
happen at extremely high voltage), then G may change on changing ~<I>.
12. If the resistance of O. l(N) aqueous solution of KC! is measured in different
cond 11ctivity cells, will the values of conductance, specific conductance and equivalent
conductance change?
Ans. : The conductance of any solution at a particular temperature, G is given by
A
G = K:. T, where K: is the conductivity, A is t~e area of the parallel electrodes and I is

the distance between the electrodes. Thus, if the cell constant .!:_
of two different cells
A
are different, the conductance will change. But the specific conductance or conductivity
K is a proportionality constant depending on the nature of the material and temperature,
and thus. will not change if the cells be changed. The equivalent conductance. A, of a
substance is related to K: and concentration, and thus, will not change on changing the
cells.
13. What is the absolute mobility of ions?
Ans. : The absolute mobility of an ion, uah' is the velocity of the ion per unit applied
force at infinite dilution. When ions are in a force field, the force imparts a component
to velocity vd. By the Newton's ~econd law.
dv
F = m. ( 1)
dt

Proh. Phv \lwrn __ 1<;


386 PROBLEMS ON PHYSICAL CHEMISTRY

where m is the mass. An ion starts off after a collision. Let the mean time between collisions
be r and there be N collisions in time t. Thus,
t
r = -. (2)
N
The average value of the velocity is the product of acceleration and average time
between collisions.
dv F
vd = r.-
dt
= r.- (3)
m

= ~.F = u 0
b.F [by definition).
m

Thus, uab is ~ it is an index of how mobile an ion is.


m
Its unit is cm.sec- 1.dyne- 1 in C.G.S. system.
14. How did you account for the abnormally high mobilities of H+ and oa-. ions
m water?
A.ns .. : The charge density of an ion increases with decrease in radius, and hence,
hydration increases. Therefore, the effective radius should increase and the velocity should
decrease. But the velocities and mobilities of H+ and OH- ions are very high. It has been
explained to be due to Grotthuss type mechanism. In case of H+ and OH- movement,
there is an effective motion of a proton or hydroxide ion, which involves the formation
and destruction of bonds through a Jong chain of water molecules. The velocity and
conductivity are controlled by the rate at which a water molecule can reorient in order
to accept and then deliver protons and hydroxide ions from and to its neighbours.

H H H H
I I I I
H-·O-H + 0 - H - H - 0 + H-0-H
87 87

H H H H
and 0
ie 1
+H-0-0-H+O-
1 ie

the whole process may be depicted as

For effective motion of H+ and OH- are needed lengthening and shorting of bonds;
reorientation for the latter,

8
H-0-----H {""; / Hf ; / Hf ; /H
'o-----H- o------H-0-----·H- 0
H/
15. Conductance of H+ ion increases as temperature increases--criticize.
Ans. : Conductance of tt+ ion increases as temperature increases. H+ has the highest
mobility of all the ions. It can be explained by Grotthuss mechanism. Mobility and hence,
ELECTROLYTIC CONDUCTANCE 387

the conductance depend on the rate at which H 2 0 or ROH molcule can reorient to accept
and transfer proton from and to neighbouring molecules. Hence, the process involves energy
barrier. Thus, as. temperature increases, the rate increases, thereby increasing the mobility
and hence, conductance of H+ ion.

16. The plot of _.!._ vs AC for a weak electrolyte has unit slope. Express dissociation
A
constant of the weak electrolyte in terms of a and A. Here all the symbols carry usual
significances.
2
a C
Ans. For a weak electrolyte dissociation constant Kd =- -
1- a'
where C = concentration of weak electrolyte
A
a = degree of dissociation = Ao

..
(;ore --- =
Kd = A or,
A A 0

Ao
1--

By the problem, A~Kd =

or, (~r Kd = 1

Kd = (~r
Problems
1. What volume of 0 2 would" be liberated from an aqueous solution of NaOH by a

current of 5 A flowing for 2 .!. hours? The temperature is 27 °C and the pressure is I atm.
2
Soln. : We know that,
l
20H- = H 20 + -0 2 + 2e.
2
Thus, 2 F charge liberates 11.2 litres of 0 2 at NTP.

Now, Po Vo P1V1
--=--
To Ti
l 1200x 1 xxl
or,
273.15 300.15
11200 x 300.15
or, x = cc.
273.15
388 PROBLEMS ON PHYSICAL CHEMISTRY

Hence,
5
96484 coulombs liberate ll. 2x x 300.l
. l't
1 res o f 0 z at 27 °C an d I atm
2 273 15
ll.2x300.15
litres
2 x 273.15 x 96484.

I 1.2 x 300.15 x 5 x 2.5 x 60x 60


5 x 2.5 x 60 x 60 ,, - - - - - - - - - - - - - litres
2x273.15 x 96484
= 2.87 litres of 0 2•
2. What volume of Hz would be liberated at the anode if 2 A current flows through
excess fused LiH for 2 hours. The temperature is 680 °C and pressure is I .5 atm.

Hints : Problem ~ I (H+ + e = _!_Hz) [Ans. 3.89 lit]


2
3. How long would it take a current of 2 A to reduce completely 240 cc of 0.1 (M)
Fez(S0 4 ) 3 to FeS04 ? How many cc of O.l(M) KzCrz0 7 could be reduced to Crz(S0 4) 3
by the same quantity of electricity?
Reaction : 2Fe3+ + 2e = 2Fez+
Therefore,
2 faraday reduces 1000 cc of I molar Fez(S0 4 ) 3
2 x 96484
coulumbs reduce I cc of I molar Fe 2(S0 4 ) 3
1000
2 x 96484
x 24 coulombs reduce 240 cc of 0.1 molar Fez(S0,4 ) 3
1000
[since 240 cc O. l(m) = 24 cc l(m))
2 x 96484 x 24
Thus, 2 x t = [where t is the time taken)
1000
2 x 96484x 24
t = -----sec
1000x2

2315616 sec
= = 38 .59 .
mmutes = 38 . 35 sec.
mm
1000
Now, Cr 2o7= + 14H+ + 6e = 2Cr 3+ + 7H20.
Thus, 6F reduces 1000 cc I (M) Crz07 =.

Hence. the. volume in this case is the .!.rd


3
of the former one, that is 80 cc.

4. How long would take a current of 5 A to reduce completely 100 cc 0.1 (M) dichromate?
How many cc of 0. I (M) Al 2 (SO 4 ) 3 can be reduced by the same amount of elecricity?
[Ans. 19.3 min. 200 cc]
5. What quantity of electricity would be required to reduce 1.5 gm of nitrobenzene
to aniline? If the potential drop across the cell is 2 volts, then how much energy in joules
is consumed in the process?
ELECTROLYTIC CONDUCTANCE 389
·~~~~~~~~~~~~~~~~~~~~-

Solo. : For the given reduction,


C 6H 5N0 2 + 6H+ + 6e = C 6H 5NH 2 + 2H 20
6 faradays of charge reduces 123 gm nitrobenzene.
That is, 123 gm nitrobenzene is reduced by 6 x 96484 coulombs.

6 x 96484 x I. 5 l b
1.5 gm nitrobenzene is reduced by cou om s
123
= 7059.8 coulombs.
Energy consumed = 2 volt x 7059.8 C = 14119.6 joules.
6. Calculate the total charge in coulomb carried by all Ca2+ ions in 500 cc O.Ol(M)
CaC1 2 solution. Assume complete ionisation.
Solo. : 1000 cc l(M) CaC1 2 carries 2 faradays = (2 x 96484) coulombs.
. . 2x96484x0.0lx500
:. 500 cc O.Ol(M) solution will carry= = 964.84 coulombs.
1000
7. The molar conductance of a solution of AlC1 3 is found to be 130 ohm- 1.cm2 .mole- 1
at 298 K. What would be its equivalent conductance?

Solo. : Equivalent conductance/\= IOOOK, where N is the normality of the solution.


N

IOOOK
/\ = - . mo Ianty
- , where C 1s . an d Z 1s
. va 1ency.
zc .

/\ = /\m , where /\m is molar conductance


z
/\ = -130 ohm- I .cm 2 .gm-eq-· J = 43.33 ohm- 1.cm 2.gm-eq- 1.
3
8. The resistance of an aq. solution containing 0.624 gm of CuS04 . 5H2 0 per 100
cc of solution in a conductance cell of cell constant 153. 7 m- 1 is 520 ohms at 298 K. Calculate
the molar and equivalent conductivity (molar mass = 249.5).
Solo. : We know that,
1
= 1(.-A =IC.- [where !:_ is cell constant, R is resistance in ohm
R l I/A A
and K is ohm- 1.cm-'J
Now, 153.7 m-1 = 1.537 cm- 1.
I l I
= -.- = - x 1.537 ohm- 1.cm-I
R A 520
= 2.9558 x 10- 3 ohm- 1.cm- 1.
6 24
6.24 gm/litre = · molar = C molar = 0.0250 I molar.
249.5
3
M o 1ar cond uctance /\m = -1000K
- = 1000x2.9558xlo- ohm-I.cm 2 .mole-I
c 0.02501
= 118.2 ohm- 1.cm 2.gm-mol- 1•
118 2
/\ = · ohm- 1.cm 2 .gm-mo1- 1 = _
59 .I o hm-l .cm2.gm-eq-l .
2
390 PROBLEMS ON PHYSICAL CHEMISTRY

9. The resistance of a electrolyte solution A is 45 ohms in a given cell and that of


electrolyte solution B is 100 ohms in the same cell. Equal volumes of solutions of both
A and B are mixed. Calculate the resistance of this mixture, again in the same cell.
Soln. : We know that,
A
-R1 = K.-.
l
A
Let - =x cm.
l

Now, R1 = K1. 1A .
I

[where K1 = conductivity of soln. A in ohm-1.cm-i


R1 = resistance of soln. A in ohm]
0.0222
. , Ki = = = x'

A
.And = Kz.-
/
[K2 is the conductivity in ohm- 1 of B soln., R2 1s the resistance in ohm of B soln.]
I I
or, K2 = R2 x
0.01
.,,

100 x
= x
In the mixture,

= 0.02222
x
x -I o hm- i .cm- I and Ki= --x-
2
0.01 I h -i
x
-I
o m .cm .
2
For the mixture, K = Ki + K2

= ( 0.01111 +0.005)
--om h -i .cm -i
x x

= 0.016111 ohm-I.cm-I.
x

= K. A [where R is the resistance of the mixture in ohm]


R I
111
= ( 0 · 01: .x)ohm-i = 0.061111 ohm-i.

I
:. R = ohms = 62.07 ohms.
0.016111
10. The resistance of an electrolyte solution A is 10 ohms and that of solution B is
50 ohms i~ the same cell. l volume of A of the same concentration is mixed with the 3
volumes of solution B of the above concentration. Calculate the approximate resistance of
the mixture, again in the same cell.
ELECTROLYTIC CONDUCTANCE 391

Soln. : Proceeding as Problem no. 9 and assuming


A
= x cm,
l
we get,

= ICA ohm-I .cm-I x x cm.

.. = l(A Xx
10
O.lohm- 1
or, l(A = -xcm
--

0.02 ohm- 1
Similarly, 1C8 =
xcm

Now the given dilution will reduce ICA to ~th and 1C


8
to 2. th of the original and IC' s
4 4
are additive.
. . for the mixture.

IC= (QJ_x_!._+ 0 · 02 x~)ohm-I.cm


x 4 x 4

0.025
= (- - +0.015)
-- h -I .cm.
om
x x
0.04 h -I
= --x
om .cm.

for the mixture, ~ = ( 0 ·~4 . x) ohm-I = 0.04 ohm-I.

R = 25 ohms.
11. Calculate A andµ of a O. l(N) solution of Al 2 (S0 4 ) 3 if its IC is 0.0108 ohm- 1.cm-I.
lOOOK
Soln. : We know that, A =- -.
N
Now, proceeding as the previous problems. [Ans. 108, 648]
12. The equivalent conductance of NaCl at infinite dilution is 126.4 and the transference
number of Na+ in NaCl is 0.40 at 25 °C. Calculate the ion conductance of Na+ and c1-.
[C.U. 1970]
Soln. : Let the ion conductance of Na+ be A+ ohm-I .cm 2 and the ion conductance
of CJ- be A_ ohm-I .cm 2. The equivalent conductance at infinite dilution is A 0 ohm-I .cm 2
= 126.4 ohm- 1.cm 2. Then,
A+ = t+A = (0.40 x 126.4) ohm-I.cm 2
0

= 50.56 ohm-l.cm2.
A._ = t_A 0

= {(1 - 0.40) x 126.4} ohm-I .cm 2


= 75.84 ohm-1.cm2•
392 PROBLEMS ON PHYSICAL CHEMISTRY

13. Calculate the ionic mobility of the cation in KCI at 25 °C, given that its transport
number is 0.49 and the equivalent conductance of KCI at infinite dilution is 150.
Solo. : We know that,
"-+ = t+A 0

That is, Fu+ = t+ A ohm- 1.cm 2.


0

1 2
u = t+A
0
ohm- .cm
+ F coulomb
By the problem,
0.49x 150 cm 2 cm 2
u+ = = 7.62 x 10-4
96484 volt. sec volt. sec
ampere .. ,
[since ohm- 1 = --- and coulomb = ampere x second)
volt
m2.
= 7.62 x 10-8
volt.sec
14. The equivalent conductance at infinite dilution of HCI, NaCl and NaOAC are
426.2 ohm- 1.cm 2, 126.5 ohm- 1.cm 2 and 91.0 ohm- 1.cm 2 respectively at 25 °C. Calculate
A for CH 3COOH.
0

A conductance cell filled with O.Ol(M) KCI has a resistance of 257.3 ohms at 25 °C.
The same cell filled with 0.2(N) CH3COOH has a resistance of 508.6 ohms. Calculate the
dissociation constant of HOAC.
[K of O.Ol(M) KCI = 1.41 x 10- 3 ohm- 1.cm- 1] [V.U. 2004; C.U. 2001]

Solo. : We know that


A HOAC = A HCI + A Na0AC - A NaCI
0 0 0 0

= (426.2 + 91 - 126.5) ohm- 1.cm 2 = 390.7 ohm- 1.cm2•

Again, let A =x cm.


l
1 A
For KCI solution, = K. - = ICX.
R l
By the problem,
1
- -- - = 1.41 x 10-3 ohm- 1.cm- 1 xx
257.3ohm
1
x =
257.3ohmx1.41x10-3 ohm- 1.cm- 1
= 2.7564 cm.
lOOOK
Again, for HOAC, A =- - ohm- 1.cm 2
N

= K.X
1 1
K = - - x - -- ohm- 1.cm- 1 = 7.133 x 10-4 ohm- 1.cm- 1.
508. 6 2. 7564
A = JOOOx K = lOOOx 7.133x 10-4 ohm-1.cm2
0.2 0.2
= 3.5665 ohm- .cm2.
1
ELECTROLYTIC CONDUCTANCE 393

a..2c
Now, Ka of HOAC is - - ,
1-a
A
where a is degree of dissociation and is
Ao
2
A2 C · (3.5665) x0.2
:. Ka = Ao (Ao -A) = _ _,,___ _,____ = 1.68 x 10-s.
390. 7(390. 7 - 3. 5665)
15. 200 ml of 0.002(M) BaC12 is added to 300 ml of 0.003(M) Na 2S04. Calculate
approximate IC of the resulting mixture. Determine transport number of c1-.
Given, /...± of CY = 76
Na+ = 50
_!_Ba++= 80
2
I
2so4 = 80.
Soln. : BaC1 2 will react with Na 2S04 to give insoluble BaS04. So after mixing the
molarity of the ions in solution is :
2
C 8 a++ = 0.002 x = 0.0008 (M)
5
2
= 0.004 x 5
= 0.0016 (M)
3
= 0.006 x = 0.0036 (M)
5
3
= 0.003 x
5
= 0.0018 (M).

Ba2+ ions are in lesser amount.

So all the Ba 2+ will be precipitated as BaS04. Strength of the remaining S04 will
be (0.0018 - 0.0008) (M'.) = O.OOI(M).
CA
Now, IC = 1000
IC = C. Ao+
± 1000

Therefore, 1<- - = O.OOl 6 76 = 1216 x 10-4 ohm-lcm-1


""L 1 1000x . . .

Similarly, = 0.001x80x2
. -- 16
. x 10-4 ohm-I .cm -I .
1000
= 0.0036X50 _- l .8 x 10 _4 o hm-I .cm-I .
1000
Adding, approximate IC of the soln. is 4.616 x 10-4 ohm-1.cm-1.

= 1.216x 10-4 = 0. 26 .
4.616x 10-4
394 PROBLEMS ON PHYSICAL CHEMISTRY

16. JOO ml of 0.002(M~ NaCl is added to JOO ml of 0.004(M) AgN0 3 . Calculate an


approximate value for the specific conductance of the mixture. [C.U. 1973]
Given, A± of Na+ = 50. I
Ag+=61.9
ci- = 76.3
NO}= 71.4.
Hints : Proceed as the previous problem. Approx. K is 2.548 x 10- 4 ohm- 1.cm- 1
= 2.548 x 10- 2 ohm- 1.cm- 1.
17. Given that A.Ag+= 54, A.c1- = 65.6, A.K+ = 84.2 and A.N _= 79.4 at a particular
03
.
temparature. A solution of 200 cc of 0.02(M) AgN0 3 is mixed with 300 cc of O.Ol(M)
KC!. Calculate the approximate value of K.
Hints : Proceed as Problem 16. [Ans. 1.25 X 10-31
18. At 25 °C, the specific conductances ·of a saturated solution of Sr SO 4 and that of water
used are 1.482 x 10-4 ohm- 1.cm- 1 and 1.5 x 10-<:i ohm- 1.cm- 1 respectively. Given the ion
conductances of Sr2+ and soi- to be 59.46 ohm- 1.cm 2 .gm-eq- 1 and 79.8 ohm- 1.cm 2 .
gm-eq- 1 respectively at this temperature. Calculate the solubility of the salt in gm.lie 1 [M.
Wt. of SrS0 4 = 182) [C.U.(H) 1996]

Solo.:
A~rso 4 = (59.46 + 79.8) ohm-I .cm 2.gm-eq-I
= 139.26 ohm- 1.cm 2.gm-eq-1.

Now, A = - - [where N 1s
1000K . norma 11ty
. ].
N
Since the salt is sparingly soluble,
0
A ""'A and Ksrso 4 = Ksrso 4 soln. - KH 2o·
Thus, K = (1.482 -- 0.015) x I o-4 ohm-I .cm-I
= 1.467 x 10-4 ohm- 1.cm-I.
JOOOK = lOOOx (l.467)x 10-4 = 1.
Hence N = 05342 x 10 _3 (N).
Ao 139.26
.
E q. Wt. IS l182 =
91
.
. . solubility in gm.lie 1 is ( 1.05342 x 10-3 x 91) = 0.096 gm.lit- 1•
19. If the eq. conductances of solutions with varying cone. of CHFOONa, NaCl and
HCI are plotted against ..jC, the intercept obtained are 91.0, 128 and 425 in ohm- 1.
cm 2 .eq- 1 at 25 °C respectively. If the resistance of 0.02 molar soln. of Clt 3COOH in a
cell with a cell constant 0 ..2061 cm-I be 888 ohms at 25 °C, what will be the degree of
dissociation of the acid at 25 °C?
Solo. : By the problem,
0

A HoAc = (91 + 425 - 128) = 388 ohm- 1.cm 2 .gm-eq-I

Again, - - - = Kohm-I.cm-Ix (Al )cm


888ohms
ELECTROLYTIC CONDUCTANCE 395

l
i.e., = K X
888 0.2061

i.e., K = 0.2061 ohm-l.cm-1


888
IOOOK IOOOx 0. 2061
A =--= = 11.60473 ohm- 1.cm 2.gm-eq-1
N 888x0.02
A 60473
a=
Ao
= l l. = 0.029909 = 0.03.
388
20. At 25 °C in a saturated solution of BaSO 4 , Ba2+ and S04 ions take 151 sec and
121 sec respectively to cover a distance of I cm across which a potential difference of
10 volts is applied. The solubility product of BaS04 at 25 °C is 10- 10 (considered in gm-
eq.lic1 unit). The conductance of the solution is 2.2 x 10-6 ohm- 1 in a cell of which the
conductance of water is 0.8 x 10-6 ohm- 1 ac 25 °C. If in the same cell conductaoce of
O.Ol(N) KCl be 1.40 x 10- 3 ohm- 1 at 25 °C, what would be its specific conductance at
the same temperature? [C.U.(H) 1994]
Solo. : By the problem,
!cm
15lsec
lcm
v_ =
12lsec
I l l
U+ = V+. - = -(cm/sec) x -(cm/volt)
~4> 151 10
cm 2
= 1510 volt.sec
1
Similarly, u_ = - -cm2.vole 1.sec-1.
1210
l cm 2
=Fu+= 96484 x --(A.sec) x
1510 volt.sec
1
= 63.896684 ohm- .cm2.gm.eq-1
96484
A- = Fu_ = - - = 79.7384 ohm- 1.cm 2.gm-eq- 1
1210

·. A~aso4 = A+ + A_ = (63.896684 + 79.7384)


= 143.64 ohm- 1.cm 2.gm-eq-1.
Now again, solubility product
Ks = 10-10.
:. solubility = ,10-5 gm-eq.lic 1.

NA 10-5 x143.64
And K =- - = - -1000
1000
--- = 1.4364 x 10-6 ohm- 1.cm- 1.

[Since BaS04 is sparingly soluble, A = A 0


]
396 PROBLEMS ON PHYSICAL CHEMISTRY

. A
Agam, G = K x -
I
Now, G = (2.2 - 0.8) x 10-6 ohm- 1 == 1.4 x 10-6 ohm- 1.

A
1.4 x 10- 6 ohm- 1 = K" x
l
A 1.4x10-{i
or, l = I.
4364
x -{i cm
10
= 0.97466 cm.

By the problem,
for KCI, G = 1.4 x 10-3ohm~I.
A
1.4 x 10-3 = I( x - .
l
1.4 x 10-3
:. K" = ohm- 1.cm- 1 = 1.4364 x 10-3 ohm-I.cm-I.
0.97466
21. The resistance of a condutivity cell was found to be 700 ohms and 800 ohms when
filled with 0.0 I (N) KCI and 0.0 I (N) AgN0 3 solution respectively. The eq. conductance
of KCI is 150 ohm- 1.cm 2. Find the eq. conductance of AgN0 3. [B.U. 1994)
Soln. : We know that,
I A
- - = K"kCI·-
RKct l

and --- = K"AgN0 3,A.


RAgN03

KAgN03 RKCI 700


= =
800
KKCI RAgN03

AAgN03
= ( l~KAgN03} ( NKCI ) = KAgN03
=
700
AKCI NAgN03 IOOOKKCI KKCI 800

[since N for both is O.Ol(N)]

A
AgN03
= AKCI· 700
800
= 150. 700
800
= 131.24 ohm-I.cm2.gm-eq-I.
22. Ksrso = 1.5 x 10-4 and KH 0 = 1.5 x 10-6 at 25 °C. Calculate solubility product
4 2
= 135.8, = 153.5, = 149.85.
0 0

of SrS0 4 • Given, A 1 A1 A°KCI . [B.U. 1993]


2SrCl2 2K2S04

Soln. : Proceed as former problem. [Ans. 2.8 x 10-71


23. A 0.02(M) KCI solution having a specific conductance of 27.7x10- 4 ohm- 1.cm- 1
gives a resistance of 175 ohms when measured in a certain conductivity cell. A 0.0 I (M)
soln. ofCH 3COONa in the same cell shows a resistance of 575 ohms. Calculate the equivalent
conductance of this CH 3COONa solution.
Soln. : Proceed as usual. [Ans. 84.3]
ELECTROLYTIC CONDUCTANCE 397

24. A saturated soln. of SrS04 shows a specific conductance (K) value 1.5 x 10- 4
ohm- 1.cm- 1 at25 °CThe solubility is 0.5 x 10- 3 mole.litre-I and the equivalent conductance
A 0 Srso = 140 ohm- 1.cm2.gm-eq- 1. Calculate the approximate value of K for water at the
4
given temparature. [B.U. 1997]
Solo. : Proceed as usual. [Ans. l x 10-51
25. The conductivities of aqueous NaCl, KC!, K2S0 4 at 25 °C at infinite dilution are
12o.45, 149.84 and 306.60 ohm- 1.cm 2.mole- 1 respectively. If 'A +at 25 °c is 50.11
Na
ohm- 1.cm2 .mole- 1, calculate the value of t _ in Na 2S0 4 at 25 °C.
so4
[1.1.T. M.Sc. Admission Test 1996, Kharagpur]
Solo. : Ao Na2S04 = 2A oNaCl + Ao KzS04 - 2A oKCI
= (2 x 126.45 + 306.60 - 2 x 149.84) ohm- 1.cm2.mole- 1
= 259.82 ohm- 1.cm 2 .mole- 1.
= A Na1S04
0
-
- A +
Na

= (259.82 - 2 x 50.11) ohm-1.cm 2 .mole-1

'A z
= ~-L._ = 159.6 = o.6 1.
A~a 2 so 4 259. 82

26. A conductivity cell was filled with 0.05 mole.dm- 3 NaOH and the resistance was
40 Q. If the cell constant is 0.4 cm- 1, find out the molar conductivity of NaOH at that
concentration. [M.Sc. Admission Test 1992, 1.1.T. Bombay]
Solo. : We know that,
A
Conductance (G) = I( x -
I
KX l
or,
R
= II A
II A
or, I(
= R

By the probem, K = ~ = 10- 2 ohm- 1.cm- 1.


40
IOOOK
Now, ANaOH = N

IOOOK IOOOx 10-2


= z.c = -----
lx0.05
= 200 ohm- 1.cm2.mol-I.

27. Calculate the equivalent conductance of 0.001 (M) H2 S04 at 298 K. The measured
conductance of the solution was found to be 8 x 10- 4 ohm-1.
[M.Sc. Admission Test 1997, 1.1.T. Bombay]
A
Solo. : We know that, G = K x - .
' l

Let us assume that, .!.... =l cm- 1.


A
398 PROBLEMS ON PHYSICAL CHEMISTRY

IOOOK IOOOK I 000 x 8 x 10-4


=--=--=
N Z.C 2x0.00I
= (4 x !000 x 1000 x 10--4 ) ohm-I.cm 2 .gm-eq-I
= 400 ohm-I .cm 2.gm-eq-I
= 800 ohm-1.cm2.mole-1•
28. In a moving boundary experiment wih 0.01 mole LiCI, the boundary in a tube
having a cross-sectional area of 0.125 cm 2 moves through 7 .3 cm in 1490 sec when a current
of 1.80 x 10- 3 ampere is used. Calculate t+. [V.U. 2005; C.U.(H) 2002]
Soln. : By the problem, a = 0.125 cm 2 , l = 7.3 cm.
Time(t) = 1490 sec.
i = 1.80 x l0- 3 ampere .
.. charge in coulomb = (l .80 x l 0-3 x 1490) = 2.682.
laFN .
. . t+ =- -
lOOOQ
(for this case N = C)

= (7.3x0.125x96484xO.Ol) = 0. 32826 = 0.33.


lOOOx 2.682 . ·
29. Determine the transference number of H+ and c1- ions from the following data
obtained by moving boundary method using CdCl 2 as the indicator electrolyte. Concentration
ofHCI soln. is 0.1 mol.dm- 3 , mass of Ag deposited in the coulometer is 0· 1209 gm, movement
of boundary = 7.5 cm and cross-sectional area of the tube is l.24 cm 2. [V.U. 2003]
Soln. : In this problem,
0 209
charge sent is ·1 faraday. Then proceeding as before (Problem no. 28) the answer
107.8
is t+= 0.83.
30. The transport number of Na ion in NaCl is 0.385 and the equivalent conductance
at infinite dilution of NaCl is 126.5 in the usual unit at 25 °C. Estimate the distance traversed
in one hour by Na ion in a very dilute solution kept in a cell with electrodes I cm apart,
when the potential difference of 30 volts is applied between the electrodes. [V.U. 2001]
Soln. : Proceed as usual. [Ans. 54.5 cm/hour]
31. The specific conductance of pure water is 38.4 x 10-9
ohm-I .cm-I at 18 °C. The
equivalent conductance at infinite dilution of H+ and OH- are 315.2 ohm- 1.cm 2.gm-eq- 1 and
173.8 ohm-I .cm 2 .gm-eq- 1 respectively. Calculate the ionic product of water at 18 °C. Is this
w.ater alkaline with respect to water at 25 °C (pH = 7)? Explain with reasons. [Given : 1
litre of water at 18 °C weighs 998.50 gm] [C.U. 2000]
Soln. : Given ~ 0 = 38.4 x 10- 9 ohm-I .cm-I
2
A°H+ = 315.2 ohm-I .cm 2 .gm-eq-I
A0 = 173.8 ohm-I .cm 2.gm-eq- 1
ow
By the problem,
I litre of water = 998.50 gm
ELECTROLYTIC CONDUCTANCE 399
~~~~~~~~~~~~~~~~~~~~

!OOOK
Now, A = - - -
N
A = !OOOK
z.c
9
C = _IOO_O_K = _IOO_O_K = 1000x38.4xl0- = 7 .853 x io-8.
N A0 (315.2+173.8)
It is the cone. of H+ and OH- ions.
Actually, water weakly dissociates into H+ and OH-.
A lOOOK I OOOK
So, CH+ = a.CH 0 = -CH 0 = CH 0 = = 7.8 x 10-8.
0
2 A 2 CHzO A 0 2 A0

998.5
(Here, CH 0 = = 55.47)
2 18
So, Kw := C 2 + = (7.8 x 10- 8 ) 2 = 6.084 x 10- 15 = 6.1 x 10- 15 .
H

pH= -log 10 CH+= -log 10(7.8 X 10-8) = (-log 107.8 + 8) = 7.108.


At this temperature (18 °C) the solution will be neutral although pH= 7.108.
Since the decomposition of water is endothermic, so (Kw)H 0 will increase with increase
2
in temperature and will decrease with decrease in temperatu re.
32. The mobility of NH: ion is 7.623 x 10-8 m2 .v- 1.sec- 1. Calculate :

(i) the ionic conductivity of the NH: ion.


(ii) the velocity of the ion if 15 volts are applied across electrodes 25 cm apart.
(iii) the transport number of the ions in NH4C2H 3Q2 solution if the mobility of
C 2Hp 2- ion is 4.239 x 10- 8 m2.v- 1.sec 1. [C.U.(H) 2003]
Solo. :
(i) We know that A+ = Fu+
By the problem, A.+ = (96484 x 7.623 x 10-B) C.m2.v-1.sec- 1
= 7.354 x 10-3 ohm- 1.m2.

(ii) Now, v+ = u+. ii<I> = (7.623x10-8 x___!2__)m2.v-I.secl x volt


l 0.25 m
= 4.574 x 10-6m.sec-1•

(iii) By the problem, t+ = ~±__ = ( 7.623 ) = 0.643.


U+ +u_ 7.623+4.239
:. tc H - = (I - 0.643) = 0.357.
2 302
33. A AgN0 3 soln. was electrolysed in a transport cell with silver electrodes. The
anode compartment contained 0.1210 gm of silver ion before electrolysis and 0.1652 gm.
after electrolysis. During eletrolysis 0.0854 gm of Ag was deposited in the cathode. Calculatte
t+ and t_. [C.U.(H) 1973]
400 PROBLEMS ON PHYSICAL CHEMISTRY

Solo. : We know that,


107.8 gm silver is deposited at cathode for the passage of I faraday .
0 0854
. . 0.0854 gm silver is deposited for the passage of · faraday.
107.8

So the charge sent = 0 ·0854 faraday. The gain in weight of Ag in the anode chamber
107.8
0.0854
for the passage of - - - faraday is, by the problem = (0.1652 - 1.1210) gm
107.8
0.0442
= 0.0442 gm = - - gm-eq.
107.8
We know that, for reactive electrodes, the passage of I faraday through a soln. leads
to an increase oft_ gm-eq. of cation (and anion and salt) in the anode chamber. Thus, the
gain in gm-eq. of Ag in the anode chamber for the passage of I faraday is :
0.0442
107.8
0. 0854
= 0.o. 0442
0854
= 0.5175644 = 0.52.
107.8
Thus, t_ = 0.52; t+ = (I - 0.52) = 0.48.
34. A CuS0 4 soln. was electrolysed in a transport cell between copper el~ctrodes. The
anode compartment contained 0.1325 gm of copper ions before electrolysis and 0.2921 gm
afterward. During electrolysis, 0.069 gm. copper was deposited at the cathode. Calculate
the transport number of Cu +2 and S04 ions.
Solo. : Proceed as Problem no. 33. [Ans. : t+ = 0.35, t_ = 0.65)
35. A AgN0 3 solution was electrolysed between silver electrodes in a transport cell.
The anode compartment contained 0.224 gm of AgN0 3 before electrolysis and 0.2921 gm
AgN0 3 afterwards. The same amount of charge deposits 0.0251 gm Cu in a copper
coulometer connected in series. Calculate t+.
Solo. : [Proceed as Problem no. 34]
63.6
E~q. wt. o f copper - -
2

('h . -
. arge sent 1s - · -x
0.-0251 --2 an d m . anod e case no. o f gm-eq Ag increase
. th"1s reactive . d
~ 63.6
(0.2921 ·· () 224)
!ll ~nc .tii"l'.c ,··.11npartment 1s
!69.8
:. [+' = 0.49.
36. A AgN0 3 !>oluuon contarnrng O.UO 739 gm ot AgN0 1 per t;;m of water is electrolysed
between silver electrodes. At the c11d ,;I the c.\µe11111t'nl tUl78 gm di ,\g 1~ found\\' have
been deposited on the cathode. The anode ~oln . Jfter dcctruly~i~. •~ found to cumain
23. I4 gm of water and 0.236 gm of AgN0 3. Calculate L+ and t . [C.Ll.(H) 1982]

Solo. : By the problem, the charge sent is 0.07S faraday.


107.8
ELECTROLYTIC CONDUCTANCE 401

The anode soln. contained (23.14 x 0.00739) gm= 0.1710046 = 0.171 gm AgN0 3
before electrolysis and 0.236 gm AgN0 3 afterwards. Hence, the increase in wt. of AgN0 3
065
· is (0.236 - 0.171) gm = 0.065 gm. That is O. gm-equivalent [eq. wt. of AgN0 3 = 169.8]
169.8 .
078
of AgN0 3 increase takes place in the anode chamber for the passage of O. faraday. We
107.8
know that, for the passage of I faraday there is an increase oft_ gm-eq. in the anode chamber.
Thus t- = 0.065I169.8 = 0.065 x 107.8 = 0.05290537 = 0.53.
' .0.078/ 107.8 169.8 0.078
t+ = (I - L) = (I - 0.53) = 0.47.
37. A CuS04 soln. containing 0.0821 gm of CuS04 per gram of water is electrolysed
between copper electrodes. At the end of the experiment, 0.0692 gm of Ag is found to
have been deposited in a silver coulometer connected in series.The anode is found to contain
50.44 gm of water and 4.1745 gm of CuS04 . Calculate the transport number of copper
ion.
Solo. : Proceed as Problem no. 36. [Ans. t+ = 0.35)
38. In a Hittorf cell with silver electrodes is placed a 0.01 mola1AgN0 3 solution !ind
a silver coulometer is connected in series. Electrolysis is continued for a time.Analysis,
after electoysis, shows
(i) 32.10 mg of Ag is deposited in the coulometec
(ii) 20.09 gm of soln. of the anode chamber of the Hittorf cell contains 39.66 mg of
Ag. Calculate t+. [C.U.(H) 1977]
Solo. : By the problem,
0.0321
charge sent = - - - faraday.
107.8
The solution contains AgN0 3 apart from water. Now, the 20.09 gm soln. from anode
0.03966
chamber contains 0.03966 gmAg, i.e., x 169.8 = 0.06247 gmAgN0 3 = 0.0625 gm
107.8 .
AgN0 3
[since 107.8 gm Ag =169.8 gm AgN03]
Thus, the amount of water in the anode chamber is (20.09 - 0.0625) gm = 20.0275 gm.
Now, the solution was 0.01 molal. That is, 1000 gm water contained (0.01 x 169.8) gm
AgN0 3 .
&
Th ere1ore, 20 .0275 gm water contame . d 0.01x169.8 x 20.0275 = 0.0340 I gm before
1000
electrolysis. Hence, the increase in wt. ofAgN0 3 in the anode chamber is (0.0625 - 0.0340 I)
gm = 0.02849 gm.
For reactive anode, there is an increase ofL gm.eq. of salt, i.e.,AgN0 3 in anode chamber
for the passage of I faraday.

Here, &1or th e passage o f - - - F, th e increase


0.0321 . . gm-eq. o f AgN0 .m the anode
m 3
. 107.8
0.02849
chamber is gm-eq.
169.8

Prob. Phy. Chem.-26


402 PROBLEMS ON PHYSICAL CHEMISTRY

.. L = 0. 02849 I 169. 8 = 0. 028~ x 107. 8 = 0. 563467 = O.S6 .


0.0321I107.8 169.8 0.0321
.. t = (I - 0.56) = 0.44.
+
39. In a Hittorf method experiment, an exactly 0.2000 molar CuSq soln. was
electrolysed between copper electrodes. After the experiment 18.2170 gm of the cathode
soln. was found to contain 0.2207 gm of Cu. Also 0.0202 gm of Ag was deposited during
the experiment in a silver coulometer connected in series. Calculate t+. [C.U.(H) 1985]
Solo. : Proceed as Problem no. 37. [Ans. t+ = 0.33]
0.0202
In short : Charge sent =- -
107.8
faradays

Cathode soln. 18.2170 gm contains 0.2207 gm Cu, i.e., 0.5538 gm Cusq. So weight
of water is (18.2170 - 0.5538) gm = 17.6632 gm.
1
~·:~
2
Initial CuS04 is (0.2 x 159.6) x gm = 0.5638 gm.

(0. 5638 - 0. 5538) gm


Therefore, loss in gm-eq. = -'---------
159.6gm I 2
Thus, i- = 0.67, t+ = 0.33.
40. A soln. of CuS04 of 0.02(M) was electrolysed between copper electrodes in a Hittorf
cell and a silver coulometer is connected in series.Analysis, after electrolysis, showed that-
(i) 44.2 mg of Ag was deposited in the silver coulometer
(ii) 30.078 gm of the soln. in the cathode chamber contains 0.02982 gm Cu.
Calculate the transport number of cu+ 2 ion.
Solo. : Proc~ed as Problem no. 38. [Ans. ~+ = 0.36]
41. A solution containing 3.02% by wt. of CuS04 was electrolysed using a Pt-cathode
and Cu-anode. After electrolysis, the cathode solution was found to weigh 54.7 gm and
to contain 0.409 gm copper. The increase in weight of the cathode was 0.408 gm. Calculate
the transport number of cu2+ and Soi- ions (Atomic weight of Cu = 63.6).
63 6
Solo. : IF deposit · gm = 31.8 gm of Cu.
2 .
0.408
.. charge sent = - - F (by the problem)
31.8
The cathode solution contains CuS04 apart from water. Now,
63.6 gm of Cu = 159.6 CuS04
gm o fc u =
0409 0.409xl59.6 fC SO
. gm o u 4
63.6
= 1.02635 gm CuS04 .
Hence, the weight of water = (54.7 - I .02635)g = 53.674 gm.
Initial percent of CuS04 is 3.02.
Thus, JOO gm soln. contains 3.02 gm CuS04 . Hence, 3.02 gm CuS04 is present in
(100 - 3.02) gm
= 96.98 gm of water.
ELECTROLYTIC CONDUCTANCE 403

96.98 gm water contains 3.02 gm of CuS04

.
53.647 gm of water contains 3.02 x 53.647 gm = 1.671 gm o f C uSO .
4
96.98
. f 0.408 F
Hence, the loss in weight of CuS04 for the passage o - -
31. 8
= (1.671 - 1·026) gm= 0.645 gm.
Now, migration of anion out. of cathode = t- gm-eq. per faraday. Hence, the loss of
salt = L gm-eq. per faraday.
By the problem,

f or the passage of - - the Ioss o f sa lt = -


0.408F 0.645
- gm-eq.
31.8 79.8
159 6
[since eq. wt. of CuS04 = · = 79.8]
2
.. for the passage of lF the loss of salt is

=
0 645179 8
·
0 645
· = · x~ = 0.6299756 = 0.63.
0. 408 I 31. 8 79. 8 0.409
Thus, t+ = 1 - L = (1 - 0.63) = 0.37.
42. 4.29 molal FeCl3 soln was electrolysed between Pt-electrodes. After electrolysis,
50 gm soln. from the cathode is found to be 3.1 molal in FeC~ and 1.4 molal in FeC~.
Calculate the transport number of Fe3+.
Solo. : This problem can be solved by two methods :
Method I. The reaction at the cathode is : Fe3+ + e = Fe 2+. Thus, production of
1 mole of Fe2 + ion means the passage of I taractay charge. Let there be x gm of water
in the cathode chamber. Now,
4
l molal in 1000 gm = 1 mole = passage of l F l.4 molal in x gm = I. xx mole
1000
4 4
=passage of I. x x F. Thus the charge sent is l. x x F. Now initially there was 4.29 molal
1000 1000
FeCl 3 . Thus, in 1000 gm, there were (4.29 x 3) moles of chloride ion (assuming complete
· · . ) . H ence, m 12 · 87 x
. x gm, th ere were - . there f ore,
10msat1on - - mo. Ies. After el ectro Iys1s,
1000

there are 3.1 x 3 x gm (for FeC13) and l. 4 x 2 x gm (for Fe Cl2 ) m


. x gm, 1.e.,
·
1000 1000

3.1-x-Jx
+ I. 4 x 2x) mo les ct- lx
12. -
=- mo Ies ct- .
( -1000 1000 1000
For the passage of 1 faraday the cathode chamber will lose L gm-equivalent of an

.
ion. Now, loss o f anion
. from ca th ode c ham ber 1s
. (-
12.- x
87- - - l
12. - x) 0.77x
= - - moles of
1000 1000 1000
. . 0.77x f f l.4x
Cl - ion, 1.e., - - gm-eq or the passage o --F.
1000 1000
404 PROBLEMS ON PHYSICAL CHEMISTRY

:. t_ = 0. 77x I IOOO = 0.77 = 0. 55 .


I. 4x I IOOO I. 4
Thus t+ = (t - t_) = (I - 0.55) = 0.45.
Method 2. Considering cation, for the passage of I faraday, I gm-eq. of Fe+3 will
4
be lost, but t+ gm-eq will move in. Now, the loss of Fe3 + should have been I. x gm-
1000
-'+ . Thus, there should be (4.29-1.4) x
eq, as IF reduced I mole of Fe- 2.89x
=- - mo Ie
1000 1000

Fe 3+. But there is


3
Ix
.
IOOO
3 2 89
mole of FeCl3 (3.1 molal). This increase, i.e., ( · 1- ·
1000
)x =
21
O. x mole is due to migration oft+ gm-eq of Fe3+ to the cathode chamber. Each Fe3+
!000
0.21xx3
ion carries 3 units of chaige. Hence, the increase in gm-eQ. is . gm-eq.
1000
3
0.21xx-- 0.21x3
Hence, t+ = IOOO = ___ = 0.45 .
l.4x 1.4
1000
43. A 3.33 molal soln. of FeC13 is electrolysed between Pt-electrodes.After electrolysis,
the soln. from cathode chamber had a molality of 2.6 in FeC~ and 0.86 in FeCl 2. Calculate
t+.
Soln. : See the previous problem to solve it. [Ans. 0-45]
44. A 2 molal solution of FeCl3 is electrolysed between Pt-electrodes. After electrolysis,
the cathode compartment soln. weighing 20 gm is found to be 1.15 molal in Feq and
I molal in FeC1 2 . Calculate tthe transport number of Fe3+ and c1-. [C.U.(H) 1999]
Soln. : See Problem no. 42. [Ans. t+ = 0.45]
45. An aq. soln. of CuS04 was electrolysed between copper electrodes.After the passage
of 1.35 x 10-3F charge, the 25.01 gm soln. from cathode chamber had a molality of 0.033.
The molality was initially 0.050. Calculate t+.
Soln. : By the problem, the chaige sent is 1.35 x w- 3 F.
Now, 25.01 gm soln. from the cathode chamber had a molctlity of 0.033. Let there
be x gm water.
Thus, 1000 gm water contains (0.033 x 159.6) gm of CuS04
33 xI 59 ·6 xgm of CuSO
x gm of water contains 0.0 4
1000
= 0.0052668 x gm of CuS04 .
. . by the problem,
x + 0.0052668 x x = 25.01
25 01
or, x= ·
1.0052668
= 24.87896745 = 24.88 g
m.
ELECTROLYTIC CONDUCTANCE 405

Thus, t~re be 24.88 gm of water in the cathode chamber Now there is a loss
o~ (0.050 - 0.033) =0.017 molality in CuS04 . Hence, the loss in weight of CuS04

24 88
= O.OI 7 x · x 159.6 = 0.0675044 gm.
1000
159.6
But eq. wt. of CuS04 = - - - = 79.8.
2

Hence, the loss .m number m . 0. 0675044 . B ut th e Ioss m


. gm-eq 1s . wt. o f sa It m
. th e
79.8
cathode chamber for the passage of IF 1s L gm-eq. The above loss is for the passage of
l.35 x 10- 3 F.
0.0675044179.8 0.0675044
Hence t
' -
= l.35x 10-3
= 79.8x l.35x 10-3
= 0.626607 = 0.63.
:. t+ = (I - t-) = (I - 0.63) = 0.37.
46. An aq. soln. of NiS04 was electrolysed between Ni-electrodes. After the passage
of 1.42 x 10- 3 F charge, 25.26 gm soln. from the cathode chamber was found to have
a molality of 0.034. The molality of the soln. was initially 0.051. Calculate t+.
[M. wt. of NiS04 = 154.7)
Solo. : Proceed as former problem. [Ans. 0.40]
47. A current of 10 mA was passed through a AgN0 3 soln. in a Hittorf cell with Ag-
electrodes for 80 minutes. After electrolysis, the cathod'e soln. weighed 40.28 gm and was
titrated with 86 cc of 0.0200 molar KSCN. The anode soln. weighed 40.40 gm and required
112 cc of 0.0200 molar KSCN. Calculate the transport number of Ag+.
Solo. : Here, charge sent = (10 x 10-3 A) x (80 x 60 sec)
= 48 coulombs
48
= - - F [= xF (let)].
96484
Let the weight of water in cathode bea gm and weight in anode beb gm. Now, passage
of x F will lead to increase in anode chamber by xt_ gm-eq. and decrease in the cathode
chamber by xt_ gm-eq.
The anode soln. required 112 cc 0.02(M) of KSCN = 112 cc 0.02(M) of AgN0 3
Now, 1000 cc l(M) of ~gN0 3 = 169.8 gm of AgN0 3
169.8xll2x0.02
112 cc 0.02(M) of AgN0 3 =
1000
= 0.380352
= 0.002249 gm-eq.
In anode the soln is 40.40 gm.
In the cathode, 1000 cc l(M)AgN~ = 169.8 gm of AgN0 3.
169 8
By the problem, 86 cc 0.02(M)AgN0 3 = · ~;~x0.0 2 = 0.292 gm
= 0.00172 gm-eq.
The cathode contains 40.28 gm soln.
So, water in anode = (40.40 - 0.380352)gm = 40.02 gm
and water in cathode = (40.28 - 0.292)gm = 39.99 gm.
406 PROBLEMS ON PHYSICAL CHEMISTRY

Let there be a' gm-eq. AgN0 3 initially in cathode and b' gm-eq. AgN0 3 in anode.
.. a' - x t- = 0.00224
b' + x (_ = 0.00172 .
. . adding, a' + b' = (0.00396) .
39 99
.. a' = · x 0.00396 = 0.00198
(39.99+40.02)
40 02
and b' = · x 0.00396 = 0.0019807.
( 40. 02 + 39. 99)
[Since, b' and a' are in the ratio of water in each chambei; i.e.,
b' 40.02
a'
= - -
39.99

or, b' = 40.02 a'.]


39.99
In cathode, loss in weight= (0.0Q198 - 0.00172) = 0.00026 gm-eq.
In anode, gain in weight = (0.00224 - 0.00198) = 0.00(;~6 gm-eq.

Hence, (_ = 0. 00026 x 96484 = 0.52


48
t + = (1 - 0.52) = 0.48.
48. A solution containing 0.4312 pn of LiCI per 100 gm of water was electrolysed
using Ag anode. After the passage of current (resulting in the deposition of 0.7394 gm of
Ag in a voltmeter) 123.61 gm solo. were found to contain 0.4606 gm of LiCI. Calculate
t+.
Solo. : Proceed as usual. [Ans. t+ = 0.33]
49. A 0.14941 wt% aq. KCI soln. at 25 °C was electrolysed in a transport cell using
two Ag-AgCI electrodes. After the experiment, it was found that 160.25 mg of Ag had
been deposited in a coulometer connected in series with the cell, and the cathode compartment
contained 120.99 gm of solo., that was 0.1940% KCI by wt. Calculatet+.
Solo. : Proceed as usual. [0.48]
SO. A BaCl 2 solution is placed between two parallel electrodes 8 cm apart and is
subjected to a potential difference of 4 volts by connecting the electrodes to an external
battery. What will be the electrical force on a Barium ion if the interionic attraction and
dipolar effect of solvent molecules be disregarded. [D = 80] [C.U.(H) 1976]
Solo. : By formula,
2x4
Force= - - x 1.6 x 10- 12 dynes
8
=1.6 x 10-12 dynes.
51. Jn a particular experiment on KCI the apparatus consisted of a tube of diameter
4.176 mm and it contained an aqueous soln. of KCI at a concentration of 0.021 mole/litre.
A steady current of 1.82 mA was passed and the cationic boundary advanced 3.18 mm
in 100 seconds. Find transport number of K+ and c1-. [C.U.(H) 1995, 2005]
Solo. : Proceed as Problem no. 28. [Ans. t+ = 0.48; t_ = 0.52]
ELECTROLYTIC CONDUCTANCE 407

52. If the equivalent conductances of solutions with varying concentration of NaOAC,


NaCl and HCI are plotted against '1c, the intercepts obtained are 9LO, 128 and 425
respectively in ohm- 1.cm 2.eq- 1 unit at 25 °C. If the resistance of 0.02 molar solution of
HOAC in a cell with cell constant 0.2061 cm- 1 be 888 ohms at 25 °C, what is the degree
of dissociation of the acid at 25 °C. (C = concentration) [C.U.(H) 1995]
Solo. : Proceed as Problem no. 19. [Ans. 0.029]
53. The transference number of the ions of l .O(N) KCI were determined by moving
boundary method using a solution of0.80(N) BaC~ as the following solution. Using a current
of 0.0142 A the time required for the boundary sweep thi·ough a volume of 0.1205 cc. was
1675 sec. What are the transport numbers of K+ and c1- ions? [V.U. 1999]
Solo. : Proceed as Problem no. 29. [Ans. t+ = 0.49]
54. In between two platinum electrodes 8.0 cm apart and with a potential diference
of 4.0 volts was placed in a very dilute aqueous solution of silver nitrate. The Ag+ and
N0 3- ions are found to move with velocities 3.2 x I o-4 cm/sec and 3.7 x 10- 4 cm/sec
respectively. Find the equivalent conductance of AgN0 3 solution and the transport number
of anion. [V.U. 1998]

Hints : v± = .u~. -L\<I>1-


0 0

A = A
0
Ag+
+A NO) = F(u+ + u_)

u_
t - --- [Ans. 133.15; 0.53]
U+ +u_

55. The resistance of a conductivity cell filled with 0.02(N) KCI at l 8°C is 17 .60
ohms. The resistance becomes 91.8 ohms when it is filled with 0.1 (N) CH_iCOOH at the
same temperature. If specific conductance of 0.02(N).KC1 is 2.399x 10-3 ohm-1.cm-1 and
ion conductance at infinite dilution of Wand CH3coo- ions are 315 ohm--' .cm2.eq-I and
35 ohm- 1.cm 2 . eq- 1, then calculate the degree of dissociation of CH_iCOOH.
[V.U. 20071

Hints : _!_ = " A


R l
cell constant = 0.0422 cm-1.
1000
KcH3COOH 1000x4.59x 10-4 - 4 59. h -I 2 -I
= - . o m .cm .eq .
c 0.1
J

. . degree of dissociation = ex = ~ = 4 59
· = 0.013.
A0 315+35
56. At 18 °C, the mobility at infinite dilution of the ammonium ion an~ chlorate ion·
are 6.6 x 10-4 cm 2.v- 1.sec- 1 and 5.7x10-4 cm 2 .v- 1.sec- 1 respectively. Calculate A m of
0

ammonium chlorate and transport number of two ions. [V.U. 2007]

Solo. : /~o\, = Fu+ = 96484 x 6.6 x 10-4


• cm2
coulomb x - -
NH! V.sec
= 63.68 ohm- 1.cm2.mole- 1.
= Fu.. = 96484 x 5.7 x lo-4
0

A
c103
= 54.99 ohm- 1.cm 2 .mote- 1.
408 PROBLEMS ON PHYSICAL CHEMISTRY

= AoNH + + Ao = 118.67 ohm-1.cm2.mole- 1


4 c103
u- 6.6
= -- - = = 0.54
U+ +u_ 6.6+5.7

t _ = I - 0.54 = 0.46.
00 3

57. The ionic conductance of Li+ and K+ are 38.7 mhos.cm 2/gm-ion and 73.5 mhos.
cm 2/gm-ion respectively. How long would it take for an ion to move from one electrode
to another (2 cm apart) of a conductivity cell where a potential diference of I 0 volts is
applied between the electrodes. [B.U. 1991]

Soln. : Ionic mobility of Li+= uu+ = ~ = 4.01 x IO-;.~cm 2 .volr- 1 .sec 1 .


96484

uK+ = ~ = 7.62 x I0-4 cm 2.volc 1.sec 1.


96484

u+ .1<j> = 4.01 x 10-4 x .!.Q = 2.01 x 10-3 cm/sec.


l 2
10
vK+ = 7 .62 x 10-4 x - = 3.81 x 10-3 cm/sec.
2
2 2
So Li+ will take = 997.51 sec and K+ will take = 524.93
2.01x10--3 3.81 x IO- 3
sec to cover 2 cm.
58. The specific conductance of pure water at a certain temperature T was found to
be 0.6x w- 7 ohm- 1.cm- 1. Find the pH of water at this temperature if A. H+ =350 0

ohm- 1.cm 2 and A. = 200 ohm- 1.cm 2. [B.U. 1998]


ow
1000K
Soln. : CH+ = 0 0 = l.091 x 10-7
A. H+ +A. OH-
pH = -log CH+ = 6.96.
59. The equivalent conductance of a O.Ol(N) CaC~ is 130.36 ohm- 1.cm 2.gm-eq- 1. What
will be the molar conductance in S.I. unit? [B.U. 2000]
Soln. : O.Ol(N) CaC1 2 = 0.005(N) CaC12
Thus l(N) CaC12 = (0.005/0.0l)(M) CaC12
Acacl = 130.36 oh'm- 1.cm 2.gm-eq- 1
2
005
Am (in SI) = 130.36 siemen (I 0-2m) 2 x ( O. mole)-!
0.01
= 0.026072 siemen.metre2.mole-I.
60. A conductivity cell of cell constant lcm- 1 shows a resistance of 6667 ohms when
filled with 0.001 (M) KCl solution at 25 °C. The same cell records a resistance of 2353
ohms when filled with 0.001(M) HCl solution at 25°C.
(i) Calculate equivalent conductance values for KCI and HCI solutions.
(ii) Calculate ion conduction of W assuming that K+ and c1- have same mobility;
consider the solutions to be infinitely dilute.
ELECTROLYTIC CONDUCTANCE 409

(iii) How far will H+ move in 10 seconds when a potential difference of 2 volts is applied
between two eletrodes placed 2 cm apart? [B.U. 2000]

Solo. : = 1(-----
R cell constant

cell constant
:. I( =
R

Thus, l(KCI = -1- = 1.49 x 1o- 4 mho .cm- 1• -


6667

lOOOK 1000xl.49xl0-4 h _
Hence, AKCI = -- = O. = 14999
. m o.cm 2.gm-eq 1
C 001

lOOO x _I_
A HCI -- 0.001 1
mh o.cm .gm-eq-.
2
2353 = 424 .99
= A°Kci = A.>
0

By the given problem, AHCI + Ac -


1

and A.,K+ = A.~1-.


o 149:'§9 2 . I
Hence, A.
c1-
= ---
2
= 74.99 mho.cm .gm-eq- .

Thus, AH+
0
= Attci 0
- A.c _
1
= 350 mho.cm2.gm-eq-1 .
utt+ = -350- = 3.63 x 10-3 cm 2 .volc 1.sec- 1 (A+ = Fu+)
96484

vH+ = uH + ~<I> = 3.63 x 10-3 x ~ = 3.63 x 10-3 cm.sec-I.


I 2
So H+ will move in 10 sec, 3.63 x 10-3 x 10 = 3.63 x 10-2 cm.
61. The equivalent conductance of a weak monobasic acid at infinite dilution is
388.5 mho.cm 2.eq-l at _25 °C. Find the specifiic conductance of 0.1 molar solution the
degree. of dissociation of which is 6%.

Soln. :)Degree of dissociation a = ~.


I A0
6
Here, a = -100 = 0.06, A0 = J88.5 mho.cm 2 .eq-l.

Thus, A = 23.31 mho.cm 2 .eq- 1.

Hence, I(= A_£ = 23.31 x ~ = 2.33 x 10-3 mho.cm-1.


' 1000 1000
62. Calculate the velocity of Li+ in 0.1 (M) LiCl when a current of IA is passed through
a column of solution with cross-sectional area 4 x 10-4 m 2. Given that equivalent con-
ductances of u+ and c1- are 4 x 10-3 and 7.5 x 10-3 respectively in mho.m2.eq-l.

Hints. : vLi+ = uLi+ ~¢>


410 PROBLEMS ON PHYSICAL CHEMISTRY

A+
Now
'
uL1.+ = _Jj_
F

1
and flcj> = i R = i -AK
- .

AL,+ . l
H ence, vLi+ = - - x 1 - -
F AKxl
10- 3 1(
Again, A = --, where, I\ in mho.m- 1 and C in gm-mole per litre.
c
3 3 3
= A.u+'X ixlq- =· 4x!0- x l"xl0-
Thus, vLi+
F ACA 96484 4xl0-4xo.1x(A +A)"
Li+ Cl-
= 9.013 x 10-s m.sec- 1•
63. Equal volumes of 0.1 (M) HCI· and 0.1 (M) NaOH are mixed so that specific
conductivity of 0.1 (M) solution of NaOH decreases to 0.006 from 0.022. On addition of
a further volume of HCl solution equal to that of the first portion added, specific conductivity
rises to 0.0170. Calculate :
(a) A for NaOH, (b) A for NaCl, (c) A for HCI, .(d) A for tt+ and OW.

I
S on.: .(a) A
NaOH
= 1000 O.I
x 0. 022
= 220 m h o.cm 2.eq-.1
IOOOx0.006
(b) CNaCI = 0·05(M), ANaCI = 0.05
. = 120 mho.cm2.eq-I.
(C) K + K = A NaCl CNaCl + A HCI CHCI
NaCl HCI l OOO I OOO

0.0170 = 120x0.033 + AHCI x0.033


1000 1000
Since, V x 0.1 = 3V x CHCI
.. CHCI = 0.033
and 2V X 0.05 = 3V X CNaCI
.. CNaCI = 0.033
AHCI = 395.15 mho.cm2.gm-eq-I.
ANaOH + AHCI - ANaCI
= Att+ + Aow = 220 + 395.15 - 120 = 495.15 mho.cm 2 .gm-eq- 1.
64. A solution of O.Ol(M) LiN0 3 and 0.02 (M) KCI is placed in a conductivity cell
whose electrodes are 4 cm apart. A potential difference of 8 Vis applied between the electrodes
and the resistance of the cell is 200 ohms. Calculate tLi+ and current carried by Li+. Given,
A.Li+ = 38.7, AN _
03
= 71.4, AK+ = 73.5, Ac 1- = 76.3.
Soln. : Current carried by u+ = /Li+ = C+ Fv+ A,
where c+ = cone. of u+ in gm-eq per cc
v+ = velocity of u+
ELECTROLYTIC CONDUCTANCE 411

A = cross-sectional area of electrode.


d<)>
Now, V+ = U+ -
1
-,

U+ = mobility of Lj+
d<)> = potential difference
I = distance between the electrodes.

Again, d<)> = _!!:_.


KA
I I
Hence, /Li+ = C+ Fu+ - = C+ A.+-.
K K

By problem, A.Li+ = 38. 7, A.N _ = 71.4, \:+ = 73.5, A.c _ = 76.3 mho.cm- 1•
03 1

Thus, K
= ACLiN03 + ACKCI
1000 1000
0.01(38. 7 + 71.4) 0.02{73.5 + 76.3) 4.1 I
= 1000
+
1000
= --mho.cm-
1000

I = d<)> = _8_ A.
R 200

Hence, /Li+= O.Ol x 38.7 x 8 x lOOO = 3.78 x 10-3 A.


1000 200x4.l

tLi+ =
Iu+ =. C+A.+ =
I K
~
K
= 0.943.

65. The resistance of a conductivity cell is 520 ohms when filled with 0.1 (M) CH 3COOH
and drops to 122 ohms when enough solid NaCl is added to make the solution 0.01 (M)
in NaCl as well. Calculate the cell constant and concentration. of H+ in the solution. Given,
ANaCI = 126, Actt COOH = 386.
3
AC 126x0.0l
Solo. : KNaCI = 1000 = 1000

By problem, RCH3COOH = 520 ohm


Rctt3COOH + NaCl = 122 ohm.
l l .
Thus, = Kctt cooH K (where K = cell constant)
520 3
l l
and 122 = ~ix K .
520
Hence, - =
122 KcH3COOH KcH3COOH

3
:. ~H cooi-t = 3.86 4
X 10- ohm- 1.cm- 1. .

Hence, cell constant = K = 3.86 x 10- 4 x 520 = 0.2008 cm-1•


412 PROBLEMS ON PHYSICAL CHEMISTRY

1000Kctt3coott
Now, ACH3COOH = 386 = C

Hence, C = l OOO x 3· 86 x l 0-4 = I .00 x 1o- 3 (M).


386
Thus, concentration of H+ = 1.00 x 10-3 (M).
66. The resistance of a conductivity cell is 220000 ohms with water, 100 ohms with
0.02 (M) KC!, I 02000 ohms with water saturated with AgCI at 25 °C. Neglecting the variation
of A with concentration, calculate :
(a) cell constant
(b) specific conductivity of the saturated solution of AgCI and
(c) solubility, product of AgCI at 25 °C.
Given, AAgCl = 126.8, AKC! = 138.~ at 25 °C.
1
Hints : 220000 = - -K, where K =cell constant
Ktt20

100 = _I_K
KKCI

102000 =
KAgCl+H20

138.3x0.02
KKCl = = 2.77 x 10- 3 ohm- 1.cm- 1
1000
Cell constant K = 2.77 x 10- 1 cm-I.
. -6
"HzO = 1.26 X 10
"AgCl + HzO = 2.72 X 10-6, KAgCI = 1.46 X 10-6

C _ C _ I000 x I. 46 x 10-6 = l.1 5 x _ .


10 5
Ag+ - Cl- - 126.8
Hence, solubility product = 1.33 x 10-10.
67. Resistance of a conductivity cell is 7000 ohms with 0.1 (M) KCI, 2600 ohms with
0.1 (M) KCI and 0.2 (M) NaCl. Given that for 0.2 (M) NaCl, t + = 0.4, Ac 1- =75. Calculate
Na •
AKCI·

R
=K x cell constant.
ELECTROLYTIC CONDUCTANCE 413

68. The molar conductances at infinite dilution for BaCl 2, KCI, K 2S0 4 and c1- are
280, 150, 300 and 76n- 1m2mo1-J respectively. Calculate the transport number of Ba2+ in
BaS04 solution at infinite ~ilution. [JAM 2011]
Hints : BaCl 2 = Ba+ 2 + 2c1-

A-m.Bac12 = Am,Ba+2 + 2 A m. c1-

Am,KCI = Am,K+ +Am,Cl-


K2S04 = 2K+ +so42
Am K1S04
·
= 2A m,K+ +A m,S0-24
BaS0 4 = Ba+2 +so42

t _ A. m,Ba +2
= 128 = 0.46.
Ba+2 - A +A 280
m.Ba +2 m.so;;2

69. In a Hittbrf experiment to determine the transference numbers in KC! solution,


the following data were obtained. Mass of both the cathode solution and middle portion
120.99 gm; per cent KC! in cathode portion 0. 19398, per cent KC! in the middle portion
0.14948. Calculate t+ if 0.16034 gm of Ag was deposited in silver coulometer connected
in series. Given that molecular weight of KC! = 74.6 and that of Ag is 107.8.
. 0.16034
Solo. : The charge sent = F.
107.8
Again, let there be x gm of water in the middle chamber.
By the data, in the middle chamber KC! present per gm of water
0.14948
= = 1.49714 x 10-3.
100-0.14948
Hence, x gm water contains 1.49714 x 10-3x gm KCI.
So mass of middle chamber ; (x + 1.49714 x 10-3x) gm
By the problem,
x(l + 1.49714 x 10-3) = 120.99.
x = 120.8091.
So total amount of KC! in middle chamber= (120.99 - 120.8091) gm= 0.1809 gm.
The cathode chamber also contains 120.8091 gm of water.
Thus, after electroly.sis amount of KCI present in cathode chamber =
0.19398
x 120.8091 gm = 0.2348 gm.
100-0.19398
Hence, increase of KC! in cathode chamber = (0.2348 - 0.1809) gm.
= 0.0539 gm
0.0539
= gm mole = 7.225 x 10-4 gm.mole.
74.6
414 PROBLEMS ON PHYSICAL CHEMISTRY

. mcrease
Th 1s . . d ue to passage o f 0. ! 603 4 F -- 1.4874 x 10-3 F.
. KCI 1s
m
107.8

7 225 10
Hence, t+ = · x -4 = 0.48.
I.4874x 10-3
Thus, t_ = i - t+ = 0.52.
CHAPTER12

IONIC EQUILIBRIUM

Required Formulae
1. Ionic product of water (Kw) is
Kw = aH+ aOH-'
where aH+ is the activity of H+ ion and a 0 H- is the activity of OH- ion.
2. pH = -log aH+
C+ C+
For dilute solutions aH+ = + . Then pH = - log+,
CH+ CH+

where C 0 + = 1 (M).
H
However, to save both space and time we willskipthedenominator. Thus, pH=- log CH+.

£..i cz
3. Ionic strength (µ) = l2 ""' 2
I I ,
i
where C; is the concentration of ith ion in moles.litre-I
and Z; is the valency of ion.

4. Mean ionic activity coefficient ("{±) is log y± = -AZ+ z_ .Jµ + Bµ.


A = 0.51 for water at 25 °C. Normally B is much smaller than A and neglected.

5. pH of weak acid = lpKa - l Iog C,


2 2
where pKa is -log 10 Ka, where Ka is the dissociation constant of weak acid and C is the
concentration of the acid in molarity.
6. If Kb be the dissociation constant of weak base, pH of the weak base

= 14 - (i pKh -ilogC} where Kw has been taken as J0-' 4 .

7. pH of a buffer of weak acid and its salt = pKa + log C.rn11


Cacid.

415
416 PROBLEMS ON PHYSICAL CHEMISTRY

8. pH of a buffer of weak base and its salt == 14 - (PKh +Jog C.,a11 )· Chase

9. pH of a salt of weak acid and strong base == 7 + lpK0 + l log C.


2 2
10. pH of a weak base and strong acid == 7 - tPKb - tlog C.
11. pH of a salt of weak acid and weak base == 7 + 2l pK0 - 2l pKb.

In all these cases [from 9 to 11] 7 is really tPKw. It is 7, because Kw is taken as 10- 14 .

12. Solubility product of a salt of M_0y == XM0 + + YAh-, so that Xa == Yb.

For a dilute solution, activity is replaced by concentration.


13. The activity of electrolyte MxAy as_ a whole,
x y
a 2 == a+ a_,
where a+ = C+"f+
a_ ;:: c_ y_.
14. Mean ionic activity,
I I
a±;:: ~ = \[c;c:r!r~ ;:: (c:cnx+y .(r!r~)x+y'
where X + Y == v == total number of ions resulting from I molecule of the electrolyte.
Now, C+ == XC and C_ == YC,
where C = molarity of the electrolyte and mean ionic activity coefficient of the electrolyte
I
;:: 'Y± = [(Y+)x(y_)Y]x+Y.
I
Thus, a± = [(xxyr)]x+r Cy±.
Short. Questions
1. Explain-"The neutralization of all bases by acids do not necessarily occur at
pH 7."
Ans. : Neutralization is the reaction between an acid and a base. The product is a
salt. At the equivalence point, no excess acid or base is present and it is thus expected
that the pH may be 7. However, due to hydrolysis, excess acid or base may be produced
and the pH may be different from 7. For example, in case of titration of weak acid by
a strong base, th~ salt contains the conjugate strong base which accepts proton from water
producing basic solution. Similarly, in case of titration of weak base and strong acid, the
pH at the end point will be less than 7. In case of titration of weak acid and weak base,
both the cation and anion are strong conjugate base and acid respectively, and thus they
will interact with water. The pH depends on degree of interaction. However, if we consider
the neutralisation of a strong base and strong acid. th.en the salt contains cations and anions,
which are weak conjugates, and there will be no interaction and pH remains 7. Thus, as
obvious, neutralisation of all acids and bases cannot take place at pH 7.
IONIC EQUILIBRIUM 417

2. What will happen if-HCI (g) is 'passed through a. saturated solution of NaCl?
Ans. : In the saturated.' solution of NaCl, aNa+ ac1- = Ks has been reached. If HCl
(g) be passed, ac1- will be greater. To keep solubility product (Ks) constant aNa+ in solution
· will be reduced and thus NaCl will precipitate.
3. Do you expect the ·solubility of CaF2 in aqueous KN0 3 solution to be same as
lower than or higher than its solubility in pure water at the same temprature? Give reasons.
Ans. : The solubility of CaF2 will increase in a solution of KN03 . The solubility of
any substance is normally not affected by the presence of another solute. However, if the
substance is an electrolyte as is CaF2 , then another salt with uncommon ion generally
increases the solubility of sparingly soluble salt. This is salt effect.
It is well-known from Debye Htickel theory that the mean ionic activity coefficient
of a salt decreases with increase in ionic strength (logy± = -AZ+Z- {rl). If we define the
product of cone. terms as Kc• the product of activity coefficient as Kr and the product
of ionic activity term as Ks, then K5 = Kc.Kr. Since Ks is a constant and Kr decreases
with increase in ionic strength, Kc must increase to keep Ks constant, i.e., the cone. of
the ions will increase. That is why- the addition of a salt with uncommon ion will increase
the solubility of salts like CaF 2.
4. The second dissociation constant of an acid is lower in magnitude than the first
dissociation constant-Explain.
Ans. : In the first dissociation of a polybasic acid HnA, H+ and Hn-t A- 1 results with
a certain K0 value. In the second dissociation Hn_ 1A- 1 ionises to Hn_2 A-2 and H+; a positive
charge is to be separated from a negative ion. This is much difficult and CH A-2 will
n-2
.
be in smaller amount. Thus. K 2 defined by K 2 = aH+·aHn-2A-2 will be much lower than
"ll I
Hn-IA-

K1 defined as K 1 = aH+·a"n-1A-l
aHnA
5. Ionic product and dissociation constant of water are not identical-explain.
Ans. : Ionic product of water (Kw) of any aq. solution is defined as

Kw = (aH+·aow ). But the process is H20 ·....- H+ + OH-

Hence, the dissociation constant is Kd = aH+·aow


llHzO

In dilute solutions, aH+• etc. are very small. Thus, Kd = CH+·Cow.


CH20
CH 2o = lrJO = 55.56, taking density of water as l.

. · · · CH+·Cow
Hence, the d 1ssoc1at1on constant Kd =~~-.-="'-- ~.
55 5 6 55.56
Hence, Kd and Kw are different.

Prob. Phy. Chem.-27


418 PROBLEMS ON PHYSICAL CHEMISTRY

6. Phenolphthalein is not a suitable indicator for the titration of NH3 with HCI~
explain.
Ans. : An indicator changes colour roughly in the pH range (pK1n-l) to (pK1n + 1),
where pK1n = -log Kin' K 1n is the indicator constant. As phenolphthalein pK1n is around
10 (exactly 9.6), so the colour change may take place in between 8.6 to 10.6. But in the
titration of HCI, a strong acid by NH 3 , a weak base, the pH at the end point is less than
7. In titrations we allowed one drop error. Within this error range the pH ch~nges generally
between 3 and 6. Thus, phenolphthalein is not a suitable indicator for titration of HCl by
NH 3 .
7. Methyl orange is a suitable indicator for the titration of strong acid and weak base.
Explain.
Ans. : In the titration of strong acid' by weak ·base the salt produced will not be at
pH = 7 due to hydrolysis. Allowing for one drop error, at the end point of the titration
of 0.1 (N) acid by 0.1 (N) base will lead to a change of pH generally from 3 to 6 or
7 depending on Kb of weak base. Indicators with pK 1n below 7 are suitable since an indicator
changes colour roughly between pH = pK1n - 1 to pK1n + 1. Methyl orange has pKa =
3.7 (around 4) and changes colour between 3.1 to 4.4 (e}\.pected 2.7 to 4.7). Thus, indicators
like methyl orange, methyl yellow, bromcresol green, etc are suitable indicators for the
titration of strong acid by weak base.
8. You are supplied with two weak acids A and B of pka values 4.8 and 5.8 respectively.
State, with reason, which one you would prefer to prepare a buffer solution of pH 5.1.

Soln. : We know, pH = pKa + log Csa1t .


Cacid

For Acid A, pKa = 4.8.


So at pH ~ 5.1,

log Csalt = pH - pKa = 0.3.


Cacid

For Acid, B, pKa = 5.8.


Thus, at pH= 5.1,

log Csalt = -0.7.


cacid

A buffer acts well within pH =pKa ± 1 with maximum capacity at pH = pKa, where

log Csalt = 0. The given pH (5.1) falls within the range between (pKa + I) to (pKa - 1)
Cacid
for both the weak acids A and B. However, for acid A, pKa is more close to 5.1 than
that of acid B. Thus, A is preferred over B to prepare the buffer solution of pH = 5.1.
Problems
1. Calculate the pH of (a) 0.1 (N) solution of CHFOOH, (b) 0.01 (N) NHpH,
(c) 0.01 (N) NH4 CI, (d) 0.001 (N) CH3COONa, (e) 0.02 (N) CHFOONH 4 . Given that
Ka for CHFOOH = 2 x 10-5 and Kb for NH40H = 2 x 10- 5 .
IONIC EQUILIBRIUM 419

Solo. : (a) pH of weak acid = 2l pKa - -zl log C.


.. . 5 ) - 1 log0.l ] = -4. 699
pH = [ 1 ( -log2x10- .- + 0.5
2 2 2
= 2.3495 + 0.5 = 2.8495 == 2.8.
(b) pH of weak base

= 14 - (~pKb -~loge)
5
= 14 - [±(-log2xl0- )-±iog0.0l]

= 14 - 2.349 + l = 12.66.
(c) pH of a salt of weak· base and strong acid

=7 - lpKb - J..Iog C = 5.65.


2 2 .
(d) pH of a salt of ~eak acid and strong base
l
=7 + 2 pKa + -zI Iog C = 7.85.
(e) pH of a salt of weak acid and weak base
1 1
=7 + 2PKa - 2PKb = 7.
2. Calculate the pH of the solution obtained by mixing 10 ml of 0.2 (N) KOH with
30 ml of 0.1 (N) CH 3COOH. Ka = 2 x 10-5 . [C.U. 1986)
Solo. : 10 ml 0.2 (N) KOH will react with 20 ml 0.1 (N) CH 3COOH to give 20
ml 0.1 (N) CH 3COOK.
But the total volume due to mixing = (I 0 + 30) ml = 40 ml.

20x0.l = 0. 5 .
40
(30- 20.)x 0.1
40
= 0.025.

Now, pH = pKa + log Csalt = -log (2 x JQ-5) + log 0.05


Cacid 0.025
= 4.699 + 0.3010 = 5.
3. In the titration of a O.l(M) weak diabasic acid (k 1 = 10-6 and k2 = 10- 10) with
a strong base, calculate the pH (i) at the start and (ii) at the first equivalence point.
[V.U. 1999)
Solo. : (i) Let the weak dibas~ acid H2A dissociate as
H2A ~ H+ +HA-
(0.l - x) x + y x
and HA- ~ H+ + A-2
x-y x+y y
H2A dissociates to give x(M) HA- while HA- dissociates to give y(M) A-2, so that
concentration of H+ = (x + y) (M).
420 PROBLEMS ON PHYSICAL CHEMISTRY

Given initial concentration of H 2 A = 0.1 (M).

C +C C +C 2
Now, K = . H HA- and K = H A-
1 2
CH2A CHA-
Given, ·K1 = 10-6 and K2 = 10-10.

Thus, 10
-6 = x( x + y) (I)
0.1-x

10-10 = (x+ y)y. (2)


x-y
Neglecting y in comparison to x, we get y = 10-IO [from Eq. (2)]
and x 2 + 10-6x - 10-7 = 0. [from Eq. (I)]
Thus, x = 3.157 x 10-4, -3.167 x 10-4.
Negfitive value of x is neglected since it is physically unacceptable.
Thus, concentration of H+ at the start
CH+ = (x + y) (M)
= (3.157 x 10~ + 10-IO) (M) = 3.15.7 x lo-4 (M).
Hence, at the start pH= - logC +
H
=- log (3.157 x 10-4) = 3.5.
(ii) At the first equivalence point in the titration with strong base BOH, H 2A will
be half-neutralized. Thus, BOH + H 2A ~ BHA + H 20.

BHA is .a~- ~cid salt. Thus, pH = ±(pk1 + pk2 ) =: ±(:-logl0-6 - log 10-IO) = 8.

4. To 50 ml of a 0.1 (M) KH2P04 solution is added to (i) 10 ml and '(ii) 50 ml of


a 0.1 (M) NaOH solution. Calculate the pH values of the resulting buffer solution in each
=
case. (Given for phosphoric acid k 1 = 7.5 x 10-3 , k 2 = 6.2 x 10-8 and k 3 4.8 x 10- 13
at 25 °C.) [V.U. 1999]
Solo. : (i) 10 ml 0.1 (M) NaOH reacts with 10 cc O.l(M) KH2P0 4 to give 10 ml
0.1 (M) KNaHP04 .
NaOH + KH2P04 KNaHP04 + H 20
Total volume of solution 60 ml =
lOxO.l 0 1
Hence, CKNaHP04 = (M) = -·- (M).
60 6
KH2P04 remaining = (50 - 10) ml or 40 ml 0.1 (M).
40x0.l O2
Hence, CKH 2ro 4 = 60
(M) = -·- (M).
3

CKNaHP0 4 O.lx3
pH = pKa + log =- log k2 + log
6
x0.
2
CKH2P04
Given k2 = 6.2 x 10-8 , Hence, pH = 6.605.
IONIC EQUILIBRIUM 421
(ii) 50 ml 0.1 (M)' NaOH will react with 50 ml 0.1 (M) KH 2P0 4 to produce 50 ml
0.1 (M) KNaHP0 4 .
NaOH + KH2P04 ----? KNaHP0 4 + H 20
KNaHP0 4 is an acid salt.
1
We know, at stage neutralization for a poly basic acid, pH = - (PKn + pKn+l ).
nth
2
It is at 2nd stage neutralization.
I
Hence, pH = 2 (pKz + pK3)

= _!_[-log (6.2 x l0-8) - log (4.8 x 10- 13)] = 9.76.


2
5. Find the pH of buffer solution consisting of 0.2(M) NH 3 and 0.1 (M) NH4 N0 3 .
For NH3, Kb = 1.6 x 10-5 at 25 °C. What is the % of change in pH after addition of
0.01 mole NaOH to 1 litre of this buffer solution? How much pH of water would change
when same amount of NaOH is added to 1 litre of water?
Soln. : (i) Given : csalt = CNH4N03 = 0.1 (M)

cbase = CNH40H= 0.2 (M)


and Kb = 1.6 x 10-5

Now, pOH = pKb + log Crntr


chase

Hence, pOH = - log (1.6 x 10-5 ) + log ..Q.:.!_ = 4.49.


0.2
Thus, pH = 14 - 4.49 = 9.51.
(ii) When 0.01 mole of NaOH is added to 11 of this buffer solution, OH- (from
NaOH) will react with NH: to given NH4 0H.
Thus, NaO~
+ NH4 N0 3 = NH4 0H + NaN0 3
O.Ol(M) 0.0l(M) 0.0l(M)
So concentration of NH4N0 3 = (0.1 - 0.0 I) (M) = 0.09 (M).
:. concentration of NH40H = (0.2 + 0.01) (M) = 0.21 (M).

Hence, pOH = - log (1.6 x 10-5) + log O.o9 = 4.43.


0.21
Thus, pH = 9.57.

Hence, m
-10
. pH = 9.57-9.51 x 100 = 063m
c'h ange m . -10.
9.51
(iii) pH of water = 7
pH of 0.01 (M) NaOH = 14 - pOH = 14 + loge = 12.
ow
12-7
Hence, % change in pH= x 100 = 71.43%.
7
422 PROBLEMS ON PHYSICAL CHEMISTRY

6. Calculate the pH of a solution obtained by mixing 50.00 ml of 0.20 M weak acid


HA (Ka = 10-5 ) and 50.00 ml of 0.20 M NaOH at room temperature. [JAM 2006]
Solo. : HA + NaOH NaA + H 20 .
50 ml of 0.20 (M) HA reacts with 50 ml of 0.2 (M) NaA.
Total volume = -I 00 ml.
50 2
Thus, strength of NaA = x O. (M) = 0.1 (M)
100
Again, NaA is salt of weak acid and strong base. So it will undergo hydrolysis. Thus,
1 1
pH =7 + zpKa + 2 log C.

Given that, Ka = 10-5 . Hence,

pH =7 + ..!..(-logJ0-5 ) + ..!..1og (0.1) = 9.


2 2
7. A solution at 25 °C contains 0.01 molar propionic acid (Ka = l.39 x 10-5 ) and
0.03 molar HCI. Calculate a for propionic acid and pH of the mixture.
Solo. : Propionic acid ionises as
H20 + CH3CH2CH2COOH ~ H3o+ + CH3CH2C'H2coo-
o.o 1 (I-a) 0.01 a 1l.Ol a
HCl ionises completely to give 0.03 (N) Hp+.
.. CH 30+ = (0.03 + 0.01 0:)

= cHJO+ .cCH3CH2CH2COO- = (0.03+0.0la)O.Ola


CcH 3cH 2rn 2cooH 0.01(1-a)
Neglecting a in comparison to 1,
K
a
= (0.03+0.0la)O.Ola = J.39x 10-5
0.01
or, 0.01 a 2 + 0·03 a ~ 1.39 x 10-5
or, 0.01 a 2 + 0.03 a - 1.39 x 10-5 =0
--0. 03 ± ~(0.03) 2 +4x0.01x1.39 x 10-5
or, a=
2.01

Neglecting (-)ve sign, since a qmnot be negative, a = 0.0009265


2
=0.00046325
= 4.63 x 10-4.
Now, C
HJ
o+ = (0.03 + O.Ola) = 0.03 + 0.01 x 4.63 x 10-4 = 0.0300046.
pH = -log C o+ = 1.5228117 "' 1.5.
HJ
8. Calculate the pH of 0.001 CM) H2S04 (given, K 2 = I.OJ x 10- 2 )
Solo. : We know that, H2S04 is a strong acid.
HzS04 + HzO ~ HS04 + H3o+
0.001 0.001 0.001
IONIC EQUILIBRIUM 423

HS04 + H20 ~ H3o+ + so4


(0.001-x) (0.00l+x) x
Now, C = (0.001 - x)
Hso4
C + = (0.001 + x)
H)O
cso4 = x.

CH o+. cso2--
.. K1 = 3 4 .
cHso4
(0.001 +x)x
or, 1.01 x 10-2 =
(0.001-x)
or, 1.01 x 10-2 x 0.001 - 1.01 x 10- 2 x = O.OOlx + x 2
or, x 2 + 0.00lx + 0.0101 x - 1.01 x 10- 2 x 0.001 = 0
or, x2 + O.Ol I Ix - 0.0101 x 0.001 = 0

2
-0.0111 ±)(0.0111) +4x Ix 0.0101x0.001
~x= - .
2
Neglecting (-)ve sign, x = 8.4551 x 10- 4 .
:. cH)O+= 1.84551 x 10-3

pH= -loge += 2.7338845 =2.73.


H)O

9. Prepare a 100 cc buffer .solution of pH= 9. You are supplied with 0.1 (N) NH40H
and 0.1 (N) HCI. Given that Kb for NH4 0H = 2 x 10- 5 . [C.U.(H) 1972]
Solo. : The resulting solution will be a buffer of weak base and its salt, whose pH
is given by

pKw - [pKh +log Csalt ] .


Cbase
By the problem,

9 = 14 - [5 - log2 +log Csalt ]


Cbase

or, Csalt = 2.
cbase
Let us consider that x cc 0.1 (N) HCI is mixed with (I 00 - x) cc 0.1 (N) NH4 0H.
Since x cc 0.1 (N) HCI will consume x cc 0.1 (N) NH 40H, the resulting mixture will
contain ( 100 - 2x) cc 0.1 (N) NH4 0H.
xxO.I
100

(!00-2x)x0.l
and cbasc = 100
424 PROBLEMS ON PHYSICAL CHEMISTRY

Csalt X
-- = -100~--2-x ·
cbase
x
Therefore, 100 _ 2x = 2

or, x = 200 - 4x or, 5x = 200

or, x = 200 = 40 cc.


5
Thus 40 cc of 0.1 (N) HCI should be mixed with ( 100 - 40) cc 0.1 (N), i.e., 60 cc
of 0.1 (N) NH40H.
10. If the final volume is I litre, how many moles of HCI will have to be added to
500 cc of 0.1 (M) Na 2C0 3 to adjust the pH to I 0.0? (Given : for H2<.i0 3, K 1 =4.52 x I o-7,
K2 =4.69x 10- 11 )
Solo. : If the neutralisation is not complete (as apparent from pH = 10),. we have
a buffer of Na 2C0 3 and NaHC0 3 . (NaHC0 3 is an acid.)

pH = pK0 + log C'.-.1t


Cacid

or, IO =-log (4.69 x 10- 11 ) + log Cs


c.
or, log Cs = (10 + log 4.69 - 11)
ca

or, Cs = 0.469.
Ca
Now, 500 cc x (M) HCI will produce 500 cc x (M) NaHC0 3 (in total volume of I litre).

:. CN HCO
a 3
= 500x
1000
= .:!.2 (M).

Now, 500 cc x (M) HCI will react with 500 cc x (M) Na 2C0 3.
Hence, there will remain 500 cc (0.1 - x) (M) Na2C0 3.

C _ 500(0.1-x) = 0.1-x (M).


NazC03 - 1000 'l9
By the problem,

(0.1-x)/2 = 0.469
x/2

or, O. J - x = 0.469 or, 0.1 - x = 0.469x


x
or, l.469x = 0.1

or, x = _Q:.!_ = 0.068 (M).


1.469
Thus, 500 cc 0.068 molar HCI, i.e.,
0.068 x 500 0 068
---- = · mole = 0.034 mole of HCI is to be added.
1000 2
IONIC EQUILIBRIUM 425

11. The degree of dissociation of pure water at 18 °C is found to be 1. 8 x 10-9 . Find


ionic product of water and its dissociation constant at 18 °C.

Soln. : Concentration of water = 1000 = 55.56 moles/litre.


18
Thus, H20. ~ W + Off
Initially : 55.56 0 0
At eq. : 55.56 (1-a) 55.56a 55.56a
a = the degree of dissociation = 1.8 x 10- 9 . '
2
Kw= CH+.Cow= (55.56xl.8x10-9 ) = 1 x 10- 14 (molenitre)z.

F urth er, Kd = CH+·Cow [Kd = d.1ssociat1on


. . constant]
CH20
1x10- 14
= Kw = = 1.8 x 10- 16 mole/litre.
CH20 55.56

12. For the reaction Ag(CN); ~ Ag++ 2CN-, the equlibrium constant at 25 °C
is 4 x 10...: 19 . Calculate CAg+ in a solution, whkh was originally 0.1 (M) in KCN and
0.03 (M) in AgN0 3 .

Soln. : Ag+ + 2CN- 1 ~ Ag(CN);

1
For this reaction, K = x _ = 2.5 x 10 18 . The initial cone. of Ag+ is 0.03 (M)
4 10 19
and CN- is 0.1 (M). Since K is so high, the constituent in lesser amount will be completely
consumed. Now, ·o.1 (M) CN- will react with 0.05 (M) Ag+. CAg+ is in lesser amount.
So Cc~ will almost be (0.1 - 0.06) = 0.04 (M) and Ag{CN); will almost be 0.03 (M).
But some Ag+ will remain. Let the CAg+ = x.
:. cCN- = (0.04 + 2x)

and CAg(cNr = (0.03 - x).


2

. 1 - cAg+ .cCN-
2

.. K - C
Ag(CN)z

_ · x.(0.04+2x) 2
or, 2.5x10 18 - (0.03-x)

Neglecting x in comparison to 0.04 and 0.03,

(0.04)2.x
2.5x10 18 = 0.03

or x = 0.03 = 7.5 x 10-18 (M).


' 2.5x10 18 x(0.04) 2
426 PROBLEMS ON PHYSICAL CHEMISTRY

13. Ka for acetic acid (HOAc) and benzoic acid (HOBz) are 1.86 x 10- 5 and
6.6 x 10- 5 respectively. Calculate the pH of a mixture of 2 .gm of HOAc and 2 gm of
HOBz dissolved in 100 cc of water.

Soln.: By the problem, there is initially lo mole of HOAc in 100 cc (60 is them.wt.
of HOAc) .
. . t.ta l mo lan.t y = 2 x I 0
. . mt = I =a.
60 3
20 10
Similarly, initial molarity of HOBz is = 122 = 6T = b.

Now the ionisation process of HOAc is


HOAc ~ H+ + OAc-
(a - x) (x + y) x
HOBz ~ H+ + OBz-
(b - y) (x + y) y

Acetic acid breaks into H+ and OAc- with CH+ = x and benzoic acid breaks into H+
and OBz- with CH+ = y. H+ is common to both. So CH+ is (x + y).
Now, by the problem,
for acetic acid,

(x+y).x = 1.86 x 10-5 .


(a-x)
(x + y).y
Similarly, (b- Y) = 6.6 x 10-S.
Neglecting x and y in comparision to a and b respectively,
(x + y)x = 1.86 x 10-S a
and (x + y)y = 6.6 x 10-S b.

:. (x + y)x = 1.86 x 10- 5 x ~ = 6.2 x 10-6 (i)

and (x + y)y = 6.6 x 10- 5 x ¥f = 1.08 x 10-S. (ii)

Adding (i) and (ii),


(x + y) 2 = l.7 X 10-S

or, (x + y) = ~l.7x 10-5 = (4.123 x 10-3).


CH+= 4.123 x 10- 3 .

:. pH = - log CH+ = - log (4.123 x 10- 3) = 2.38.


14. 300 cc of a solution of HOAc (m. wt. = 60) containing 10 gm/I (Ka= 1.86 x10- 5 )
is mixed with 200 cc of solution of phenyl acetic acid (m.wt. = 136) containing 5 gm/I
(Ka = 5.42 x 10- 5 ). Calculate the pH of the mixture.

10
Soln. : Initial molarity of acetic acid = 60 = 0.1667.
IONIC EQUILIBRIUM 427

Initial molarity of phenyl acetic acid = 5 = 0.0368.


136
After mixing, molarity of HO Ac =· ~83 x 0.1667 = 0.1002 (M)
200
and molarity of phenyl acetic acid = x 0.0368 = 0.0147 (M).
500
Let Cr = CAcO- = x,
I

CA2 •= CPHAcO- ;:: y,

so that for both the acids ctt+ = (x + y). Thus, at equilibrium the compositions are as
shown :
HA 1 ~

(0.1002 - x) (x + y) x
HA2 ~ W + Ai
(0.0147 - y) (x + y) y
Neglectirig x in comparison to 0.1 and y in comparison to 0.015,

1.86 x H)-5 = (x+y)x.


0.1002
.. (x + y)x = 1.86 x 10-6. (A)
And (x + y)y = 5.42 x 10- 5 x 0.015 = 0.7978 x 10-6 (B)
Adding (A) and (B),
'(x + y)2 = 2.673 x 10-6.
(x + y) = 1.63 x 10-3.
pH = - log CH+= - log (1.63 x 10-3) = 2.79.
15. Calculate the ionic strength of 0.04 (M) solution of Al 2(S0 4) 3.
Soln. : Alz(S0 4)3 completely ionises as Al 3+ and so~- .
.
Alz(S04)3 ~ 2Al 3+ + 3 so~-
CA13+ = (2 x 0.04) (M)
cso~-= (3 x 0.04) (M)

ZA13+ = +3

z.so2--
4
- -2 .

.. ionic strength(µ)= i LC;Z;2 = i[(2x0.04)x(3) 2 +(3x0.04)x2 2 ]

= [(0.04 x 9) + (3 x 0.04 x 2)] = 0.6.


16. Using Debye-Hi.ickel limiting law ca~ulate the m~an activity co-efficients of0.001
(M) aq. solutions of K3 [Fe(CN)6] and K4 [Ft(CN)6). Oebye-Hi.ickel constant is 0.51.
[M.Sc. Admission Test 1995, 1.1.T. Bombay]
Soln. : Mean ionic activity coefficient log (y±) = -AZ+Z_ {µ +Bµ.
428 PROBLEMS.ON PHYSICAL CHEMISTRY

B is much smaller than A and is neglected.


Now, to calculate y± we ~ave to calculate at first the value of·µ.

For K 3[Fe(CN)6 ], K 3[Fe(CN)6 ] ~ 3K+ + Fe(CN)~­


CK + = (0.001 x 3)

CFe(CN)~- = 0.00 I

.. µK3[Fe(CN)6] =~[(3x0.001)x12+(0.00!)x32],,;, ~[0.003 + 0.009] = 0.006 .

.. log(y±)K 3[Fe(CN)6j = (-0.51xlx3x-v'0.006).


y± = 0.761178844 = 0.76
For K4 [Fe(CN)6],

K4 [Fe(CN)6 ] ~ 4K+ + Fe(CN):-.


CK+.= (0.001 x 4)

CFe(CN)~- = 0.001.

I
µ K4 [Fe· (CN )6 = l2 [(o · 001x4)x 12 +{O · 001)x42]· = l[0.004
2 + 0.016] = 0.01.

. . y± = 0.625173 = 0.6.
17. The solubility product· of CaF2 at 27 °C is 3.55 x 10- 11 . Using Debye-Hi.ickel
limiting law calculate the solubility of CaF2 at 27 °C in moles/litre.
[M.Sc. Admission Test 1990, 1.1.T. Kharagpur]
Solo. : CaF2(s) ~ Ca 2+ + 2F -

.. Ks = aca2+·a;_
0

aca+2
= Y+C+ = Y+C
ar = y_C_ = y_2C
Ks = (y+C) (y_2C) 2 = 4C3 yi.

Since, mean ionic activity coefficient y± = VY+ y2_ .


From Debye-Huckel limiting law, log y± = -AZ+ Z_ .[Ii.
Now, ionic strength (µ) of CaF 2 is very small and is taken as zero, i.e., µ 0.
Therefore, its activity = concentration.

log(y±)caF = 0.
2
(y±) = I, :. K5 = 4C3
or, 3.55 x 10- I I = 4C3
IONIC EQUILIBRIUM 429

or, c3 = 3.55 x_10-tt = 8.875 x 10-12


4
or, C = 2.07 x 10- 4 mole/litre.
18. Calculate the ionic strength of a solution obtained by mixing equal volume of
O.oI (M) NaCl and 0.02 (M) AICJ 3 solution. [C.U.(H) 1989]
Solo. : Due to mixing of the two solutions of equal volume, the concentration of
each solution will be the half of the given concentration.

Therefore, CNaCI = O.OI (M) = 0.005 (M)


2

CAICl3 = 0.~2 (M) = 0.01 (M)

= 0.005 (M)
= 0.01 (M)
= [0.005 + 3 x 0.01] (M) = [0.005 + 0.03] (M) =0.035 (M).

µ =-i [(o.oo5)x 12 +(O.Ol)x3 2 +(0.035)x1 2 ]= 0.065.


19. Calculate the mean ionic activity coefficient at 25 °C of a O.oI (M) solution of
K3 [Fe(CN)6]. Given Debye-Htickel constant A for water at 25 °C = 0.51.
The solubility of CaF2 in water at 18 °C is 2.04 x Io- 4 moles/litre. Calculate the
solubility in 0.01 (M) NaF solution. [C.U.(H) 1988]
Solo. : For the I st part see Problem no. 12.
2nd part : Since CaF2 is sparingly soluble in water, its ionic sti;ength is negligible.
Therefore, its activity = concentration.
CaF2 ~ Ca 2+ + 2P-
.. Ks = C.(2C)2
[Ks is the solubility product and C is the solubility in moles/litre]
. . 4C3 = Ks or, Ks = 4 x (2.04 x 10-4 )3.
Now, let the solubility of CaF2 in presence of NaF is C'(M).
:. then Cea2 + = C' (M)
and CF- = (2C' + 0.01) (M) "' 0.01 (M).
[since 0.01 is much higher in comparison to 2C']
2
Now, Ks= aCa2+ .a 2F_ = ("'+c2+)("'-C
I Ca F-
I
) = yi.C'.(0.01) 2.

Now, log y± = -AZ+Z-{µ_.


For 0.01 (M) NaF solution µ = t L C;Z;2
= t [0.01{1)2 +0.01 x (1) 2] = 0.01.
.. Jog y± = -0.51 x 2 x I x ..,/Q.Ol
.. y± = 0.79.
430 PROBLEMS ON PHYSICAL CHEMISTRY

Now, we have calculated before, Ks = 'Y~ .C'.(0.01) 2.


putting the values of Ks and y±, we have
4 x (2.04 x 10-4)3 = (0.79) 3 x C' x (0.01)2.
3
. ' 4 x (2. 04 x 10-4 ) -7 .
or, C = 3 2 = 6.89 x 10 molenatre.
(0.79) x(0.01)
20. Calculate the ionic strength of a solution containing 0.01 (N) NaCl and 0.02 (N)
N~S0 .
4 .
Solo. : See solution to Problem no. 14.
21. Find out the solubility of AgCl in a solution where Cc1- = 10- 1 (N). To a saturated
solution of AgCl in water, I o- 8 (N) c1- is added. Find out the amount of AgCl precipitated
in gm/litre. The solubility product (K5 ) of AgCl is 1 x 10- 10 .

Solo. : Ks = CAg+·Cc1_
Since Cc1- = 10-1 (N),

:. CAg+ = Ks 10-10
-c = - -1 -- 10-9 .
ci- 107
Therefore, 10-9 moles/litre AgCl will dissolve. The solubility in gm/litre
= 10-9 x 143.5 = l.435 x 10-7 gm. [m. wt. of AgCl = 143.5]
Now, if the solution is initially saturated in water, then there are !o- 5 gm-mole/litre
o_f AgCl, i.e., 1.435 x 10-3 gm/litre of AgCI.
. -10
Now CA 8+.Cc 1- = 10 .
Let CAg+ coming from AgCl in solution be x. So, if _l o- 8 (N) Cc1- is added,
x (x + 10- 8 ) = 10-JO
or, x 2 + 10-8.t - 10- 10 =0
or, x
-10-s +.J10- 16 +4.10- 10 = -10-s +.J4x 10-10 [ .,. 10-16 « 4 x 10-101
= 2
= 0.9995 x 10-5 .
.. AgCl precipitated = oo- 5 - 0.9995 x 10- 5 ) gm-mole/litre
= 0.0005 x 10- 5 gm-mole/litre
= 0.0005 x 10- 5 x 143.5 gm/liter
= 7.175
x 10-7 gm/litre. .
22. Calculate the solubility product of Ag2Cr0 4 . Given that its solubility is 7x l o- gm-
5

molenitre. [V.U. 2004]

Solo. : Ag 2Cr0 4 (gJ ~ 2Ag+ + Cro~-


C =7 x 10- 5
CrO~-
CAg+ =2 x 7 x 10- 5

.. (Ks)Ag
2
CrO
4
= C2
Ag+ .
C
CrO 2 -
= {(2x7x10-5 ) 2 x7xl0-5 } = 1.372 X 10-tz.
4
IONIC EQUILIBRIUM 431

23. Calculate s;onc in a saturated solution of H 2 S which is approximately 0.1 (M). ·


K 1 = 9.1 x 10-8 and K2 =
1.2 x 10- 1s.
Solo. : H 2 S ~ H+ + HS-

:. K = CH+.CHs- = 9.1 x 10-s.


1
. CH2S
And HS- ~ H+ +, s2-.

:. K = cH+ · cJ- = 1.2 x 10- 15 .


2
cHS-
S ince K 2 is very small, it. may be assumed that Cs2- is very small. Thus, we may
safely assume that CH+ = CHS-· Since K1 is also very small, CH s = 0.1 (M).
2
.. CH+= CHS-= ~Kl XO.I =~o-8 XO.I = 9.5 x 10- 5.

And Cs2- = 1.2 x 10- 15 .


2
( c +) x c 2-
Again , K 1K 2 = Hc s
HzS
I. Ix 10.-23
= 1.2 x 10- 15 .
(cH+ f
Problem ~ Calculate Cs2- when CH+ = 0.3 and 10-7 .
[Ans. 1.2 x 10-22 ; I· I x 10-91
24. The solubility product of CaF2 at 25 °C is 1.6 x 1~- 10 . Should precipitation occur
when 50 ml of 5 x 10-2 (M) Ca(N03) 2 is mixed with 50 ml of 4.0 x 10-4 (M) NaF solution?
If precipitation occurs, how much CaF2 will be precipitated? [C.U.(H) 1986]
Solo. : By the problem, after mixing
the concentration of Ca 2 +

= 50 x 5 x 10- 2 (M) = 2.5 x 10- 2 (M)


100
and C.,..- = 50 x 4 x 10-4 (M) = 2 x 10- 4 (M) .
• r TOO
2
Now, CcaZ+·C!- = (2.5xl0- )x(2xl0-4)2 = 10 x 10-10 = 10-9.

But (K)
S CaF
= 1.6 x 10- 10.
2
Since the product of concentration terms is greater than K5 , there will be precipitation.
Let x moles/litre Ca 2+ ion be precipitated. Then 2x moles/litre r will be precipitated.

Ks = Cca z+.C2F-
or, 1.6 x I 0- 10 = (2.5 x 10-2 - x) (2 x 10-4 - 2x) 2 •
The result will be a equation eontaining x3 which is difficult to solve. But we know
that 2.5 x 10- 2 >> x.
432 PROBLEMS ON PHYSICAL CHEMISTRY

:. 2.5 x 10-2 (2 x 10-4 - 2x) 2 = 1.6 x 10-10.


:. 0.1 x2 - 2 x 10-5 x + 8.4 x 10- 10 =0
or, x = 1.4 x 10- 4 or, 6 x 10-5 .
If x is 1.4 x 10-4 , then 2x = (2 x 1.4 x 10- 4 ) which is greater than 2 x 10-4 . This
is not possible:
Therefore, x =6 x 10-5 molar.
But the volume of the solution is I00 cc, therefore, 6 x 10-6 moles of CaF2 will
be precipitated, i.e., 78 x 6 x 10-6 = 4.68 x 10-4 gm of CaF2 .
25. The solubility product of Pbl is 7.47 x 10-9 at 15 °C and 1.39 x 10- 8 at 25 °C.
2
Calculate (i) the molar heat of solution of Pbl 2 and (ii) the solubility in moles/litre at 75 °C.
. [C.U.(H) 2005]
Soln. : By van't Hoff equation,

In (Ks )2 = Aff T2 - Ti
(Ks) 1 R T1T2 '

where (K5) 2 is the solubility product at T2 K, (K5) 1 is the solubility product at T 1 K and
Affis a constant heat of reaction. For the given case,
Pbl 2 ~ Pb 2+ +. 21-
By the problem,
T1 = 288.15, (K5 ) 1 = 7.47
x 10-9
T2 '= 298.15, (K5 ) 2 = 1.39 x 10-8

.. In 13.9
7.47
= Aff( 288.15 10x 298.15 ) .
R
. . !J.H = 8.314x288.15x298.15 In 13.9 .
IO 7.47
= 44355.8345 joules = 44.36 ~·:.
= K at 75 °C, i.e., 348.15 K
In K = 44355. 834 x _ _5_o_ _ =··~. 5698603
1.39x10-8 8. 314 348.15 x 298.15
K
or, ---...,,.. = 13.064
l.39x 10-8
· or, K = 1.81589 x 10- 7 = 1.82 x 10-7.
If the solubility is x mole/litre, then
x.(2x) 2 = 1.82 x 10-7
or, 4x3 = 1.82 x 10-7
or, x = 3.57
x 10- 3 = 0.00357 mole/litre.
26. The specific conductance of a saturared solution of AgCI is l .55x10- 6 ohm- 1.cm- 1•
The mobilities of c1- and Ag+ ions are 5.6 x 10-4 cm per sec and 6.8 x 10- 4 cm per sec
respectively under unit potential gradient. Calculate the solubility product of AgCI.
[C.U. 1983]

Solo. : A<;,.gCI = A:g+ +A.~ 1 - = Fu+ + Fu_ = (u+ + u_)F


= (5 .6 x 10-4 + 6.8 x 10-4 ) x 96484 = 119.64.
IONIC EQUILIBRIUM 433

Now, A = IOOOK [N = normality] .'


N
= lOOK [Z is the valancy = I]
zc
c = IOOOK
Z.A
Since the solution is extremely dilute, A is almost equal to A 0 , i.e., A Ao
I 000 x I. 5.5 x 10--<i
= ---~-- = 1.29555 x 10-5 .
119.64

· KS
•" = CAg+·CCl- -- C 2
= ( 1.29555 x 10-5) 2 = 1.68 x 10- 10.
27. The solubility product of PbS04 is 1.31 x 10- 8 . Calculate the number of moles
of PbS0 4 that can be dissolved in 5 litres of I x 10- 3 (M) Na2S04. Show whether PbS0 4
will be precipitated if 20 cc of. 2 x 10- 4 (M) Pb(N0 3)2 is mixed with 80 cc of
Ix 10- 4 (M) N32S04. [C.U.(H) 19721
Soln.:
PbS0 4(s) ~ Pb2+ + so-42
x x
N32S04 ~ 2Na+ + S04 =
1st Part :
x(x+ 10-3) = 1.31 x 10-8
Solving, x = 1.293 x 10-s.
Thus in 5 litres moles of PbSO 4 is
=
(5 X 1.293 X 10-5) = 6.465 X J0- 5 •
2nd Part : Initial cone. of Pb 2+

= 2 x J&_ x 10-4 (M) = 2 x 0.2 x 10-4 (M).


l\N .
so4= x 10-4 (M) = 0.8 x 10-4 (M).
Initial cone. of = I x
1~
:. CPb2+ .C50= = (2 x 0.2 x 10-4) x (0.8 x 10-4)
4
= 0.32 x 10-8.
This is less than (K5 ) PbS04. So there will be' nc precipitation of PbS04•
28. If 20 cc of 2 x 10-3 (M) Pb(N03)2 is mixed with 80 cc of I x 10-4 (M) Na 2S04,
will there be precipitation? If so, then calculate the amount of precipitation in gm.
[Given that (K5 )PbS04 = 1.31 x I0-8]
Soln. : The cone. of Pb2+ is

= I~ x 2 ~ 10-3 .(M) = 0.2 x 2 x 10-3 (M) = 4_ x 10~4 (M)

and cone. ~f so~- is=-~ -x I x 10-4 (M) = 0.8 x 10-4 (M).

Proh. Pliv rhf'm -?~


434 PROBLEMS ON PHYSICAL CHEMISTRY

The product of these two cone. terms is greater than 1.31 x 10- 8 . Hence, there will
be precipitation.
Let x mole/litre will be precipitated.
Then (8 x 10- 5 - x) (4 x 10-4 - x) = 1.31 x 10-s.
Solving, x = 4.35 x 1o- 5 (M) [neglecting (+) sign]
The solution is 100 cc.
Thus number of moles precipitated = 4.35 x l o-6 mole
= (4.35 x 10-6 x 302) gm = 0.0013 gm.
[M. wt. of PbS0 4 = 302)
29. At 25 °C, the solubility product of AgBr0 3 is 5.77 x 10-5 . Using the Debye-Htickel
limiting law, calculate its solubility in (a) pure HzO (b) 0.01 (M) KBr0 3 .
Soln. : (a) We know that,

Ks= aAg+·aer03 = 5.77 x 10-5.

The amount of a +and a 8 r0_ in solutions are so insignificant that the ionic strength
. Ag 3
(µ) of solution is almost zero, so that y± = 1.

= CAg+·CBrO) = x2.

x = ~ 5. 77 x 10-5 = 7.596 x 10-3 (l\j).


(b) In a 0.01 (M) solution of KBr0 3 the ionic strength is

µ =i Ic;z;2=i[o.01 (1) 2 + 0.01 x (1) 2J =i[2 x 0.011 = o.oi.

. . log r" ~ - AZ+Z- {µ_ + Bµ


B is very small in comparison to A.

So, log y" = -0.51 x 1 x 1..Jo.01 = -0.051


•• 'Y:!: = 0.889.

Now, Ks = aAg+·aero3 = ( CAg+. 'Y +).( Cer03 ·.'Y-)


= (x."f+){(x+O.OI)y_} = x(x+O.Ol)y; (since Y+'Y- = "(~).
x (x + 0.01) (0.889) 2 = 5.77 x 10-5
or, x2 + O.Olx - 7.30 x 10-5 = 0

x = -O.Ol±~(0.01) 2 +4x7.30xl0-5 = -0.01±~3.920xl0-4


2 2
=
-0.01 ±0.01980
2
Neglecting (-)ve sign,
-O.Ol+0.01980
x = = 0.0049 (M).
2
IONIC EQUILIBRIUM. 435

30. A soln. is 0.1 (M) in Cl- and 0.001 (M) in CrO~. ff solid AgN0 3 is gradually
added to the solutions, which will precipitate first, AgCl or Ag 2Cr0 4? Assume that the addition
causes no change in volume. Given that solubility products of AgCI andAg2Cr04 are respectively
· 1.7 x 10- 10 and 1.9 x 10- 12 . What will be the cone. of cl-when Ag 2Cr0 4 begins to precipitate?
What percentage of c1- originally present will remain in solution at this point?
Solo. : C Ag+ required to precipitate Ag 2Cr0 4 is

a2 +aero= = {Ks)Ag2cr0
Ag 4 4

a2Ag+ = 1. 9x10- 12
or,
acr0=
.4

[Assuming, a At = C Ag+ and acr0i- = Cei-04 l

c2 - 1. 9 x 10- 12 1. 9x10- 12
Ag+-
ccro4- = - 0.001
[Initial cone. of Ccr _ = O.OOl(M)]
024

= 1.9
x 10- 9
.. CAg+ ~ 4.36 x 10-5 .
Now, since initial cone. of c1- is 0.1 (M),
C Ag+ required to precipitate AgCl is calculated as follows :

· · (Ks) AgCl = CAg +. cCl- [asuming activity = cone.]

.. C += (Ks)AgCl = 1. 7X10-10 = 1.7 X 10-9.


Ag CCI_ 0.1
Since cone. of Ag+ required for precipitation of Ag2Cr0 4 is greater than cone. of Ag+
required for precipitation of AgCI, therefore, if solid AgN0 3 is gradually added to the
solution, AgCI will precipitate first.
When Ag2Cr04 starts precipitating, the cone. of Ag+ in soln. is 4.35 x 10-5 (M).
Thus CCI- at this stage is
1.7 x 10- 10 = 4.35 x 10-' x Cc1-
l. 7x 10-10
or, CCI- = . (M) = 3.9 x 10-6 (M).
4.35x10-5
Initially there is 0.1 (M) of Cc.-·

Hence, percentage of c1- remaining = 3· 9 x 10-6


x 100 0.0039. =
0.1
0

31. A solution containing 0.01 (M) ZnCl 2 and MnCl 2 is saturated with H 2S. The
solubility products of ZnS and MnS are l x 10-22 and 5.6 x 10- 14 respectively. What
is the pH at which MnS will form precipitate? What will be the cone. of Zn 2+ remaining
at the pH? (For H2S, ~ = K 1K2 = l.l x 10-21)
Solo. : The solution contains O.ot (M) ZnCJ 2 ·and O.Ol(M) M?Cl 2.
Now the cone. of s= required for precipitation of MuS is calculated as follows :
CMn2+ .Cs2- = (Ks)MnS
436 PROBLEMS ON PHYSICAL CHEMISTRY

or, C52- 5.6xI0-16 (M) = 5.6 x I0-14 (M).


0.01

and HS- = H+ + s=
K
2
= CH+·CHs-
CHs-

" K IK 2 = C~+·Cs=
~,--->'-
CH 2S
Now when MnS will precipitate, the cone. of s2- will be
= 5.6 x I0- 14 .
At this cone. CH+ will be

= K1K2 = C~+ .Cs2- = 1.1 x I0-21


CH2S
21
or c2 = 1.1 x I0- x O. l [ c = 0.1 for saturated soln.]
, H+ 5. 6 X I0-14 H2S
= 0.0196 x I0-7 = 0.196 x IO-s.
CH+ =:= 0.443 x I 0-4 (M)
pH, = -log CH+ = -log (0.443 x lo-4) = 4.35.
Now, at this pH C52 _ = 5.6 x 10-14.
Let the cone. of Zn 2+ = cZn 2+·

- (Ks)znS - IX I0-22
or, Cz 2+ -
n C52-
-
5.6x I0- 14
= 1.79 x 10-9 (M).

32. IO cc of 0.1 (M) NaOH soln. are added to 50 cc of 1.1 (M) HOAc. What will
be the pH of the resulting mixture"? Given that K0 for HOAc 1.75 x I0-5 . =
. [C.U. 1977)
Solo. : IO cc 0.1 (M) NaOH = IO x 0.1 cc I (M) =I cc I (M).
And 50 cc I.I (M) HOAc = (50 x I.I) cc I (M) = 55 cc I· (M).
Now, I cc I (M) NaOH will consume I cc I (M) HOAc and produce I cc I (M) saJ.t.
Thus, HOAc remaining will be = (55 - I) cc I (M) = 54 cc I (M).
Total volume of the mixture = 60 cc.

Now cone. of salt = fa x I (M)

and cone. of acid =~ x I (M).


IONIC EQUILIBRIUM 437

Now, pH = pKa + log Csalt = -log (1.75 x 10-s) + log _!_


Cacict 54
= 4.757 - 1.7324 = 3.0246 = 3.0.
33. It is desired to prepare a buffer solution of pH 4.5 using HOAc and NaOAc. Given
that Ka for HOAc = I.8 x 10-5 . Calculate the weight of CH3COONa that must be added
to I litre of I (N) HOAc.
Solo. : We know that pH of a buffer of weak acid and its salt is

pKa + log Csa1t


Cacict

or, 4.5 = -log (1.8 x 10-5 ) + log csalt .


cacid

log Csalt = 4.5 + log (l.8 x 10-5 ).


Cacid

csalt = 0.5692.
cacid
Now, let x (N) CH3C00Na will have to be mixed with I (N) CHFOOH.

Therefore, T= 0.5692. or, x = 0.5692 (N) = 0.5692 x 82 = 46.67 gm.


[ ·: m. wt of CH3COONa = 82]

34. A definite volume of an aq. ~ HOAc (Ka = 1.8 x 10-5) is titrated with a s_trong
NaOH soln. It is found that 75 equal sized drops of Ni.OH, added from a burette, effect
the complete neutralisation. Find the pH values when the acid soln. is neutralised to the
extent of 20%, 40% and 90% respectively. Neglect volume change due to addition of alkali.
[C.U.(H) 1979]
Solo. : Addition of alkali to a weak acid, produces a buffer of, weak acid and its salt.

pH = pKa + log Csa1t


cacid
When 20% is neutralised, then 20% of salt and 80% of acid remain in solution.
Cs 20
pH = -log (l.8 x 10-5 ) + log CA = 4.745 + log 80 = 4.145. == 4.1.

For 40% neutralisation process,

pH= 4.745 + log 40 = 4.57 4.6.


60
For 90% neutralisation process,

pH = 4.745 + log 90 = 5.6992 = 5.7.


JO
35. A buffer solution is made from 0.06 mole of HOAc and 0.04 mole of NaOAc
per litre. What is the pH of the solution? Calculate the changed pH resulting from the
addition of (i) O.oI5 mole HCI (ii) 0.03 mole of NaOH per litre of the solution.
[Ka = 1.8 x 10-5 ]
438 PROBLEMS ON PHYSICAL CHEMISTRY

Soln. : The pH of the solution will be

pH = pK0 + log Cs
CA
5 0.04
= -log (1.8 x 10- ) + log-- = 4.568 "'4.6.
0.06
(i) 0.015 mole acid will react with Aco- and produce (0.04 - 0.015) mole= 0.025
mole salt and (0.06 + 0.015) = 0.075 mole acid.
40
:. pH = pK0 + log Cs = 4.745 + log-.= 4.745 - 0.4771
CA 60
= 4.2679 "' 4.3.
(ii) When 0.03 mole NaOH is added to the solution, then it pfoduces (0.04 + 0.03)
mole = 0.07 mole salt and (0.06 - 0.03) = 0.03 mole acid.
0.07 .
:. pH = pK0 + log .£-As = 4.745 + log - - = 4.745 + 0.37 = 5.112 = S.l.
1..-. 0.03
36. Calculate the concentration of H+, H 2As04, HAs04 and As04 in 0.5(M) solution
of H3 As0 4 . Given, K 1 = 2.5 x 10-4 ; K2 = 5.6 x 10- 8 ; K3 = 3 x 10- 12 .
Soln. : We know that,
H20 + H3As0 4 = H3o+ + H 2As04.

C +·C
H30 H2As04

CH3As04

H20 + H 2Aso4- =Hp++ HAso4=


C +·C _
H30 HAso4

cH2Aso4

K3 = CH30+·CAs ~ - .
4
cHAs04-
Since K 2 and K 3 are much smaller than Kp
.. CH
30
+ = CHzAs0_ = x (say).
4

:. K1 = (0.xxx
5 - x)
= 2.5 x 10-
4

or, x2 = 2.5 x 10- 4 (0.5 - x)


or, x2 + 2.5 x 10-4 x - 2.5 x 0.5 x 10- 4 = 0.
Solving, x = 0.011 [neglecting (-)ve term]
IONJC EQUILIBRIUM 439

C +·C _
~ Now, K2 = HJO HAso4

cH2Aso4
= CHAS 02 _
4
[since CH.
30
+= C
H2As0 4
_]

•• CHAso~- = 5.6 X 10-8 (M).

c HJO+ .cAso~-
Again, K3 =
K2

or, c 1.527 x l0- 17 (M) = 1.53 x 10-7(M).


Aso~-

37. Calculate H30+, H 2PO:;, HPoi- and P0 4 3- cone. in 0.1 (M) H3P0.4 at 25 °C.
[Given : K 1 = 7.52 x lo- 3, K 2 = 6.23 x l0- 8 ar:d K 3 = 4.8 x 10-13]
Soln. : Proceed as Problem 32.
[CH o+
3
= CH2P0_ =0.0239 (M); CH 3PO4 =0.076l·(M); C _ ,
HP0 2
=6.23 x . 10- 8 (M);
4 4

CP0_ 3 = l.25 x 10- 18 (M)]


4

38. You are supplied with 0.12 (N) strong acid HX and r0.15 (N) weak base BOH
of Kb = 2 x 10- 5 . Prepare 110 cc of a buffer of pH 9.
Soln. : We know that, the pH of a weak base and its salt is

14 -[pKb+log Csalt ]·
Cbase

.. 9 = 14 -[5-log2+log~J
or, log 2 = log Cs
c;;
or, Cs = 2.
CB
Let us take x cc of HX and (110 - x) cc BOH.
Now, x cc of O.J 2 (N) HX will react with x cc of 0.12 (N) BOH to give x cc of
0.12 (N) salt.
Since the final volume = 110 cc,
xx0.12
:. csalt = 110
Now, let y cc of. 0.15 (N) BOH react with x cc of 0.12 (N) HX.
xx0.12
.. y = 0.15
440 PROBLEMS ON PHYSICAL CHEMISTRY

2
Thus, the remaining BOH is (110-x- xxO.l ).
0.15.
So the strength of the remaining BOH
0 27
= ( 110- · x)o.15
0.15 = 16.5-0.27x
110 110 .

NOW, Csalt = 2.
cbase

xx 0.12
..,....,..-=-1;..;l~O~- =2 or,
xx0.12 =2
16.5-0.27x 16.5-0.27x
110

or, 0.12x = 16.S x 2 - 0.27 x 2x or, 0.66x = 16.5 x 2

or, x = 16.5x 2 = 50.


0.66
Hence, 50 cc of 0.12 (N) HX is to be mixed with (I IO - 50) cc of 0.15 (N) BOH
or, 60 cc of 0.15 (N) BOH.
39. Prepare a 1000 cc buffer soln. of pH 5 with 0.5 (N) HA (Ka =2 x 10-5 ) and
0.2 (N) NaOH.
Cs
Hints : pH = pKa + log CA

Cs
c;= 2

Let us take x cc 0.2 (N) Oir.


C _ xx0.2
·· salt - IOOO
Now, x cc 0.2 (N) NaOH will consume y cc 0.5 (N) HA.

.·. y = xx0.2
0.5

(1000-x- xx0.
2
)o.5
So C
acid -
- 0. 5
lOOO
· = 500-0. 7x
1000
xx0.2
2.
500-0.7x
:. x = 625 cc base.
:. (1000 - x) = 375 cc acid.
40. You have two stock solution :
'HOAc (Ka = 1.86 x 10-5) and PhCH2COOH (Ka = 5.42 x 10-5 ) containing IO gm/
litre and 5 gm/litre of the acids respectively.
IONIC EQUILIBRIUM 441

By mixing how many litres of PhCH 2COOH with 1 litre of HOAC, you can prepare
a solution of C + = 1.5 x 10-3?
H30

6
Soln. ; Let K litre of PhCH2COOH (HA 2) containing 1~ = 0.0368 mole/litre is added

to 1 litre of HOAc (HA 1) containing !8- = 0.1667 mole/litre.


They will both ionise to tt+ and A1- or Ai.
Let the equation composition be shown as belC1W :

HAI
___.. tt+ + A1-
~

(0.166T-x) x
1.5 x 10-3
l+ K l+K
___..
HA2 ~
H+ + Ai
(0.0368-y)K yK
1.5 x 10- 3
l+K l+K

1.5x10-3 x _x_
Hence, 0. 1667 _ l+K
x = 1.86 x 10-s
l +K
x 1.86x10-5
or, ----= 3
0.1667-x l.5x10'- ·
Neglecting x in comparison to 0.1667,

_x_ = 1.24 x 10-2 .


0.1667
x = 1.24 x 10-2 x 0.1667; :. x = 2.07 x 10-3.

I.5x10-3 x L
Again, -----=1-...:+_.K.:t.- = S.42 x 10-5 .
(0.0368-y)K
l+K
Neglecting y in comparison to 0.0368,
3
l.5xl0- y = 5.42 x 10-s.
0.0368
5
:. y = 5. 42 x 10- x 0. 0368 = 1.33 x 10-3.
1.5 x 10-3
· By the problem,

CH+= (2.07xl0-3+1.33xl0-3K) = 1.5 x 10-3, :. K = 3.4 litres.


l+K
41. 60 cc of 0.015 molar BaC12 solution is mixed with 40 cc of 0.0125 molar KC!
solution.Calculate the ionic strength of the solution. [B.U. 1987]
Soln. : Proceed as shown earlier. [Ans. 0.032)
442 ' PROBLEMS ON PHYSICAL CHEMISTRY

42•. At 25 °C and at a dilution of 32 litres the equivalent conductance of a salt of


weak base and strong acid is 98.5 ohm- 1.cm 2, while thar of a strong acid at the same
dilution is 383 ohm- 1.cm 2. On adding successive quantities of the free base to the solution
of the salt, the equivalent conductance could be finally reduced to 89.7 ohm- 1.cm 2. Calculate
the degree of hydrolysis, and hydrolysis constant. [C.U.(H) 1983]
Solo. : If ex be the degree of hydrolysis of the salt and Ac be the equivalent conductance
of the unhydrolysed salt, then
A-Ac
ex = --~-[A = eq. conductance of the solution].
AHA -Ac
A= (I - ex)Ac + exAHA .
AHA= eq. conductance of strong acid.
Now, A = 98.5 ohm- 1.cm2
Ac = 89.7 ohm- 1.cm2
AHA = 383 ohm- 1.cm 2.

So, ex = 98.5 - 89. 7 = 0.03.


383- 89. 7
Concentration of the solution is C =__!__ (N).
32
(0. 03)2 x _!_
Hydrolysis constant (Kh) = 1ex~S =
v. (1-0.03)
32
'
= 2.899 x 10-s.

43. Calculate the pH of a 0.1 (M) K 3PO 4 solution. The third dissociation constant of
ortho-phosphoric acid is 1.3 x 10- 12 . The hydrolysis proceeds only in the first step. Assume
Kw = I x 10- 14. Do not neglect ex.
Solo. : The hydrolysis reaction is :
P0 4 3- + Hp = HPOi- + Olr
Initial : 0.1 0 0
Equilibrium : 0.1 (l - ex) O.lex O.lex
1x10- 14
K"
h
= Kw -
K0 - 1.3x10- 12
= 7.692 x 10-3 .

2
= 1-
ex C
ex·
. . ex 2C = 7.692 x 10- 3 = 7.7 x 10- 3 .
1-ex
.. ex2 c + 7.7 x 10- 3 ex - 7.7 x 10- 3 = o.

-7. 7x10- 3 +~(7.7x10-3 ) 2 +4x7.7x10-3 x 0.1


. . ex = (since, C = O. IM)
2x 0.1

-7. 7x 10-3 +~5. 929x 10-5 +3.08x10-3


= 2x0.1
-7. 7x10-3 +0.056029 0.04833
2 x 0.1
= 0.2
= 0.2416.
IONIC EQUILIBRIUM 443

C0 w =a x c = (0.2416 x 0.1) = 0.02416


pOH = 1.6168 = 1.62
pH = (14 - 1.62) = 12.38.
If we neglect a, we can use the formula : pH = 7 + 21 pK3 + 21 log C = 12.44.

44. Will Fe(OH) 3 (s) precipitate from buffer solution prepared by mixing 0.5 (M)
HOAc and 0.15 (M) NaOAc at 25 °C, if the solution contains 0.25 (M) Fe 3+?
Ksp of Fe(OH)3 =4 x 10- 38 , Ka of HOAc = ).74 x 10-s.
Solo. : For weak acid and its salt the pH of the solution is :

pKa + log -ElL


c
cacid

or, -log CH+ = -log Ka + log _s


c
CA

or, CH + -- KaCA
· - = l.74 x w-s x ~ = 5.8 x w-s.
Cs 0.15
Kw w-14
· cOH- --
·· -C
tt+
= 5.8x w-5 = 1.7 x 10.- 10.

Now, CFe3+·Cbw = 0.25 x (l.7 x rn- 10) 3 = l.23 x w-30.

As the ionic product of Fe (0H) 3, i.e., l.23 x rn- 30 is greater than the solubility
product, i.e., 4 x rn-38 . So Fe(OH)3 will precipitate.
45. The passage of C0 2 gas through a saturated solution of FeC0 3 leads to the foHowing
reaction at 30 °C :

FeC0 3 + H 2C0 3 ~ Fe 2+ + 2 HCOj'


Given that the solubility of FeC0 3 is 6.6 x w-
6 moles/litre and the solubility of C0
2
is 0.665 volume/litre. Calculate the pH of the solution and equilibrium constant of the
above reaction. (For H2C0 3, K 1 = 3.7 x rn-7 and K 2 = 5 x 10-11)
Solo. : 22.414 vol at 273.15 °K l mole =
22 14 303 5
·
1.e., .4 273.15
x ·1 VO l. at 30 °C -== 24 .87 VO·.1 - l mole

l
:. l vol. =- -
24.87
mole

665
i.e., 0.665 vol. = O. mole = 0.0267 mole/litre
24.87 '·
H2C03 + HzO ~ H 30+ + HCOj'
(l - <X1)C a1C a1C

.. K1 = a~C.
1-o.1
Since a 1 is small in 'comparison to l,
444 PROBLEMS ON PHYSICAL CHEMISTRY

K 3. 7x 10-7
•• CX1 = -C1 = = 3.72 X 10-3 •
0.0267
. • CHco- = cx 1C = (3.72 x 10-3 x 0.0267) = 9.93 x 10-5 mole.litre-I.
3

For the reaction, FeC03 + H2C03 ~ Fe2+ + 2 HCO]

_ CFe2+·C~CO) _ (6.6x I0-6)(9.93x 10-


5
)2
K
- CH2C03 - 0.0267

[ Ctt c 03
2
= 0.0267 mole, since cx 1 < < I]
.. K = 2.44 x 10- 12 . .
Now, Ctt 3o+ == CHco3 ·

Since CH 30+ resulting from 2nd dissociation is negligible in comparison to the first
dissociation constant,
:. pH= -loge + = 4.002 = 4.
H30
46. The dissociation constant of NH40H is 1.8 x 10-5 . The solubility product of
Mg(OH) 2 is 1.22 x 10- 11 . How many grams of solid NH4Cl must be added to a mixture
of 50 cc (N) NH 40H solution and 50 cc (N) MgC1 2 solution so that ppt. of Mg(OH) 2 just
disappears? It is assumed that the volume of the solution is not changed by dissolving
solid NH4 CI and the dissociation of the salt is complete. [B.U. 2001]
Hints : Cone. of OH- required so that ppt. of Mg(OH) 2 just disappears
= 6.99 x l0--6(M).
Let x gm of NH4Cl be added so that dissociation of NH40H is suppressed.
NH40H ~ NH~ + OH-

C + x6.99x 10-6
J.8 X 10-5 = _N_H~4- - - - -
0.5
C + = l .288(M).
NH 4

1.288
Thus, x = - - x 100 x 53.5 = 6.89 gm.
1000
47. The value of equivalent conductance (A) of a O.Ol(M) solution of a weak acid
(HA) is 60 at 298 K while the value of A 0 is 360 at the same temperature. Calculate
pKa of the acid considering the activities. [B.U. 2000]
Solo. : A weak acid HA dissociates into H+ and A-.
HA~ H+ +A-

(1-a)C cxC cxC

K·=
cH+CA_ = cxzc
c
CHA I-ex
IONIC EQUILIBRIUM 445

Hence, pKa = -LogKa = - logKc - 2Jog y±.


A
Now, a = Ao
Here, A = 60 mho.cm 2
Ao = 360 mho.cm 2
.. (l = 0.167 and
Kc = 0.167 2 x0.01 = 3.35 x 1o - 4.
l -0.167
Again, logy± = -AZ+Z_ {jl
µ = ~LC;Zl = ~[cH+.1 2 +CA_.1 2 J = ~[aC+ac]
i

= ac = 0.167 x o.ot = 0.00167.


logy±= -0.51 x 1 x 1 x .Jo.00167 = -0.021
Thus, pKa = -log(3.35 x 10- 4) -2 x (-0.021) = 3.52.
48. Calculate the ionic strength of a solution obtained by mixing aqueous solution
of 20 ml of 0.05 (M) MgCl 2. 30 ml of 0.05 (M) Na2S04, 25 ml of 0.04 (M) AlClj and
25 ml of 0.01 (M) glucose at a given temperature.
Solo. : Total valume = 20 + 30 + 25 + 25 = l 00 ml.
Thus, CMgCl2 = 0.Ql (M), CNa s04 = 0.ol5 (M), CAICl = 0.01 (M).
2 3
Glucose is non-electrolyte. So it is not considered.
MgCl 2 = Mg+ 2 + 2c1-
Na2S04 = 2Na+ + So4- 2
AlC1 3 = Al+3 + 3Cl-

l 2
µ = -C.Z.
2 I I

l
= -(0.01 x 22 + 2 x O.ol5 x 12 + 0.01 x 32 + 0.015 x 22 + (2 x 0.01
2
+ 3 x 0.01) x 12].
= 0.135.
49. Solubility of MgC03 at a particular temperature is 10-3 moles/litre. Calculate is
solubility in 0.1 (M) KN0 3 solution. [B.U. 19941

Solo. : · MgC0 3 ~ Mg+ 2 + co32


For MgC0 3 solubility product, Ksp
446 PROBLEMS ON PHYSICAL CHEMISTRY

Since MgC0 3 is sparin~ly soluble in water,


"(± = } and thus aMg+2 ':;::, .CMg+2 and acoJ2 =
Here, C +2 = C _2 = C = 10- 3(M).
Mg co3
Thus, Ksp = (10-3)2 = 10-6.
Let the solubility of MgC0 3 be C' in presence of 0. I
Now, Ksp = C'2 ("(±) 2 = 10-6

log "(± = -AZ+Z_ ..jµ.


Now, µ = !.2 [0. I x I 2 + 0. I x I 2] = 0. I
Thus, log "(± = -0.51 x 2 x 2 x .J(fi
•• 'Y± = 0.226.
Hence, 10-6 = C'2 x (0.226)2
.. C' = 10-3/0.226 = 4.42" x 10-3 moles/litre.
SO. Solubility of a sparingly solub1$! salt (MA) at 25 °C is 10-4 (M). Calculate its
solubility in O. l(M) MCI solution. [consider the effect of ionic strength. The Debye-Htickel
constant A at 25 °C = 0.5 I] [B.U. 2000]

Hints : Ksp = 10-8 = "(~C'O.I


. Jog "(± = -0.51 x I x I x .J(fi
'Y± = 0.6898.
Hence, C = 2.102 x 10-7 mole/litre.
51. Solubility of AgCI is 10-5 mole/litre. Calculate the solubility in (a) water (b) 0.01
(M) NaCl, (c) O.oI (M) NaN0 3 (d) 0.001 (M) Ca(N0 3) 2. Arrange the solubility of AgCI
in decreasing order.
Soln. : (a} In water, "(± = 1.
Thus, CAg+ = Cc1- = to-5.
(b) Let the solubility be C 1 mole/litre in 0.01 (M) NaCl.
Thus, CAg+ = C 1(M), Cc1- = (0.01 + C 1) .. O.OI(M) ..

Ionic strength, µ = _!_2 [0.01.1 2 + 0.01·I 2] = 0.0 I.


log 'Y± = -A.Z+Z-..jµ = -0.51 x 1 :\< 1 x ..Jo.01 = -0.051.
.. 'Y±. = 0.89.
So, 10- 10 = C1 x O.oI x (0.89) 2, :. C 1 = 1.26 x 10-8 mole/litre.
(c) Let the solubility of AgCI be C2 mole/litre in O.Ol(M) NaN0 3.
µ = O.Ql and "(± = 0.89
Now, CAg+ = Cz(M)•and Cc1- = C2(M).
Hence, 10- 10 = Ci x (0.89) 2..
:. c2 = 1.12 x 10-5 mole/litre.
IONIC EQUILIBRIUM 447
(d) Let the solubility of AgCl be C 3 mole/litre in O.Ol(M) Ca(N0 3)2.
Hence, CAg+ = t'c 1- = C3(M).

µ = ~[0.01 x 22 + 2 x 0.01 x 12] = 0.03.


logy± = --0.51 x 2 x l x ,,,Jo.o3
:. 'Y± = 0.66.
Hence, 10-IO = Cf x (0.66) 2, :. C3 = 1.52 X 10-S mole/litre.
Thus, solubility of AgCl in Ca(N03)2 > NaN03 > H 20 > NaCl.
52. To one litre of a solution of 0.1 (M) in acetic acid and 0.1 (M) in CH 3COONa.
1 cc of 10 (N) HCl is added. What will be the change in pH? The change in volume
may be neglected. K0 = 1.8 x 10-5 at 25 °C. [B.U. 1979]

Soin. : The pH of the solution before addition of HCl = pK 0


+ log Cs .
CA
Here, Cs= O.l(M), CA= 0.l(M)

Hence, pH = -log(l.8 x 10- 5) + log~ = 4·74.


0.1
lee of lO(N) HCI, i.e., 10-2 mole HCl is added. It will react with 1o- 2 mole CH 3COONa
to produce CH3COOH.
Thus, Cs = 10- 1 -.10- 2 = 0.09 mole
CA = 10- 1 + 10-2 = 0.11 mole

Hence, ;pH

= PKa + 1 0.09
og~-
0.11
= 4.74 - 0.09 = 4.65.
Thus, pH w~ll decrese by 0.09 unit.
53. Using Debye-Htickel limiting law calculate the activity of a 2 - l electrolyte at
a molarity of 0.01 aqueous solution at 15 °C. [A = 0.50 at 15 °C]
Soln. : 0.01 '~olar 2 -1 electrolyte (MA 2) dissociate as MA 2 = M+ 2 + 2A-.
Hence, CM+2 = O.Ol(M) CA- = 0.02 molar.

So, ionic strength, µ = ..!.. [0.01 x 22 + ·0.02 x 12] = 0.03 ..


2
logy± = - 0.50 x 2 x l x ,,,Jo.03:
:. mean ionic activity coefficient of MA 2 = 0.67.
Thus, activity of the electrolyte MA2 = a2 = aM+2 a!-

= (C~+2'Y M+2 )(Cr"( A- )2


= 0.01 x 0.022 x ·'Y~
= 0.01 x 0.022 x 0.673
= 1.2 x to':'"6 ,
CHAPTER13

EMF

Required Formulae
1. Work done in carrying positive charge from point a to p0int b,

. = qi (<l>b - <l>a) = q;L\<I> (<l>b > <l>a),


W
where <l>b = electric potential at b; <l>a = electric potential at a.
If Z; = valency of an ion,
W = Z;el\<I>.
2. Work done on potentiometer for the passage of electrons from anode to cathode
in a galvanic cell.
W = -nFE,
E = emf of the cell.
n = number of moles of electron required to write a balanced cell reaction.
3. L\G =W = -nFE,
where L\G is the free energy change in joule. ~is equation is strictly true for reversible
cell potential.

~· (ill = nF { ~ ~~) P- E}.


where Ml
0E)
. change
= enthalpy for a cell reaction

and ( = temperature coefficient.


'OT P

5. E = ~~ In ~: , wher'e Ka = equilibrium constant for a reaction takjng place in a


cell,
Qa = reaction quotient, R = 8.314 J.mole-t.K-'.

448
EMF 449

6. E z+
IM
= IfJM z+ ~ + 0.05916 Jog.a at 25 oc,
M
Z M~
where E = electrode potential in volt.
MZ+(M

= standard electrode potential in volt.


~Z+IM
= activity of Mz+.

7. E = 2RT In.fl.. where C2 and C 1 are concentration of the species in a dilute


F · C1
solution.
E = emf of the concentration cell without transport.

8. E = t_ 2~T Jn%-, ~here t_ = transport number of.negative ion.


E = emf of the concentration cell with transport.

9. Eup = (r - t ) RT In (a± )2
- + F (a±) '
1
where t_ and t+ are the transport number of -ve and +ve ions respectively. (a±) 1 and (a±)2
are the mean ionic activities of solutions 1 and 2 in Uffi and RHE respectively.
10. µMz+ = µ Mz+ + z.qi.F.
1
where Z = valency of the ion
if> = electric potential
F =Faraday
µ + = chemical potential of the ion
Mz
j.I + = electrochemical potential of the ion.
Mz
Short Questions
1. Define Electrochemical potential.
Ans. : The difference in chemical potentiaf of an ion with ana without the electric
potential is ZFlf>, where if> is the electric potential and ZF is the charge per mole on the
ion. The chemical potential of an ion ili the presence of an electric potential ZFlf> is the
electrochemical potential j.I. It follows that iI; = µ; + Z;FIf>, where iI; = electrochemical
potential, µi = chemical potential, Z; = valency of ith ion.
When there is no charge (Z; = 0), the chemical and electrochemical potential are equal.
2. What is the meaning of the statement?-The standard electrode potential of the
Fe3+/Fe 2+ is 0·77 volt.
Ans. : It means that the potential of the electrode Fe3+/Fe2+ where the reaction is Fe3+
+ e ~ Fe 2+ at unit activities of the ions is 0.77 volt with reference to that of standard
hydrogen electrode potential taken as zero. This is the standard 'emf of the cell-
Pt, H 2 (g, I atm) I H+ 11 Fe3+ I Fe 2+, Pt, i.e., it is true einf of the cell.

Prob. Phy. Chem.-29


450 PROBLEMS ON PHYSICAL CHEMISTRY

H+ Fe 3 + Fe 2 +, Pt
Pt. H 2 (g, I atm)
a H+ =I (a ;+ = I a +z =I
Fe 1) Fe

3. Does the emf of a cell depend on temperature?


Ans. : The emf of a cell should depend on temperature.
4. Is the emf of a cell measured potential of a cell?
Ans. : No, it is not the measured potential of the cell. It is the measured potential
when the electrodes are joined by a wire of infinite resistance, i.e., when no current is
flowing and no net reaction is taking place within the cell.
5. Does positive E 0 refer to feasibility of a reaction under the given ·set of condition?
Ans. : The positive E 0 only indicates that the given reaction at unit activities has
an equilibrium constant K greater than unity. Thus, when equilibrium is reached, the product
may have a predominance in the given case. But, it tells nothing about rate, nothing about
whether it will actually proceed towards right under the given set of condition or even
at unit activities.
6. The standard potentials of Cu++ I Cu and Zn++ I Zn systems are reported with different
signs, Why? Which one is positive according to your convention?
Ans. : The standard polentials £ 0 are reported with (+)ve or (-)ve signs with respect
to that of standard hydrogen electrode, protential of which is taken as zero. The hydrogen
electrode is taken as either left-hand electrode or right-hand electrode in two different
conventions. Thus, reported values of E 0 refer to a reference cell. If the emf of the cell
I I
with Cu++ Cu is positive and ~;iving current at unit activities, then emf of the cell with Zn++ Zn
is negative and is not giving current and vice versa, and that is why they are reported with
different signs in our conventions which is the international convention wher~ the hydrogen
electrode is L.H.E. Thus, £ 0 of Cu++ ICu system is the standard emf of the cell,
Pt, H 2 (g, I atm) I H+ 11 Cu++ I Cu,
where the reaction is H 2 + Cu++ =Cu + 2H+ per two faraday. The reaction has equilibrium
constant K > I at unit activities and E 0 is +ve.
I
E 0 of Zn++ Zn system refers to the cell Pt,, H 2 (g, I atm) I H+ I IZn++ IZn.
Here, equilibrium constant K < I at unit activities and E° is negative.
7. Why is the decomposition potential of most acid solutions 1.7 volts?
Ans. : The decomposition voltage of an electrolyte is the minimum voltage that must
be applied to a particular solution of an electrolyte in order that current of appreciable
strength shall be able to pass. The decomposition voltage of aqueous solution of most acids
and bases were found to be around 1.7 volt. This leads to the conclusion that the same
electrolytic processes are occuring in each case, that is, the decomposition of water to
hydrogen and oxygen. The theoretical ~oltage for this process is 1.23 volt. The extra value
is due to polarisation of electrodes.
8. Decomposition voltage of dilute salt solutions are greater than decomposition voltage
of common acids.-Explain.
'.!!'·
Ans. : The electrolysis of dilute solution of salt like Na 2S0 4 , K 2 S04 , etc in electrolytic
cell also lead to electrolysis of water. But in such cases, the decomposition voltage is greater
EMF 451

than the voltage needed for electrolysis of dilute acids (i.e., 1.7 volt). This is beleived
to be due to the fact that acid is created following the removal of O~ at the anode, and
alkali is created at the cathode following the removal of W ions. Thus, in those cases,
the ions are not discharged from the same solutions. ·w ions are discharged from a more
alkaline and OH- ions are discharged from a more acidic solution resulting in an increase
in overvoltage and decon1Position voltage.
9. "Passage of electricity through dilute solution of H 2S04 , H 3P0 4 , etc. leads to the
decomposition of water, but passage of electricity through dilute solution of HCI leads
to the decomposition of HCI at a lower voltage."-Explain.
Ans. : This is due to over~oltage. The theoretical decomposition voltage of water
is 1.23 volt. Due to overvoltage, from dilute acid solution like H 2S0 4 , H 3P0 4 , etc., the
decomposition starts at 1.7 volts. Halogen acids like HCI decompose at a lesser voltage.
Because of negligible overvoltage, HCI decomposes at around 1.3 volts. (discharge potential
of c1- = 1.3 volt) and it decomposes to H2 and Cl 2 • However, if the voltage rises to 1.7
volt, water begins to get electrolysed from dilute HCI solutions.
10. "Passage of electricity through dilute solution at both H2S04 and Na 2S0 4 leads
to the electrolysis of water, but in the second qise, higher voltage is needed."-Why?
Ans. : [Hints : See Question no. 8.]
11. Explain why--
KC! or NH 4N0 3 is preferred to make salt bridge.
Ans. : The salt bridge is used for the purpose of eliminating liquid junction potential.
(LJP). The expression of LJP is

MX(a±)1iMX(a±) 2

Eup = (t_ - t+) RT In (a±)2'


F (a±)1
where t+ and t_ are transport numbers of cations and anions. (a,..)'s are mean ionic activities
'r
and all other terms have usual meaning.

Now, velocities at K+ and c1- or NH~ anq N03 are nearly same. i.e., t+ = t_. So
LJP is almost eliminated. That is why KC! or NH 4N0 3 is preferred to make salt bridge.
12. The standard potential of hydrogen electrode is taken as zero at all temperatures.-
Explain. · ... -.

An.s. : The standard potential £ 0 and emf E of cell depend on temperature. E° ·


. H""\82: '
is taken as zero at 25 °C. It may not be zero at other temperature, but then we can only
determine the temperature coefficient of emf of cells. The temperature coefficient of electrode
cannot be found out. For determining temperature coefficient of electrodes, £ 0 of H+ H 2 I
electrode may be taken as zero at all temperatures. Then the determined temperature coefficient
of a cell with H+ IH2 as L.H.E. will be the temperature coefficient of R.H.E.
13. Distinguish between single electrode potential and standard electrode potential.
Ans. : Single electrode potential is the electrode potential of an electrode whereas
I
standard electrode potential £ 0 is the emf of a cell with H+ H 2 as L.H.E. at unit activities
of all reactants and products.
452 PROBLEMS ON PHYSICAL CHEMISTRY

14. Can you use quinhydrone electrode above pH = 8? Explain.


··Ans.: Quinhydrone electrode is an electrode where the reaction is Q + 2H+ + 2e = QH2•

V
0 00 •

where Q is quinone 3:nd H2Q is Q.In alkaline medium, above or at pH = 8, quinone


O OH

may be attacked by Off to give different products· of aldol type and the equilibrium is
disturbed. So, it cannot be used at or above pH = 8.
15. Explain : The density of H 2S04 in a lead storage battery increases as it is charged.
Ans. : The reaction in the acid storage cell is :
Pb0 2 + Pb + 2H2S04 = 2PbS04 + 2Hz0.
The reaction from right to left is the reaction which takes place during charging. Thus,
H2S04 is produced and Hp is consumed. That is why the density of H 2S0 4 increases.
16. c1-, AgCl(s) I Ag electrode can be treated as Ag+ I Ag electrode. Explain.
Ans. : The reaction taking place in the electrode CC, AgCl(s) I Ag is
AgCI = Ag+ + c1-
Ag+ + e =Ag
Adding, AgCI + e = Ag + er
But AgCI as shown dissociates completely to Ag+ whichever amount of AgCI goes
I I
in solution. Therefore, er, AgCI Ag can be treated as Ag+ Ag electrode, with the restriction

that a + = K, . K5 is the solubility product of the cell.


Ag a
Cl-

17. For ~n irreversible cell nFE' ~RT In Keq-Explaii:i


Ans. : The electriciil work in a cell for the passage of charge nF is' Wet = nFE 0

0
But this is equal to -LiG for a reversible cell only. Thus, for reversible cell, nFE' =RT In K.
This is not)rue for irreversible cell, because the Nemst equation is not applicable for these
cells.
THE CONSTRUCTION OF CELLS :
The cell construction for any reaction involves the following steps
(i) Writing the reaction ionically, if possible.
(ii) Selection of redox pairs or choosing the case where reduction (consumption) of
an ion or a substance has taken place. That system will constitute the R.H.E.
(iii) Choosing the •case where oxidation has taken place. This system will be L.H.E.
If some reactant or product of R.H.E. is absent in the final equations, then the
L.H.E. should contain th.at reactant or product. In some cases, we may have to
choose L.H.E. first. Then the above applies for R.H.E.
(iv) Then the probable cell has to be written, and the cell reaction has to be checked.
If .it fits (usually it will fit), then the constructed cell is the required cell.
Remember that solution of any salt contains the ions. However, weak electrolytes and
insoluble salts cannot be written ionically. Also remember that production of positive ion
0

and consumption of negative ion are comparable to oxidation for our electrode construction
EMF 453

process. Similarly, consumption of positive ions and production of negative ions are
comparable to reduction. Any electrode should be reversible with respect to at least one

io_n. For example, let us consider the reaction H 2 + -!oz = HzO.


In this reaction, Oz is consumed. Therefore, there must be one electrode containing
Oz, i.e., OH-j0 2 , Pt. where the reaction is lo 2 +Hp+ 2e = 20H-. Thus, it is the R.H.E.
2
But the final equation does not contain OH-. So it must be consumed in L.H.E. With Hz
remaining, the electrode at R.H.E. should be OH-IH2 . As L.H.E. the electrode is Pt, Hz(g:
I
l atm) OH-. Hence, the probable cell :
I
Pt, H2 (g, l atm) ott-1 Oz (g, l atm), Pt
18. Construct cell where the following reactions take place :
(i) 2KMn0 4 + 3H2S0 4 + 5Hz0 2 = KzS0 4 + 2MnS04 + 8Hp + 502
Ans. : Writing ionically :

2K + + 2Mn 04 + 6H+ + 3 so~- + 5Hz0 2

= 2K+ + so~- + 2Mn 2+ + 2 so~- + 8Hz0 + 50z

2Mn 04 + 6H+ + 5H20z = 2Mnz+ + 8Hp + 50 2

In this reaction Mn04 is reduced to Mnz+. Thus, Mno41Mn 2+ should be R.H.E. and
remaining 0 2 and HzOz should be a part of L.H.E. The electrode is Oz (g, I atm) IHz0 2 ,
H+ as L.H.E. Therefore, the cell is :

Pt, Oi(g, l atm) I HzOz. H+ I I H+; Mn 04 I Mn2+, Pt


The reaction in this cell is as follows :
Reaction at R.H.E. : [Mn 04 + SH+ + Se = Mn2+ + 4H 20] x 2
Reaction at L.H.E. : Oz + 2H+ + 2e = H 0z
2 as R.H.E.
Reaction at L.H.E. : [H20 2 = Oz + 2H+ + 2e] x 5

Adding : 2Mn 04 + 6H+ + 5Hz0z = 2Mn + + 8Hz0


2 + 50z

(ii) -!-Hz + -!-c1 2 = HCl


Ans. : Writing ionically : -!Hz + iClz = H+ + er.
Hence, the cell is :
Pt, H2 (g, l atm) IH+, c1- I Clz (g, l atm), Pt.
The reaction will fit. Therefore, tPe written cell is the required cell.
(iii) HCl + NaOH = NaCl + HzO
Ans. : Writing ionically : H+ + Cl- + Na+ + OH- = Na+ + CJ- + Hp.
Thus, the net reaction is H+ + OH- = HzO.
In this reaction, H+ is consumed, therefore, H+ I Hz electrode will be R.H.E., where
the reaction is H+ + e = --!- Hz.
454 PROBLEMS ON PHYSICAL CHEMISTRY

Final equation does not contain Hz. It must be consumed at L.H.E .. With OH- remaining
the L.H.E. as R.H.E. is :
O.W I Hz I
(g, I atm). Hence, as L.H.E. it is Hz(g, I atm) OW. The reaction at
I
L.H.E. is 2H2 + Oir = H 20 + e. The cell is, thus,
Pt, Hz(g, I atm) IOH- I IH+ I Hz (g, I atm), Pt.
(iv) Zn + PbO = ZnO + Pb.
In this reaction PbO is reduced to Pb. Thus, R.H.E. should be OW, PbO(s) Pb(Hg), I
Pt.
Naturally Zn is oxidised to ZnO. Hence, L.H.E. is Zn I ZnO (s), oH-. Thus, the
probable cell is :
Zn I ZnO(s); OW, PbO (s) I Pb(Hg), Pt.
The reaction taking place in the cell at the R.H.E. ..
PbO + Hp = Pb + + 20H-
2

Pbz+ + 2e = Pb
Adding, PbO + Hp + 2e = Pb + 20W ·
Obviously the reaction at L.H.E. would be :
Zn + 20H- = ZnO
+ HzO + 2e
Adding : Zn + PbO = ZnO + Pt
(v) 2Ag + HgzC1 2 = 2Hg + 2AgCl
The cell is "•g IAgCl, CJ-, Hg2c1 2 IHg, Pt.
Proceeding in the line of (iv) the cell may be constructed as shown above. Both the
electrodes are reversible with respect to er.
19. Construct cells where the following reactions take place.
(i) P4 + 3NaOH + 3Hp = 3NafI2POz + PH3
(ii) 3Fe 2+ = 2Fe3+ + Fe
(iii) CaCl 2 + Na 2C03 = CaC0 3 + 2NaCl
(iv) CuS0 4 (C 1) = CuS04 (C 2)
(v) K2Cr20 7 + 7H2S04 + 6KI = 4K 2S04 + Cr 2(S0 4) 3 + 7Hp + 312
(vi) Ag+ + Br - = AgBr
(vii) Agl = Ag - + 1-
(viii) ZnC1 2 = Zn + Cl 2
(ix) PbO + H 20 = PbZ+ + 20H-

(x) lClz + Br - = c1- + lBrz


2 2
(xi) PbC1 2 (s) + 2Agl (s) = 2AgCl (s) + Pbl 2 (s)
Hints : Proceed as the usual method, choose redox pair. Only in the last cell the
reaction probably does not take place in water solution. However, we can construct cell-
both the electrodes are 3rd kind.
EMF 455

In this reaction Pbl 2 is produced. Therefore, Pblz should be a part of R.H.E. with
Ag, Agl present. The electrode is :
Pb 2+, Pbl 2(s), Agl(s) I Ag,
where the reaction is :
2Agl = 2Ag+ + 2I-
2Ag+ + 2e =Ag
Pb 2+ + 21- = Pbl 2
Adding : 2Agl + 2e + Pb 2+ = Pbl 2 + 2Ag
The L.H.E. is Ag I AgCl (s), PbClz(s), Pb 2+.
Now the reaction at L.H.E. as R.H.E. :
2AgCI = 2Ag+ + c1-
2Ag+ + 2e = 2Ag
Pb 2+ + 2c1- = PbCl 2
Adding : 2AgCI + Pb 2+ + 2e = 2Ag + PbC1 2
Now at the L.H.E. the reaction is 2Ag + PbCl 2 = 2AgCl + 2e + Pb 2+
Adding R.H.E. and L.H.E. : PbC1 2 + 2Agl = 2AgCI + Pbl 2
Hence, the probable cell is :
Ag IAgCI (s), PbClz(s), Pb 2 +, ~bl 2 (s), Agl(s) IAg.·

a( E 0
IT) Aff 0
20. Show that, ( ) = --.
() l/ T nF

Soln. :
a(E°tT) a(-tiG 0 /T) .
= __,__ ___.... (smce b.G 0 = -nFE0 )
()(JI T) nFd(I IT) '

= nFI [ b.Go I ()b.G ]


T ()(I IT)

= - I [ -b.G 0 + T
nF
()b.G
--
()T
0
J (smce,
. ()(-TI ) = --I dT)
T2
0
Aff • db.G 0
== - - - (smce b.G 0 = Aff0 - Tb.S = Aff0 +T - - )
nF ' ()T .
. Numerical Problems
0 0
0 0

l.Giventhat Es n4+js n+2=0.15V and ESn +21 Sn =-0.136V.Calculate µ sn 4+, µ sn +2 and


Eo
sn 4 +1sn.
0
0

Soln. : µ = ZF. E 21 Sn . [Z = no. of charge, F = Faraday]


+
Sn+2 Sn

= 2 x 96484 x (- 0.136) J.mole- 1 = -26.2 kJ.mole-1.


456 PROBLEMS ON PHYSICAL CHEMISTRY

The reactions are


Sn4 + +'2e ~ Sn 2+!!.G 0 = -2 x F x 0.15V
Sn+ 2 + 2e ~Sn !lG0 = -2 x F x (-0.136)V.
Adding Sn 4+ + 4e ~ Sn!lG 0 = -2 x F x 0.014 V.
. . -2 x F x 0.014 =- 4 x F x E°.

( £0Sn 4 +1sn ) = -
0.014 = 0.007 v.
2- .
µ 0 4+
Sn
=4 x 96484 x 0.007 J.mole- 1 = 2.70 kJ.mol- 1•

2. Given that t;n+2lsn = - 0.14V and E~++jpb = - O. I 3V. Cal6ulate the molarity of
Sn++ ion when Sn is added to a 0.02 M solution of Pb(N0 3) 2 at 25 °C.
Soln. : The reaction is Sn + Pb++ = Sn++ + Pb. This reaction occurring fri the cell
Pt Sn I Sn++ I I Pb++ I Pb(Hgf, 'i>t
E~eu
= -0.13 - (-0.14) = --0.13 + 0.14 = 0.01 volt.
.. !!.Go = -RT lnK = -nFE".
.. In K = nF .E° = 2x96484x0.0l = 0.778467456.
RT 8.314x 298.15
.. K = 2.178 =-2.18.
Let the molarity of Sn++ =x at equilibrium, then that of Pb++ = 0.02 - x.
c
The equilibrium constant of the reaction in the cell is Csn++
Ph++
x
.·or, = 2.18
0.02-x
or, 3.18 x = 0.0436 or, x = 0.0137

i.e., csn++ = 0.0137 (M).


3. What is the equilibrium concentration of Fe++ when a piece of iron is dipped in
a 0.02 (M) CdS0 4 solution at 25 °C. "

Given, E~e++!Fe = --0.441V, E~d++icd = -0.403V.


Soln. : Proceed as Problem 2.

C
Fe++
= 0.019 (M).

1 ++ = 1.51 V and E° ++ = 1.23 V in acid illution. Calculate


0

4. Given that £ _
Mn0 4 Mn Mn02 1Mn
Eo
Mno 41Mno 2 in acid solution.

Solo. : Mno- + SH+ + 5e


4
~ Mn+++ 4Hp, !lG 0 = -5F x (1.51)
Mn++ + 2H20 ~ Mn0 2 + 4H+ + 2e, !lG 0 = -2F (-1.23)
EMF 457

Mn04 + 4H+ + 3e ~ Mn02 + 2H20~ !J.G 0 = -3FE


0
Adding,
:. (-7.55 x F) + (2.46 x F) = -3F x E°
or, -7.55 + 2.46 = -3 x E°
5.o9
•·• E0 -- -
- - . VO It -- 1696
, VO It,
-3
5. Calculate !J.G at 25 °C and 1 atm for reaction lH 2(g) + AgCI =Ag+ H+ +er.
2
Given that E of the cell Pt, H 2(g) HCI, AgCI I
Ag is 0.35252 V at 25 °C. I
Soln. : Here, E = 0.35252 V,
, F = 96484 coulombs

For the r~action, lH2(g) + AgCI .,; Ag + H+ + Cr, n = I.


2
:. !J.G = -nFE
= -1 x 96484 coloumbs x 0.35252 volt = -34012.53 J = -34.01 kJ.
6. For the cell H 2 (l atm) I HBr (a = l), AgBr IAg, E is given as
E = 0.07131 - 4.99 x 10-4 (t - 25) -3.45 x 10-6 (t - 25)2, where t is temperature
in °C. Cakulate !J.G, AS and Aff in joules for the reaction taking place in the cell at 25 °C.
Soln. : E = 0.07131 - 4.99 x 10-4 (t - 25) - 3.45 x 10-6 (t - 25) 2

Thus, (aE)
'OT P
= - 4.99 x 10-4
.
- 3.45 x 10-6 x 21 + 2 x 25 x 3.45 x 10-6

Hence, at 25 °C ( ~~) P = -4.99 x 10-4 - 3.45 x 10-6 x 2 x 25 + 2 x 25 x 3.45 x 10-6

= -4.99 x 10-4 volt.K-1.


E = 0.07131 volt at 25 °C
The reaction in the cell for the passage of 2 faraday :
2 AgBr + 2e = 2Ag + 2Br-
H2 ,= 2H+. + 2e

2AgBr + H 2 = 2Ag + 2H+ + 2Bc


Here n = 2.
Hence, !J.G = - nFE = - 2 x 96484 x 0.01731 J = -13760.55 J = -13.761 kJ.

!:JS = nF(oE) = 96.29 J.


'OT P

Hence, Aff = nF { ~ ~~) E} P-

=2 x 96484 coulomb (298.15 x (-4.99 x 10- 4 ) - 0.07131} volt


= - 42469.72 J = - 42.47 kJ,
7. For a particular cell, E at 20 °C, 25 °C and. 30 °C are 0.0663 V, 0.06839 V and
0.07045 V respectively. Calculate !J.G, !:JS and Aff for the reaction taking place in the cell
at 25 °C.
458 PROBLEMS ON PHYSICAL CHEMISTRY

Solo.: ((JE) = E2 -E1 = 0.06839-0.0663 = 4.18 x 10-4 volt.K-1.


CJT p • T2 -Ti 5

Again, (g~)P = 0.07035;0.06839 = 3.92 x 10-4 volt.K-1 .

. . taking average, ( g~) P = 4.05 x I o- 4 volt.K- 1.

.. T(~~)P = 298.15 x 4.05 x 10-4volt at 25°C.


Now, liH = nF { T( ~~) P _ E} I = x 96484 (298.15 x 4.05 x Io- 4
- 0.06839)

= 96484 (0.12075 - 0.06839) [taking n =I for this cell]


= 5.051 kJ.
tiG = -nFE = -I x 96486 coulombs x 0.06839 volt [taking n =I for this cell]
= - 6.598 kJ =- 6.6 kJ.

liS = nF(~E) = I x 96484 coulombs x 4.05 x 10- 4 volt. K-1 [n = I]


uT PA .
= 39.076 J.K-1.mot-t.
8. The emf of the cell Cd ICdC1 2 (Im), AgCl (s) IAg is 0.675 volt at 25 °C. The
temperature coefficient of the cell is - 6.5 x I0-4 volt/K. F.ind the changes in heat content.
and entropy for the electrochemical reaction that occurs in the cell when one faraday of
electricity is drawn from it. [C.U. 1975]
Hints : Proceed as Problem 7.
In the cell, for the passage of I faraday the reaction is
AgCl + e = Ag + c1-

.!.cd = .!.cct+2 + e
2 2

AgCl + .!.cd
2
=Ag + .!.cd+2 +c1-
2
liH = - 83.83 kJ/mol, tiS = - 62.72 J.K-1.mo1-1.
9. At 0 °C, a calorimetric determination of liH for the reaction Zn + 2AgCl = ZnC1 2
+ 2Ag yields the ~alue -52.50 kcal. The emf of the cell is 1.015 volts. Calculate the
temperature coefficient of the cell. [C.U. 1979]
Solo. : liH = -52.50 kcal = -219.66 x 103 J.
For the cell reaction n = 2; T = 273.15 K. E = 1.015 volt.

We know, liH = nF { r( ~~) E} , where ( ~~)


P_ P is temperature coefficient.

:. -219.66 x 10
3
=2 x 96484 {273.15(~~)? -1.015}
or, (g~)P = -4.514 x 10- 4 volt.K- 1.
EMF 459

I I
10. The Pb PbCl 2 (fused) Cl 2 (g), l atm shows an "emf :
E = 1.2467 - 6.50 x 10-4 (t - 55) V; t is expressed in °C.
What are the values of ent~alpy and entropy of formation of PbCl 2 at 500 °C?
Solo. : Proceed as problem (6). Here n =2
Aff = -281.73 kl/mole
f!,,S = -125.43 J.K-l.mole-1
11. The emf of the cell (V) glass electrode I buffer solution 11 calomel electrode was
found to be 0.0232 volt at 25 °C when the buffer has- a pH = 2.5. The emf increased to
0.112 volt when another buffer solution is used. What is the pH of the latter buffer?
[C.U. 1979)

Solo. : By the problem, E = Ecal - ( E~ - 0.059 pH)


.. 0.0232 = (£cal - £~) + 0.059 (2.5)
or, 0.0232 = x + 0.1475 (where Eca1 - E~ = x).
. . x = 0.0232 - 0.1475 = - 0.1243.
Under new condition, E = 0.112 volt,
:. 0.112 = -0.1243 + 0.059 pH

or pH= 0·112+0·1243 = 4005 "' 4.


' 0·059 .
12. The emf of the cell obtained by coupling through a salt bridge with quinhydrone
electrode (E° ::::: 0.6994 V) dipping into a buffer solution and a saturated calomel electrode
(E = 0.2415 V) is 0.0042 V. What is the pH of the buffer? [C.U. 1982)
Solo .. : Ecen = EQuinhydrone - Ecalomel
::::: (0.6994 - 0.059 pH) - 0.2415 = 0.4579 - 0.059 pH
Given, Eceu = 0.0042
0.0042 = 0.4579 - 0.059 pH
pH ;:::: 7.689 = 7.7.
13. Calculate the emf of a cell at 25 °C which comprises a saturated calomel electrode
coupled through a salt bridge, with a platinum black electrode dipping into a solution A
and H 2 gas at l atm is bubbled over the platinum surface. The solution A was prepared

by mixing 10 ml ( ~ )NaOH solution with 90 ml ( 1 ~ )Hcl solution. (Oxidation potential


of saturated calomel electrode =-
0.242 V at 25 °C) · [C.U. 1976; V.U. 1989]
Solo. : Eeeu ::::: Eca1 -EH+/H = 0.242 - 0.059 log aH+ = 0.242 + 0.059 pH.
2

~) NaOH with 90 ml ( ~ )HCI.


Now, the solution A is prepared by mixing 10 ml (
1
[90 cc of(i~) (M) is 0.9 cc of l(M)

10 cc O.l(M) =lee l(M)]


. . C0 H- = O.lcc l(M) in 100 cc.
460 PROBLEMS ON PHYSICAL CHEMISTRY

Thus, C0 H- in NaOH = -2:.!_(M) = 10-3 (M).


100
.. pOH = 3.
.. pH = 14 - 3 = 11 at 25 °C.
.. Eceu = (0.059 X 11) + 0.242 = 0.891 V.
14. Write down the cell reaction and calculate the emf, the free energy change and
the equilibrium constant of the reaction taking place at 25 °C of the following cells :
[C.U. 1984)
(i) H2 (I atm) IHBr (a± = 0.,2), Hg2Br2 (sf IHg
Given, £ 0
HglHg2Br2(s).Br-
= - 0.1385 V.
Soln. : Reaction for the passage of two faradays :
Reaction at R.H.E. : HgzBr2 + 2e ~ 2Hg + 2Bc
Reaction at L.H.E. : Hz ~ 2H+ + 2e
.. cell reaction : HgzBrz + Hz ~ 2Hg + 2H+ + 2Bc

Given, ~glHg 2 Br 2 .Br- = - 0.1385 V


:. E = (0.1385) - 0.059 log aH+.a8 r
= 0.1385 - 0.059 log a±2 [a± = mean ionic activity]
.. E = 0.1385 - 0.059 x 2 x Jog 0.2
= 0.1385 + 0.0821 = 0.220 v.
.. !lG = -nFE = -2 x 96484 x 0.220 J = - 42.64 kj.
Now, RT In K = nFE°

or, In K = nFE°, = 2 x 96484 x 0.1385


RT 8. 314 x 298.15
or, In K = 10.78177
:. K = 4.81 x 104.
(ii) Zn Zn++ (a = 0.01) 11 Fe 3+(a=0.1)IFe++(a=O.OOI) I Pt

Given, £0 = 0.761 V
Znlzn++
E° = - 0.771 v.
Fe 2+1Fe 3+
Soln. : The cell reaction is Zn + 2Fe3+ = zn++ + 2Fe 2+.

( a 2+)2.a ++
-~
.. E = (0.771 + 0.761)
2
log Fe
( aFe3+ r. azn
Zn

2
or, E = I.532 _ 0.059 lo {0.001) x {0.01)
2 g {0.1) 2 x{I)
EMF 461

or, E = 1.532 - 0.059 log 10- 6


2
.. E = 1.532 .+ 0.059 x 3
.. E = 1.702 V.
.. !l.G = -nFE = -(2 x 96484 x 1.702) J = -328.43 kJ.
.. !l.G = -nFE° = -RT In K
0

or, In K = 2x96484x 1.532


8.314 x 298.15
:. K = 6.229 x 1051 .
15. The potential of the cell, Zn (s) I ZnCl 2 (0.01021 M), AgCI IAg was found to
be 1.1566 at 25 °C. Given that E° ++i = -0.7618, E° _ = 0.224. Calculate Y+·-
Zn Zn Cl .AgCI Ag 1

Solo. : The cell reaction is Zn + 2AgCI ~ Zn++ + 2Cl- + 2Ag

Now, E° -E° ·
Cl-,AgCllAg zn++lzn

= 0.224 - (- 0.7618)V
= (0.224 + 0.7618)V = 0.9858 V.

E = (EoCl- AgCllAg _ Eozn+2izn )- -


0.059 2 .
••
· 2- (ogaZn +2 .aCl -
Since ' for the salt M xA Y' axay
+ -
= r"vcx+y
.r y±x+y
a a2 = 1l22c3y 3 = 4c3y 3.
zn+ 2 c1- ± ±

Hence, 1.1566 = 0.9858 _0.059


--log
( 4 C3
. +·'Y+
3)
2 - -
or, l.1566 = 0.9858 - 0.0591 x log (2) - 3x 0.059 Jog Cy+
2 -
or, 1.1566 = 0.9858 - 0.01776 + 0.176238 - l.5 x 0.059 logy±.

:. log 'Y+ = _ 0.01232 .


- l.5x0.059
or, y± = 0.726.
16. What will be the emf of the cell at 25 °C,
H 2 (l atm) I 0.5 (N) formic acid II l(N) acetic acid I H2 (l atm),
if the diffusion potential be neglected? The dissociation· constants of formic and acetic
acids are 17.7 x 10- 5 an~ 1.8 x 10- 5 respectively: Which will act as the positive electrode?
[C.U. 1975; V.U. 1988]
Hints : For the passage of one faraday I gm atom of H+ ions passes from 1(N) AcOH

a
solution to 0.5 (N) formic acid solution, emf of the cell is - RT In tt+(o.5NHCOOH)
F a
H+(I NAct>H)

But CH+ of a weak acid with acidity constant K0 is ~ c. K , where C concentration


0
=
of the acid, assuming CH+ == aH+. since activity coefficients are unknown.
462 PROBLEMS ON PHYSICAL CHEMISTRY

I
(8.85x 10:_5)2 ___ 0.059 log 8.85
E = - 0.059 log I
2 1.80
= - O.02 VO
I
t.
(1.8x 10-5 )2

Thus, R.H.E. should be -ve electrode and L.H.E. should be +ve electrode.

17. For the reaction Fe= Fe 3+ + 3e; E"


,
=0.036 V and E 0 3+ 2+ = 0.771 V. Calculate
Fe 1Fe
~e2+IFe. Is the reaction Fe + 2Fe3+ = 3Fe 2+ spontaneous in the forward or the reverse
direction? [Delhi University (Hons.) 1977]
Soln. : Given
Fe3+ + 3e ~ Fe; - 0.036 V · -3 x F x (- 0.036)
Fe2+ ~ Fe 3+ + e; -0.771 V -1 x F x (- 0.771)
Adding, Fe 2+ + 2e ~ Fe; -2xFxE"

·. E" ___ {3x0.036+1.77)


. = - 0.439 v =- 0.44 v
2
The cell in which the reaction,
Fe + 2Fe3+ ~ 3Fe2+, takes place is Fe I Fe2+ I I Fe3+ I Fe2+, Pt.
The E" of the cell is E" = (0.771 + 0.44) volt = 1.21 V.
So, the reaction is spontaneous in forward direction.
18. Calculate the pH of the solution for which a quinhydrone electrode E 0 = 0.6944 V
coupled with a saturated calomel electrode E = 0.2415 V through a salt bridge shows no
measurable emf~ [C.U.(H) 1984; V.U. 1988]
Hints : Proceed as before. By the. problem, 0.4529 V = 0.059 pH.
:. pH= 7.76.

19. The:. ~"lubility product of Fe(OH)3 at 25 °C is 10- 38 · 1 and E° 3+ = - 0.036 V.


Fe Fe 1

LaKulau;. tbe standard emf for OH-. Fe(OH)3 Fe electro?e. I


Solo. : We know that E" (2nd kind with an insoluble salt MS)

E':.cell = RT
nF In Ks.
0 059
= · in Ks, where the temperature is 25 °C.
n
0 0

E • = E + 0.059 JogK
OW,Fe(OH)3iFe Fe 3+!Fe -3- s·

0 059
= -0.036 + · JogJ0-3 8·1 = -0.036 - 0.059x38.l = -0.785 V.
3 3
20. What is the equilibrium concentrat.ion of NF+ when a piece of iron is dipped in

a 0.005 (M) solution of NiS0 4 at 25 °C. Given that E


0

+i
Fe 2 Fe
= - 0.441 volt and

E~i++iNi = - 0.24 v.
EMF 463

Solo. : The reaction is Fe + Ni++ = Fe++ + Ni which occurs in the cell


Fe I Fe++ I· I Ni++ I Ni
£ = - 0.24 + 0.441 = 0.201 v.
0

.. l!J.G 0 = -nFE0 = -2 x 96484 x 0.201 = -RT In K.


.. K = 6.2444 x 106.
K is so high, almost all of Ni++ will react to give Fe 2+. Let coefficient of Ni++
remaining be x. Thus, CFe 2+ = 0.005 - x. Now, ass~ming activity = concentration,
6.2444 X 106 = ~Fe2+ = 0. 005 - ~
Ni++ X

Neglecting x in comparison to 0.005,


5
concentration x = 0.00 = 8.0071 x lo- 10 .
6.2444xl0 6
, Thus, the equlibrium concentration of Ni++ will be 8.01 x 10 -to molar.
21. An aqueous solution containing 0.01 (M) Fe (Cl0 4h. 0.01 (M) Fe(Cl0 4 ) 2 and
0.01 (M) HC10 4 was titrated with a concentrated NaOH solution at room temperature
(30 °C), so that changes in volume were negligible. Calculate the redox potential of
Fe 3+ IFe 2+ system at pH values 2.2, 4.2, 6.0, 8.2 and 10 assuming new species formed
during titration were Fe(OH) 3 and Fe(OH) 2 only. [E° for system Fe 3+ IFe+ 2 = + 0.77 V.
Solubility product of Fe(OH) 3 = 10-3?.l and that of. Fe(OH) 2 = 10-18.4 [C.U. 1973]

Solo. : The redox potential of Fe3+ IFe 2 + system by the problem is

2.3026RT. aF 3+
E = 0.77 + log-e-
F a Fe 2+
a 3
= 0.77 + 0.06014 Iog-a--
Fe + at 30 °C
Fe2+
c
= 0.77 + 0.06 log CFe 3+ .
Fe 2 +
Assuming activity to be ·equal to concentration.

When Fe(OH) 3 appears, ~Fe3+ X C~w = Ks .


. 10-37.l
.. CFe3+=~·
ow
10-18.4
And also when Fe(OH) 2 appears, C
2
Fe+
=-C2- .
ow
Now, at the given pH values (pH increasing as a result of addition of NaOH); assuming
K w = l0- 14 '
464 PROBLEMS ON PHYSICAL CHEMISTRY

CH+ CoH- = Kw cFez+x cOH-


2 Result
.PH .

2.2 10-2.2. 10-ll.8 10-2x10-35.4-= 10-37.4 10-2x10-23.6 = 10-25.6 No ppt.

4.2 lo-4·2 10-9.8 10-31.4 10-21.6 Fe(OH) 3 ppted.

6.0 1~ 10-8 10-26 10-18 Both ppted.

Thus, at pH = 2.2,
E = 0.77 + 0.06 log O.Ol = 0.77 y,
0.01 .
10-37.1 /10-29:4
At pH 4.2, E = 0.77 + 0.06 log = 0.77 + 0.06 log 10-5·7
0:01
= 0.77 - 0.342 = 0.428 v.
At pH = 6, 8.2 and 10.

10-37·1
= 0.77 + 0.06 log --r.;-;- - 0.06 log Cmr
10-10-4

= --0.352 - 0.06 log C 0 H-


.. at pH= 6,
E = - 0.352 - 0.06 log 10-8 = 0.128 V.
At pH= 8.2,
E = - 0.352 - 0.06 logl0-5.8
= - 0.352 + 0.348 = - 0.004 v.
At pH= 10,
E = - 0.352 - 0.06 log 10-4 = - 0.112 V.
Thus, although there is no W or OW in the Fe3+ I Fe 2+ system needed for balancing,
E depends on pH because of precipitation of hydroxides.
22. In l(M) HCI0 4 solution, the formal potentials for the different stages in the
reduction of Mn04 are given in the following emf diagram :
• 0·564V _ 2·26V
Mno4 - Mn04 - Mno 2

Mn++ ~ Mn3+ ..o.9svl

Calculate the formal potential for the reaction

Mn04~ Mn++ and Mn04 ~Mn++ [C.U. 1979]


EMF 465

Soln. : We know, /1G is additive. Thus.


Efonnal (V) 110

Mn0-4 + e = MnO~ 0.564 -O.S64 F(l)


MnO~ + 4H+ + 2e = Mn0 2 + 2Hz0 2.26 -4.S2 F(2)
Mn0 2 + 4H+ + e = Mn 3+ + 2H20 0.95 - 0.9S F(3)
Mn 3+ + e = Mn++ l.S l - l.S l F(4)

Adding, Mn04 + 8H+ + Se = Mn++ + 4H20


!1G= -(0.S64 + 4.S2 + 0.9S + l.SJ)F = -S FE
E = l.S088 V.
Adding (2) to (4),

MnO~ + 8H+ + Se = Mn++ + 4Hz0


!1G = -(4.52 + 0.9S + I.SI) F = -SFE.
.. E = 1.754 V.
23. The emf of Weston standard cell is l.01S30 Vat 20 °C and 1.01807 Vat 25 '<C.
Calculate /1G, !1H and llS for the cell reaction at 2S 0 C. [C.U. 1982)

Soln. : (aE)
dT P
= l.01807- l.OlS30
s
= S.S4 x lQ-4 volt/deg.

llS = nF(dE)
dT p
Here, n =2
.. llS = 2 x 96484 x S.S4 x 10-4 J/K = 106.9 J/K.
!1G = -nFE = -2 x 96484 x 1.01807
= -196.45 kJ.

l1H = nF {r(~~)P -E} = 2 x 96484 (29~.lS x S.S4 x 10-4 - 1.01807)

= -164.58 kJ.
24. The potential of the cell Cd ICdl2 (a2), Agl(s) IAg is 0.2860 Vat 2S 0 C. Calculate
the mean ionic activity and activity of the electrolyte.

Given, E°
1- ,Agl\Ag
= - 0.1522 V and E° ++\
Cd . Cd
= - 0.403
.
V.

Soln. : £ 0

Cell
= - 0.1S22 + 0.403 = 0.2508 V.
The reaction is Cd + 2Agl ~ 2Ag + Cd++ + 2r

Thus at 2S °C, E = E° - o.os 9 log 2


2 aCd++'a1_

or, 0.2860 = 0.2508 - O.OS9 log a 2


2
Prob. Phy. Chem.-30
466 PROBLEMS ON PHYSICAL CHEMISTRY

(0.2860-0.2508)x2
.. Ioga 2 =- _ =-1.1932203.
0 059
.. a2 = 0.0640884 = 0.0641.

Now a 2 = al.
:. a± = 0.400184 = 0.40.
I
25. The emf measurements of the cell H 2 (I atm) HCI (m), AgCl(s) Ag were made I
using two HCI solution of diff. molalities with the following results :
m = 0.0032 0.1238
E (volts) = 0.5205 0.3420
Assuming infinite dilution for the first solution, find E° of the cell and hence calculate
the mean activity coefficient of HCl in the second solution. [C.U. 1973]
Soln. : For the passage of 1 faraday, the reaction taking place in the cell is

lH 2 (g) + AgCI (s) =


Ag(s) + H+ + er
2
.. E = E° - 0.059 log aH+.ac 1- at 25 °C
=£ 0
- 2 x 0.059 log a±.
By the problem, E° =E + 0.118 log m for 1st solution
= 0.5205 + 0.118 log 0.0032
= 0.2261 v.
Thus, for the second solution,
E = 0.2261 - 0.118 log my±
or, 0.3420 = 0.2261 - 0.118 log 0.1238 - 0.118 logy±.
:. y± = 0.841561 "" 0.84.
26. A hydrogen electrode and a calomel half-cell are used to determine the pH of
a solution on a mountain top when the pressure is 500 m of Hg. The pressure was wrongly
assumed to be 760 mm of Hg and the pH was found to be 4; what is the correct pH?
Soln. : The cell is H 2 (P) I solution II KC!, Hg2Cl 2 (s) I Hg
E = £2 - £1

= £2 - (Ef +0.059log
. . PHz
a~~] at 25 °C

0.059
= Ecalomel + 0.059 pH + - - log PH .
2 2

Thus, ~-~:; = 4 by the problem a'sR~ming PH 2 =I atm.

Thus.·· E- Ez = pH + _!_Jog PH
0.059 2 2.

:. 4 = pH + _!_Jog 500
2 760
EMF 467

or, pH =4 - 1 log 500 = 4.09092.


2 760
. . pH == 4.09.
The above is true even at any other temperature.
27. A silver electrode is immersed in 100 cc of 0.1 (M) KC\. What will be the potential
of the electrode at 25 °C after addition of (a) 30 cc, (b) 45 cc, (c) 50 cc, (d) 55 cc and
(e) 70 cc of 0.2 (M) AgN0 3 solution? Given that at 25 °C, E:g+JAg = 0.80 V and the
solubility product of AgCI is 2.3 x 10- 10:
Solo. : Let us consider a cell :
Ag I Ag+ II c1-. AgCI (s) I Ag.
In this cell the reaction is :
AgCI ~ Ag+ + ti-. · ·
RT
Therefore, E = E° -7 In a;..g+·ac 1- .

At equilibrium, E = 0 and aAg+·ac1- = Ks.


Ks = solubility product of AgCI.
.. E° = FRT\n K
s
£0 - £0
£~ell= c1-.AgCilAg Ag+!Ag

.. £
0

Cl-.AgCtlAg
£ =
Ag+!Ag
0

+ 0.059 log 2.3 x 10-IO 0.2313 V. =


• KC\.
Stock solution is 100 cc 0.1 (M) KCl = 10 cc of l (M) KC!. == 50 cc of 0.2(M)
Different amounts of 0.2 (M) AgN0 3 solution are added. After addition of a trace
of 0.2 (M) AgN0 3, the electrode becomes c1-. AgCl IAg (AgN0 3 reacts with KC! giving
AgCI), where the reaction is
AgCI + e ~ Ag + c1-.
1
:. E = E° + 0.059 log -
a
c1- . ·
= 0.2313 - 0.059 log ac1-.
Here, activity coefficient, as it is not given, is considered as unity.
After addition of 50 cc of 0.2 (M) AgN0 3 the electrode becomes Ag+ IAg, where
the reaction is Ag+ + e = Ag.
Here, E = 0.80 + 0.059 log CAg+
cc of 0.2(M_) Total vol. Cc1- or CAg+ E in volt
AgN0 3 added

30 cc 130 cc 0.32 volt

45 cc 145 cc 0.36 volt


468 PROBLEMS ON PHYSICAL CHEMISTRY

50 cc 150 cc Cc1- ~ ~2.3x 10- 10 0.5 volt


== 1.516 x' 10-s (M)

55 cc· 155 cc _l_(M) 0.67 volt


155
70 cc 170 cc l~O (M) 0.7Q4 volt

28. Calculate the potential difference between a standard hydrogen electrode and half-
cells comprising platinum foil coated with platinum black in contact with ~2 at a pressure
of I atm into the following solution at 25 °C.
(i) 10- 3 (M) Ba(OH) 2
(ii) 0.1 (M) acetic acid
(iii) a mixture of 20 cc of 0.25 (M) HOAc and 80 c of 0.1 (M) NaOH.
. Solo. : Proceed as Problem 27.

0
==-(£OH-!Hz -0.059logaOH- )

K
== - 0.059 log Kw + 0.059 log-w
al'i+

== - 0.059 log aH+ == 0.059 pH volt.

(ii) E ==E0 - E
H+IH2 H+!Hz

== - 0.059 log aH+(O. IM CH3COOH)

== - 0.059 log~ K0 CcH3cooH volt.

(iii) E == E° - E
H+!Hz OW/Hz

== - 0.059 log aH+ == 0.059 pH volt (pOH == 1.52)


[Ans. (i) 0.67 volt, (ii) 0.17 volt, (iii) 0. 74 VJ
29. The emf of the cell Cd ICd(N03 )z (0.05M), KCN(2M) II Cd(N03)z(O. I 9M)I Cd
is found to be 0.578 volt.
Calculate the instability constant of the complex Cd(CN) 4 •
Solo. : It is a concentration cell without transport for the passage of 2 faradays.
Cd++ from 0.19 (M) passes to Cd++"(0.05) molar which is taken as x, because it has
taken part in complex formation Cd(CN)~ .

C ++(0.19M)
.. E == ~
00 log Cd .
2 x

.. log 0.19 == 0.578x2


x 0.059
EMF 469

or, log 0.19 = 19.5932.


x
0.19 = 3.9194 x 1019
x
. . x = 4.847 x 10-21 ~

The instability constant of Cd(CN)~ i.s the equilibrium constant K of the reaction
Cd(CN)~ ~ Cd++ + 4CN-.
Since, x is very small, almost all of Cd++ has been consumed.

Thus, C remaining = x and C _ formed = (0.05 - x)


Cd++ Cd(CN)4

.. C
cw
=2 - [4 x {(0.05) - x}] = 1.8 + 4x.

cCd++. cCN-
4
.. K = CCd(CN)4-

:. K = x(l.8+4x) 4 .
(0.05-x)
Neglecting x in comparison to 0.05 and 1.8.

:. K = 4·847x10-21 x(l·8)4 = 1.0176 x 10-18.


0·05
30. The solubility of iodine in water at 25 °C is l.33 x 10-3 moles/litre. What is the
cone. of 1- in a saturated solution of 12 in water at 45 °C, when
C 1- = 0.500 (M)
Given, Eo = 0.5355 V
1-112

£7- 3 = 0.5365
11
_ V at 25 °C.

Soln. : In a saturated solution of 12 in 1-, the reaction is 12 + 1- ~ 1


3
0.5 x
The reaction takes place in the cell
13 I r I li{s)
Reaction at R.H.E., 12 (s) + 2e = 2r
at L.H.E., 31- = 13 + 2e
12 (s) + r = 13
a1- a
3 __5_~ = l]
.. K = a12 x al_ = [ ·: a 1 (s)
a,-0.5 2

(I)
470 PROBLEMS ON PHYSICAL CHEMISTRY

At equilibrium, £cell = 0.
:. 0 = Eo - RT1n K
2F

or, 0 = E° _ 0.05916 10 g K (2)


2

or, £ 0 =
0.059161 0 g K.
2
:. log K = - 0.033806.
:. K = 0.92510996.
:. x = 0.5 x K = 0.4626 mole.
If we choose a different cell,
13 I 1- I 12 (aq. solution),
then we have to know £ of 1- I 12 (aq.). The reaction in the cell
0

12 (aq) + 2e = 21-
3r =~ + 2e
At equilibrium, E 0
r-lr2(aq)
- E°
1-1r3
= 0.059161og .'\.
2
12 (aq) + 1- = 13
0.5 x
Let us consider the ceil
12 (aq. solution) I1-I 12 (s).
The reaction is

I 1 (aq.) + e
2 2

112 (s) = J_ 12 (aq.)


2 2
1/2
AG = AG 0
+ RT In ar2(aq)
al2(s)

E = E° _ E° _ 0.05916 log at2 (aq).


rltz(s) rl12(aq)
2
At equilibrium,
0 05916
o = E°rl12(s) - E°rlt2(aq) - ·
2
log 0.00133.

:. E°,- I!z (aq ) = 0.5355 - 0.0~916 log 0.00133

= 0.62057 volt = 0.6206 volt.


Putting the value of eq. (I),
EMF 471

log
K =. (0.6206-_0.5365)x2 = 0.084x2
= 2.84313.
0.05916 0.05916
K = 696.84 == 697.

al_
Now, K= -~3-
a1_a12(aq)
= O·OOt:f3x0·5 = 696 ·84 ·
x = 0.5 x 696.84 x 0.00133 == 0.463 (M).
If it is assumed that solid 12 is removed from saturated solution,

696.85 = x
(0.00133-x)x0.5

x = 696.696.85x0.5+1
85 x 0.00133x 0.5 0.463
= '349.425 = 0.001325 (M).

31. 25 ml of 0.2 (M) CuS0 4 is mixed with 25 ml of 1.0 (M) KI. Calculate the

concentration of Cu++ ion in the resulting solution. (Given, E~u++lcu+ = 0.17 V,

E'j_ r = 0.53 V) and solublity product Ks of Cul


121
=5 x 10-12_ [C.U. 1986]
2

Soln. : The reaction is Cu++ + 21- = Cul + l 1. ( 1)


' 2.2
I I
This reaction takes place in the cell 12 1-, Cu++ Cul (s) with electrode reaction
at R.H.E., Cu++ + I:-- + e = Cul

at L.H.E., I - = 112 + e.
2
Now, E _ ++i
I ,Cu Cul
= £ 1-.cu++ Cul + 0.059 log aCu ++.a_at
0

1I
25 °C. (2)

Again, considering the R.H.E as Cu++ ICu+ electrode (Cul is sparingly soluble).

a a
E
0

1-.eu++icu1 -- £ cu++icu+ + 0 ·059 log -cu++


- = 0.17 + 0.059 log cu++ .
a cu+I K_,. I al_
By the problem,
EI_,Cu ++I Cul = (0.17 - 0.059 log Ks) + 0.059 log acu++ .a1_ . (3)
Equating eqn. (2) and eqn. (3),
0

£ = 0.17 - 0.059 log 5 x 10-12 = 0.8367 volt.


1-.cu1lcu++
Thus, E:eu= 0.8367 - 0.53 = 0.3067.
But, /J.G 0 = -FE° = -RT In K so that
0 3067
log K = · at 25°C = 5.199.
0.059
:. K = 1.58 x 10s.
Since, K for reaction (I) is so high, it may be assumed that Cu++, the ion with lower
concentration, is completely consumed. The concentration of 1- will be around
(0.5 - 2 x 0.1) = 0.3 (M).
472 PROBLEMS ON PHYSICAL CHEMISTRY

If x be the concentration of Cu++ remaining, then by equation ( l ), the concentration


of 1- remaining at equilibrium is 0.3 + 2x.
1/2
.·. K = llcu1xa,22 = ---....,...-
2 [a "" C] .
acu++ xa,_ ccu++'c,_

l
Thus, l.58 x 10 5 = 2.
x(0.3+2x)
Neglecting 2x in comparison to 0.3,

1
x = =
7.03 x 10-s (M).
l.58xl0 5 x{0.3)2
32. The solublity of iodine in water at 25 °C is 1.33 x I0- 3 "tnoles !litre. Calculate
the equilibrium constant of the reaction 1- + 12 = 13 if e<;- n- = 0.534 V
3

and £ 0
,- /12
= 0.535 V.

Solo. : The reaction is really 12 (aq) + 1- = 13.


So, we have to determine E° of 1- I 12 (aq.).
Consider the cell 12 (aq.) 1- 12 (s). I I
The reaction is _!_1 2 (s) + e ~ 1-
2
1- ~ _!_1 2 (aq.) + e
2
l1 2 (s) = _!_1 2 (aq.)
2 2
:. E = E° _ 0.05916 log a,2(aq.) .
2 a,2ls)
At equilibrium,

0 = E~_llz(s) - e<;-llz(aq) - 0.05916 log 0.00133.


2
.. E°
rl~~q)
= 0.535 - 0.05916 log 0.00133 = 0.62 volt.
2
The reaction takes place in the cell 13Ir11 2 ( aq.)
E° = (0.62 - 0.534) volt = 0.086 volt.
.. RT In K = nFE° = 2 x 96484 x 0.086 .

.. In K = 2x96484x0.086.
8.314 x 298.15
. . In K = 6.69482,
.. K = 808.2.
EMF 473

33. Given that, 12 (aq.) + 2e = 21-.


E° = 0.6197 V and 13 + 2e = 31-, £ 0 .= 0.5355 V.

Calculate K for the reaction 12 (aq) + 1- = 13.


Solo. : Proceed as Problem no. 32. [An~. K = 708]
34. Calculate the· emf of the cell :
I
Pt, Pb(Hg) PbSOis),CuS0 4 (a 1); CuS0 4 (a2), PbSOis), I Pb(Hg), Pt
where the molarity of the solution
a 1 = 0.2 (M), y± = 0.1 l and a 2 = 0.02 (M), y± = 0.32.
(t+ )cu2+ · = 0.37, which is the negative electrode.
Solo. : R.H.E. : The reaction is-
PbSO 4 + 2e = Pb + so~- (a 2)
Net change for the passage of one faraday :

Gain of so!- : i gm-mole


' t
Loss of So!- : ..::;.. gm-mole.
2

Net gain of so~- (..!..- 1- ) = !.±. gm-mole.


2 2 2

R.H.E. : Net gain of Cu 2+ = !.±.2 gm-mole of CuS0 4


(a2 )

L.H.E. : The reaction is Pb + so~- = PbS04 + 2e

Loss of so~- : l gm-mole


2
Gain of so~- : ~ gm-mole
2

Net loss of so!- : !.±. gm-mole


2
t
Net loss of Cu 2+ : ...±.. gm-mole of CuSOia 1).
2

So for the passage of one faraday, 1+ gm-mole of CuS04 passes from a 1 to a 2.


2
:. f)..G = f)..Gcu2+ + f)..G so2 -
4
2 2

= f!.±.RTdlna
2 ~
z+ + f!.±.RTdlna 2
2 w~
I I

= 1+ RT In
2
474 PROBLEMS ON PHYSICAL CHEMISTRY

= t+ RT ln !!1. + !±_RT In a2
2 a1 2 a1

= t+ RT ln a2.

= t+ RT ln C2Y± = t+ RT ln Cz x0.32
C1Y± C1 x0.ll

= t+ RT Jn 0.02 x 0.32
0.2x0.I I

E = _ liG = _ RT t ln 0. 02 x 0. 32
nF nF + 0. 2 x 0.11
= 0.0117 V (where n = I, t+ = 0.37)
= 0.012 v.
Since the emf is positive, the L.H.E. is negative electrode.

35. liG 0 for the reaction Hz (g, 1 atm) + (g, 1 atm) = HzO (I) is -237.2 kJ
l Oz
2
at 298 K. Calculate the E 0 _ ( )' The cell potential producing the reaction,
OH 1Oz g

HzO (I) = H+ (aH+ = I) + OH- (a


ow
= I), is - 0.828 V. (i)

Solo. : The reaction Hz + t0z = HzO (I) takes place in the cell.

Pt, Hz (g, I atm) IOW I Oz (g, 1 atm), Pt. Let the standard potential be E°.
0
Then E is :

E° = 237200 = 1.2292193 v.
2x 96484
E° of the cell is :

.. 1.229 (V) =E~wlo2 - ~WIH2. (ii)

The cell producing the reaction, Hp (I) = H+ + OH-. is H2 I tt+ II cm-1 H2.
But the standard cell potential of the cell Hz IH+ II oH-1 Hz is = - 0.828 V.

Putting in (ii), 1.229 = ~wloz - (-0.828 V) = ~wloz + 0.828 V.

:. E° I
OW02
= 1.229 - 0.828 = 0.401 V.
EMF 475

36. Evaluate the emf of the cell at 298 K neglecting the LJP
Pt, H 2 (g, 1 atm) ICH 3COOH(lM) II HCOOH(0.5M) I ·0 2 (g, 1 atm), Pt

Given, E° = 0.401 V
owlo 2
Kw = 1.0?8 x 10- 14

( Ka )CH3COOH = 1.85 x 10-5


5
( K a ) HCOOH = 17. 7 x 10- . ,
Soln. : The emf of the cell E = E2 - E1, where E2 is the potential of R.H.E. and
E 1 is the potential of L.H.E. as R.H.E.

Reaction at OH_ I Oz electrode.


1
2 oz + HzO + 2e = 20W
E = E° - RT In a 2 = E° I - 0.059 log Kw
owlo2 owlo 2 nF ow ow o 2 aH+

Hence, E° +i = E° _ - 0.059 Jog Kw (since, a0 w = Kw )


. H Oz OH 102 aH+

As H+ I Oz electrode reaction at R.H.E.,


1
2 oz + 2H+ + 2e = HzO
Ez = E°H+lo2 + -
0.059 log a 2 +at
- H
25 °C and 1 atm
2
= E~+
102 + 0.059 log aH+

= ~wloz - 0.059 log Kw + 0.059 Jog aH+.

Since "(± is not known, the concentration in molarity is used for activity.
:. £ 2 = 0.401 - 0.059 log Kw + 0.059 log (CH+)
= 0.40 I - (- 0.82579) + 0.059 log CH+

= 1.22679 + 0.059 log ~CHCCX>H ( K0 )HCOOH

CHCOOH (K0 )HCOOH = 0.5(M) X 17.7 X 10-S = 8.85 X w=-s(M)

= 1.22679 + 0.059 log ~8.85X10-S


= 1.22679 - 0.119565 = l.107 (V).
For L.H.E. the reaction is H+ (CH3COOH) + e = lH 2
(1 atm).
2
E1 =.E° + 0.059 Jog aH+
= 0 + 0.059 log c H+ .
476 PROBLEMS ON PHYSICAL CHEMISTRY

For l mole CH3COOH,


1
'Vfc CH3COOH (Ka )CH3COOH =
5
CH+= 'Y 1x1.85x10-
E1 = 0.059 log c = --0.1396.
H+
.. E = E2 - E 1 = 1.107 + 0.1396 = 1.25 V.
37. During discharge of a lead storage battery the density of H2S04 falls from
1.295 gm/cc to 1.140 gm/cc'. H 2S0 4 of density 1.295 gm.cc- 1 is 39.2% H 2S04 by weight
and H2S04 of density 1.140 gm/cc is 20.8% H 2S0 4 by weight. The battery holds 3.6 litres
of acid and the volume remains practically constant during discharge. Calculate the number
of ampere-hour for which the bat~ery must be used.
Solo. : The net reaction of discharge for the passage of one faraday 1s
Pb(s) + Pb0 2(s) + 2H 2S04 = 2PbS04 (s) + 2Hz0 (I).
As per reaction the equivalent weigh.t = 98 (molecular weight).
Number of equivalent of H2S04 present initially,
39 2
3600 ml x 1.295 gm/cc x · xJ_ = 18.648 equivalent.
100 98
Number of equivalent present after discharging,
20 8
3600 x 1.14 x · x __!_ = 8.7105.
100 98
Number of equivalent loss = 9.9375 eq .
.Quantity of charges = 9.9375 x 96484 = 958809.75 coulombs
= 958809.75 ampere-sec.
So the required ampere-hour for which battery must be used
958809.75
= = 266.34 ampere-hour.
3600
38. A current of 4.2 micro-ampere is passed through the solution of AgN0 3 for 30
minutes using Pt electrode. A uniforn single larg~ Ag atom deposited carries 45% of cathode
surface. What is the total surface area of cathode? It is known that one Ag atom covers
5.404 x IQ-16 cm2.
Solo. : Total charges pass = 4.2 x IC)-(; x 30 x 60 = 7.56 x 10-3 C = 0.0756 C.
108 756
"' · ht of A g d epos1t
ne1g · ed at ca th ode = x0.0 -- 8.4629 x 10-5 .
96484
8.46x10-5
Total number of Ag atoms = · x 6.022 x 1023 = 4.718 x 10n
108
So surface area at cathode covers, 4.718 x 1011 x 5.404 x IQ-16
= 254.9607 cmz
254.982x 100
. . total surface of the cathode = = 566.58 cm 2•
45
EMF 477

39. For the react.ion :


Hi (g) + 0 2 (g) - - 7 HzO (I) at 25 °C
0
b.G = -237.2 kJ and Aff0 = -285.9 kJ.
(i) Devise a cell in which above reaction would take place.
(ii) What is £ 0 of the cell?
(iii) At what rate does E° change with T around 25 °C?
[M.Sc. Admission Test, 1.1.T. Kharagpur 1996]
Soln. : The cell is : Pt, H2 (g, l atm) I NaOH I 0 2 (g, l atm), Pt
b.G 0 = -237.2 kJ = -237200 J.
, , AffO : -285.9 kJ : -285900 J,
0
E° = _ llG = - 237200. = 1.2292 V = 1.23 V.
nF 2x96484

M1°= nF H~~), -E"}


-285900 = 2 x %484 {w8.1s(~), -1.229}
0
0
2 x 96484 x 298.15 ( £ ) = -285900 + 1.229 x 2 x 96484
oT P

( a£oT
)
P
= -8.472 x 10-4 •

40. Devise a cell where the net reaction is Zn + cu2+ ~ zn2+ + Cu.
2
Standard reduction potential (E°) values for Zn + Zn and Cu 2+ Cu electrodes are I I
- 0.76 V and 0.34 V respectively. Calculate the equilibrium constant of the reaction.
[V.U. 2005]
Soln. : The cell is Zn I Zn 2+ I I Cu 2+ ICu.
For reaction balancing two electrons are needed, that is n = 2.
:. -2 xF x E° = -RT In K.
E° = {0.34 - (-0.76)} volt= l.10 volt.
Then proceed as earlier.
Equilibrium constant K = 1.546 x 1037.
41. Calculate the free energy change and the equilibrium constant of the reaction that
takes place in the following cell at 27 °C :
Cu I Cu+~ (a = 0.01) 11 Fe2+ (a = 0.001), Fe3+ (a = 0.01) I Pt

Given, E° = 0.34 volt, £


0
= 0.77 volt. [V.U. 2004]
Cu 2 +1cu Fe 3+iFe 2 +

Soln. : Proceed as earlier. For the cell, £ 0 = 0.43 volt. So, K = 2.7589 x 1014 and
b.G = -105.94 kJ,
478 PROBLEMS ON PHYSICAL CHEMISTRY

42. Write cell reactions for the following electrochemical cell :


Fe, Fe2+ (0.1 M) 11 Cd 2+ (0.00 I M), Cd
assuming that activities equal concentration. Also (i) calculate the emf of the cell and
determine the direction of the spontaneous reaction, and (ii) calcu1ate the equilibrium
constant of the cell reaction. Given, E°
. Cd2+1cd
= -0.40 V and E° 2+
Fe 1Fe
= - 0.44V.
[V.U. 2003]
Soln. : The reaction for the given cell,
·Cd+2 + 2e = Cd
Fe = Fe+2 + 2e
Cd+ 2 + Fe = Cd + Fe+2
-nFE' = -RT in K
2 x96484x0.04
.. In K = 8.314 x 298.15
(since, n = 2)
K = 22.51.

A t 25 oc , E = 0 .04 V - - - o gaaFe2
0.0591
2
--
+
Cd 2

0.0591 O.I
= 0 .04 - -2- ogQOOI

= 0.04 - 0.0295 log 100


= 0.04 - 0.059 = - 0.019 volt.
Hence, for the given cell, L.H.E. is not the negative electrode. Current is passing from
R.H.E. to L.H.E., i.e., R.H.E. is negative electrode. and should be written as L.H.E.
43. The measure emfs are 0.06839 V at 25 °C and 0.07048 V at 30 °C for the following
cell Ag, AgBr (s) I K.Br solution I Hg 2Br2(s) I Hg. Calculate, after stating the reaction
involved the values of free energy change and entropy change of the process at 25 °C.
[M.Sc. Admission Test 1999, 1.1.T. Khargapur]
Soln. : By the problem,

aE) = E30°c - E2s c 0

( aT p 5
= 4.18 x J0--4 volt/K.
The reaction lakes place in the cell,
Hg 2Br 2 + 2Ag = 2Hg + 2AgBr
.1.G 0
= -nFE' = -13.2 kJ,

Af0 = 2x F x (aE) = 80.66 J.K-1.


aT p
EMF 479

44. The emf of a galvanic cell :


H2 I aq. solution (pH = x) II IN calomel is 0.5164 V at 25 °C, when the barometric
pressure is 754.1 mm of Hg. Given that electrode potential of the calomel electrode is
0.28 volt and the vapour pressure of water is 23.8 mm Hg, calculate the value of x, i.e.,
pH of the aqueous solution. [M.Sc. Admission Test 1993, 1.1.T. Bombay]

Solo. : For the L.H.E. the reaction as R.H.E. is H+ + e = ..!.H


2 2
.

RT a+
E = E° +7ln ~(;tt2)

.. E = 0.059 log aH+ - +0.059 log PH ,


2
since E° = 0,

l
= - 0.059 pH - 20.059 log PH .
2
By the problem,
096
PH
2
= 754.1-23.8
760
atm. = . atm.

E = Ecal - EH+ttt2·

.. 0.5164 = 0.28 + 0.059 pH + +0.059 log 0.96.

.. pH-;= 4.015 = 4.02.


45. Write the cell reaction and calculate the emf of the cell
Cu!Cu 2 + (4M) II Ag+ (0.1 M) I Ag at 25 °C from the following data :

· = 0.80 V. [M.Sc. Ad~ission Test 1993, 1.1.T. Bombay]


E°Cu 2+ 1Cu = 0.34 V, E°Ag +i Ag

Solo. : Proceed as earlier.


Ecell= 0.38 volt
[Use concentration for activities.]
46. Calculate the solubility product of PbC1 2 at 25 °C.
E° for PbC1 2(s) I Pb(s) = - 0.268 V and E° for Pb 2+ I Pb (s) = - 0.126 V.
[M.Sc. Admission Test 1994, 1.1.T. Bombay]
Solo. : We know that for the reaction PbC1 2 = Pb+2 + 2Cl-, the cell is
Pt, Pb (Hg) I Pb 2+ II c1-, PbC1 2(s) I Pb(Hg), Pt.
For the given cell,

E° =; ~l-.PbCl 2 1Pb - e;b2+IPb = -0.268 - (-0.126) V = -0·142 V.

. .. -0.142 = -2-
0.059 1o~ K s

. . Jog Ks = -O.l42x2
0.059
.. log K.s = - 4.81~, .. Ks = 1.53 x 10-5 .
480 PROBLEMS ON PHYSICAL CHEMISTRY

47. The potential (£) of the c~ll Pt I Hz (g, I atm) I NaOH (aq. 0. mot/kg), NaCl r
(aq. 0.01125 mol/kg I AgCI (s) I Ag at 25 °C was found to be 1.04864 V. Calculate pH
and the standard free energy change of autoprotolysis of water at 25 °C. Assume activity
coefficients to be equal to unity. Given, E~gCllAg = 0.2223 V.
[M.Sc. Admission Test 1996, 1.1.T. Bombay]
Solo. : The cell given is :
Pt, H2(g, l atm) I NaOH (0.1 molal), NaCl (0.01125 molal), AgCI I Ag.
Ecell = ER.H.E. - EL.H.E.

= (E°c1-.Agc1 Ag _RTlna
1 F ci- )-(E°+
H tt 2
+0.0591ogaH +)
1

= (0.2223 - 0.059 log 0.01125) - (0 - 0.059 pH).


·· Ecell = 0.3373 + 0.059 pH
.. 1.04864 = 0.3373 + 0.059 pH
.. pH = 12.057.
= 8.77 x 10- 13 , :.
Kw = CH+.Coir = 8.77 x 10- 14 .
=-RT In Kw= -8.314 x 298.15 In (8.77 x 10-1 4)
= 74525 joule = 74.52 kj.

48. The standard potentials of cu2+ I Cu and Cu+ I Cu electrodes are 0.34 volt and
0.52 volt respectively. Calculate the standard potential for Cu2+ electrode.
[M.Sc. Admission Test 1991, 1.1.T. Kharagpur]
Soln. : Given, E° t:i,.Go
(i) Cu 2+ + 2e = Cu 0.34 v -2F x 0.34
(ii) Cu+ + e = Cu 0.52 v -F x 0.52
(iii) Cu 2+ + e = Cu+ E° -F~ell
Reaction (iii) is obtained by subtracting (2) from (I).
[The cell is Cu I cu+ II cu+ 2 I cu+]
.. t:i,.G~ = t:i,.Gf - t:i,.G2
-FE° = -0.16 F
.. ~ = + 0.16 volt.
49. The equilibrium constant for the reaction 2Cu+ =
Cu 2+ + Cu(s) at 25 °C is
6
1.646 x 10 . (i) Construct a cell in which the reaction could occur, (ii) Calculate the standard
emf for the cell at 25 °C. (iii) Evaluate the standard oxidation potential of the Cu I cu+
electrode at 25 °c l The standard potential of cu+ I cu+ 2 electrode = - 0.153 v at 25 °c1
[C.U. 2004]
Solo. : The reaction, 2Cu+ = Cu2+ + Cu takes place in the cell.
Pt, cu+ I cu2+ II cu+ I Cu.
0
!:i,.G for the cell is -2 x F x E° = -RT In K.

.. E° RT In ( 1.646 x I06 ) = O. 0 59 log K = 0.366 V.


F I
EMF 481

0.366 = E°Cu +1Cu -(-0.153)


.. ~u+lcu = (0.366 - 0.153)V = 0.213 V.

Therefore, standard oxidation potential of Cu I cu+ = - 0.213 v.


Oxidation potential is negative of reduction potential.

50. Pt, H (g, 1 atm) IHA2 (PKa = 6) II HA1 (PKa = 4) IH2 (g, I atm )I Pt
z C 1at25°C C 1 at25°C

Calculate the emf of the above cell at 25 °C. What are the assumptions which must
be made in order to solve the problem. [C.U. 2005)
Soln. : The reaction taking place in the cell for the passage of IF :
At the R.H.E., ff+ (pKO) = 4) + e = Hz.
At the L.H.E., Hz = ff+ (pK0 z = 6) + e
Adding, H+ (pK0 = 4) ~ H+ (pK0 = 6)
)· z
llG = RT In aH+(PKa2 =6 )
aH+ (PKai = 4)
.. E =In ·aH+(PKai = 4 )
aH+(pK02 =6)
Assuming that Y± = 1, att+ = CH+,
CH+(PKai =4)
E = -RT In. ---"'"--'----...;..
F CH+(pK02 =6)
At 25 °C,

E = 0.059 log
C +(PK =
H OJ
4)
CH+(PKai =6)

. = 0.059 logCH+(PKai =4) - 0.059 logCH+(pK02 =6)


= -0.059 pH (HA1) + 0.059 pH (HA2).
Any weak add HA dissociates as
HA~ W +A-
At equilibrium, HA
C{l-a)
If the initial concentration is C and ex is the degree of dissociation, then
482 PROBLEMS ON PHYSICAL CHEMISTRY

Neglecting a in comparison to 1,
Ka = a 2C .
.. CH+= aC = ~C.Ka .

. . pH ~ .!_pKa - .!_log C.
2 2
Neglecting a in comparison to I,
Ka = a 2C .
. . CH+= aC = ~C.K0 •

. . pH = .!_pKa - .!_log C.
2 2
. . by the problem,

E = -0.059Gx4-±togC1) + 0.059.Gx6-±togC1) = .059(3 - 2) => 0.059V.

The assumptions made up :


( I) There is no LJP.
(2) Degree of dissociation of weak acid is negligible in comparison to I.
(3) The addition or removal of a finite no. of gm mole does not alter the concentration.
(4) Activity coefficient is unity.
51. Calculate the emf, the free en~rgy change and the equilibrium constant of the
reaction involved for the following cell at 25 °C.
Cu I cu+2 {a = 0.01) II Fe+ 2 (a = 0.001), Fe+ 3 (a = 0.01) I Pt.
0
Given, E 0 ++ = 0.34 V, E +3 +z = 0.77V. [V.U. 2007]
Cu ,Cu Fe ,Fe
Solo. : Same as Problem 41.
[Ans. E = 0.548 V, L1G = -105.75 kJ, K = 3.45 x 10 14]
52. For the following standard cell, Cd(Hg) I CdS0 4: Hp(s) Hg2S04 (s) l'Hg, the
emf is given as a function of temperature t in °C by the following expression:
E = 1.01845 - (4.05 x 10-5 ) (t - 25) - (9.5 x 10-7 ) (t - 20) 2 .
Write down the cell reaction at each electrode. Calculate L1G, Aff and !lS for cell
reaction at 25 °C.
Solo. : Reaction at RH.E. : Hg2SOis) + 2e = Hg + S042 .
Reaction at L.H.E. : Cd = Cd+ 2 + 2e.
Hence, reaction in the cell is Hg2S0 4 (s) + Cd = Hg .+ Cd+ 2 + S042 .

oT£) p = -5
0
Proceed as Problem 6, ( x 10-sv.K- 1 at 25 °C.

E = 1.01843 V at 25 °C
[Ans. L1G = -196.52 kJ, Aff = -199.40 kJ, !lS = -9.65 J.K- 1]
EMF 483

53. Calculate the emf of the following cell at 298 K :


Hz(Pt) 1 atm I HCI (a = 0.1) I Hz(Pt) 0.1 atm. [B.U. 2004]

l
Solo. : H+ (a = 0.1) + e = -Hz (0.1 atm)
2
I
-Hz (1 atm)
2
= H+ (a= 0.1) + e

l
Adding, ..!_Hz (I atm) = -Hz (0.1 atm)
2 2
This is a concentration cell.

RT P~;(o.1 atm)
Thus, E = - - In l/Z ( ) (since, E~+ /Hz = 0)
F PH 2 l atm
= 0.0295 v.
54. The emf of the following cell is 1.71 volts at 25 °C, Zn I zn++(a = 0.0 l) 11 Fe++
(a = 0.01), Fe+ 3 (a = 0.01) I J;>t.
Calculate the equilibrium constant of the reaction :
Zn + 2Fe+ 3 = zn+-+ · + 2Fe+z at 25 °C. [B.U. 1978]
Solo. : The reaction for the cell :
Zn I zn++ (a = 0.01) II
Fe++ (a = 0.001), Fe+ 3 (a = 0.01) I Pt is
Zn + 2Fe+ 3 = zn++ + 2Fe+Z.

Hence, E = E° -
RT
2F
In
azn++(aFe+2 r (n = 2).
{aFe+3) azn
2

or,
1. 7 ! = E° _ 0.059 l
2 og
O.Olx(0.001)
(0.01)2
(
azn =
l)
.

E° = 1.592 v.
Proceed as Problem 14. [Ans. K = 6.652 x JO~i3]
55. A silver electrode is dipped into a 0.1 molal KCl solution saturated with AgCJ.
The emf of the cell formed by combining this electrode with 0.1 (N) calomel electrode
is found to be 0.0494 volt at 25 °C. Calculate the solubility product of AgCl at 25 °C.
(Potential of 0.1 (N) calomel electrode for oxidation is - 0.3338 volt, the standard oxidation
potential of the silver electrode is - 0.799 volt at 25 °C. The activity of chloride ion in
the solution is 0.077) [B.U. 1979)
Solo. : For the cell Ag I AgCl, KCl II O.l(N) calomel,
E = 0.0494 volt at 25 °C.
Ered for 0.1 (N) calomel electrode =. 0.3338.

E~g+[Ag = 0.799.
Now, Ecell = Ecalomel (1)
AgCI = Ag+ + c1- (2)
Ag+ + e =Ag (3)
484 PROBLEMS ON PHYSICAL CHEMISTRY

Adding ( l) and (2), AgCl + e = Ag + c1-


E°Cl-,AgCllAg = E0
Cl-,AgCllAg
- 0.059 log a
Cl-

= ~g+IAg + 0.059 log aAg+ (disregarding eqn. 2)

= £ Ag+IAg
0
+ 0.059 log~
a
(Ks = solubility product of AgCl).
c1-

Hence, E~l-.AgCllAg = ~g+IAg + 0.059 log Ks.


Thus, from equation ( l ),
0.0494 = 0.3338 0
- ( £ Ag +i
Ag
+ 0.059 log Ks - 0.059 log ac 1-)

= 0.3338 - 0.799 - 0.059 log Ks + 0.059 log 0.077.


Hence, Ks = 1.46 x 10-10,

56. A solution of Fe+ 2 is titrated potentiometrically using ce+4 solution. Calculate


the emf of the redox electrode thus formed when (i) 50% of Fe+ 2 is titrated, (ii) 90%
of Fe+ 2 is titrated and (iii) 110% titration is done.
Given, Fe+2 ~ Fe+ 3+ e E" = - 0.77 volts
and Fe+ 2 + Ce+4 ~ Fe+ 3 + ce+ 3 K = 1014 [B.U. 1986]

Solo. : Given, E°Fe +JI,Fe +z = 0.77 volts.

For '50% and 90% titration the electrode is Fe+ 31Fe+ 2.

RT I aFe+2
(i) At 50% titration E = ~e+JIFe+z - F n --
aFe+3

= 0.77 v.
(ii) At 90% titration, E = 0.77 - 0.059 log = 0.82 V.

(iii) At !00% titration, equilibrium is attained.


At equilibrium, E = 0 = E" - 0.059 log K.

or, ~e+4/ce+3 = ~e+31ce+2 +0.059 log K (the cell is Fe+2 1 Fe+3 II Ce+4 I ce+3)
= 1.596 volt.
At 110% titration,

100
= 1.596 - 0.059 log
10
= 1.537 volt:
EMF 485

57. Two hydrogen electrodes at one atmosphere pressure each are inserted into 0.01 (N)
NaOH and 0.01 (N) NH40H solution respectively. They are connected internally by a salt
bridge. If the teperature be 25 °C, what will be the emf of the cell? [Given, dissociation constant
of NH 40H = 1.85 x 1o-5 and R =8.31 volt.coulomb/deg). Which electrode does act as positive
terminal of the cell? [B.U. 1989]
Solo. : Let the cell be :
Hz (l atm) I NaOH (0.0IN) II Nl1 40H (0.0IN) I Hz (I atm)
Reaction occuring in the cell,
oH-(0.01 N NaOH) = oH- (0.01 N NH40H).

a
RT OW(O.OIN NaOH)
EceII= -In
F

= 0.059 log ~ O.OJ = 0.081 V.


1.85x10-5 xO. OJ

Hence, NH40H I0Hz acts as R.H.E. and the other as L.H.E.


58. A saturated calomel electrode is coupled through a salt bridge with a quinhydrone
electrode dipping in 0.1 M NH4Cl. The observed emf at 25 °C is 0.152 volts. Find the
dissociation constant of NH40H. Oxidation potential of saturated calomel = - 0.242 V and
standard oxidation potential of quinhydrone electrode. is = - 0.699 volt at 2? °C.
[C.U. 1977)
Hints : For the cell.
Pt, Hg I HgzClz(s), KCl II solution Q I QHz. Pt.
Ecell = 0.699 - 0.059 pH - 0.242.
Since NH4Cl is a salt of strong acid (HCI) and weak base (NH4 0H),
pH = 7 - pKb - logC [Ans... : Kb = 2.19 x 10-51

59. Calculate the junction potential at 25 °C of HCl (0.1 N) : HCl (0.0 I N), the
transference number of the cation being 0.83. Assume concentration to be identical with
activities. [C.U. 1978]

. 'd .
So In. : L 1qu1 . . 1 E
Junction potentia = I.JP = (t_ - RT (a±)2
t+)- ln -(-)- .
F a± 1

t+ = 0.83, t_ =I - 0.83 = 0.17


Eup = (0.17 - 0.83) x 0.059 Jog O. OJ = 0.039 volt.
0.1
486 PROBLEMS ON PHYSICAL CHEMISTRY

60. For the following cell, write down the cell reaction, and calculate emf; free energy
change; and equilibrium constant for the cell reaction.
Hz (g, 1 atm) I HzS04 (m = 0.05, Y± = 0.340), HgzSOis) j Hg.
0
Give.n, £ Z(OX) =-0.6141 V. [C.U. 1985)
Hg+IHgzS04(s).so4

Solo. : Reaction at R.H.E., HgzS0 4 (s) + 2e = 2Hg + S042


Reaction at R.H.E., Hz(g, 1 atm) = 2H+ + 2e

Cell reaction, Hg 2S04 (s) + Hz (I atm) = 2Hg + 2H+ + S042

= Eocell - RT log (aH+)Z


2F
(aso42)
=.cso4-.HgzS0
r;-0
, 0 - DO
c,
+ - o. 059 Iog (aH+ )z (
aso-2 / 'j
4(s)IHg H IHz 2 4
0.059 z I 3 3
=0.6141- log2.l.C.y±(E°H+ltt 2 =0!
2
0 059
= 0.6141 - ·
2
log (4 x 0.05 3 x 0.343) = 0.7529 V.

!l.G = - nFE = -2 x 96484 x 0.7529 = -145.29 k.J.


nF
In K = -E°
RT
2
log K =- - x 0.6141.
0.059
:: K = 6.56 x 1o2°.
.61. The thermodynamic dissociation constant of a weak acid (HA) is 1.0 x 10-5 at
25 °C. Find the standard potential (E°) for the half-cell reaction:
HA(aq.) + e ~ A-(aq.) + Hz(g). [B.U. 1999]

[Ans. E° = - 0.295 V]
62. A glass electrode is used in conjunction with a calomel electrode. The following
data points are obtained
pH emf (V)
4.01 0.1246
6.99 0.3012
Calculate the value of F, the Faraday constant. [B.U. 1998)
Solo. : For glass electrode in conjunction with calomel electrode,
E = £cal - Eg
EMF 487

= x - R: X 2.303 log a 8+ ( Ecal - E~ = X)


8.314x298.15x2.303 H 5708.72 H
=x+ - - - - - - - - p = x + p .
F F
5708 72
So, 0.1246 = x + · x 4.01
F
5708 72
and 0.3012 =x · +
x 6.99.
F
Hence, F = 96331 coulombs.
63. Calculate the emf of the cell
Pt I H2 (l atm) I HA 1(0.1M) II HA 2(0.l M) I H 2(1 atm) I Pt, where HA 1 and
HA 2 are two weak acids of dissociation constants 10- 4 and 10-5 (M) respectively.
[B.U. 1994]

a + )
Hints : E = - 0.059 log H (HAi [Ans. E = - 0.0295 V]
a
tt+(HA2)
64. The standard electrode potentials for
zn+ 2 + 2e = Zn EJ.98 = - 0.76 V
and Co+ 2 + 2e = Co ~98 = -0.28 v
(i) Which of the two metals is more noble?
(ii) If a solution containing l gm-mole/litre of both ZnS0 4 and CoS0 4 is electrolysed
with Pt-electrodes at 25 °C, what should be the approximate concentration of cobalt ion
when deposition of Zn starts. [C.U. 1994]
Solo. : (i) Since, E° +21 < E° +zi , Co is more noble as co+ 2 is more readily
Zn Zn CO CO
reduced to Co than zn+ 2 to Zn.
Thus, Zn will be more easily oxidized to zn+ 2 than Co to Co+ 2.
(ii) Co+ 2 will deposit first since its reduction potential is greater than zn+ 2 . But
reduction potential of co+ 2 will decrease due to loss of its concentration in solution and
when its reduction potential at certain concentration of co+ 2 equals that of zn+ 2 , then zn+ 2
will start deposition. Let the concentration of co+ 2 be x(M), when
Eco+ 21co = Ezn+ 2tzn .
0.059 0.059
or, Ec 0 +21c 0 + -
2
- log x = Ez0 +21Zn + 2
log l at 25°C.

0.059
or, - 0.28 + - - log x = - 0.76.
2
x = 5.36 x 10- 17 mole I litre.
65. Standard electrode potential for the Ag+ I Ag electrode is 0.7991 V at 25 °C and
solubility product for Agl is 8.2 x 10- 17 . What is the standard electrode potential for
1-, Agl I Ag?
Solo. : The required cell for measuring solubility product of Agl is
Ag I Ag+ II 1-, Ag I I Ag.
488 PROBLEMS ON PHYSICAL CHEMISTRY

The reaction at R.H.E. Agl ~ Ag+ + 1- (l)


Ag+ + e =Ag (2)

Adding ( 1) and (2), Agl = Ag + 1-


Hence, £~-.AgllAg = £~-.AgIJAg - 0.059 log a,- (at 25 °C).
= E~g+JAg + 0.059 log aAg+ (disregarding equation 1)

= E~g+JAg + 0.059 log :s (Ks = solubility product = aAg+ ar)..


I
0 0
Thus, £ I _ I
,Ag! Ag
= £ Ag +J Ag + 0.059 log Ks

= 0.7991 + 0.059 log (8.2 x 10- 17) = - 0.149 V.


66. If (a±)I and (a±)z are 9.05 x 10-3 and 1.75 x 10-2 respectively and if.mobilities
of hydrogen and chloride ions in a solution of hydrochloric acid are 33.71 x 10- 4 cm 2•
v- 1.sec 2 and 6.84 x 1o- 4 cm 2.v- 1.sec 2 respectively, calculate the liquid junction potential.
[V.U. 1991]
Solo. : Let the cell be

(~)
Eup = (t_ - t+) x 0.059 log ( )
a±I

t =-
u_
-= 6.84xlq-4 = O.l 7
- U+ + U_ 33. 71x10-4+6.84x10-4
t+ = 0.83

I. 75x10-2
Hence, Eup = (0.17 - 0.83) x 0.059 log .0SxI0_3 = -0.011 V.
9
CHAPTER14

ADSORPTION, CATALYSIS, COLLOIDS

Required Formulae
(a) Adsorption :

(i) Langmuir equation : (J = KAP , where KA= adsorption equilibrium constant


l+KAP
(J = fraction of surface covered.
(ii) If the substances A and B are adsorbed and the partial pressures PA' P8 are at
equilibrium, then

(iii) Freundlich equation : ~ = (J = KP 11n, where K and n are constant, and


m
where x = amount of adsorbate adsorbed
m = amount of adsorbent
P = equilibrium pressure.
(iv) dlnKP = Aff .
~ RT2
0
But, by Langmuir equation KP =
1-0
0
:. In K + In P = In
1-0
At constant coverage, d In K + d In P = O.
dT dT
. dlnP _ Mi
.. ( f f - - RT2 .

P Aff
i.e:,i tn-1..=-
. P1 R
[T1 -T2 ] ,where · the terms.have usual meaning.
T 1T2

489
490 PROBLEMS ON PHYSICAL CHEMISTRY

(b) Colloids :

(i) S = x 3002 in volt, where 7J = coefficient of viscosity, 11$ = potential


47tr]V
D.11$11 l
gradient and v = velocity in cm/sec, D = dielectric constant.
4
(ii) l; = m1 Vx: x 3002 in volt, where D =dielectric constant, I= current in ampere,
D.l.
V = volume moving per unit area per second
l; = zeta potel}tial in volt and K = specific conductance.

In these equations, l1<\l -= potential gradient in volt/cm.


l
... ) v
(m = D.l1$!l.l; ·m c mlsec. [II
a terms havmg
. . 'fi1cance]
usuaI s1gm
47tT] x 300 2

(iv) NA =

where NA = Avogadro number


(p - p 0 ) = difference in densities between sol solution (p) and pure solvent (p 0 ).
(h 2 - h 1) = the difference in heights where the no. of particle's reading has been
taken.
n 1 = no. of particles at height h1
n 2 = no. of particles at height h2
T = temperature in K.
r = radius of the sol particles
R = ideal gas constant in ergs.
(c) Catalysis :
(i) J = ko + kci Cc1 + kc2Cc2'
where k0 = specific rate in absence of catalyst
kCJ = catalytic coefficient of catalyst C 1
cc 1 = concentration of catalyst C 1

kcz and cc are respectively catalytic coefficient and catalyst concentration


2
of catalyst 2.
(ii) J = J o + kf/BCHB+kB_CB_'

where J = first order rate constant or total catalytic coefficient being equal to
I dC
c. dt
C = concentration of the reactant, t = time.
I

(iii) J = k fl + . KJ ,
ADSORPTION, CATALYSIS, COLLOIDS 491

where kH+ = catalytic coefficient of H+


Ka = acidity constant of the weak .acid.
(iv) In enzyme catalysis,
Maximum rate R
Turnover number = Initial enzyme concentration
=- 0
CE
·

Short Questions
(a) Adsorption :
(i) What happens in common cases of absorption?
Ans. : In absorption the foreign substances undergo uniform penetration into the field
of force that exists between atoms, ions or molecules, in the bulk of the absorbent. In
some cases, this will lead to the formation of homogenous mixture, that is solution. In
many other cases this may lead to·compound formation.
(ii) "Physisorption must be exothermic."-Explain.
Ans. : Physisorption is a weak adsorption taking place in the surface of a solid
where van der Waals type forces are _operative. The process is spontaneous, and thus
L'!G =!'l.H - TL'! S must be negative. Since the translation freedom of the adsorbate is reduced
on its adsorption, !!.S is negative. Thus, !'l.H has to be negative to make L'!G negative.
Therefore, physisorption has to be exothermic.
(iii) "Adsorption has to be exothermic."-Comment.
Ans. : Adsorption is a spontaneous process with L'!G being (-)ve. For gas with three
degrees of freedom adsorption means the loss of one degree of freedom (as adsorbed gas
molecule is mobile in 2 dimensions). For immobile type of chemisorption, translational
degrees of freedom are reduced to zero. Thus, !!.S is negative and !'l.H has to be negative
for L'!G to be negative.
In this approach, the adsorbent has been treated as completely inert. But, there are
cases when it may swell making !!.S to be (+)ve. What is more, in case of dissociative
chemisorption, !!.S may be positive. For. these cases, at low temperature T!!.S may be
sufficiently negative so that !'l.H need not be negative to make L'!G negative. It can be +ve.
Adsorption of H 2 on glass at room temperature has been found to be endothermic.
(iv) Why do electrolytes increase the surface tension of a liquid?
Ans. : Electrolytes attract solvent molecules very strongly. The solvent-solute
interaction is stronger than solvent-solvent interaction. In the bulk, the amount of solvent
available is larger and solvent molecules may surround the solute molecules from all sides.
This increased opportunity in bulk for attraction leads to depletion of ions from the surface.
As a consequence, the surface solvent molecule apparently feels an inner side pull and

surface tension (y) increases. This is also" given by Gibbs equation. r 2 = __£_. dy. Since
RT de
r2, surface excess, is negative, so dy is (+)ve. i.e., increase in concentration leads to
de
increase in surface tension.
492 PROBLEMS ON PHYSICAL CHEMISTRY

(v) Show that for a gas obeying Lang1J1uir equation a plot of In~ vs. ()will be linear
p

with a slope of -l. What will be the slope of In V vs. V plot?


p

Ans. : By Langmuir equation 0 = KA.P


l+KA.P
or, _0_ = KA or, ~ = KA (I - 0 ).
1-0 p

. . In~ = In KA + In (I - 0 ).
p
When 0 is small, In (I - 0) = -0.
0
So In - = In KA - 0. (I)
p

Hence, a plot of In~ vs. 0 will be linear with -I as slope.


p
. 0
Agam, = -v
vm
Putting the value of 0 in eqn. (!),
v
In--= In KA -
v
PVm Vm
v v
In- = In (KA. Vm) - -
p vm
Hence, a plot of In~ 1
vs. V will be linear with - - - as slope.
p vm
(vi) Write down B.E.T. equation. Explain all the terms.

Ans. : B.E.T. equation is : F = _I_+ C - I .F.


V(l-F) VmC Vm.C

F = _!_, where P is pressure at which the adsorption is started.


Po
and P0 = saturated vapour pressure of the adsorbate as a liquid.
_ -(£1-EL)/RT
C - e '
where £ 1 = heat of chemisorption in the first layer
Ei = heat of liquefaction of adsorbate gas.
Vm = monolayer volume (generally reported at NTP).
(vii) Write the B.E.T. equation. Under what condition it is converted to Langmuir
equation?

Ans. : B.E.T. equation is F = _I_+ C-1 .F·


V(l-F) VmC Vm.C
[where all the terms have usual meanings]
ADSORPTION, CATALYSIS, COLLOIDS 493

Let C >> l.
Now, equilibrium between gaseous and liquid adsorbate is
l
g~I. K= Po'
In B.E.T. method, adsorption is comparable with liquefaction.
Let P0 >> P.

.. P = . + _!__ which is the Langmuir equation with K = Ka.


V Vm K Vm
(viii) Explain the following :
(a) The decomposition of NH 3 on tungsten is of zero order.
(b) The decomposition of NzO on gold is of first order.
(c) The recombination of H atoms on gold is of second order.
(d) The decomposition rate of NH3 on Pt depends on p
(NH3)
/P(H2) .
(e) The decomposition rate of NH 3 on Mo is strongly retarded by N2 , but does
not approach zero as the surface becomes saturated with nitrogen.
Use Langmuir-Hinshelwood model and be as brief as possible.
Solo. : (a) Ammonia is very strongly adsorbed on tungsten surface so that rate
R = k2.fJ, where k2 = rate constant, () = fraction of surface covered.

.. R = k2 • KAP , where KA = adsorption equilibrium constant, P = pressure of


(l +KAP)
the adsorbate. For very strong adsorption KA.P > > l, so that

R = k2
KAP. :. R = k2.P0 •
KAP
Hence this is of zero order..
(b) N20 is weakly adsorbed on gold surface, so that
R = k 2.e
= k2 • ( KAP ) [all terms have usual significance]
1+KAP ·
In case of NzO, l >> KAP.
:. R = k 2 .KAP,
which is of the first order.
494 P~OBLEMS ON PHYSICAL CHEMISTRY

(c) H atoms are weakly adsorbed on gold. And the rate is proportional to the collision
of two·'act~O'rbed H atoms .
2
. . Rate = k2 0 2 = k2 ( KHPH ) l > > KH.PH.
l+KH.PH

2 2
:. Rate = k2 . (KH) (PH) .

Thus, the rate is of second order. Here, KH is adsorption equilibrrium constant of H


atom, PH is the partial pressure of H atom.
(d) Product H2 of NH3 decomposition is strongly adsorbed on active centres, where
NH3 is weakly _adsorbed. Let KA be the adsorption equilibrium constant of hydrogen
maintained at a pressure PA· KB and PB are those of NH 3 . Then Rate = k2 .0A. OB .

.. Rate= k{l+K:;~:AK8 P8 )(l+K:;~:8K8 P8 )


Now, KAPA>> I and KAPA>> KBPB.

As already indicated, PB = PNHand PA = PH 2 •


3
p
. . E.ate = k. ~, where k is a constant and is equal to k2 K-;. 1 K8 .
PH2
(e) There are obviously at least two types of active centres on Mo surface. In one
type of active centre, NH3 is weakly adsorbed and N 2 is strongly adsorbed so that rate
PNH3
=KA.--.
PN2

So, as decomposition proceeds, pN increases and the rate decreases. But obviously,
2
there is at least another type of active centre of low concentration on which only NH3
is adsorbed. So, rate of decomposition of NH3 does not become zero.
[For details see the chapter on Adsorption of Physical Chemistry, Vol. II by
Dr S. Pahari.] ·
(b) Colloids :
(i) "Deltas are formed at a place where rivers pour water into sea."-Comment.
Ans. : River water contains mud which is clay in colloidal form. These are stable due
to charge. In the sea, there are salts. So, when rivers pour water into sea, clays are mixed
with salts. Their charges are neutralised. From colloids, these become suspension and get
precipitated and continuous deposition through years leads to the formation of a delta.
(ii) "Colloid solutions are thermodynamically unstable."-Comment.
Ans. : Colloids are particles being in between crystalloids and suspensions in size.
Colloid particles are so large (about 100-1000 A in diameter) that tl'ley have a distinguishing
surface. So, if a large number of such particles fuse, the total surface area and hence,
ADSORPTION, CATALYSIS, COLLOIDS 495

free· energy will decre~se. Therefore, colloids are thermodynamically unstable showing a
tendency to coalesce. The reason of their stability is electrokinetic-their motion and
adsorbed charges on them prevent collision of two such particles.
(iii) What is the absolute primary role of double layer?
Ans. : The primary role of double layer is to confer kinetic stability to the
thermodynamically unstable colloids.
(iv) If a very small atnount of AlC1 3 is added to gold sol, the gold flocculates, but
·if a large quantity of AlC1 3 is added, there is no flocculation.-Why?
Ans. : Gold sols are lyophobic and negatively charged. Addition of small amount
of A1Cl 3 thus leads to reduction of 1;,-potential and hence, flocculation of the sol. However,
if a· large amount is added, greater amount of AP+ is adsorbed making gold sol positive
and kinetically stable.
(v) "Presence of H2S is essential in As 2S3 sol, though H 2S ionises and should be
precipitated as As 2S3."-Explain.
Ans. : As 2S 3 sol can exist as a sol, only if it adsorbs some ion. As 2S3 sol owes its
stability mainly to adsorbed s= ion. Because of adsorption of charge, small particles cannot
collide directly to yield a precipitate. H 2S ionises weakly to H+ and S= ions. The
concentrations of s= is extremely small. But the adsorption of this amount of S= confers
some 1;,-potential to the suspensoid and As 2S3 soln. is stabilised. And, thus, although passage
of H 2S to As 3+ solution. precipitates As 2S3 ; some extra H 2S must be present for existence
of As 2S3 sol.
(vi) Why do colloids precipitate when they are heated?
Ans. : Colliding colloidal particles can break through the double layer and coalesce,
only if the collision is sufficiently energetic to disrupt the layers of ions and solvating
molecules. Of course, this is also possible, if as a result· of brisk motion, the surface
accumulation of charge is stirred away. This may happen at high temperature, when the

velocity (K.E.. = .!.mv2 ) is expected to be high, and that is why sols precipitate when they
2
are heated.
(vii) "Uncharged colloids exist and are important, both in aqueous and nonaqueous
media."-Explain their stability.
Ans. : A colloidal sol is a heterogenous system and the units tend to coalesce which
is a process that reduces negative free energy and hence; these coagulate. The adsorbed
charges confer kinetic stability. However, solvation also is another important factor which
may prevent the collisions of particles. These may not break through their solvent layers.
The solvation may be a result of dipole-dipole attraction. We generally speak of hydrogen
bonds with surface in aqueous media and van der Waals attraction in non-aqueous media
for explaining the reason of solvation.
(viii) "Lyophobic colloids flocculate only when the charges on the collidal particles
are completely neutralised by addition of electrolytes."-Comment.
Ans. : The above assertion is not completely correct. Colloids are kinetically stable
due to 1;,-potential resulting from the adsorption of charge by the particles. The repulsion
of the charge is a factor why the particles cannot collide and hence, cannot coalesce to
496 PROBLEMS ON PHYSICAL CHEMISTRY

form precipit<fte. When electrolytes are added, the charge is neutralised. Completely
neutralised lyophobic colloids are unstable and must form precipitate. However, colliding
colloidal particles can break through the double layer if the collision is sufficiently energetic
to disrupt the layers of ions and solvating molecules. Thus, at every temperature, there
will be a minimum value of /;-potential below which this is possible. This has been found
to be about 0.02-0.03 volts. Thus, for coagulations, complete neutralisation is not needed
(partial neutralisation is sufficient).
(ix) Acids and Bases are particularly effective as peptising agent-Why?
Hints : H+ and Oir ions are especially strongly adsorbed by uncharged surface.
(x) Organic salts are more powerful than corresponding inorganic salts.-Why?
Hints : Organic ions are more strongly adsorbed on charged surface in comparison
to inorganic salts.
(xi) On what factors does the course of coagulation depend?
Ans. : Mainly two factors-Brownian motion of the particles and interaction between
the particles when they are close enough.
(xii) We speak of zeta-potential in case of colloids but not in case of salt solutions,
although both colloidal particles and ions in salts are charged.-Why?
Ans. : Salt solutions result from dissolution of salts in a solvent which is generally
water. This process is spontaneous with negative free energy. However, the solution is
homogenous and there is no surface of separation between ions and solvent. In case of
colloidal sol, the solution is heterogenous for the large size of colloidal particle which
are stable· kinetically due to the adsorbed charge on them so that they are repelled from
one another. Thus, there is a surface with charge separation, and naturally a potential
difference is set up. A part of the potential is zeta-potential and we speak of /;-potential
in discussing this.
(xiii) Both emulsifier and peptising agents stabilize coJloids but their actions are
different-Explain. c;
Ans. : Colloids are stabilized by peptising agents because colloid particles adsorb
preferably one of the ions of peptising agents at their surface. As a result, !;-potential
- originates and colloids become stable kinetically.
In case of emulsion, the two parts of the emulsifier are adsorbed at the two exposed
liquid surfaces and the interfacial tension is reduced. The nature of the emulsifier determines
the type of emulsion. The film curves toward the phase on the surface with higher tension.
Univalent cation soap-like sodium oleate is more wetted by water, the surface ~ension is
lower on water side. Calcium or aluminium oleates are less soluble in water-these are
more easily wetted by oil, and water surface is curved giving water/oil emulsion. Thus,
no doubt there is adsorption and double layer, but the emulsifier acts by reducing surface
tension of a particular surface due to greater wetting (greater preferential adsorption).
(c) Catalysis :
(i) A catalyst remains unchanged at the end of reaction.-Explain.
Ans. : A catalyst is both the reactant and the product of a reaction. It takes part in
a reaction to form a stable or unstable intermediate /. This may be depicted with reactant
ADSORPTION, CATALYSIS, COLLOIDS 497
R, product P and intermediate I as follows
R+C~l---tP+C.

Thus, the catalyst is regenerated in the final stage. So, the above statement is true.
Although, it is chemically unchanged but physical form may be changed.
(ii) A catalyst cannot affect the position of equilibrium of a reaction.-Explain.
Ans. : Let us consider a reversible reaction,
kl
A~B,
k_I
where k1 and k_ 1 are the specific rates for the forward and backward reactions respectively.
In the given case, the steps are elementary so that the equilibrium constants,
A -EilRT
K = -k_k1 ' ". K i·e
=---'--_~E2-l~R=T '
1 A .e2

(E1 -E2)
. . uncatalysed K = ~~ .e -~.
A catalyst reduces· the activation energy by x, so that the activation energy now is
(E1 - x). Similarly, activation energy for the backward reaction is (E2 - x). There is no
reason for the change of frequency factor A. Therefore, equilibrium constant of the catalysed
reaction [(C is

' c -[(E1-x)-(E2-x)]
[(C = -c.e
A1 RT
A2
(E1-E2)
= ~.e----,u- = K.
A2
There is no reason why AC and A will be different. So, KC= K; the equilibrium position
is not changed.
(iii) "A catalyst eliminates undesirable side .products."-Comment
Ans. : A catalyst is specific and reduces the activation energy of a specific path by
taking part in the formation of the activated complex of the path. Let a substance A
decompose to product B and D in two different paths.
A--7 B
A--7 D
If the activation energies are comparable, then B and D will occur together. Let a
catalyst C1 reduce activation energy of A --7 B pathway.
Thus, in presence of Cl' B will be formed first, and if collected, B will be the sole
product. Thus, the catalyst C1 eliminates the undesirable side product, D. Similarly, another
catalyst C2 may eliminate B and give solely D.
(iv) Negative catalysts are not catalysts.-Explain.
Ans, : The so-called negative catalysts are substances that retard the rates of reaction.
These substances can do so by increasing activation energy (and /or by decreasing the
498 PROBLEMS ON PHYSICAL CHEMISTRY

frequency factor). But, catalysts are defined as substances that increase the rate of reactions.
Catalysts generally reduce the activation energy by taking part in the activated complex
formation. They may do so also by increasing frequency factor. Thus, negative catalysts
are not catalysts. They are nowadays termed as inhibitors.
(v) It is not essential for a catalyst to decrease activation energy. Comment.

Ans. : A catalyst increases rate by increasing specific rate k. But k = A. e-E/RT, where
A = frequency factor, E = activation energy.
Hence, a catalyst may increase k by decreasing E. But it also may increase k by
increasing A, and without decreasing E. It may even increase E and yet increase rate if
A is increased sufficiently.
(vi) If a catalyst does not take part in intermediate formation, then there is limit of
high concentration up to which the rate will depend on the catalyst concentration.
Ans. : Let a catalyst C catalyses a reaction A to P by just activating A to A* and
not taking part in the formation of A*.
That is,
k1
A+ C ~A*+ C.
k_l
Now, A* decomposes to give P.
k2
A*~·P.

Thus, r = k2.CA*·
Assuming ~teady state in A*,
' k1CA.Cc = k_1.CA*Cc + k2.CA*•

k1CA-Cc
k_1Cc +k2

So, r = k2k1CA.Cc .
L1Cc +k2
So, at high Cc, r is independent of catalyst concentration.
(vii) For the reaction R + C----+ P, rate = kCc-CR, when R is reactant, and C is
catalyst. So when Cc = 0, the rate is zero. Comment. [V.U. 2002]
·Ans. : For a catalytic reaction, R ----+ P,
we define, a first order rate consunt.

J = __!_ dCR = ko + kc-Cc,


CR dt
where CR = constant of the reactant
k0 = first order rate constant without catalyst
kc = catalytic coefficient
Cc = catalyst concentration.
ADSORPTION, CATALYSIS, COLLOIDS 499

But, kc-Cc is so much greater than k0 that k0 is neglected. But when Cc is zero kcCc
is zero. Rate constant becomes
J = k0 • Hence the rate is not zero.
(viii) If a catalyst catalyses the forward reaction, then it must catalyse the backward
reaction.-Explain.
Ans. : All reactions are reversible. Let us consider a reaction, A ~ B.

p
Rate---•

The energy vs. reaction coordinate diagram

E 1 is the activation energy of the forward path and E2 is the activation energy of
the backward path. If k1 is the specific rate constant of the forward reaction and k2 is
the same for backward reaction, then

A -E1/RT (E E )
K = !:l_ = 1e _ E
I RT
2
= ~ e-~, where A 1 and A 2 are frequency factors.
· k2 Aze A2
Let the catalyst reduce the activation energy by x. Then activation energy in presence
of catalyst = Ee, = E1 - x

and equilibrium constant Kc in presence of catalyst becomes [(C =


Then, [(C may become equal to K only if E' is E 2 - x.
Thus, if a catalyst catalyses the forward reactio_n, it must catalyse the backward reaction
by the same way.
(ix) What are enzymes?
Ans. : Enzymes occuring in some natural juices, plants, animals, etc. possess incredible
capacity of bringing about many complex chemical reactions. Enzymes are large protein
molecules, namely-diastate, urease, pepsin, zymase, etc. They have much high molecular
weights, usually between 104 -106 . Enzymes disperse in water forming colloidal solution
and are sometimes referred to as microheterogenous catalysts.
(x) What is Michaelis constant?
Ans. : Michaelis constant is a characteristic of enzyme catalysed reaction. Let an
enzyme E react with reactant S to give product P. The reaction has been suggested to
follow the mechanism-
k1
E+S
500 PROBLEMS ON PHYSICAL CHEMISTRY

Here, k1 is the specific rate constant of ES forming reaction.


k_ 1 and k2 are specific rate constants of ES decomposition.

k_, + k2
Michaelis constant KM is defined as KM = --k-,-·

Its unit is moles/litre.


(xi) What is turnover number?
Ans. : The turnover number of an enzyme is the maximum number of moles of product
produced by I mole of the enzyme in unit time. Obviously, it is also the maximum number
of molecules of product produced by one enzyme molecule in unit time.
Numerical Problems
(a) Adsorption :
1. A long chain fatty acid of molecular weight 256 has a density of 0.82 gm/cm 3.
If 0.102 mg of the acid is required to form a close packed monolayer film over 500 cm 2
of water surface, find the cross section and length of the acid molecule. . [C.U. 1985]

02 1 3
Soln. : Let a be the cross section and h be the length. By the problem, O. l x o-
256
moles occupy 500 sq. cm.

3
Thus, 0.102 x I 0- x 6.022 x I 023 x a = 500.
256
:. a = 2.08386 x 10-15 cm 2 = 20.8 sq. A.
Again, v x p = m
or, 500 x ·h x 0.82 = 0.102 x 10-3
.. h = 2.4878 x I 0-7 = 24.9 A.
2. 5.2 x J()-5 gm of stearic acid (M = 284) when placed on water a surface film could
occupy an area of 225 sq. cm as a close packed monolayer, Find cross-sectional area of
stearic acid molecule. . [C.U. 1979]
Soln. : If a be the cross-sectional area of the acid molecule, then by the problem,
5 23
5.2x 10- x6.022x10 x a= _
225
284
225 x 284 •
:. a = x 10-1s = 2.04059 x 10-15 cm2 = 20.4059 sq. A.
6.022 x 5.2
3. In a laboratory experiment 60 cc of a solution originally 0.21 (M) in AcOH was
perm.itted to come to equilibrium with a IO gm sample of charcoal. After equilibrium,
20 cc of AcOH required 30 cc of 0.11 (M) NaOH for neutralisation. Calculate the equilibrium
concentration of the acid and the weight of CH3COOH adsorbed per gm.

Soln. : Equilibrium concentration = 30x0.1 I = 0.165 (M).


20
No. of moles of AcOH initially = 60x0.21 = 0.0126.
1000
No. of moles of AcOH at equilibrium= 60x0.165 = 0.0099.
1000
ADSORPTION, CATALYSIS. COLLOIDS 501

So, no. of moles of AcOH adsorbed = (0.0126 - 0.0099) = 0.0027 moles


= (0.0027 x 60) gm = 0.162 gm.
162
· adsorbe d per gm
. . wt. of acetic. acid = O. -- 0.0162 gm.
10
4. For the adsorption of a substance from aqueous solution by charcoal at 25 °C. the
Freundlich constants are n =3.0 and K =0.50, the concentration of solution being expressed
in gm/litre and the amount being adsorbed in gm/gm. What weight of substance will be
adsorbed by 2 gm of charcoal from 1 litre of a solution containing originally 2 gm of
the substance? [C.U.(H) 1981]

I
Soln. : We know, ~ = K.Cli.
m
By the problem, C = (2 - x) gm, K = 0.5, n = 3, m = 2 gm.
X I
= 0.5 (2-x)3
2
or, x = (2-x)3I
or, x3 =2 - x or, x3 + x - 2 =0
or, (x - 1) (x2 + x + 2) = 0, :. x = I.
i.e., 1 gm of the substance will be adsorbed.
5. The adsorption of a dye from solution on charcoal is governed by the Freundlich
isotherm in which n = 2 and K = 6.8. The concentration of the dye is expressed in milli-
moles/cc and the amount adsorbed in millimoles/gm. If 10 gm of the charcoal be shaken
with 100 cc of a 0.1 (M) solution of the dye, then what will be the equilibrium concentration
of the solution.?
Soln. : We know that 0.1 (M) = 0.1 x 10-3 moles in 1 cc = 0.1 millimoles/cc.
Thus, C before adsorption is 0.1 millimoles/cc. Let x millimoles/cc be adsorbed. Hence,
Cat equilibrium is 0.1 -x. But 10 gm of charcoal adsorbs IOOx millimoles from 100 cc.

So, for -
x = K.cn,
-I
m
1
X = lOOx; K = 6.8, m = 10, C = 0.1 - x, n = 2·
Thus, lOOx = 6.8~(0.l-x)
10
i.e., IOx = 6.8~(0.l-x)
i.e., 100x2 = 46.24 (0.1 - x)
i.e., 100x2 + 46.24x - 4.624 = 0.
:. x = 0.0845 millimoles/cc = 0.0845 (M).
:. the equilibrium concentration = (0.1 - 0.0845) = 0.0155 (M).
6. The adsorption of a dye from solution on charcoal is governed by the Freundlich
isotherm, where n = 2 and K = 6.8. The concentration of the dye is expressed in milli-
moles/cc and the amount adsorber' 111 i111l1111. ''es/gm. If 10 gm of the charcoal is shaken
502 PROBLEMS ON PHYSICAL CHEMISTRY

with (a) 1 litre, (b) 10 cc at 0.1 (M) of the dye, then what will be the equilibrium
concentration of the solution? [V.U. 1995]
Hints : Proceed as Problem 5.

(a) The equation is IOOOx = 6.8~(0.l-x).


IO
[A~s. O. l_ - x = 0.081 (M) J

(b) The equation is _!9x = 6.8 ~(0.1-x)


IO
[Ans. 0.1 - x = 0.00022 (M)J
7. Calculate the surface area per gm of a finely powdered solid for which the volume
of Ni(g) adsorbed per gm at STP to form a monolayer was found to be 3.01 cc. Given
area of N 2 molecule is 16.2 sq. A.
Solo. : At NTP, 22400 cc contains 6.022 x I0 23 N2 molecules.
23
3.01 cc contains 6 · 022 x I0 x 3. 0 I
)
molecules.
( 22400

. 6.022 x I0 23 x 3.01
Hence, the surface area 1s x !6.2 x 10--1 6 sq. cm
22400
= 131091.41 sq. cm = 13.U sq. m.
8. For the adsorption of a substance from aq. solution by a solid at 20 °C, the Freundlich
constants are n = 2 and K = 0.5, the concentration in solution being expressed in gm/
litre and the amount being expressed in gm/gm. What weight of the substance will be
adsorbed by 2 gm of the solid from one litre of solution containing initially 3 gm of the
substance?
Hints : Proceed as Problem 4.
Here the equation is :
t I
.:. . . = (0.5) (3-x)2.
2
x = 1.3 gm.
9. The adsorption of a gas is described by Langmuir isotherm with K0 = 0.85 kPa-- 1. Find
the pressure at which the surface coverage is (a) 15%, (b) 95%.
Solo. : (a) 15% coverage means, e = 0.15.
Ka.P
.. by Langmuir equation, 0.15 = l+Ka.P

0.85kP; 1
or, 0.15 = 1+0.85kP; 1
I.
or,
0.15 = 0.85P + 1

or, 0.85 = 0.85P


0.15
0.15
or, 0.85P = 0.85
ADSORPTION, CATALYSIS, COLLOIDS 503

0.15xl
:. P = 2 = 0.2076 kPa = 0.21 kPa.
(0.85)
(b) 95% coverage means, e= 0.95.
Therefore, proceeding as in (a),

0.05
0.95 0.85P
:. p = _o_.9-5- =22.35 kPa.
0.05x0.85
10. A certain solid sample adsorbs 0.44 mg of CO when the pressure of the gas is
26.0 kPa and the temperature is 300 K. The amount adsorbed when the pressure is 3 kPa
and temperature is 300 K is 0.19 mg. The Langmuir isotherm describes the above process.
Calculate the fraction of surface covered.

vve know,
So In. : UT e = _!i!_,
l+KP
KP1 = ad sorpt1on
. equi·1·b.
= ---'--,
l+KP
where K 1 num constant.
1

=
KP2 . A gam,
. e
1 _ w1
1+ KP 2 e2 - W2 .

~=
I+ KP2 = Pi 1 + KP2
Wz KP2 P2 · 1+ KR,

By the problem, w1 = 0.44 mg, w2 = 0.19 mg, P1 = 26 kPa, P2 =3 kPa.

0.44 = 26
1+3K
- - -
~.19 3 1+26K
or, K = 0.184.
0.184x26
Now, fraction of surface covered, e1 = = 0.83.
l+0.184x26
0.184x3
e2 = I +0.184x 3
= o.36.
11. A certain solid sample adsorbs 0.52 mg of hydrogen when the pressure of the
gas is 34 kPa and the temperature is 35 °C. The amount adsorbed when pressure is
5 kPa at the same temperature is 0.21 mg. The Langmuir isotherm describes the above
process. Calculate the fraction of surface covered.
Soln. : Proceed as Problem 10. [Ans. e1 =0.75, e2 =0.30)
12. A solid with surface area independent of temperature adsorbs 2 mg of a gas, when
in contact with the gas at a pressure of 10 kPa and a temperature of 283 °K. The system
follows Langmuir equation. The amount of heat absorbed when 2.00 millimole of the gas
is desorbed in 80 J. What is the equilibrium pressure for the adsorption of 2 mg of the
gas at a temperature of 323 K?
Soln. : By the problem, P 1 = 10 kPa, T1 = 283 K, T2 = 323 K, P 2 =?
504 PROBLEMS ON PHYSICAL CHEMISTRY

Heat absorbed for 2 millimole = 80 J


" millimole = 40 J
mole = 40 x 103 = 40 kJ.
. . t:.H0 = - 40 kJ, where t:.H0 = heat of adsorption/mole.
din P d In Ka _ t:.Ha
We know that, ---;;;:- = - -;rr- - - RT2 ,
when e is temperature independent.

or, In P2 = 40x 10
3
[ -40 J
IO 8.314 323x 283

or, In p2 = 2. 105334.
IO
:. P2 = IO x 8.2098 kPa = 82.0984 kPa = 82.1 kPa.
13. A solid in contact with a gas at a pressure of 10 kPa at 300 K adsorbs 2 mg
of the gas. The system follows Langmuir equation. The heat released for the adsorption
of 1 millimole is 15 J. What is the equilibrium pressure for the adsorption of 2 mg of
the gas at 37 °C. Assume that the surface area is independent of temperature.
Soln. : Proceed as Problem 12.

In P2 = t:.H
Pi R
-T2 ]·
.1iT2
[7i [Ans. P 2 = 12.14 kPa]

14. The chemisorption of H2 on manganese is activated. It proceeds at a rate 35%


faster at 1000 K than at 600 K. Calculate the activation energy of chemisorption.

Soln. : We know that In -2 k =E[T-T] _g_ -


2
1-- ,
k1 R T1T2

where k 1 and k 2 are specific rate constants at temperature T 1 and T2 respectively.


By the problem,
k2 = 135, k1 = 100, T2 = 1000 K, T1 = 600 K.

.. In 135 = __!!y_x( 400 )


100 8. 314 600 x 1000
or, E0 = 3.74 kJ,
15. Argon was absorbed on charcoal to the extent of 0.835 gm/cc at a pressure of
2.4 atm and a temperature of 190 K. But at 250 K the same amount can be adsorbed only
when the pressure is 16 atm. Calculate the heat of adsorption.

Solo. : We know that In P2 =Mi


Pi R
-T2 ]·
1iT2
[7i
By the problem, P 2 = 16 atm, P 1 = 2.4 atm., T 1 = 190 K, T2 = 250 K.
ADSORPTION, CATALYSIS, COLLOIDS 505

16
190x250x8.314ln-
.. !iH =- 2.4
60
.. t;,,.H = -12.5 kJ.
16. Adsorption is studied on a clean surface. Let a solid having density of !'urface sites
1.3 x 10 16 cm- 2 be placed in a high vacuum chamber at a pressure of 1.00 x 10- 12 mm of
Hg at 298.15 K. Let there be air of average molecular weight 29 be present. How Jong will
it take to occupy I% of surface sites, assuming that ~very striking molecule is adsorbed.

Soln.: Number of molecules striking the surface at 298.15 Kand 1.00 x 10- 12 mm
of Hg
PNA
= --)2rcMRT
12
1 00 x 10-
· x 1.01325 x 10 5 ) NI m 2 x 6.022 x 10 23 mole- 1
( 760
=
~2 x rt x 29 x 10-3 kg.mole- 1 x 8.314 J.K- 1 .mole- 1 x 298.15K
= 3.778 x 10 12 m-2 .sec- 1 x 10-3 = 3.778 x 108 cm-2.sec- 1.
I~ one· second the fraction of surface sites occupied by the molecules

= 3.778x!Os = 2.9 x 10-s.


1.3x10 16
Hence, time taken by the molecules to occupy I% of the surface sites

-- 3. 778x108 -- 3 •45 x 1"'"


u- sec.
1.3xl0 16
17. From the following data for the adsorption of N2 gas on a solid surface, calculate
the constant involved in Langmuir isotherm; P = oo, V1 = 180 cc, P = 3.5 atm, V2 = 100
cc/gm [V = volume of gas adsorbed). [8.U. 1996)

Soln. : By Langmuir equation, 0 = ~ = __!!__.


Vm l+KP
Now, P 1 = 00
, V1 = 180 cc/gm; P2 = 3.5 atm, V2 = 100 cc/gm.
180
Thus, -
100
=
l+KP2 • 1+3.SK
= ---
KP
(since, P 1 = oo) = 3.SK
2
K = 0.36 atm- 1•

18. Adsorption of N2(g) on a piece of solid graphite was studied at 97 K. The volumes
of adsorbate are reported at different gas pressures
Platm 3.49 10 16.70 25.70 29.55
Vice 3.28 4.57 4.98 5.27 5.36
Assuming that the Langmuir equation is obeyed, determine K 0 and Vm' the monolayer
volume.
506 PROBLEMS ON PHYSICAL CHEMISTRY

Solo. : By Langmuir equation,


v
vm
Choosing any two readings,
3.28
4.57

4.98
= 16.70Ka
.
1+25.70Ka K
, a
= O.32 atm
-I
5.27 1+16.70Ka 25.70Ka

_2_5._7_0_K~a- 1+29.55Ka
= -1+25.70Ka Ka __ O.26 atm -I .
5.27
. ,
5.36 29.55Ka

Thus, Ka = 0.37+0.32+0.26 = 0.32 atm-1.


3
Choosing any one reading,
5.36 0.32x29.55
vm l+0.32x29.55.
.. vm = 5.927 cc.

19. At 27 °C and at a pressure of 1 atm, only 0.25 fraction of the total surface of
an iron catalyst is covered by molecular nitrogen. What is the standard free energy change
for adsorption pressure at 27 °C? [V.U. 1998)
1
Solo. : By Langmuir isotherm, 0.25 = x Ka (since, P =I atm)
l+Ka
Ka = 0.33 atm- 1
!1G 0 =-RT In Ka = - 8.314 x (273.15 + 27) x In 0.33 = 2.767 kJ.
20. A film containing 4.22 x I o-5 gm fatty acid of molar mass 228 gm spread on
water was compressed to a monolayer at 25 °C occupying an area of 245 sq. cm. The
density of fatty acid is 0.812 gm/cc. Calculate (a) the area of cross section and (b) the
length of the molecule (c). Also calculate the surface tension if surface tension for water
at 25 °C is 0.072 Nm- 1
Solo. : Proceed as Problem I.
(a) a = area of cross section = 21.98 sq. A
(b) h = length of the molecule = 21.21 A
(c) Now, cr = area occupied by one mole of fatty acid
= 6.022 x 10 23 x 21.98 sq. A
= 6.022 x 1023 x 21.98 x 10- 20 sq. m c10- 10 m = I A)
Now, 7tCT = RT (1t = surface pressure)

8.314x(273.15+25) I I
Hence, 7t = 6.0222xl0 23 x21.98xl0- 20 N.m- = O.Ol 37 N.m- ·

Yo - Y = 7t.
Thus, surface tension of the solution, y =y 0
- 7t = 0.072 - 0.0187 = 0.053 N.m- 1•
21. Find t.Hacts for N 2(g) at 1.03 bar given that 155 cm 3 is adsorbed by 1 gm charcoal
at 88 K and 15 cm 3 at 273 K. [V.U. 2000]
ADSORPTION, CATALYSIS, COLLOIDS 507

Solo. : Let k 1 and k 2 be desorption rate constants at T 1 and T2 respectively. Then,


using Arrhenius equation,

In~ = Ed x Tz -Ti ,
k1 R T 1T 2

where Ed = activation. energy for desorption ..


Now, rate of desorption is inversely proportional to volumes adsorbed. So, by the
problem,

In~=~ x 273-88
IS 8.314 273x88
.. Ed = 252) .39438 J.
Now, assuming physical adsorption to be non-activated,
since Ea - Ed = tJfads' Ed = -tJfads (when Ea = 0).
Thus, tJfads = -2521.39 J = -2.5214 kJ.
(b) Colloids :
1. A quartz particle of diameter l x 10-14 cm in aqueous suspension at 25 °C
(llw = 0.8903 CP) migrates with a velocity of 3 x 10-3 cm/sec under an applied potential
gradient of IO V.cm- 1. Calculate the zeta-potential. Given, the dielectric constant of water
is 78.30. [C.U. 1986)
41tllv
Solo. 1; = x 3002 volt
D.( ~$)
Here, Tl = 8.903 x 10-3 poise

D = 78.30, v = 3 x I 0-3 cm/sec, M = I0 V.cm- 1.

:. 1; = 4 x 3.142 x 8. 903 x 10-3 x 3 x 10-3 x 3002 = 0.0385837 v = 0.039 v.


78.30x 10
2. The volume of a liquid transported through porous diaphragm is 1.8 cc/per sec.
If the viscosity of the liquid is 8 x I0-3 poise, and the specific conductance of the solution
is found to be 1.61 x 10-3 ohm-1.cm-1, calculate the 1;-potential in volts when the flowing
current was found to be 6 amperes and dielectric constants of the liquid is 80.

4
Solo. : We know that, 1; = 7tT1 VK x 9 x 104 volt.
DI
Here, Tl = 8 x I0-3 poise, V = 1.8 cc, K' = 1.61 x I0- 3 ohm-1.cm-1, I = 6 A.

4 x 3.142 x 8 x 10-3 x 1. 8 x l.61x10- 3 x 9 x 10 4


:. 1; =
80x6
= 0.0546331 = 0.055 volts.
3. Calculate the electrophoretic velocity of colloidal particles with zeta-potential of
0.102 volt in a medium with dielectric constant 80 and viscosity coefficient of7.8 x I0-3 poise
at 25 °C, if the applied potential gradient is 4.8 volt/cm.

Solo. : The velocity in cm/sec is given by v = l;DE


47tT1x300 2
508 PROBLEMS ON PHYSICAL CHEMISTRY

Here, 1;; = 0.102, 17 = 7.8 x 10-3 , D = 80, E = 4.8.

:. v = _ _ _o_.1_0_2_x_8_0_x_4~._8_~ = 4.44 x 10-3 cm/sec.


4 x 3.142x 7.8x 10-3 x 300 2
·4. In equilibrium distribution of gold sol, in the gravitation field, the following data
were recorded :
Radius of the particle = 629 A.
t!.h = h2 - h 1 = 44.6 x 10-4 cm.
Density of sol= 19.31 gm/cc, density of pure solvent= 0.89 gm/cc, number of particles,
n 1 = 327.6, n 2 = 40.34. T = 293 ~· Calculate Avogadro number.

ne know, N0 =
So In. : uT RT In~.
4
- 7tr3 (p - Po )gtih nz
3
7
.. NA= 3x8}14xl0 x293 In 327.6
4x3.142x(629x10-8 ) (19.31-0.89)x98Jx44.6x 10-4 40.34

= 6.07235 x 1023 ""' 6.07 x 1023.


5. The volume of liquid transported through fine capillaries is 10 cc per sec. If the
viscosity of the liquid is 0.8 CP, the specific conductance of the solution is 2 x 10- 3
ohm-1.cm- 1 and the current flowing is 10 amperes, then calculate potential if the dielectric
constant is 80.

Solo. : We know, 1;; = 47tTJ VK x 9 x J04


DI
Here, Tl = 0.8 CP = 8.0 x 1o- 3 poise:
V = 10 cc, K = 2 x 10-3 ohm-l.cm- 1 , D = 80.0, I = 10 amperes.
Putting in equation, 1;; = 0.226224 V = 0.23 V.
6. If in the above case, K = 1.43 x 10- 3 , I = 6 amperes, what will be 1;;?
Solo. : Proceed as in Problem 5. [Ans. 1;; = 0.27 VJ
7. A quartz particle of diameter 1 x lo- 4
cm in aq. suspension at 25 °C migrates
at a velocity of 3 x 10-8 cm/sec under potential gradient of 10 V/cm. Calculate zeta-potential
at the quartz water interface (TJ of water = 0.89 CP, dielectric constant of water.= 78.3 ).
[V.U. 1994)
Solo. : Proceed as Problem 1. [Ans. s = 3.86 x 10- 7 VJ
8. Calculate the electrophoretic velocity of colloidal particles with zeta potential of
0.04 V in a medium with dielectric constant 78 and viscosity coefficient 8.9 x 10- 3 poise
at 25 °C, if the applied potential gradient is 10.4 V/cm.

ne kn ow,
S0 In. : UT I'
.. = --"'-
47tTJV X 3002.
D. ~
I

D. tic)> .1;;
v = I
47tTJ x 300 2
ADSORPTION, CATALYSIS, COLLOIDS 509

Here, D = 78, 6<)> = 10.4 V/cm, s = 0.04 V, 1J = 8.9 x 10- 3 poise


l
Putting the values,
78x10.4 x 0.04
cm/sec = 3.2 x 10-3 cm/sec.
v = 4 x 3.142 x 8. 9 x 10-3 x (300) 2

9. The flocculation value of ZnCl 2 for As 2S3 sol is 0.68 (millimoles of electrolyte
needed per litre of sol). How many ml of 0.24(M) ZnC1 2 will be sufficient to start the
coagulation of 300 ml of sol?
Soln. : Let x ml of 0.24 (M) ZnC1 2 be needed.
:. total volume = (300 + x) ml.
By the problem, 0.24 (M) means 0.24 moles in 1 litre sol = 0.24 millimole/cc.
Thus, 0.24x millimole is needed.
Now, 0.68 millimole is needed per litre.
68
O. millimole is needed per cc.
1000

0. 68 (300 + x) is needed for (300 + x) cc.


1000

0.68x(300+x) = 0.24x
1000
or, 0.68 (300 + x) = 240x or, 204 = 239.32x.
204
. . x = - - = 0.8524 :. x = 0.85 ml.
239.32
10. The diameter of the micelle of a surfactant is 4 nm and the area of the head
group of the constituent surfactant molecule is 0.20 nm 2 • Calculate the number of surfactant
molecules in the micelle. [V.U. 1999)
Soln. : Diameter of the micelle = 4 nm.
:. radius = 2 nm.
:. total surface area covered = 41t(2)2 (nm)2 = 50.24 (nm)2.
Now, total number of surfactant molecules (assuming close packing)

= 50.24 "' 251.


0.2
11. 100 cc of a silver sol contains 3 x 1010 particles each of diameter 100 nm. If
the solution requires 2.7 x 10- 4 gm of Al3+ to attain iso-electric point. Calculate the charge
density of silver particles. [V.U. 1998)
Soln. : Diameter of each particle= 100 nm = 100 x 10-.9 m = I0-7 m = 10-s cm .

. . surface area covered by each particle = 4. x 3.142 x ( 10 -


2
5
Jcm 2

4 x 3.142 x 10-25
= 4
cm 2 .
510 PROBLEMS ON PHYSICAL CHEMISTRY

Since, the sol contains 3 x 1010 particles, so total surface area covered
= (3x10 10 x3.142x10-25 )cm2 = (3x 3.142x10- 15 )cm2.
Now, charge on I mole (27 gm) of Al 3+ =3 x 96500 coulombs .
. . charge on 2.7 x 10-4 gm At3+

3 96500 2 7 1
= x x · x 0-4 coulombs = 2.895 coulombs.
27
This charge neutralises the charge on sol particles of total surface area

= (3x3.14x 10- 15 )cm 2 •


2.895 14 2
. . charge density = 15 = 3.07 x 10 coulombs/cm .
3 x 3.14 x 10-
12. 2 gm of a water insoluble substance of density 0.89 gm/cm 3 is dispersed in 2 litre
of water, leading to the formation of a colloidal sol containing I 0 14 particles of spherical
shape per mm 3. Determine the radius of the particle.
Solo. : I mm 3 of the sol contains 1014 particles.
.. I cm3 of the sol solution contains 1017 particles.
Now, 2 litres of the sol solution contain 2 x 103 x 1017 particles
=2
x I a20 particles.
Given mass = 2 gm.
Density = 0.89 gm/cm 3.

:. volume = - 2- = 2.2471 cm3.


0.89
2.2471
So, volume of each particle = = 1.12359 x 10-20 cc.
2x10 20
Hence, taking radius r (in cm),

47tr3 = 1.12359 x I~ -20 cc.


l
.. r = (2. 68237 x 10-21 )3 = 2.98052 x 10 - 7 cm
= 1.38944 x 10-7 cm = 13.89A.
13. In a coagulation experiment 5 ml of Agl is mixed with distilled water and 0.1 (M)
solution of NaCl so that total volume is 10 ml. It has been found that all tubes containing
more than 4.9 ml of NaCl coagulates within 5 minutes. What will be the flocculation value
of NaCl for Agl?.
Solo. : Flocculation value is expressed in millimole/litre of electrolyte.
Given experiment, a tube of I 0 ml is taken containing 5 ml of Agl sol and x cc of
NaCl solution, so that the total solution is 0.1 (M) NaCl.
Flocculation takes place when NaCl added is 4.9 ml. •
0.1 (M) solution means 0.1 mole per litre, i.e.,
0.1 x I Q-3 mole/cc, i.e., 0.1 millimole/cc.
ADSORPTION, CATALYSIS, COLLOIDS 511

So, in the given ~oagulation 4.9 ml of NaCl is added so that in the total volume of
I 0 cc, there is 4.9 ml NaCl. Thus, there is 4.9 x 0.1 = 0.49 millimole in 10 cc, i.e.,
0.49 x 100 millimole/litre.
Thus, the flocculation value is 0.49 x 100 = 49 millimole/litre.
(c) Catalysis :
1. The hydrolysis of an ester is catalysed by acid HA, A- ions and Hp+. The rate
constant of the reaction in the following 3 solutions are given in the table :
Cone. of HA Cone. of A- Rate constant
Soln. I 0.2 (M) 0.2 (M) 6.18 x l(J-3
Soln. II 0.2 (M) 0.1 (M) 5.95 ·x 10-3
Soln. III 0.2 (M) 0.05 (M) 8.48 x 10-3
Find the catalytic coefficent of H3Q+, A- and HA. (Given, pKa of the acid 4.75)
Solo. : We know that first order rate constant J =k 0
+ k 1CHA + k2C A- + k3CH+,
where k0 = non-catalytic rate constant.
k1, k2 and k3 are respectively the catalytic coefficients of HA, A- and H+.
k0 is so small that it is neglected. CH+ values calculated for these three buffers are
the following :

. c
Solution (I) ----7 pH = pKd + log--tL.. = 4.75 + log.2.2 = 4.75.
CHA 0.2
:. C
tt+
= 1.78 x 10- (M).
5

Solution (2) ----7 pH = 4.75 + log_QJ_ = 4.75 - (}:3010 = 4.45.


0.2
:. C +
H
= 3.55 x 10-5 (M).

5
Solution (3) ----7 pH = 4.75 + log 0.0 = 4.75 - 0.602 = 4.15.
0.2
:. C = 7.08 x 10 - 5 (M).
tt+
Therefore, by the problem,
0.2k 1 + 0.2k2 + 1.78 x 10 - 5 k3 = 6.18 x 10-3 (i)
Again, 0.2k 1 + 0.1k2 + 3.55 x 10 -5 k3 = 5.95 x 10 -3 (ii)
0.2k 1 + 0105k 2 + 7.08 x 10-5· k 3 = 8.48 x 10-3. (iii)
Subtracting equation (ii) from equation (i),
0.1 k2 - 1.77 x 10 -5 k3 = 0.23 x 10 -3. (iv)
Subtracting equation (iii) from equation (ii),
0.05 k2 - 3.53 x 10 - 5 k3 = -2.53 x 10 -3. (v)
Multiplying (v) by 2,
0.1k 2 .... 7.06x10-5 k3 =-5.06x 10-3. (vi)
Subtracting equation (iv) from equation (vi),
512 PROBLEMS ON PHYSICAL CHEMISTRY

k - 5.29xl0-3 =102.
3 - 5.29x 10~5
Putting k3 in (vi),
0.1 k2 = 7.06 x 19-3 - 5.06 x 10-3
or, 0.1 k2 = 2 x 10-3, :. k2 = 2 x 10-2 .
Putting k2 in equation ( 1),
0.2 kI + 0.2 (2 X 10-2) + 1.78 X 10-S (102) = 6.18 X 10-3
or, 0.2 kI =4 x I 0 -4
or, kI =2 x 10 -3.
2. A reaction is catalysed by acetic acid, acetate ions and proton. The rate constant of
the reaction at 25 °C in two solutions, one containing 0.2 mole acetic acid and acetate ions
and the other containing 0.2 mole acetic acid and 0.1 mole acetate ions are 4.4 x 1o-3
sec-I and 2.4 x 10-3 sec-I respectively. Find the catalytic coefficients of acetate ions and
acetic acid respectively. (assuming C +to be low enough to be ignored) [V.U. 1995)
H30

Soln. : Proceed as in Problem no. I.


However, here the two equations are :
kHA X 0.2 + kA- X 0.2 = 4.4 X 10-3 (I)
kA X 0.2 + kA- X 0.1 = 2.4 X Jo-3 (2)
.. ~A-= 2 x 10-2 sec-I. (gm/1)-I
kHA =2 x 10-3 sec-1. (gm/1)-I.
3. The hydrolysis of a substance is specifically catalysed by hydrogen ions, the rate
constants being given by k = 4.7 x 10-2 C
H30
+sec-I. When the substance was dissolved
in I o-3 (M) solution of an acid HA, the rate constant was found to be 3.2 x 1o-5 sec-I.
Calculate the dissociation constant of the acid HA.
Soln. : For the acid HA,
HA + H 20 ~ H 30+ + A-
(1 - a.)C a.C a.C
where a. = degree of dissociation, C = concentration in molarity.

2
. . Ka = a. C , if I >> a., a. = ~.
1-a. fc
.. C + =• ~K0 .C.
H30
Thus, by the problem,
3.2 x 10-s = 4.7 x 10 -2 ..[K;C
= 4.7 x J0- 2 ~Kaxl0-3 •
.. Ka x 10 -3 = 4.6355 x 10-7
or, Ka = 4.6355 x 10-4 "'4.6 X to-4.
ADSORPTION, CATALYSIS, COLLOIDS Sl3

4. H 30+ acts as a homogenous catalyst in the reaction of inversion of sucrose. At


pH =3 the reaction in aqueous solution proceeds with constant half-life of SO minutes .
. What value of half-life would you expect at pH 4? = [C.U. 1997]
Soln. : The first order rate constant J = kH+.CH+,
where kH+ = catalytic coefficient of H+ ion
CH+ = concentraton of H+ ion.

Now, J = 0.693
t112

In the first case, J .= 0.693 at pH =3 and CH+ = 10 -3.


so
kH+ = 0.693 = 13.86.
sox 10-3
In the second case, when pH = 4, CH+ = l 0 -4 .

So, in the second case, J = 0.693 x 10-4 = 1.386 x JQ-3.


sox 10-3
:. J = 0.693.
soo
But J ~ 0.693
t112
0.693 0.693
or, t112 = -J- = _l._3_8_6_x_10 3 = 500
___ min.

5. The hydrolysis of a substance is catalysed by H 3o+ and the rate is given by


r = S.58x 10 - 2cH 30+sec-1 .
When the substance is dissolved in 10-2 (M) solution of an acid HA, the rate constant
was found to be 2.4 x 10-5 . Calculate the dissociation constant of acid.
Solo. : Considering the acid HA, the dissociation equation is
HA~ H+ +A-
.. (l - cx)C a.C cxC.
Cl = degree of dissociation, C = molar concentration.
2
cx C
.. K
a
= --
l-cx
.;, cx2c ( ·: l >> ex)

.. Cl= ft.
c
H30+
= cx.C = "'1Ka.C.
.. r=S.S8x 10-2 ~Ka.C.
514 PROBLEMS ON PHYSICAL CHEMISTRY

When C = I 0 - 2 (M), the rate constant is 2.4 x I 0 - 5 .

Hence, 2.4 x IO - 5 = 5.58 x · 10-2 {io-2 -.;;_f<a-


2
or, (2.4x10-5 )2= (5.58x10- 2 ) x I0-2 x Ka.

.. Ka= 1.85 x io-5•


6. The first order constant for a catalyst by a weak acid HA is 8.4 x IO - 5 sec- 1.
For this reaction kH+ = 1.4 x I 0 - 2 sec- 1 . Calculate the acidity constant K0 by using
I
J = kH+. (Ku)2.
Soln. : Given, J = first order rate constant = 8.4 x IO -5 sec-1.
kH+ = 1.4 x IO - 2 secl.

Now, 8.4 x 10-5 = 1.4 x I0-2 x (Ka)"i


2
or, (8.4x 10-5 ) =(1.4x 10-2 )2.Ka

7. A reaction is catalysed by acetic acid, acetate ions and H3o+. The rate constants
of the reaction at 25 °C in two solutions, one containing 0.2 mole of acetic acid and acetate
and another containing 0.2 mole of acetic acid and 0.1 mole of acetate, are 4.4 x I0-3
sec- 1 and 2.4 x 10-- 3 sec- 1 respectively. Find the catalytic coefficients of acetic acid and
acetate at the same temperature. [Assume H 3o+ concentration to be low enough to be
ignored]. . [V.U. 1994]
Soln. : Given,
Cone. of CH 3COOH Cone. of CH3COO - Rate constant
Solution I 0.2(M) 0.2(M) 4.4 x I0-3 sec I
Solution II 0.2(M) O.l(M) 2.4 x 10-3 sec- 1

We know, J = kH~CHA + kA- CA- (neglecting CH 30 +)

Solution I : 4.4 X 10-3 = kHA 0.2 + 0.2 kA- (I)


Solution II : 2.4'x 10-3 = 0.2 kHA + 0.1 kA - (2)
Solving equations (I) and (2), we get
kA- = 0.02
kHA = 0.002.
8. An enzyme following Michaelis-Menton kinetics was found to have highest activity
at 37 °C and pH 7.0. If maximum velocity V max for the enzyme was 2.4 x 10--4
mol. J- 1.s- 1 with an initial ezyme concentration.IE]= 2.4 nmol, calculate turnover frequency.
[JAM 2007]

2.4x 10-4
Solo. : Turnover frequency = . xI _
24 0 9
= HP moles/sec.
CHAPTER15

QUANTUM MECHANICS

Required Formulae
1. Black body radiation
Stefan-Boltzmann law :
RT = crT 4 ,
where RT = total energy radiated per unit time per unit area of black body
T = absolute temperature
cr = Stefan-Boltzmann constant = 5.67 x 10-8 W/m 2K4.
If v max is the frequency at which Ry (v) is maximum 'at temperature T, then according
to Wien's displacement law,
Vmax oc T.
2. Photoelectric effect :

lmv 2 = hv - W = hv - hv 0 ,
2
where m is the mass of election, v is the velocity of the ejected electron, v is the frequency
of radiation striking the metal surface, Wis the work-function of the metal, v0 is the threshold
or cut-off frequency and h is Planck constant.

eV0 = 2I mv2 ,
where V0 is the stopping potential, e is the charge on the electron and m and v have their
usual meanings.

3. If a particle of mass m moves with velocity v, its de Brogle wavelength /.., = !!_ = !:_,
mv p
where p is the momentum of the particle.
4. If an electron of charge e and mass m is accelerated through a potential difference

of V. then its kinetic energy -imv2 = eV, where v is the velocity of electron.
2
P = eV or, p = .,/2me V .
2m
515
516 PROBLEMS ON PHYSICAL CHEMISTRY

h
Hence, associated wavelength A = -J 2 me v .
5. Heisenberg uncertainty principle :
If /J. x is the uncertainty in the position x, /J.px is the uncertainty in the momentum pX'
ti h
/J. x !J.px ~ 2' where 1i = 27t.
• If !J.E is the uncertainty in the .. energy E and fit is the uncertainty in the time t,

!J.E!J.t~f!.
2
6. Probability of finding a particle described by wave function \jf between a and b,
b

P = J'l'\vdx.
a

7. If N is a .normalization constant and N'lf' = 'If, then


+~

or, J
~ 2 'I'• \jltix = 1.

8. Particle in a box :
2 2
Energy of the particle in one dimensional box, £ 11 =
(n = l, 2, 3, ... , etc.),
n h
. 8ma 2
where £ 11 is the energy of the particle of mass m at nth level and a is the length of the
box.
(n+l) 2h2 n 2h 2 (2n+l)h 2
Excitation energy, /J.E = En+I - E -
n - sma~
, - --2 -
8ma - o
8ma~

Energy at the particle of mass m in a cube of volume a3 ,

(n; +n~ +n?)h 2 n2h2


En = =
8ma 2 8ma 2
nx = ], 2, 3, ... , etc.

ny = ], 2, 3, ... , etc.
nz = l, 2, 3, ... , etc.
Wave function of a particle in one dimensional box of length a at nth energy state,

11r = (2 sin mu .
'I'll I/a a
Wave function associated with a particle in a cube of volume a3 ,

( 12) sm-a-sm-a-sm--"tl.
3
. nx1tX . ny1tY . n"1tZ
'l'n = I/a
9. For a particle of mass m possessing energy E tunnelling through a potential barrier
of width a and height V with E < V,
T = transmission coefficient

= 2
2 I a ~ . ,
cosh ~a+- ( ---. ) smh~ ~
4 R n
QUANTUM MECHANICS 517

where ~ = }_~2m(V-E)
h .

t
10. Energy of a harmonic oscillator at nth state, En = ( n + }v, n = 0, 1, 2, ... etc.,

where v is the frequency of oscillation. v = 2~ {f, where k is the force constant and
µ is reduced mass.
+~

11. If \jl 1 and \jl2 are orthogonal, J'1':'1' 2dx = 0.

Short Questions

1. Photoelectric work function and ionization potential of a metal are not the same-
comment. [B.U.(H) 1999]
Ans. : Work function, a characteristic energy of a metal, is defined as the minimum
energy an electron must possess to leave the metal surface overcoming the attractive forces
that normally bind the electron to the metal.
On the other hand, ionization potential is the energy required to remove an electron
from an isolated gaseous atom.
Hence, it is evident from the definitions that photoelectric work function and ionization
potential of a metal cannot be the same.
2. In a photoelectric experiment when the emitter metal is irradiated with light of
two different frequencies the following observations were made :
(i) electrons could not be emitted by using a very intense beam of light having
wavelength A.1.
(ii) electrons could be easi.ly emitted using a weakly intense beam of weavelength
"-z.
How do you explain the phenomena? Which wavelength is shorter. [B.U.(H) 1996]
Ans. : Light of frequency v consists of particles, called photons, each with energy
E of magnitude hv, where h is Planck constant. When a single photon strikes the metal
surface, its entire energy is absorbed by an electron. Now, to escape the metal surface
overcoming the attractive forces electron requires a minimum energy. So, if the impinging
frequency is not sufficient, ejection of electron would never occur, no matter how intense
the light is.
518 PROBLEMS ON PHYSICAL CHEMISTRY

Energy of Che photon constituting light is hv or h f, where A. is the wavelength of


light. So, lower the value of A., the greater will be the- energy of photon. Now, the weakly
intense light of wavelength A. 2 has more energy than the other with wavelength A.1. Thus,
the weakly intense light has shorter wavelength.
3. Write notes on :
(i) The Stefan-Boltzmann law.
(ii) The Wien's displacement law indicating the quantities involved. [C.U.(H) 2002]
Ans. : (i) Stefan-Boltzmann law states that the total energy radiated (Rr) per unit
area per unit time from the surface of a black body at a particular temperature T increases
with the increasing temperature.
Rr = cr T4
cr = Stefan-Boltzmann constant.
(ii) The frequency at which maximum radiation per unit time per unit area of the
surface of the black body occurs at particular temperature T shifts towards higher frequency
as T increases.
vmax oc: T
vmax = frequency at which radiation power is most intense.
4. Make use of de Broglie relationship to cak:ulate the total energy of H atom in
the Bohr model, where radius is &ssum~d to be equal to a 0 . [B.U.(H) 1996]
Ans. : De Broglie relation relates w::.velength (A) of matter wave associated with the
motion of a material particle to its momentum p.

According to de Broglie relationship, A. = !!.p . where h = Planck constant.

In the Bohr model, electron of charge -e and mass m is moving with velocity v around
nucleus of charge +e in a circular orbit of radius a 0 . Circumference of allowed orbits must
accomodate integral multiple of de Broglie wavelength associated with the moving electron.
Hence. nA. = 27t a 0 , n = I, 2, 3, .. ., etc.

or, A. = 2rtao (!)


n
Now, p = h

nh =fl!!_, where, h
Thus, p
2rtao
= Go
ti== 27t. (2)
According to Bohr, there exists a mechanical equilibrium between centrifugal force
and centripetal force in the orbit.
2
Centrifugal force = mv
Centrifugal force = Coulomb force between electron and nucleus

= e2
;;0
QUANTUM MECHANICS 519

At equilibrium,

(3)

Again, p = mv = n..Jj_, (4)


ao

e2 -- n2ti2 n2*-2
Thus, ma a2m2 , :. ao = _1rr_. (5)
O o Cm
Total energy of an electron in hydrogen atom = kinetic energy + potential energy

= p2 e2
2m -a;;
2 2 2
n 1i e me 4
= 2
a m - ao =- n 21i 2 [putting the value of a 0 from equation (5))
6 2
5. How does the de Broglie hypothesis lead to one important postulate made by Bohr
in his theory of hydrogen atom? [B.UAJ) 1997]
Ans. : De Broglie hypothesis states that every moving particle has wave associated
with it that governs its motion.

Thus, according to de Broglie hypothesis, A = ~,


where A is the de Broglie wavelength and pis the momentum of the particle. Now, according
to Bohr model of hydrogen atom, an electron of mass m moves in a circular orbit of radius
r ·with velocity v.
Electron has a wave associated with it. Now, the circumference of a stable orbit must
contain integral number of the wavelength A.
Hence, n'A = 27tr, n = I, 2, 3, ... , etc.

A. = 27tr.
n
(I)

h
Again, by de Broglie relation, p =X
or, p = ;;, [putting the value of A from (!)]
mvr = nn, (2)
h
where Ii = 27t , p = .mv.
Equation (2) predicts that angular momentum (mvr) of electron is allowed to have
only certain discrete values and this is the important postulate made by Bohr in his theory
of hydrogen atom regarding the quantization of angular momentum.
6. Write action integrals for radial and angular motions in atom. [B.U.(H) 2001]

Ans. : f Ld¢> = n~h, where L = angular momentum, ¢> = polar coordinate.

n~ = I. 2, 3, ... , etc.

f P,dr = n,h,
520 PROBLEMS ON PHYSICAL CHEMISTRY

where p, = radial momentum, r = polar coordinate, h = Planck constant.


n, = 0, 1, 2, ... , etc.
7. Write down the dimensions (in M, L, T) for force, angular momentum and action.
[B.U.(H) 1996]
Ans. : Dimension of force = MLT-2 ( ·: force = mass x acceleration)
Dimension of action = ML2T- 1 ( ·: action = energy x time).
Dimension of angular momentum = ML2T-1 ( ·: angular momentum = mass x velocity
x distance).
8. State Wilson-Sommerfeld quantization rule.
Ans. : Every physical system in which coordinates are periodic functions of time obeys

a quantum condition. This condition is f Pqdq = nh (n = 0, 1, 2,' ... ), where Pq is the


momentum associated with the coordinate q, n is a quantum number that takes on only

integral values, and f means that the integration extends over one period of q.

The above relation is known as Wilson-Sommerfeld quantization rule.


9. Explain Bohr correspondence principle, taking the case of H atom.
[B.U.(H) 2002]
Ans. : Correspondence principle states that quantum theory approaches classical theory
asymptotically in the limit of large quantum number.
In the Bohr model of hydrogen atom, an electron of mass m and charge -e is rotating
around nucleus of charge +e in a circular orbit of radius r.
According to Bohr theory of H atom for a stationary orbit,
mv 2
(I)
r
mv 2
where - - = centrifugal force
r
e2
72 = centripetal force
and mvr = ntt (n = 1, 2, ... ) (2)
where mvr = angular momentum
n = quantum number
n = 2~ , h = Planck constant.
e2
From (l), we get r = mv 2 • (3)

Putting the value of r in equation (2), we get


e2
v = nfz · (4)
2 2
Hence, r = n 1i (5)
me 2 .
QUANTUM MECHANICS 521

Now, if v be the frequency of radiation that an elec~ron would emit when it jumps
.from (n + I )th orbit to nth orbit, then
hv = En+I - En

= me 4 [~ _ _ _l _ 2 ]
2J1 n . (n+I)
[since energy of Bohr orbit = E,, =- f =-
r
mi\ ]
· 2n 1i
4 2 2
or, v = I me [{n+l) -n ]
27t"2h 3 n 2 (n+I) 2

or, v = me4 (2n+I)


227t1i 3n4 .

:. v =~3
[since n >> I, n+I = n, 2n+I = 2n]. (6)
27t1i nJ
In classical physics, frequency (v c1) of radiation radiated by an rotating electron,
v
Yc1 = 21tr
2
e me 2
= - I -. [putting the values from eqns. (4) and (5)]
27t nh 2
n h2 ·
me 4
= 2
7tn 3 h 3 = v [from eqn. (6)].

10. Apply Wilson-Sommerfeld quantization rule to obtain an energy expression for


a particle in one dimensional box.
Ans. : The particle is moving back and forth within a one-dimensional box of dimension
a. So, the distance covered by the particle in one period is 2a.
Now, by Wilson-Sommerfeld quantization rule,

f Pxdx =nh, where Px is linear mo~entum along x direction, n is the quantum number
that takes on only integral value, h is Planck constant, and f means that the integration
is taken over one period of motion.

Thus, fPxdx = Pxf dx (since Px is constant as no force is acting on the particle)

= p. 2a = nh.
:. p = nh.
2a
2 2
Energy of the particle = E =b
L.m
= n h
8ma2
, where m = mass of the particle.

11. Prove that .!!-__ is not Hermitian in nature. [V.U.(H) 1997)


dx
Soln. : An operator A is said Hermitian if it obeys the following relation
522 PROBLEMS ON PHYSICAL CHEMISTRY

+~ •
Now,
f \jf 2 d\jl I dx
dx

-- - +J~'1'2 dxd '1'1dx


* (since \j/ 1
and \j/ 2
become zero at infinity).

Hence, !!_ is not Hermitian, rather it is an anti-Hermitian operator.


dx
12. Prove that Hermitian operator gives us real eigenvalues [V.U.(H) 1997]

Solo. : Let A be a Hermitian operator and \jl is its eigenfunction with eigenvalue
a, such that,

Al'V) = ai'I') ( 1)

(Ai'V)f = (A'VI = a·('l'I· (2)

Since A is Hermitian, a* is complex conjugate of a.

Now, ( \j/IAl\j/) = a (\jl I'I') (3)

Again, ( \j/IAl\j/) = (A \jl I\jl) (applying turn-over rule)

= a* ('l'l'I')· (4)
Now, subtracting (4) from (3), we get
(a - a*) ('l'l'I') =0
or, a - a* =0 (since, ( \jf I'I') -;C 0)
a= a*.
Thus, a is real.
13. Explain the terms : linear operator, eigenvalue and stationary state.
[V.U.(H) 2001]

Solo. : Linear operator : An operator A is called linear if it possesses the following


properties :
(i) for any arbitrary functions ftx) and g(x),
-
A (fix) + g(x))
-
= Aftx) -
+ A g(x) and
- - d
(ii) Ac ftx) = c Afix), where c is an arbitrary constant. e.g., -
dx
Eigenvalue : If the action of an operator on a function gives back the same function
multiplied by a constant, then the constant is called eignevalue. e.g., H'lf = E\jf.
r.: is the eigenvalue of the opt>rator H.
QUANTUM MECHANICS 523
Stationary state : A state which does not evolve with time is called stationary state.
For stationary state, the probability density 'l'*'I'· where 'I' represents the state, does not
change with time.
14. Does the function e-iErlh\jl(x) represent a stationary state?
Soln. : Given, 'lf(x, t) = \jl(x) e-iEr I h.

'l'*(x, t) 'I' (x, t) ="l'*(x) e-iEr 1h\jl(x) e-1£11 h


= 'lf*(x) \jl(x).
Thus, the probability density remains constant in time. Hence, the given function
represents a stationary state.
15. Show that if two operators commute, they have simultaneous non-degenerate
eigenfunctions. [V.U.(H) 1994]
Soln. : Let I'I') '
be an eigenfunction of the operator A with an eigenvalue a.
Then Al'I') = al'lf). (I)
"
Let B be another operator which commutes with A .

Hence, [.4,8]1'1'} = 0
or, ABl'I') - BAl'lf) = 0.

:. A(81'1')) = 8(Al'lf)). (2)

But, BAl'I') = aBl'I') [from equation (l)] (3)


Comparing equations (2) and (3), we get

(4)

Thus, Bl 'I') A with the eigenvalue a.


is an eigenfunction of

But, I'I') is. the only eigenfunction of A with the eigenvalue a, since I'I') 1s non-
degenerate eigenfunction of A .

Hence, Bl'lf) cannot be a new eigenfunction of A with the same value a. It must
be the function I'I') with some constant.

' "
Thus, A and B have same set of non-degenerate eigenfunctions.
16. Prove that if two eigenfunctions of a Hermitian operator have different eigenvalues,
then the wave functions are orthogonal. [V.U.(H) 2000]
I I
Soln. : Let 'I' 1 ) and '1'2) be two eigenfunctions of a Hermitian operator A with
eigenvalues al and az
respectively. .
Thus, Al"'') = ad"'') ( 1)

Al"' 2) = a2 I'I' 2) (2)


524 PROBLEMS ON PHYSICAL CHEMISTRY

Hence, ( \jl ij.41'1' 2) = a2 ('I' d'I' 2) [by equaion (2)]. (3)

Again, ('I' ij.41'1' 2 ) = (A 'I' ij 'I' 2 ) [by tum-over rule]

= a 1 ('1' 11'1' 2 ) [from equation (l)] (4)


Now, subtracting equation (3) from equation (4), we get

a1 ('l'1 l'l'2) - a2('1'1 l'l'2) = 0


or, (a 1 - a 2) ('l'i 1'1' 2) = 0.

Since, a1 ;Ca2,('l'i l'1' 2) = 0, i.e., J\jl;\jl 2d't = 0


Hence, I'I' 1) andI'I' 2) are orthogonal.
17. If two operators a and p are Hermitian, then find out the condition when up
will be Hermitian. [V.U.(H) 1995]
Solo. : Since a and p are Hermitian,

('I' dal'I' 2) = (U\j/ d'I' 2) (1)

("' 1IPI"'2) = (P'I' d"' 2). (2)

Now, ('I' daPJ'I' 2) = (U\j/ i!Pl'I' 2 ) [by tum-over rule]

= (Pa'I' d'I' 2) [by tum-over rule].


If up =.pa, i.e., [a, Pl = 0, then

('I' daPl'I' 2) = (uP'I' 1l'I' 2) ·


Thus, if a and P commutes, up will be Hermitian.
18. Show that momentum operator is Hermitian.

Solo. : Momentum operator = Px


A

= - in ax .
a

Since, \j/ 1 and \j/2 vanish at infinity,

in f \jl 2 a\j/ I dx
+- •

ax

= L'l'z(-in :J
+- •

'l';dx [ ·: (-if= i ].
QUANTUM MECHANICS 525

19. If \j/1 and \j/2_are eigenfunctions of linear operator A with the same eigenvalues
a, show that any linear combination of \j/1 and \j/2 will also be an eigenfunction of A
with the same eigenvalue. [C.U.(H) 2000]
.
Solo. : Given, = a\j/1
A \j/ 1
A'1'2 = a\jf2 ·
Let \jf = C1\j/1 + C2\jf2, where C1 and C2 are combining coefficients.
Hence,
.
= A (C 1'V 1 + C2W2 ) = A C1'V 1 + AC W l ·:A
2 2 is linear operator]

= cl A'1'1 + C2A '1'2 [·:A is linear operator]


= aC1'1'1 + aC2'1'2 = a (C1'1'1 + C2'1'2) = a\jf.

20. Under what condition is . B')2


(A+
Solo. : (A + B)2 = (.4 + B)(A + B)
= A2 + AB + BA + B2
= A2 +AB+AB+B2 [if AB= BA, i.e., [A,B] =OJ

= A2 + 2AB + B2 .
Hence, if A and B commute,

(A+ Bf= A2 +2AB+ B2 .


21. Evaluate [;n, Px]. [C.U.(H) 2005]

Solo.: [xn,flx]'I' = XnPx'l'-PxXn\jf


.,,, n O'lf .,,, a ( n ) [ • .,,, a
= -lriX :;--- + lri :;---- X 'If ·: Px = -lri OX]
ux ux

'n •
[x •Px
l = 'fi nx l
11-I
.

22. Given an arbitrary operator A, show that AtA is always Hermitian.

Solo. : ('lfijAtAj\j/ 2) = (A'1' 1IAl'1' 2) [by tum-over rule]

= (At A 'If i l'I' 2) [by tum-over rule]

Again, ('I' ijA Al'lf = ((At Af 'I' ii 'I'


t 2) 2)
526 PROBLEMS ON PHYSICAL CHEMISTRY

( t )t = A t A.
Thus, A A
Hence, AtA is always Hermitian.
" "' ) ,.. " ,.. "'
23. If two operators A and B are Hermitian, prove that AB+ BA is always Hermitian.

Solo. :

= (.8tA\vii'1' 2 ) + (AtBt'Vil'1' 2 ) [by turn-over rule]

= (BA'Vd'V 2 ) + (AB'V 1l'I' 2 ) [ ·: A and B are Hermitian]

= ((BA+AB)'Vil'V 2 )
= ((AB+ BA )'1' ii '1'·2 ) ·
Again, ( \jl ijAB + BA'"' 2) = ((AB+ BA r ii 2)
\jl \jl [by turn-over rule].

Hence, (A.8+.BAf = AB+BA.


Thus, AB+ BA is always Hermitian.
24. Determine, citing reasons, whether each of the following functions is acceptable
or not as a wave function over the indicated intervals :
(i) e-x (0, oo); (ii) e-x (- oo, oo); (iii) sin-Ix (-1, Ij; (iv) e -I xi (- oo, oo)
[C.U.(H) 2002]
Ans. : (i) e-x is acceptable as it is finite (i.e., vanishes at infinity), continuous, single-

valued and its first derivative or slope ( d~x ) is also continuous within

the indicated interval.


(ii) e-x is not acceptable within the given interval as it diverges as x tends to
- oo.

(iii) sin-1 x is not acceptable, since it is a multivalued function.

(iv) e-1 xi is not acceptable as its first derivative ( d::xl) is not continuous

at x = 0.
2
25. Test whether the following functions are eigenfunctions of the operator ( d )
dx 2 .

Write also the corresponding eigenvalues :


(i) sin x ; (ii) sin x cos x ; (iii) A sin m.x + B cos nx. [B.U.(H) 2004]

. d2 . d . . I I
I : (1) --, sm x = -cos x
Son. -~m x, •genva ue = - .
dx- dx
QUANTUM MECHANICS 527

(ii) ~sin x cos x =l ~sin 2x = !!_cos 2x = -2 sin 2x, eigenvalue= -2.


dx2 - 2 dx.2 dx

(iii) ~(A sin m.x + B cos nx) = .!!._ (m A


cos m.x - n B sin nx)
dx2 dx
= -m2A sin m.x - n2B cos nx.
.
(A sm . . f . f d2
m.x + B cos nx) 1s not e1gen unction o - , .
dx"
26. An operator ( A) is defined as : A=
x l_ . Find A2 . [B.U.(H) 2003]
dx
Soln. : Given, A = x l_. Let 'I' be any function.
OX
2
Thus, A \jf = A.A'lf = x.l_ (x.l_)'I'
OX ox

= (xl_+x
ox
2
L)'I'·
ox 2
=x 0 + x2 02

r.
ox ox 2 .

27. Expand the operator ( !+ x [B.U.(H) 2001]

Ans. : Let 'I' be any function.

2
= ( -,,
d +x-+1+x
d
dx"
2
J'V· dx

= -dxd22 +x-+x
d
dx
2
+ 1.

28. Find the dimension of h : . [B.U.(H) 2003]


0

Ans. : h ! has a dimension of momentum, i.e., MLT-1.

29. Show that H is Hermitian. [V.U.(H) 1996]


h2 d2
Soln. : H= --- +V(x)
2m dx 2
2
Since n and V are real numbers, they are automatically Hermitian.
2m

Now,

+= d
= J'I': da.
dx
dx [putting _h = a]
dx
528 PROBLEMS ON PHYSICAL CHEMISTRY

• )+-
= ('i'iCX -- -
f
+- •
d\jf I wJx
dx

* ,. d
= - Jd\jf 1 a.dx
+oo
[since \jf1 and 'I' 2 a vanish at infinity]
dx d.x
+- •
= - J d'lf1 d'lf2 d.x[since a= d'lf2 l
dx dx d.x

=- [( '1'2 .)+- +-L


d\jf I
dx -- -
d 2\jf •I
d.x2 \jfzdx
l
=
+oo
f 2
\jf d \jf I
2~·
*
,,_

d2 A

Hence, d.x 2 is Hermitian. Thus, H is Hermitian.

30. What is meant by precise value in quantum mechanics?


Ans. : If a wave function associated with a system is eigenfunction of an operator
which corresponds to a dynamical property in classical mechanics, then the wave function
will always give some value of the property whenever is acted on by the corresponding
operator. So there cannot remain any uncertainty in value and thus, it is actually a precise
value. Free particle is associated with the wave function eikx which is eigenfunction of

the operator (-iii).!!:._ representing momentum of the particle. Thus,


dx

-iii .!!:._ (eikx) = lik (eikx).


dx
So the free particle has precise value of momentum and the value is lik.
31. Calculate degeneracy of the level having an energy of 14 (h 2/8ma 2 ) for a particle
of mass m confined in a cubic box of dimension a. [V.U.(H) 2002]
Solo. : Energy of the particle in a cubic box.
En = Enx + Eny +Enz
( n2
x +n2y +n2)h2
z
= 8ma 2
= (I)

where m = mass of the particle, a = edge of the cube.


Given, n 2 = 14
Or • n x2 +n 2y +n 2= 14 t .

Now 12 + 22 + 32 = 14. (2)


Numbers on the L.H.S. of the equation (2) can be arranged in the following 6 different
ways among themselves, i.e ..
QUANTUM ME€HANICS 529

nx ny n,
1. 2 3
1 3 2
2 1 3
2 3 1
3 2
3 2
So, the given energy level is six-fold degenerate.
32. Can the zero point energy of a particle in a box be zero? Answer with reasons.
[V.U.(H) 1995]
or,
Why is the value of n =0 of the quantum number not permitted for a particle in
a one-dimensional box. [C.U.(H) 2001]
Ans. : Energy of a particle of mass m confined in a one-dimensional box of length
2
= fm = 8ma
2
a is E n h:, where pis the momentum of the particle. When n = 0, E Is zero.

It means that momentum of the particle is also zero. Hence, there is no uncertainty in
its momentum, that is, tlp = 0. Now, to maintain Heisenberg uncertainty principle which

states ax tlp ~ !! , the position of the particle has to be absolutely uncertain. But the particle
2
is confined within a box of finite length a. So, the uncertainty in the position (ax) of
the particle cannot exceed the length of the box. Thus, the zero point energy of the particle
in a box cannot be zero, since otherwise it would violate uncertainty principle.
33. For a particle in a box problem, show that the energy levels are continuous, if
either the mass of the particle or the length of the box becomes large. [V.U.(H) 2001]
Soln. : The energy of the nth energy level of the particle confined in a box
n2h2
= E" = 8mt2'
where h = Planck constant, m = mass of the particle, l = length of the box.
The energy difference between two successive energy levels = En+l - En

(n+ l)h 2 n 2h2 (2n + l}h 2


= 8ml 2 8m/ 2 = 8mt 2
Separation between two successive energy levels decreases if either the mass of the
particle or the length of the box becomes large.
34. Show that the error in the de Broglie wavelength (A.) is related to the error in

velocity (v) by the relation dA. = - !:_ dv. [B.U.(H) 2003]


v
Soln. : According to de Broglie formula, A. = p'
h (I)

.where A. is the wavelength of the particle with momentum p.


h
Now, from (I), we get A. = mv [·: p = mv]
Prob. Phy. Chem.-34
530 PROBLEMS ON PHYSICAL CHEMISTRY

where m = mass of the particle, v = velocity of the particle.


-
:. d'A =- h dv = - h dv
mv 2 pv

=- ~ dv [from equation (I)].


v
35. For a particle in a cubical box write down the energy values for the condition:
nx + ny + nz =
4 and indicate the level of degeneracy, if any. [C.U.(H) 2000]

Solo. : nx + ny + nz = 4
Now, I + l + 2 = 4. (l)

Thus energy of the particle in a cubical box, E = En x +En y +En z

= h2
8ma2
(n2 +n2 +n2)
x Y z
= _tL
8ma2
(12+12 +22) =

where m = mass of the particle, a = edge of the cube.


Again, nx ny n,
l l 2
2 l
2
2
Each of the combinations mentioned above corresponds to the same energy 6h
8ma 2 ·
Hence, the level is three-fold degenerate.

. (x2 -a2)
36. Show that the function 'lf(x) = (x-a) is not acceptable for describ~ng the state
of a particle in a one-dimensional box ranging from x =-2a to x = 2a. [B.U.(H) 1999]

. (x2-a2)
Solo. : Given, 'lf(X) = (x-a) .
At x = -2a.
_ 4a 2 -a 2
'lf(X) -
-2a-a
= -a.
At x = 2a,
2
-a 2
'lf(X) = 4a2a-a = 3a.
So, 'lf(x) does not vanish at. the boundary. Hence, the given 'lf(X) is not acceptable
for describing the particle confined in a one-dimensional box ranging from -2a to 2a.
37. From the definition of linear harmonic oscillator, obtain the expression for the
potential energy and hence, construct the Hamiltonian operator for the same.
[C.U.(H) 2001]
Solo. : A linear restoring force (F) acts on particle executing simple harmonic
oscillation which is prope>rtional to its displacement, x, from the centre, namely
F = -kx, (l)
where k = force constant.
QUANTUM MECHANICS 531

Now, F(x) = - av(x), dV(x) = -F(x) dx (2)


ax
or, dV (x) = kx dx [from equation (l)]. (3)
Integrating equation (3), we get

V(x) = i1cx2 , [since at x = 0, ·V(x) = OJ (4)


V(x) = potential energy of the particle.
[ p = momentum of the particle, '!1 = mass of the particle.]
·2
Hamiltonian operator for a linear harmonic oscillator, H = fm +-!- kX 2 .
Now, p
A

= -In ax'
• a

1i a 2 2
1 2
Thus, H = - 2m ax2 + 2kx
38. Obtain the energy expression for a particle moving to and fro within a one-
dimensional box of leQgth a using de Broglie equation.
Solo. : The energy of a particle moving to and fro within a box of length a,
p2
(I)
E = 2m'
where p is momentum of the particle and m is its mass.
By de Broglie hypothesis every moving object has wave associated with its motion, i.e.,
h
p = x-· (2)

where /... is the wavelength of the wave associated with the ·particle having momentum
p. Again, in one traversal of the region, the particle covers a distance of 2a.
An integral multiple of the matter wave must fit in the distance covered by the particle
in one back and forth motion.
Hence, 2a = n/....
.. ').., = 2a. (3)
n
p2 h2
Thus, E = 2m - 2m/...2 [from equation (2)]

=h
2h2
[from equation (3)].
8ma
39. What is the ·ground state energy of a 3-dimensional harmonic oscillator?
Solo. : Energy of the 3-dimensional harmonic oscillator,
532 PROBLEMS ON PHYSICAL CHEMISTRY

Thus, ground state energy = E0 = lhv [since n = O].


2
40. What is the degeneracy of a level having energy lhv for a 3-dimensional harmonic
2
oscillator? ·
Soln. : Energy of the nth level of a 3-dimensional harmonic oscillator,

En = ( nx +ny +n, +i)hv = (n+i)hv.

3
Now, n+2 - 7
- 2
n = 2.
Thus, nx + ny + n, =2
nx ny
1 0
0
0 l'
2 0 0
0 0 2
0 2 0
Thus, the given energy level is six-fold degenerate.
41. Comment on zero-point energy of a one-dimensional harmonic oscillator.
Ans. : Energy of a linear harmonic oscillator,
2
E = J!.L..
2m 2
+ _!_ kx2
'
where Px is momentum of the oscillator and x is displacement from the origin. If E is
zero, Px and x are also zero. It means that there are no uncertainties in both position (.1x) ·
and momentum (!!.px) and both can be measured simultaneously with an arbitrary precision,

i.e., L\x !).px = 0. But uncertainty principle requires L\x !!.px ~ !!.. . So, if L\x is zero, !!.px
2
has to be infinite and vice versa. Both '!!.px and L\x cannot be zero simultaneously. Thus,
zero point energy of the oscillator cannot be zero.
42. Show that En of hydrogen atom is n2-fold degenerate. [V.U.(H) 1995]
4
Soln. : Energy of a hydrogen atom is given by En = - mf 2 ,
2n 1i
where n is the principal quantum number, m is the mass of an electron and e is its charge.
The corresponding eigenfunction of energy depends on n, I (azimuthal quantum number],
and m magnetic quantum number and is represented as 'Vn/m·
Now, for every value of n there are n number of I with minimum value 0 and maximum
value (n - 1), and again, for every value of I there are (2/ + l) number of m ranging.
from -/ to + I. So, situations could occur when different states corresponding to different
I and m but same n (e.g., \jl ntm, \jl nl'm') belong to th.e same energy state which depends
only on n, e.g., for a given n, total number of l is
(0, 1, 2, ... , (n - l) =) n
and total number of m is
QUANTUM MECHANICS 533
l + 3 + 5 + ... + (2n - 1) = n 2 (since, for each I, number of m = 21 + l)
and hence, £" of a hydrogen atom is n2.... fold degenerate.

43. What is the degree of degeneracy of Y1m with respect to the eigenvalues of iJ-?
Solo. : L2 Y1m = I (I + 1) 1i 2 Yim
So the eigenvalue depends only on I, not on m. But for each value of I, there are
21 + 1 number of m and the form of eigenfunction Y1m is governed by I and m. So, situations
are likely to occur when several states represented by different Yim (different in respect
to m, for example, Y1m , Yim'• Yim", etc.) correspond to same eigenvalue which is
independent of m.

, Hence, the degree of degeneracy of Y1m with respect to L2 is (21 + 1).


44. Show that eikx is an eigenfunction of the operator Px. What is the eigenvalue?

o o. : p' 'xfe'
SI a:
= 'lid
"""i"""" eikx = 1ikikx
-1 e .
ax
:. eigenvalue is 1ik.
45. Analyse whether the following functions are even or odd or neither :
(a) tan x, (b) ex, (c) 15, (d) x cos hx, (e) 2 - 2x, (f) (2 + x) (2 - x), (g)
2
(h) xe-x , (i) xe-x, U) x sin x, (k) eLt.
Solo. : (a) f (-x) = tan (-x) = -tan x = -f(x), odd
(b) f (-x) = e-x #ex
:. f(x) = eX ?f.f(-x)
'# -f (--x), neither even nor odd.
(c) -15 ;C 15, neither even nor odd. -.
1 1
(d) f(x) = x cosh x =-x (ex+ e-x) = --(-x) (e-x + &) = (-x) cosh(-x)
2 2
= -f (-x). odd.
(e) f(-x) = 2 - 2(-x) '#2 + 2x '#2 - 2x
:. f(x) = 2 - 2x ;Cj(-x)
?f.-f (-x), neither even nor odd.
(f) f (-x) = (2 - x) (2 + x) =f(x), even.
(g) f(-x) = {-x)2 e-<-x>2 = x 2e-x2 = f(x), even.
(h) f(-x) = (-x) e-<-x> 2 = -xe-x2= = -f(x), odd.
(i) f(-x) = (-x)e-(-x) = -xeX ;Cxe-x
:. f(x) = xe-x '#f(-x)
;C-f (-x), neither even nor odd.
U) f (-x) = -x sin(-x) = x sin x, even.
(k) eix = cos x
+ i sin x '# cos x - i sin x
;C - = cos x - i sin x),
(cos x - i sin x) (where e-ix
neither even nor odd.
534 PROBLEMS ON PHYSICAL CHEMISTRY

46. Classify these operators into linear or non-linear :

(a) x 2 -d22
dx
, (b) ( )2, (c) f dx, (d) exp, (e) In , (f) '
-y, (g) -d , (h) cos.
dx
x=I
Solo. : Let ftx) and g(x) be functions of x, and c be any constant.
d2 d2 d2
(a) x 2 - 2 [cf(x)] = cx2 - 2 ftx), and x2- [f(x) + g(x)]
dx dx dx 2
. d2 d2
= x2 -if (x) + x2 - 2 g(x), linear.
dx dx
(b) (j.(x) + g(x)) ;t. (f (x)) 2 + (g(x)) 2
2

(cf (x)) 2 ;t. elf (x)) 2 , non-linear.

(c) j(f(x)+g(x))dx = Jf(x)dx+ Jg(x)dx

Jcf(x )dx = J
c f ( x )dx, linear.

(d) exp[{ (x) + g(x)] ;r. exp <J (x)) + exp(g(x))


exp[cf(x)] -.r.c exp[f(x)], non linear.
n n
(e) Lcf(x) = c Lf(x)
x=I x=I
n n n
Lf(x) + g(x) =Lf(x) + Lg(x), linear.
.t=I x=I x=I

(f) ~f(x)+g(x) -.r. ~f(x) +~g(x)

~cf(x) -.r.c~f(x), non-linear.

(g) .!!_ is linear by the same argument as in (a).


dx
(h) cos (j(x) + g(x)) ;r. cosftx) + cos g(x)
cos (cftx)) ;r. c cos ftx), non-linear.

47. Given a linear operator A<P = a</J, find the value of eA </J.
• + A'2 + A'3
I : e A =I+ A 3! + ...
A

Son. 2!
I is identity operator, defined by l</J = <P for all </J.

Hence, eA<P
.
= ( 1 +A+~+~+
'2 '3 )
... <P
2! 3!

= <P + a</J + i.__ <P + ~ <P + ...


2! 3!
(since, A~ = a</J , A2<P = Aa<P = a A<P = a2<fJ, and so on)

= (1+a+~~ +~; + ... )<P = ea</J.


QUANTUM MECHANICS 535

48. Prove that i [a, Pl is ·Hermitian, if a and P are Hermitian.

Soln. : ('I' iii[ a, P]j'I' 2) = ('I' iii( ap - Pa )j'I' 2)


= ('I' 1liaPl'I' 2) - ('I' 1liPal'I' 2)
= (-iPa'I' ii 'I' 2 ) - (-iaP'I' ii 'I' 2 ) (since a and P are Hermitian)

= (-i(Pa - aP)'I' ii 'I' 2) = (i( ap- Pa )'I' ii 'I' 2) = (ila, Pl'I' ii 'I' 2) ·
49. What are the consequences of measurement when (a) two operators commute?
(b) two operators do not commute?
Ans. : When two operators commute, it means that the corresponding observables
can simultaneously be measured with an absolute precision, i.e., their simultaneous
measurements do not have any uncertainties.
When two operators do not commute, the corresponding observables cannot be
measured simultaneously and precisely; i.e., their simultaneous measurements are subject
to certain uncertainties.
SO. (a) Interpret (i) \jf, (ii) 'l'*'V dx.
(b) What is node?
Ans. : (a) (i) 'I' is a wave function that depends on coordinates and time, and contains
all the information about the dynamical properties of the system. It
is actually a probability amplitud~ and can be real or complex.
(ii) 'l'*'I'
is the probability of finding a particle between x and x + dx at
the PQSition x.
(b) Notle is a PQint after passing through which wave function (\jl) changes
sign. At node \jl is zero.
51. Test whether the following functions are acceptable or not within the indicated
range:
2
(i) e-x (- oo, oo); (ii) e-x cos x (0, oo); (iii) lco, oo); (iv) tan x (0, 7t); (v) sin x (0, 7t);
x
(vi) sinxco, oo); (vii) e-icl>(O, 2n;).
x
2
Ans. : (i) e-x is acceptable as it is c:;ontinuous, single-valued, finite, i.e., does not
diverge at any point between - 00 to + oo and its first derivative or slope is also continuous.
(ii) e-X COS X is acceptable due tO the Same reaSOnS aS in (i).

(ii.iY _!_ is not acceptable since it dlverges at 0.


x
(iv) tan .x is not acceptable, since it diverges at 7t

(v) sin x is acceptable due to the same reasons as in (i).

(vi) sin x is acceptable due to the same reasons as in (i). It is finite at x = 0.


x
(vii) eic!> is. acceptable since it is continuous, single-valued, and finite within the given
range of <)>.(eicl> = cos <)> + i sin <)>) ..
536 PROBLEMS ON PHYSICAL CHEMISTRY

52. Show which of the following functions are acceptable : (i) e-lxl (- oo, oo);
(ii) cos Ix I (-27t, 2rt); (iii) sin I x I (-27t, 2rt).
Soln. : (i) f (x) = e-1 x I
= e-x (when x ~ 0)
= eX (when x ~ 0)
f (x) is continuous, finite, single-valued.

Now,y' =
= eX (when x ~ 0).
Thus, e-1 x I is not acceptable within the given range as its derivative diverges at
x = 0.
Cii) t <x) = cos ~I,
= cosx (x ~ 0)
= cosx (x ~ 0)

y' = d( coslxl) = -sin x (x ~ 0)


dx
= -sinx (x ~ 0).
f (x) is continuous, finite, single-valued and its derivative is also continuous. Hence,
it is acceptable.
(iii) f (x) = sinIx I
= sinx (x ~ 0)
= -sin x (x ~ 0).
f (x) is continuous, finite, single-valued.
But y' = cos x (x ~ 0)
= -cos x (x ~ 0).
So, y' is not continuous at x = 0.
Hence, sin Ix I is not acceptable.
53. What is the meaning of nodes of wave functions? How do you explain the passage
of electron across the node? Do the 'signs' on the two sides of a node make any difference
in the electronic charge distribution?
Ans. : Answer to the first part of the question has been given in 50(b).
Electron does not have any well-defined trajectory as a consequence of uncertainty
principle. At node its uncertainty in momentum will be infinite by virtue of uncertainty
principle as its position is now precisely known. Uncertainty in momentum must be
comparable to its momentum in magnitude and it is found at the other side of the node.
Electronic charge distribution is equal to the product of the wavefunctions, viz, \jl*\jl.
So the different signs of wavefunction on the two sides of a node do not make any difference
in electronic charge distribution.
QUANTUM MECHANICS 537

54. Why are the wave functions associated witk different states of a particle in a box
orthogonal?
Ans. : Wave functions of a particle in a box are energy eigenfunctions and they are
non-degenerate. Non-degenerate eigenfunctions of a Hermitian operator are orthogonal.
Hence, wave functions of a particle in a box are orthogonal as Hamiltonian is a Hermitian
operator.
55. Functions 'l't and \j12 are individually normalised and are mutually orthogonal to
each other. Normalise 'I' = 'l'i +
3 11i \j12 . [1.1.T.B. M.Sc. Admission Test 2000]
Soln. : Let 'If' = N\jl.

By the problem, ('1''1'1'') =

or, N 2 ('l'l'I') =
or, N2('1'1 +"3'1'21'1'1 +"3'1'2) = 1

2
or, N [('I' ii 'I' 1) + -{3 ('I' 2I'1' 1) + -{3 ('I' ii 'I' 2) + 3( 'I' 2I'I' 2)] =l
or, N 2 [1+3] = 1 [Given, ('1'il'1' 2) = ('1' 21'1' 2 ) = 1 and ('l'i 1'1' 2 ) = ('1' 2 1'1' 2 ) = O]
or, N 2 = L :. N = .!..
4 2
56. The experimental results of the photoelectric effect data for metals Cs and Na are
given in the figure below. Explain why the Cs-line is to the left of the Na-line in terms of
the 'atomic structure ideas known to you. [I.I.T.B. M.Sc. Admission Test 2000]

5.2 5.6
Frequency (in 1014 Hz)

Ans. : The given figure shows that the frequency of light required to eject an electron
is greater for Na than Cs. It happens as Cs has outermost electrons in 6s shell while Na
has that in 3s shell and screening effect of innermost electrons is more pronounced on
6s electron than on 3s electron. Hence, the former electron is more loosely bound than
the latter and thus, requires less energy to escape attraction to nucleus.
57. A cubic box with each side measuring IOA (with zero potential inside and infinite
potential outside) contains a system of 4 quantum particles. Find the degeneracy of the
lowest energy state of the system. [C.U. 2003]
Ans. : Second energy state of cube is 3-fold degenerate. Each energy state can
accomodate at most 2 quantum particles. Lowest energy state will accomodate 2 particles.
Remaining 2 particles can occupy the 2nd energy state in the following different ways :
538 PROBLEMS ON PHYSICAL CHEMISTRY

(En xEnyEn)
z (E211) (E112) (E121)
2 0 0
0 2 0
0 0 2
l 0
0 1
0
Hence, degeneracy of lowest energy state is six.
58. Prove that commutator of two Hermitian operators is always anti-Hermitian.

Hints : Prove (AB- BA) is anti-Hermitian. Proceed as problem 23.

59. What is radial distribution function? '


Ans. : Radial distribution function, 2
R;
1(r )r determines the probability of finding
an electron anywhere between two concentric spheres of radii r and r + dr, i.e., it gives
the probability of finding the electron at a distance r from the nucleus.
60. Identify the following hydrogenic orbitals with given radial parts as (i) N 1 (~ 2 - r)
exp[-a2r] and (ii) N 2r exp[-a3r], where Ni, ai, ~i are constants. Explain your answer using
plots of the orbitals. [C.U. 2006)
N (~
Ans. : Let 'l'i = 1 2 - r) e-°'lr, '1'2 = N 2 r e-a3r,
'I' 1 is zero only at r = ~ 2 and after passing through which it changes sign. So it has
only one node at r = ~ 2 , whereas '1' 2 has no node but becomes zero at nucleus.
So, 'l'i is 2s orbital, since for 2s orbital number of node = n - 1 - l = 2 - 0 - 1 = 1
and s orbital has non-zero ampitude at nucleus and '1'2 is 2p orbital since for 2p orbital,
number of node= 2 - 1 - 1 = 0.

\jf I

r
r
61. Explain whether the following functions \jf, - \jf, 2i\jf represent the same state,
given 'I' is real. [C.U. 2006)
Soln. : Let \jf 1 = \jf, '1'2 = -\jf, \jf 3 = 2i\jf, where \jf is real.
Now, \jf 1*'I' 1 = \jf*\jf
'1'2 *'1'2 = 'l'*'I'
'1'3*'1'3 = 4 i*i'l'*'I' = 4\jf*\jf "' 'l'*'I' (4 is only a number)
Probability densities for \jf 1, \jf2 and '1'3 are same and so they represent the same state.
62. What is meant by degeneracy? Find the quantum number associated with energy
2
17
level h of a particle in a cubical box and hence, indicate the degree of degeneracy.
8ma 2
[C.U.(H) 2005)
QUA>ITUM MECHANICS 539
Solo. : If several linearly independent eigenfunctions of an operator correspond to
the same eigenvalue, the number of eigenfunctions is the degeneracy with respect to that
operator representing some property.
For 2nd part of the problem, proceed as Problem no. 35.
Here, nx2 + .ny2 + n:2 = 17
or, 22 + 22 + 32 = 17.
The given energy level is 3-fold degenerate.
63. Determine [x, pyl. [t, E].
Solo. : [x, Pyl 'If (x, y) = xpy 'If (x, y) - PyX 'If (x, y)

= -inx a'lf(x,y) + inx a'lf(x.y) [p =-iii a"' l


ay ay y ay

= 0.

E = ma"'
at
[t, E] 'If (x, t) = tE 'If (x, t) - Et 'If (x, t)

= iii t gt 'If (x, t) - iii ~ t 'If (x, t)

= iii t gt 'If (x, t) - iii t gt 'If (x, t) - iii 'If (x, t)

= - iii 'If (x, t)


[t, E] =-iii.
64. Find the average (x) of a particle confined in a one-dimensional box. Would thi_s
location be right place to find the particle if it is in second energy state.
Hints : Let the particle be moving in a one-dimensional box of length a between

0 to a. (x) = Jsin n:x xsin n:x dx = ;; Jx( I -cos 2:7tX )dx= ~


0 0

i.e., (x) is at the centre of the box.


In 2nd state, 'If = 0 at the centre of the box. Thus, in second energy state particle
will not be found at this location.
65. The wave function of a quantum state of hydrogen atom with principal quantum
3

number n =2 is '1'2 im (r. 0, <!>) -b(_.!_)


= -y327t ao
2
(2-..!:...)exp(--r-)·
ao 2ao

(i) Identify the values of quantum number l and m and hence, the atomic orbital.
(ii) Find where the radial node of the wave function occurs. [JAM 2011)
Hints : (i) Since the wave function has no angular part, both the quantum numbers
l and m are zero, and the given wavefunction con;-esponds to 2s atomic orbital.
(ii) See Prob. 31 in the section Numerical Problems of this chapter.
'"'
540 PROBLEMS ON PHYSICAL CHEMISTRY

66. An atomic orbital is described by the weave function

1 r )
-( -
\j/(r) = ~e ao , where a0 is the Bohr radius.
'V 1tag

Given : d't = r2 sin e dr de d<j> and j rne-llr dr = p~;I (n is a positive integer).


0

(a) Identify the atomic orbital and calculate the mean or the average radius of this
orbital in terms of a0 .
(b) Calculate the most probable radius (in terms of a0 ) at which an electron will
be found when it occupies this orbital.
Hints : (a) ls orbital : See problem 27 for (a) and problem 47 for (b) in numerical
section.

Numerical Problems
1. When aluminium is irradiated with light, the maximum energy of ejected electron
is 2.3 eV for ').. = 2000 A and 0.223 eV for ').. = 3000 A. Calculate Planck constant and
the work function of aluminium. What is threshold frequency for aluminium?
Solo. : Given, energies of ejected electrons :

hv I - W = hc- - W = (2.3 X l.6 X 10-19) J (I)


A1
c
and h - - W = (0.23 x l.6 x I0-19) J. (2)
"A.2
Given, "A. 1 = 2000 A and "A.z = 3000 A.
(2.3-0.23)x J.6x 10- 19 1
Hence, h =
c(J_ __
"A.1
l )
"A.2

(2.07x1.6x10- 19 )1
= 8 1 1 1
( 2.9979 x I0 )m.sec- ( - )m- 1
2000 x 10- 10 3000 x 10-10
= 6.628 x 10-34 J.sec.

Thus, work function : W = h~ - 2.3 x 1.6 x 10-19


"A.,

(6.628x 10-34 )J.sec x (2. 9979x 108 ) m.sec- 1


= - (2.3 x 1.6 x 10-19) J
( 2000 X 10-IO) m

= 6.255 x 10-19 J = 3.91 eV.


QUANTUM MECHANICS 541
Threshold frequ~ncy of aluminium,

19
= (6.255x10- )J
[·:hv 9 =W]
(6.628x 10-34 ) J.sec
= 9.437 x 1014 Hz.
2. Calculate the de Broglie wavelength of an electron that has been accelerated through
a potential difference of 300 volts.

Solo. : A. = h , where A. is de Broglie wavelength.


"12meV
34
= 6.626x 10- J.sec
-:="=============================~
= 7.0812 x 10-11 m =0.70812 A.
~2 x (9.109 x 10-31 ) kg x (1.602x10- 19 x 300 )1
3. The work function for sodium metal is 1.82 eV. Calculate the threshold frequency
for sodium. [C.U.(H) 2002]
Solo. : hv 0 = W = 1.82 eV, where W = work function, v 0 = threshold frequency.

Vo= (1.82xl.602x10-19)1 = 4.40 x 1014 sec-I= 4.40 x 1014 Hz.


(6. 626 x 10-34 ) J. sec
4. Calculate the wavelength at which a cavity at 3000 °K will radiate most per unit
wavelength.

SoIo. : By W1en ~
. ,s d"1sp1acement Iaw, 11.max -- constant -- 2.898x10-3m oK
T 3000°K
= 9.660
m = 9660 A.
x I 0-7
5. At a given temperature, for a black body cavity, A.max = 6000 A. What will be
the ~ax if temperature of the cavity is decreased such that the rate of emission of spectral
radiation decreases two-fold?
Solo. : According to Stefan-Boltzmann law,
RT = oT 4, (I)
where RT is total emissive power at temperature T, and o is a constant.

N OW, RT' RT
=1 at T' .

Here, Rr = -f
R
= o T' 4
. (2)
Dividing equation (I) by equation (2),
4
T) .!.
( T' = 2, :. TT = 24.
By Wien's displacement law,

Amax T = Amax T'.

Amax = Amax I,
T
= 6000 A X 2{- = 7135.24 A.
542 PROBLEMS ON PHYSICAL CHEMISTRY

6. 2.14 eV is required to remove an electron from caesium. (a) What is the cut-off
wavelength for photoelectric emission from caesium? (b) Will a light of 'A= 6000 A liberate
an electron from caesium?
Soln. : (a) Given, hV 0 = 2.14 ev:

or, ~: = 2.14 eV,

34
Ao = (6.626x 10- )1.secx(2.9979x !0 )m.sec-
8 1

(2.14x1.602x10- 19 )1
= 5.79418 x 10-7 m = 5794.18 A.
(b) Caesium will not show photoelectric effect wi.th a light of 'A = 6000 A, since its
cut-off wavelength is 5794.18 A.
7. Potassium surface is irradiated with a radiation of 4000 A. Work function of
potassium is 2.30 eV. What is the kinetic energy of (a) fastest and (b) slowest emitted
electrons? (c) What is maximum possible wavelength capable of removing electron from
potassium surface? (d) What is stopping potential? (e) How many electrons will liberate
per unit time from the surface if the intensity of incident radiation is l .5 W /m 2 ?
Solo. : (a) Maximum kinetic energy of ejected electron :

..!..mv2 = E = hv - W
2
( 6.626x10-34 ) 1.sec x (2. 9979x10 8 ) m.sec- 1
= x _ m
4000 10 10
( 2. 3 x l. 602 x,w-• 9 ) joule

= 1.28142 x 10 -19 J. (l)


(b) Kinetic energy of slowest emitted electron = 0.
(c) Work function = 2.3 eV.
hv0 = 2.3 x 1.602 x 10 -19 1. (2)

Hence, h {;- = 2.3 x 1.602 x 10 -19,

.. Ao
34
= (6.626xl0- )1.secx(2.9979xl0
8
) m.sec-• = 5 .39111 x I0-7 m
19
(2. 3xl.602x10- )1
= 5391.l l A. (3)
Hence, maximum possible value of 'A that can remove an electron = 5391.l l A.
(d) If stopP.ing potential is V0 , then
eV0 = 1.28142 x l0-19 1 [from l]
9
.• VQ= l.28142x1q-• 1 =0.799V[1=CxV]
l.602x 10- 19 c
(e) Let the number of liberated photoelectrons be n.
Now, nhv = 1.5 W/m2.

1.5 W i m 2 x(4000x10- 10 )m
n = = 3.0205 x to 18/m2 .sec. .
(6. 626 x 10- 34
)1. sec x ( 2. 9979 x l 08
)m.sec- 1
QUANTUM MECHANICS 543

8. The stopping _potential for photoelectrons emitted from a surface irradiated by a


light of wavelength /... = 3000 A is 1.91 V. When the incident wavelength is ch.anged, the
potential is found to be 0.90 V. What is the new wavelength?

Solo. : Given, .!.mvz = kinetic energy of liberated electron =


2
1.91 eV = he - W (where A. = 3000 A) and (I) ••
/... .

0.90 eV = ~~ - W (Let A. be a new wavelength.) (2).

Subtracting equation (2) from equation (I),


I.OJ x 1.602 x 10-1 9 J

= (6.626xI0-34 ) I.sec x (2.9979xl0


8
) m.sec- 1. (
3000
_x\o-'om -i,)
A.' = 3.9702 x 10-1 m = 3910.2 A.
9. (a) What is the wavelength associated with a ball of 50 gm moving at a speed
of 500 m/sec? (b) Why is the wave nature of ball not revealed through diffraction effects
from solid?
34
· 626 x 10- I.sec
Solo. : (a) Wavelength, A. = h = h
p = (so x6 10- 3
) kg x 500 m/sec
mv
= 2.65 x 10-35 m = 2.65 x 10-25 A.
(b) A. associated with the ball is too much smaller than the interatomic separation
( == IA) in solid to be detected through diffraction pattern.
10. Calculate the de Broglie wavelength of an electron moving with kinetic energy
500 eV.

Solo. : de Broglie wavelength, /... = h


p

= .J2mh KE [KE = kinetic energy]

{6. 626 x 10-34 ) J. sec


=
~2x(9.109x10- 31 )kg x 500ev x (1.602 x 10-19 )1 I eV
= 5.485 x 10-11 m = o.549 A.
11. Calculate the smallest possible uncertainty in the position of an electron q:ioving
with a velocity of 3 x 107 m/sec, ·
h
Solo. : Uncertainty in position Ax =
47tm~v

(6. 626 x 1.0-34 ) J. sec


= = 1.931 x 10-12 m = 1.93 pm.
4 x 3.14x (9.109x10-31 ) kg x 3x10 7 m.sec- 1
12. Find the uncertainty in the momentum of a particle when· its position is det\!nnined
within 0.01 cm. Find also the uncertainty. in the velocity of an electron, a proton and an
ex-particle respectively when they are located within 5 x 10-10 m.
544 PROBLEMS ON PHYSICAL CHEMISTRY

Solo. : Uncertainty in the momentum of the particle,

h (6.626xl0- 34 )J.sec
!lp = 47tL.U = 4 x 3.14 x 0.01cmx10-2 m I cm = 5.28 x
A_ 10-31 kg.m.sec-1.

Uncertainty in velocity of an electron :

~v =
34
h _ (6.626xl0- )J.sec
= 1.16 x 10s m.sec-1.
47tm& - 4x3.14x(9.l09xl0-31 )kgx(5xl0- 10 )m

Uncertainty in velocity of proton :


34
Av= (6.626xl0- )J.sec _ _
u - 63104
• m.sec 1.
4 x3.14 x ( l.672 x 10- ) kg x x 10-~o )m.
27
(s
UncertaiRty in the velocity of an a-particle :

(6.6~6x 10-34 )J.sec


~v =
~xl0-
3
4x3.14x( )kgx(sx10-10 )m
6.022x10 23
= 15.88 m.sec- 1.
_;
13. Show that fly) = (l6y4 - 48y2 + l2)e 2 is an eigenfunction of the operator

yz - ~ . Find the eigenvalue.


dy2

Solo. : (l -; 2

2 )(l6y4 - 48y2 + 12) e -f


y2 d2 i
= (16y6 - 48y4 + l2y2)e-2 - -2
(l6y4 - 48y2 + 12)e-2
dy
d 2B
= A--
dy2 .

i
dB = (-16y5 + ll2y3 - lOSy)e-2
dy
2
2
d B = (l6y6 - 192y4 + 444y2 - 108)e-1i-.
dy2
d 2B .
Now, A - - - gives
dy2

2~~
:y2 J (y)
;
(
l- = (l44y4 - 432y 2 + 108)e-2 = 9fly) ..
Thus, eigenvalue is 9.
QUANTUM MECHANICS 545
2
14. Show that e -tx 2
is an eigenfunction of the operator d
dx 2
- x2 . Find the eigenvalue.

Soln.:

I 2
=- e-!x . Hence, eigenvalue is -1.
IS. Polynomials P0 (x) = a0, P1 (x) = a 1 + b1t, P2 (x) = a2 + biX + CiX2 are the first three
members of an orthogonal set of functions in the interval - l S x S 1. Evaluate the constants
a0 , a 1, b 1, etc.

Soln. : 1
-1
Pci'(x)dx =
+I
or, Ja~d:t =
-I
1, :. Do =
~
1 (l)

Again,

+I +I

or, aoa1 f dx + aobi f xdx =0


-I -I

or, aaa1 = o. :. 01 = 0 [ ·: ao = l ] . (2)


12
JPl(x)dx =
+I
Now,
-I
+I

or, J(al +q
-I
2 2
x +2a1 1x)dx = 1b or, b~(~)= 1. [ ·: from equaton ('.t), a 1 =OJ

(3)

+I

Again, JP P dx =
0 2 0
-I

+I .

or, J(aoa2 +aob2x+aoc2x2)dx = o


-I
or, 2aaa2 = -~aoc2.

.• Oi = --1-· (4)

Prob. Phv. Chem.-J5


546 PROBLEMS ON PHYSICAL CHEMISTRY

Now,

+l

or, 2
j(b1a2 x+b1b2 x +b1c2 x )dx
3
=O
-1
or, b 1b2 = 0.
b2 =0 [ ·: from equation (3), b 1 = .Ji]. (5)

+1

Again, f Pi
-1
dx =l
+1 +1 +1

or, Ja~dx+
-1
2 4
J2aic 2 x dx+ Jcix dx [·:from equation (5), b2
-1 -1
= O]

Thus, c2 = -3a 2 = -
3.JS
{2 .
2
16. The speed of an electron is measured to be I 00 m/sec with an uncertainty of
0.1 %? Find the minimum uncertainty in locating the position of the electron.
Solo. : v = l 00 mlsec
6v = 0.001 x 100 m/sec = 0.1 m/sec.
Ap = mdv = (9.109 x I0-31) kg x 0.1 m/sec = 9.109 x 10-32 kg. m/sec.
34
(6.626xI0- )J.sec ·
& = _h_ = ) = 5.792 x 10-4 m.
47tAp 4x3.14x(9.109x10-32 kg.m/sec
17. Calculate the de Broglie wavelength of an electron travelling at l.00% speed of
. light._ [C.U.(H) 2002]
=
S'olo. : Velocity v (2.9979 x l 08) m/sec x 10-2 =
2.9979 x 106 m/sec.

Wavelength, A = h = (6. 626 x 10-34) J. sec


(9.109 x 10-31 ) kg x ( 2. 9979 x 106 ) m /sec
mv
=
2.426 x Hr-10 m 2.43 A. =
18. Average life-time of an excited atom is 10-s sec.
(a) What is th.e minimum uncertainty in frequency of the radiation emitted by the
atom while decaying to ground state?
(b) What is the uncertainty in energy of the excited state?
(c) Calculate the natural width (6v/v) of the spectral lines of sodium atom at
A.= 5890 A.

Solo. : (a) Uncertainty in frequency, 6v = 4 ~ 1 = 4 x 3. 14 x 10_8 sec


= 7.96 x 106 sec-t.
QUANTUM MECHANICS 547

(b) Uncertainty in energy


= M.v = (6.626 x 10-34) 1.sec x 7.96 x 106 sec- 1
= 5.275 x 10-27 J = 3.29 x 10-S eV..
(c) A. = 5890 A= %·
6
sec- 1 6 1 10
:. tw = 7.96x10c 7. 96 x 10 sec- x5890 x 10- m
= -------.,,8-~---
2. 9979 x I 0 m I sec
= 1.56 x 10 -8.
v

19. Assume that 'lf(x) =csin ( m;:) is a wave function of a particle in a box of length
L. Verify that it satisfies H'lf = E'lf. What is the value of E so obtained? Calculate c from
normalization of 'I'· [B.U.(H) 2000)

Solo. : H'lf = __fi_


2
L2
87t m dx
c sin
·
(!!E.)
L

= nzh27t2 c sin (n1tX) = n1h22 c sin (n1tXL ) = E'lf.


87t 2mL2 · L 8mL
n2h2
E = 8mL2 .

c2 J.
L

sm 2 -n7tX dx
L
=
0

or, c: f (I-cos 2nL1tX )dx = I [ ·: cos 2x = I - 2 sin2 x]


0

or, c; [(x)~ -(5in2~~/tL):] =I


or, ,2 x L2 =l [since n = l, 2, 3, ... , etc., sin 2n7t = OJ

.. e=I-f.
20. Consider a particle is moving to and fro in a box of length a.
(a) Find the averge value of Px• for n =l state. Comment on result.
(b) If the particle is in its ground state, what is the probability of finding the particle
in tlie range E. to 2a ? Find the same for the particle in its first and second
3 3 i
excited states.

(Px) J'l'~Px'lfidx . 7tX ( - I.,.na- ) sm-


= -a2 fa sm- . 7tX d
Solo. : (a) =
0
a ox a x
[ l/f for particle in a box is ~sin n~]
548 PROBLEMS ON PHYSICAL CHEMISTRY

, a
. (COS--
27tX)a
~ -itz3-.~J sin 1tX cos 1tX dx
= -i!!._ Ja · 27tX dx ~.!!!-__2 a 0 = O.
a a a 2 sm--
a ·a a 2rc/a
0 0 q.
Probability 0f finding positive value of momentum for a particle in a box is exactly
same as that of its negative value of momentum and they compensate each other to gh:e
zero average momentum.

(b) Probability of finding the particle in the ground state in the range J to 2a,
3
2a/3

f'V~'V1dx = 3-
a
J
2 3
sin 2 1tX
a
dx (since 'Vn fl sin n1tX,
='\/a a
here n = I)
a/3 a/3

= I
5- I ( . 4rc . 2rc)
2rc sm5-sm5
= _31 + O·rc866 = 0.609.

Probability of finding the particle in the first excited state within the range !! to 2a ,
. 3 3 I

'
2a/3 2a/3 ' .
= ~ Jsin 2 2
: dx =~ J(1-cos 4
: )dx = j- 1 8 4
4 rc(sin ;-siri 3rc) =0.196.
a/3 a/3
2
Probability of finding the particle in the 2nd excited state between } to ; .

2a/3 213 213 .6


J'V;'V3dx
a/3
=~
a/3
sin
23
:dx = ~ a/3 (1-cos :X)dx·
Proceeding in the same way, probability = 0.333.
21. The wave function of a certain state of a system is the linear combination 'V =
ff 4>; + ~ <1> 2 , where $ 1 and <1> 2 are energy eigenfunctions with distinct eigenvalues E1

a11d £ 2 respectively. What is the probability that the system energy will be observed with
E1'f [B.U.(H) 1994]
QUANTUM MECHANICS 549

(since <!>1 and <!>2 are non-degenerate eigenfuqctions of H,


f <1>: H$ 2d-c = f <l>;H$ 1d-c = 0)

= .!_E1 ff<!>
II
d-c+iE2 ff<!>
22
d't (given, H¢1 = E1¢z, H¢2 = E2¢.i)
4 4

The probability that the system energy will be observed with E 1 is


4
22. A particle with a mass 6.65 x 10-27 kg is confined in an infinite square well
of width a. The energy of the third level is 2 x 10-_24 J. Calculate a.

Son.:
l E = __
n2h2
8ma 2
3 x (6.6626x10- 34 )1 .sec
.. a= (-: n = 3)
~8 x( 6.65x10- 27
) kg x( 2x10-
24
)1
= 6.094 x lQ-<.I m = 6.09 nm.
23. A particle of mass I()-{> kg is rolling on the smooth floor of a 1 x l Q-4 m wide
box with a speed of 3.3313 x 1Q-3 m/sec. Applying particle in a box problem, calculate
the quantum number corresponding to the translational energy of a ball. Will you consider
the energy to be quantized in a practical sense?

Solo. : Kinetic energy, T = -l mv 2 = n2h2


2 8ma 2
2 2 2
or, n2 = 4m v a
h2

n = 2xl0--6kgxl0-4mx3.33l3xl0-3m/sec = 1x1021.
6.626x10-34 J.sec
2
till= E - E = (2n+l)h2 = (2x10 21 +t)(6.626xto- 34 1.sec)
n+ I n 2
2
8ma 8x10--6 kg x (10-4 m)
= l.098 x IQ-32 J.
Thus, the energy may not be considered quantized in a practical sense.
24. An electron is confined to a molecule of length 2 nm. What is the minimum energy
and minimum excitation energy from the state? What is the probability of locating the
electron between x = 0 and x = 0.5 nm?
2
( 6. 626 x 10-34 J. sec )
Solo. : Minimum Energy, E =~ =
8ma 2
8 x (9.109 x 10-31 ) kg x ( 2 x 10-9 m )
= 1.51 x 10-20 J.
550 PROBLEMS ON PHYSICAL CHEMISTRY

Minimum excitation energy, llE = {2n + I)h2 2


8ma
2
= __3_x_{~6_.6_2_6_x_I_o_-_34_J_.s_e_c)__-,,.. =4.52 x 10-20 J.
8x9.109x10-31 kgx {2x10-9 m )2
Probability of locating electron between x =0 and x = 0.5 nm,

I"'•'"
b

'Yl'YI
dx = ~T Jsin 1txI sin 1txI dx
0.5
(since \jln
ti
= -I sin - I
n1tx
)
a 0

= !oJ.5(1-cos27tx)dx = ![(x)o.5
0
1 (sin27tx)0.5]
__
l I I 27t I 0
0

0 5
= · -- = 0.091 (·: I= 2 nm).
I 21t
25. Calculate the first excitation energy of a proton confined to a region roughly equal
to the diameter of a nucleus (10-15 m). Calculate the probability that in its ground state
it will be found between x = 0.25/ to x = 0. 75/.
Solo. : First excitation energy of proton :

llE = (2n+I)h 2
8ma 2

3 x (6.626 x 10-34 J.sec )2


=
S x 1.672x10-21 kg x (10- 15 m)2
= 9.S47 x 10-11 J = 614 x 106 eV = 614 MeV.
Probability of locating the particle within 0.25/ to 0.75/,
0.751
=- 2
l
I. 21txdx
sm -
l
0.251

=
0.251
T°T (I -cos
2
~ )dx = f = 0.82.
[o.5/ -
2~ (sin
2
~ l:::J
26. If benzene is regarded as a square box of side 0.4 nm containing 67t-electrons,
then calculate the wavelength of light to promote a 7t-electron to the first excited state.

4h 2 + ~
10h 2 +5h 2 2
Energy of excited state, Eex =~ 2 2
-- 2
+-
2
Sh- = 27h
Sme a 8me a 2 Sm a 2 ·
8me a 8me a e
QUANTUM MECHANICS 551

2
or, /... = 8cmea
3h
2
= 8 x (2. 9979 x 108 ) m.sec- 1 x (9.109 x 10- 31 ) kg x (0.4 x 10-9 m )
3x6.626x10-34 J.sec
= 1.758 x 10-7 m = 175.8 run:
1 x

27. A particle has the wave function \jf(r) = ( ~ )2 e-~,where a 0 is constant. Find (r).

Solo. : (r) = J'I'• (r)r\jf(r)41tr2dr


0

3!
=
(~r
28. Calculate the zero point energy of a particle of mass 1o-3 kg constrained to move
only along x-axis when it is fixed to a spring which is stretched 10-2 metres by a force
of 0.1 newton.

Solo. : By Hook's law, force constant k = F = O.lN = 10 kg.sec-2 .


-x 10-2 m

Frequency v = 1 {k
21t -v-m
= lOkg.sec-2 = 15.92 Hz.
27t 10-3 kg

Zero point energy, E0 = -1 hv


2

= ~ x (6. 626 x 10-34 )lsec x 15.92 Hz = 5.27 x J0-33 J.

29. Verify that the function Ae ( --} Bx2) is an eigenfunction of· linear harmonic

oscillator, where B = ~(':). [V.U.(H) 2001]


2 1
Solo.: H = -h-a- 2
1 2
"' +-kx
2m ax 2
(1)

1 2
'I' = Ae --zBx (2)

:-. :::. 1 2 _lsx2


Now ~ = ~Ae-2Bx = -ABx e 2 = 'I'' (3)
ax ax

and a1"' a"'' =.-AB e--zBx


~=-
1 2 1 2
+ AB 2x2 e --zBx = -8(1 - Bx2)'1f. (4)
ax ax
552 PROBLEMS ON PHYSICAL CHEMISTRY

[putting the value of \jl from equation (2)]


2
ti2-i1-
=-2m '1' I 2
Thus, H\jl
ox 2 +-kx
2
\jl

= ~B(I-Bx2)v+..!..kx2'1' [from equation (4)]


2m . 2

= -n2. -..J,;;k 2
- \ j l -h- . mk
- x 2\jl+-I kx2 'I' (given , B = ..J,;;kn )
2m n 2m n2 2

= !!_ {k'V _ _!_kx 2\jl+.!.kx 2\jl = .!.ri {k\jl


2v-;;; 2 2 2 v-;;;
= ..!.. hv\jl (where v = _I_ fI and Ii = 2h1t )
2 27tVli
= E\jl.
I
Hence, E= -hv.
2
I r

30. The ls wave function for H atom is \jl = (7ta~ f2 e 00


• Calculate the probability
of the electron being within a distance of a0 (Bohr radius) from the nucleus.
[V.U.(H) 2002; C.U.(H) 2005]

Soln. : P1• = J
0
'V•'Vd't

Do n 2n
= J
0
2
J
\jl•\jlr dr sin6d6 dq> [d't
0
J
0
= r2sin6 drd6 dq>]

= ~(['~~~F+~dr]
= _i_[a~e- +a°"J;e-~:dr] =_· +.i.[[re-~: JPe-~: drl
a3
0
-2
2
0
2 e-2
a2
0 --
2
]"o _
0
2
0 --
. ao o . ao
QUANTUM MECHANICS 553

= -2e-2 - 2 e-2 + -
2
e
al -~r dr
0

ao o
= -2e-2 - 2e-2 - e- 2 + l = 0.3233.
31. The radial wave function for 2s orbital of a hydrogen atom is given by

R20 = N( 2- ~ }- 2 ~, N is constant. (i) Determine the number and location of node


(s) in 2s wave function. (ii) Write down the expression for radial distribution function of
a 2s electron and sketch the radial distribution curve. [C.U.(H) 2002]
Solo. : (i) The number of radial node = n - l - l.
For 2s orbital, n = 2, l = 0.
Hence, the number of node = 2 - l = l.
(ii) Radial distribution function for 2s electron,

where r = distance from nucleus.

At the location of node, R20 = 0.

or, N(2- ~}-2~ = 0

or, ·2 - = 0 (since N and e 2ao are not zero)


..!_
ao
r=2ao.
Hence, ·node is at 2a0 distance from nucleus.
32. Estimate the wavelength of the light absorbed when a it-electron of butadiene
is excited from the highest occupied energy level to the lowest vacant energy level. For
the sake of simplicity assume that the it-electrons of butadiene move in a one-dimensional
box of length 7.0 A. [C.U.(H) 2004)
Solo. : Ground state energy of butadiene,
2 2
2h
Eg, = -~2 + 8h
- -2·
8mea 8mea
554 PROBLEMS ON PHYSICAL CHEMISTRY

Energy of lhe excited state,


2
= -2JE._2 + 4h • + -2fi:_.
8mea 8mea 2 8mea 2

l:!E = Eex - Egr = hv = h I =


Or, 1
/\, = 8mea 2c
5h
2
= 8 x ( 9.109x10- 31 ) kg x ( 7x10- 10 m ) x ( 2. 9979 x.10 8 ) m. sec- 1

S x( 6.626x10- 34 ) J.sec
= 3.23 I x 10-7m = 3231 A.
33. The 2s wave function for H atom i~ 'l'zs =N( 2 - ~ }-fao, where N is a constant,
r is the distance from nucleus and a0 is Bohr radius. Find the distance from the nucleus
at which radial probability density is maximum.
Soln. : Radial probability density for 2s electron = 'l';s'l' 2,,r 2 = [R 20 (r)J 2r2.
At the distance where radial probability density is maximum,
2 2
d[Rzo(r)] r =O
dr

or, =0

or, .i_2 '- a6' + 4 = 0 (smce,


· r ~
~ 0)
a0 0

.l± 36 16
ao ;;--;;-
or, r = z° 0
= 3a 0 ± ./5a0 •
;;:0
=
r1 5.236 a 0 and r2 0.764 tlo· =
Radial probability density for 'Vis becomes maximum at 5.236 a0 and at 0.764 a0 as
well.
34. Evaluate transmission coefficients for (i) electron (ii) proton for penetrating a
rectangular potential barrier of height 12 MeV and thickness 10-14 m. Both the particles
have total energies 5 MeV.
Soln. : Transmission coefficient,
I
T = 2
cosh 2 Pa+-I ( - p) smh
CX - - A~
. 2 ,.,..
4 p a
QUANTUM MECHANICS 555

13 = 2mV(i-E.).
2
n v
a = ../2mE
h
Here, a = 10-14 m, E =5 MeV, V = 12 MeV.
(i) For electron,

p= Zn l2x(9.l09xl0- 31 )kgx(7xl0 6 )evx(t.602xl0- 19 )JteV


6. 626 x 10-34 J. sec 'I
= l.355 x 1013 m-1.
T = 0.981.
(ii) For proton,

p= ~ 2n_ 12 x (t. 672x10-27 )kg x (7 x 106 )ev x ( l.602 x 10-19 )1 I eV


34
(6.626x10 )J.sec I/
= 5.81 x 1014 m .
:. T = 3.494 ~ 10-5•
35. Calculate the transmission coefficients of electron tunnelling through a rectangular
barrier of height 2 eV and width (i) 2 A, (ii) 4 A. Electron has total energy l eV.
Soln. : E =l eV, V = 2 eV, a = 2 A = 2 x 10-10 m.

2rt~2 x (9.109 x 10-31 kg) x l eV x ( l.602 x 10-19 )JI eV


p = 6.626xl0-34 J.sec
= 5.12 x 109 m- 1.
. . transmission coefficient,
T = 0.404.
When a= 4A = 4 x 10-10 m-1,
T = 0.064.
36. Normalise the function eim•.
21t
Solo.·: N2 Je_,.,,..e,.""d<I> = I (N = normalisation constant)
0

or, N2 J
0
d<I> =l or, N2.27t = I.
1
N = "Jri'
37. If 'If is an eigenfunction of operator a with eigenvalue a, then 13'1' is an eigenfunction
of ex with eigenvalue a + k provided [ex, 131 kj3. =
Solo. : Given, Call - j3cx)'lf = kP'lf.
.. al3w = {Jal/f + kll'lf = 13a\lf + kll'lf (since aw = a'lf)
Hence, al3w = (a + k)j3'1f.
556 PROBLEMS ON PHYSICAL CHEMISTRY

38. An operator T; is defined as i; \jl (x) = \jl(x + i).

(a). Is T; linear? (b) Evaluate : (fr -3f1 +z) x2 •


Soln. : (a) If i; c\jl (x) =c\jl (x + i), where c is any constant; T; will ..be called linear.

(b) (f?-3f1 +z)x2


= f 1 (x + 1)2 - 3(x + 1)2 + 2x2 [ ·: i;\jl(x) = \jl(x + i)]
= (x + 2) 2 - 3(x + 1)2 + 2x2 =I - 2x.
39. Calculate the probability of a particle in· the right-half of a one-dimensional box
in its first excited state. Will all the other states give the same result? Explain.

Soln. : \jl (x) = f2 sin 27tX , when n :::: 2


"\/a a .
a
2
-2 f sm
. 2 -dx
27tX
a a
0
a
"!
= -!zJ(1-cos 4 :;-x~ (·: 2 sin 2
x =l - cos 2x)
0

= ![E...
a 2 47t
_..E_(sin 47tX)!1
a
= _21 .
0

The probability of finding the particle is same in the two halves of the box for all
the states of the particle.
Function of the particle in a box is the combination of two momentum eigenfunctions.

sin n:;x = l ( .mu


i e'a
.mu) (·:
- e -•a eix = cos x + i sin x)
2

= ~i (eikx _ e-ikx) (where k =~7t ).


So, the probability that the particle will possess positive momentum (lik) is same as
the particle will possess negative momentum (lik) and hence, the probability of finding
the particle in the left-half is same as the probability of finding the particle in the right-half.
40. Show that the functions of simple harmonic. oscillator

= (4l7t )4 2xe-T
I 2
x
\jf1 (x) are orthogonal.
QUANTUM MECHANICS 557

= ( 211t) v2Jl+-
Jxe-x /2dx+ [(-x)e-x /~dx = 0.
- 2 2 l
Since the integral is an odd function of x its magnitude from -oo to +oo must be zero.
41. A particle of mass m moving in one dimension between x =a and x =b is described
by a wave function \j/'= A, where A is normalisation constant. (a) Calculate A, (b) What
x
is the average value of x?

Soln. : (a) 'I' = A


x
b

f"'."' =
a
b
or, Aif x2dx =
a

or, A2 (b-a)= I,
ab
:. A=~ b-a
ab .

b b
(b) x = f'lf~X\jldx = ab Jxdx
b-a ;r
a
a '
b

= b~af ~ = ~tri!.
b-a a a
42. The velocity of a rifle bullet is 1000 m/sec. If it weighs 30 gm, calculate the
minimum precision in measurement of its position when its momentum is not perturbed
by more than one part in a million.
Soln. : v = 1000 m/sec
.1v = 1000 106'
m/sec x 1

Hence, !lp = m.1v = 30 x l()-3 kg x J0--3 m/sec = 3 x 10--s kg.m/sec.

Thus, .1x = 2!lp


Ii

_J_.s_ec_ _ = J.76 x
34
or, .1x = __6_._62_6_x_I_o_-_
5
l()-30 m.
4x7tx(3x10- ) kg.111 /sec
43. Show that the wave functions corresponding to different energy levels for a particle
in a box are orthogonal.

Soln. : "'" = i/a


fI sin mtx [n
a
= I, 2, 3, .. ., etc.]

'I'm = J! sin m1tX

a
[m = I, 2, 3, ... ~etc.]
558 PROBLEMS ON PHYSICAL CHEMISTRY

J'l'n'lfmdx = a2 fa sm7sm-a
. n1tx . m1tX .= aI fa [cos(n-m)-a-cos(n+m)-a
1tX 1tX]
0 0 0

= -al [ -l- ( sm(n-m)-


. 1tX)a - -l - ( sm(n+m)-
. · 1tX)a]
n-m a 0 n+m a 0
= 0 [since sin 0 = 0 and sin (n - m)Zt = sin (n + m)7t = 0
as n and m are integer numbers]
44. A stream of electrons each having an energy E =4 eV is directed towards a potential
barrier of height 5 eV. The width of the barrier is 2.1 x 10-7 cm. Calculate the fraction
transmitted.

Hints : ~ = -v
1 I2m(V-
1l
' E)

27t~2x(9.109x10- 31 )kg x (5 -4)eV x {1.602 x 10-19 )u eV


= 6.626x 10-34 I.sec
= 5.12 x 10- 19 m- 1•
~ = 5.12 x 10..:19 x 2.1 x 10-9 = 10.76.
T = 1.3 x 10-9 .
2
45. A particle has the wave funcion 'II = z(i+a ) e-1 x I sin a.x. Calculate
a2
(x) and .

2
2(1+a2) -Ix! 2(1+a )
Soln.: 2 e sin a.x = Ae-1 x lsin a.x, where A =
·"' = a
\jl 1 = Ae-x sin a.x 0 :s;x :::; oo
\j/2 = A~ sin a.x - oo :s;x :::; 0

+- 0

= J'1'1X'lf1dx + f 'lf2X'lf2dx
0
QUANTUM MECHANICS 559

(p)) =-A'>
2
[f <" J. '1'1 d<+ '1'2 ~2 '12 .U l
= -2A'n2 [ l( 1-n
2
),-" ,;n
2
ax.U + 2,,-hJ ,;""''°'"'"' l
=-2h 2 [(1-a 2 )±-1] [integreating two terms by parts]

= n2 (1 + a2).
Hints :
- .
Je-2 x sin 2 ax dx = ±Je-lx (1
- - cos 2a.x)dx
0

[
0

-
l
1 e-2x e-2x
= - ----
2 2 4(1 +a 2 )
(2asin2ax-2cos2ax)
0

-
·Je-2 x sinaxcosaxdx
·o

= ±J-e- x sin 2a.x dx


2
=2
(e-2x)~

4(l+a
2
)
.i

[-2sin 2ax- 2acos2ax ]~


0

= 4(1 +a 2 ) •

46. Normalize the function 'II = e-zrlao.


Solo. :
-
J
N2 \jl4Ttr 2dr =l
0

-
J
or, N247t e-lzrfaor 2dr=
0

2
O<, N41<. (!J 2
=I

2- .(- z )3 .-,
or, N - 1 .... N = (-Ttl )1;2 ( azo )3/2 .
ao 7t
560 PROBLEMS ON PHYSICAL CHEMISTRY

47. Is wave function for H atom is 'Vis= Ne-'10o, where N is a constant, r is a distance
from the nucleus and.U(J is Bohr radius. Find the most probable value of r in Is state.
Solo; : Radial probability density = R?0 r 2 .
d( R2 ,2)
=-0
dr

or, ~(e-2rl0or2) = O
dr

or, 2re-2rla0 _ l:.__ r2 e -2r/ao =0


ao
or, 2re-2 ' 10 o(l- ~) = 0
or, I - ~ =0 (since, re-2' 10o ;e 0).
ao
.. r =a0 = distance from nucleus at which radial probablity density is maximum.
48. A particle is confined within a one-dimensional box of length a. Find the minimum
possible energy it will have from uncertainty principle.
Solo. : The maximum uncertainty in position of the particle =length of box within.
which the particle is confined.
Hence, Ax = a.

By uncertainty principle, Axtlpx ~~


Hence, minimum uncertainty in momentum of the· particle
h h
= tlpx = 2& = 2a ·
Now, energy of the particle confined in a box,

E = (µ;).
2m
For particle in a box, tip; = (µ;).
tip2 h2
Hence,E = __ x = -- .
2m 8ma 2
49. Calculate the minimum uncertainty in position for an automobile of mass 500
kg m9ving with speed 50 ± 0.001 Jcm.hr- 1•
Comment on the _result [h = 6.627 x 10-34 J.sec]. [C.U. 2001)
Solo. : By the problem, ~vx = 2 x 0.001 km.hr- 1.
tlpx ~ m~vx =
500 kg x 0.002 km.hr- 1 0.278 N.sec. =
By uncertainty principle, Ax~px = !!:_2
6.627xl0_34 10-34m.
:. ~x = 2x0.278
=
4X7tX0.278
m = 1.89 x

Ax is too small to be detected. Hence, uncertainty principle is not impo~ant for


macroscopic bodies like microscopic bodies.
CHAPTER 16

SPECTROSCOPY AND SOME PHYSICAL PROPERTIES

Required Formulae

1. If an angle 0 i_s subtended by two bond moments µ 1 and µ 2 , then resultant dipole
moment

µ = ~µf+µ~+2µ 1 .µ 2 cos0·
.e
If µ1 = µ7.·µ = 2µ1 cos-.
2
. . p D-1 M
2• MI
o ar po I anzatton, m =- -- ,
D+2 p
where D = dielectric constant
M = molecular weight
p = density.
3. Molar refraction, Rm = n22 -1M
n +2 f)'
where n = refractive index.

4.
pm = ~JtNA( Cx 0 +<Xe+
3~T)
<Xa = atomic polarizability
<Xe = electronic polarizability
µ = permanent dipole moment
k = Boltzmann constant
T = temperature
NA = Avogadro number.
S. Specific rotation, [<X K= 1OO<Xobs ,
LC
<Xobs = observed angle of rotation
561
Prob. Phy. Chem.-36 .
562 PROBLEMS ON PHYSICAL CHEMISTRY
..
= length of polarimeter in dm
C = cons;entration of substrate in gm per I 00 ml of solution.
[a]1 x M
Molecular rotation [Ml,_ = /. , where M = molar mass.
1

. 100
6. For a solution mixture of A and B, molar refraction of solution
= RM = XARM(A) + Xfl?M(IJ),
where XA = mole fraction of A
RM(A) = molar refraction of A
X8 = mole fraction of B
RM(B) = molar refraction of B
= n:-1 (XAMA +X8 Ms)
n +2 Psot
= refractive index of solution
= molar mass of A
= molar mass of B.
7. Rotational constant, B = _h_cm-1 =
87t 2 /c
where I = moment of inertia = µr2
c = velocity of light
h = Planck constant, µ = reduced mass
r = bond length.
8. Rotational energy, E = BJ (J + I) cm-1.
9. Spaci!1g between the lines in rotation spectrum = 28.
10. Vibrational energy = (n + ~) hv joule = (n + ~) v sec-1.

Vibr~tional
11. frequency, ii = _l_
2 7tC v-µfI cm-1 = _l_ fI
2 7t v-µ
sec-1,

where k = force constant.


12. Spacing between lines in vibration spectrum =v sec-1. ·

13. Ratio of populations at two energy levels J and J' : NJ =e-(c1-cr)/KT,


Nr
El' Er = energy of the levels J and J'
N1 , Nr = population of tlie respective energy levels.
2J + l -(EJ-Ej' )/KT
If degeneracy of the levels = 2J + I, then N1 ! N1 , = -21'+1
-e .

Short Questions
1. What is polarizability? What is polarization? Can a molecule with zero dipole
moment undergo polarization?
Ans. : Polarizability is the dipole moment induced in molecule by an external electric
field of unit strength. If µind is the induced dipole moment and E is the strength of the
applied field, then µind oc E or, µind = a E, where a is the polarizability.
SPECTROSCOPY AND SOME PHYSICAL PROPERTIES 563

It is the measure of easiness with which a molecule can be polarized.


Polarization (P) is the dipole moment induced by an external field per unit volume.
A molecule with zero dipole moment can be oolarized by the distortion of its electronic
distribution and nuclei by an applied field.
2. Why is CCl 4 molecule is non-polar although the C-CI bond is polar?
[B.U.(H) 2002)
Ans. : Net dipole moment of a molecule is the vector addition of all the bond moments.
Now, bond moment of one C-Cl bond is exactly opposite to the vector addition of other
three C-CI bond moments, since CCI 4 has tetrahedrally symmetrical structure.
3. Find out the dimension of dipole moment µ. What is the SI unit of dipole moment?
How do the induced polarization and the orientation polarization depend on temperature?
[B.U. 2003]
Ans. : Dimension of dipole-moment : current x time x length = ITL
SI unit of dipole moment µ.: C-m (coulomb-metre)
Polarization is induced in both polar and non-polar moleeules by the displacement
of electronic distribution and nuclei towards opposite ends of the external electric field.
This type of polarization is called distortion polarization and it is independent of
temperature. But polar molecules can undergo another type of polarization, called orientation
polarization, in addition to distoration polarization. Orientation polarization occurs due to
alignments of molecular dipoles along external field and it depends on temperature. As
temperature increases, the random orientation of the molecular dipoles also increase and
it, in tum, decreases the tendency of these dipoles to orient along field direction, and thus
orientation pofarization decreases. Thus, for polar molecules induced polarization depends
on temperature while for non-polar molecules it is temperature independent.
4. Show that sum of the bond moment of BCI 3 molecule is zero.

Ans. : The shape of BC13 molecule is triangular planar


Cl

B
t
Cl/ ~Cl

Let the bond moment of each C-Cl bond be µ making an angle of 120° with each
other. The resultant moment of two C-Cl bond moments

= µ' = ~µ 2 +µ 2 +2µ 2 cos120° = ~2µ 2 -µ 2 = µ. Cl



It is opposite to the remaining one C-Cl bond moment. µ <;. µ
c?'t ~Cl
µ'
Hence, the sum of the bond moments =µ - µ' =µ - µ = 0.
5. Explain the graphical plot required to determine the dipole moment of a polar
substance from the molar polarization values measured at a number of temperatures.
[B.U. 1997]
564 PROBLEMS ON PHYSICAL CHEMISTRY

Ans. : For polar substance orientation polarizability a.0 is directly pl'bportional to its
dipole moment µ in the applied field direction and inversely proportional to temperature
T of the system, namely
µ2
a.0· = where k is Bbltzmann constant.
3KT'
Total polarizability a., is sum of distortion (a.d) and orientation (a.0 ) polarizabilities
CX.1 = a.d + a.o
According to Debye equation,

pm = ±nNA a.I
3 Pm /µl
= ~7tNA (ad+ 3~T). ,' Slope=-
or, Pm ,. •• •• 3k

where pm = molar polarization


NA =Avogadro number.
Hence, Pm of polar molecules depends on temperature. Measuring a number of different

Pm values at different temperatures and plotting those values against ~, the dipole moment

µ can be determined from the slope ( ~;}


6. How does molar polarization vary with temperature? How would you determine
the dipole moment of a molecule using the variation of molar polarization with temparature?
Ans. : Same as the answer of Q. 5.
7. What is the CGS unit of orientation polarizability?
2
Ans. : Orientation polarizability, a.0 = L, where µ = permanent dipole moment,
3kT ·
k = Boltzmann constant, T = temperature.
2
. . (esu.cm)
H ence, its unit = ( _)
erg.K 1 K

= (esu.cm)
2
=
( dynei cm cm r = cm 3.
erg dyne.cm
8. What is the SI unit of orientation polarization?

Ans. : Orientation polarization, ~0 =


3
! 0
( ~~}
where e0 = permitivity of vacuum.
µ = permanent dipole moment
k = Boltzmann constant
T = temperature in °K.
SPECTROSCOPY AND SOME PHYSICAL PROPERTIES 565

= = m3.
(c 2 .N- 1.m-2 )(N. m)
9. Is it possible to distinguish a polar molecule from a non-polar molecule from the
temperature variation of molar polarization? [C.U.(H) 1996]
Ans. : In polar molecule sum total of distortion polarization Pd and orientation
polarization P0 contribute to the total induced polarization (Pt) while in non-polar molecule
only the distortion polarization constitutes the total polarization (Pt).
Hence, Pt = P0 + Pd (for polar molecules).
P, = Pd (for non-polar molecule).
So for polar molecules molar polarization,

pm = j1tNA(ad +aµ) = j1tNA( ad+~~)


where NA = Avogadro Number
ad = distortion polarizability
aµ = orientation polarizability
µ = permanent dipole moment,
k = Boltzmann constant
and T = temperature in kelvin,
and for non-polar molecule,

Pm = .34 1tNAad:
ad is characteristic of the molecule and is independent of temperature, and thus Pm
for non-polar molecule is also independent of temperature. But for polar molecules, besides
ad, <Xµ also contributes to Pm and aµ
depends on temperature. So, Pm for polar molecule
varies with temperature. It decreases with increasing temperature while the same for non-
polar molecule remains constant.

'
_/ Polar molecule

}!tNA"d I '---~~~~~~~-
Non-polar molecule

T
So, observing temperature vanatton of molar polarization (Pm), it is possible to
distinguish polar molecule from non-polar molecule.
10. Show that the polarizability has a dimension of volume? Arrange the following
molecules in the order of the decreasing polarizability HCI, HF, HI, HBr.
566 PROBLEMS ON PHYSICAL CHEMISTRY

Ans. Polarizability, ex = induced dipole moment


external electric field
= .!::
= charge x distance = Q x L = /}.
E charge/(distance)
2
Q/ L2
Hence, polarizability has the dimension of volume.
Polarizability decreases successively from HI to HF in the order HI> HBr >HCI >HF,
as the bond length decreases successively from HI to HF in the same order.

11. What will be the nature of total polarization P 1 versus J_ for CH 2CI and C 6H6 ?
T
[B.U. 2003]
Ans. : CH 2CI is polar molecule and C6H6 is non-polar molecule.

CH 2CI

P,

I
•'
T
The reason is given iii answer to Q. 9.
12. Atoms are f1on-polar but a molecule may be either polar or non-polar--discuss.
[C.U. 1993]
Ans. : In isolated atom, negative electronic charge is spherically disposed around the
positively charged nucleus. This symmetric charge distribution makes atom non-polar. But
a molecule may be hbmonuclear or heteronuclear. In homonuclear molecule again the
symmetric charge distribution makes the atom non-polar. In heteronuclear molecule due
to difference in eletronegativity of the atoms the negative charge distribution shifts towards
more electronegative atom, thereby resulting in an asymmetric distribution of charge density.
This asymmetry is dipole moment. It makes a molecule polar. However, if the molecule
consists of more than 2 atoms, and vector addition of all the bond moments becomes null,
then the molecule will be non-polar.
13. Molar polarization value of 0 2 and CH4 are independent of temperature while
those of HCI gas and CH3CI gas decrease with increase in temperature. Explain with proper
reason. [C.U.(H) 2003]
Ans. : For non-polar molecules two factors contribute to its total molar polarization
Pm, namely (i) electric (Pe) and (ii) atomic polarization (Pa), i.e., Pm = Pe + Pa.
Electronic and atomic polarization arise due to distortion of electronic distribution
and nucleus respectively towards opposite ends of the external electric field. Together thex
are called distortion polarization ( Pd). It is independent of temperature, since it is evident
from the definition that the ease with which charge distribution will be distorted depends
SPECTROSCOPY AND SOME PHYSICAL PROPERTIES 567

on the external field and,Qn the molecule itself. 0 2 and CH 4 are non-polar molecule and
hence their molar polarization is independent of temperature.
In case of polar molecule another type of polarization, called orientation polarization,
( P0 ) is present, i.e., Pm =
P0 + Pe + Pa =
P0 + Pd. Now, the rise in temperature increases
random orientation of molecular dipoles, thereby reducing the tendency of the dipoles to
orient along the field direction which in turn decreases orientation polarization. In other
words, it has an inverse relation with temperature. HCl and CH3Cl are polar molecules
and so their molar polarization decreases with increase in temperature.
14. Total molar polarization is partly temperature dependent and partly temperature
independent-why? [B.U. 1999)
Ans. : For non-polar molecules total molar polarization is completely temperature
independent while for polar molecule it is partly temperature dependent and partly
temperature independent.
See Q. 13.
15. Write Clausius-Mosottii equation in CGS and SI systems.
Clausius-Mosottii equation relates the distortion polarizability of the molecule to the
dielectric constant of the non-polar substance.
Ans. : CGS System :

D-1 M =P. - 4rtN


D+2 p m - 3 Aud

D = dielectric constant of the medium


M = molar mass of the medium
p = density of the medium
NA = Avogadro number
ad = distortion polarizability
Pm = molar polarization.
SI System :

(fo)-1
E = permitivity of medium
E0 = permitivity of vacuum.
Other symbols signify usual meaning.
16. Why is p-dichlorobenzene is non-polar but p-dihydroxybenzene is polar?
Ans. : In p-dichlorobenzene two Cl atoms are in the same plane of the benzene ring
and thus the vector sum of two C-Cl bond moments are zero as they are oppositely directed.
But, in the case of p-dihydroxybenzene two OH bonds are not coplanar with benzene,
rather they are directed at an angle to the benzene ring and thus, it is polar.
,!"'

568 PROBLEMS ON PHYSICAL CHEMISTRY

Cl

p-dichlorobenzene. ¢ Cl
µ=0

p-dihydroxybenzene.
HO
er ~

~
µ ¢. 0

17. S02 has dipole moment of 1.6 D while C02 has zero dipole moment-explain.
Ans. : S02 is non-linear molecule and thus, two S-0 bond moments are at an angle 8

with each other giving rise to net dipole moment which is equal to 2µ 50 cos i. /f's~
C0 2 is planar molecule and its two C-0 bond moments are oppositely directed making
an angle of e = 180° and hence, C02 is non-polar. 0 C 0 ==
18. Three different dichlorobenzenes have the dipole moments (i) 1.48 D, (ii) 0 and
(iii) 2.25 D. Identify the compounds.
Ans. : Net dipole moment µ is vector addition of all the bond moments. If µ 1 and
µ 2 make an angle 0 with each other, then

µ = ~µ? + µ~ + 2µ1µ2 cose'


e
if µI =µ 2, µ = 2µ I COS - ,
2

For o-dichlorobenzene, 0 = 60°.


:. µ =2µ1 cos 30° = .J3µ1.
Cl

Fo• m-dichlorobenzene, 0 = 120'. 0


~CI
Cl

And for p-dichlorobenzene, 6

:. µ = 0.
= 180°.
$ Cl
SPECTROSCOPY AND SOME PHYSICAL PROPERTIES 569

Hence, ortho has the highest dipole moment and para has th~ least. Thus (i) is ortho,
(ii) is para and (iii) is meta.
19. What is specific rotation?
Ans. : Specific rotation is defined as the angle of rotation of the plane of linearly
polarized light produced by 1 decimetre length of solution containing 1 gram of the optically
active substance per ml and is represented as

[ex]~ = IOOexobs
LC
[ex]~ = specific rotation measured at temperature t using sodium D line.
ex000 = observed angle of rotation
= length (in dm) of the column of solution
C = concentration of the substance in gram per 100 ml of solution.
20. On which factors does the angle of rotation of the plane of linearly polarized
light produced by an optically active substance depend?
Ans. : The factors are :
(i) the nature of substance,
(ii) concentration of the solution,
(iii) length of the solution traversed by polarized light,
(iv) nature of solvent,
(v) wavelength of the light used,
(vi) temperature.
21. What are the units of molar polarization and molar refraction?
Ans. : Both molar polarization and molar refraction have the unit cm3/mol or m3/mol.
22. Write Debye equation in CGS and SI systems?
Ans. : Debye equation relates the total polarizability, i.e., orientation and distortion
polarizability of the polar molecule to the dielectric constant of the molecule.
CGS System:
D- LM P. 4 7tN µ2 )
D+rp-= m=3
< (

A exd+3kT'

where D = dielectric constant of the substance


M = molar mass of the substance
p = density
pm = molar polarization
NA = Avagadro number
exd = distortion polarizability
µ = permanent dipole moment
k = Boltzmann constant
T = temperature.
570 PROBLEMS ON PHYSICAL CHEMISTRY

SI System ~

c = permitivity of medium
c0 = permitivity of vacuum.
All other symbols signify the usual meaning.
23. The dipole moment of C0 2 is zero but that for HzO is 1.85 D - explain.
[C.U. 1994]
Ans. : C0 2 is a linear molecule having the structure 0 =C =0 while H 20 is non-

linear having the structure H'.Q, . For further information see Q. 17.
8
24. What is polarizability volume? What is its unit?

Ans. : Polarizability volume, a' = __Q,_,


47tco
where a = polarizability of the molecule
c0 = permitivity of vacuum.
_, c2 2
. f J . .m
U mt o a, = _1 2 _1 = m 3 .
J .C .m
25. What is the unit of polarizability in SI system?

Ans. : Unit of polarizability, a(= µ;d)


=
C.m1 = c2.m.N-l = i-Ic2.m2.
N.C-
(unit of dipole moment = C.m, unit of E = N.C-1 )
26. What is Lorentz-Lorenz equation?
Ans. : Lorentz-Lorenz equation relates molar polarization ( Pm) to molar refraction
(Rm).

2
Thus, it is written as P =R = n - I M (I)
m m n2 +2 -P'
where n is the refractive index of the substance, M is its molar mass and p is its density.
This relation holds, both for polar or non-polar molecules, if the alternation frequency
of external field is in optical region (> I 0 15 Hz). Now, at optical region Pm contains only
electronic polarization ( Pe).

It is written as Pm= Pe = .±7t


1
NA ae, (2)
where NA is Avogadro number and a, is electronic polarizability.
Thus, it is evident from equations (I) and (2),
SPECTROSCOPY AND SOME PHYSICAL PROPERTIES 571

Rm = i7t NA ae. (3)


3
Equation (3) is also known as Lorentz-Lorenz equation.
27. Why does Maxwell relation D = n2 hold for non-polar molecule but not for polar
molecule?
Ans; : For polar molecule dielectric constant (D) depends on electronic polarizability
e . (cx.0 ) while for non-polar
(a ), atomic polarizability (ex.a) and orientation polarizability
molecule D depends only on cx.e and a. by the following relations,

D-lM = -43 7tNA ( cx.e + CX.o +ex.a) (for polar molecule)


D+2 p
D-IM
and - - -
D+2 p
= -43 7tN A (a, +ex.a) (for non-polar molecule),
·

where Mis molar mass and pis the density. Now, D is measured using alternating current
of frequency less than J06 Hz, when all the three polarizabilities are present. But refractive
index (n) is measured employing optical technique at the frequency greater than 1015 Hz,
when cx.0 and ex.a cease to exist. Refractive index is related to polarizability by the relation

n:- l M = ±NAae. Since, cx. 0


is altogether absent in case of non-polar molecule and CJ.a
n +2 p 3
contributes minimal, D = n2 holds for them, but a 0 contributes significantly for polar
molecule and thus, D = n2 does not hold for them.
28. What is the essential condition for Raman activity of a molecule? What are Stokes
and anti-Stokes lines in the Raman spectrum? Mention two important applications of Raman
spectroscopy. [C.U. 1995]
Ans. : If polarizability of molecule changes with its vibration, the molecule is Raman
active and this is the essential condition for the molecule to be Raman active.
Stokes lines in the Raman spectrum correspond to the radiation scattered by the
molecule with a frequency lower than that of incident radiation.
Anti-Stokes lines in the Raman spectrum are due to the radiation scattered by the·
molecule with a frequency higher than that of incident radiation.
Two important applications of Raman spectroscopy are :
(i) determination of bond length of molecule
(ii) determination of structure of molecule.
29. How many normal modes are possible in C 2H 6, C 6H 6, Hi\), HCI and C0 2?
Ans. : The number of normal modes are 3N-5 for linear molecule and 3N-6 for non-
linear molecule, where N is the number of atoms.
Hence, the number of normal modes are for
C2H6 3 x 8 - 6 = 18
C6H6 3 x 12 - 6 = 30
H 20 3 x 3- 6 = 3
HCl 3 x 2 - 5 = 1
C0 2 3 x 3 - 5 = 4.
572 PROBLEMS ON PHYSICAL CHEMISTRY

30. Why water cannot be used as a solvent in infrared spectroscopy?


• Ans. : Water absorbs strongly in infrared region. So, infrared spectroscopy of the sample
under investigation cannot be obtained without interference from the absorption of water
if water is used as solvent and thu~, it cannot be used as a solvent in infrared spectroscopy.
31. What are the essential conditions for a compound to be infrared active and Raman
active? Explain with suitable examples. [C.U.(H) 2002]
Ans. : If the dipole moment of the molecule changes with vibration, the molecule
is infrared active, e.g., H20 is IR active as its dipole moment changes with vibration, whereas
H2 is not inactive as H2 is non-polar molecule and during vibration it does not generate
moment. Again, although C0 2 is non-polar molecule, during antisymmetric stretching and
bending, it generates dipole moment which changes with vibrations and hence, these
particular vibrations are only inactive.
If polarizability of the molecules changes with vibration, the molecule is Raman active,
e.g., both H20 and H2 are Raman active as their polarizability changes with vibration.
But only symmetric stretching of C0 2 is Raman active, as polarizability changes only during
this type of stretching.
32. Which of the following molecules show (a) rotational spectrum, (b) infrared
spectrum, (c) Raman spectrum?
HCl, CS 2 , Br2 , CC1 4 , C 2H 4
Ans. : HCl will show (i) rotational, (ii) infrared and (iii) Raman spectrum, since it
has permanent dipole moment which changes with rotation and vibration and its
polarizability also changes with vibration.
CS 2 (S = C = S) will not show rotational spectrum, as it does not possess permanent
dipole moment. But, its asymmetric stretching and bending are infrared active as dipole
moment changes during these vibrations, and its symmetric stretching is Raman active as
this stretching changes its polarizability.
Br2 will show only Raman spectrum for the same reasons given above.
CC1 4 will show only Raman spectrum for the same reasons.>
C 2H4 will give Raman spectrum and infrared spectrum (only for assymetry stretching
and bending).
33. What are Stokes and anti-Stokes lines in Raman spectrum? Why are Stokes lines
more intense than anti-Stokes lines?
Ans. : Radiation scattered by molecules with a frequency lower than the frequency
(V) of incident radiation gives rise to Stokes lines. If v is the frequency of incident radiation,
then Stokes lines appear with the frequency vm ~m < v), where (v - vm) corresponds
to the energy absorbed by molecules to go to higher energy state from lower one.
Radiation scattered by molecule with a frequency higher than the frequency (v) of
the incident radiation gives rise to anti-Stokes lines. If v is the frequency of incident
radiation, then anti-Stokes lines appear with frequencyvm(v m> v)where(v m- v) corresponds
to energy absorbed by molecules to come down from higher energy level to lower one.
Spectral intensity depends on the popualtion of the molecules of the energy level from
which transition occurs. Population of lower energy level is greater than that of higher
one at normal temperatures. So Stokes lines are more intense than the anti-Stokes lines.
34. Deduce the expression for the possible transition energies associated with rotation
vibration spectra of a diatomic molecule. Hence, show that the vibration-rotation spectra
'
SPECTROSCOPY AND SOME PHYSICAL PROPERTIES 573

would consist of lines with equal spacing on either side of band origin. [C.U.(H) 2000]
Ans. : Vibrational transition from n = 0 state to n = 1 state is accompanied by rotational
transition. Let the rotational level in n = 0 state be designated by J and in n = 1 state
by J'.
Now, rotational-vibrational energy of diatomic molecule (using simple harmonic
oscillator and rigid-rotator approximation)

= Erot,vib =BJ (1 + 1) + ( n+~)vo,


where v= 0
vibrational frequency in cm- 1

B = rotational constant =
·
-!}-
8n k
cm-1.

(h =Planck constant, I= moment of inertia= µr 2, µ =reduced mass, r =:bond length,


c = velocity of light)
So, transition energy in rotation-vibration spectrum of diatomic molecule =
!1£ = £1'.n=I -£1,n=O

= B/ (J' + 1) + 2v 0
- BJ(J + 1) - _!_ v0
2 2
= v0 + B(J' - J) (J' + J + 1) (assuming B remains the same for all the vibrational
levels).
The rotational selection rule is !1 J = ± 1.
So, when !1 J = + l, J' - J = l or, J' = J + 1,
then !1£ = v 0
+ 28 J', where J' = l, 2, 3, .. ., etc. (I)
When !1 J = -1, J' - J = -1, : . J = J' + 1,
t.hen !1£ = ii0 - 28 J, where J = l, 2, 3, .. ., etc. (2)
Combining (1) and (2),
!1£ = v0 + 2Bm, where m =± l, ± 2, ± 3, .. ., etc.
The frequency v
0 is called band origin which remains absent in the spectrum.
Lines to high frequency region (on the right side of band origin) correspond to +ve
values of m and lines to low frequency region (on the left side of band origin) corresponds
to -ve values of m. So rotational vibrational spectra consists· of equally spaced lines with
the separation of 2B between them on either side of the band origin.

v0 v(cJT1-1)
35. Show that the lines in the rotation spectrum of a diatomic molecule are equispaced
under rigid rotator approximation. [B.U.(H) 2004]
Ans. : Under rigid rotator approximation, rotational energy of diatomic molecule,
£rot = BJ (1 + 1) cm-1, J = 0, l, 2,

where B h . 1
= Sn 2 le = rotattona constant..
(h = Planck constant, I .= motnent of inertia, c =. speed of
. light)
J = rotational quiJntum number.
574 PROBLEMS ON PHYSICAL CHEMISTRY

Now, under this approximation selection rule for rotational transition is ~ J =± I.


So, for the transition J = 0 ~ J = I,
transition energy, = 2B cm-I
~E

and for the transition J = I ~ J = 2,


~E = 6B - 2B = 4B cm-I, and so on.

So the lines in rotation spectrum are equally spaced with the separation of 2B cm- 1
in between them
36. State Franck-Condon principle.
Ans. : Franck-Condon principle states that an electronic transition takes place so rapidly
in comparison with the nuclear motion that nuclei maintain their initial position and
momentum just after the transition. It happens as the nuclei are much heavier than the
electrons. According to this principle electronic transition from lower electronic state to
higher one takes place vertically keeping internuclear distance and nuclear momentum intact.
37. Why are the spectral lines broad?
Ans. : Three factors contribute to the broadening of the spectral line, namely,
(i) Doppler broadening, (ii) Collisional broadening, (iii) life-time broadening.
(i) Doppler broadening : Molecules emitting radiation are in ceaseless random motion.
So there will be Doppler shift of frequency of radiation. ~f the molecules approach observer,
then the frequency will shift towards higher region and if molecules move away from
observer, it will shift towards lower region.
(ii) Collisional broadening : It arises from the collision between the molecules, or
between the molecules and the container. If the mean time between the collisi<?ns is Meo/,
1
then the resulting collisional broadening is Meo/ "" / •
2ulco/
(iii) Life-time broadening : Uncertainty principles is responsible for this broadening.
If life-time of the molecule in excited state is flt, then the resulting broadening is
n
tlE =
2M.
38. Determine various degrees of freedom of HCl, CH3CI, C 6H 6, H 2CO, C02 , NH3 ,
CH4, HzO.
Ans. : Total number of degrees of freedom of a molecule consisting of N atoms is
3N. Of these 3N degrees of freedom, 3 degrees of freedom are for translation, 2 (for linear)
or 3 (for non-linear) for rotation and the rest constitutes vibrational degrees of freedom.
Molecule Translational Rotational Vibrational Total
HCI 3 2 I 6
CH3CI 3 3 9 15
C6H6 3 3 30 36
H2CO 3 3 6 12
c~ 3 2 4 9
NH3 3 3 6 12
CH4 3 3 9 15
HzO 3 3 3 9
39. Which of the following molecules are microwave active-H2, HCI, CH 4, CH 3I,
H 20, SF6 ?
Ans. : HCI, CH3I, H 20 are microwave active, since they have permanent dipole moment.
SPECTROSCOPY AND SOME PHYSICAL PROPERTIES 575

40. Why is laser used in Raman spectrometer?


Ans. : Laser is used as a source of radiation in Raman spectrometer to irradiate the
sample, since intensity of scattered radiation is very low and the shifts, measured in
frequency, of scattered radiation from incident radiation are quite small. So, highly intense
monochromatic radiation from laser is required to make the shifts detectable.
41. Write the differences between harmonic and anharmonic oscillator.
Solo. :
Harmonic Oscillator Anharmonic Oscillator
1. Energy of harmonic oscillator 1. Energy of anharmonic oscillator

En= (n+±)hv, n= 1,2,3, .. ., etc., En= (n+±)hve-(n+±Y hv~e


where v = frequency of vibration where xe = anharmonicity constant.

= 211t ff·
n = vibrational. quantum number.
2. Energy levels are equispaced. 2. Separation between successive energy
En+\ - En= hv levels decreases as n increases.
En+\ - En = hve(l-2x/n+l))
3. Selection rule for transition is 3. Selection rule for transition is
/),. n = ±1. /).n = ±1, ±2, ±3, .. ., etc.

Numerical Problems
1. Calculate the specific rotation of sucrose if a 20% solution of it when placed in
a 2-decimetre polarimeter tube shows an angle of rotation of 26.4°. [C.U.(H) 1994]

26
Solo. : Specific rotation of sucrose = [a]1 = IOOaobs = lOOx .4 = 66°.
D IC 2x20
2. The dipole moment of water molecule is 1.84 D. Calculate the 0-H bond moment,
the bond angle being 105°. [C.U.(H) 2001]
Estimate the % ioric character of the bond, if 0-H length is 96 pm.
Solo. : Let µO-H be the bond moment of OH bond.
0

H
A 9=105° H

= 2µ0H COS~ = 2µ0H COS 52.5°


2
:. µ0-H = 1.51 D.
If H 20 is l 00% ionic, then bond moment of OH will be
µOH ion = 4.8 X 10- 10 X 96 x 10-10 esu.cm
= 460.8 x 10-20 esu.cm (10- 18 esu.cm = lD)
= 4.608 D.
576 PROBLEMS ON PHYSICAL CHEMISTRY

H ence, m ' ·
70 1omc
1. 478
ch aracter ·.= - - x 100 = 32.0701.70.
4.608
. 3. Dipole moment for HCI molecule is 1.03 D and internuclear separation is I .275 A.
Calculate the % ionic character of the bond. [Pune B.Sc. 1986]
Soln. : µHCI, ion = 4.8 x 10-10 x 1.275 x 10-S esu.cm
= 6.12 x 10-18 esu.cm = 6. 12 D.
3
Hence, % ionic character = µobs =
1.o x100 = 16.83%.
x 100
µion 6. 12
4.-Dielectric constant of a liquid of molecular weight II2 is 4.288. Its refractive index
is 1.348 and its density is 1.108 gm/cc at 25 °C. Calculate molar polarization and molar
refraction.
Soln. : D = 4.288, n = 1.348; M = ll2, r = 1.108 gm/cc.

Hence, molar polarization =P = D-IM


m D+2 p

= (4.288-1) ~
( 4.288 + 2). 1.108
cc. mole- 1 = 52.86 cc. mole-I.

= (I.(I. 348)2) +- Ix~


2 2
Moar
I f . R n - I .M
re ract1on = m = n 2 + 2 p l.I0
= 21.64 cc. mole-I.
348 2 8
5. From the data gi~en in the last problem, determine dipole moment of the compound
assuming atomic polarization is 5% of electronic polarization.
Soln. : Electronic polarization = Pe = Rm =
21.64 cc.moJ-1.

Atomic polarization = Pa = _2_ x P. = 1.082 cc.moI-I.


100
Molar polarization = Pm = 52.86 cc.mol-1
NowPm=Pa+Pe+P0
47tNAµ2
P0 = orientation polarization = kT
9
Thus, 52.86 cc.mo1-1

_ · _ _ 4xxx6.022xJ0 23 xµ 2 (esu.cm) 2
- 1.0 82 cc. mo 1 1 + 21 .64 cc. mo 1 1 + ( 7 1 1
x 8.314~!0 erg.K- .mole- x . K
9 298 150
6. 022 x 10 23 .

:. µ = 1.21. x 10-18 esu.cm = 1.21 D.


6. Dipole moments of C 6H 5N02 , C 6H 5Cl and p-nitrotoluene are 3.93 D, 1.55 D and
4.39 D respectively. Predict the dipole moments of m-nitrotolune, o-nitrotolune, ortho- and
meta-dichlorobenzene and o-dinitrobenzene.
Soln. : Given,

©
N02 Cl

µ" 3.930 µ = 4.39D


© µ = l.55D
SPECTROSCOPY AND SOME PHYSICAL PROPERTIES 577

©.
CH3

Dipole moment of tolune = 4.39 D - 3.93 D = 0.46 D.

Hence, bond moments of C-Cl, C-N02, C-CH3 are 1.55 D, 3.93 D and 0.46 D
respectively.

(a) µ

= -~(0.46) 2 +(3.93) 2 +2x0.46x3.93xcosl20°


= 3.72 D.

~
=(0.46) 2 +(3.93) 2 +2x0.46x3.93xcos60° = 4.18 D.'

©rCI
Cl

(c)

=2 µC-CI cos 30° = 2 x 1.55 x cos 30° D = 2.68 D.

Cl

(d)
©-Cl µ = 2>Jc-ei cos (120°/2) = 2µc-e 1cos 60° = l.55 D.

N02

(e) ©rN~ µ = 2 >'c-No2cos (60°/2) = 2µc-NOz cos 30°

= 2 x 3.93 x cos 30° D = 6.806 D.

Prob. Phy. Chem.-37


578 PROBLEMS ON PHYSJCAL CHEMISTRY

7. In the pure rotational spectrum of CO, ii0 -4 1 = 3.84235 cm-1. Calculate the
internuclear distance of the molecule, if µ co = 11.38365 x 11-27 kg? [C.U.(H) 2003]
Solo. : Given, 2B = 3.84235 cm- . 1
.·. B =-1.921175 cm-1.
h h
Now,B
= 87t2le = 87t 2µr 2c
h
or, ,.:i =
6.626 x 10-34 J.sec
= 8 x 7t 2 x 11.38365 x 10-27 kg x 2. 9979 x 10 8 m I sec x I. 921175 x l 00 cm -I
= 1.28 x 10-20 m.
r = 1.13 x 10-10 m = 1.13 A.
8. The fundamental and first overtone transition of NO molecule are ceptred at 1876
•cm-I and 3724 cm-1 respectively. ·
Evaluate (i) the equilibrium vibrational frequency, (ii) the exact zero point energy.
[C.U.(H) 2003]

Solo. : Vibration energy/= En = [ ( n + ~ )ve -( n + ~ J 1


vexe] cm- ,

where n = vibrational quantum number,


ii = equilibrium vibrational frequency in cm-1
e
x, = anharmonicity constant.
(i) Given, for transition from n = 0 to n = l state,

&0--41= 1876 cm-1 = iie -2iiexe. (I)


For tansition from n = 0 to n = 2 state,
&o--42 = 3724 cm- 1 = 2iie-6iiexe. (2)
Multiplying equation (I) by 3, we get
5628 cm-I = 3iie - 6v.x.. (3)
Subtracting equation (2) from equation (3 ), we get iie = 1904 cm- 1.
Hence, equilibrium vibrational frequency = 1904 cm-1 •
Now, from equation ( l ),
1876 cm-I = Ve (l - 2xe)

or, I
_ 2x = 1876cm-1 = 0.9853.
e l904cm- 1
•• Xe = 0.0074.
(ii) The exact zero point energy
l _ I_ 1904 _1 1904x0.0074 _1
=. En = 0 = 2v· -4v.x. = -2-cm - 4 cm

= 948.48 cm-1•
SPECTROSCOPY AND SOME PHYSICAL PROPERTIES 579

9. Chlorobenzene has µ = 1.55 D and nitrobenzene µ = 3.80 D. Estimate the dipole


moments of meta-dinitrobenzene, ortho-dichlorobenzene and meta-chloronitrobenzene. The
observed moments are 3.90 D, 2.25 D and 3.40 D. How would you explain any discrepancy?
[B.U. 1990]
Ans.. : Bond moments of C-Cl and C-N0 2 are l.55 D and 3.8'0 D respectively.

Hence, µ = 2µC-NOz cos (120°/2) = 3.8 D.

©rc1
Cl

µ = 2µC-Cl cos (60°/2) = 2.68 D.

= ~(1.55) 2 +{3. 8) 2 + 2 x l. 55x3.8xcos120° D = 3.3 lD.


For o-dichlorobenzene calculated value differs much from the observed value. This
is due to 'ortho effect' which arises due to close proximity of two groups introducing
repulsion which thereby increases the bond angle subtended by two groups.
10. The dipole moment of HCl is 1.03 D and its bond length is 1.27 A. Calculate
(i) the charge on the constituent atoms and (ii) the percentage of ionic character.
[B.U. 1987; V.U. 1996]
Solo. : (i) Dipole moment of HCl =charge on the constituent atom (Oe) x bond length
.. l.03 D = Be x l.27 x 10-s cm

Th f
ere ore,
oe =
1.03x10-1sesu.cm
1. 27 x 10-8 cm
= 8.11x10-11 esu.

(ii) See Problem no. 3.


11. A compound AB having partly ionic and partly covalent characters has the dipole
moment 1.03 debye. Show how the per cent ionic character may be calculated on the basis
of charge (4.8 x 10-10 esu) separated by equilibrium bond length 1.3 A. [B.U. 1985]
Hints : If AB is completely ionic, then µcal = 4.8 x l 0-10 esu x 1.3 A.

% of ionic character = µcal x 100 = 16.51 %.


µobs
12. Suppose there are two energy levels and the difference between them is 2kT.
Calculate the ratio of number of molecules in the two levels. [B.U. 2001]
Soln. : Let N; and Ni be the number of molecules in the two levels i and j.
580 PROBLEMS ON PHYSICAL CHEMISTRY

Ej - Ei = 2 kT
N1· -- e-Ej/kT -- e-(cj-c;)kr -2 - 0135
· --- e - ..
N; e-c;fkT
13. The bond angle in H 2S is 97°, while its dipole moment is 0.95 D. Calculate
S-H bond moment. [V.U.(H) 2002]
Soln. : Let µs..H be the bond moment of S-H bond.

:. 0.95 D = 2µ5-H COS~, µS-H = 0.7168 D.


2
14. The spacing between rotational lines in the IR spectrum of CO is 3.86 cm-1.
Calculate the bond length of C-0 bond. [V.U.(H) 2002]
Soln. : Spacing between rotational lines in IR spectrum = 2B, where B = rotational
constant. ·
Given, 2B = 3 .86 cm-1.,.
. . B = 1.93 cm-1

B. = _h_ = 1.93 cm-1


87t 2 /c
I = µr2
23 2
(12xl6)/(6.022xl0 ) 12xl6
µ = - (12+16)/(6.022xl0 23
) = 28x6.022xl0
23

= l.139 x 1()723 gm= 1.139 x lQ-26 kg .

.. r = ~ = l.13 x 10-10 m = 1.13A.


~Sr?µdi
15. The molar polarization of a substance is 46 cm3 .mol-1 at 25 °C. Calculate the dipole
moment of the m"olecule if the distortion polarizability 1s 1.808 x 10-23 cm3 .
[V.U.(H) 2001]
Solo. : Molar polarization= Pm = distortion polarization + orientation polarization
=pd+ Po.
Given,
Pm = 46 cm3.moJ-1, T = 273.15 + 25 = 298.15 °K.

Pa = t7tNA ad, ad = l.808 ~ 10-23 cm 3.

= '1-x 7t x 6.022 x 1()23 x l.808 x I0-23 = 45.61 cm3.mo1-1.

P0 =Pm - Pa= 0.39 cm 3.mol- 1.

po = 4~µ2
9kT

µ = ~A . = _3_ /P 0 xRxT
l"4ltN; 2 X NA ~ 7t

3 0. 39 x 8. 314 x I 0 7 x 298.15
= 23
2 x 6. 022 x 10 7t
19
= 1.38 x IO - esu.cm = 0.138 D.

SPECTROSCOPY AND SOME PHYSICAL PROPERTIES 581

16. At 27 °C 1.1 (M) lactic acid (C 3H60 3) solution taken in a polarimeter tube of
10 cm length rotates the plane polarized light by +2.3°. Calculate the molar rotation of
lactic acid. [C.U. 1996)
Solo. : Molecular weight of C 3H60 3 = M = 90 gm
C = 1.1 (M) lactic acid = 99 gm lactic acid/1000 cc, l = ldm,cx0 bs =. +2.3°.
[cxK = lOOcx 0 bs _ 100x2.3 = 23 .230
l. c l x 9.9

[M]~ = [cxK x M = 23.23x 90 = 20.91 0.


100 100
17. The refractive index of benzene at 298 K for light of wave length 600 nm is
1.498. The density of benzene is 0.844 x HP kg.m-3. Estimate the polarization of benzene.
[V.U.(H) 1999)
Solo. : Benzene is non-polar molecule. So D = n2 and P0 = 0.
n 2 -1 M
Hence, Pe =_n 2 +
2
.P
n = 1.498, M = 78 x 10-3 kg
p = 0.844 x 103 kg.m-3.
Neglecting atomic polarization (P0 ) since it is negligibly small,

l.4.98 2 -1 78x10-3 l
P - P - m3.mo e-1
e - m - l.498 2 +2 0.844xl0 3
= 2.71 x 10-5 m3.mole-1 = 27.1 cm3.mole-1 •
18. The rotational spectrum of HCI has lines 21. l cm-1 apart. Calculate the moment
of inertia and bond length of HCl. [V.U.(H) 1995)

Hints : 2B = 21.l ·cm-1, B = _h_


2
cm-1
87t /c
I = moment of inertia,
/ = µ,:Z, µ = m1m2 , r = bond length
m1 +m2
I = 2.653 x 10-40 gm.cm 2 , r = 1.28 A.
19. The dipole moment of H20 is 1.84 debye. Calculate H-OH bond angle, given
HO bond moment = 1.5 D. [C.U. 1986)
Solo. : Let the H-OH bond angle be 0.

Hence, l.84 D = 2 x 1.5 D x cos ~. :. 0 = 104.34°.


2
20. The refractive index of gaseous normal alkane C"H2" +2 is 1.00139 at STP. If atomic
refractions for H and C are 1.10 cc/mole and 2.42 cc/mole respectively, find the molecular
formula of alkane. [C.U. 1984)
2
Solo. : Molar refraction Rm = n - 1 M
n2 + 2 · p

Now at STP, molar volume = M = 22400 cc.


p
582 PROBLEMS ON PHYSICAL CHEMISTRY

2
R - (1.00l 39 ) - l x 22400 = 20.75 cc/mole.
m - (1.00139) 2 +2
Now atomic refractions are additive and are added to make the whole of molar
refraction.
Hence, 20.75 = 2.42 n + l.10 (2n + 2)
or,' 4.62 n = 18.55, :. n = 4.02 == 4.
Hence, molecular formula of alkane CnHzn+Z is C 4H 10 •
21. The density of SiHBr3 (mol. wt. 269) is 2.690 gm/cc. At 25 °C its refractive index
is 1.578 and dielectric constant is 3.570. Estimate its dipole moment in debyes (neglecting
atomic polarization). [C.U. 1999]
Hints : Pm = Pe + Pa + P0

pa = 0, Pe = Rm = n2 -1 x M
n 2 +2 p
D-l M
--x
D+2 p

p = 4 µ2
o )Tr.NA X 3kT
µ = 0.79 D.
22. The dipole moments of HCI, HBr and m
are 3.44. 2.64 and 1.00 respectively
in J0-30 coulomb-metre unit. Express them in debye. If equilibrium internuclear distances
are 127 pm. 141 pm and 161 pm respectively, find the fractional charges on the halogen
atoms and examine whether they are in order of the electronegativities of the halogens.
[B.U. 1999]
Solo. : l coulomb = 3 x 109 esu
So, µHCI = 3.44 x 10-30, coulomb.metre x 3 x 109 esu/coulomb x 102 cm/metre
= 1.032 x 10-18 esu.cm = 1.032 debye
µHBr = 2.64 x 10-30 x 3 x 1011 = 0.792 x 10-1 8 esu.cm = 0.792 deby~.
µHI = l x 10-30 x 3 x 10 11 = 3 x 10 -l9 esu.cm = 0.3 debye.
If r is internuclear distance and Oe is fractional charges on atoms, then
µ=oexr.
18
For HCI, r = 127 pm, .. oe = l.032x10- esu.cm = 8.12 x 10-11 esu.
127x10- 10 cm
19
For HBr, r = 2.64 pm, .. oe = 7. 92 x 10- esu.cm = 5.62 x 10-11 esu.
141x10- 10 cm

For HI, r = 161 pm, :. oe


= 3x10-19 esu.cm = 1.86 x 10-11 esu.
161x10- 10 cm
They are in the order of decreasing electronegativities of halogens.
23. Given atomic masses ofH = 1.0078 amu, Cl= 34.98 amu and the mean internuclear
distance of H-CI is 1.275 A. Calculate the reduced mass and moment of inertia of HCI.
[Pune B.Sc. 1985]
SPECTROSCOPY AND SOME PHYSICAl: PROPERTIES 583

Soln. : Reduced mass, µ = m1 m2 = (M1M2)/N1


(m1 +m2) (M1 +M2)NA
3
= 1. 0078 x 34. 98 x 10- k = 1.626 x 10-27 k .
(1.0078+34.98)x6.022xl0 23 g g

Moment of inertia, I = µ r1 (r = bond length)


= l.626 x 10-2 7 kg x (l.275 x 10-10 m)2
= 2.644 x 10-47 kg. m2.
24. The rotational spectrum of HF has lines 41.9 cm-1 apart. Calculate the moment
of inertia and bond length in HF. [Delhi B.Sc.(H) 1976]
Soln. : Given, 28 = 41.9 cm-1 (B = rotational constant)
B = 20.95 cm--1

Now, B = _h_2
cm--1.
87t /c
. . 6.626x10-- 34 J.sec
. . moment o f mertla, / = 2
8 x 7t x 20. 95 cm-- 1 x 2. 9979 x 10 10 cm I sec
= 1.34 x lo-4 7 kg.m2.

Now, reduced mass, µ = (M1M2)/N1


(M1 +M2)NA
19x10-3 k
= 20 x 6. 022 x 1023 g = l.578 x 10--27 kg.

Hence, bond length, r = ff


7 2
= 1.34x10--4 kg. m = 9.21 x 10 -11 m = 0.92 A.
1. 578 x 10--27 kg
25. Calculate the ratio of molecules in rotational level J = 2 relative to J = 0 for
a molecule with B = 1.2 cm-- 1 at 300 K.

Soln.. N1=2 e--E1=2fkT


• Nl=O = e--E1=ofkT
.
N1=2 = population at J =2 level
N1=0 = population at J = 0 level
E1=2 = BJ (J + 1) = 68 cm--1, E1 = 0 =0

or, N1=2 = e --(El=z-El=o)/kr = e--6Bhc/kT


Nl=O
6xl.2cm- 1 x2.9979x10 10 cm/secx6.626x10--34 J .sec
300 Kxl.38066xl0--23 J.K-1
=e = 0.966.
But Jth rotational level is 2J + 1 degenerate, Thus, considering degeneracy

N1=2 = 5 x e --(E1=z-E1=0)/kr = 4.83.


Nl=O
584 PROBLEMS ON PHYSICAL CHEMISTRY

26. Spacing between lines in microwave spectra of H1 1127 is 13.2 cm-1 . Calculate
the bond length of HI · [V.U. 1994]
Soln. : Given, 2B = 13.2 cm-1, :. B = 6.6 cm-1.

Hence, moment of inertia I = h. cm-1


81t 2Bc
6.626x 10-34 J.sec
=
8 x 7t 2 x 6.6cm- 1 x2. 9979 x 10 10 cm.sec- 1
= 4.241 x lQ-47 kg.m2.
127x10- 3
[µ = M 1 M2 /N~
= 128 x 6. 022 x 10 23 kg
(M1 +M2)/NA
= 1.648 x 10-27 kg]

v-µfI
Thus, bond length r = 4.241x10-47 kgm 2
=
. I. 648 x 10-27 k~
= 1.604 x 10-10 m = 1.604 A.
27. C-H stretching vibration in organic; compounds occur near 2900 cm-1. Appro-
ximately what wave number would be the C-D stretching vibration? [B.U.(H) 1992]

Soln. : ii = -·1-
27tC vµfI cm-1 = 2900 cm-1 .

Reduced mass of C-H,


12x10-3
µ = 23 kg = 1.53 x 10-27 kg.
13x6.022X 10

= 47t2 x 1.53 x 10-27 kg x (2.9979 x 1010 cm/sec x 2900 cm-1) 2


= 456.512 kg. sec-2.
Now, reduced mass of C-D,
3
µ' = 12 x 2 x 10- kg = 2.846 x 10 - 27 kg.
23
(l2+2)x6.022x 10
_2 I fr 1 456.512 kg.sec-2 _
212624
_1
v = 27tc~µ' =27tx2.9979xl0 10 cm.sec- 1 2.846xllo-27 kg - • cm·
28. Absorption JT\axima in the I.R. spectra of H 1Br 80 was observed as 37.8 nm.
Calculate the force constant of the bond between HBr.
80xlxI0-3 k
Soln. : Reduced mass of HBr, µ
= {80+ l)x 6.022x10 23 g
= 1.64 x 10 - 27 kg.

A, = c = 37.8 nm = 37.8 x 10-9 m.


v
8 1
v = 2.9979xl0 m.sec- = 7.93 x 101s sec-I.
37.8x 10-9 m
SPECTROSCOPY AND SOME PHYSICAL PROPERTIES 585

7.93 x 1015 sec- 1 = -21-7t 1. 64 x 10-


k 27
kg
.

:. force constant k = 47t 2 X (1.93 x 1015 sec-1)2 x l.64x J0-27 kg= 4.1x106 N.m-1.
29. In I.R. spectrum of H 1CJ35, the centre of fundamental band has a frequency of
8.6 x 1013 Hz. Calculate force constant.

Soln. : Reduced mass of HCI, µ = (


,
) 35x I
35+ I x6.022x 10-·
01 gm = 1.614 x I0-
24 gm.

force consant, k = 47t2v2µ = 47t 2 x (8.6 x 1013 sec-1)2 x 1.614 x I 0- 24 gm


• = 4.7 x 105 dyne.cm-1.
.. 30. The vibrational Raman spectrum of chlorine molecule shows a series of Stokes
lines separated by 0.98 cm-1 and a similar series of anti-Stokes lines. What is the bond
length of CI 2? (At. wt. = 35)
Soln. : Stokes lines and anti-Stokes lines are separated by 48 cm- 1.
Given, 4B = 0.98 cm-1.
B = 0.245 cm-1.

3
Reduced mass of CI 2 , µ = 35 x 35 x I 0- = 2. 91 x 10-26 kg.
(35+35)x6.022x 10 23
h h
Now, B =- 2- =
87t /c 2
87t µr 2 c ·

Hence, bond length r = ~


f8728~
6.626 x 10-34 J.sec
= 87t 2 x 0. 245 cm- 1 x 2. 91x10-26 kg x 2. 9979 x 10 10 cm.sec- 1

= 1.98 x io-10 m = 1.98 A.


31. What is the value of J in the most highly populated rotational level of ICI
(B = 0.114 cm-1) and CH4 (B = 5.24 cm- 1) at room temperature?
Soln. : Jmax for ICI

1.38066 x 10-23 J. K- 1 x 298.15 K I


= 2
2.0114 cm- x6.626x10-34 J.sec x 2. 9979 x 10 10 cm.sec- 1
1

1.38066 x 10-23 J. K- 1 x 298.15 K _ _!_ = 4.


=
2 x 5.24cm- x 6.626 x 10-34 J.secx 2. 9979 x 10 10 cm. sec- 1
1
2
32. Using 435.8 nm lines mercury as a source of radiation a Raman line was observed
at 444.7 nm. What is the Raman shift?
586 PROBLEMS ON PHYSICAL CHEMISTRY

Solo. : A.source = 435.8 nm = 435.8 x 10-7 cm ..


V source = (Asource)2 = 22946.31 cm-I.
vemitted = 444.7 nm = 444.7 x 10-7 cm.
V emitted = (Aemitted)-1 = 22487.07 cm-I.
Hence, Raman shift·= vsource - vemitted = 459.24 cm-1.
33. Irradiation of CCl 4 with 436 nm lines of mercury line gave lines at 439.9 run,
441.8 nm, 444.6 run and 450.7 nm respectively in the Raman spectrum. Calculate the shifts
in cm-1.
Solo. : A.source = 436 run = 436 x 10-7 cm.
V source = (Asource)-I = 22935.78 cm-1.
A1 = 439.9 nm = 439.9 x 10-7 .cm.
v1 = 22732.44 cm-1
Ll V 1 = V source - V 1 = 203.34 cm-1•
Ai = 441.8 nm = 441.8 x 10-7 cm
dVz = v source - Yz = 301.1 cm-1.
A- 3 = 444.6 run = 444.6 x 10-7 cm
dV3 = v source - V3 = 443.65 cm-1.
A-4 = 450.7 nm = 450.7 x 10-7 cm
Llv 4 = v source - v4 = 748.07 cm-1.
34. The wave numbers recorded in the vibrational spectra of the following mole-
cules in their n =0 states are: HD 3627 cm-1, DCI 1990 cm-1, HCI 2885 cm-1 and D 2
2990 cm-1. Calculate the energy change of the reaction : DCI + HD = D2 + HCI.
Solo. : Total vibrational energy of D2 + HCI,

E = ~hc(v0z +vHc1)
= lx 6.626 x J0-34 I.sec x 2.9979 x 1010 cm.sec-1 x (2885 + 2990) cm-1
2
= 5.84 x I0-20 J.
Total vibrational energy of (DCI + HD),
E' - l
-2hc (- -
Y0z +vHCI
)

= l x 6.626 x 1Q-34 I.sec x 2.9979 x 1010 cm.sec-I x (3627 + l 990)cm-1


2
= 5.58 x 10-20 J.
Energy change of the reaction,
M = [(E' - E) x NA] J.moJe-1
= (5.84 - 5.58) x 10-20 J x 6.022 x J023 moJe-1 = 1.56 kJ.mole-1.
35. The hydrogen chloride rotati~nal constant B = I 0.59 cm-1 and fundamental and
first overtone vibration lines are shown at 2885 cm-1 and 5679 cm-1 respectively. Calculate
(i) rotational energy for the molecule at J = 2.
(ii) zero point energy of the molecule,
(iii) hot band wave numbers. [Delhi B.Sc.(H) 1982]
SPECTROSCOPY AND SOME PHYSICAL PROPERTIES 587

Solo. : (i) Rotational line is given by,


v = BJ (J + 1) cm-1 = 10.59 cm-1 x 2 x 3 = 63.54 cm-1•
(ii) Vibrational energy,

En= v.(n+~) - yexe(n+~)2cm- 1 .


Given, for the transition from n = 0 to n = 1,
dEo-+l = 2885 cm-1 = Ve - 2 \iexe (1)
and for the transition from n 0 to n 2, = =
dE -+ = 5679 cm-1 = 2\ie - 6xe\ie.
0 2
(2)
Multiplying equation (1) by 2, we get
5770 cm-1 = 2\ie -_4xe\ie. (3)
Subtracting equation (2) from equation (3), we get
2xe -v e = 91 cm- 1. (4)
Adding equation (4) and equation (1),
\ie = (2885 + 91) cm-1 = 2976 cm-1.

Thus, from equation (4), xe = 9lcm-l = 0.0153. (5)


2x2976cm- 1
Hence, zero point energy,

En=O = _!_ Ve
2
-(.!.)
2
2
\i e x e

= ~ve(l-~x,) = 2x2976cm-
1 1( . 1 .)
l-2x0.0153

= 1476.62 cm-1.
(iii) Hot bond wave numbers corresponding to transition from n = 1 to n = 2,
(6)
Either putting the values of v, and xe in (6) or subtracting (2) from (3), we obtain
dE 1-+ 2 = 2794 cm-1•
36. The C = 0 bond energy in acetone is 728 kJ.mol-1. Will a light of 250 nm be
able to break the bond ? [B.U. 1999]
Solo. : Energy of one photon of wavelength 250 nm,

E = h £ = 6.626xl0-34 J.secx2.9979x10 8 m.sec- 1 = 7.946 x 10-19 J.


/... 250x 10-9 m
Energy of one Einstein of photon = NAE = 6.022 x 1()23 x 7.946 x 10-.19 J.
= 478.51 kJ.
It is less than C =0 bond ~neregy. So the bond will not break.
37. The refractive index of CCI 4 for sodium D line at 20 °C is 1.457 and its density
is 1.595 gm/cc. Calculate the molar refraction. If the atomic refraction of carbon be 2.42,
calculate atomic refraction of chlorine. [C.U. 1981]
588 PROBLEMS ON PHYSICAL CHEMISTRY

2
Solo. : Molar refraction = Rm = nn2 +-12 pM
n = refractive index = l .457
M = molecular weight of CCl4 = 154 gm
p = density = 1.595 gm/cc.

Th R _ (1. 457}2 - l 154 gm


us, m - (1.457)2+2.l.595gm/cc = 26.29 cc.

Atomic refractions are additive, Thus,


26.29 = atomic refraction of carbon + 4 x atomic refraction of chlorine
= 2.42 + 4 R0
R0 = 5.97
cc.
38. Two optically active substances A and B have specific rotations of +50° and -30°
respectively. A mixture containing 6 gm A 'and 4 gm B is dissolved in water and the solution
is made up to 50 cc. Find the angle of rotation for the solution kept in a 5 cm polarimeter
tube. [C.U. 1985)
Solo. : If C is concentration in gm per 100 cc, l (dm) is length of the tube, and a

is angle of rotation, then specific rotation = [a] = IOOa.


IC
For A, [a] = +50°, l = 5 cm = 0.5 dm, CA = 12gm.
lOOcc
. _ lOOaA . _
. , 50 - , . . CXA - 30 .
0.5x12
8gm
For B [a]
'
= -30° ' C8 =- -
IOOcc

. _ 100a . _ o
.. -30 - - -8, .. CX8 - -1.2.
0.5x8
So, the angle of rotation for the solution = aA + a 8 = 3° - 1.2° = 1.8°.
39. What is the frequency of the rotational line shown by a diatomic molecule, having
the.moment of inertia value 16.5 x I(T40 gm. cm 2 , the excited molecule being in the quantum
state J = 2? [B.U. 1983]
Solo. : According to the selection rule of rotational transition !!J =± 1.
Here J for excited state is 2. Hence, transition occurred from J =l state.
Energy for J = l state = E = BJ (J +l)cm-1 = 2B cm-1.
1
For J = 2 state, £2 = 8.2 (2 + l) = 68 cm-1.
So, !iE = E2 - E1 = 4B cm-1.
Hence, wave number of frequency of the rotational line shown by a diatomic molecule
27
= 48 = ~ = 4x6.626xl0- erg.sec = 6 .79 cm-i.
2
8rt /c 8x rt x16.Sx l0'.:46gm.cm 2 x2.9979x10 10 cm I sec
2

40. The molar polarization of tluorobenzene vapour is 70.62 cm3.mole-1 at 351 K and
62.47 cm3.mole-1 at 423 K. Calculate the dipole moment of molecule. [V.U. 2000]
SPECTROSCOPY AND SOME PHYSICAL PROPERTIES 589

Solo. : Molar polarization, Pm =A + B. ·


T

A=

B=

B
Given, 70.62 cm3.mole-1 = A + (1)
351K
B
and 62.47 cm3.moJe-1 =A + (2)
423K
Subtracting (2) from ( 1), we get

8.15 cm3.mole-1 = B(-1 1


- - -- ) .
35 lK 423K
.. B = 16806.32 cm3.K.mole-1.
2
Thus, 16806.32 cm3.K.mole-1 = i rrN µ ·NA
3 A- 3R

µ2 = 9x16806.32cm 3 • K. mo1- 1 x8.314x10 7 erg. mo1- 1• K- 1


2
4X 7tX (6.022X10 23 )
= 2.76 x 10--36 esu2 .cm2 .
µ = 1.66 x 10--18 esu.cm· = 1.66 D.
41. Calculate the moment of inertia of CO molecule having isotopic combination
12 6
c o1 given rco = l.13 A. [B.U. 1987]

12x 16
Solo. : Reduced mass µco = = 1.14 x l Q-23 gm
(12+ 16) x 6.022x10 23
So moment of inertia = I = µr2
= l.14 x 10-23 gm x (l.13 x lQ-8)2 cm2
= 1.46 x lo-39 gm.cm 2 •
42. The fundamental vibrational frequency of HCl is 2885 cm-1. Assuming HCl and
DCl may be treated as simple harmonic oscillator, calculate the fundamental frequency
of DCl (atomic mass of chlorine = 35 dalton). [1.1.T., M.Sc. Admission Test]
Solo. : Reduced mass of HCl,
2
(l x 35)/(6.022x10 23 }
= = (1+35)/6.022xl0 23 gm

= 36x6.022x10
35 23 gm = 1.61 x l0-24 gm.

= · (2 x·35 ) gm = 3.14 x 10-24 gm.


(2 + 35) x 6.022x10 23
590 PROBLEMS ON PHYSICAL CHEMISTRY

= 2885 cm-1 = - -1~


--·
27tC µHCI

k = (2885 cm-1)2 x (2rtc) 2 µHCI.

_"oc1 =I- ~
- - , k = -2
Voc1X (2 7tC )2 xµDCI·
27tC µDCI
Thus,
2
(2885 cm-1)2 x (27tc) 2 µHCI =V6c1X{27te) xµDCI

or, v2 = (2885cm-')2 xl.6Ix10-24gm = (4267640.84 cm-1)2.


DCI 3. I4 X 10-24 gm
:. Voci = 2065.83 cm-1 •
43. Calculate the reduced mass of 11 27 Cl35. The bond length of the molecule is 2.32A.
Calculate its moment of inertia. [B.U. 1993]
Hints : I = µr2 = 2.45 x 10-38 gm.cm2.
See Problem no. 41.
44. How many times does a molecule of H 1Cl35 rotate per second in the J = I rotational
level? Given, B = 10.6 cm-1. [B.U. 1989]

Soln. : Classical rotational energy = .1..Joo 2.


2
Quantum mechanical rotational energy in J =I state = BJ (J + l) cm-1

= Bhcl (J + I) joule = 2Bhc joule (J = I)

Now, 1/co 2 = 2Bhc


or, co 2 = 4Bhc
r-
= 4Bhcx8rt 2 Bc [·: B = h .. I= --1!-J
h Rrt21r' Rn: 2 Br
= 32B rt c2
2 2

co = 4B1tc .J2 = 4 x I0.6 cm-1 x 7t x 2.9979 x 1010 cm/sec x ../2


=5.65 x 1012 radian.sec-1 =8.99 x 1011 revolutions.sec-1 (dividing by 27t).
45. A liquid of molecular weight I I 2 and density I. I 08 gm/cc. has D.E constant 4.29
and R.I I .348. Calculate its molar polarization and molar refraction. [C.U. 1986]

Hints : Pm = D- I M , (52.87 cc.mole-I)


D+2 p
2
R = n - I M (21.64 cc.mole-1).
m n2 +2 p
46. The refractive index of benzene at 298 K for light of wavelength 600 nm is I .498.
The density of benzene is 0.844 x I03 kg.m-3 . Estimate the polarizability of benzene.
[V.U. 1993]
Soln. : See Problem no. I 7.

Pm = 27. I cm3 .mole-1 = ~rt NA a [a = polarizability]


SPECTROSCOPY AND SOME PHYSICAL PROPERTIES 591

3 - I
a. =
27. lcm . mole- x 3 = 1.07 x I0-23 cm3.
4x1t x 6.022x10 23
47. The dipole moment of o-xylene is 0.693 D. Find the dipole moment of to lune.
What will be the value in SI units ? [B.U. 1993]

CH3
~CH3
Solo. : LSdJ µo-xylene = 0.693D.

Let µ be bond moment of C-CH 3.


.
Gtven, 60°
0.693 = 2µ cos T.
.. µ = 0.40D.
1 D = 10-18 esu.cm
1 C.m = 3 x 109 esu x 102 cm = 3 x 10 11 esu.cm.
10-18
1D = 3xl0 11 C.m = 3.33 x rn-30 coulomb.metre.

:. 0.4 D = 1.332 x 10-30 C.m ..


48. A liquid of mo!. wt. 18 and density 0.995 x 103 kg.m-3 has a dielectric constant
78.5 and refractive index 1.383. Calculate the values of its molar polarization, molar
refraction and dipole moment neglecting atomic polarization. [V.U. 1989]
Hints : Density = p = 0.995 x 103 kg.m-3 = 0.995 gm.cm-3
p
m
= D-1
~-
M
D+2 p

Rm ;,,, 1 M = Pe
n2 -
n+2 p
pm = po + Pe + pa [Pa = 0]
2
Po = Pm - Pe = j.7tNA ~T (T = 298.15 °K)
µ = 0.8 x 10 -ls esu.cm = 0.8 D.
49. How many revolutions does a molecule Na23c135 make in J =9 state? The rotational
constant of NaCl is 6500 MHz.

Solo. : Classical rotational energy = ~ /ro2.


Quantum mechanical energy = hBJ (J + 1).
Now, llro2 = hBJ (J + 1)
2
or, ro2 = 2hBJ(J + 1) = 2hBJ (J + 1) x 8rt 2 B h h
-h- [B = 8rt 2 / •• I = 8rt 2B]
I
= 16 B2rt2 J(J + 1) .

.. co =4rtB ~J(J+l) =4nB~9(9+1) [J=9]

= 4rt (6500 x 106 sec-1)-[90 .F 7.75 x 1011 radian.sec-1


= 1.23 x 1011 revolutions.sec -1.
592 PROBLEMS ON PHYSICAL CHEMISTRY

50. In the near I.R. spectrum of CO there is an intense band at 2144 cm-1• Calculate
(a) the fundamental vibration frequency of CO, (b) the period 9f vibration, (c) the force
constant, (d) the zero point energy in cal/mole. [B.U. 1990)
Solo. : (a) v= = 6.43 x 10I3 sec-t.
2144 cm- 1, v = \ic
13
(b) 6.43 x 10 vibrations per sec all executed in one sec. So time for one vibration
1
= period of vibration = = 1.55 x 10-I4 sec.
6.43x IO 13

I {I . 12xl6 23
(c) v = 2rt~µ· whereµ= (12+16)x6.022xl023 = l.139 x 10- gm.
k = 1.86 x 106 gm.sec-2 = 1.86 x 106 dyne.cm-I.

(d) ~;o point energy = =· thv ±x


6.626 x 10~ 34 J. sec x 6.43 x 1013 sec- 1
= 2.13 x 10-20 J = 5.09 x 10-21 cal [4.184 J = l cal]
Thus, zero point energy per mole
= 5.09 x 10-21 x 6.022 x 1023 = 3065.19 cal.mole-I.
51. K39 CI 35 has an intense line at 278.0 cm- 1, in far infrared spectrum. Calcu-
late the force constant and the period of vibration of K 39Cl 35.
Solo. : Force constant, k = 4rt 2 v2c2µKCI dyne.cth-1.

39x35
µKCI = {39 + 35) X 6. 022 X 1023 = 3.063 X J0-23 gm

v = 278.0 cm-1
k = 8.39 x 104 dyne.cm-t.
Period of vibration, T := _!_ = ~ = 1.19 x 10-13 sec.
V CV
52. Calculate the refractive index of a mixture of CH 30H and C 2H50H contain-
ing 0.75 mole fraction of CH30H. Given, ·
PcHJOH = 0.792 gm.cm- , 3 Pc HsOH = 0.789 gm.cm- 3
2
nCH30H = 1.328 nc2H50H l.36 l l =
Pmixture = 0.7903 gm.cm- 3
where p = density, n = refractive index.
2
n -l M
Solo. : Molar refraction for CH30H = RcH 3oH = n2 + p
2
2
= (1.3 28 ) -l x_E_ = 8.197 cm 3.
(1:328)2 + 2 0. 792
2
. . {l.3611) -l 46 902 3
Molar refraction for C 2H50H = Re H OH
2 5
= (l.3611) 2 +2 x - - = 12.
0.789
cm .

Molar refraction for solution = 0.75 x 8.197 + 0.25 x 12.902 = 9.37.


N ow, Rsol = -n22-I- x -M,01
·-
n +2 Pso1
SPECTROSCOPY AND SOME PHYSICAt; PROPERTIES 593

where n = refractive index of the solution.


2
n - l 0. 75 x 32+ 0. 25 x 46
Hence, 9.37 = .-
2
·- x
.n +2 0. 7903
n2 - l
or, ~
n +2
= 0.2085, :. n = 1.34.
53. A microwave spectrometer capable of operating only between 60 and 90 cm- 1
was used to observe rotational spectra of HI and DI. Absorptions were measured as follows
for three successive lines :
HI (cm- 1) DI (cm- 1)
64.275 65.070
77.130 71.577
89.985 78.084
Determine the J values between which transition occus for the first line listed above
for HI and DI and find the B values for each molecule. Hence, find the atomic wt. of
D. (At. wt. of I = 127, H = l). State the assumption used for calculation. [C.U. 2006)
Soln : Both HI and DI are assumed to be rigid rotor. Hence, distance between two
adjacent lines in the rotational spectra = 28.
By the problem, for HI molecule,
28 = (77.130 - 64.275) cm- 1
= (89.985 - 77.130) cm- 1 = 12.855 cm- 1. (1)
B = 6.4275 cm-1. (2)
For DI molecule,
28' = (71.577 - 65.070) cm- 1
= (78.084 - 71.577) cm-1
= 6.507 cm- 1 (3)
B' = 3.2535 cm- 1• (4)
Again, energy separation of two adjacent rotational levels = 28(1 + l ).
By the problem, for HI
28(1 + 1) = 64.275 cm-1
or, 12.855 cm- 1 (1 + 1) = 64.275 cm-1 [from Eq. (l)]
or, l + 1 =5
.. l = 4.
And for DI 6.507 (1 + l) cm- 1 = 65.07 [from Eq. (3)]
or, l + l = 10
.. l = 9.
Thus, for HI the first line appears due to the transition from l =4 ~ l =5 and
for DI, transition from l = 9 ~ l = 10 gives rise to the first line.

= (M1M1M2/Ni
+M2)/NA

127
= 128x6.022x10 23
24
gm = 1.648 x l o- gm.
594 PROBLEMS ON PHYSICAL CHEMISTRY

h
Now, B = ·81t 2µr l
C

2 6. 626.x 10-27 erg. sec


·· 'HI= 8xn 2 x6.4275cm- 1 xl.648xlo- 24 gmx2.9979xl0 10 cm.sec- 1
= 2.642 x 10- 16 cm 2.
Hence, rHI = 1.625 x 10-8 cm.
Bond length remains same within experimental error on isotopic substitution.
Rene, rHI = r 01 = 1.625 x 10-8 cm.

6. 626 x I 0-27 erg. sec


.. µDI = µ' = ~---~~--~~~~--..::'---~--~~~~-
8 x n 2 x 3.2535cm- 1 x(1.625x10-8 cm)2 x 2.9979x I0 10 cm.sec- 1

= 3.258 x 10-24 gm.


Let atomic weight of D be x gm.
_ __ 127x
T h US, 3 .25 8 X 10 24
(127 +x) x 6.022x10 23

127x
or, 1.962 = 127+x
I I
or, -+-
127 x
=- -
I. 962
x = 1.99 "" 2, i.e., atomic weight of D is 2 gm.
CHAPTER17

STATISTICAL THERMODYNAMICS

Required Formulae
1. If Ei and Ej are the energies of ith energy level with degeneracy gi and jth energy
level with degeneracy g respectively,
1
N -lk;
·_;=~ (1)
N ge-lkf'
1 . J

where Ni = population at ith energy level


Nj = population at jth energy level
1
f3 = - (k = Boltzmann constant, T = temperature)
kT

g.e- lk; -lk1


§J!__
and Ni=-'-- N
q ' J
= q
(2)
The above relations are known as Boltzmann relation,
where N = total numbers of molecules
q = partition function =Lg;e -lk; .
j

2. If n identical but distinguishable particles are arranged in i energy levels in such


a way that E1 energy level accomodates n 1, Ez accomodates n 2, Ei accomodates ni and
so on, the total distinct number of ways of realising such distribution,

W= n! =n~.
n 1 ! n2 !. .. n; ! ; n1 !

Short Questions
1. (a) Consider an isolated system containing N molecules, distributed among the
possible non-degenerate energy levels represented by E1, E2, .. ., etc. Assuming the expression
for thermodynamic probability, show that under equilibrium conditions number of molecules
(N) in the level E; is given by Ni = A exp (-bei), where A and b are constants.
595
596 PROBLEMS ON PHYSICAL CHEMISTRY

1
(b) Answer the following questions taking b = - - :
kT
(i) Show that Ni+I < Ni for any finite temperature.
(ii) O~tain A in terms of b and show that it is related to the partition function.
(iii) What is the probability that a molecule has an energy Ei? [C.U.(H) 2001]
Ans. : (a) Let N 1 molecules be in the energy level £ 1, N2 molecules in the level £ 2
and so on. · ·

LN; = N, i = I, 2, .. ., etc.

N!
So, weight of the configuration = thermodynamic probability = W =
N1!N2! ...
N!
or, In W = l n - - - - -
N1 !N2 !N3!...
= lnN/ -In (N 1! N2 ! N;! ... ) =In N/ - (In N 1! t In N2 ! + fn N3 ! + ... )

or, In W =In N! - LlnN;! i = I, 2, ... , etc.

Using Stirling's approximation


In W = (N In N - N) - L(
N; In N; - N;)

= NlnN- N_- LN;lnN;+N (since,LN;=N)


i

= N'InN - LN;lnN; = LN;lnN - LN;lnN;.


i i i
At equilibrium, the configuration which will dominate can be found out from the
maximum value of W which is a function of Ni. At the maximum value of W,
d In W = 0

or, -Ui Ni In Ni - 'LN/l In Ni = 0 (since L,N; = N = constant; L,dN; = 0)

:. Ui Ni In Ni= 0.
Maximum value of W is subject to two constants, namely,

l N;E; = E, where E = total energy


i

and LN; = N, i = I, 2, ... , etc.

Now, L dN; = 0 and L E;dN; = 0,

since E and N are constant.


STATISTICAL THERMODYNAMICS 597

Applying Lagrange's method of multipliers L (a+ bE; +In N,.) dN; =0, where a and
b are constants.
Ni are all independent.
So, a + bEi + In Ni =0 is true for every i.
Thus, In Ni =- a - bEi.
N.l = e-0 x e-bei =A e-bei (since e-a is constant).

l e·
_1
(b) (i) b = -. Thus, Ni = Ae kT
kT

_1
e kT is a monotonically decreasing function of energy.
_!i_
Number of molecules in E1 energy level at temperature T = N1 = Ae kT.
-e;+t
Number of molecules in Ei+I energy level at temperature T = Ni+I = Ae kT
Now, Ei < Ei+J·
Thus, e-e;fkT > e-e;+1/kT (k and Tare constants).

Hence, Ni > Ni+ 1 (Proved).

(ii)

or,

or,

Partition function = q =Le -be; .


N
.. A =
q
Ne-be;
(iii) Ni =
:r;e-he;
N; e-be;
or,
N
= probability that a molecule has an energy £ 1. =~
£./-be;

2. What is thermodynamic probability? [C.U.{H) 2002]


Ans. : Thermodynamic probability is the distinct number of ways of realising a
particular distribution.
598 PROBLEMS ON PHYSICAL CHEMISTRY

Let 4 particles (a, b, c, d) occupy 2 cells in the following way :


Cell I 3
Cell 2 1
This can only be done in 4 ways.
Cell 1 abc abd adc bed
Cell 2 d c b d
So thermodynamic probability of the above distribution is 4.
3. What is the condition for applying Boltzmann relation?
Ans. : Two following conditions should be ensured for applying Boltzmann relation :
(i) The system must be in thermal equilibrium at temperature T.
(ii) The system must contain fixed number of particles.
4. What is the significance of partition function?
A~s. : Partition function q = L g;e -Pe; ,
i
where gi = degeneracy of ith energy level,
E; = energy of ith energy level and

/3 = Lg;e-J3e;.
q describes how the total energy of the system is partitioned among several energy
states at temperature T.
At absolute zero, q = g0.
Thus, at T = 0. q becomes equal to degeneracy of ground state, i.e., all the molecules
come down to ground state.
As T increases, q also increases. So at high temperatures all the energy states of the
system are accessible to a molecule.
So, q also indicates the number of energy states that are thermally accessible to a
molecule at temperature T ·
5. Which state function of the system is related to the maximum value of
thermodynamic probability and how? [C.U. 2002]
Ans. : Entropy (S) is related to the maximum value of thermodynamic probability W.
S = k In W, k = Boltzmann constant.
If W of any distribution of some particles is .greater than the others, entropy will also
be greater for it. •
6. Show that the entropy is a logarithmic function of thermodynamic probability.
Ans. : Entropy, S, is the function of thermodynamic probability, W.
:. S =f(W).
Let a system having entropy Sand thermodynamic probability W be divided into two
non-interacting parts with entropies S 1 and S2 and the corresponding thermodynamic
probabilities are W1 and W2.
Then, S = S1 + S2 = f(W 1) + f(W2) = f(W) = f(W 1, W2)
f(W 1, W2) = f(W 1) + f(W2) suggests, that the function is logarithmic. -
Thus, S = a In W + b, where a and b are constants.
STATISTICAL THERMODYNAMICS 599
7. Show that the relation connection S of a system with the partition function Q is

given by S = Nk 8 In Q + E ,
T
where E = internal energy of the system,
k8 = Boltzmann constant
N = number of molecules in the system
T = temperature of the sy~tem. [C.U.(H) 2003]
N!
Ans. : If W is thermodynamic probability, then W = N N 1 N 1
,
1
I· 2· 3·

where N = N 1 + N2 + N3 + .. ., N 1, N2, N3, .. ., etc. are the number of molecules in


energy levels.
E1, E2, E3, ...

Now, entropy= S = k 8 In W = k8 (lnN!- f1=1


1nN;!l

= k 8 ( N In N - ~ N; In N;) [applying Stiding's apprnximation]

L
~ ~ N ~
= -Nk 8 -·- I n - - [since, according to Boltzmann relation, -1. = _e_]
Q Q N Q

Ler'£ie;
E
since j
= £ =
Q N
8. Consider a system of n molecules, distributed among non-degenerate energy levels
represented by Eo• E1, Ez· .. ., etc. Write down the expression for molecular partition function
for the system. Show that internal energy ( U) of a system can be expressed as

U = nkT2(olnQ) .
oT v
where k is the Boltzmann constant, T and V being the temperature and volume of the system
respectively. [C.U. 2002]

Solo. : Partition function, Q = Ie-[3£; i = 0, l, 2, .. ., etc.


600 PROBLEMS ON PHYSICAL CHEMISTRY

L/-f}e;E;
Average in. ternal energy, U = _.,~>-[le;
. ,.,;--- .
Now, ~:e-fle;e; = _I[ oe;:; )v = -( :p Q)v
f3 = kT

( ~~) v = - k~2 •

Thus, U = __!_(oQ) = kT2 (olnQ) .


Q ap v . ar v

:. U = n V = nkT2 (a oTQ) .
In
v
9. The molecules of a gas have two energy states, zero and £, and degeneracies 8i
and 8 2 , respectively. Write down the expression for molecular partition function.
Ans. : Molecular partition function, Q = 8i e-flei + 8 2 e-flez
Since, e 1 = 0, ti = £,

I
Q = 8 1 + 82e-{3£ (where f3 = - , k = Boltzmann
kT
constant, T = temperature)
10. What is ensemble?
Ans. : An ensemble is an imaginary collection of large number of systems each having
same macroscoping properties but different microscopic properties.
11. How do you deflne canonical ensemble, grand canonical ensemble and microcanonical
ensemble?
Ans. : Canonical ensemble : Each member of the imaginary collection of systems
has same N, V, T, where N is the number of particles of single component, V is volume
of each member, and T is temperature.
Grand canonical : Each member has same µ, V, T, where µ is chemical potential.
Microcanonical ensemble : Each member has the same N, V, E, where E is the energy
of each component.
12. If N molecules are distributed among the possible non-degenerate energy level
etc. in an isolated system, the· number of molecules N; in the level E; can be
£ 1, £ 2, .. .,

represented by N; =N i -[le;
e-fle; . Find the value of {3.

N! N!
Solo. : Thermodynamic probability =W = =
N1 !N2 !N3 ! LN;!.
STATISTICAL THERMODYNAMICS 601
Entropy, S = k lnW, ~ere k = Boltzmann's constant.
In W = NIn N - L N; In N; (applying Stirling's approximation)

N
= I.N,.lnN - """N,.
£.J
lnN,. =I.JV.In-' =-IN.In
• N '
Le-131:;
e- ;
131:

= fJE+ N In Q
(since, L N;E; = E = total energy of the system and LN; =. N)
i

Thus, S = k {JE + N k In Q.
Now, (aEas)v = k/3 + kE(aP)
aE v
+ Nk(alnQ)
aE v
= k/3 + kE(ap) + Nk(a1nQ) (aQ)(ap) .
aE v aQ v ap aE v
Now, Q = L e-lk; .
Hence, ~~ = - ~ E;elk; .

Thus, (as)
aE v
= k/3 + kE(ap) - Nk le-lk;e; (ap)
aE v Q aE v
= k/3 + kE(ap) ~ NkE·(aQ)
aE v ap
= k/3 (since NE = E). (l)
Again, TdS = dE + PdV.
.. (~!) = T (2)

Equating ( 1) and (2), f3 = -1


kT
13. Write down the vibrational partition function.

Ans. : Vibrational energy = En = ( n + ~) hv, where n = 0, 1, 2, ... , etc.

and v = - I ~
- = v1'b··.rat1ona
. I frequency,
21t µ •..
where k is force constant of the molecule and µ = reduced mass.
602 PROBLEMS ON PHYSICAL CHEMISTRY

Hence, ,yibiational partition function

~ ~ e-P(n+- )hv = e-~hv/2 k.Je-Phvn


~
1

""'e-Pt:n = """ 2

n=O n n=O
J e-f}hv/2 I
= e-/Jhv/2 (I +e-Phv + e-2/Jhv + ... ) = 1-e-f}hv
(since, I + x + x2 = --).
1-x
14. Derive Barometric distribution formula from Boltzmann energy distribution.
Ans. : If N particles are distributed among the possible non-degenerate energy levels
£ 1, £ 2, ... , ~tc, in such a way that N 1 particles are in £ 1, N2 particles are in Ez and so
on, the number of particles Ni in ith energy level at certain temperature T can be written
as under :

e -Pt:;
N; = N
(1)
N
Le-Pt:;
i=I

where i = I, 2, ... , N and f3 =


kT
Equation (I) is known as Boltzmann relation.
So if Ni and N/ are the number of particles in Ei and £ energy levels, then according
1
to Boltzmann relat10n,

N
_, -
-
e-PI:;
- -
Pt: -
- e -P(e;-eJ·) (2)
Nj e- J

Equation (2) can describe distribution of particles in atmosphere at temperature T,


where gravitational energy varies with altitude.
Let N 0 and Nh be the number of particles at the ground level and at altitude h

respectively at temperature T. Thus, Nh = e-P(eh-eo) = e-P Ile.


No
The difference of energy ~E represents the potential energy difference of the particles
at heiglit h with respect to ground. Thus, ~E ~ mgh, where m is mass of the particle and
g is gravitational force.
mgh
So, N = N e-/Jmgh = N
0 0
e-Tr. (3)
The above equation is known as barometric distribution formula.
15. Calculate the contribution of a vibrational degrees of freedom to the average energy
of a molecule. Comment on its value at high temperature.
.. . e-Phv/2
Ans. Vibrational part1t1on function, qvib = -~hv ,
1-e

. where P = k~ and energy of nth vibrational level = En = ( n + ~) hv, n = 0, I, 2.

Now for an assembly of N number of molecules,


STATISTICAL THERMODYNAMIC:S 603

( E. ) = NkT2 dlnqvib.
v1b dT
Here, N = 1, Thus,
= kT2 d In qvib
dT

d hv --
hv ] ]
- kT2- - - - - I n 1-e kT
- dT [ 2kT (

hv
-kT hV 2
hv 1 e . - 2 .kT
= kT2 -.-+ kT
2 kT2 hv
1-e kT

hv hv
= 2+ e~hv -1 ·
If kT » hv (at high temperature),

= -hv + kT ("• hv)


e.,.,v ==I+-
2 kT

= kT (since hv « kn.
2
16. State the difference between Maxwell-Boltzmann, Bose-Einstein and Fermi-Dirac
statistics.
Ans.

Maxwell-Boltzmann Bose-Einstein Fermi-Diroc


statistics statistics statistics

(a) Particles are distin- (a) Particles are industin- (a) Particles are indistin-
guishable guishable guishable
(b) Particles do not have (b) Particles have integral (b) Particles have half-
spin spin integral spin
(c) No symmetry require- (c) Wave function should be (c) Wave function should
ment is imposed on symmetric over inter- be antisymmetric over
the corresponding wave changing any two interchanging any two
function of the system particles particles.
over interchanging any
two particles.
(d) No restriction over (d) No restriction. (d) Energy state can acco-
occupancy of an modate only two
energy state particles with opposite
spin.
604 PROBLEMS ON PHYSICAL CHEMISTRY

Numerical Problems
1. (a) Consider 20 molecules divided equally between four non-degenerate energy
levels. What is the thermodynamic probability (W) for this distribution? (b) How does the
value of W change if one molecule is removed from one level and added to another?
[C.U.(H) 2001]
20
Solo. : (a) Number of molecules in each energy level = = 5.
4
Total number of molecules = 20.
W= ~2-0 -!- = 1.17 x 10 10•
5!5!5!5!
(b) If one molecule is removed from one level and added to another
2
W1 = 0! = 9.78 X 109..
5!5!6!4!
2. Suppose a molecule has two energy levels £ 1 = 0 and ~ = kT. Calculate (i) the
partition function and (ii) ratio of the number of molecules in the two levels [k =Boltzmann
constant and T is the temperature in KJ [C.U. 2001]
Solo. : (i) Partition function q = e-ei/kT + e-E2/kT =l + e-1 = 1.37.
(ii) Let N 1 and N2 be the number of molecules at £ 1 and ~ energy levels respectively.

N1
-
N2 - ~
e-ei/kT
= e-{E1-E2 )/kT = e .
(smce, e 1 = 0, P-
-'-
= k1)
= 2.72.
3. Consider a system of six distinguishable particles. One..of the macrostates has the
following distribution of particles :
Energy OE le 2£ 3£ 4£
No. of particles 0 .O 2 2 2
Calculate its themodynamic probability. [C.U. 2002]
6!
Soln. : Thermodynamic probability, W = 2!2!2! = 90.
4. Calculate the number of ways of arranging 5 different particles among 3 energy
ways levels such that one energy level has l particle, two have 2 each. [C.U. 2003]
Solo. : The total number of ways, W

- ~-30
- 1!2!2! - .
5. Calculate the difference in entropy between two macrostates, I and II, where six
distinguishable particles are distributed in 3 different energy levels (0, e and 2£) in the
following manner :
Energy 0 e 2£
macrostate I 3 3 0
macrostate II 2 2 2
STATISTICAL THERMODYNAMICS 605
Soln. : Entropy, S = k In W,
where W is thermodyrramic probability
and k is Baltzmann 's constant.
6!
For macrostate 1, W1 = = 20.
3!3!
6!
For macrostate 2, W2 = = 90.
2!2!2!
So the difference in entropy. between two macrostates, is ,,; S2 - S1
w . 90
=kin W2 - kin W1 = kln-2 =kin
W1 20
= 1.38066 x 10-23 x 1.504 J.K-1
= 2.079 x 10-23 J.K-1.
6. Calculate the rotational partition function of a diatomic molecule at high temperature.
Soln. : Rotational energy of Jth level = E1 = BJ (J + 1),
h2
where B = 8rt2/

h = Planck constant
I = moment of inertia:
Rotational partional function
-
= qrot(D = L/21 + l)ePBJ(J+l), where f3 =
J=O kT
Degeneracy of Jth level = 2J + 1.

Since 1_ is quite small at ordinary temperature for most molecules, the sum can
kT
be approximated by an integral. Hence, at high temperature,

qror<D =
- -
J(21 + I)e-BJ(J+1)/kT = Je-BJ(J+l)/kTd{ J(J + 1)} = k; =
0 0

7. Find the ratio of population of two states such that Eb - Ea = kT. Degeneracy
of corresponding energy levels are 3 and l.
Soln. : Let Nb and Na be the population of state b and state a.
gb = 3, 8a =1
N 8a e -Ea/kT l
Hence, _a _ _ _ -(Ea-Eb)/kT _ !!_ _ 0 906
- E /kT - e - - • •
Nb 8be- b 3 3
8. The energy difference between ground state and first vibrational state for 0 2 is
3.139 x 10-20 J. At what temperature would the 2nd vibrational state have half of the
population of the 1st vibrational state?
Soln. : Degeneracy of each vibrational state = 1.
Hence, Nn=I = 2 = e-( E1 -E2 )/kT = e3.139xJ0-20/rxl.3Sx10-23
Nn=2
606 PROBLEMS ON PHYSICAL CHEMISTRY

or, In 2 = 3.139 x 10-201y x l.38 x 10-23


:. T = 3281.61 K.
9. Two distinguishable p~icles are distributed among four boxes. How many
distributions are possible? Calculate thermodynamic probability of each distribution and
commeJM on most prqbable distribution. Calculate also the probability of getting any one
arrangement.
Solo, : Only two distributions are ,possible.
('i) 2 in same box (2000)
(ii) 2 in different two boxes with one in each box (llOO).

Thermodynamic probability of distribution (2000) = 4 P1 = 4.


Thermodynamic probability of distribution ( 1100) = 4
P2 = 12.
So the 2nd distribution is most probable since it has highest thermodynamic probability
or microstates. ,
Total number of arrangement = 24 = 16 (also 12 + 4 = 16).
So probability of getting any one arrangement = /6
10. Suppose, five letters of the word 'LASER' is written on separate cards. The cards
are shuffled and arranged in a row. Calculate the probability of obtaining the word 'LASER'
Solo. : Total number of ways the five letters can be; arranged taking one at a time
=~ = 120.

So· probability of obtaining the word 'LASER' = ~.


120
11. Suppose, 2 distinguishable particles are distributed among 3 degenerate energy
states. How many arangements are possible? What will happen if the particles are (i) bosons
(ii) fermions? ·
Solo. : Here degeneracy = g = 3.
Number of particles = N = 2.

Total number of arrangements = W = gN = 32 = 9.

AB
AB
A B
A B
A B
B A
I} A
B A
STATISTICAL THERMODYNAMICS 607

(g+ N-1)' 4!
(i) For bosons, W
-
=
. N!(g-1)!
· =- =6 ..
2!2!
E1 Ez ~,.
AA
AA
AA·
A A
A A
A A

(ii) For fermions, W = 8 CN = 3C2 =~ = :3.


2H!

A A
CHAPTER18

MISCELLANEOUS PROBLEMS

Theoretical Problems
(a) Answer the following in brief as much as possible :
1. What is the SI unit of R?
Ans. : The SI unit of R is J.K-l .mo1-1.
2. What is the SI unit of van der Waals constant a?
Ans. : J.m 3 .mol-2 .
3. What is the relation between bar and torr?
Ans. : l atm = 760 torr
= l.01325 bar
760
l bar =- - - torr
1.01325
= 750.06 torr.
4. Find the llf<lximum number of degrees of freedom. pf a one-component system. ·
Ans. : By phase rule, P + F =·c +·2,
P = no. of phase
F = degrees of freedom
C = no. of component.
F =C-P+2=l+2-P
=3 - P.
Minimum phase = I.
:. ·maximum degrees of freedom =3 - l = 2.
S. Find the degrees of freedom of an aqueous solution of sucrose and glucose.
Ans. : By phase rule, F =C - P + 2.
Here C = 2, P = I
F = 2 - l + 2 = 3.
608 ..
MISCELLANEOUS PROBLEMS 609
6. How is entropy related to probability?
Ans. : If Wis thermodynamic probability, Sis entropy and k is Boltzmann constant,
R
which is equal to - [NA =Avogadro number],
NA .
S = k In W
7. Remark whether Q = 0 for a cyclic process.
Ans. : Q is not a state function, so Q ;C 0.
8. Remark whether every closed system is isolated.
Ans. : An isolated system cannot exchange both matter and energy with the
surrounding. Closed systems exchange heat (energy) with the surrounding. Thus,
clost{d system is not an isolated one.
9. Express l J in kg.m 2.sec-2.
Ans. : Unit of energy is joule.
l J = force x distance
Dimension of l J = Mr.:r--2 x L = MI}Y-2.
So, l J = l kg.m 2 .sec2 .
10. Express l watt in kcal/day.
lJ 1Jx60 1Jx60x60x24 864001
Ans. : 1 watt = !sec
= = =
minute day day

86400
= - - cal/day = 20650.0956 cal/day = 20.65 kcal/day.
4.184
11. Remark whether TB should be greater than Tc·
Ans. : Tc is the highest temperature at which a gas can be liquefyed by application
of pressure alone. TB is the temperature at which a gas can behave ideally, since
then intermolecular attraction and repulsion balances each other. As temperature
increases, intermolecular attraction starts becoming negligible. Thus, TB > Tc.
12. Remark whether t::,S = 0 for any process in isolated system.

Ans. : t::,S = Qrev for an isolated system. But for spontaneous changes, the heat
T .
change is irreversible, and, Qirrev = 0 for an ·isolated system. But there is no relation
between irreversible heat change and entropy change. So t::,S is not zero for any
spontaneous change in an isolated system.
13. When should a liquid have zero surface tension?
Ans. : Surface tension, y, is a surface property. At critical temperature (Tc) and
pressure (Pc), there is no surface. Hence, at Tc, y = 0.
2
14. For a distribution P(u) du = Ae-u du with u (-00 to +oo). Find the most probable
value.
Ans. : u = 0 [for the most probable speed,
d 2
- (Ae-u ) du = 0
du
2
:::::) -2uAe-u = 0, :. u = OJ

Prob. Phy, Chem.-39


610 PROBLEMS ON PHYSICAL CHEMISTRY

15. Define compressibility factor.


If at certain temperature, Pi and P are the pressures of ideal gas and real gas
respectively at the same molar volume or at the same pressure, Vi and V are the
molar volumes of ideal gas and real gas respectively, then compressibility factor,
p v p PV1
z ==
P;
== == nRT == nRT
V;
V1

16. Remark if the pH range for boric acid-borax buffer lies in the acidic range.
Ans. ·: This is a buffer of weak acid and weak base. Pk0 of boric acid is 9.27.
Its actual buffer range is 8.27 to 10.27. Thus, pH range of the syst~m is alkaline.
17. What is the driving force of osmosis?
Ans. : The driving force of osmosis is the chemical potential. The increase of
chemical potential in a solvent-solution system decreases the tendency of the
solvent to pass to solution. So the driving force is the chemical potential.
18. Draw a volume-temperature diagram for an isothermal expansion.·
Ans.

r
T i
v j
v- T-

19. Plot P vs. for an ideal gas at constant T.


v
Ans. : This is a linear plot passing through the origin:

i
p

1-
v
20. What is the basic difference between volume and molar volume?
Ans. : Volume is an extensive property, molar volume is an intensive variable.
21. What physical property does the dimension ML21 2 correspond to?
Ans. : It corresponds to energy.
22. What is the dimension of viscocity coefficient (fl)?
de
Ans. : ML- 11 1 [F == 77A. - ]
dy
MISCELLANEOUS PROBLEMS 611
23. Define reduced temperature.
Ans. : If the temperatur~ of a gas in a particular state is T, and Tc is its critical
T
temperature, then - = Tr = reduced temperature.
Tc
1
24. What kind of average speed is involved in PV = mnc 2.
3
Ans. : The average speed is r.m.s.
d 3x
25. If x is the length and y is the time, then find the dimension of dy 3 .

d 3x
Ans. : The dimension is [L13], because
dy3
= dyd . dyd . dx
dy

26. Explain : PV is a state function.


Ans.: For state l, PV is P 1V1 and for state 2, PV is P 2 V2 irrespective of how
the change takes place. Thus, PV is a state function.
27. Explain if Z in PV = ZnRT for a real gas is intensive.

Ans. : By definition, Z = PV . V is molar volume Vm· Thus, Z =


PVm . A11
nRT n RT
these quantities are intensive. Therefore, Z is an intensive property (In case of
Z, parts are not added to give the whole).
28. Find the degrees of freedom in the reaction mixture CaC0 3 = CaO + C0 2 .
Ans. ; There are three components, CaC0 3 , CaO and C0 2 . But there is an equation
relating them, so the no. of components is 2 (3 - I = 2).
No. of phases, P = 3.
Hence, deg~ees of freedom, F =C - P + 2
. =2-3+2=1.
29. Remark whether heat cannot be completely converted into work.
Ans. : Heat can be completely converted into work in a process involved in state
change. As for example, isothermal reversible or irreversible expansion of an ideal
gas. However, heat cannot be completely converted into work in a cyclic process.

30. Identify X and Y in (ap)


aT
_= (as)
vax r

Ans. : By Maxwell equation we know that, ( aP) v -- (as)


aT av T

Hence, X = V and Y = T.
31. Explain whether a pure system with four phases can co-exist.
Ans. : We know that by phase rule, P + F = C + 2,
P is the no. of phases,
F is the degree of freedom,
C is the no. of components,
He!lce, F = C - P + 2.
612 PROBLEMS ON PHYSICAL CHEMISTRY

A pure system means a single compone_nt.


.. F= I - 4 + 2
= -1,
Hence, four phases cannot co-exist.
32. Giv~n, Y = aX2 . How do you get a· linear plot?
Ans. : By taking logarithm,
log Y = log a + 2 log X.
Thus, plot of log Y vs. log X is linear with 2 as slope and log a as intercept.
33. When does solubility decrease with temperature?
Ans. : If dissolution is exothermic, e.g., heat of solution is negative,. then solubility
will decrease with increase in temperatue.
34. How does Kw change with temperature?
Ans. : Kw is the equilibrium constant of the reaction, H 20 = H+ + OH-. This
is endothermic and hence, Kw will increase with increase in temperatrure.
35. W·hat pH range will be suitable for acetic acid-acetate buffer?
Ans. : The best buffer region is pH with (pKa + I) and (pKa - I), where pKa
is the negative logarithm of Ka of acetic acid and pKa =4.75. Therefore, the pH
range is 3.75-5.75.
36. For an aqueous solution of varying pH, plot pH vs. pOH at a given temperature.
Ans. : Since pH is varying, taking pH as abcissa (X-axis) and pOH as ordinate
(Y-axis), the curve is shown below : y
(0, 14)

We kn0w that, pH + pOH = 14. i


(pOH)
So when pH = I, pOH = 13.
=(14-pH)
Hence, it will be a linear plot
with 14 as intercept and -I as slope.
~---------
(0,0) pH -
x
37. Why do not two isothermals intersect each other?
Ans. : Isothermals are plotted in PV-curve in general. Isothermal plot is a
, rectangular hyperbola and other isothermals are parallel to it. Each point in the
same isothermal plot are in the given temperature. A plot for P-V for an ideal
gas is shown below :

Jsothermals for an ideal gas at


temperatures T 1• T2 and T3
T3 > Tz > Tl. p
T3
Tz
.,___ _ _ _ _ _ _ _ _ _ T1

v-
MISCELLANEOUS PROBLEMS 613

38. Viscosity and conductivity of a gas must be related to another. Why?


Ans. : In viscosity there is momentum transfer-it is transpoi:t of momentum.
p2
Theremal conductivity is due to transport of K.E. Kinetic energy (K.E.) is m,
2
where P is momentum. So they must be related.
39. State the principle of corresponding states.
Ans. : The principle of corresponding states predicts that at the same reduced
temperature (Tr) and reduced pressure (Pr) all gases have the same reduced volume

(Vr). Here reduced temperature (Tr) is !.._ Pr is .!.._ and Vr is ~. where Pc,
Tc pc Ve
Ve and Tc are the critical pressure, volume and temperature of the gas respectively
and P, V and Tare pressure, volume and temperature of the same gas respectively.
40. When is an adiabatic process isoentropic?

Ans. : We know that llS = Qrev , where llS is change in entropy, Qrev is reversible
T
heat change and T is temperature. Thus, for adiabatic reversible change, llS =
0, i.e., adiabatic reversible change is isoentropic.
41. In a cryoscopic study of solutions, which concentration unit is preferable-molarity
or molality? Why?
Ans.: The unit chosen is molality. Because in cryoscopic change, there'is a change
of temperature and the molality does not depend on temperature (molarity depends
on temperature).
42. What is the dimension of enthalpy?
Ans. : Enthalpy has the unJt of energy. Thus, its dimension is Ml..2'12 .
43. Iodine usually sublimes but it can be melted. How?
Ans. : For the iodine system the triple point is above l atm. Natura~y solid iodine
does not have a normal melting point. It sublimes at l atm. But above the triple
point which is much above 1 atm, solid iodine can be melted.
44. Should an ideal gas have a viscosity? Why?
Ans. : In case of gases, viscosity is due to momentum transfer by collisions. In
ideal gases there can be collisions. So they will have viscosity.
45. Does the B.P. of water change with pressure? How?
Ans. : B.P. of any substance depends on pressure. Dependence is given by
Clapeyron equation and Clausius-Clapeyron equation. By Clausius-Clapeyron
equation, writing in integrated form,

In P2 = Af-1 T1 - T1
P1 R T1T2 '

where P 2 is the pressure at temperature T2 ,


P 1 is the pressure at temperature T 1,
!:l.H is the change in enthalpy, i.e., latent heat for evaporation,
R is the universal gas constant.
614 PROBLEMS ON PHYSICAL CHEMISTRY

46. What is the dimension of surface tension?


Ans. : Surface tensio? is the force per unit length.

Thus, the dimension is ML'r-2 x _!_ = [Mi2].


l
47. What is Clausius inequality?

Ans. : We know that, dS = dQrev and any aQ :::;; dQrev .


T

.. dS ~ aQ. This is the Clausius inequality.


T

dQrev is reversible heat change and dQ is heat change in spontaneous process.


48. Why is partial molar temperature of a component in a mixture irrelevant?
Ans. : Temperature (7) is an intensive property, and a system at equilibrium has
a same temperature in all its parts and in the surrounding. So the partial molar
temperature is meaningless.
49. How does chemical potential of a solvent will change on addition of solute?
Ans. : The chemical potential of solvent is ~t<tted as µf (T, P). Addition of solute
changes the mole fraction from l to X 1, where X 1 is the mole fraction of solvent.
So now µ 1 =
µf (T, P) + RT \11 X1, X1 is a fraction. Hence, In X1 is negative
and the chemical potential will be reduced.
SO. Why is o~t.vald dilution law meaningless in case of aqueous HCI solution?
Ans. : Ostwald dilution law is applicable for weak electrolytes BA ionising to
B+ and A-, BA ~ B+ + A- and states that as concentration decreases degree
of ionization (ex) increases. But HCI in aqueous solution is completely ionized
to H+ and c1- at all dilutions and hence, Ostwald dilution law cannot be applied.
SI. Why t:.S is not a state function?
2
Ans. : t:.S is JdS and it does not depend on path, being the same always for
I
the same state change. So dS is an exact differential. State functions characterize
a particular state. t:.S. does not characterize a particular state and hence, is not
a state function.
S2. In the titration of NH40H with HCI, the pH at the neutralization point is not
7--comment.
Ans. : In the titration of NH40H with HCI, at the neutralization point there is
NH 4CI. But NH: ions undergo hydrolysis and pH becomes < 7 due to presence
of some extra H+ arising from hydrolysis.
NH4e + Hp ~ NH 3 + H 3oe (ft9 is written as H 30e)
S3. I atm pressure = ? torr?
Ans. : I atm = 760 torr = 760 mm of Hg.
MISCELLANEOUS PROBLEMS 615

S4. What is the dimension of relative lowering of vapour pressure?


Ans. : If the V.P. of pure solvent is pO and that of solution is P at a particular
0
temperature, then relative lowering is p
p
~ p. Thus, relative lowering has no

dimension-it is a pure number.


SS. How does Carnot cycle look in T-S diagram?
Ans. : Carnot cycle looks like a parallelogram. The first change is reversible
isothermal, the second change is reversible adiabatic, i.e., isoentropic. The third
change is isothermal reversible and the last change is isoentropic.

Change
i
T Change
41 : l Chonge 2

Change 3

s-
S6. What is the S.I. unit of chemical potential?
Ans. : The S.I. unit of chemical potential (µ;) is J.mol- 1.
S7. What condition must be satisfied for a binary liquid mixture to show azeotropism?
Ans ..: If the binary liquid solution deviates from Raoult's law by increase or
decrease vapour pressure (V.P.) and there is a composition at which V.P. becomes
maximum ot minimum, then an azeotrope results.
SS. How does the pH of water depend on temperature?
Ans. : Kw increases as temperature increases. Hence, CH+ increases with rise of
temperature. Thus, pH given by -log 10CH+ should decrease.
S9. Cite two examples of reversible process.
Ans. : Boiling of a liquid in a closed vessel with movable piston at constant normal
P and T, and similarly melting of a solid at constant normal pressure and
temperature are examples of reversible process.
60. Write down any one form of Gibbs-Helmholtz equation.

Ans. : Let the Gibbs free energy be G, and H be enthalpy, then G =H + T ( oG)
ar p

is one form of Gibbs-Helmholtz equation.


61. A reaction of the type A(g) + A(g)~A 2 (g) is expected to be exothermic.-Explain.
Ans. : For a spontaneous change t:i.G should be negative. But t:i.G = t:i.H - Tt:i.S,
where t:i.G is the change in Gibbs free energy,
t:i.H is the change in enthalpy,
t:i.S is the change in entropy.
For the given change t:i.S has to be negative. Let it be - Y.
:. l:lG = t:i.H + TY. Hence, t:i.G can be -ve, only if t:i.H is -ve. The change is,
therefore, exothermic.
616 PROBLEMS ON PHYSICAL CHEMISTRY

62. State the basic law that governs the solvent extraction method.
Ans. : The law is Nernst distribution law which states that if there is a pair of
immiscible liquids and a solute is soluble in both the solvents with C 1 being the
concentration of solvent 1 and Cz being the concentration of solvent 2. Then

..s_ = D, distribution coefficient, is constant at a constant temperature.


C2
63. What is the dimension of change?
Ans. : Kept to itself the change has no dimension. However, it will have the
dimension of property whose change is given.
64. What does P signify in PdV type of work?
Ans. : PdV is the mechanical work done by or upon a system. P in PdV is external
pressure which becomes equal to internal pressure in reversible change.
65. What is thixotropy?
Ans. : Thixotropic gels form sol easily when shaken or pressed. It is a property
any gel can get at a certain condition. Thixotropy is a pr(')perty of the initial stages
of gelation.
66. Why is alum more effective than NaCl in seperating out clay from river water?
Ans. : Clay is in colloid form and it is precipitated as ions are added. According
to Schulze-Hardy rule the precipitating effect of ions increases as the valency of
ions increases. Alum consists of Al 3+, and NaCl consists of Na+. Thus, coagulating
power of Al 3+ is much greater. That is why alum is more effective.
67. Write the cell reaction if two equivalent of electricity is passed through the cell-
Hz(g), pt I H+ (aq), c1-(aq), AgCl(s) I Ag(s).
Ans. : Reaction at the R.H.E.,
2AgCI .__ 2 Age + 2c1-
2AgEl:l + 2e .__ 2 Ag
Adding, 2AgCI + 2e 2 Ag+ 2c1-
Reaction at the L.H.E. as R.H.E.,
2H+ + 2e =Hz
Now, reaction at L.H.E. as L.H.E., Hz = 2H+ + 2e
So the cell reaction is
2AgCl + Hz = 2Ag + 2H+ + 2c1- '.
or, 2AgCI + Hz = 2Ag
+ 2HCI. . :'.
1
68. Give example of one reversible and one irreversible cells. /f,
Ans. : Reversible cell, H2 (g) Pt I H+ (aq), Cr-(aq), AgCl(s) I Ag(sr" .
Irreversible cell, Zn I H 2S04 I Cu.

69. Wh at .ts the unit. of - - "1or an nth order reaction.


dlnk . ?
dt
Ans. : The unit is sec- 1.
70. What is the sign of entropy of activation when the transition state' is formed by
an associative mechanism? ;~ .•'
Ans. : It will generally be negative, since the activated partiCl~ ls;-ope and there
are more than one reactant. ·
MISCELLANEOUS PROBLEMS 617

71. What is the dimension of f pdq, where p is the generalized momentum conjugate
to the generalized coordinate q?

Ans. : f pdq = nh. Thus, f pdq has the same dimension as h, i.e.,

Ml-21'-2 x T = [ML21 1]

72. Write Gibbs adsorption isotherm.


c dy
Ans. : - RT. dC = r2,
where r2 is the surface excess of solute per unit area,
C is the concentration of solute
r is surface tension of solvent.
73. Calculate the momentum of an electron which is characterized by a de Broglie
wavelength of 100 pm.

Ans. : We know that A. = h


p
34
or P
'
= -hA. = 6.625x10- J.sec
lOOxI0- 10 m
= 6.626 x 10- 14 kg.m.sec- 1.

74. Write an expression of the Hamiltonian operator for a particle in a one dimensional
box.
h2 az
Ans. H -- - 87t 2 m. ox 2 , since V = 0.

75. Give an example of a cationic surfactant.


Ans. : The cationic surfactant may be lauryl amine hydrochloride
CH3(CH2 ) 11 NH3+c1-.
76. Name two experiments in support of (i) wave nature and (ii) particle nature of
electron.
Ans. : The experiments of finding elm ratio of an electron support the particle
nature of electron. Electron diffraction studies prove the wave nature of electron.
77. Name two experiments in support of (i) wave nature and (ii) particle nature of
radiation.
Ans. : The interference of light is an example of wave nature of radiation.
Photoelectric effect is one of the example indicating the particle nature of radiation.
78. Arrange boiling, Boyle and critical temperatures (Tb, T 8 and Tc) in increasing
order with explanation.
Ans. : The increasing order is Tb < Tc < T 8 .
In the boiling temperature the liquid vaporizes at 1 atm. At the critical temperature
the liquid state disappears and at the Boyle temperature the gas behaves as an
ideal gas.
79. Plot energy v~. n for the particle in a box in one dimension.
n2h2
Ans. : We know, E = - -2 •
8ma
618 PROBLEMS ON PHYSICAL CHEMISTRY

h2 nzh2
For a particular system,
8
ma 2 is constant (K). So,
8
ma 2 = Kn 2.
Therefore, the plot is parabolic and a plot for n = 1, 2, 3 is shown below :

9K -----------------

2
--n-
80. Give an example of a photochemical reaction which may be used to measure
intensity of radiation.

"' k rate of reaction


Ans. : vve now, <p =
la

:. = rate of reaction .
10
<p
In the range of radiation (250 - 400) nm the quantum efficiency of the decom-
position of uranyl oxalate is 0.57. Hence, measurement of the rate. of reaction
will lead to I a calculation.
81. Express mean ionic activity of Al 2(S04 ) 3 in terms of ionic activities.
Solo. : Al 2(S0 4) 3 ionizes as

Alz(SO 4)3 ~ 2Al 3+ + 3 so4 2


C 2C 3C

.. Ionic activity a2 = a2A13+· a3soi


.. a± = \{a;
.. a+ = C+'Y+· a_ = C_y_
.. a± = ~(2c)2. y~.(3C)3.y: = ~l08C5 . y~
. . a± = (108) 115.c.y±
r(5/2)
82. Calculate ~() .
r 3/2
Solo. : r(n+I) = nr(n)
r( %) = %r(%) = %x ~ r( I) = ~.

Again, r(l) = =
2
_!_ r( I)
2
_!_.
2
MISCELLANEOUS PROBLEMS 619

r(5/2) 3/4 3
r(3/2)
= 1;2
=
2
83. When does the Bragg reflection condition lead to /,., ~d?

Ans. : We know that Bragg's equation is


n/,., = 2d sine [e = 0°-90°].
When n = 2, A, = d sine,
d=·~.
sine
d ~A, (since, O < sine < 1).
84. For identical experimental conditions the first order Bragg reflection from a plane
of cubic crystal comes up at 5.9° and 5.85° respectively at 20 °C and 50 °C.
Calculate coefficient of cubic expansion of the solid.

Hints : Coefficient of cubic expansion = a. = I


V aT
(av) p

t.., = 2d sine
dV d3 . 3e
= __2Q.. - 1 sm 20 - 1.
v dio = sin3 e5o

Thus, a. = _!_(sin: ezo


30 sin e 50
-1) = S.59 x 10-4 K-1.
.
85. Potassium crystallizes in a body centred cubic lattice with cell length 5.2 A.
Determine (i) distance between (a) nearest neighbours and (b) next-nearest
neighbours (ii) no. of (a) nearest neighbours and (b) next-nearest neighbours for .
each K atom and (iii) density of lattice.
Hints :

(i) (a) b = 2{j 0

x 5.2 = 4.50 A (along body diagonal). a

(b) a = 5.2 A (along cell edge). '


'

(ii) (a) 8 (at 8 corners).


(b) 6 (along cell edges). G,
''
'
(iii) 0.921 gm.cm-3. ''

86. Show that ry 2cr4 /m has the dimension of energy where the terms have usual
meaning.
Ans.: Dimension of T] = ML-111
Dimension of CJ =L
Dimension of m =M
TJ2CJ4 M2L-2T-2L4
Thus, dimension of = ----- = ML212
m M
= dimension of energy.
620 PROBLEMS ON PHYSICAL CHEMISTRY

87. Show that (PdV + VdP) is an exact differential.


Solo. : Let df = PdV + VdP = MdX + NdY,
where M = P, dx = dV. N = V dy = dP.

If ( aa; l = ( ~~) y' then df is an exact differential.

Here, (aP)
aP v = I = (av)
av p

Thus, df is an exact differential.


88. Calculate the fraction of molecules of a gas having energy greater than 50 RT.

Solo. : dNE = 27t(-l-)3/2 E112e-E/RTdE


N rrl?T '
where E = energy per mole.
~ 3/2
:. N>E = J27t(-l-) Elf2e-E/RTdE.
N 7tRT
E

After integrating, we have

N~E. = 2 ( 1t;T r2 e-EIRT

= 2 (50RT.
1tRT
)l/ e-SORTIRT =. 1.54. ·. 10-21,
2
X

89. Mean free path is independent of temperature-comment.


kT
=A =
Ans. : Mean free path of a gas ~
27t0'2 p .
For a gas oi constant volume, T remains constant. Hence, A does not change
p
with change of temperature.
90. Consider the equilibrium,
A(g) ~ B(g) + C(g)
At a constant pressure of I atm. A dissociates to the extent of 50% at 500 K.
Calculate .::10° (K.J. moJ- 1) for the reaction. [JAM 2008]
Hints : Initial moles of A =a mol.

At equilibrium, moles of A = !!.. , moles of B = moles of C = !!.. .


2 2

Total number of moles = 3a


2

p 1
p/3a2
!!.. 3
= 3
2
MISCELLANEOUS PROBLEMS 621
91. Consider the following redox system
Q + 2H+ + -2e ==:;; QH2, E° = 0.699 V
Calculate pH of the solution at 298 K, if the redox potential of the system is
0.817 V. . [JAM 2008]
Hints : E = E° + 0.059 pH
E = 0.817 V, E° = 0.699V, pH= 2.
92. A stream of 0 2 molecules at 500 K exits from a pin ~ole in an oven and strikes
a slit that selects the molecules travelling in a specific direction. Given that the
pressure outside the oven is 2.5 x 10-7 atm, estimate the maximum distance at
which slit must be placed from the pin hole, in order to produce a collimated
beam of oxygen. (Radius of 0 2 = 1.8 x 10- 10 m)
Hints : To get a collimated beam, the maximum distance of the slit from the
pin hole ·should be same as mean free path of 0 2 molecules. :·

A, = Ji2mJ R;PNA
= 0.4733 m.

93. A photochei:nical reaction was carried out using a monochromatic radiation (490
nm) of intensity 100 W. When the sample was irradi~ted for 30 min, 0.3 mo!
of the reactant was decomposed. Estimate the quantum efficiency. [JAM 2008]
Hints : Energy corresponding to 1 einstein of photon of A, = 490 nm

= NAhc = 244088.7121 J.
"A

50
x l 00 x 30 x 60
Energy absorbed in einstein = lOO = 0.369.
244088. 7121
03
.. $ = · = 0.813.
0.369
94. Assuming ideal behaviour calculate L\U 0 at STP for the reaction :
CHig) + 202(g) ~ C02(g) + 2H20 (I), L\H° = -606.9 kJ.moJ- 1.
Hints : L\H 0 = L\U 0 - 2RT (for the reaction)
L\U 0 = - 602.358 kJ.mol- 1
95. Find the eigenvalues of the following matrix, given that 2 i; one of the eigenvalues.

0 -2
-1

1-x l
Solo.: 0-x -2 =0
-1 1-x

or, (1 - x) {(0 - x) (l - x) - 2} + l {-2 - (1 - x)} + 1 (-1 + x) = 0


622 PROBLEMS ON PHYSICAL CHEMISTRY

or, 3x + 2x2 - 6 - x3 = 0 or, (x - 2) (3 - x 2) = 0.


. . x = 2, x = ± -J3.
The eigenvalues are : 2, + -J3, -"13.
96. At 600 K and 200 bar, a I : 3 (molar ratio) mixture of A 2 and B 2 reacts to
form an equilibrium mixture containing xABJ =
0.60. Assuming ideal gas
behaviour, calculate KP for the reaction :
Ai(g) + 3Bi(g) === 2AB 3(g) [JAM 2007]
Solo. : Given : xABJ = 0.6.
Thus, xA2 +x82 = 0.4.
Now by the problem, nA2 n 82 = I : 3.

Let nA2 -i:- n 82 + nABJ = y.


Thus, nA2 + n 82 = 0.4 y,
Hence, nA2 = 0.4y x "4I = O. ly, n 82 = 0.4y x "43 = 0.3y.

XA
2
= -nA2
y
= 0.1, and x82 = nBz
y
= 0.3.
Now' K p = Kx (P)A"f
Here, !J.y = -2

0.6 2
2 = 3.42 x 10-3 .
O.lx0.33x ( 200 )
l.01325
97. A 50 mL 0.05 M solution of Fe(II) is titrated with 0.05M solution of Ce(IV)
in the presence of dilute H 2S04 at 25 °C. Calculate the equivalence point potential
and the equation constant Kin tenns of log K. [ £'!{ +3 +2 )
Fe IFe
=+ 0.75 V, £'!{
Ce
-1-4 + )
ICe 3

= 1.45 VJ [JAM 2007]


Hints : !J.G 0
= -RT In K = -nFE° = -2.303 RT log K
E° = 0.7 v
log K = 11.83.
98. The vapour pressure of D20 at 20 °C is 745 mm Hg when 15 gm of a non-volatile
compound is dissolved in 200 gm of D 2 0, the pressure changes to 730 mm of
Hg. Assmping applicability of Rault's law, calculate the molecular weight of the
compound. [JAM 2007]

Hints : P = P0 Xo2o

200
Here P = 730 mm of Hg, P 0 = 745 mm of Hg and h 020 = = 10.
200
MISCELLANEOUS PROBLEMS 623

and W2 = 15 gm.

Hence~ if M2 is molecular weight of unknown compound, M2 = 73gm.


99. The reaction Fk1~
L_
k_ 1 trans

is first order in both directions. At 25 °C, the equilibrium constant (K) of this
reaction is 0.4. If0.115 mo!. dm-3 of cis isomer is allowed to equilibrate, Calculate
the equilibrium concentration of each isomer.
Soln .. : Hints : Let at equilibrium concentration of trans-isomer = x mol.dm-3
Then concentration of cis-isomer = (0.115 - x) mol-dm- 3.
x
:. K = 0.115-x = 0.4, :. x = 0.033 mol.dm-3.

100. Consider the 41t electrons in G¥clobuSadiene to be free particles in a 2-dimensional


square box of length 2 A. Calculate the wavelength of the electronic transition
from the HOMO to LUMO. Write down the normalized wave functions for the
occupied degenerate states. [JAM 2007]

Solo.: For 2 dimensional square box, energy of a state = En n


x Y
=(n; + n;) 8ml
h\ .

For cyclobutadie~e. Egr = £ 11 x 2 + £ 12 .x 1 + £ 21 x 1


2 2 2 2
2h 5h 5h 14h
= -8ml
-x 2 2 + - -2x l + - -2x l =
8ml 8ml 8ml 2 ·
Eex = £ 11 x 2 + £ 12 (or £ 21 ) x 1 + £ 22 x
2 2 2
4h 5h 8h 17h 2
= -8ml
-2+ - -2 + - -
8ml 8ml 2
=
8ml 2 •

3h 2
!iE = Eex - Egr = 8ml2 .

3h 2 c
N ow, - - 2 = h-
8ml 1.,·
Here l = 2 A.
3x6.626x10-34 J.sec 2. 9979 x 108 m. sec- 1
Thus, 2
=
8x(9.109x10-34 kg) x (2x10- 10 ) m 2 'A

8 2
A= 2.9979xl0 m.sec- 1 x8x(9.109xl0- 31 )kgx(2x10- 10 ) m 2
·· 3 x 6.626 x 10-34 J.sec
= 4.396 x 10-8m = 43.96 nm.

v12 =~sin 1tX sin 2 1ty (here l = 2 A)


l l l
2 . 27tX . 1ty
v21 =-sm--sm-.
l l l
624 PROBLEMS ON PHYSICAL CHEMISTRY

101. When an ideal monoatomic gas is expanded from 1.5 bar, 24.8 L and 298 K
into an •evaculttted container, the final volume becomes 49.6 L. Calculate Ml,
Af and /lG for the process.
Hints : This process is isothermal for ideal gas.
Ml= 0

Af = nR In Vz = 5.76 J.K- 1
V1
!1G = -TAf = -l.72 kJ.
102. The reaction Nz + 3Hz = 2NH3 is carried out at 300 K by mixing Nz and Hz.
The standard free energy of formation of NH 3 is -16.4 kJ/mol. After one hour
of mixing, the particle pressures of Nz. Hz and NH 3 are 50 bars, 2 bars and
200 bars respectively. What is the reaction free energy at this stage of the reaction?
Hints : t:i.G = t:i.G 0 + RT In Qp

t:i.G
0
= 2t:i.G;,NH 3 - 3t:i.a;.Hz - t:i.a;.N 2
= -16.4 x 2 = -32.8 kJ.
2
{200)
Thus, t:i.G = -32.8 x 103 + 8.314 x 300 In = -21.3 kJ.
50x2 3
103. One mole of a salt of type MX is dissolved in 1.00 kg of water. The freezing
point of the solution is -2.4 °C. Calculate the percent dissociation of the salt
in water.
Hints : t:i.T
1
= ik1m, m = 1 molal
i-1
a= - -
y-1
The per cent dissociation = 29%
104. At 20 °C, the vapour pressure of two pure liquids X and Y which form an ideal
solution are 70 torr and 20 torr respectively. If the mole fraction of X in solution
is 0.5, find the mole fractions of X and Y in the vapour phase in equilibrium.

Hints ~ P = P~Xs + P~Ys


Again, P = Px + PY = PXvap + PYvap·
Given, P~ = 70 torr, P~ = 20 torr, Xs = 0.5.
Thus, xvap = 0.78, yvap = 0.22.
105. The equilibrium constant for the reaction
Fe30 4 (s) + CO (g) ~ 3Fe0 (s) + COz (g)
at 600 °C. is 1.00. If a mixture initially consisting of I mole of Fe30 4 , 2 moles
of CO, 0.5 mole of FeO and 0.3 _mole of COz is heated to 600 °C at constant
total pressure of 5 atmospheres, how many moles of each substance would be
present at equilibrium? [JAM 2006]
Hints : Fe 30 4 (s) + CO (g) ~ 3Fe0 (s) + COz (g)
(a - x) (b - x) (c + 3x) (d + x)
Let a, b, c and d be initial moles of Fe30 4 ,CO. FeO and COz respectively and
· x moles of each reactant are converted to products. Since Fe30 4 and FeO are solid,
MISCELLANEOUS PROBLEMS 625

Pco2 Xco2 P Xco2


Kp = - - = - - - = - - = Kx,
Pea XcoP Xco
where P is the total pressure.
Now at equilibrium total number of moles =b - x +x +d =b + d.
d+x b-x
Hence, Xco2 = .b + d and Xco = b +d

0.3+x
Given that Kp = l, b = 2 moles, d = 0.3 moles. Thus, = 2-x '
:. x = 0.85
moles.
Hence at equilibrium,
nFe 304 = (l - 0.85) moles = 0.15 moles
nco = 2 - 0.85 = 1.15 moles
nFeO = (0.5 + 0.85) moles = 1.35 moles
nc02 = (0.3 + 0.85) moles = l.15 moles.
106. The Zn +1zn ~alf cell (£ = -0.762 V) is connected to a Cu 2+icu half cell (E
0
2 0

= 0.340 V). What is the value of for spontaneous conversion of chemical energy
to electrical energy? What is the value of log 10 K, where K is the equilibrium
constant? Use 2.303 RT/F = 0.06 [JAM 2009]
Hints : Pt, Zn I zn+2 II cu+ 2 I Cu, Pt
Eo II = £0 2 - £0 2
ce Cu+ /Cu zn+ /Zn

!1G
0
=-nFE;eu = -RT lnK = -RT 2.303 log K
F
log K = - l
2.303RT - 0.06.
107. The limiting ionic molar conductivities of K+ 1 and c1- are 73.5 and 76.5 S cm 2
mo1- 1 respectively. If the molar conductivity of 0.1 M KCI solution is 130.0 S
cm 2 moJ- 1, calculate the Kohlrausch's constant for KCI solution. [JAM 2006j
Sol. : Hints : A =A 0
- k.,/C
k = kohlrausch 's constant

c = 0.1, Thus, k = 63.245 S cm2 moi- 312.


108. (a) Given that /1G =-nFE, derive the expression for the temperature dependence
of the cell potential (E) in terms of the change in entropy (11.5).
(b) For a cell reaction, E (at 25 °C) = 1.26 V, n =2 and AS= -96.5 JK- 1moi- 1.
Calculate E at 85 °C.
Hints : !1G (25 °C) = -nFE

(o!iG)
oT .p
= -l:lS
.

Prob. Phy. Chem.-40


626 PROBLEMS ON PHYSICAL CHEMISTRY

11.G (85 °C) = !lG(25 °C) - AS.60 = -nFE (85 °C)


E (85 °C) = l.23 volt.
109. The rate constant k for a second order reaction P + Q ~Products is expressed

as. log K = 20 - 3
000, wher~ the concentration is in mol.lic 1, Tis in absolute
T
temperature and time is in minutes. The initial concentrations of both the reactants
are 0.5 M. Calculate the activation energy and half-life of the reaction at 27 °C
(R = 2 cal K- 1 mol- 1) [JAM 2006]
Hints : K = Ae -Eal RT

log K = log A - Ea
2.303RT

Given that Ea = 3000.


2.303R
:. E0 = 13818 cal mot-I,

1
Thus, t 1 = - -= 2 x 10-9 min- 1
Ka
2
110. (a) A container is partitioned into two compartments, one of which contains ·2
moles of He while the other contains 3 moles of Ar. The gases are ideal. The
temperature is 300 K and the pressure is 1 bar.
R = 0.88 l bar moi- 1 K- 1, In (2/5) = -0.92, In (3/5) = -0.51.
(i) What is the total Gibbs free energy of the two gases?
(ii) If the partition between the two compartments is removed and the gases are
allowed to mix, then what is the Gibbs free energy of the mixture?
(iii) What is the change in enthalpy in this process?
(b) Obtain (i) the molar heat of formation of CHig) and (ii) the average
C-H bond energy, to the nearest kilojoule (kJ), from the given data :
!lH (kJ mo1- 1)
(I) CH4 (g) ~ CH3(g) + H(g) 435
(2) CH3 (g) ~ CH 2(g) + H(g) 444
(3) . CHi(g) ~ CH(g) + H(g) 444
(4) CH (g) ~ C(g) + H(g) 339
(5) C (graphite) ~ C(g) 717
(6) H 2 (g) ~ 2H(g) 436 [JAM 2009]
Hints : (a) (i) G = G + nRT ln P 0

Po = 1 bar, P = 1 bar
G = G0 •
3
(ii) G Ar= G~ +3RTln-P
5
P = 1 bar

GHe + GAr = G~e + G~, - 83.93 I-bar.


(iii) 11.Hmix = 0.
MISCELLANEOUS PROBLEMS 627

(b) (i) C(graphite) + 2H 2(g) = -CH4 w 1 = -73 kJ.mole- 1.

(ii) Average C-H bond energy = 7} == 18 kJ.mole- 1.

Objective Type Questions


Part A
[Four choices are given under each of the following questions. Choose the correct
answer in each case. Write your answers as (a), (b), (c) or (d).]
1. The resultant of two vectors of magnitude µ making an angle of e with respect
to each other is :
(a) 2µcose (b) 2µcos9/2 (c) 2µsine (d) 2µcos29.
2. For A ---1 B, Afi is given by

(a) I CpdT (b) dCP! dT (c) cp x T


(d) CPIT.

A B

Temperature (T)
3. (3N/2)! can be evaluated only when the number N is
(a) odd (b) prime (c) even (d) without restriction.
4. For 300 MHz, the corresponding wavelength is :
(a) l m (b) 2 m (c) 3 m (d) 0.5 m.
5. For an ideal gas undergoing free expansion under adiabatic conditions,
(a) !1U < 0, 11T < 0 (b) !1U < 0, !1T > 0,
(c) !1U > 0, 11T > 0 (d) !1U = 0, 11T = 0.
6. For the process A ~ B,

(a) !1U = UB - UA (b) Qtotal = QB - QA (c) Wtotal = WB - WA


(d) !1U = !1H + P!1V.
7. The potential V (rA' r8 , R) =(e 2/47t€ 0 ) [l/R - llrA - 1/r8 ], where R is the nucleus-
nucleus distance and rA and r8 are the nucleus-electron distances.
The term(s)
(a) llR is attractive, llrA and llr8 are repulsive
(b) llR is repulsive, llrA and llr8 are attractive
(c) 1/R is attractive, llrA and l!r8 are attractive
(d) 1/R is repulsive, llrA and llr8 are repulsive.
8. The number of vibrational degrees of freedom in water molecule is
(a) I (b) 3 (c) 2 (d) 4.
9. The partition coefficient for the distribution of benzoic acid between H20 and
CC1 4 mixture at a given temperature :
628 PROBLEMS ON PHYSICAL CHEMISTRY

(a) increases with increasing ratio of H 20 : CCl 4


(b) decreases with increasing ratio of H 20 : CCl 4
(c) increases with increasing concentration of benzoic acid
(d) ~emains constant.
· IO. Given that X + Y <=>A + B (with forward and backward rate constants of k 1 and
k2 ) and A <=> C + D (with forward and backward rate constants of k 3 and k4 ).
The equilibrium constant for X + 'Y <=> B + C + D is -
(a) (k 11k2) - (k 3/k4 ) (b) (k 1/k2) x (k4 /k 3 ) (c) (kifk 2) x (k 3/k 4)
(d) (k 1-k2)/ (krk4 ).
II. The rate of the following reaction 2NO(g) + Oi(g) = 2N0 2(g) 1s

(a) _J_ d[N0 2 ]1dt (b) _J_d[NO]ldt (c) d[NO]!dt (d) 2d[NO]!dt.
2 2
I2. The correct half-cell reaction for the electrode c1-, AgCl/Ag is
(a) Ag+(aq) + e- = Ag(s) (b) AgCl(s) + e- = Ag(s) + c1- (aq)
(c) AgCl(s) = Ag+(aq) + cI-(aq) (d) Ag+(aq) + Ci-(aq) = AgCl(s).
13. If vP is the most probable velocity, < v > is the mean velocity, and < v2 > 112 is
the rms velocity, then
(a) < v2 >112 > < v >> vp (b) < v >> < v2 > 1/2 > vp
(c) VP< <v2>1/2> <V> (d) VP= <v2>1/2 = <V>.
I4. The equivalent conductance of 0.1 mol.1- 1 acetic acid is 39 ohm- 1.cm 2 .equiv- 1.
The equivalent conductance of acetic acid at infinite dilution is 390 ohm- 1.cm 2.
equiv- 1. The degree of ionization of 0.1 mol.1- 1 acetic acid solution is :
(a) 0.2 (b) 0.1 (c) 0.01 (d) 0.5.
IS. The amount of electricity required to deposit one equivalent of silver ion from
0.1 molar silver nitrate solution in coulomb.moJ- 1 is :
(a) 96485 (b) 48243 (c) 192270 (d) 9648.
I6. The correct quantum numbers for an electron in the 3pz orbital are :
(a) n = 3, l = 1, m 1 = 0, ms = +l/2 (b) n = 3, l = 2, m1 = l, ms = -112
(c) n = 3, l = I, m1 = -1, ms= +l/2 (d) n = 3, l = 0, m1 = 1, ms= -l/2.
I7. The entropy of vaporization of toluene is 90 J.K- 1.mot- 1 at its boiling point of
110 °C. The heat of vaporization in kJ.mole- 1 is :
(a) 9.9 (b) 34.47 (c) 19.8 (d) 0.00 l.
I8. For the reaction,
CO(g) + 2H 2(g) ---; CH30H(I)
The free energy of formation of CO(g) and CHpH(I) are -137 kJ.moI- 1 and
-166.kJ.mol- 1 respectively at 298 K. This reaction is :
(a) spontaneous as written (b) non-spontaneous as written
(c) spontaneous in the reverse direction (d) spontaneity cannot be predicted.
I9. The distribution of probability density calculated from Maxwell-Boltzmann
statistics in an ar.bitrarily chosen direction spreads with
(a) increasing temperature (b) decreasing temperature
(c) temperature independent (d) increasing velocity.
MISCELLANEOUS PROBLEMS 629
20. The reaction A --t B is spontaneous at :

x z

A B
Extent of reaction
(a) W (b) X (c) Y (d) Z.
Part B
[Answer to the point]
21. For the reaction PCI 5(g) = PCl 3(g)+ Cli(g) :
(A) Calculate the equilibrium constant using the following data at 25 °C.
Substance Ml 0 (k,J.mol-1) S0 (J.K- 1.mol- 1)
PCI 3(g) -287 312
PC1 5(g) -375 365
Cli(g) 223
(B) Using the result from above, calculate the equilibrium constant of
2PCl 5(g) = 2PCI 3(g) + 2Cli(g).
22. The observed freezing point depression for an aqueous acetic acid solution of
molality = 10-3 mol.kg- 1 is 2 x 10-3 K. If K/HzO) = 1.86 K.moJ- 1.kg, (i) calculate
the depre~sion in freezing point, (ii) find the degree of ionization of acetic acid.
23. Starting with the definition of G, show that for a closed system in the absence
of non-expansion work,
v = (oG(aP)T' s = -(oG/oT)p·
24. Calculate the work done by 2 mo! of a monoatomic perfect gas initially at 25 °C,
undergoing the following transformations-
(i) isothermal reversible expansion from 10 L to 40 L.
(ii) adiabatic reversible expansion from 10 L to 40 L.
25. (i) Draw the phase diagram of water up to 2 atm and 110 °C.
(ii) Based upon the variation of molar Gibbs function (or chemical potential) with
temperature, explain why the melting point of ice decreases with increasing
pressure.
26. Calculate the Ksp ofHg2CI 2 ; E 0 (Hg 2 2 +/Hg) = 0.79 V; E 0 (Cl-,Hg 2Cl 2/Hg,) = 0.27 V.
27. (i) For the reaction A --t B, sketch the variation of the concentration of A with time
when (a) it is a zero order, and (b) first order reaction with respect to A.
(ii) Estimate the fraction of molecules with kinetic energy greater than or equal
to 100 kJ.mo1- 1 at 300 K.
(iii) Explain in maximum two lines the meaning of frequency factor in the
Arrhenius equation.
630 PROBLEMS ON PHYSICAL CHEMISTRY

28. A particle is represented in space with the following wave function. Choose the
correct option and justify the answer in two lines (maximum).
(a) The particle ... exist at 2 where the wave function is negative. (can/cannot)
(b) The kinetic energy of the particle at 3 is ... than that at I. (more/less)
(c) The probability of finding the particle at 5 is ... than at l. (more/less)
(d) The probability of finding the particle may become zero as it moves along
X-direction. (true/false)
(e) The momentum of the particle decreases as it moves aim~ X-direction. (true/
false)

Part C
[Four choices are given under wch (>j the following questions. Choose the correct
answer in each case. Write your answers as (a), (b), (c) or (d).]
29. Which of the following 0.1 molar solutions has the lowest freezing point?
(a) urea (ti) glucose (c) KCI (d) Na 2S04 .
30. Which of the following gas molecules has the longest free path under the same
conditions of temperature and pressure?
(a) 0 2 (b) H 2 (c) N 2 (d) Cl 2.
31. An element retains one-tenth of its original radioactivity after 2.303 ti.me units.
The half-life in the same unit is :
(a) 2.303 (b) 0.0693 (c) 0.693 (d) 0.2303.
32. The trajectory of a particle in Brownian motion is
(a) linear (b) zigzag (c) oscillatory (d) curved.
33. The difference, (!.:.H - !.:.U), in the heats of combustion of glucose at constant
pressure and constant volume at 25 °C is :
(a) zero (b) -I2RT (c) I2RT (d) 6RT.
34. The molar heat capacity at constant volume is independent of temperature for
(a) hydrogen gas (b) water vapour (c) helium gas (d) all gases.
35. Identify the exact differential :

(a) <fqrev (b) <fqrev IT (c) <fqirrev (d) <fq;rreJ T.


36. The acid-catalyzed hydrolysis of an ester is found to obey a pseudo first order
kinetics. The corresponding rate constant is independent of :
(a) the temperature
(b) the acid concentration
MISCELLANEOUS PROBLEMS 63i
(c) the ester concentration
(d) none of the above.
37. If the specific rate constant k is to be equal to the pre-exponential factor A in
the Arrhenius equation, then
(a) the reaction temperature is very low
(b) the reaction is a catalysed one
(c) the reaction is molecular in nature
(d) the reaction does not need any activation energy.
38. According to the equipartition principle, the average vibrational energy of the linear
N 20 molecule at 27 °C is approximately :
(a) 7200 J.mot- 1 (b) 9600 J.mo1-1 (c) 4800 J.mot- 1 (d) 6000 J.mot- 1.
39. The ionic mobility (µ) and ionic conductivity (/....) are related by the equation
(a) µ = ')JF (b) µ = A. (c) µ = nA.F (d) µ = A.F
40. The conductivity in ohm- 1.m 2.mo1- 1 of M+ ion is 70 and that of A- is 60. The
specific conductance of the saturated solution of MA is 1.3 x I 0-{) ohm- 1.m- 1. The
solubility of the salt in mou- 1 is :
(a) 10-3 (b) 10--4 (c) 10-5 (d) 10-1 1.
41. The role of the supporting eletrolyte in polarography is to
(a) maintain the residual current
(b) increase the diffusion current
(c) suppress the diffusion current
(d) limit the maximum current.
42. For the reaction Np4(g) ~ 2N0 2(g), the degree of dissociation at 50 °C and
I atm pressure is found to be 0.38. When the reaction is carried out in the presence
of an inert gas with a partial pressure of 0.25 atm (total pressure is still I atm),
the degree of dissociation wiJI :
(a) increase (b) decrease (c) remain unaffected (d) drastically decrease.
43. The first ionization constants of acetic acid, chloroacetic acid, carbonic acid and
boric acid are 1.75 x 10-5, 1.38 x 10-3, 4.69 x 10- 11 and5.79 x 10- 11 respectively.
To have the maximum buffer action between pH 4 and 5, one should have a mixture
of the following acid and its salt.
(a) chloroacetic acid (b) boric acid (c) carbonic acid (d) acetic acid.
44. Which of the following statements is true for the equivalent conductance of an
aqueous solution of KC!?
(a) It is independent of concentration.
(b) It inceases linearly with concentration.
(c) It increases linearly with increase in [concentration] 112 .
(d) It decreases with increase in [concentration] 1' 2 .
45. The enthalpy change for a cell reaction is given by :
(a) nC/T2-T1) (b) nF[T(oE/(JT)p - E] (c) [nFE + (oE/oT)pl

(d) [T(oE/oT)P-E] I nF.


632 PROBLEMS ON PHYSICAL CHEMISTRY

46. After electrolysis, the pH of an aqueous NaCl solution :


(a) increases (b) decreases (c) remains unaffected (d) is unpredictable.
47. A drop of ink put into a glass of water mixes uniformly with the solvent as time
progresses. This is due to the :
(a) gravitational force (b) minimization of potential energy
(c) maximization of entropy (d) osmotic pressure of water.
48. The specific heat of a stable solid can be obtained from
(a) the Gibbs-Helmholtz equation
(b) the Kirchhoff's equation
(c) the first law of thermodynamics
(d) the second law of thermodynamics.
49. For an adiabatic irreversible expansion of a perfect gas, which of the following
relations is correct?
(a) llT = -PextllV!Cv (b) llT = -PextllVICP
(c) llT = e-nR ln[V(final)/V(initial)] (d) !lT = 0.

50. For the reaction Nz(g) + 3H2(g) ~ 2NH3(g), llH 0 =- 46. l l kJ.moJ- 1 at 25 °C.
The equilibrium constant of this reaction will increase with :
(a) increase in temperature (b) increase in pressure (c) equilibrium constant does
not change by changing temperature or pressure (d) lowering temperature.
51. The reaction in a given direction is spontaneous if the change in Helmholtz
Function, A, is negative at :
(a) constant P and T (b) constant V and T
(c) constant H and S (d) constant T and S.
52. The relationship between a (edge length) and r (atomic radius) for a primitive
cubic unit cell is :
(a) a = 2r (b) (2) 112 a = 4r (c) (3) 112 a = 4r (d) a = 4r.
53. Ionic solids, in general, are associated with one of the following defects :
(a) interstitial (b) vacancy (c) substitutional (d) Frenkel.
54. How many mirror planes of symmetry are present in CH3CJ?
(a) none (b) l (c) 2 (d) 3.
55. Absorption of molecules in microwave region of the electromagnetic spectrum
is due to :
(a) elecronic transitions (b) vibrational transitions
(c) rotational transitions (d) nuclear spin transitions.
56. The attractive part of van der Waals interactions is due to
(a) ion-dipole interactions (b) dipole-dipole interactions
(c) dipole-induced dipole interactions (d) induced dipole-induced dipole
interactions.
57. Number of radial nodes in the 2pz orbital is :
(a) 0 (b) l (c) 2 (~) 3.
MISCELLANEOUS PROBLEMS 633
58. Between two wave functions, the one'.with larger number of nodes represents the
state with :
(a) higher energy (b) lower energy (c) same energy (d) zero energy.
. .
59. An ideal gas kept in a vessel at a temperature T, has :
(a) unequal average velocity along x-, y- and z-directions
(b) zero average veloccity along x-direction
(c) equal non-zero average velocity along x-, y- and z-directions
(d) unequal average speed along x-, y- and z-directions.
60. For a van der Waals gas having constants a and b,
(a) a and b are state functions and state parameters
(b) a and b are state functions but not state parameters
(c) P and T are state functions but not state parameters
(d) P, V and T are all state functions and state parameters.
61. Above 1100 K, iron undergoes a phase transition from a body-centred cubic to
another cubic structure accompanied by an increase in density. The new lattice
structure is :
(a) simple cubic (b) face centred cubic (c) CsCl type (d) orthorhombic.
62. At the upper critical solution temperature of phenol-water system, at a given
pressure the degree of freedom is :
(a) 2 (b) 3 (c) 0 (d) l.
63. Maximum entropy of mixing is achieved when two ideal gases, X and Y, are mixed
in the amounts of :
(a) l mole of X and 3 moles of Y (b) 3. moles of X and I mole of Y
(c) 2 moles of X and 3 moles of Y (d) 2 moles of X and 2 moles of· Y.
64. Two pieces of soft iron have equal mass and are at same temperature and pressure.
One of the pieces is placed on a permanent magnet, and the other on a wooden
block. The two pieces will have :
(a) same Gibbs free energy (b) different enthalpy
(c) same entropy (d) different entropy.
65. In the reaction, C 3H 8(g) + 50 2(g) = 3C0z(g) + 4Hz0 (g), 6.6 moles of C0 2 were
formed. The extent of reaction, ;, is :
(a) 2.2 (b) 1.6 (c) 1.3 (d) 6.6.
66. In the relations/statements given below, the correct one is
(a) L\G = Ml + SL\T + TL\S (b) L1S = 0 for all adiabatic processes.
(c) a solution component at unit activity is in its standard state.
(d) activity and activity coefficient are intensive properties.
67. A liquid solution of two components :
(a) will always freeze entirely at one temperature
(b) will never freeze entirely at one temperature
(c) will have frozen solid phase containing both the components
(d) will never freeze.
634 PROBLEMS ON PHYSICAL CHEMISTRY

68. The molecule whose bond vibration is both IR and Raman active is
(a) Hz (b) Nz (c) HI (d) Oz.
69. Trouton rule states that :
(a) Af11Tb is constant for normal liquids (b) Af1IT is constant for all liquids
1
(c) .1S > 0 for spontaneous processes (d) .:1G ~ 0 as equilibrium is approached.
70. Two electrodes, Ag+ I Ag, and ci- 1, AgCI I Ag dipped into a solution of aqueous
AgN0 3 will have :
(a) same E and E 0 values (b) different E and E 0 values
(c) same £ 0 value, but different E values (d) different E 0 values, but same E value.
71. The standard heat of formation at 298 K of :
(a) Hz, Dz and HD are all equal to zero (b) Hz, Dz, and HD are all non-zero
(c) Hz and Dz are zero, but that of HD is non-zero (d) HD is zero, but those
of Hz and Dz are non-zero. ·
Part D
72. Indicate whether the following statements are true (T) or false (F) :

(a) The molecule-ion o; is more stable than the neutral molecule Oz.
(b) The species Oz is diamagnetic.
(c) The 3p 2 orbital of the hydrogen atom has no radial node.

(d) .ix !lpx ~


n
l
(e) Diamond has a structure of covalent network.
(f) Compared to the infrared transitions, electronic transitions occur at longer
wavelength.
(g) Hz molecule displays a pure rotational spectrum.
(h) If two gases are at the same temperature, their molecules have the same
average kinetic energy.
(i) At 600 mm pressure I gm of charcoal absorbs about 10 ml of Nz at 0 °C,
20 ml at -29 °C and 45 ml at - 78 °C. These data indicate that adsorption
of Nz on charcoal is accompanied by evolution of heat. ·
G) If !lG ads is zero, then the following adsorption isotherm is obeyed :
KP
e= l+KP
Part E
73. Choose the correct statement(s).
(a) A hypothetical absolute temperature scale ( 0 A) is defined by assigning 0 °C
= 100 °A. The boiling point of water on this scale is :
(i) I 00 ° A (ii) 136.61 °A (iii) 200 °A.
(b) The following plot for a fixed amount of an ideal gas at constant temperature
will not be linear :
(i) P against V (ii) P against llV (iii) PV against P.
MISCELLANEOUS PROBLEMS 635

(c) The root mean square speed for a collection of gas molecule at absolute
temperature T is :
(i) (8RT I rtM) 112 (ii) (3RT/M) 112 (iii) (2RT!M) 112 .
(d) The height (h) to which a liquid will rise or be depressed in a capillary tube
is related to the radius of the tube (r) by :
(i) h = 2y I prg (ii) h = ry I 2pg (iii) h = prg I 2y.
(e) The molality of an aqueous solution that has a boiling point elevation of
1 K is :
(i) 0.95 mol.kg- 1 (ii) 1.95 mol.kg- 1 (iii) 3.95 mol.kg- 1.
(f) The entropy of a perfect crystal at 0 K is :
(i) > 0 (ii) 0 (iii) < 0.
(g) A change in the free energy is related to the enthalpy change, the entropy
change and temperature by :
(i) D.G = M - Tfi.S (ii) D.G = D.U - Tfi.S
(iii) D.G = Ml - Tfi.S (iv) D.G = Ml + Tfi.S.
(g) In SI units, pressure is expressed in
(i) Pa (ii) bar (iii) atm (iv) torr.
(i) A spontaneous change is always accompanied by
(i) an increase in the entropy oMhe universe
(ii) a decrease of the entropy of the universe
(iii) an unaltered entropy of the universe.
(j) At equilibrium,
(i) D.G > 0 (ii) D.G < 0 (iii) D.G =0 (iv) D.G = -RT In K.

Part F
74. State whether the following statements are true (T) or false (F).
(a) In order to minimize liquid junction potentials, one must use a salt bridge
containing a salt such that t+ = t_.
(b) The order of a reaction is always a positive integer.
(c) The standard reduction potential EJ of a certain redox couple is greater than
~ , the standard reduction potential of another redox couple. Therefore,
is a stronger reducing agent than 2.
(d) The dissociation constant of a salt depends upon the solvent used.
(e) The total energy of the universe is variable.
(f) If D.G has a negative value, the process is not spont~neous.
(g) Entropy of the universe remai11s constant for a reversible process.
(h) Enthalpy of the universe decreases in an irreversible process.
(i) The change in internal energy is the heat change that takes place at constant
volume.
(j) The enthalpy change is the heat change occuring at constant temperature.
636 PROW.EMS ON PHYSICAL CHEMISTRY

Part G
75. Choose the correct statement(s).
(a) An increase in the volume of an ideal gas brings with it :
(i) a decrease in entropy (ii) no change in entropy (iii) an increase in entropy
(iv) none of the above.
(b) In homonuclear diatomic bond strength :
(i) increases (ii) decreases (iii) remains the same with increase in bond order.
(c) In heteronuclear diatomic bond length :
(i) increases (ii) decreases (iii) remains the same with increase in bond order.
(d) The probability density for finding the Is electron in a hydrogen atom is
maximum at :
(i) r =0 (ii) r = a0 (iii) r = a0 !2 (iv) r = x.
(e) Compounds of various metals give characteristic colours to a flame. These
colours are due to :
(i) nuclear spin transitions (ii) vibrational transitions (iii) electronic transitions.
(f) In fee metal, the number of atoms per unit cell is
(i) 8 (ii) 14 (iii) 6 (iv) 4.
(g) The rate constants of two reactions, k 1 and k 2, are equal at a certain
temperature T. If at a higher temperature r, k 1' > k 2', then
(i) Ea. I < Ea. 2 (ii) Ea. I = Ea. 2 (iii) Ea. I > Ea. 2 .
(h) A current of 965 mA flows between two copper electrodes immersed in a
solution of CuS04 in an electrolytic cell for 200 sec. The mass of copper
deposited on the cathode is
(i) 0.635 gm (ii) 6.35 gm (iii) 0.0635 gm (iv) 0.00635 gm.
(i) The partition coefficient Kofa solute in two immiscible solvents depends
on the temperature. This dependence can be rep~esented as

(i) K oc T (ii) K oc _!__ (iii) K oc Tn (iv) K oc ellT


. T
(v) none of the above.
(j) The unit of rate constant for a zero-order reaction is
(i) sec- 1 (ii) mol.litre- 1.sec- 1 (iii) litre.mo1- 1.sec 1 (iv) I.

Part H
76. Choose the most appropriate answer from the alternatives given
(a) The ionic product of water at 5 °C is 1.9 x 10- 13 . The pH of neutral water
at this temperature is
(i) 7 (ii) 6.50 (iii) 6.36 (iv) - 6.36.
(b) Cars with catalytic converters must use unleaded fuel because
(i) it is mon; fuel efficient (ii) it gives higher mileage (iii) lead poisons the
catalyst (iv) lead is harmful for the environment.
(c) The rate of a reaction, A + B ~ products,
MISCELLANEOUS PROBLEMS 637
remains unchanged when the concentration of A is doubled and B is halved.
It is also-unchanged when the concentration of A is halved and B' is doubled.
The rate expression is of the form :
(i) [Af[B] (ii) [A] [B] 2 (iii) [A] 2[B] 2 (iv) [A] 2[B]112.
(d) The ion conductance at infinite dilution at 25 °C of the proton is 349.8
ohm- 1.cm 2 and the acetate ion is 40.9 ohm- 1. Therefore, the equivalent
conductance of acetic acid at infinite dilution is :
(i) 390.7 ohm- 1.cm 2 (ii) 308.9 ohm- 1.cm 2 (iii) 431.6 ohm- 1.cm 2
(iv) 268.0 ohm-1.cm2.
(e) If two gases are at the same temperature, their molecules have same :
(i) rms velocity (ii) average speed (iii) average kinetic energy (iv) average
potential energy.
(f) The orthorhombic crystal system is :
(i) a =b =c (ii) a = b ;Cc (iii) a ;Cb ;Cc (iv) none of
a = f3 = r = 90° a = f3 = r = 90° r
a= f3 = = 90° the above.
(g) A gas is allowed to expand from Vi to VJ against constant external pressure,
P ex. The work done by the gas is :
(i) 0 (ii) Pex (VJ -V) (iii) -nRT In CV/
V) (iv) nR In V/V;.
(h) Bohr's basic quantum postulate was :
(i) mvr = nhl21t (ii) E = hv (iii) c = v/... (iv) all of the above.
(i) A wave function is acceptable, only if it is :
(i) single valued (ii) continuous (iii) square integrable (iv) all of mentioned
earlier.

(j) The outermost electron in H2 will be in the orbital


(i) l<Jg (ii) 2<Jg (iii) l<Ju (iv) 3<Jg.

Part I
77. State whether the following are True or False :
(a) All real gases can be liquefied at room temperature.
(b) If a strong acid is added to a saturated solution of a salt of a weak acid,
more salt will precipitate out.
(c) The equivalent conductance of a strong electrolyte increases with decrease
in concentration.
(d) For an elementary reaction the order is same as the molecularity of the
reaction.
(e) The extensive use of CFCs (chlorofluorocarbons) is responsible for the Green
House Effect.
(f) The number of components in the equilibrium CaO(s) + C0 2(g) = CaC03(s)
is 2.
(g) The freezing point of water increases with increasing pressure.
(h) The entropy of the system does not change for an adiabat~c irreversible
change.
(i) If a perfect gas expands against zero external pressure under adiabatic
conditions, its temperature falls.
638 PROBLEMS ON PHYSICAL CHEMISTRY

(j) The quantum number l represents the magnitude of the electron's orbital
angular momentum.
Part J
Choose the correct answers :
78. Of 'the following statements, which is not true for an ideal gas?
(a) (au;av) 7 = o Cc) (oU/oT)v = o
(b) (oU/oP)T = 0 (d) (oH/oP)T = 0.

79. The pH of a 10-3 M solution of a weak acid HA is 4.0. The dissociation constant
of HA is :
(a) 1 x 10-3 (b) 1 x 10-4 (c) 1 x 10-5 (d) I x 10-6 .
80. The following reaction is carried out at constant volume and constant temperature :
2NO(g) + 2Hz(g) ~ Nz(g) + 2HzO(g).
The equilibrium constant for the above reaction can be calculated by knowing the
(a) initial concentration of all the species and the total final pressure of the system.
(b) initial concentration of all the species.
(c) equilibrium concentration of any three species.
(d) mole fraction of two species at equilibrium and the total final pressure of
the system.
81. The criterion for spontaneity and equilibrium of a system is
(a) (dG)7V. ::::; 0 (c) (dU)S.P ::::; 0
(b) (dA)P.T ::::; 0 (d) (dH)S.P ::::; 0.
82. The spontaneous cell reaction and the emf of an electrochemical cell are known
at a given temperature. The thermodynamic function (s) which can be determined
from this information is/are :
(a) 6.S only (c) Ml only
(b) !!..G only (d) !!..G, Ml and 6.S.
83. For one mole of an ideal gas, (oV/oP) 7 is equal to
(a) -VIP (c) -RT/P
(b) RT/P2 (d) RTIP.
84. The liquid-vapour phase diagram for two miscible liquids A and Bis given below.
In a mixture of A and B, the mole fraction of A (XA) = 0.25. When the mixture
is distilled in a column at I atmosphere,

85

75

65 Tb= b?iling point at I atm

0·0 0·25 0·50 0·75 1·0


XA
MISCELLANEOUS PROBLEMS 639
(a) it is possible to separate both A and B in pure form.
(b) it is po~ible to separate A only in pure form.
(c) it is impossible to separate either A or B in pure form.
(d) it is possible to separate B only in pure form.
85. The concentrations of NaCl, MgC1 2 and FeC1 3 in a 250 ml aqueous solution are
I M, 0.5, and 0.0005 M, respectively. The solubility product of AgCl is 2.8 x
10- 10 . Half a gram of AgCl is added to the solution. The concentration of Ag+
in this solution is :
(a) 1.4 x 10-lO M (b) 0.7 x 10-lO M
(c) 2.8 x 10-lO M (d) 1.86 X 10-IO M.

86. The width of the Maxwell-Boltzmann velocity distribution of a gas


(a) decreases as the temperature increases.
(b) increases as the temperature increases.
(c) remains unchanged with change in temperature.
(d) increases or decreases with temperature depending on the gas.
87. The half-life of a reaction is independent of the initial concentration of the reactants,
if :
(a) the reaction is zeroth order, (b) the reaction is first order
(c) the reaction is very fast, (d) the reaction is very slow.
88. In a simple free radical initiated polymerization, the rate of polymerization varies
with the initiator concentrations [/] as :
(a) [/]o (b) [/] 112 ( c) [/] (d) [/]2.
89. Which one of the following statements is correct for the equilibrium reaction
H2(g) + Cl 2(g) ~ 2HCl(g) + heat :
(a) increase in temperature shifts the equilibrium to the right
(b) increase in pressure shifts the equilibrium to the left
(c) change in pressure has no noticeable effect on the equilibrium
(d) decrease in pressure shifts the equilibrium to the left.
90. For the following mechanism,

N0 2(g) + F 2(g) ~ N0 2F(g) + F(g), slow


l
F(g) + N0 2(g) -y:-+ N0 2F(g), fast
2
The rate law is given by
(a) rate = k 1k2[N0 2] 2
(b) rate = k 1[N0 2][F2]
(c) rate·= k2[F][N0 2]
(d) rate = k 1[N0 2FJ [F]
91. Four gases, C0 2 , S0 2, Cl 2 , and H20 are kept at the same temperature and pressure
in separate containers of equal volume. The gas which has the maximum density
is
·(a) C0 2 (b) S02 (c) Cl 2 (d) Hp.
640 PROBLEMS ON PHYSICAL CHEMISTRY

92. The phase diagram for a pure compound is shown below :

400
liquid

E'
(;J
300
'-'
~
::l
200-
"'~"'
0..

100

200 300 400


temp. (K)
At a temperature 'greater than 210 K and pressure of 60 atm, the compound exists as :
(a) both solid and gas at equilibrium
(b) vapour
(c) solid
(d) vapour and liquid in equilibrium.
93. The frequency of light of wavelength A. = 500 nm is :
(a) 599.6 Hz (b) 5.996 x 101 4 Hz
(c) 59.96 x 108 Hz (cl) 5.926 x 10- 14 Hz.
94. A photophysical process which leads to non-radiative decay is called :
(a) fluorescence (b) internal conversion (c) absorption (d) phosphorescence.
95. An allloy of gold and copper crystallizes in a cubic-lattice with gold atoms
occupying the corners of the cube and copper atoms the centre of each of the
cube faces. The empirical formula of the alloy is :
(a) AuCu (b) AuiCu (c) AuCu 3 (d) Au 3Cu.
96. Find the radius, r, of the smallest atom that can fit in the centre of an octahedron,
made from 6 ligand atoms, each with radius R.
(a) 0.2 R (b) 0.414 R (c) 0.65 R (d) R.
97. The point group symmetry of acetylene is :
(a) C2h (b) C2v (c) D_h (d) C-v·
98. The zero point energy of an electron confined in a I-dimensional box of length
a is :
(a) h2/(2rr,2ma 2 ) (b) h2/(4rc2ma 2 )
(c) h2/(8rc2 ma2 ) (d) h2/(l 6rc2ma2).
99. According to the equipartition theorem, the heat capacity of HF at constant volume
will be :
3R 7R
(a) - (b) SR (c) (d) 6R.
2 2
MISCELLANEOUS PROBLEMS 641

100. The 3d atomic orbital has


(a) one angular node (b) one radial node
(c) one angular and one radial nodes (d) two angular nodes.
101. The bond length of a homonuclear diatomic 'molecule can be determined by
(a) IR spectroscopy (b) Microwave spectroscopy
(c) Raman spectroscopy · (d) NMR spectroscopy.
102. What are the coordination numbers of cs+ and c1- in the caesium chlorid~
structure?
(a) 4 (b) 6 (c) 8 (d) 12.
103. In a diffraction experiment, Bragg reflections are observed for an fee lattice, only
when:
(a) h, k, I values are all odd or all even
(b) h, k, I values are all odd
· (c) h, k, I values are all even
(d) h+k+l values are even:
104. According to band theory of solids, conduction occurs in metals because
(a) valence ban~ is full
(b) band gap has a finite value
(c) the valence band and conduction band overlap
(d) transition occurs from valence band to conduction band
105. The zero point vibrational energy for the deuterium (D 2) molecule is
(a) low.er than that for H2 . (b) higher than that for H2
(c) equal to that for H 2 (d) equal to zero.
106. The vapour pressure of an aqueous solution of KN0 3 at 100 °C is 750 torr. The
activity of water then is : ·
(a) 0.987 (b) 1.01 (c) 1.00 (d) 0.
107. The maximum work that may be obtained from a chemical reaction is given by
the change in the :
(a) enthalpy (b) entropy
(c) Gibbs free-energy (d) internal 'energy.
108. In the 1H NMR spectrum, a compound of molecular formula C4H6 exhibited only
a singlet at o1.8 ppm. The compounc!. is :
(a) 1-butyne (b) 2-butyne (c) 1,3-butadiene (d) cyclobutene.
109. The point group of [PtC1 4] 2- is :
(a) Cs (b) C 2v (c) D2d (d) D4h.
110. By the uncertainty principle, the uncertainty in position (&) and the uncertainty
in· momentum (!lp) of a particle are related as (h = Planck, constant) :
(a) &.!lp Sh/ 47t (b) &.!lp ;;::.h / 47t (c) &.!lp = 0 (d) &.!lp > 1.
111. With increasing temperature, the equilibrium constant of a reversible reaction tend.s
towards :
(a) 0 (b) 1 (c) -1 (d) oc.

Prob. Phy. Chem.--41


642 PROBLEMS ON PHYSICAL CHEMISTRY

112. The absorption of infrared radiation by a molecule is accompanied by a change in :


(a) the zero point energy (b) the dipole moment
(c) the nuclear spin (d) the electronic state.
113. A chemical reaction that is acompanied by a decrease in entropy occurs
(a) not at all (b) if the enthalpy change is sufficiently negative
(c) only at 0 K (d) if the enthalpy change is sufficiently positive.
114. In the SchrOdinger equation, H'I' = E'I', 'I' represents :
(a) wavelength (b) wave function (c) probability density (d) potential energy.
115. Suppose, the transformation of A to B occurs by both 'll reversible first-order reaction
and a reversible second-order reaction involving H+ ion.
k1 k3
A~B A+H+~B+H+
k2 k4
The relationship between these four rate constants is
(a) k 1k2 = k3k4(b) k 1k 3 = k 2k4 (c) k 1k4 = k2k3 .
116. A chemicaf... reaction will be favoured by :
(a) negative values of t'!J/ 0 (b) negative values of f).G 0
(c) positive values of ,15° (d) all of the three.
117. If the heats of reaction of the reactions :
. I I
C + 0 2(g) = C0 2(g), C + o2(g) = CO(g) and CO(g) +
2 ZOi(g) =COi(g) are
Q, Q 1 and Q2, respectively, then :
(a) Q = QIQ2 (b) Q =
QI + Q2 (c) Q =
QI - Q2.
118. Same quantity of heat is supplied to three reversible engines working between
heat res·ervoirs with the source temperatures of T 1, T2, T3, respectively, and the
sink temperature of T K. If T1 > T2 > T3, then the maximum work available will
be in the order :
(a) WI > W2 > W3 (b) W3 > W2 > WI (c) WI > W2 < W3.
119. The distance between 110 planes of a simple cube crystal with a lattice side of
1.5 A is :
(a) t.5 A (b) o.75 A (c) t.06 A Cd) o.53 A.
120. The half-life is independent of initial concentration of the reactant :
(a) only for a zero-order (b) only for a first-order
(c) for both zero and first-orders reaction.
121. Radiative transition from a higher excited singlet electronic state to a lower singlet
electronic state is called :
(a) fluorescence (b) phosphorescence (c) internal conversion.
122. According to the equipartition principle the heat capacity at constant volume (C)
for C0 2 gas is :
(a) 6.5R (b) SR (c) 9R (d) 8.5R.
123. The pair that shows critical solution temperature in a composition, temperature
phase diagram is :
(a) phenol-water (b) phenol-benzene (c) chloroform-water (d) methanol-water.
MISCELLANEOUS PROBLEMS 643

124. Under the equilibrium conditions for the reaction C(s) + C0 2(g) ~ 2CO(g),
the total pressure is 12 atm. The value of KP is :
(a) 16 (b) 0.5 (c) 2 (d) 32.
125. The crystal structure which does not have aqy axis of rotational symmetry is :
(a) triclinic (b) orthorhombic (c) hexagonal (d) cubic.
126. Among the following statements the one that is NOT true for a zero-order reaction
is :
(a) t 112 is directly propotional to the initial concentration of the reactant
(b) the time taken for 75% reaction is 1.5 times that of. t 112 .
(c) the rate constant has the dimension of concentration·time- 1.
(d) the concentration of the reactant decreases exponentially with time.
127. The ratio of the most probable velocity of hydrogen to oxygen gas is :
(a) 2.5 (b) 4 (c) 4.5 (d) 16.
128. For an irreversible adiabatic expansion of a perfect gas from volume V; to v the
1
change in entropy of the gas is :
(a) nR In WJ'V) (b) zero (c) less than zero (d) greater than zero.
129. The latent heat of vaporization of bromine ([) at 59 °C at 1 atm is 29.76 kJ/mote.
The tiU of this process is :
(a) 127 kJ (b) 29 kJ (c) 27.5 kJ (d) 27 kJ.
130. A solution containing 2.423 gm of a sulphur in 100 gm of naphthalene (m.p.
80.1 °C) gave a freezing point depression of 0.64 °C. The latent heat of fusion
of naphthalene is 35.7 cal/gm. The molecular. formula of sulphur in the solution
is :
(a) S4 (b) S2 (c) S6 (d) S 8.
131. In comparison to the frequency of the IR transition the NMR transition frequency
is :
(a) much higher (b) much lower (c) almost same (d) none of these.
132. The microwave active molecule is :
(a) C02 (b) HCl (c) p-dinitrobenzene (d) acetylene.
133. As per the_ uncertainty principle, !lx.!!.py =
(a) h (b) zero (c) /.... (d) h/2rt.
134. The dipole ~ment of toluene is m 1 debye. The dipole moment of a-xylene will
be:
(a) 3 {3 m 1 (b) {3 m 1 (c) m 1 (d) 0.
135. The non-radiative transition taking place between S 1 and T1 states is known as :
(a) internal conversion (IC) (b) intersystem crossing (ISC) (c) fluorescence
(d) phosphorescence.
136. For a linear molecule of n atoms in two dimensional space, the number of
vibrational degrees of freedom is :
(a) 2n - 6 (b) 2n - 5 (c) 2n - 3 (d) 2n - 4.
137. For the elementary gas phase reaction CO+ N0 2 ~ C0 2 +NO, the activation
energy is found to be 116 kJ.mo1- 1. The enthalpies of formation of CO, N0 2 , C0 2
644 PROBLEMS ON PHYSICAL CHEMISTRY

and NO are :_ 110 kJ.moi- 1, +33 kJ.moI- 1, -394 kJ.moI- 1 and +90 kJ.mo1-1,
respectively.
The activation energy in kJ.moJ- 1 for the reverse reaction is :
(a) 343 (b) -227 (c) 227 (d) 116.
138. The heat of a chemical reaction in solution may be determined by
(a) infrared spectroscopy (b) differential thermal analysis
(c) X-ray analysis (d). electron energy loss spectroscopy.
139. The term 'symbol' for the first excited state of the helium atom is :
(a) 150 (b) 351 (c) 3po (d) lp1·
140. The following diagram shows the heating curve for one mole of a s'ubstance that
melts at 250 K and boils at 600 K. The horizontal portion of the graph marked
'X' represents :

700
600
g
~ 500
~...
0
0.. 400
E
B 300
200
0 150 300 450 600 750
time (second)

(a) the heat required to bring the substance to its melting point.,
(b) the melting of the substance.
(c) the enthalpy of vaporization of the substance.
(d) the heat required· to boil the substance.
141. A constant current 200 mA is passed for one minute through a copper coulometer
to facilitate the reaction: Cu 2+ + 2e- - - 7 cu 0. The charge recorded by coulometer
is :
(a) 6C (b) 8C (c) 12C (d) 16C.
142. Of the following, the metal that is present in the respiratory protein haemocyanin
is :
(a) vanadium (b) iron (c) molybdenum (d) copper.
143. An involatile solute is dissolved in a volatile solvent. Which of the curves below
. best represents the variation of solvent vapour pressure P with solute mole fraction
X2:

p~l p~l pt=jl i~~~:J


0 X2 0 X2 . Xz . , Xz
(a) (b) (c) (d)
MISCELLANEOUS PROBLEMS 645
144. The equivalent conductance and the specific conductance of a saturated AgCl
solution are 138.3 ohm- 1·cm 2 and 1.81 x 10-6 ohm- 1·cm- 1, respectively. The
solubility of AgCl in equivalents·litre- 1 is :
(a) 1.31 x 10-5 (b) 1.31 x 10-4 (c) 1.31 x 10-3 (d) 1.31 x 10-2 .
145. The point groups to which CH 4 and CH3Cl belong, respectively, are
(a) Td and Td (b) Td and C3v (c) C4 v and C3v (d) C3v and C2v.
146. Choose the correct statement :
(a) Carbon dioxide gas does not show rotational Raman spectrum.
(b) Carbon dioxide gas does not absorb IR radiation.
(c) Carbon dioxide gas does not absorb microwave radiation.
(d) Carbon dioxide gas does not absorb in the UV region.
147. The dipole moment of chlorobenzene is 1.69 D. The dipole moment of
m-dichlorobenzene is expected to be roughly
(a) 1.69 D (b) 1.69./2D {c) 0 D (d) l.69./3D.
148. The number of nodal surfaces for an atomic orbital having principal quantum
number n and azimuthal quantum number I is equal to :
(a) n (b) l (c) n + l - I (d) n - l.
149. When an electron is removed from oxygen molecule to form o; ion,
(a) the bond length and bond order both increase.
(b) the bond length and bond order both decrease.
(c) the bond length decreases and bond order increases .
. (d) the bond length increases and bond order decreases.
150. In an experiment to determine the molecular weight of a substance using the
measurement of osmotic pressure, 50% of the compound was in the dimerised
form in the solution. If the actual molecular weight was 120, the measured
molecular weight would be :
(a) 240 (b) 60 (c) 180 (d) 160.

151. In a kinetics experiment, the observed rate law was - dC = kC 112 . The units of
dt
the rate constant would be :
(a) sec- 1 (b) M 112 sec- 1 (c) M- 112 sec- 1 (d) M- 1 sec- 112 .
152. The number of lines present in the ESR spectrum of methyl radical is
(a) one (b) four (c) three (d) six.
153. The number of formula units of NaCl present in a unit cell is
(a) one (b) two (c) three (d) four.
154. Arsenic in germanium is a :
(a) p-type conductor (b) n-type conductor
(c) ionic conductor (d) metallic conductor.
155. The reduction potentials forCu 2+(aq)+ e ~ cu+(aq) and Cu 2+(aq) + 2e ~ Cu(s)
are 0.15 and 0.34, respectively. Accordingly, the reduction potential for
Cu+(aq) + e ----t Cu(s) will be :
(a) 0.53 V (b) 0.19 V (c) 0.17 V (d) 0.05 \.
646 PROBLEMS ON PHYSICAL CHEMISTRY

156. Calcium carbonate is heated in a closed container and the equilibrium CaCOis)
~ CaO(s) + C0 2 is established. This system has :

(a) 3 phases and 2 independent components


(b) 2 phases and 2 independent components
(c) 2 phases and 3 independent components
(d) 3 phases and 3 independent components.
157. The solubility of Ag 2Cr0 4 is I x 10-4 M. The solubility product is
(a) 4 x 10- 1z (b) I x 10-8 (c) 2 x 10-1z (d) I x 10- 1z.

158. For a ls atomic orbital, the value of 1'1'12 is :


(a) a minimum at the nucleus
(b) a maximum at a distance of one Bohr from the nucleus
(c) zero at the nucleus
(d) a maximum at the nucleus.
159. A container is divided into two halves by a diaphragm. Both sides of the diaphragm
are filled with nitrogen at I atm pressure and at 30C K. When .the diaphragm
is removed, the antropy of mixing !!.S is :
(a) Af =0 (b) Af > 0 (c) Af < 0 (d) undt>fined.
160. In a first order reaction, the reactant concentration reduces from I mole/litre to
0.25 mole/litre in 100 min. Tne l:ialf-life of the reaction is :
(a) 50 min (b) 75 min (c) 200 min (d) 25 min.
161. When Hz gal': and 0 2 gas are at the same temperature, the average speeds of Hz
molecules and D 2 molecules are in the ratio :

(a) VH2 =I (b) VH2 = ../2 (c) VH2 = 2 (d) VH2 = 11../2.
vo2 vo2 Vo2 Voz
162. The number of molecules of an ideal gas in a volume of 8.2 litres at 380 torr
pressure and 27 °C temperature will be :
(a) 1.0 x 10z3 (b) 1 x wzz 2.00 .-----,.-----r---...----,.---r-1
(c) 6.02 x 102Z (d) 12.04 x 10z3.

163. The dependence of -


PV on tem-
1.80
RT
perature and pressure of gaseous f... 1.60
~
nitrogen is shown in the figure. ......
I;:..
We, therefore, can infer that : '-<., 1.40
(a) volume occupied by a mol- NI
ecule is less at lower pressure
1.20
(b) the molecules feel intermo-
lecular attractions only at large
separation . 1.00

0-80
0 200 400 600 800 I000

P/bar
MISCELLANEOUS PROBLEMS 647
(c) intermolecular attractive force is influenced by temperature
(d) even at zero pressure the molecules experience intermolecular attraction.
164. Crompton effect demonstrates that
(a) electrons have wave-like behaviour
(b) electrons have both wave- and particle-like behaviour
(c) photons have both particle and wave characteristics
(d) photons act like particles.
165. According to classical equipartition principle, the heat capacity of monoatomic
solids should be close to (in J/mole/K unit) :
(a) 22.4 (b) 24.1 (c) 24.9 (d) 27.6.
166. Among the following orbitals of a homonuclear diatomic molecule, the bonding
one is :
(a) lcru (b) 2cru (c) lrc (d) lrcu.
8
167. Among the following molecules, the shortest bond length is to be found in
(a) C2 (b) N 2 (c) 0 2 (d) F 2 .
168. Bond length of a homonuclear diatomic molecule can be measured using :
(a) microwave absorption spectroscopy
(b) Raman spectroscopy
(c) infrared absorption spectroscopy
(d) electron microscopy.
169. The state of one mole of an ideal monoatomic gas is changed reversibly from
(1 lit, 10 bar) to (10 lit, 1 bar) by two paths shown in the figure. The correct
statement is :
(a) work done by the gas and change of its internal energy are more for the
path-I than for the path-II.
(b) work done for bcith the paths is same but internal energy changes are different.
(c) work done by the gas for path-II is less but energy change for both the paths
is same.
(d) work done by path-I is more but internal energy of the gas in both the states
is the same.
170. For the transformation of the system
described in Q. 169,
(a) the entropy and the free energy
Path I
of the system will remain un-
changed 10
ta
.0
(b) entropy change of the system
will be more for path-I than for
path-II
(c) entropy will increase by the ~
l
same amount for both the paths
(d) entropy and free energy changes
I
cannot be computed based on Path II
the given data. l 10
--v-Oit)
648 PROBLEMS ON PHYSICAL CHEMISTRY

171. For an. adiabatic reversible expansion of an ideal gas, the plot of log P versus
log V is linear with a slope equal
.
to (y = Cp /Cv} :
(a) l!y (b) -1/y (c) r (d) -Y.
172. The half-life of a reactant in a chemical reaction for initial concentration 0.1 and
0.4 mol/lit are 200 sec and 50 sec, respectively. The order of the reaction is :
(a) 0 (b) 1 (c) 2 (d) 3.
k1 k
173. For a bimolecular reaction, A + B AB -1. C. The product fonnation is
k2
observed to follow the rate law, rate= (k 1kz!k_ 1) [A](B]. Therefore, we can infer
that :
(a) k 1 > k2 > k_ 1 (b) k2 > k 1 > k'- 1 (c) k_ 1 » k2 (d) k 1 < k_ 1
174, The pH of a buffer solution containing 4 x 10- 3 mole of acetic acid (pK0 = 4.76)
and 0.4 mole of sodium acetate will be :
(a) 6.76 (b) 4.76 (c) 2.76 (d) 0.76.
175. The law that relates the solubility of a gas to its pressure called
(a) Raoult's law (b) the distribution law (c) Henry's law (d) Ostwald's law.
176. A crystal having the lattice parameters a = b -;t!.c and ex = p= 90°
(a) is monoclinic. (b) is tetragonal or hexagonal.
(c) is orthorhombic. (d) has y = 90°.
177. For a semiconductor at 0 K :
(a) there are no states at the Fenni energy
(b) the conduction band is fully filled
(c) the valence and conduction bands overlap
(d) the electrical conductivity is finite.
178. For which of the following systems is a quantum number of zero not allowed?
(a) rigid rotor (b) harmonic oscillator
(c) particle in a box (d) anharmonic oscillator.
179. If x is a position operator and Px, PY' Pz are the momentum operators along
the x-, y- and z- directions respectively, with P2 = P; + P; + P1, then which of
the following commutators is non-zero?
(a) [x, Px] (b) [P 2, Px] (c) [P 2, Pz] (d) [x, Pz].
180. Let l/11 .and l/fz be the only linearly independent eigenfunctions having identical
eigenvalues E. Which of the following statements is true?
(a) l/f1l/12 is also an acceptable eigenfunction for the same state with eigenvalue
£2.
(b) I/Ii l/12 is also an acceptable eigenfunction for the same state with eigenvalue
E.
(c) al/f1 + bl/f2 is also an acceptable eigenfunction for the same state with
eigenvalue 2E.
(d) al/f1 + bl/f2 is also an acceptable eigenfunction for the same state with
eigenvalue E.
MISCELLANEOUS PROBLEMS 649
181. A black body kept at room temperature would emit broad band radiation peaking
in the:
'
(a) infrared, (b) visi!Jle (c) radio-frequency (d) X-ray.
182. The graph below shows the constant volume heat capacity Cv for· 1 mole of a
diatomic molecule in the gaseous state as a function of temperature. R is the gas
constant.

(7/2)R

CV (512)R

(3/2)R

IO 100 1000

T(K)

Which of the following statements is not true for the system?


(a) Below 100 K Cv is due largely to translational degrees of freedom.
(b) Vibrational degrees of freedom contribute significantly to Cv only above about
1000 K.
(c) Cv for translation and rotation is ,,, (5/2) R.
(d) Between IO K and 100 K, rotational degrees contribute most to Cv.
183. The graphs below show the P-V isotherms of an ideal gas at the temperatures
T1 and r 2. In which of these does the dashed line correspond to the process of
adiabatic compression between the volume V1 and V2?

(a) (b)
p p

Tl
Tl
Tz
Tz
Vz v vt
Vz v vi

(c) (d)

p p

Tl Tl
T2 Tz
vi v v2 vi v Vz
650 PROBLEMS ON PHYSICAL CHEMISTRY

184. The graph below shows the approximate variation of the molar Gibbs free energy
Gas a function of the pressure Pat constant temperature for diamond (solid line)
and g~aphite (dashed line).

diam on~
____---;--;, ; ~
~

graphite

15 kbar
p
Which of the following statements does not follow from the figure?
(a) Below 15 kbar, graphite is more stable than diamond.
(b) The molar volume of graphite is less than that of diamond.
(c) The molar volume of graphite is more than that of diamond.
(d) Diamond is more stable than graphite above 15 kbar.
185. The graph below shows the P-T phase diagram of pure water :

y
p

T
Which of the following statements about the figure is not true?
(a) At the point Y, water has .one degree of freedom.
(b) Point X is the triple point of water.
(c) The molar entropy of water at Q is greater than that at Z.
(d) Solid and liquid water coexist at the point R.
186. Which of the following has the highest heat capacity according to the classical
equipartition theorem?
(a) HCN (b) ClF (c) Hp (d) Ar.
187. If the vibrational frequency of HCI is v, the vibrational frequency of DCI
(deuterium chloride) is :
(a) (l/2)v (b) v (c) (ll./2)v (d) ../2v.
188. Two identical bubbles of air form at the bottom of a lake, then rise to the surface,
expanding as they do so (because the pressure at the surface of the lake is less
than that at the bottom). Bubble A rises isothermally while bubble B rises
MISCELLANEOUS PROBLEMS 651
adiabatically. Assuming the air in the bubbles is a monoatomic ideal gas, we can
conclude that at the surface :
(a) volume of bubble A > volume of bubble B
(b) volume of bubble A < volume of bubble B
(c) volume of bubble A = volume of bubble B
(d) the volumes are indeterminate.
189. Two different monoatomic ideal gases A and B, each with the same energy U,
volume V and mole number N occupies the two halves of a divided chamber.
If the partition between the chambers is removed and the gases are allowed to
mix, then the total change in entropy is given by :
(a) Nk 8 In 2 (b) Nk 8 In (1/2) (c) Nk 8 In 4 (d) (N/2)k 8 In 2.
190. A liquid is kept in a closed vessel. If a glass plate of negligible mass and with
a small hole is placed on top of the liquid surface, the vapour pressure of the
liquid in the vessel would :
(a) increase (b) remain the same (c) decrease (d) be changed in a manner that
cannot be predicted.
191. The dissipated work oWd done by a system at pressure P against an external
pressure Pext leading to a volume change dV is defined as
oWd = (P - pext) dV.
Which of the following statements is not true?
(a) awd > 0, for a spontaneous process.
(b) awd > 0, for a reversible process.
(c) awd > 0, when p > pext'
(d) awd > 0, when p < pext'
192. The emf of the cell Zn I Zn 2+( 1 M) II Cu2+( 1 M) I Cu is 1.1 V. If the standard
potential of zn+ 2 I Zn is -0.78 V, what is the oxidation potential of Cu I Cu 2+?
(a) 1.88 V (b) 0.34 V (c) - 0.34 (d) -1.88 V.
193. If a reaction goes to completion in finite time, the order of the reaction is :
(a) zero (b) first (c) second (d) third.
194. According to the equipartition principle the heat capacity at constant volume (Cy)
for ethylene gas is :
(a) llR (b) l.5R (c) 0.15R (d) .15R.
195. An aqueous solution coataining a mole of a non-volatile solute freezes at -0.186
°C. The elevation of boiling point of the same aqueous solution (Kf = 1.86,
Kb = 0.512). would be :
(a) 0.186 (b) 0.512 (c) 0.00512 (d) 0.0512.
196. In a system, when the chemical potential of each component is the same for all
the phases, the equilibrium is said to be :
(a) metastable (b) composition (c) thermal (d) mechanical.
197. Among the singlet (S), doublet (D) and triplet <n
electronic states, phosphore
scence involves transition between :
(a) S and S (b) S and T (c) D and D (d) T and T.
652 PROBLEMS ON PHYSICAL CHEMISTRY

198. Ml and !J.U for the reaction Fe20 3 (s) + 3H2(g) ~ 2Fe(s) + 3H20(1) at constant
tempef-ature are related as :
(a) Ml = !J.U (b) Ml = !J.U + RT (c) Ml = !J.U + 3RT (d) Ml = !J.U - 3RT.
199. The half-lite time for a reaction at initial concentration of 0.1 mol.1- 1 and
0.4 mol.1- 1 are 200 sec and 50 sec respectively. The order of the reaction is
(a) 0 (b) 2 (c) 3 (d) I.
200. On a particular day water boiled at 99.5 °C in Calcutta and 96.1 °C at Darjeeling.
Elevation of boiling point of a 0.0 I molal sucrose solution on that day was
(a) less in Darjeeling (b) equal both in Calcutta and at Darjeeling (c) less in
Calcutta.
201. As the frequency of the alternating current used in its measurement is increased,
the conductance of an aqueous electrolytic solution :
(a) attains a limiting maximum value (b) remains unchanged (c) decreases linearly.
202. The differential heat of dilution of a solution at any given concentration of the
solute, is the difference of enthalpies :
(a) per mole of solute in the solution and in its pure state.
(b) per mole of solvent in the solution and in its pure state.
(c) sum total of the two.
203. The buffer capacity will be maximum at the
(a) half neutralization point of a weak acid by a weak base
(b) half neutralization point of a weak acid by a strong base
(c) full neutralization of a strong acid by a strong base.
204. In a homogeneous reaction R ~ P, the time needed for the first 5% conversion
of R is independent of its initial concentration. The reaction is :
(a) zero order in R (b) first order in R (c) second order in P.
205. Joule-Thomson coefficient of a van der Waals gas is determined by
(a) only the pressure correction factor 'a'
(b) only the volume correction factor 'b'
(c) both 'a' and 'b'.
206. A saturated solution of Na 3P0 4 , which is completely dissociated to its component
ions, contains an excess of the salt. For the system
(a) P = 2, C = 2, F = 2
(b) P = 3, C = 3, F = 2
(c) P = 3, C = 2, F = I.
207. One mole of an ideal gas absorbs 300 calories of heat to expand irreversibly from
10 litres of volume at 27 °C to 100 litres of volume at 27 °C. The corresponding
change in entropy is :
(a) 4.605 e.u. (b) 1 e.u. (c) 11.1 e.u.
208. Two reactions I and II, have the same values of rate constants at 30 °C. But
activation energy of I is greater than that of II (E1 > E 11 ). On raising the temperature
to 50 °C, the relative increase in rates of the two reactions will be :
(a) less for reaction I (b) more for reaction I (c) the same for both the reactions.
MISCELLANEOU&. PROBLEMS 653
209. For the reactiQJ1 N2 (g) + 3H2 (g) ==
2NH 3(g), the thermodynamic equilibrium
cbnstant Ka has the unit of :
(a) atmosphere-2 (b) mole-2.litre 2 (c) number.
210. For a reversible reaction having Ka < 1 and /1H 0 < 0 at a given temperature :
(a) !:i.G 0 and !:i.S 0 are both +ve
(b) !:i.G 0 is +ve, t:i.S 0 is -ve
(c) !:i.G 0 is -ve, t:i.S0 is +ve.
211. As the temperature of a gas increases, the difference in population of gas molecules
having most probable velocity and average velocity
(a) decr~ases, (b) increases, (c) remains the same.
212. Molal boiling point elevation constants of water and ethanol are 0.51 and 1.22
respectively at l atmosphere pressure. A 0.01 M solution of urea will have osmotic
pressure :
(a) same both in ethanol and in water (b) higher in water (c) higher in ethanol.
213. For a reaction A + B -7 Product, the overall rate of formation of production is
doubled when concentration of A only is doubled. However, when that of B only
is doubled the rate remains unaffected. The overall order of the reaction is
(a) zero (b) one (c) two.
214. Energy of activation is :
(a) free energy change of overall reaction
(b) potential energy difference between the transition state and the reactant
(c) potential energy difference between the transition state and the product.
215. The molar free energy of a gas is invariant at
(a) constant temperature (b) constant pressure (c) constant temperature and constant
pressure.
216. The pH .at half-neutralisation point of a weak acid by a strong base is given by :
1
(a) pH =pK 0
(b) pH = 2pK 0
(c) pH = -zPKa.
217. When a pure liquid coexists with the vapour and solid, the system is :
(a) bivariant (b) mono-variant (c) non-variant.
218. When one mole of ice is converted to water at 0°C and l .atm, the work done
(l.atm). is :
(a) l.l x· 10-4 (b) 2.0 x 10-3 (c) 2.0 x 10-4 (d) l.l x 10-s.
219. For a zero order reaction, unit of the rate constant is expressed as
(a) M,1s- 1 (b) M°s- 1 (c) M- 1s- 1 (d) MOsO
220. When l Oo gm of water is reversibly heated from 50 °C to 75 °C at l atm, the
change in entropy (J.K- 1) of the universe is
(a) 0.35 (b) 0.31 (c) 0 (d) 3:f.

221. If the electron were spin ~ particles, instead of spin _!_, then the number of
2 2.
electrons that can be accomodated in a level are :
(a) 2 (b) 3 (c) 4 (d) 5.
654 PROBLEMS ON PHYSICAL CHEMISTRY

222. For a particle in a cubic box, the total number of quantum numbers needed to
specify its state is :
(a) I (b) 2 (c) 3 (d) 9.
223. With increasing pressure, the temperature range over which the liquid state is stable :
(a) decreases (b) increases (c) remains constant
(d) decreases till the critical pressure and then increases.
224. Given that the most probable speed of oxygen gas is 1000 m.s- 1, the mean/average
speed (ms- 1) under the same condition is :
(a) 1224 (b) 1128 (c) 886 (d) 816.
225. I x I o-6 moles of the enzyme carbonic anhydrase dehydrates H2C03 to produce .
0.6 mo! of C0 2 per second. The turnover number of the enzyme is :
(a) NA x 6 x 10-5 (b) (1/6) x I0- 5 (c) (6 x I05)/NA (d) 6 x 105.

226. For a reaction with rate equation - dC = KC1, C0 and C are the concentrations
dt
of the reactant at time 0 and t respectively. If IO minutes were required for C0
to become Cof2, the time required for C0 to become Cof4 is :
(a) IO min (b) 20 min (c) 30 min (d) 40 min.
227. For an ideal solution formed by mixing of pure liquids A and B :
(a) Aflmixing =0 (b) Aflmixing < 0 (c) Aflmixing > 0 (d) ~mixing ~ 0
228. The relationship between the equilibrium constant K1 for the reaction :

CO(g) + °2I Oz(g) ~ C0 2 (g)

and the equilibrium constant K2 for the reaction :


2CO(g) + Oz(g) ~ 2C02(g) is :

(a) 2K 1 = K2 (b) K1 = Ki (c) K 1 = K2 (d) K12 = K2.


229. For H-like atoms, the ground state energy is proportional to :

(a)
µ
(b) -
z2 (c) µZ2
l
(d) -:-:-::;r.
? µ µZ
where µ is the reduced mass and z is the nuclear charge.

230. The value of the integral Je-x x dx


2
is :

231. For the raction aA ~ products, the plot of l~I versus time (t) gives a straight

line. The order of the reaction is :


(a) 0 (b) I (c) 2 (d) 3.
232. The pH of a solution prepared from 0.005 mole of Ba(OH) 2 in 100 cc water is :
(a) IO (b) 12 '(c) 11 (d} 13.
MISCELL.ANEOUS PROBLEMS 655
233. For an electron whose x-positional uncertainty is 1 x 10- 10 m, the uncertainty
in the x-component of the velocity in ms- 1 will be of the order of :
(Data : me =9 x lo- 31 kg, h = 6.6 x 10-3 4 Js)
(a) 106 (b) 109 (c) 10 12 (d) 10 15.
234. For the following system in equilibrium CaC0 3(s) ==
CaO(s) + C0 2(g), the
number of components (C), phases (P) and degrees of freedom (F), respectively,
are
(a) 2,2,2 (b) 1,3,0 (c) 3,3,2 (d) 2,3, l.

235. Given that E°Fe 2+ /Fe = -0.44 V and E°Fe 3+ /Fe 2+ = 0.77 V, the E°Fe3+/Fe is :

(a) 1.21 V (b) 0.33 V (c) -0.036 V (d) 0.036 V.


236. The solid-liquid phase diagram
700 ~---------------.
for the Mg-Zn system is shown
in the adjoining figure where the 600

vertical line at X(Mg) = 0.33 u 500


represents the formation of a E::: 400 1:!#~--#-3--1
0
#7
congruent melting compound
MgZn 2 . The figure is divided into 300 #4 #6
seven regions depending upon 200+-~--,r--+---r--..--.---.--...--.--i
0.00 O. IO 0.20 0.30 0.40 0.50 0.60 0.70 0.80 0.90 1.00
the physical state of the system.
Zn x (Mg) Mg
The composition of the region #6
represents :
(a) single phase of a solution of Mg and Zn
(b) two-phase region between solid Zn and solid MgZn 2
(c) two-phase region between liquid and solid MgZn 2
(d) two-phase region between solid Mg and solid MgZn 2 .
237. In the extraction of metals from their ores, which one of the following reduction
methods can bring about a non-spontaneous reduction?
(a) electrolytic reduction (b) reduction by carbon (c) reduction by another
metal (d) reduction by hydrogen.
238. The plot of Gibb's free energy G and
i
the reaction A ==
the extent of a reaction ~ is given for
B. Ifµ A and µ 8 are
the chemical potentials of A and B
>.

i8
respectively, the Incorrect statement is <!::
"'
(a) at point X, µA > µ 8 ~ _ _ _ _ __
o.__
(b) at point Y, !lG = 0 0 Extent of reaction 4

(c) at point Z, µA > µ 8


(d) at equilibrium, the composition of
the reaction mixture can be identified.
239. For a cyclic process performed by an ideal gas, changes in some thermodynamic
functions are zero. Indicate the set in which all the functions are zero.
(a) W, llU, !:ili, l'lG (b) Q, t::.S, !:ili, M (c) Q, !lU, t::.S, !lG (d) !lU, t::.S,
!:ili, M.
656 PROBLEMS ON PHYSICAL CHEMISTRY
~

240. Which one of the follo';Ving species is the conjugate base of Ho-?
(a) Hp (b) o2- (c) 02 (d) O~.
241. An aqueous solution containing O.OlM FeCl 3 and 0.06M HCI0 4 has the same
ionic strength as a solution of : •
(a) 0.09M NaCl (b) 0.04M Na2S0 4 (c) 0.06M CuS0 4 (d) 0.03M H 3P0 4 .
242. Which one of the following figures, showing kinetic energy of the ejected electron
versus the frequency (v) of the incident photon, represents the Einstein's
photoelectric effect?

(a) (b) (c)


~L'_. (d)

o v(r')~

243. The standard potential of a Daniel cell is + I.IO V and the equilibrium constant
for the cell reaction is l.5 x 1037 . It can be concluded that :
(a) zinc oxidises copper (b) displacement of copper by zinc goes to near
completion (c) copper oxidises zinc (d) displacement of zinc by copper goes
to completion :
244. The normalisation constant 'A' for' the wave function V(<I>) = Ae0m$) where 0 :5
<I> :5 '..}[ is :
(a) 1/-ffii, . (b) ~ (c) 27t (d) 1/../2.
245. The pH of a l.O x 10-3 M solution of a weak acid HA is 4.0. The acid dissociation
constant K0 is :
(a) l.O x 10-3 (b) l.O x 10-4 (c) 1.0 x 10-5 (d) 2.0 x 10-5
246. The overlap between the atomic orbitals sketched below is :

~y-
~x+~x
zvy
(a) pos1t1ve (b) negative (c) zero (d) no overlap.
247. The ionic radii of Ca 2+ and P- are 100 pm and 133 pm respectively. The

lli ·
coordination number of Ca 2+ in the ionic solid will be :
(a) 8 (b) 6 (c) 4 (d) 2.
248. The phase diagram of a pure substance is sketched here.
The num~ of degrees of freedom at points P 1, P2 and P3, ! p
3
P'];
respectively, are : · •P 1
(a) 2, l, 0 (b) 1, 2, 0 (c) 2, 0, l (d) 0, 2, l. T~

249. The solubility products (Ksr) for three salts MX, MY 2 and MZ3 are 1 x 10-8,
4 x 10-9 and 27 x 10-8, respectively. The solubilities of these saltsfollow the order :
(a) MX > MY 2 > MZ 3 (b) MZ 3 > MY 2 > MX (c) M~ > MX > MY2
(d) ·MY 2 > MX > MZ3.
MISCELLANEOUS PROBLEMS 657
250. The temperature <n
dependence of the equilibrium Constant (K) of a chemical
reaction is correctly described by the following statement :
(a) For an endothermic reaction, the slope of In K vs. l/T plot is positive.
(b) For an exothermic reaction, K is proportional to T.
(c) For an exothermic reaction, K at a higher temperature is lower th.an K at
a lower temperature
(d) If t'l.H is independent of temperature, the change in K with T is smaller at
lower temperatures.
251. When the concentration of K+ across a cell members drops from 0.01 M to 0.001
M, the potential difference across the members is :
(a) 0.0 V (b) 0.0059 V (c) 0.059 V (d) 0.59 V.
252. The statement that is correct for both electrochemical (galvanic) cells and
electrolytic cells is :
(a) !t..G =-nFE (b) free energy decreases in bOth cells
(c) the cell potentials are temperature independent
(d) chemical energy is converted ioto electrical energy in both cells.
253. The molar heat capacity at constant volume of a colourless gas is found to be
25 J.mol- 1K-.1 at room temperature. The gas must be : •
(a) N 2 (b) 02 (c) C0 2 (d) S02 .
I
254. The wave function for a particle (moving in a hng) is (27tf2 exp{2icp), where·
cp is the polar angle. The prabability of finding the particle in a small interval

dcp when the value of cp =~ is :


2
(a) dcp (b) d¢i/27t (c) dcp exp (i7t) (d) dcp exp (irt)/2rt.
255. An electric current of 0.965 ampere is passed for 2000 seconds through a solution
containing [Cu(CH3CN) 4]+ and metallic copper is deposited at the cathode. The
amount of Cu deposited is :
(a) 0.005 mol (b) 0.01 mol (c) 0.02 mol (d) 0.04 mol.
256. The Maxwell-Boltzmann distribution for molecular
speeds is shown in the following figure.
In the figure, H is the height of the peak, L is the
location of the maximum and Wis the width at
half-height. As the t~mperature is decreased,
i
(a) H increases, L decreases and W increases
(b) H increases, L decreases and W decreases
(c) H decreases, L decreases and W increases
L v--+
(d) H decreases, L decreases and W decreases.
257. A system undergoes two cyclic processes 1 and 2. Process is reversible and
process 2 is irreversible. The correct statement relating to the two process is :
(a) llS (for process 1) ::: 0, while llS (for process 2) ;t: 0
(b) Qcyclic ::: 0 for process 1 and Qcyclic ;t: 0 for process 2

Prob. Phy. Chem.-42


658 PROBLEMS ON PHYSICAL CHEMISTRY

(c) more heat can be converted to work in process I than in process 2


(d) more work can be converted to heat in process I than in process 2.
258. The molar internal energy of a gas at temperature T is Um(T). The molar internal
energy at T = 0 is' Um(O). The correct expression· that relates these two with
appropriate contributions is :
(a) Um(1) = Um(O) + 3RT [linear molecule; translation only]

(b) Um(1) = Um(O) + ~RT [linear molecule; translation and rotation only]
2

(c) Um(1) = Um(O) + 3-RT [nonlinear molecule; translation and rotation only]
2
(d) Um(1) = Um(O) + RT [nonlinear molecule; translation only]

259~ If a particle has linear momentum p=-2l + J+ k at position r =3l - J+ k, then


its angular momentum is :

(a) T+2k (b) -2i -5] +k (c) 5i -2} (d) 2t +5}-k.


260. If 'If is the eigenfunction to the Hamiltonian operator with ex as the eigenvalue,
then ex must be
(a) positive (b) negative (c) an integer (d) real.
261. A quantum mechanical particle of mass m free to rotate on the surface of a sphere
2
. r 1s
o f rad ms . .m the state wit !Oli- • Th e. degeneracy o f th'1s state 1s
. h energy - . :
mr 2
(a) 20 (b) 10 (c) 9 (d) 4.
262. Choose the INCORRECT statement among the following :
(a) 'Yhen ideal gases are mixed, the entropy of mixing is always positive.
(b) At equilibrium, the chemical potential of a species is the same in all of the
phases of the system.
(c) The total pressure of a mixture of ideal gases is equal to the sum of the
partial pressure of each gas in the mixture.
(d) When a gas is allowed to expand, the maximum work is obtained when the
process is carried out irreversibly.
. 263. The work done during the free expansion of one mole of an ideal gas at 27 °C to
twice its original volume is (given : RT =2494 J mo1-!, In 2 =0.7, log 2 = 0.3) :
(a) 1746 J mo1- 1 (b) -1746 J mol- 1 (c) zero (d) 748.2 J mo1- 1
264. Choose the correct order of the diffu~on coefficients of the following at 298K. .
P : H+ in water, Q : OH- in water, R : H 2 0 in water, S : Sucrose in water,
(a) P > Q > R > S (b) S > R > Q > P (c) S > Q > R > P .
(d) P > R ::> Q > S.

265. Two matrices are given as X = G~) and Y = (~ ~} If xr is the transpose


of X, then what .would be xrY?

(d) 44 28)
( 12 12 .
MISCELLANEOUS P~OBLEMS 659
266. Addition of 1.0 gm of a compound to 10 gm of water increases the boiling point
by 0.3 °C. The amount of compound needed to propare a 500 ml of 0.1 M solution
is (given : assume negligible dissociation or association of the compound, boiling
point constant Kb of water =_ 0.513 K kg mol- 1) :
(a) 0.855 gm (b) 17.1 gm (c) 8.55 gin (d) 85.5 gm
267: The molar conductivity of 0.009 M aqueous solution of a weak acid (HA) is 0.005
S m 2 mo1- 1 and the limiting molar conductivity of HA is 0.05 S m2 mol- 1 at
298 K. Assuming activity coefficients to' be unity, the acid dissociation constant
(K0 ) or HA at this temperature is
(a) l x 10--4 (b) 0.1 (c) 9 x 10--4 (d) l.l x 10-5 .
268. The structure of rock salt consists of :
(a) a cubic close-packed array of anions with cations in all the octahedral sites
(b) a cubic close-packed array of cations with anions in all the tetrahedral sites
(c) a hexagonal close-packed array of anions with cations in all the octahedral sites
(d) a cubic close-packed array of anions with cations in all the tetrahedral sites.
269. For an ideal gas the plot that is nonlinear is :
(a) PY vs. T (b) PV vs. P, at constant T (c) P vs. V, at constant T
(d) In P vs._ In V at constant T.
270. Consider two identical containers, one with l mole of H2 and the other with l
mole of He. If the root-mean square (RMS) velocities of the two •gases are the
same then the ratio of the temperatures, T(H2)/T(He) is
(a) 1/2 (b) 2 (c) 1/{2 (d) ./2.
271. An electron moves around the nuc~us in a circular
orbit according to the Bohr model. The radial
vector r
and the instantaneous linear momentum
vector p are shown in the diagram :
The direction of the angular momentum vector is :
(a) along r (b) along p (c) opposite to p
(d) perpendicular to both r and p.
272. X and Y are transformed co-ordinates obtained from p and q as follows

The correct set of linear equations that represent X and Y are :


(a) X=a1p+aiq (b) X=a 1p+°')q (c) X=aip+a 4 q (d) X=a 1p+a4q
Y=°'Jp+~q Y=aip+~q Y=~p+°'Jq Y=<Jip+°')q
273. Which of the following is not a solution of the equation :
d2x 2
dt2 +cox = 0.
(a) x =A cos coi (b) x =A sin cot (c) x = At2 (d) x = ~(eiwr +e-iwr).
660 PROBLEMS ON PHYSICAL CHEMISTRY

274. An electron is found in an orbital with one radial node and two angular nodes.
Which orbital the electron is in? ·
(a) ls (b) 2p (c) 3d (d) 4d.
275. The acceptable valence shell electronic arrangement is
(a)
Q!l Q!l [!] D (b) [!!] [!] [!] [!]
2s 2p 2s 2p

(c)
Q!l [!] [!] [!] (d)
Q!l [!] [!] [!]
2s 2p 2s 2p
276. If Ksp is the solubility product of a sparingly soluble salt A3X 2 , t~en its solubility
is :
1/3 (
(a) ( K,p/108 ) (b) K,P )1/5 (c)
(
K,.P/72 )1/5 (d)
(
K,P )1/2 .
277. For the formation of B from A, heat liberated is 20 kJ mo1-- 1• If the activation
energy for the reaction B ~A is 100 kJ mol-- 1, then the activation energy (in
kJ moJ-- 1) for the reaction A~ B is :
(a) 120 (b) 100 (c) 80 (d) 60.
278. For the reaction A + B - t Z, the concentration of Z at time t is given by [Z]
= [A] 1 = 0 ( 1 - e--kt) + [Z] 1 = 0 , where k is the rate constant. The rate law is :

(a) - d[ Z] = k[A] (b) d[ Z] = k[A] (c) d[ z] ::: k[A] (d) d[ Z] = k[A][B].


dt dt dt dt .
279. Among the following, the incorrect statement is :
(a) diamond and graphite are two allotropes of carbon
(b) in diamond, each carbon is sp 3 hybridized
(c) in graphite, each carbon is sp2 hybridized
(d) graphite shows high electrical conductivity in one direction only.
280. The pH of a 1 x 10--8 M HCl solution is close to :
(a) 8.0 (b) 7.l (c) 6.9 (d) 6.0.
281. The indicator phenolphthalein changes colour at pH - 9. This indicator is not
suitable for accurate determination of the end point in the titration of :
(a) CH3COOH with NaOH (b) HCI with NH4 0H
(c) HCI with NaOH (d) HCI with KOH.
282. Based on the principle of equipartition of energy, the molar heat capacity of C0 2
at constant volume cv.m is :
(a) 3.5R (b) 6R (c) 6.5R (d) 9R.
283. One mole of a van der Waals gas undergoes reversible isothermal transformation from
an initial volume V1 to a final volume V2. The expression for the work done is :

V2 -b
(b) -RT ln--.+a I --
(- I )
V1 -b V1 V2

(c) RT In Pz
P1
(d) V2--b
RT l n
V1-b
- - a (-
' -- .
V1 V2
I)
MISCELLANEOUS PROBLEMS 661

284. The scalar product of two vectors u and v, where u = 2i + 3j - 5k and v = ·i


+ j + 3k, is :
(a) -10 (b) 2i + 3j - l5k (c) 3i + 4j - 2k (d) 10.
285. The minimum concentration of silver ions that is required to s1:art the precipitation
of Ag2S (Ksp = l x io- 5 1) in a 0.1 M solution of s2- is :
(a) l x io-49 M (b) 1 x 10-50 M (c) 1 x 10-26 M (d) l x io-25 M.
286. Identify the correct statement regarding Einstein's photoelectric effect :
(a) The number of electrons ejected depends on the wavelength of incident
radiation.
(b) Electron ejection can occur at any wavelength of incident radiation.
(c) The number of electrons ejected at a given incident wavelength depends on
the intensity of the radiation.
(d) The kinetic energy of the ejected electrons is independent of the wavelength
of incident radiation.
287. The hydrolysis constant (Kh) of NH4Cl is 5.6 x 10- 10 . The concentration of H3 o+
in a 0.1 M solutio~ of NH4 Cl at equilibrium is :

(a) .J5.6x 10- 11 (b) ~5.6x 10- 10 (c) 5.6 x 10- 10 (d) 2.8 x lo-s
288. The acid dissociation constant (Ka) for HCOOH, CH3 COOH, CH2 CICOO~ and
HCN at 25 °C are 1.8 x 10-4, 1.8 x 10-5 , l.4 x. io-3 and 4.8 x 10- 10 , respectively.
The acid that gives highest pH at the equivalence point when 0.2 M solution of
each acid is titrated with a 0.2 M solution of sodium hydroxide is :
(a) HCOOH (b) CH3COOH (c) CH 2CICOOH (d) HCN.
289. For an ideal gas undergoing reversible Carnot cycle, the plot of enthalpy (H) versus
entropy (S) is :

(a) Hl_S.:;i_ Hl_g_ Hlg_ Hlg_


(b) (c) (d)

s s s s

Short Questions
1. Devise a cell in which the following reaction takes place. Write down the half-
cell reactions. 2H 2(g) + 0 2(g) = 2H 20(1) [Ans. H2 I OH-I 0 2]
2..Calculate the minimum pressure needed to convert diamond to graphite at 298 K.
Given, densities of diamond and graphite remain constant at 3.52 gm.cm- 3 and 2.25
gm.cm- 3 respectively, and 6.G'} of diamond is 2.90 kJ per mole at 298 K. ( 1.51 x
109 Pa)
3. (a) An ideal gas at 300 Kand l.O atm pressure was passed at a rate of 100 ml/sec
through a tube which is thermally insulated from outside, but fitted inside with a
heater which dissipates heat at a rate of 0.5 J per second. The gas flows into the
atmosphere at a temperature of 30. l K. Calculate the heat capacity ratio, y, of the
gas. [Ans. 1.4]
662 PROBLEMS ON PHYSICAL CHEMISTRY

4. (a) For a noncatalytic ·reaction, A ~ 8, the conversion is linear with time. The
reaction goes to completion in 4 hours at 300 K and in 2 hours at 315 K.
Find out the order of the reaction, and the energy of activation.
[Ans. 36.31 kJ]
(b) Two glass capillary tubes, one with a square bore and the other with a circular
bore of same cross-sectional area, were dipped into water. Show analytically
in which case there will be higher capillary rise. [Ans. square]
5. When an excited nucleus decays, it emits gamma-rays. If the half-life of radioactive
decay is 69.3 x 10- 14 second, what is the uncertainty in the energy of the emitted
gamma-rays? [Ans. 3.804xI0-23 J]
6. Calculate the probability that a particle in a one-dimensional box of side a, is
I
to be found between 0 and a I 2. [Ans. -]
2
7. (a) Calculate the unit cell dimensions of NaCl if its density is 2.17 kg.m-3.
[Ans. a = 5.75 nm]
(b) For a reaction 2A + B ~ C, if the concentration of A is doubled the reaction
rate increases by a factor of four. If the concentration of B is tripled, the
reaction rate increases by a factor of nine. What is the ,1rder of the reaction
with respect to. A and with respect to B? Whrn B is present in large excess
and the-initial concentration of A is 0 1 moles/litre, the concentration of A
decreases to ·0.05 moles/litre ·after 5 ,,econds. What is the rate constant of
the reaction? What 1s the re:>ction rate?
[Ans. r = kC~C~, k = 2 1.mol- 1.sec- 1]
(c) Write •h..: SchrOdinger equation for the electronic wave function of H 2 .
8. (a) Show that in a simple cubic structure only about 52% of the space is filled
by the atoms or spherical ions constituting the lattice.
(b) Show that for the CsCI structure the minimum cation to anion radius ratio
is 0.732.
(c) At the triple point of water, the degree of freedom is zero, and the degree
of freedom is also zero for the liquid-vapour equilibrium at 300 K-explain.
(d) It is easier to break chemical bonds at higher temperatures-justify.
(e) All vibrations of C0 2 are not infrared active-{!xplain.
9. (a) Is it possible to measure precisely and simultaneously the x-coordinate and
the y-component of the linear momentum of an electron? Justify in brief.
(b) Functions: 'l'i and '1'2 are individually normalized and are mutually orthogonal
to each other. Normalize 'I' = 'I' 1 + 3 112'1'2 .
(c) The experimental results
of the photoelectric effect
data for metals Cs and Na
x x
are given in the figure.
Na X
Explain why the Cs line
is to the left of the Na line
in terms of the atomic
structure ideas known to
you.
5.2 5.6 Frequency (in 10 14 Hz)
./..
MISCELLANEOUS PROBLEMS 663

(d) Consider the folllowing wave function for a particle in a box.

'P(x)
0 L
3U4

At what positions is the likelihood of finding the particle maximum? At what


L 3L . L
positions is it minimum? [Ans. max - , - mm=-]
4 4 2
10. (a) At what compositions corresponding to the points (X, Y or Z) is the reaction
A ~ B not spontaneous? Why?

x z
Free energy tJ
y

PureA Composition of the mixture Pure B

(b) For a given substance, which line of Figure A below belongs to the gaseous
phase and which line ~elongs to the liquid phase? Why? (A = gas, B = liquid)

Property
Free energy
G

0
0 T~
T ~(Pressure constant)

Figure A Figure B

(c) For the reaction A + B = C, K is the thermodynamic equilibrium constant.


l
If ("dK/"dP)T = x, what values can x take and why? [Ans. -1
p
(d) For a reaction X2 + Y2 = 2XY, the dependence of the change in heat capacity
and entropy change of the reaction on temperature are shown in the Figure
B above. Are the reactants stabilized by heating? Rationalize.
664 PROBLEMS ON PHYSICAL CHEMISTRY

(e) Fill in the box C in the following. Calculate the entropy change for each step and
verify that entropy is a state function. Given, Cp(H20, l) =75.5 J.K- 1.mol- 1
. .
1
CµCH 20, g) = 33:8 J.K- .mol- 1. Mlvap(H 20, 0 °C) = 43.773 kJ.moI- 1. Mlvap
(HzO, l 00 °C) = 40.67 kJ.mo1-1.

z HzO (g), I atm, I 00 °C

HzO (g), O·O I atm, 100 °C E

B HzO (I), I atm, 100 °C HzO (g), O·O I atm, 0 °C D

A HzO (!), I atm, 0 °C


[Ans. HzO(I), l atm, J00°C]
11. (a) Equivalent conductance A of 0.0005N HCI is 422.74 ohm- 1.cm 2 at 25 °C.
Calculate the conductance of this solution. [Ans. 2.l lxl0-4]
(b) The accompanying figure shows conductometric titration curves for a weak
acid against a weak base, a strong acid against a strong base and a weak
acid against a strong base. Identify which one of these corresponds to I, II
and III.

II
[ I strong acid vs. strong base
II weak acid vs. strong base
III weak acid vs. weak base]
. , , . , - - - - - - - III

Volume of Base

(c) If the approximate molar conductance of a l : l electrolyte is 120 ohm- 1cm 2,


the molar conductance of a l : 2 electrolyte is :
(i) 120 ohm- 1.cm 2 (ii) 180 ohm- 1.cm 2 or (iii) 240 ohm- 1.cm 2.
(d) For a zero order reaction, the half-life depends on the logarithm of the initial
concentration. Is this statement correct? Justify your answer.
12. (a) Calculate the constant pressure molar heat capacity of hydrogen gas from
the data given below :
For the reaction, H2(g) + ..!..o2 (g) ~ H 20(g)
2
Mi0 (25 °C) = -241.8 kJ.mol-1, Afl 0 (100 °C) = -242.6 kJ.mol- 1. At constant
pressure, molar heat capacity of 0 2(g) = 29.37 J.K- 1.mol-2 , molar heat
MISCELLANEOUS PROBLEMS 665
capacity of H 20(g) =33.58 J.K- 1.mo1-- 1. Assume that the heat capacities are
independent of temperature. [Ans. 29.56 J.K- 1.moJ- 1]
(b) For the reaction, N2(g) + 2H2 (g) ~ 2NH3(g), f).G 0 =-33 kJ.mo1- 1 at 25 °C.
and
If the partial pressures of N2 (g), H 2(g), NH3 (g) are 3 bar, 1 bar and 4 ear
respectively, in which direction the reaction will process? [Ans. forward]
Using the data given above calculate the standard Gibbs function of formation
of NH 3? [Ans. -33 kJ.moJ- 1]
13. (a) A gas at 250 K and 15 atm has a molar volume 12% smaller than that
calculated from the perfect gas law. Calculate (a) the compression factor
(b) the molar volume of the gas. (c) Which forces are dominating--attractive
or repulsive? [Ans. (a) 0.88 (b) 1.203 (c) attractive]
(b) The change in the Gibbs function of a certain constant pressure process was
found to fit the expression. f).G = -92.6 + 40.3/T,
where f).G and T are in k.J and K respectively. Calculate the value of f).S
for the process at 25 °C. [Ans. 0.4533 J.K- 1]
(c) A monoatomic perfect gas is expanded to twice its volume and simultaneously
heated to twice its initial temperature. Calculate the change in its entropy.
[Ans. 14.41 J.K- 1.mo1-1]
(d) The fo1\owing data was recorded during the potentiometric titration of
25.0 ml of ammonium iron (II) sulphate solution with 0.1095 M cerium (IV)
sulphate solution using platinum and saturated calomel electrodes.

ce+4 solution added, ml E, mV M/f).V, mVml- 1

22·00 523

22·50 543 40

22·60 550 70

22·70 557 70

22·80 565 80

22·90 575 100

23·00 590 150

23·10 620 300

23·20 860 2400

23·30 915 550

23-40 944 290

23·50 958 140

24·00 986 56

Calculate the concentration of the ammonium iron(II) sulphate solution.


[Ans. 0.203 M]
-·.,

666 PROBLEMS ON PHYSICAL CHEMISTRY

14. The normal boiling point of a binary solution of A and B with mole fraction xA
= 0.6589 is 88 °C. At this tempera_ture the vapour pressures of pure A and B are
957 and 379.5 torr, respectively. (a) Is this solution ideal? (b) What is the
composition of the vapour abqve the solution?
[Ans. (a) ideal, (b) XA = 0.83, X8 = 0.17]
15. A monolayer of N 2 molecule is adsorbed on the surface of 1.00 gm of an Fe/
Al 2 0 3 catalyst at 77 K. Upon warming the N 2 gas occupies 2.86 cm 3 at 0 °C
and 760 torr. What is the surface area of the catalyst if the effective area of a
N2 molecule is 0.167 nm 2? [Ans. 12.83 m2]
16. (a) Calculate the resistance of a conductivity cell containing a solution with
specific conductance, JC =·0.001 ohm- 1.cm- 1, and electrodes l cm x 2 cm
are held 0.5 cm apart. [Ans. 250 ·ohms]
(b) The ionic radii of Mg 2+ io11 is 65 pm and 0 2- ion is 140 pm. Find the
coordination number in MgO.
[Ans. around each Mg+ 2 ion there are 6 c1- 1 ions and vice versa]
17. Using the following thermochemical equations,
Ag(s) ~ Ag+(aq) + e = 105.6 kJ
Af1

_!_ Clig) + e ---t Cl-(aq) Af1 =-167.2 kJ


2
AgCl(s) ~ Ag+(aq) + Cl-(aq) Af1 =65.5 kJ
predict the effect of raising temperature on the equilibrium constant of the reaction
given below :
I
Ag(s) + -Cl 2(g) ---t AgCl(s). [Ans. yield will decrease]
2 .
18. (a) The emf of the cell
Ag(s)jAgCl(satd.), KCl(0.05 mol.dm- 3 )! AgN0 3 (0.1 mol.dm-3 )jAg(s) is
0.43 I V at 298 K. The mean activity coefficient of KCI is 0.8 I 7 and that
of AgN0 3 is 0.723. Calculate the solubility product of AgCI at 25 °C.
[Ans. I .46 x 10-IOJ
(b) The conversion of A to B and C goes through the following mechanism

2A~I~B+C.
kz k4
Show that the equilibrium constant (K) for the overall reaction is : K = kik3 •
kzk4
(c) Express how the equilibrium of the reaction :
N2 + 3H2 ~ 2NH3
will be affected if inert gas Ar is added to the system at constant pressure
and constant volume. [Ans. yield decrel!ses, no effect]
19. (a) Using quantization condition for the de Broglie wavelength on a ring, derive
the expression for rotational energy levels of a rigid homonuclear diatomic
rotor of bond length 2r.
(b) The fundamental vibrational frequency of HCI is 2885 cm- 1. Assuming that
HCI and DCI may be treated as simple harmonic oscillator, calculate the
fundamental frequency of DCI (atomic mass of chlorine = 35 dalton).
[Ans. 2066 cm-1]
MISCELLANEOUS PROBLEMS 661.
(c) Find the condition under which the function f(x) = a sin ax + b cos ax can
be eigenfunction of -id/qx. [Ans. a = b] •
20. If 4 moles of a MX2 salt in 1 kg of water raises the boiling point of water by
3.2 K, calculate the degree of dissociation of MX 2 in the solution. (for water,
Kb = 0.5 K kg mol- 1) [Ans. a = 0.3] [JAM 2011]
21. Equilibrium constant for a reaction doubles as the temperature is increased from
300 K to 600 K. Calculate the standard reaction enthalpy (in kJ mol- 1) assuming
it to be constant in this temperature range. (given : R = 8.3 JK- 1 mol- 1, In 2
= 0.7) [Ans. 34.86 J] [JAM 2010]
22. A 50 mL solution of 0.1 M monoprotic acid (Ka = 1 x 10-6 at 298 K) is titrated
with 0.1 M NaOH at 298 K. Calculate the [H+] of the solution after the addition
of 50 mL of NaOH at this temperature. (given : Kw = 1 x 10- 14 at 298 K)
[Ans. 8.85] [JAM 2010]
23. One mole of an ideal gas initially at 300 K and at a pressure of 10 atm undergoes
adiabatic expansion
(i) reversibly and
(ii) irreversibly against a constant external pressure of 2 atm until the final
pressure becomes equal to the external pressure.
Calculate ~Ssystem for (i) and (ii). For (ii), express the final answer in terms
of R.

Given : Molar heat capacity at constant volume C\.!


,m
=32R
[Ans. (i) = 0, (ii) = 3.67 R] [JAM ·2011]
24. For the following equilibrium at 300 ·°C,
N 20 4 (g) ~ 2N02 (g).
Calculate KP when N 20 4 is 30% dissociated and the total pressure is 2 bar.
[Ans. 0.79] [JAM 2011]
668 PROBLEMS ON PHYSICAL CHEMISTRY

Short Questions without Answers


1. Define fine structure of spec_~al lines.
2. The frequency of _light incident on a metal surface is doubled. What will be the
effect on kinetic energy of emitted photoelectrons?
3. Write the dimension of angular momentum from its definition.
4. Write down the action integral for angular motion of electron in an atom.
5. Arrange the following molecules in order of increasing polarisability : HCI, HF,
HI, HBr.
6. Name the important ·physical quantity for a diatomic molecule, which is obtained
from its rotational constant 'B'.
7. For hydrogen ls-state, 1'1'1 2-shows maximum at the origin-state true or false.
8. Write down the values of L, S and J for the 2P312 term.
9. What is the main reason behind splitting of the atomic spectral lines into fine
structrue?
10. Is the photoelectric work function of a metal the same as its ionization potential?
11. Why does not the lowest energy state in H atom have n = O?
12. What is the potential of the half-cell H 2 (Pt) (0.01 atm) I H+(a = 0.1) at 298 K?
13. Give the cell representation of any standard cell.
14. Name an electrode which can be used to determine c1- ion concentration of a
NaCl solution.
15. Draw the differential plot for potentiometric titration of HCI vs. NaOH.
16. Express mean ionic activity of Al 2(S04 ) 3 in terms of ionic activities.
17. Suggest a suitable linear plot for variation of Freundlich isotherm.
18. Draw schematically an energy profile diagram for a uncatalysed reaction.
19. State Walden's rule.
20. Plot schematically µ1 T vs. 1/T for a van der Waals gas.

21. What is the dimension of _:!L_, where yand C respectively are surface tension
dlnC .
and concentration.
22. Write down the dimension of R.

23. Dra~ log k vs. {µ plot for inversion of sucrose catalysed by HCI.
24. Calculate Cv for H 2 according to the equipartition theorem.
25. Write down SchrOdinger time dependent equation.
26. Draw the wave function for the 2nd quantum state of a particle in a one-dimensional
box (0 ~x ~a).
27. Which one of the electronic transitions n-m* or 7r~7t*, will appear at longer
wavelength?

28. Write the dimension of it1 :e [0 is angle].

29. Give an example of a photosensitized reaction.

30. Find a suitable eigenfunction of the operator j_


ax
MISCELLANEOUS PROBLEMS 669
31. What is the dimension of entropy?
32. Remark on -the statement "solubility increases with temperature".
33. Define an ideal solution.

34. What condition is imposed on f dx for x to be a state function?

35. What is the relation between eV and J?


36. Show that (PdV - VdP) is not an exact differential.
37. Identify X and Y in Henry Jaw : X oc Y.
38. Remark if any reversible isothermal process is isoenthalpic.
39. Explain whether the partition co-efficient is independent of temperature.
40. What is meant by "critical solution temperature"?
41. Cite an example of an isothermal-isobaric process.
42. How many phases can generally exist for a I-component system at STP?
43. What is the unit of rate constant for a 2nd order reaction?
44. What is the dimension of surface tension?
45. Define "triple point". ·
46. For a certain equilibrium .1G 0 = 0, find the equilibrium constant.
47. Define chemical potential. What is its dimension?
48. Entropy of an isolated system is constant--criticize.
49. Draw curves to shbw the pressure dependence of compressibility factor (Z) for
H 2 and He.
50. The solubility prpduct of a sparingly soluble salt MA 3 is 2.7 x 10-7 . Find its
solubility assuming complete dissociation.
51. Indicate a simple experimental device by which pure HD can be distinguished
from an equimolecular mixture of H 2 and D 2 (D = heavy hydrozen).
52. Justify/criticise whether [iJ(G/D/ o( 1/T)]p is a state function.
53. What does 'zero' in 'zero-point energy' signify?
54. Arrange in order of increasing translational frequencies for electronic, NMR and
vibrational spectra.

55. Show that the function aed0 is an eigenfunction of the operator !!._. What is the
dx
eigenvalue?
56. Arrive the dimension for 'energy x time'.
57. State whether the threshold frequency in photoelectric phenomena is a charac-
teristic of light used or that of the material employed.
58. Why is CCl 4 molecule non-polar although the C-CI bond is polar?
.
··"
.
59. The electronic and vibrational transitions produce the electronic and vibrational
spectra. What are the transitions responsible for NMR spectrum?
60. Write an equation which describes the variation of A with temperature.
61. Write Boltzmann distribution formula for the case of degenerate energy levels.
62. The rate of reaction is 1.0 x 10-5 mol.dm- 3 .s- 1. Calculate the rate in molecule
cm-3.s- 1 unit.
670 PROBLEMS ON PHYSICAL CHEMISTRY

63. Give an example of a photosensitized reaction which is of immense important


to mankind.
64. Defin~ zeta potential.
65. Represent the Weston Cadmium cell using the usual convention.
66. Which of the two solutions, aq. solution of NaCl or an alcoholic solution of NaCl
of same conentration will show more non-ideality and why?
67. If pH =7 - (log x)/2, find Kw of pure water.
68. Explain if 'a' in van der Waals equation follows additivity.
69. How does the photoelectric work function depend on the frequency of light?

70. Expand the operator ( ~ + x J2.


71. Compare the dimen~ions
.
of action and angular momentum .
72. How is the stationary Bohr orbit of an atom defined in terms of de Broglie waves?
73. Verify whether the operator tJ. 2 is linear.
74. How does the energy associated with a wave function vary with the number of
nodes?
7S. What is the value of quantum yield for a primary process in photochemistry?
76. Should an ideal gas have viscosity? Why?
77. What is Clausius inequality?
78. A reaction of the type A(g) + A(g) ~ A 2(g) is expected to be exothennic.-Explain.
79. What condition must be satisfied for a binary liquid mixture to show azerotropism?
80. 12 usually sublimes, but it can be melted.-Explain.
81. How can you depict Boyle's law by a straight line plot other than PV vs. V
(or P)?
82. (i) Draw the P-T phase diagram of water. (ii) Label the different regions in this
diagram. (iii) On the diagram, show the liquid-vapour equilibrium for a dilute
solution of NaCl, with the help of a dashed curve. [JAM 2009]
83. The temperature dependence of the Gibbs free energy G is :

'd(G/T)) =-!!_,
( 'dT p T2
Obtain the expression for the temperature dependence of the equilibrium constant
K. Given that Af-!0 = A + BT (where A and B ·are constants). [JAM 2009]
84. The Maxwell. probability distribution of molecular speeds for a gas is

F( v)dv
.
=4 7tv 2 (__!!!.__) 3/2 ( 2)
exp - mv dv,
2TtkT 2kT
where v is the speed, m the mass of a gas molecule and k the Boltzmann constant.
(i) Use F(v) to show that the most probable speed u is given by the expression

u=rn.
(ii) Use R = 8 JK- 1 moJ- 1 !n the above expression to calculate the ump for CH4 (g)
at 127 °C. [JAM 2011]
MISCELLANEOUS PROBLEMS 671

Answers to Objective Type Questions

Part A 34, {c) (assuming it Part D


1. (b) (to be an ideal gas) 72. (a) F
2. (a) 35. (b) (b) F
3. (c) 36. (c) (c) F
4. (a) 37. (d) (d) T
5. (d) 38. (b) (e) T
6. (a) 39. (a) (f) F
7.(b)
40. (c)
8. (b) (g) F
41. (a) (h) T
9. (d)
42. (a) (i) T
10. (c)
11. (b) 43. (d) (j) F
12. (b) 44. (d) Part E
13. (a) 45. (b) 73. (a) (ii)
14. (b) 46. (a) (b) (i)
15. (a) 47. (c) (c) (ii)
16. (a) 48. (c) (d) (i)
17. (b) 49. (a) (e) (ii)
18. (a) so. (d)
(f) (ii)
19. (a)
51. (b)
20. (a) (g) (iii)
52. (a) (h) (i)
Part ·B 53. (d) (i) (i) .
21. (A) 2.9x10-7 54. (d) (j) (iii)
(B) 8.4lx10- 14
55. (c)
22. 0.08
Part F
56. (d)
74. (a) T
24. (i) 6.87 kJ 57. (a) (b) F
(ii) 4.48 kJ
58. (a) (c) F
26. 2.36x!0- 18 59. (b) (d) T
27. 2.76x10-t 7
60. (c)
(e) F
28. (a) can 61. (b)
(f) F
(b) more
62. (c) (g) T
(c) less
63. (c) (h) F
(d) true
64. (d)
(i) T
(e) false
65. (a) (j) F
Part C 66. (d)
29. (d) Part G
67. (c)
30. (b) 75. (a) (iii)
68. (c) .
31. (c) 69. (a) ·(b) (i)
32. ,. (b) 70. (b) (c) (ii)
33. (a) 71. (c) (d) (ii)
672 I PROBLEMS ON PHYSICAL CHEMISTRY
(e) (iii) 89. (c) 130. (d)
(t) (iv) 90. (b) 131. (b)
(g) (iii) 91. (b) 132. (b)
(h) (iii) 92. (b) 133. (b)
(i) (v) 93. (b) 134. (b)
(j) (ii) 94. (b) 135. (b)
Part H 95. (a) 136. (c)
76. (a) (iii) 96. (b) 137. (a)
(b) (iv) 97. (c) 138. (b)
(c) (iii) 98. (c) 139. (b)
(d) (i) 99. (c) 140. (b)
(e) (iii) 100. (d) 141. (c)

(t) (iii) 101. (c) 142. (d)


(g) (ii) 102. (c) 143. (a)

(h) (i) 103. (a) 144. (a)

(i) (iv) 104. (c) 145. (b)


(j) (iii) 105. (a) 146. (c)

Part I 106. (a) 147. (a)


107. (c) 148. (b)
77. (a) F
108. (b) 149. (c)
(b) F
109. (d) 150. (d)
(c) T
110. (b) 151. (b)
(d) T
111. (b) 152. (b)
(e) F
112. (b) 153. (d)
(t) T
113. (b) 154. (b)
(g) F
114. (b) 155. (a)
(h) F
115. (c) 156. (a)
(i) F
116. (b) 157. (a)
(j) F
117. (b) 158. (d)
Part J
118. (a) 159. (a)
78. (c)
119. (c) 160. (a)
79. (c)
120. (b) 161. (b)
80. (a)
121. (a) 162. (a)
81. (d)
122. (a) 163. (c)
82. (b) 164. (d)
123. (a)
83. (a)
165. (c)
124. (a)
84. (d) 166. (d)
125. (a)
85. (a) 167. (b)
126. (d)
86. (b) 127. (b) 168. (b)
87. (b) 128. (d) 169. (c)
88. (b) 129. (d) 170. (c)
MISCELLANEOUS PROBLEMS 673
171. (d) 212•. (c) 252. (b)
172. (c) 213. (b) 253. (d)
173. (c) 214. (b) 254. (b)
174. (a) 215. (c) 255. (c)
175. (c) 216. (a) 256. (b)
176. (b) 217. (c) 257. (c)
177. (a) 218. (b) 258. (b)
178. (c) 219. (a) 259. (b)
179. (a) 220. (c) 260. (d)
180. (d) 221. (c) 261. (c)
181. (a) 222. (c) 262. (d)
182. (d) 223. (a) 263. (c)
183. (a) 224. (b) 264. (a)
184. (c) 225. (d) 265. (b)
185. (a) 226. (c) 266. (c)
186. (a) 227. (a) 267. (a)
187. (c)
228. (d) 268. (a)
188. (a) 269. (c)
229. (c)
189. (c) 270. (a)
230. (d)
190. (b) in. Cd)
231. (c)
191. (b) 272. (b)
232. (d)
192. (c) 273. (c)
233. (a)
193. (a) 274. (d)
234. (d)
194. (d) 275. (c)
235. (c)
195. (d)
236. (d) 276. (a)
196. (b)
237. (a) 277. (a)
197. (b)
238. (c) 278. (b)
198. (d)
.239. (d) 279. (d)
199. (a)
240. (b) 280. (c)
200. (a)
241. (b) 281. (a)
201. (a)
242. (a) 282. (c)
202. (a)
243. (c) 283. (b)
203. (b)
244. (a) 284. (a)
204. (b)
245. (c) 285. (d)
205. (c)
246. (a) 286. (c)
206. (a)
247. (b) 287. (a)
207. (a)
248. (a) 288. (d)
208. (b)
249. (c) 289. (b)
209. (c)
210. (b) 250. (c)
211. (b) 251. (b)

Prob. Phy. Chem.-43

You might also like